Aap Prep 2024
Aap Prep 2024
An 8-year-old is being evaluated before undergoing skin-prick allergy testing. They have a history of
moderate persistent asthma, mild atopic dermatitis, and severe allergic rhinitis. Two weeks ago, they were
evaluated by their pediatrician for a cough and wheezing and a 5-day course of oral steroids was prescribed.
Their medications include daily inhaled corticosteroids, a daily intranasal steroid, an oral antihistamine at
bedtime, and a short-acting β-agonist inhaler as needed. The child uses emollients for her dry skin daily and
a low-potency topical steroid cream on occasion, most recently 2 months ago.
Of the following, the factor MOST likely to interfere with this child’s diagnostic testing is their
Correct answer is C
PREP Pearl(s)
Allergy testing should be obtained only in conjunction with a thorough history and physical
examination.
Skin-prick testing and serum-specific IgE tests identify immediate hypersensitivity reactions to specific
allergens, which may facilitate selective avoidance and immunotherapy.
Oral and topical antihistamines interfere with skin-prick testing, and their use must be discontinued
before testing.
Critique
Of the response choices, oral antihistamine use is most likely to interfere with skin-prick testing (SPT). Skin-
prick testing is performed by trained allergists in the office setting to detect an IgE response to specific
allergens. Testing involves puncturing the skin and introducing specific allergens into the epidermal layer. The
resulting wheal and erythema are measured. Histamine is used as a positive control and saline as a negative
control. Because histamine is an important wheal and flare response mediator, antihistamine use affects the
interpretation of SPT.
Short courses of oral corticosteroids and daily inhaled or intranasal steroids do not affect SPT results.
Although use of potent topical steroids may affect SPT results, the child in the vignette used only low-potency
topical steroids, and their last use was 2 months ago. Atopic dermatitis is a contraindication for SPT only if
the location and severity of skin inflammation interfere with interpretation of the test. The childin the
vignette has mild atopic dermatitis, which should not interfere with SPT.
Exposure to allergens with subsequent observation of the clinical response in a controlled setting is the
reference standard for diagnostic allergy testing (eg, oral food challenge, aeroallergen organ provocation).
However, at times these tests may not be practical or appropriate (owing to patient safety concerns). Skin-
prick and serum-specific IgE testing are used to identify specific food and environmental triggers with IgE-
mediated immediate hypersensitivity reactions. The results guide selective avoidance and immunotherapy
(for refractory cases). These tests are not useful in identifying T-cell mediated delayed reactions.
Allergy testing is most useful when symptoms are associated with specific triggers. A positive allergy test
result identifies sensitization to a trigger but does not correlate with symptoms. Clinicians must be aware of
conditions that may lead to false-positive or -negative results and interpret results in the context of history
and physical examination findings.
Low cost
Immediate availability of results
Trained personnel
Serum-specific IgE tests are widely available and do not require discontinuation of medications; however,
they are expensive, and their results may take a few days. Cross reactivity to food proteins can cause false-
positive results, which may lead to unnecessary food avoidance. The sensitivity and specificity of SPT and
serum-specific IgE testing for the detection of food and inhaled allergens are shown in Table 2. The large
number of allergens available on some testing panels may yield clinically irrelevant positive results and
unnecessary allergen avoidance. These large panels may also be expensive.
Suggested Reading(s)
Muthupalaniappen L, Jamil A. Prick, patch or blood test? A simple guide to allergy testing. Malays Fam
Physician. 2021;16(2):19-26. doi:10.51866/rv1141
Sicherer SH, Wood RA; American Academy of Pediatrics Section on Allergy and Immunology.. Allergy
testing in childhood: using allergen-specific IgE tests. Pediatrics. 2012;129(1):193-197.
doi:10.1542/peds.2011-2382
Wong AG, Lomas JM. Allergy testing and immunotherapy. Pediatr Rev. 2019;40(5):219-228.
doi:10.1542/pir.2018-0126
Rosetti A. Anaphylaxis. Point-of-Care Quick Reference. Pediatric Care Online. American Academy of
Pediatrics. December 22, 2022. Accessed September 1, 2023. Pediatric Care Online
Content Domain
Allergy
7DEOH6HQVLWLYLW\DQG6SHFLͤFLW\RI$OOHUJ\7HVWV
Test Sensitivity— 6SHFLͤFLW\̱ Sensitivity— 6SHFLͤFLW\̱
)RRG$OOHUJHQ )RRG$OOHUJHQ Inhaled Inhaled
PREPSA2024_C.indd 169
$OOHUJHQ $OOHUJHQ 9/19/23 12:06 PM
Skin 20%-60% 30%-70% 80%-90% 70%-95%
prick
6SHFLͤF 38%-60% 60%-80% 64%-69% 85%-90%
serum
IgE
Courtesy of S. Naganathan
AAP PREP 2024 - Question 2/267 - Anatomy Question 1/1
An 8-year-old girl is seen for a routine health supervision visit. She is new to the practice and has not had
recent medical care. She has no symptoms of illness, but her mother reports that her daughter’s lips have
looked bluish for more than a year.
On physical examination, the girl is afebrile and appears well except for mild cyanosis of her lips and
earlobes. Her weight and height are at the 75th percentile for age. Her vital signs are a blood pressure of
80/45 mm Hg; a heart rate of 75 beats/min; a respiratory rate of 15 breaths/min; and an oxygen saturation of
80% in room air via pulse oximetry, which increases to 84% with 100% oxygen by nonrebreather mask. Other
than moderate clubbing of her fingers and toes, the girl’s physical examination findings are normal.
Chest radiography shows an ill-defined density in the left upper lobe, clear lungs, and a normal cardiothymic
silhouette.
A. bronchiectasis
C. cystic brosis
Correct answer is D
PREP Pearl(s)
Digital clubbing in children is most often found in association with cystic fibrosis.
Chronic pulmonary infections such as bronchiectasis, tuberculosis, and HIV disease may cause digital
clubbing.
Digital clubbing may appear acutely in children with empyema.
Critique
The girl in the vignette has asymptomatic cyanosis, digital clubbing, hypoxia, and a left upper lobe density on
chest radiography. These findings are most consistent with a pulmonary arteriovenous malformation (AVM).
Chest computed tomography with intravenous contrast confirmed an AVM in the same area as the left upper
lobe density.
All of the other response choices could produce cyanosis and digital clubbing, but they would involve
additional signs, symptoms, or both. The girl has no chronic wet cough or diffuse radiographic changes to
suggest bronchiectasis. She has normal growth and no symptoms of malabsorption or chronic cough to
suggest cystic fibrosis (CF). Congenital cyanotic heart disease is unlikely with no symptoms of cardiovascular
compromise, normal cardiac examination findings, and no radiographic evidence of a cardiac abnormality.
Hippocrates noted digital clubbing (Figure) in an individual with empyema; this is thought to be the first
recognized clinical sign in medicine. Clubbing was then described in association with lung cancer,
bronchiectasis, hepatic cirrhosis, and cyanotic congenital heart disease. The list of associations has increased
over time. The pathophysiology of clubbing involves vascular endothelial growth factor, a platelet-derived
factor induced by hypoxia.
The Table outlines the conditions associated with clubbing in children. The most common cause of digital
clubbing in children is CF. However, with the advent of new treatments that address the defective cystic
fibrosis transmembrane conductance regulator protein resulting from the underlying pathologic genetic
variant, the incidence of all CF complications has decreased. With early surgical correction of many forms of
cyanotic congenital heart disease, the incidence of clubbing in these children has also decreased.
Long-term pulmonary infections and their complications are the other major causes of digital clubbing in
children. Bronchiectasis, chronic tuberculosis, and HIV infections are associated with digital clubbing,
especially in areas where tuberculosis and HIV are endemic. Up to 30% of children with chronic tuberculosis
in endemic areas have clubbing.
Inflammatory bowel diseases and biliary cirrhosis have a rare association with digital clubbing. There is a
familial form of clubbing with no other disease association. Unilateral clubbing may occur when a localized
vascular lesion produces an arteriovenous shunt, as well as in children with hemiplegia.
In all of these conditions, digital clubbing is a delayed finding after a prolonged disease course. The exception
is empyema, which may be associated with an acute onset of digital clubbing.
Hypertrophic osteoarthropathy, a rare cause of clubbing in children, is associated with periostitis (usually of
the long bones), painful joint enlargement, and digital clubbing. In children, hypertrophic osteoarthropathy
may be inherited (~20%) or may be seen in the context of CF. Cardiac, gastrointestinal, and hepatic diseases
are rarely associated with hypertrophic osteoarthropathy.
Suggested Reading(s)
Bhat MA, Plunk MR, Basel DG, Hanson SJ. Cyanosis in a previously well child. Pediatr Rev.
2021;42(11):619-624. doi:10.1542/pir.2020-002055
Chang AB, Oppenheimer JJ, Weinberger M, Rubin B, Irwin RS. Children with chronic wet or productive
cough: treatment and investigations—a systematic review. Chest. 2016;149(1):120-142.
doi:10.1378/chest.15-2065
Essouma M, Nkeck JR, Agbor VN, Noubiap JJ. Epidemiology of digital clubbing and hypertrophic
osteoarthropathy. J Clin Rheumatol. 2022;28(2):104-110. doi:10.1097/RHU.0000000000001830
Homnick D. Pediatric physical examination: interpretation of findings. In: McInerny TK, Adam HM,
Campbell DE, Foy JM, Kamat DM, eds. American Academy of Pediatrics Textbook of Pediatric Care. 2nd
ed. American Academy of Pediatrics; 2016:chap 16. Accessed September 1, 2023. Pediatric Care Online
Content Domain
Anatomy
Normal Clubbing
Hyponychial angle
C B C
B
A
A
Schamroth sign
Adapted and reprinted with permission from Adapted from Pasterkamp H. The history and
physical examination. In: Wilmott RW, Boat TF, Bush A, et al, eds. Kendig and Chernick’s Disorders
of the Respiratory Tract in Children. 8th ed. Philadelphia, PA: Elsevier Saunders; 2012: 127
A 24-month-old boy is seen for a health supervision visit. His medical history reveals recurrent ear infections
with chronic middle ear effusion. His parents are concerned about his lack of language skills. He speaks
about 10 words and babbles sequential sounds that are mostly unintelligible. His motor and social
milestones are appropriate for age. His parents are bilingual and speak both English and Spanish at home.
The boy has 2 healthy siblings, a 5-year-old brother who received speech therapy and an 8-year-old sister.
His growth parameters and physical examination findings are normal.
Of the following, the MOST significant risk factor for this child’s condition is his
A. bilingual family
Correct answer is C
PREP Pearl(s)
Language is the most common domain of developmental concern in preschool-aged children.
Risk factors for language delay include any etiology causing global developmental delay as well as male
sex, family history of speech and language disorders, moderate to severe hearing loss, and lack of
environmental stimulation.
A bilingual household, chronic middle ear effusion, and late birth order are not factors associated with
language delays.
Critique
The 24-month-old boy in the vignette has significant language delay, with only 10 words and speech that is
mostly unintelligible. From ages 18 to 24 months, there typically is rapid advancement in speech and
language milestones. At age 2 years, children should have a vocabulary of almost 200 words and use 2-word
phrases, and 50% of their speech should be intelligible to strangers.
Of the response choices, a family history of speech delay is the most significant risk factor for this boy’s
language delay. A family history of speech delay is associated with a 3-times-higher risk of speech problems.
Other factors associated with language delay include male sex, family history of language and reading
disorders, low socioeconomic status, and an environment with poor stimulation and/or psychosocial stress.
Speech delay should not be considered a normal variation in language development. Boys may start speaking
words slightly later than girls but catch up within a few months. A bilingual home is not a risk factor for
speech delay. Bilingual children should have a combined vocabulary similar to monolingual children.
Although initially there may be mixing of words from both languages, children learn quickly to separate
languages. Chronic middle ear effusion can cause mild conductive hearing loss but is not typically associated
with significant language delay. It is a misconception that late birth order children speak later than first-born
children. Later birth order children are less likely to hear only adult-generated speech directed towards
them.
Language is the most common domain of developmental delay in preschool children, with a prevalence of 2%
to 20%, and often has no identifiable cause. The Table outlines the factors associated with delayed
achievement of language milestones.
For a child with language delay, a thorough history and physical examination may yield clues that can direct
further evaluation. All children with speech delay should undergo a hearing screening. Moderate to severe
hearing loss can lead to poor language acquisition as well as other developmental delays.
Early identification of developmental delay allows for early intervention and can improve outcomes in these
children. The American Academy of Pediatrics recommends developmental screening with a validated
screening tool at health supervision visits for all children at ages 6, 18, and 30 months.
Suggested Reading(s)
Feldman HM. How young children learn language and speech. Pediatr Rev. 2019;40 (8):398-411.
doi:10.1542/pir.2017-0325
McQuiston S, Kloczko N. Speech and language development: monitoring process and problems.
Pediatr Rev. 2011;32(6):230-238. doi:10.1542/pir.32-6-230
Rosenberg M. Speech and language concerns. In: McInerny TK, Adam HM, Campbell DE, DeWitt TG, Foy
JM, Kamat DM, eds. American Academy of Pediatrics Textbook of Pediatric Care. American Academy of
Pediatrics; 2023. Accessed September 1, 2023. Pediatric Care Online
Scharf RJ, Scharf GJ, Stroustrup A. Developmental milestones. Pediatr Rev. 2016;37(1):25-37.
doi:10.1542/pir.2014-0103
Content Domain
Behavioral/Developmental
Courtesy of S. Naganathan
AAP PREP 2024 - Question 4/267 Behavioral/Developmental Question 2/9
A 4-year-old boy is undergoing follow-up evaluation regarding behavioral concerns. At his recent health
supervision visit, the boy’s mother reported that he is constantly on the go, does not pay attention to
directions, requires frequent reminders to complete tasks, often acts impulsively, and would “spend hours”
using his tablet, if allowed. During that visit, the boy jumped off the examination table multiple times,
attempted to leave the examination room, and would not remain seated. Today, the boy is very focused on
completing a puzzle on his tablet. His developmental milestones have been on track. Responses to the
behavioral questionnaires completed by his parents and day care teacher are consistent with clinically
significant and impairing levels of inattention, hyperactivity, and impulsivity. The mother asks whether her
son has attention-deficit/hyperactivity disorder and, if so, what can be done for him.
Of the following, the BEST next step in this boy’s care is to
A. explain that he does not meet criteria for this diagnosis because he is able to focus well on his
tablet
D. recommend that the boy return in 2 years for an evaluation when he is old enough for a
diagnosis to be made
Correct answer is C
PREP Pearl(s)
Attention-deficit/hyperactivity disorder can be diagnosed as early as age 4 years.
The first line of treatment for preschool-aged children with attention-deficit/hyperactivity disorder is
behavioral management. For those who continue to have a moderate to severe level of impairment
with behavioral interventions, medication can be considered.
Attention-deficit hyperactivity disorder evaluation includes assessment for comorbid conditions
including anxiety, learning disorders, oppositional defiant disorder, and mood disorders. The risk of
experiencing comorbidities varies according to age.
Critique
The boy in the vignette meets criteria for a diagnosis of attention-deficit/hyperactivity disorder (ADHD),
combined presentation. His developmentally inappropriate levels of hyperactivity and inattention are
resulting in functional impairment in more than one setting (home and day care). Of the response choices,
providing information on parent-directed behavioral management and scheduling follow-up in 3 months is
the best next management step.
Behavioral intervention is the first line of treatment for preschool-aged children (ages 4-6 years) who have
ADHD. Parent training should include education about developmentally appropriate expectations and
promoting positive behaviors while decreasing negative behaviors. It is important to follow up with the family
within an appropriate time frame (eg, 3 months) to monitor the child’s response to behavioral therapy.
Stimulant medication (eg, methylphenidate) may be considered for preschool-aged children with ADHD who
continue to have a moderate level of functional impairment with behavioral therapy alone. Medication is not
recommended as the first line of treatment for a preschool-aged child.
One of the most common neurodevelopmental disorders, ADHD has a prevalence of 8.9% in children
between the ages of 3 and 17 years. The prevalence of ADHD increases with age. Boys are twice as likely to be
diagnosed with ADHD as are girls. The rate of ADHD is higher in children with other developmental disorders
(eg, fragile X syndrome, epilepsy, autism, and intellectual disability).
Evaluation for ADHD should include a detailed medical history (including prenatal, birth, past medical issues)
and family history (ADHD, other behavioral and neurodevelopmental disorders). Risk factors for ADHD
include the following:
Before initiating a stimulant medication, the practitioner should inquire about any family history of cardiac
conditions including congenital defects, syndromes associated with cardiac disorders, arrhythmias, and
sudden cardiac death.
To make a diagnosis of ADHD, the following information regarding the child’s behavior should be elicited:
Types
Setting(s)
Frequency
Related safety concerns
The signs and symptoms of ADHD must be present in more than 1 setting and result in functional
impairment in at least 1 domain (academic, social, or adaptive). In most cases, these occur before age 12
years. The diagnostic criteria for the 3 types of ADHD are outlined in the Table.
The 4-year-old boy in the vignette is not too young to be diagnosed with ADHD. According to the 2020
American Academy of Pediatrics clinical practice guidelines, there is insufficient evidence for children younger
than 4 years to be diagnosed with ADHD; however, these children can be referred for behavioral
interventions without a diagnosis.
Conditions co-occurring with ADHD in preschool-aged children may include developmental delay, autism,
hearing loss, and language disorders. In school-aged children, comorbid conditions may include learning
disorders, anxiety, and oppositional defiant disorder. In addition to the conditions seen in school-aged
children, adolescents with ADHD are at higher risk of developing substance use disorder, sleep deprivation,
and mood disorders. Hyperfocusing on a preferred activity or task is seen in children and adolescents of all
ages with ADHD. This behavior contributes to the overall difficulty with attention regulation and ability to shift
attention between tasks.
Suggested Reading(s)
Barbaresi WJ, Campbell L, Diekroger EA, et al. Society for Developmental and Behavioral Pediatrics
clinical practice guideline for the assessment and treatment of children and adolescents with complex
attention-deficit/hyperactivity disorder. J Dev Behav Pediatr. 202;41(suppl 2S):S35-S57.
doi:10.1097/DBP.0000000000000770
Leslie LK, Guevara JP. Attention-deficit/hyperactivity disorder. In: McInerny TK, Adam HM, Campbell DE,
Foy JM, Kamat DM, eds. American Academy of Pediatrics Textbook of Pediatric Care. 2nd ed. American
Academy of Pediatrics; 2016:chap 220. Accessed March 4, 2023. Pediatric Care Online
Rajaprakash M, Leppert ML. Attention-deficit/hyperactivity disorder. Pediatr Rev. 2022;43(3):135-147.
doi:10.1542/pir.2020-000612
Wolraich ML, Hagan JF, Allan C, et al; Subcommittee on Children and Adolescents with Attention-
Deficit/Hyperactive Disorder. Clinical practice guideline for the diagnosis, evaluation, and treatment of
attention-deficit/hyperactivity disorder in children and adolescents. Pediatrics. 2019;144(4):e20192528.
doi:10.1542/peds.2019-2528
Content Domain
Behavioral/Developmental
7DEOH'LDJQRVWLF&ULWHULDIRUWKH7\SHVRI$WWHQWLRQ'HͤFLW
Hyperactivity Disorder.
$WWHQWLRQ'HͤFLW+\SHUDFWLYLW\'LVRUGHU7\SH Diagnostic Criteria
Courtesy of S Monteiro
AAP PREP 2024 - Question 5/267 Behavioral/Developmental Question 3/9
A 10-year-old boy is seen in the office with his maternal aunt to establish care. The aunt reports that before
being permanently placed with her after his mother’s parental rights were terminated owing to substance
use and neglect, the boy had been moving “back and forth” between his mother’s home and multiple foster
homes for the past 5 years. The aunt has contacted his case worker to get his medical and educational
records so that she can enroll him in school. The boy’s mother dropped out of school in the 9th grade, after
she started using drugs, and his aunt is trying to avoid “repeating history.” The boy takes multiple
medications to help with his behavior, and his aunt is wondering if their use can be discontinued because “he
has a stable home now and does not need them.” The boy’s physical examination findings are normal.
A. agree that the medications can be discontinued as he is now in a stable living situation with a
caring family member
B. perform developmental and mental health screening in the office and refer for further
assessment and medication management
C. recommend that the medication use be continued until medical and school records can be
obtained from his case worker
D. request that the aunt reach out to his most recent foster family to determine what each
medication was treating
Correct answer is B
PREP Pearl(s)
Children and adolescents in the foster care system have higher rates of developmental and psychiatric
disorders. As adults, this population is at increased risk of experiencing low educational attainment,
mental health disorders, unemployment, homelessness, and posttraumatic stress disorder.
Comprehensive assessment, including developmental and mental health screening, should be
performed by pediatricians within 30 days of a child’s entry into foster care; longitudinal monitoring
also should be conducted.
Among the challenges of providing care to children in the foster care system are lack of access to
medical and educational records; varying levels of education of foster parents regarding the impact of
trauma on children in foster care and these children’s risks of experiencing mental health disorders
and developmental disabilities; and variable coordination of care by caseworkers.
Critique
The boy in the vignette has been placed in the care of his maternal aunt. Because this is a new placement for
the boy, developmental and mental health screening should occur along with relevant referrals (eg, mental
health, developmental and behavioral pediatrics) for assessment and medication management.
The American Academy of Pediatrics classifies children in the foster care system as children with special
health care needs. Among those in foster care, up to 80% enter the system with a mental health condition
and 40% of school-aged children have learning challenges (American Academy of Pediatrics Council on Foster
Care, 2015). Children in foster care are more likely to receive special education services, change schools in
the middle of the school year, experience grade retention, and drop out of school as adolescents. As adults,
this population is at increased risk of experiencing low educational attainment, mental health disorders,
unemployment, homelessness, and post-traumatic stress disorder.
Given the frequency of psychiatric and neurodevelopmental disorders in this population, it is recommended
that a comprehensive health assessment, including developmental and mental health screening, be
completed within 30 days of entry into the foster care system. Reassessment should be conducted 60 to 90
days after entry into foster care, and continued monitoring should occur semiannually to assess adjustment
to placement.
Although some studies have shown greater stability with placement in kinship care, there are associated
challenges. Kinship caregivers are generally older and have limited access to services compared with
nonrelative foster parents. Additionally, oversight of and subsidies provided for kinship care may be limited.
Given the boy’s history of multiple placements (which, for many, results in difficulties forming a secure
attachment to a caregiver) along with the history of maternal substance use and neglect, it is recommended
that the boy continue to use his medications until he can be assessed by a mental health professional, a
development and behavioral pediatrics professional, or both. Childhood trauma has been associated with
difficulties in emotional regulation, aggression, inattention, hyperactivity, and impulsivity. Data regarding
children with Medicaid insurance demonstrate that children in foster care are prescribed psychotropic
medications at a much higher rate and with a higher rate of polypharmacy compared with other children.
One of the many challenges of establishing a medical home for a child in the foster care system or in kinship
care is a lack of comprehensive medical and educational records (eg, immunization records, medication
history, surgical history, and psychosocial/family history). Caseworkers face similar challenges, and they may
not have the training needed to manage medical and educational needs with the information available to
them. Thus, it is not recommended to delay medication management decisions for the boy in the vignette
until records are obtained from his case worker. Requesting that the aunt reach out to his most recent foster
family to determine what the boy’s medications were treating is not the best next step in the boy’s care, as it
is unclear what the family knew or understood about his diagnoses and treatment. Foster parents often
receive little education regarding the impact of trauma, risk of experiencing mental health disorders and
developmental disabilities, coordinating care among medical and mental health care providers, and ensuring
adequate educational support.
Pediatricians caring for children in the foster care system must be aware of and understand the impact of
trauma, adverse childhood experiences, and neglect on early development, as well as the resulting risk of
experiencing neurodevelopmental and psychiatric disorders through childhood and adolescence.
Pediatricians play a key role as the child’s medical home, identifying, addressing, and coordinating mental
health, developmental, and educational needs. This role includes comprehensive screening for mental health
and developmental disorders, referral to a specialist for further assessment when appropriate, and
longitudinal monitoring. Ongoing communication between caseworkers and foster parents is also required to
ensure all of a child’s needs are met.
Suggested Reading(s)
Gilgoff R, Singh L, Koita K, Gentile B, Marques SS. Adverse childhood experiences, outcomes, and
interventions. Pediatr Clin North Am. 2020;67(2):259-273. doi:10.1016/j.pcl.2019.12.001
Okun A. Foster care. Pediatr Rev. 2016;37(12):546-547. doi:10.1542/pir.2016-0076
Szilagyi M. The pediatric role in the care of children in foster and kinship care. Pediatr Rev.
2012;33(11):496-507. doi:10.1542/pir.33-11-496
Szilagyi MA, Rosen DS, Rubin D, Zlotnik S; Council on Foster Care, Adoption, and Kinship Care;
Committee on Adolescence; Council on Early Childhood. Health care issues for children and
adolescents in foster care and kinship care. Pediatrics. 2015;136(4):e1142-66. doi:10.1542/peds.2015-
2656
Szilagyi M, Jee SH. Children in foster or kinship care. In: McInerny TK, Adam HM, Campbell DE, Foy JM,
Kamat DM, eds. American Academy of Pediatrics Textbook of Pediatric Care. 2nd ed. American
Academy of Pediatrics; 2016:chap 72. Accessed September 1, 2023. Pediatric Care Online
Content Domain
Behavioral/Developmental
The correct answer is: perform developmental and mental health screening in the office and refer for further
assessment and medication management
A 14-year-old with no previous medical problems is seen by his primary care provider for evaluation of
difficulty focusing. His grades have declined over the past 6 months. He has no history of attention-
deficit/hyperactivity disorder (ADHD). He recently got into trouble at school for having a vape device. The
adolescent privately admits to daily use of marijuana and nicotine (vaped) for the past year.
The boy’s mother smokes marijuana daily to help with stress relief and for perceived health benefits. She
also uses nicotine products daily and takes a stimulant for ADHD. She believes that her son is old enough to
make his own health decisions and is not worried about his smoking, because she also smoked at his age.
Education is provided on cognitive development and the effect of substance use during adolescence on the
risk of future substance use disorders and addiction. Readiness for change is assessed for both the boy and
his mother.
Correct answer is D
PREP Pearl(s)
The prefrontal cortex controls executive brain function and higher-level decision-making, and it is not
fully developed until age 21 years or later.
Daily or near-daily substance use can impair normal adolescent cognitive functioning and brain
development.
Treatment of comorbid psychological conditions is imperative in the management of adolescent
substance use.
Critique
The boy in the vignette reports difficulty focusing and daily use of marijuana and nicotine. The best next step
in his care is to screen for comorbid psychological disorders that may be contributing to his substance use.
Mood disorders, anxiety disorders, and attention-deficit/hyperactivity disorder (ADHD) are common
comorbid conditions in adolescents who use substances. Treatment of any comorbid psychological disorders
is imperative for the successful management of substance use. For diagnostic accuracy, individuals should
not be under the influence of substances during psychological evaluations.
Urine drug screening would not be the best next step in treating this boy. High false-positive and false-
negative rates can confound assessments and delay appropriate management. An evaluation for ADHD
should be performed before this boy receives any stimulant medication. A follow-up visit should be
scheduled sooner than 6 months.
Adolescence is commonly divided into three phases: early (ages 10 to 13 years), middle (ages 14 to 17 years),
and late (ages 18 to 21 years). Early adolescents typically have concrete thinking, are egocentric, and are
becoming more concerned with how they are perceived by their peers. Middle adolescents begin to develop
more abstract thinking, but they allow strong emotions to drive many of their decisions. Late adolescents can
better navigate decisions and begin to develop their own values and goals. The prefrontal cortex, the area of
the brain that controls executive function and higher-level decision making, does not fully develop until age
21 years or later.
The boy and his mother should be counseled that at age 14 years, children lack the full capacity to make
healthy decisions for themselves because the brain is still developing and forming connections. Normal
adolescent brains have a reward center that responds to high levels of activity (the “gas pedal”) with a
prefrontal cortex (the “brake pedal”) that is not fully developed. This combination can lead to risky behaviors
that are commonly seen in adolescents. To be successful and safe, adolescents require structure, monitoring,
support, and accountability.
The boy in the vignette has engaged in daily substance use for the past year. While the brain is developing,
substance use can decrease cognitive function and alter normal development. Thus, because of his daily
substance use, the boy may be functioning at a cognitive level that is lower than expected for his
chronological age.
Suggested Reading(s)
Allen B, Waterman H. Stages of adolescence. HealthyChildren.org. Accessed September 1, 2023.
https://www.healthychildren.org
Levy S, Bagley S. Substance use: initial approach in primary care. In: McInerny TK, Adam HM, Campbell
DE, DeWitt TG, Foy JM, Kamat DM, eds. American Academy of Pediatrics Textbook of Pediatric Care.
2nd ed. American Academy of Pediatrics; 2021:chap 198. Accessed September 1, 2023. Pediatric Care
Online
Preventing drug misuse and addiction: the best strategy. National Institute on Drug Abuse. Accessed
September1, 2023. https://nida.nih.gov/publications/drugs-brains-behavior-science-
addiction/preventing-drug-misuse-addiction-best-strategy
Content Domain
Behavioral/Developmental
A 2-year-old boy is seen in the office for a health supervision visit. He is lying on his back crying on the
examination room floor while his mother is holding a toy and telling him that he is not allowed to play until
the end of the visit. After several seconds of crying, he stops vocalizing, appears limp, and his face looks blue.
The incident lasts approximately 30 seconds after which his skin color returns to baseline, he opens his eyes,
sits up, and is alert while being comforted by his mother, who is frightened and distressed. She states that
similar events have been occuring at home a few days a week when she attempts to discipline him. She
denies any extended loss of consciousness, shaking of extremities, or post-event sleepiness with prior
episodes. His findings on physical examination after the event are unremarkable.
Review of his medical record shows that he is underimmunized. He had a recent visit to the emergency
department for a fever, at which time he had a normal complete blood count. There is a family history of a
paternal aunt who, as a child, was under the care of a neurologist for unknown reasons.
A. encourage his mother to avoid potential trigger events, such as enforcing rules or withholding
toys
B. reassure his mother that this condition is not harmful and acknowledge her fear
C. refer the patient to the emergency department for further evaluation and management
D. refer the patient to a pediatric neurologist for further evaluation and management
Correct answer is B
PREP Pearl(s)
A breath-holding spell is an involuntary reflex linked to the autonomic nervous system, typically in
response to anger or fear, that results in transient cerebral anoxia.
Parents should be reassured that breath-holding spells are not harmful.
Diagnostic evaluation or referral to a specialist for breath holding spells is not necessary unless the
diagnosis is uncertain or there are concerns for an underlying condition, such as a seizure disorder.
Critique
The child in the vignette experienced a breath-holding spell. These events have no long-term sequelae or
negative effects on development. Breath-holding spells can be frightening for the parent to witness. The best
next step in management is to reassure his mother that this condition is not harmful and acknowledge her
fear.
A breath-holding spell is an involuntary reflex linked to the autonomic nervous system that results in
transient cerebral anoxia. It occurs most commonly between ages 1 and 3 years, but can be seen in infants
and older toddlers. Spells usually last less than 1 minute, occurring 1 time in a day. Breath-holding spells can
be classified into 2 categories, cyanotic and pallid. Typically, when anger serves as a trigger there are cyanotic
skin changes, whereas sudden fear can result in pallor. Cyanotic breath-holding spells are more common.
The child in the vignette demonstrated a typical example of a spell, beginning with an intense crying event
when his toy was taken away, then becoming limp with facial cyanosis. Affected children often lose
consciousness.
Although the exact cause of breath-holding spells is unknown, the family history may reveal family members
who have experienced similar episodes, suggesting a genetic component. Iron-deficiency anemia may be
associated with breath-holding spells; iron supplementation has been shown to improve the condition. The
boy in the vignette’s recent complete blood count result was normal. The differential diagnosis includes a
seizure, and the diagnoses can sometimes be difficult to distinguish. Unlike seizures, breath-holding spells
typically have an identifiable trigger, and the child is immediately alert once it is over. A breath-holding spell is
a clinical diagnosis; diagnostic testing is usually not indicated. Resolution of spells typically occurs by 5 years
of age without intervention.
Unlike breath-holding spells, tantrums are an expected developmental response in toddlers as they are trying
to exert independence and control over their lives while still depending on others to care for them. Children
can have difficulty understanding and regulating their emotions, as well as naming their feelings as language
develops; tantrums are one way they can express themselves until they learn appropriate response
techniques.
Although it may feel easier for parents to avoid potential breath-holding spell triggers, they should be
reminded that these spells are an involuntary response and encouraged to enforce age-appropriate
discipline. Given that the child in the vignette had a witnessed event, is clinically stable with normal physical
examination findings, and has had similar benign prior events, a referral to the emergency department or a
neurologist, diagnostic studies, and medication are not indicated at this time. However, if there is diagnostic
uncertainty or concern for an underlying disorder, further evaluation may be indicated.
Breath-holding spells can be a scary event for the parent and panic may be the first response. Pediatricians
should provide reassurance and reinforce that these spells are not dangerous. It can be helpful to remind
parents to remain calm and work through supportive steps to care for their child during a spell. Parents
should also be reminded to never shake a baby.
Suggested Reading(s)
Adam H. Temper tantrums and breath-holding spells. Point-of-Care Quick Reference. Pediatric Care
Online. American Academy of Pediatrics; 2022. doi:10.1542/aap.ppcqr.396460
Leung AKC, Leung AAM, Wong AHC, Hon KL. Breath-holding spells in pediatrics: a narrative review of
the current evidence. Curr Pediatr Rev. 2019;15(1):22-29. doi:10.2174/1573396314666181113094047
Prazar GW. Temper tantrums and breath-holding spells. In: McInerny TK, Adam HM, Campbell DE,
DeWitt TG, Foy JM, Kamat DM, eds. American Academy of Pediatrics Textbook of Pediatric Care.
American Academy of Pediatrics; 2023. Accessed September 1, 2023. Pediatric Care Online
Sege RD, Siegel BS; Council on Child Abuse and Neglect, Committee on Psychosocial Aspects of Child
and Family Health. Effective discipline to raise healthy children. Pediatrics. 2018;142(6):e20183112.
doi:10.1542/peds.2018-3112
Content Domain
Behavioral/Developmental
The correct answer is: reassure his mother that this condition is not harmful and acknowledge her fear
View Peer Results
AAP PREP 2024 - Question 8/267 Behavioral/Developmental Question 6/9
A 7-year-old girl is undergoing evaluation of inattention noted by her teacher since she started 2nd grade this
year. The girl previously looked forward to going to school, but her mother reports it is now difficult to get
her up and out the door every morning. Homework time in the afternoon is challenging. The girl and her
mother are spending almost 2 hours on work that should be completed in 20 minutes. The teacher has
started to work with the girl in a small group of students for 15 minutes each day, but this has not seemed to
help with completing homework assignments covering the same material. The girl’s mother does not have
concerns regarding inattention, hyperactivity, or impulsivity at home. The girl received speech therapy until
kindergarten. She has no other notable medical history. The family history is notable only for an uncle with
learning difficulties who did not graduate from high school.
Of the following, the BEST next best step in this girl’s care is to
A. have the mother and the girl’s teacher complete a standardized behavioral questionnaire
C. recommend that the girl work with a tutor at home and schedule follow-up in 3 months
D. start stimulant medication to treat the inattention and schedule follow-up in 1 month
Correct answer is B
PREP Pearl(s)
Children with learning disabilities have higher rates of comorbid psychiatric conditions including
attention-deficit/hyperactivity disorder, anxiety, and depression; behavioral symptoms may be the
presenting concern.
Children with a history of a speech and language disorder are at increased risk of developing language-
based learning disorders (eg, dyslexia) and should be closely monitored as they enter school.
Pediatricians play an important role in the identification and care of children with learning disabilities
and act as a liaison between the school and community.
Critique
The girl in the vignette is exhibiting symptoms that raise concern about a learning disability. The best next
step in her care is to recommend that she undergo a comprehensive psychoeducational evaluation at
school.
A learning disability is present when there is a discrepancy between an individual’s intellectual ability and his
or her academic performance. Areas of deficiency can include reading, mathematics, and written expression.
In the past, psychoeducational testing demonstrating a discrepancy between intelligence and achievement
scores was required to qualify for special education support. Updates to the Individuals with Disabilities
Education Act in 2004 state that support and interventions should be provided to students who are struggling
while the formal evaluation process is completed and a specific diagnosis is made. The girl in the vignette has
been having a difficult time in school, and she has not demonstrated a successful response to intervention
(small group work). A full psychoeducational evaluation would be the best next step to determine if she has a
specific learning disorder.
A reading disability (dyslexia), the most common form of learning disability, is characterized by deficits in
phonological processing (sounding out words), reading fluency, and reading comprehension. A history of
speech and language difficulties is frequently associated with dyslexia and is an indication for close
monitoring as a child enters school. Other risk factors for dyslexia include a family history of learning
disabilities and prematurity. Some affected children are identified at a later age, especially if they have strong
skills in rote memorization, a higher cognitive ability, or other successful coping strategies.
Behavioral problems are frequently reported in children with learning disabilities and may be the initial
concern raised. Children may appear anxious about attending school, act out, exhibit “class clown” behavior,
or have declining grades. Children with learning disabilities have higher rates of comorbid psychiatric
conditions, including attention-deficit/hyperactivity disorder (ADHD), anxiety, and depression; these children
should routinely undergo assessment for these conditions. The diagnosis of ADHD requires symptoms to be
noted in at least 2 settings. Although this girl’s teacher reports inattention at school, the girl’s mother has not
noticed any behaviors that raise concern about ADHD (inattention, hyperactivity, impulsivity) in the home
setting. Therefore, behavioral questionnaires could be helpful but would not be the best next management
step. Similarly, initiation of stimulant medication would not be appropriate, as a diagnosis of ADHD has not
been established.
Tutoring could be helpful for this child; however, it is more important for her to have a psychoeducational
assessment to help determine her learning needs. Despite many children’s showing early signs of dyslexia
(eg, difficulties associating letters with their individual sounds, difficulties with rhyming, problems with rapid
naming of objects), most children are not diagnosed until the end of 2nd grade or beginning of 3rd grade. This
delay in diagnosis may result in an educational gap that is difficult for many to overcome. Individuals with
learning disabilities are at increased risk of dropping out of high school, not attending higher educational
programming, and entering the juvenile justice system. Adults with learning disabilities experience higher
levels of unemployment and are more likely to earn lower wages than peers with higher educational
attainment.
Pediatricians play an instrumental role in the identification and management of children with learning
disabilities through longitudinal monitoring and facilitating evaluation and interventions when indicated.
When concern about a learning disability arises, a thorough history should be obtained, including pregnancy
and birth, chronic medical problems, education (age at school entry, previous daycare/preschool attendance),
previous educational evaluation or therapies, developmental milestones achieved, and family history.
Pediatricians can facilitate the completion of a psychoeducational assessment and support the receipt of
appropriate educational support and interventions for children affected by a learning disability.
Suggested Reading(s)
Bailet LL. Learning disorders. In: McInerny TK, Adam HM, Campbell DE, Foy JM, Kamat DM, eds.
American Academy of Pediatrics Textbook of Pediatric Care. American Academy of Pediatrics; 2023.
Accessed September 1, 2023. Pediatric Care Online
Hulme C, Snowling MJ. Reading disorders and dyslexia. Curr Opin Pediatr. 2016;28(6):731-735.
doi:10.1097/MOP.0000000000000411
Rimrodt SL, Lipkin PH. Learning disabilities and school failure. Pediatr Rev. 2011;32(8):315-324.
doi:10.1542/pir.32-8-315
Sanfilippo J, Ness M, Petscher Y, Rappaport L, Zuckerman B, Gaab N. Reintroducing dyslexia: early
identification and implications for pediatric practice. Pediatrics. 2020;146(1):e20193046.
doi:10.1542/peds.2019-3046
Content Domain
Behavioral/developmental
A 7-year-old boy is brought to the office by his mother to discuss the results of his recent school evaluation. A review
of the evaluation report reveals that the boy has an intellectual disability. He has a history of global developmental
delay and has been receiving physical, occupational, and speech therapies since age 2 years. He has not undergone
a medical evaluation related to this delay. The boy was born at full term, had a negative newborn screening test
result, and has no history of seizures. During the pregnancy, his mother did not take medication (other than
prenatal vitamins), use illicit drugs or alcohol, or have an infection. There is no family history of intellectual disability
or known genetic disorders. The boy’s physical examination findings are unremarkable.
D. order urine organic acid, amino acid, glycosaminoglycan, and oligosaccharide testing
Correct answer is C
PREP Pearl(s)
Decisions regarding the laboratory and imaging evaluation of individuals with intellectual disability (ID)
should be made on the basis of the findings of a thorough history and physical examination. The
recommended evaluation for nonsyndromic ID is a chromosomal microarray analysis.
Intellectual disability is defined as a deficit in both cognitive and adaptive functioning. It can be further
categorized according to severity, which facilitates appropriate counseling of families regarding
appropriate expectations and planning for support.
For children with ID, practitioners in the medical home play a vital role in reviewing educational
support and ensuring goals are individualized and appropriate.
Critique
The boy in the vignette is living with an intellectual disability (ID) without history or physical examination
findings suggestive of a specific genetic syndrome. He is therefore classified as having a nonsyndromic ID of
unknown etiology. The best next step in this boy’s care is to order a chromosomal microarray, which has a
diagnostic yield of 15% to 20% in this clinical scenario. Testing for fragile X would be indicated if the boy had
clinical features of the condition (eg, macrocephaly, prominent ears, hyperextensible joints, enlarged testes in
pubertal boys) or if the chromosomal microarray result is normal.
Individuals living with ID have deficits in both cognitive and adaptive functioning. Cognitive functioning is
measured by standardized, culturally appropriate instruments that test intelligence (eg, IQ testing). Scores
falling at least 2 standard deviations below the mean (eg, IQ ≤70) are consistent with an ID. Adaptive
functioning (conceptual, social, and practical domains, which contribute to an individual’s overall ability to
take care of themselves and function in the community) is measured through standardized questionnaires.
Intellectual disability is further classified according to severity level (Table).
The etiology of ID can be genetic or environmental (eg, prenatal infection, alcohol, other teratogen exposure);
alternatively, ID may occur secondary to another condition (eg, traumatic brain injury, prematurity, inborn
error of metabolism, neurologic disorder). For many individuals with ID, the cause remains unknown.
A thorough history and physical examination should be performed when evaluating a child with global
developmental delay (a diagnosis used until age 6 years) or intellectual disability. Findings should guide
decisions regarding laboratory evaluation and neuroimaging.
An inborn error of metabolism should be suspected in the context of neurologic findings (eg, hypotonia,
ataxia, spasticity, seizures), gastrointestinal symptoms, growth deficiency, or atypical odor. The boy in the
vignette does not have any of these symptoms or signs; therefore, testing of urine organic acids, amino acids,
glycosaminoglycan, and oligosaccharides is not indicated. Because the boy does not have a history of
seizures and has no focal neurologic findings, neuroimaging is not indicated. It would be inappropriate to
inform his mother that an evaluation is not indicated, because a chromosomal microarray analysis would be
recommended in this case.
Children living with an ID should have appropriate school support and educational intervention, provided in
the least restrictive environment as is appropriate (ideally a regular classroom setting with peers). Children
with moderate to severe ID may require a smaller, self-contained setting with more support and a higher
teacher-to-student ratio. Practitioners in the medical home play a role in monitoring a child’s educational
setting and the supports they have in place, as well as ensuring the goals in the child’s individualized
education plan are appropriate. Resources obtained outside of school may include private therapies (speech,
occupational, physical) and community resources.
Care of individuals with ID in the medical home includes assessment for comorbidities. Seizure, respiratory
and gastrointestinal disorders, and cerebral palsy are more prevalent among individuals with moderate to
profound ID. Other neurobehavioral disorders (eg, attention-deficit/hyperactivity disorder, anxiety,
depression, mood disorders) also occur more frequently in individuals with ID and should be assessed for
and treated appropriately.
Suggested Reading(s)
Moeschler JB, Shevell M; Committee on Genetics. Comprehensive evaluation of the child with
intellectual disability or global developmental delays. Pediatrics. 2014;134(3):e903-e918.
doi:10.1542/peds.2014-1839
Purugganan O. Intellectual disabilities. Pediatr Rev. 2018;39(6):299-309. doi:10.1542/pir.2016-0116
Phelps RA, Cohen WI. Intellectual disability. In: McInerny TK, Adam HM, Campbell DE, Foy JM, Kamat
DM, eds. American Academy of Pediatrics Textbook of Pediatric Care. 2nd ed. American Academy of
Pediatrics; 2016:chap 278. Accessed September 1, 2023. Pediatric Care Online
Content Domain
Behavioral/Developmental
&DQSHUIRUPEDVLFDFWLYLWLHVRI
GDLO\OLYLQJDQGFRQWULEXWHWR
KRXVHKROGWDVNV
6HYHUH 5 25- 8SWRSUHVFKRRO /LPLWHGFRPPXQLFDWLRQDELOLWLHV
40 DQGXQGHUVWDQGLQJRIFRQFHSWV
VXFKDVWLPH
$GDSWHGDQGUHSULQWHGZLWKSHUPLVVLRQIURP3XUXJJDQDQ2,QWHOOHFWXDOGLVDELOLWLHVPediatr
Rev.
AAP PREP 2024 - Question 10/267 Behavioral/Developmental Question 8/9
A 3-year-old girl is seen in the office for a health supervision visit. Findings on developmental surveillance
include that she enjoys playing with her toys next to other children, she is able to tell you her first name
when asked, she can put on her jacket by herself but cannot unbutton her shirt, and she can draw a circle
when shown how.
Of the following, this child MOST likely has a developmental delay in the domain of
A. cognition
B. language/communication
C. motor skills
D. social-emotional skills
Correct answer is D
PREP Pearl(s)
Developmental surveillance should be performed at each health supervision visit from birth to at least
5 years of age.
The core domains of development are cognition, language/communication, motor, and social-
emotional.
By 3 years of age, 75% of children will have achieved the social-emotional developmental milestones of
noticing other children and joining them to play and calming down within 10 minutes after a parent or
caregiver leaves them.
Critique
The child in the vignette has achieved age-appropriate developmental milestones in the
language/communication, cognition, and motor skill domains. However, at 3 years of age, she is still engaging
in parallel play rather than interactive play with other children. This finding indicates a possible delay in her
social-emotional development and should prompt developmental screening using a validated tool (Table).
Recent evidence suggests that 1 in 6 children have a developmental delay in at least 1 domain; however, less
than 25% of these children receive early intervention services before the age of 3 years. Early identification of
children with developmental delay through surveillance at health supervision visits is one of the most
important roles of the pediatrician in the medical home.
Developmental surveillance is defined as “the process of recognizing children who may be at risk for
developmental delays.” This surveillance should be performed at every health supervision visit from birth to
at least 5 years of age, facilitating open and regular communication between the parent or caregiver and the
pediatrician about the child’s development.
The American Academy of Pediatrics (AAP) 2020 Clinical Report Promoting Optimal Development: Identifying
Infants and Young Children With Developmental Disorders Through Developmental Surveillance and
Screening defines 6 components of developmental surveillance:
Eliciting and attending to the parental or caregiver concerns about their child’s development
Obtaining, documenting, and maintaining a developmental history
Making accurate and informed observations of the child
Identifying risks, strengths, and protective factors for the child and the family
Maintaining an accurate record of the process and findings
Sharing (with permission) and obtaining opinions and findings with other professionals (eg, child care
providers, early intervention providers, preschool teachers, and developmental specialists)
The AAP recommends routine developmental screening with a standardized tool at the 9-, 18-, and 30-month
health supervision visits. Evidence has demonstrated that pediatricians cannot solely rely on their judgment
to identify delays in development. In addition, autism spectrum disorder screening is recommended at the
18- and 24- month visits. The pediatrician should strongly consider developmental screening at any age when
a concern is elicited from parents or caregivers or on routine surveillance.
Developmental evaluation (in contrast to screening) is a rigorous assessment using a validated diagnostic
evaluation instrument (eg, Bayley Scales of Infant and Toddler Development) to identify specific
developmental disorders. This type of assessment is performed by trained practitioners.
Suggested Reading(s)
Centers for Disease Control and Prevention. Learn the Signs. Act Early.
https://www.cdc.gov/ncbddd/actearly/index.html
Gleason MM. Emotional or behavioral problems in children younger than 5 years. Point-of-Care Quick
Reference. Pediatric Care Online. American Academy of Pediatrics. March 23, 2022. Accessed March 4,
2023. Pediatric Care Online
Lipkin PH, Macias MM. Promoting optimal development: identifying infants and young children with
developmental disorders through developmental surveillance and screening. Pediatrics.
2020;145(1):e20193449. doi:10.1542/peds.2019-3449
Zubler JM, Wiggins LD, Macias MM, et al. Evidence-informed milestones for developmental surveillance
tools. Pediatrics. 2022;149(3):e2021052138. doi:10.1542/peds.2021-052138
Content Domain
Behavioral/Developmental
AAP Bright Futures AAP Bright Futures Ages & Stages Bayley Scales of
Previsit guidelines (4th ed) Questionnaires Infant and Toddler
Questionnaires (3rd ed) Development (3rd ed)
American Speech- AAP Pediatrics in
Language- Hearing Review articles Ages & Stages Beery-Buktenica
Association Questionnaires: Developmental Test
development charts Social-Emotional of Visual-Motor
Integration (6th ed)
AAP brochure “Is 0RGLͤHG&KHFNOLVW
Your One-Year- Old for Autism in BRIGANCE Early
Communicating Toddlers, Revised Childhood Screens III
With You?”
Parents’ The Capute Scales:
CDC Learn the Evaluation of Cognitive Adaptive
Signs. Act Early. Developmental Test/Clinical
Checklists Status With Linguistic and
Developmental Auditory Milestone
FIRST WORDS Milestones Scale
Project 16 × 16
Survey of Well- MacArthur-Bates
Being in Young Communicative
Children Development
Inventories (2nd ed)
Mullen Scales of
Early Learning
Peabody
Developmental
Motor Scales (2nd
ed)
Preschool Language
Scale-5
*
Diagnostic evaluation tools were cross referenced when there was lack of agreement
supporting a milestone or age of a milestone across other data sources. Not all milestones
were cross referenced with diagnostic resources.
Reprinted with permission from Zubler JM, Wiggins LD, Macias MM, et al.
Evidence-informed milestones for developmental surveillance tools. Pediatrics.
2022;149(3):e2021052138.
AAP PREP 2024 - Question 11/267 Behavioral/Developmental Question 9/9
A healthy 2-year-old is seen for a health supervision visit. His parents are concerned about temper tantrums
and biting that started about a month ago. They read that this behavior could be a sign of autism. There have
been no changes in his daily routine. His parents do not expose him to electronic media and settle their
differences amicably. Both parents work from home. They share parenting responsibilities equally and have
a consistent parenting style that emphasizes positive reinforcement of desired behaviors. The boy is
developing normally. The family history is unremarkable. His physical examination findings are normal.
Of the following, the MOST likely reason for this boy’s behavior is
A. age-appropriate behavior
D. poor parenting
Correct answer is A
PREP Pearl(s)
Temper tantrums and biting are age-appropriate behaviors for 1- to 4-year-olds.
Positive parenting strategies (eg, avoiding triggers, offering acceptable choices so children feel in
control, and redirection) are recommended for managing temper tantrums and biting.
Consistency among caregivers, regardless of the strategy, is critical in managing tantrums and biting.
Critique
The 2-year-old child in the vignette is exhibiting new-onset temper tantrums and biting in the setting of an
apparently safe, stable, and nurturing environment. His behavior is therefore most likely age-appropriate. He
does not have developmental delays or other behavioral concerns, so his behavior is unlikely to be indicative
of autism spectrum disorder or a mental health disorder. His parents have a consistent parenting style that
emphasizes positive reinforcement of desired behaviors, so his behavior is unlikely to be a result of poor
parenting.
Temper tantrums and biting, although developmentally normal for children aged 1 to 4 years, are a common
source of frustration and concern for families. Normalizing the behavior for this age group and framing it as a
way for children to express themselves, especially when they do not yet have language skills, can be helpful
for caregivers. This perspective is especially helpful to those who may fear this behavior reflects faulty
parenting skills.
After providing reassurance, guidance on how a caregiver’s reaction to tantrums and biting can either
encourage or discourage continuation of the behavior, can help families maintain a positive relationship with
their child. For instance, gentle biting may at times be considered endearing (eg, nibbling when the child does
not have teeth), leading caregivers to unintentionally give positive reinforcement. If this feedback continues
as the child develops more independence and willfulness, they may not be able to differentiate between
playful and aggressive biting. Temper tantrums’ inherent disruptiveness may cause caregivers to bring
attention to the behavior while attempting to extinguish it; this may unwittingly encourage the child to
continue, or even escalate, the behavior to achieve a desired outcome.
Positive parenting strategies (eg, avoiding triggers, offering acceptable choices so children feel in control, and
redirection) are recommended for managing temper tantrums and biting. Not giving in to the child’s
demands and ignoring the tantrum can also be helpful. Further exploration of domains of influence (Table)
can help parents understand the reasons for temper tantrums. If tantrums and biting persist beyond age 5 to
6 years, referral to a specialist may be warranted.
Consistency among caregivers, regardless of the strategy, is critical in managing tantrums and biting. Working
together to focus on a child’s strengths can help the family enjoy their time together. Tantrums and biting do
not need to be a major defining aspect of the parent-child relationship, but just one part of interacting with a
growing, developing human being.
Suggested Reading(s)
Hagan JF, Shaw JS, Duncan PM. Promoting mental health. In: Hagan JF, Shaw JS, Duncan PM, eds. Bright
Futures: Guidelines for Health Supervision of Infants, Children, and Adolescents. 4th ed. American
Academy of Pediatrics; 2017:115-150.
Prazar GE. Temper tantrums and breath-holding spells. In: McInerny TK, Adam HM, Campbell DE,
DeWitt TG, Foy JM, Kamat DM, eds. American Academy of Pediatrics Textbook of Pediatric Care.
American Academy of Pediatrics; 2016:chap 201. Accessed March 7, 2023. Pediatric Care Online
White EM, Frazier KF, Scharf RJ. Temper tantrums and breath-holding spells. Pediatr Rev.
2022;43(7):411-413. doi:10.1542/pir.2021-005137
Content Domain
Behavioral/Developmental
Adapted from Hagan JF, Shaw JS, Duncan PM. Promoting mental health. In: Hagan JF,
Shaw JS, Duncan PM, eds. Bright Futures: Guidelines for Health Supervision of Infants,
Children, and Adolescents. 4th ed. American Academy of Pediatrics; 2017:127.
AAP PREP 2024 - Question 12/267 Cardiology Question 1/9
A 15-year-old girl is seen for a health supervision visit. She is new to the practice, having moved to the area 6
months ago. She has a history of tetralogy of Fallot, for which she had surgery at 5 days of age and again at 6
months of age. She has not undergone any additional cardiac interventions. She was routinely followed up by
a cardiologist until 7 years of age; since then, the family has moved several times and has had difficulty
establishing cardiologic care. The girl is otherwise healthy, with no concerns noted at her health supervision
visits.
The girl is not involved in any competitive sports; her only activity is physical education class, in which she
participates fully without difficulty. She received speech therapy from 2 to 8 years of age. She currently has
hypernasal speech, and some words are hard to understand. She has struggled academically since late
elementary school. The girl does not receive any academic assistance, and she does not have an
individualized education plan. Her academic issues have been attributed to frequent school changes. On
physical examination, she has an elevated body mass index (>95th percentile) and normal blood pressure
and heart rate. She has a well-healed median sternotomy scar. There is a harsh III/VI systolic murmur and a
II/IV diastolic murmur along the left sternal border. The remainder of her examination findings are normal.
Of the following, the MOST concerning issue identified at this patient’s visit is the
D. school di culties
Correct answer is D
PREP Pearl(s)
A basic understanding of common forms of congenital heart disease is vital to providing long-term
pediatric care for this population.
Children with repaired and palliated cyanotic congenital heart disease are at high risk of cardiac
comorbidities and require lifelong cardiology follow-up.
Noncardiac comorbidities are common in the congenital heart disease population, especially in
patients with more complex congenital heart disease and/or genetic abnormalities.
Critique
In the vignette, the most concerning issue identified at this visit is the girl’s consistent difficulties with school
performance, especially given her age and lack of any formal educational assessment, assistance, or
accommodation. This issue needs to be addressed promptly. Her other issues of concern, including her
chronic cardiac disease, can be addressed in a stepwise fashion.
Tetralogy of Fallot (TOF), the most common cyanotic congenital heart disease (CCHD), occurs in
approximately 4 to 5 per 10,000 live births and makes up approximately 7% to 10% of all congenital heart
disease (CHD). The other commonly known forms of CCHD include transposition of the great arteries,
hypoplastic left heart syndrome, tricuspid atresia, truncus arteriosus, pulmonary atresia with intact
ventricular septum, and total anomalous pulmonary venous return. There are many other forms of CCHD.
Children with repaired TOF are frequently encountered in general pediatric practice because of the
prevalence of TOF and its excellent operative and postoperative survival rate. A basic knowledge of the short-
and long-term complications of this condition can assist pediatricians in providing optimal care. This is true
for all forms of CHD, but especially for CCHD.
Anatomically, TOF consists of a large ventricular septal defect (VSD; the position of which leads to override of
the aorta) and various combinations of right ventricular (RV) outflow tract obstruction (subvalvar, valvar, and
supravalvar) (Figure). The degree of obstruction to pulmonary blood flow determines the physiology, with
some neonates having completely normal or low normal oxygen saturation and some requiring ductal
patency to provide adequate pulmonary blood flow. The last component of TOF, RV hypertrophy, occurs in
response to the abnormal physiology. This girl’s surgical history suggests she required an additional source of
pulmonary blood flow as a neonate, as evidenced by her surgery at 5 days of age, which is generally provided
through the creation of a systemic to pulmonary artery shunt. Her surgery at 6 months of age was likely a
complete repair of her defect with VSD closure and augmentation of her RV outflow tract (often including the
pulmonary arteries). Children with an acceptable level of desaturation or normal or near-normal saturations
typically undergo surgical intervention between the ages of 3 and 6 months.
Courtesy of A. Johnson
In the United States, the surgical outcomes for TOF are excellent; the 25-year survival approaches 95% for
children undergoing typical or simple TOF repair. The original anatomy and surgical repair history help define
the risk of medium- and long-term issues that may arise, including the following:
Children with repaired TOF are often asymptomatic from a cardiac standpoint but require lifelong cardiologic
surveillance for the known complications. As opposed to simpler forms of CHD (eg, isolated atrial septal
defect or VSD), most children with repaired TOF will require additional cardiac intervention or surgery.
Among the more common later interventions are (1) placement of a competent pulmonary valve (surgically
or via a catheter-based approach; long-standing severe pulmonary regurgitation leads to progressive RV
dilation and the risk of irreversible RV dysfunction) and (2) surgical or catheter-based pulmonary artery
intervention. Lapses in cardiologic care may occur when parents of children with TOF mistakenly believe that
reintervention rates are low or nonexistent.
It is vital for practitioners to recognize that all forms of CHD, especially CCHD, place children at increased risk
for noncardiac morbidity. Neurodevelopmental disorders are very common in this population, with the
highest incidence in children with CCHD and those requiring cardiac surgery as infants. These conditions may
include disorders of language development, disorders of gross and/or fine motor development, inattention,
impulsivity, depression, anxiety, visual or spatial difficulties, and disorders of executive functioning. Some
conditions may not be evident until children reach higher levels of schooling, when greater independence is
required and the work becomes more complex. The higher prevalence of genetic abnormalities associated
with CHD also increases the risk of developmental delay. Developmental delays and learning disorders
leading to academic struggles can exacerbate psychosocial stressors and psychiatric conditions.
The adolescent in the vignette has both abnormal speech and academic difficulties, which are suggestive of
DiGeorge syndrome. The incidence of 22q11.2 deletion in children with TOF is 10% to 15%. The phenotypic
presentation of DiGeorge syndrome is variable; developmental delays are very common.
This girl’s cardiac examination findings are not unusual for a patient with repaired TOF at this age and
consistent with residual RV outflow tract disease. She clearly needs to reestablish cardiologic care; however,
her overall clinical status and lack of symptoms are reassuring. Her obesity is a long-term cardiac risk factor
and known comorbidity in the CHD population. Her obesity will need to be addressed; however, it is not her
most urgent issue. This adolescent’s academic concerns are paramount and should be addressed
expediently.
Suggested Reading(s)
Bellinger DC, Rivkin MJ, DeMaso D, et al. Adolescents with tetralogy of Fallot: neuropsychological
assessment and structural brain imaging. Cardiol Young. 2015;25(2):338-347.
doi:10.1017/S1047951114000031
Lantin-Hermoso MR, Berger S, Bhatt AB, et al; Section on Cardiology; Cardiac Surgery. The care of
children with congenital heart disease in their primary medical home. Pediatrics.
2017;140(5):e20172607. doi:10.1542/peds.2017-2607
Marino BS, Lipkin PH, Newburger JW, et al; American Heart Association Congenital Heart Defects
Committee, Council on Cardiovascular Disease in the Young, Council on Cardiovascular Nursing, and
Stroke Council. Neurodevelopmental outcomes in children with congenital heart disease: evaluation
and management: a scientific statement from the American Heart Association. Circulation.
2012;126(9):1143-1172. doi:10.1161/CIR.0b013e318265ee8a
Puri K, Allen HD, Qureshi AM. Congenital heart disease. Pediatr Rev. 2017;38(10):471-486.
doi:10.1542/pir.2017-0032
Smith CA, McCracken C, Thomas AS, et al. Long-term outcomes of tetralogy of Fallot: a study from the
Pediatric Cardiac Care Consortium. JAMA Cardiol. 2019;4(1):34-41. doi:10.1001/jamacardio.2018.4255
Content Domain
Cardiology
A 12-month-old boy is seen in the emergency department for cough, fever, and decreased oral intake for the
past 2 days. He has had the normal number of wet diapers and no diarrhea. He has a history of hypoplastic
left heart syndrome (HLHS), which was initially palliated with a Norwood-type repair and at 5 months of age
with a bidirectional Glenn procedure (cavopulmonary anastomosis). The boy has been doing well since his
last surgery and receives consistent cardiologic follow-up. The cardiologist had no concerns at the last clinic
visit. His vital signs are as follows: temperature, 38 °C; heart rate, 170 beats/min; respiratory rate, 50
breaths/min; blood pressure, 95/50 mm Hg; and oxygen saturation, 67% in room air (average of 80%-85%
since last surgery).
He is alert and interactive, with mildly increased work of breathing without acute distress. On lung
auscultation, air movement is adequate and equal bilaterally, with mild wheezing and without stridor. He has
well-healed surgical scars. Capillary refill time is 2 to 3 seconds. The remainder of his examination findings
are normal.
B. chest radiography
D. supplemental oxygen
Correct answer is D
PREP Pearl(s)
Management of children with cyanotic congenital heart disease should not be based on classification
into a single category; optimal care requires a basic understanding of the specific cardiac lesion, their
surgical history, and current cardiac physiologic mechanisms.
Some patients with cyanotic congenital heart disease remain at a baseline level of desaturation for the
first several years of life, specifically those undergoing staged surgical palliation.
Many of the mechanisms for acute desaturation in patients with cyanotic congenital heart disease
(CHD) mimic causes seen in the general pediatric population, but often the cardiac physiologic
mechanisms are quite different with CHD, leading to more significant levels of hypoxia.
Critique
The boy in the vignette, who has hypoplastic left heart syndrome (HLHS), is desaturated from his baseline
oxygen level in the presence of respiratory symptoms. Therefore, the best next step in his management is to
provide supplemental oxygen. The mode of delivery should be based on clinical status and the initial clinical
response. In a child without significant respiratory distress, a nasal cannula or simple face mask are
reasonable options.
The boy’s fever, tachypnea, pulmonary examination, and decreased oxygen saturation level suggest a
respiratory infection with lower respiratory tract involvement (eg, viral bronchiolitis). Children with a history
of cyanotic congenital heart disease (CHD) often have abnormal cardiac physiologic findings based on their
underlying disease process and surgical interventions. Management of an intercurrent acute disease, even a
common viral infection, in a child with complex, cyanotic CHD can be daunting, but often typical interventions
for the specific acute disease are still warranted.
Many physicians fear providing supplemental oxygen to children with complex cyanotic CHD. This is based on
the concept that administration of supplemental oxygen can cause pulmonary vasodilation, leading to
increased pulmonary blood flow at the expense of systemic blood flow; this process would then lead to an
increase in pulmonary symptoms and decreased end organ perfusion. However, this physiology is generally
present in patients with HLHS with ductal-dependent blood flow before first-stage palliation and to a lesser
extent after stage 1 palliation (either Norwood type repair or shunt placement). Management of children with
cyanotic CHD based on classification into a single category that does not take into consideration their specific
cardiac lesion, surgical history, and current cardiac physiology is not helpful and may be detrimental to care.
Approximately 25% of infants with CHD have critical CHD, defined as a condition requiring cardiac surgery or
other cardiac procedure(s) in the first year after birth. In the United States, just more than 7,000 infants each
year are born with critical CHD. This boy has HLHS, a form of critical CHD seen in approximately 2 per 10,000
births and representing approximately 2% to 3% of all CHD. Classic HLHS is uniformly fatal without
intervention, which includes immediate stabilization in the neonatal period with prostaglandin E
administration to maintain a patent ductus arteriosus. Stabilization is followed by stage 1 palliation
(Norwood-type intervention), providing unobstructed systemic blood flow and adequate pulmonary blood
flow. Neonates who are receiving prostaglandin or are past stage 1 of palliation have tenuous circulation;
they are described as having single-ventricle physiologic findings. As such, alterations in systemic or
pulmonary vascular resistance lead to changes in both systemic and pulmonary blood flow. Any perturbation
in baseline physiologic findings can significantly impact clinical status, which accounts for the high mortality
rate in children before and after stage 1 palliation.
Stage 2 palliation (bidirectional Glenn anastomosis) alters the cardiac physiologic mechanisms and allows for
passive blood flow to the lungs from the superior vena cava. This procedure is typically performed between
the ages of 3 and 6 months to allow for both physical growth and pulmonary vascular resistance to decrease.
Children who are past stage 2 of palliation remain cyanotic because inferior vena cava blood still enters the
heart and mixes with pulmonary venous return (typical saturations of 75%-85%). Overall, the circulation or
physiologic mechanisms are more stable after stage 2 palliation.
Stage 3 palliation (Fontan procedure) generally occurs between the ages of 3 and 6 years. As a result of this
procedure, all systemic venous return is passively directed to the pulmonary circulation. Saturations after
stage 3 are typically low to near normal. The cardiac physiologic mechanisms are again altered with this
intervention, leading to greater clinical or physiologic stability. However, this sets the stage for chronic cardiac
and noncardiac morbidity.
In a patient with post–stage 2 palliation HLHS, causes of desaturation can be classified into the following
categories:
Pulmonary venous desaturation
Pleural effusion
Pneumonia or infection
Pneumothorax
Pulmonary edema
Systemic venous desaturation
Anemia
Increased oxygen consumption
Low cardiac output
Decreased pulmonary blood flow
Increased pulmonary vascular resistance
Pulmonary artery distortion
Ventilation/perfusion (V/Q) mismatch is common in a variety of pulmonary conditions (eg, bronchiolitis with
and without CHD) and leads to pulmonary venous desaturation. Pulmonary vascular resistance may be
increased due to atelectasis and lung volumes below or above functional residual capacity. Changes in
pulmonary vascular resistance have a greater effect in the presence of passive pulmonary blood flow, as seen
in this patient. Other factors leading to desaturation may include anemia (secondary to inadequate red blood
cell production in the presence of chronic hypoxia) or the combination of an inadequate increase in
myocardial contractility (due to inherent issues with a systemic right ventricle) and increased cellular oxygen
consumption resulting from the intercurrent illness. This boy’s history, vital signs, and examination findings
suggest that systemic saturations may be improved by supplemental oxygen administration to decrease total
V/Q mismatch and potentially lead to pulmonary vasodilation.
Although dehydration can lead to a decrease in intravascular volume and effective pulmonary blood flow, an
intravenous fluid bolus would not be the best initial step in the management of this child. Children that are
past stage 2 palliation are preload dependent; however, this boy’s findings do not suggest significant
dehydration; his tachycardia is likely due to fever. Careful assessment of intravascular volume status is
required during any viral respiratory illness due to increased insensible losses.
Laboratory evaluation may be warranted in this child’s evaluation but is not urgent based on his clinical
presentation. Empiric antibiotics are not required for this boy unless his evaluation raises concern for an
underlying bacterial process. Similarly, chest radiography may be helpful in diagnosing the cause of his acute
condition, but his level of oxygen desaturation is a more pressing issue. The boy’s physical examination does
not suggest a primary pulmonary process for which the results of chest radiography would significantly alter
the initial management. Simple interventions (eg, fever control) may lead to a decrease in heart rate,
respiratory rate, and metabolic demand, which can improve cardiopulmonary interactions and overall clinical
status.
Suggested Reading(s)
Javed R, Cetta F, Said SM, Olson TM, O'Leary PW, Qureshi MY. Hypoplastic left heart syndrome: an
overview for primary care providers. Pediatr Rev. 2019;40(7):344-353. doi:10.1542/pir.2018-0005
Lantin-Hermoso MR, Berger S, Bhatt AB, et al; Section on Cardiology; Cardiac Surgery. The care of
children with congenital heart disease in their primary medical home. Pediatrics.
2017;140(5):e20172607. doi:10.1542/peds.2017-2607
McCulloch MA, Gajarski RJ. Congenital and acquired heart disease. In: McInerny TK, Adam HM,
Campbell DE, DeWitt TG, Foy JM, Kamat DM, eds. American Academy of Pediatrics Textbook of Pediatric
Care. American Academy of Pediatrics; 2023. Accessed October 14, 2021. Pediatric Care Online
Puri K, Allen HD, Qureshi AM. Congenital heart disease. Pediatr Rev. 2017;38(10):471-486.
doi:10.1542/pir.2017-0032
Ronai C, Batlivala SP, Nguyen QT, et al. Paediatrician's guide to post-operative care for functionally
univentricular CHD: a review. Cardiol Young. 2022;32(10):1527-1553. doi:10.1017/S1047951122002943
Content Domain
Cardiology
An 8-year-old boy is seen for recurrent episodes of sharp, midsternal, nonradiating chest pain for the past
year. The pain occurs randomly over the day, mostly at rest, about 2 to 3 times a month. He had 2 short-lived
episodes during physical education class. The pain can be fleeting or last up to 10 minutes. There are no
associated symptoms. The boy is otherwise healthy and receives routine general pediatric care. His parents
are concerned because his 70-year-old grandmother with type II diabetes and hypertension was recently
diagnosed with coronary artery disease. He has normal vital signs and physical examination findings.
A. order echocardiography
B. reassure and counsel the family; plan short- and long-term symptom monitoring
C. refer to a cardiologist and restrict from physical education class until evaluated
Correct answer is B
PREP Pearl(s)
Pediatric chest pain is common and rarely cardiac in nature.
A thorough personal history, family history, and physical examination is the key to determining the
need for further testing or specialty referral.
Physicians need to be familiar with the signs and symptoms suggestive of benign chest pain,
noncardiac chest pain, and cardiac chest pain.
Critique
The best next step in management for the boy in the vignette is to reassure and counsel the family, and plan
short- and long-term symptom monitoring.
Chest pain in the pediatric population is common, accounting for over a half a million medical visits annually.
This symptom is among the most frequent reasons for pediatric cardiology referral. Most cases are benign,
and primary cardiac etiologies are rare (0% to 5%). The etiologies for chest pain span multiple organ systems
(cardiac, respiratory, gastrointestinal, musculoskeletal, and psychiatric/psychogenic). Idiopathic and
musculoskeletal causes are common.
The boy in the vignette’s history and physical examination findings demonstrate no clear “red flags” for a
primary cardiac etiology. Typically, cardiac pain does not occur randomly, is not well localized to the anterior
chest, and is not sharp, stinging, or burning in nature. Long-standing, nonprogressive chest pain is also
unlikely to be cardiac in origin.
History findings that raise concern for a cardiac etiology of chest pain include:
Physical examination findings that raise concern for a cardiac etiology of chest pain include:
Children with recurring chest pain at rest may also experience episodes with exercise; it is important to
determine if the pain is similar in both circumstances.
A past medical history of cardiac disease (congenital or acquired) is an important factor that may influence
the decision to pursue further evaluation. Atrial and ventricular septal defects and mild valve abnormalities
generally do not increase the risk of ischemic heart disease. Congenital heart diseases that involve surgical
manipulation or repair of the coronary arteries and acquired diseases (eg, Kawasaki disease) can increase the
risk for myocardial ischemia. Age is an important factor; chest pain in children younger than age 8 years is
much less commonly associated with a primary cardiac etiology. Young children often confuse chest pain
with other symptoms.
The management of children with nonspecific, subacute to chronic, recurring chest pain who are at low risk
for primary cardiac disease is guided by the findings of a thorough personal and family history and physical
examination. When there are no concerning findings, reassurance and appropriate counseling regarding
signs or symptoms requiring follow-up should be provided. Performance of electrocardiography (ECG) should
be based on the degree of suspicion for a cardiac cause, rather than as a reflexive test. Electrocardiography is
a relatively poor screening test when there is a low prevalence of disease and often leads to unnecessary
testing to evaluate nonspecific findings (ST and T wave changes, isolated voltage criteria for ventricular
hypertrophy, mild conduction delays). Its use in low-risk situations is an inefficient use of medical resources.
Echocardiography should be performed when there is a strong suspicion for structural or functional cardiac
disease. In most cases, referral to a cardiologist for evaluation and determination of the need for
echocardiography is the most cost-effective approach to evaluation of children with chest pain concerning for
a cardiac etiology.
It is appropriate for primary care physicians to stratify the likelihood of a cardiac etiology for chest pain, and,
if low, continue the primary care of this symptom. Cardiology consultation should be sought in the following
cases:
Symptoms suggestive of a cardiac condition develop, new and concerning family history
information is obtained, or there are concerning physical examination changes.
Activity restriction is not needed when there is a low likelihood of a primary cardiac condition.
Such restrictions can lead to unnecessary periods of inactivity/exclusion from sports
participation and negatively impact the psychosocial aspect of children’s lives.
Unless the patient has ongoing chest pain, abnormal examination findings, highly concerning associated
symptoms, or concern regarding access to follow-up with primary or subspecialty care, urgent referral for
further cardiac evaluation/testing is not warranted and is an inefficient use of medical resources. While a
family history of early coronary artery disease (generally defined as <55 years old in males and <65 years old
in females) is concerning, the likelihood of this having any bearing in a young child with cardiac symptoms is
exceptionally low. An important caveat is a family history of early sudden death, especially if the cause of
death is not known. An unclear or confusing family history of unexplained death in a first-degree relative
(parent or sibling), which could be cardiac in origin, should prompt cardiology referral. Recommendations for
situations with non-first-degree relatives are not as clear.
Suggested Reading(s)
Barbut G, Needleman JP. Pediatric chest pain. Pediatr Rev. 2020;41(9):469-480. doi:10.1542/pir.2019-
0058
Chest pain. Point-of-Care Quick Reference. Pediatric Care Online. November 5, 2014. Accessed April 16,
2023. Pediatric Care Online
Friedman KG, Alexander ME. Chest pain and syncope in children: a practical approach to the diagnosis
of cardiac disease. J Pediatr. 2013;163(3):896-901.e1-3. doi:10.1016/j.jpeds.2013.05.001
Powell AW, Pater CM, Chin C, Wittekind SG, Mays WA, Anderson JB, Statile CJ. Implementation of a
pediatric chest pain local consensus guideline decreases the total tests performed without negatively
affecting the yield of abnormal cardiac results. Pediatr Cardiol. 2020;41(8):1580-1586.
doi:10.1007/s00246-020-02414-y
Saleeb SF, Li WY, Warren SZ, Lock JE. Effectiveness of screening for life-threatening chest pain in
children. Pediatrics. 2011;128(5):e1062-e1068. doi:10.1542/peds.2011-0408
Writing Group for Echocardiography in Outpatient Pediatric Cardiology; Campbell RM, Douglas PS,
Eidem BW, Lai WW, Lopez L, Sachdeva R; American College of Cardiology Appropriate Use Criteria Task
Force; American Academy of Pediatrics; American Heart Association; American Society of
Echocardiography; Heart Rhythm Society; Society for Cardiovascular Angiography and Interventions;
Society of Cardiovascular Computed Tomography; Society for Cardiovascular Magnetic Resonance;
Society of Pediatric Echocardiography. ACC/AAP/AHA/ASE/HRS/SCAI/SCCT/SCMR/SOPE 2014
appropriate use criteria for initial transthoracic echocardiography in outpatient pediatric cardiology: a
report of the American College of Cardiology Appropriate Use Criteria Task Force, American Academy
of Pediatrics, American Heart Association, American Society of Echocardiography, Heart Rhythm
Society, Society for Cardiovascular Angiography and Interventions, Society of Cardiovascular Computed
Tomography, Society for Cardiovascular Magnetic Resonance, and Society of Pediatric
Echocardiography. J Am Soc Echocardiogr. 2014;27(12):1247-1266. doi:10.1016/j.echo.2014.10.002
Content Domain
Cardiology
A 10-year-old child with no significant medical history is seen in the emergency department for an acute
onset of chest pain earlier in the day. Four days ago, she was diagnosed with influenza A following 3 days of
fevers and cough. These symptoms have significantly decreased, and she has been afebrile for 48 hours. She
denies dyspnea or palpitations. The chest pain is constant but worsens with coughing and deep breathing.
She has a heart rate of 100 beats/min, respiratory rate of 20 breaths/min, and normal blood pressure for her
age. She has no reproducible chest pain to palpation. The remainder of her physical examination findings are
normal. Electrocardiography results are shown (Figure).
Courtesy of M. Carr
D. prescribe a 10-day course of a nonsteroidal anti-in ammatory drug and follow up in 2 days
Correct answer is B
PREP Pearl(s)
The most common etiology of pericarditis in the pediatric population is viral or postviral.
Pediatric pericarditis has a broad differential and may be the initial presentation of a systemic
inflammatory disorder.
Pericarditis must be differentiated from myocarditis and perimyocarditis, as each diagnosis has
specific implications regarding evaluation, treatment, and activity restrictions.
Critique
This patient’s clinical presentation and electrocardiography (ECG) findings are most suggestive of acute
pericarditis, likely secondary to influenza. The child’s ECG demonstrates mild sinus tachycardia, diffuse ST
segment elevation, and mild PR depression, which are all classic findings of pericarditis. However, as acute
myocarditis can present with similar symptoms, it is important to rule out any significant myocardial
component before making decisions regarding disposition and treatment. Affected children may have
myocardial dysfunction without significant symptoms. Up to one-third of patients with pericarditis have a
mild elevation in troponin level and are deemed to have myopericarditis. Perimyocarditis is diagnosed when
the main abnormality is myocarditis, but there is pericardial involvement demonstrated by symptoms,
pericardial effusion, or typical ECG changes.
Although pericarditis is an inflammatory condition, empiric treatment with oral steroids is not the standard of
care. Data in both pediatric and adult patients suggest a higher rate of recurrence of symptoms when
steroids are used as primary treatment.
Chest radiography would be an appropriate diagnostic test for a child with pleuritic chest pain. However, this
patient’s clinical symptoms and ECG findings are more consistent with pericardial inflammation. While chest
radiography can be helpful in identifying a large pericardial effusion, small- and moderate-sized pericardial
effusions can be missed by chest radiography and cannot be ruled out in the presence of a normal cardiac
silhouette or cardiothoracic ratio.
In general, oseltamivir is effective in decreasing symptoms from influenza A when started within 48 hours of
symptom onset. However, the Red Book 2021-24 Report of the Committee on Infectious Diseases
recommends treatment with oseltamivir for "any hospitalized child with suspected or confirmed influenza
disease, regardless of the duration of symptoms." Oseltamivir would have no impact on pericarditis
symptoms or prognosis.
Nonsteroidal anti-inflammatory drugs (NSAIDs) are often used to treat pericarditis and may be an
appropriate treatment option for this patient, but it is important to rule out myocardial involvement before
beginning anti-inflammatory treatment. Pericarditis is typically viral or postviral in nature. Idiopathic
pericarditis is also common, mostly likely due to the inability to isolate a specific viral pathogen with routine
testing such as a respiratory viral panel. The differential diagnosis of pediatric pericarditis is broad and
includes many infectious and noninfectious etiologies (Table).
Children with bacterial pericarditis are typically very ill. Fungal etiologies are uncommon in
immunocompetent hosts. Primary and metastatic neoplastic etiologies of pericarditis are very rare in
children. However, neoplasms in the anterior mediastinum may lead to secondary pericardial inflammation.
Metabolic etiologies more commonly present with pericardial effusions, which may be minimally
symptomatic unless they are large. Any child who has undergone cardiac surgery or cardiac catheterization is
at risk for post-pericardiotomy syndrome, which typically presents with fever, chest pain, and fatigue from
one to several weeks post-surgery/intervention. Pericarditis can be the presenting abnormality of a systemic
inflammatory condition. Several autoinflammatory conditions that present in childhood can lead to serositis,
including pericarditis. It is important to obtain a thorough noncardiac history to assess for signs and
symptoms pointing to a systemic inflammatory disease.
Most children diagnosed with pericarditis are hospitalized for further evaluation, management of pain, and
treatment of inflammation. All children with an elevated troponin level should be hospitalized. Evaluation of
pericarditis typically includes serial ECGs and baseline echocardiography to assess for a significant pericardial
effusion or evidence of decreased cardiac function, which would suggest more significant myocardial
inflammation. Treatment with NSAIDs for several weeks is the mainstay of therapy. The addition of a several-
month course of colchicine is being used more commonly; recent evidence shows that it may decrease
recurrence. The recurrence rate of pericarditis is approximately 10%, with a higher incidence among children
whose pharmacologic treatment was stopped prematurely. Participation in competitive activity after an
episode of pericarditis should be restricted for a minimum of 3 months. Longer duration restrictions may be
recommended if there is concomitant myocarditis or recurrence of pericarditis-type symptoms.
Suggested Reading(s)
American Academy of Pediatrics. Congenital and acquired heart disease. Point-of-Care Quick
Reference. Point-of-Care Quick Reference. Pediatric Care Online. American Academy of Pediatrics.
2019. Accessed September 1, 2022. Pediatric Care Online
American Academy of Pediatrics. Influenza. In: Kimberlin DW, Barnett ED, Lynfield R, Sawyer MH, eds.
Red Book: 2021-2024 Report of the Committee on Infectious Diseases. 32nd ed. American Academy of
Pediatrics; 2023. Accessed September 1, 2023 Red Book Online
Schwier NC, Tsui J, Perrine JA, Guidry CM, Mathew J. Current pharmacotherapy management of
children and adults with pericarditis: prospectus for improved outcomes. Pharmacotherapy.
2021;41(12):1041-1055. doi:10.1002/phar.2640
Tunuguntla H, Jeewa A, Denfield SW. Acute myocarditis and pericarditis in children. Pediatr Rev.
2019;40(1):14-25. doi:10.1542/pir.2018-0044
Chiabrando JG, Bonaventura A, Vecchié A, et al. Management of acute and recurrent pericarditis: JACC
state-of-the-art review. J Am Coll Cardiol. 2020;75(1):76-92. doi:10.1016/j.jacc.2019.11.021
Content Domain
Cardiology
A 17-year-old boy is seen in the emergency department for 7 days of episodic fevers, fatigue, and occasional
chills. He has a history of tetralogy of Fallot for which he underwent complete repair as an infant and primary
percutaneous pulmonary valve placement 1 year ago. He follows up regularly with a cardiologist; his last
appointment was 6 months ago. The boy was seen by his primary care physician 2 days ago at which time
crackles were heard during his pulmonary examination and he was prescribed azithromycin for presumed
pneumonia. His symptoms have persisted despite initiation of antibiotic treatment. He takes 81 mg/d of
aspirin and no other regular medications and has been taking acetaminophen as needed for fever. He has
had no recent illnesses and was in his usual state of health before the initiation of symptoms.
The adolescent is febrile, has mild tachycardia, and has normal blood pressure. Physical examination reveals
clear lung fields with no increased work of breathing, a III/VI systolic ejection murmur, a II/IV diastolic
murmur, no hepatomegaly, and normal perfusion. The remainder of his examination findings are normal.
A. administer a normal saline bolus and alternate ibuprofen and acetaminophen for fever control
Correct answer is D
PREP Pearl(s)
Infective endocarditis should be considered in a child with congenital heart disease or repaired
congenital heart disease with prolonged fever.
It is imperative to obtain at least 2 independent blood cultures if there is a suspicion for infective
endocarditis.
Appropriate counseling regarding the risk of infective endocarditis in higher-risk patients is an
important part of routine care.
Critique
The best next step in management of the adolescent in the vignette is to obtain at least 2 independent blood
cultures. His history of repaired congenital cardiac disease and pulmonary valve replacement 1 year ago put
him at higher risk for infective endocarditis (IE). Additionally, he has symptoms (fever and chills) suggestive of
a systemic infectious or inflammatory process. He appears compensated and clinically stable; however, with
his history and clinical findings, he requires an evaluation to help determine the cause of his presentation,
including blood cultures. Infective endocarditis can be life-threatening, with a mortality rate of 5% to 10%.
Children with congenital heart disease (CHD) are at higher risk for IE than children with structurally normal
hearts; the level of risk is dependent on the underlying cardiac disease, history of interventions or operations,
and residual lesions. Approximately half of all children diagnosed with IE with concomitant CHD have had
prior cardiac surgery. Ten percent of cases of pediatric IE occur in children with structurally normal hearts,
although most have other risk factors (eg, central venous catheters) and many are infants. Because IE is
uncommon in children, this diagnosis may not readily occur to some pediatric practitioners, which may lead
to a delay in diagnosis.
The pathogenesis of IE is thought to be due to endothelial changes (damaged or denuded tissue), leading to
platelet and fibrin deposition and the development of a nidus for bacterial colonization. Although several
bacterial pathogens can be seen in IE, a handful of bacteria predominate. Therefore, it appears that the
combination of the endothelial changes in the presence of specific bacteria confers risk. The risk associated
with endothelial change can be extrapolated to nonnative tissue or hardware, such as intracardiac patches,
prosthetic cardiac valves, central venous catheters, percutaneously placed cardiac devices (valves, atrial
septal defect or patent ductus arteriosus closure devices, or pacemaker leads). The most common bacterial
pathogens associated with IE are staphylococcal species (aureus and epidermidis) and streptococcal species
(mitis, oralis, and sanguinis). Other bacterial causes include β-hemolytic Streptococcus, Enterococcus, and the
HACEK organisms (Haemophilus, Aggregatibacter, Cardiobacterium, Eikenella, and Kingella).
The diagnosis of IE can be made using the modified Duke criteria (Table). On the basis of his risk factors, this
adolescent requires further evaluation with cardiac imaging (usually transthoracic echocardiography, but
transesophageal echocardiography or computed tomography may be required). If a child being evaluated for
IE is clinically stable and cardiac imaging is not definitive for a vegetation, initiation of antibiotic treatment can
be held pending blood culture results. If there is hemodynamic instability or abnormal findings on cardiac
imaging, treatment with broad-spectrum parenteral antibiotics should be started, after obtaining at least 2
blood cultures. Expert consultation should be sought, preferably with collaboration between cardiology and
infectious disease specialists, to ensure optimal management. The choice of antibiotics and duration of
therapy is based on the specific bacteria isolated, but in all cases several weeks of parenteral antibiotics are
required. The incidence of endocarditis involving percutaneously placed pulmonary valves is increasing as
this intervention becomes more commonplace in children with CHD. Surgical intervention for children with
acute IE is uncommon and reserved for those who experience embolic phenomena, significant valvular
dysfunction, congestive heart failure, or atrioventricular block. Prompt recognition of these complications is
important.
An unexplained fever for 7 days in a child with a history of CHD and pulmonary valve replacement should
prompt further evaluation; completion of the previously prescribed antibiotic without further evaluation,
including blood cultures, is not appropriate for the boy in the vignette. Similarly, basing current management
decisions on a chest radiograph used to confirm or discount the prior diagnosis of pneumonia, without
exploring alternative causes of the symptoms, is not appropriate. Although this adolescent could be
dehydrated due to insensible losses from fever and may benefit from a normal saline bolus, he is currently
hemodynamically stable, and this approach would not further the evaluation and ultimate treatment of the
cause of his condition. Similarly, providing fever control does not address the diagnosis and treatment of the
cause of his symptoms.
Practitioners need to be familiar with the guidelines for antibiotic prophylaxis for certain interventions (eg,
dental procedures). In 2007, the American Heart Association made significant changes to the
recommendations regarding the cardiac lesions that require preprocedural antibiotics and increased the
focus on the prevention of dental disease and the importance of routine dental care in high-risk patients and
patients with CHD. These preventive measures should be stressed for all patients with CHD and repaired
CHD and are considered an integral part of routine longitudinal care.
Suggested Reading(s)
Baddour LM, Cahill TJ, Prendergast BD. Infective endocarditis complicating transcatheter pulmonary
valve replacement: more data, more to learn. J Am Coll Cardiol. 2021;78(6):590-593.
doi:10.1016/j.jacc.2021.05.043
Baddour LM, Wilson WR, Bayer AS, et al; American Heart Association Committee on Rheumatic Fever,
Endocarditis, and Kawasaki Disease of the Council on Cardiovascular Disease in the Young, Council on
Clinical Cardiology, Council on Cardiovascular Surgery and Anesthesia, and Stroke Council. Infective
endocarditis in adults: diagnosis, antimicrobial therapy, and management of complications: a scientific
statement for healthcare professionals from the American Heart Association. Circulation.
2015;132(15):1435-1486. doi:10.1161/CIR.0000000000000296
Cahill TJ, Baddour LM, Habib G, et al. Challenges in infective endocarditis. J Am Coll Cardiol.
2017;69(3):325-344. doi:10.1016/j.jacc.2016.10.066
Wilson WR, Gewitz M, Lockhart PB, et al; American Heart Association Young Hearts Rheumatic Fever,
Endocarditis and Kawasaki Disease Committee of the Council on Lifelong Congenital Heart Disease and
Heart Health in the Young; Council on Cardiovascular and Stroke Nursing; and the Council on Quality of
Care and Outcomes Research. Prevention of viridans group streptococcal infective endocarditis: a
scientific statement from the American Heart Association. Circulation. 2021;143(20):e963-e978.
doi:10.1161/CIR.0000000000000969
Wilson W, Taubert KA, Gewitz M, et al; American Heart Association Rheumatic Fever, Endocarditis, and
Kawasaki Disease Committee; American Heart Association Council on Cardiovascular Disease in the
Young; American Heart Association Council on Clinical Cardiology; American Heart Association Council
on Cardiovascular Surgery and Anesthesia; Quality of Care and Outcomes Research Interdisciplinary
Working Group. Prevention of infective endocarditis: guidelines from the American Heart Association: a
guideline from the American Heart Association Rheumatic Fever, Endocarditis, and Kawasaki Disease
Committee, Council on Cardiovascular Disease in the Young, and the Council on Clinical Cardiology,
Council on Cardiovascular Surgery and Anesthesia, and the Quality of Care and Outcomes Research
Interdisciplinary Working Group. Circulation. 2007;116(15):1736-1754.
doi:10.1161/CIRCULATIONAHA.106.183095
Content Domain
Cardiology
Adapted and reprinted with permission from Li JS, Sexton DJ, Mick N, et al. Proposed
PRGLͤFDWLRQWR'XNHFULWHULDRIGLDJQRVLVRILQIHFWLYHHQGRFDUGLWLVClin Infect Dis. 2000;
30(4):633-638.
AAP PREP 2024 - Question 17/267 Cardiology Question 6/9
A 15-year-old boy with low-grade fever, malaise, and abdominal pain is admitted to the pediatric inpatient
unit of a small community hospital. He has no history of vomiting or diarrhea. His appetite has been poor for
3 days. He has no significant medical history and takes no medications or supplements. Treatment in the
emergency department included acetaminophen for fever and two 500-mL normal saline boluses for
tachycardia, followed by intravenous fluids at a maintenance rate. Abdominal ultrasonography was
performed, the results of which were not concerning for appendicitis.
On arrival at the pediatric department, the boy appears uncomfortable. He continues to experience dull
abdominal pain. His vital signs are as follows: temperature, 38 °C; heart rate, 125 beats/min; respiratory rate,
27 breaths/min; blood pressure (right arm), 110/87 mm Hg; and oxygen saturation, 96% in room air.
The boy is alert and oriented. On cardiac examination, he has an S3 gallop and no murmur. He has diffuse
abdominal pain to deep palpation but no rebound tenderness. He has normal pulses. His extremities are
somewhat cool with no peripheral edema; capillary refill time is mildly prolonged. The remainder of his
physical examination findings are normal.
Chest radiography (Figure 1) and electrocardiography (Figure 2) are performed. N-terminal pro–brain
natriuretic peptide and troponin levels are elevated.
Courtesy of M. Carr
Correct answer is D
PREP Pearl(s)
Myocarditis is a rare and often underrecognized disease process.
Symptoms of myocarditis in children can vary from very mild, nonspecific findings to cardiogenic
shock.
Children with acute myocarditis and signs of cardiac dysfunction require care in a facility capable of
providing pediatric intensive care and mechanical support.
Critique
The most appropriate next step in this boy’s management is to arrange for urgent transfer to a higher level of
medical care. This adolescent’s clinical presentation (history, vital signs, and physical examination findings) is
concerning for acute, compensated heart failure, possibly due to myocarditis. He is at risk for rapid
decompensation and requires care at a center capable of the full spectrum of pediatric intensive care,
including mechanical support.
Pediatric heart failure is uncommon, and the symptoms and clinical presentation can mimic other disease
processes, often leading to a delay in diagnosis. Pediatric myocarditis is also rare; symptoms can range from
mild and nonspecific to acute decompensated heart failure and cardiogenic shock. Myocarditis, an
inflammatory disease of the myocardium, can affect children of any age; there is a male predominance in
older children. Approximately 5% of noninfant pediatric deaths have been attributed to myocarditis, which is
a known cause of sudden cardiac death in the pediatric population. The most common cause of myocarditis
is a viral infection. The causes of myocarditis are listed in Table; there is significant overlap with the causes of
pericarditis.
Children with myocarditis are often misdiagnosed. Mild myocarditis symptoms can be similar to those of a
typical viral illness. More significant cases of myocarditis can present with chest pain and/or a combination of
systemic symptoms (eg, fever, fatigue, and malaise). Because myocardial inflammation can serve as a
substrate for arrhythmia, some affected children present with palpitations or tachycardia. Frequent
premature ventricular contractions, although more often benign in the pediatric age group, can be seen with
myocarditis.
Severe cases of myocarditis generally present as either fulminant myocarditis or a more subacute heart
failure. Children with fulminant myocarditis appear quite ill; they can be misdiagnosed with sepsis or another
severe systemic illness. Often, because of the acute onset and short timeline to cardiac dysfunction in these
cases, the ventricles are not significantly dilated; the cardiac size on chest radiography may be normal.
Fulminant myocarditis can be associated with ventricular ectopy and ventricular arrhythmia. Children with
significant cardiac dysfunction may not present with chest pain or other typical cardiac symptoms;
gastrointestinal concerns are very common. Children with cardiac dysfunction generally remain
hemodynamically compensated for a longer period than do adults but can decompensate quickly, especially
with additional significant stressors (eg, intubation). This decompensation often leads to cardiac arrest, which
can be difficult to recover from, even with appropriate resuscitative efforts. Mechanical support, including
extracorporeal membrane oxygenation (ECMO), may be required. Interestingly, children with fulminant
myocarditis who require mechanical support have a relatively high degree of myocardial recovery when
compared with other forms of heart failure.
The next form of severe myocarditis presents as progressive ventricular dysfunction, which occurs as a result
of both the initial injury (caused by the specific virus) and ongoing myocardial inflammation (mediated by
cytokines and other host inflammatory responses). These children often present several weeks after the
acute viral infection; they have a more insidious course, with a slow progression of heart failure symptoms
due to an evolving dilated cardiomyopathy.
The adolescent in the vignette most likely has fulminant myocarditis. This diagnosis is supported by the short
duration of symptoms and active fever, suggesting an acute viral infection. His abdominal pain without
associated diarrhea or vomiting makes acute viral gastroenteritis less likely. His tachycardia, after
intravascular volume expansion, is out of proportion to his level of fever and hydration. Persistent
tachycardia that is not responsive to intravenous (IV) fluid administration should raise concern for cardiac
dysfunction. Overzealous IV fluid administration may lead to pulmonary edema and a worsening clinical
status, as well as the development of hepatomegaly; children receiving IV fluids should be monitored for
these effects. Additional findings of concern include his mild tachypnea, low normal blood pressure, and
narrow pulse pressure. The S3 gallop is concerning for ventricular dysfunction. Normal pulmonary
examination findings are not unusual in children with cardiac dysfunction.
Although this adolescent does not have significant cardiomegaly on chest radiography, his
electrocardiography findings are abnormal (significantly low voltages and diffuse ST- and T-wave changes);
these findings are consistent with myocarditis. Additionally, his N-terminal pro–brain natriuretic peptide level
is elevated (suggesting ventricular stretch or dilation or dysfunction), as is his troponin level (indicative of
myocardial injury from ischemia or inflammation). Although echocardiography would be helpful to confirm
the degree, there is ample evidence supporting significant myocardial dysfunction.
Aggressive volume resuscitation should be avoided if there is a concern for cardiac dysfunction. Treatment
with a nonsteroidal anti-inflammatory drug could be considered if pericarditis were suspected but is
generally contraindicated in the treatment of myocarditis. Fever control can be helpful to decrease oxygen
consumption, but empiric antibiotic therapy is not indicated in this situation. The boy’s abdominal
examination findings are not suggestive of an acute abdomen, and his physical examination and laboratory
findings raise concern for a cardiac condition; thus, he does not require a surgical evaluation at this time. The
boy requires urgent transfer to a facility that can provide a higher level of care, preferably while he remains in
a somewhat compensated state.
Suggested Reading(s)
Ammirati E, Frigerio M, Adler ED, et al. Management of acute myocarditis and chronic inflammatory
cardiomyopathy: an expert consensus document. Circ Heart Fail. 2020;13(11):e007405.
doi:10.1161/CIRCHEARTFAILURE.120.007405
Law YM, Lal AK, Chen S, et al; American Heart Association Pediatric Heart Failure and Transplantation
Committee of the Council on Lifelong Congenital Heart Disease and Heart Health in the Young and
Stroke Council. Diagnosis and management of myocarditis in children: a scientific statement from the
American Heart Association. Circulation. 2021;144(6):e123-e135. doi:10.1161/CIR.0000000000001001
Tunuguntla H, Jeewa A, Denfield SW. Acute myocarditis and pericarditis in children. Pediatr Rev.
2019;40(1):14-25. doi:10.1542/pir.2018-0044. PMID: 30600275.
Content Domain
Cardiology
Courtesy of M. Carr
AAP PREP 2024 - Question 18/267 Cardiology Question 7/9
A 16-year-old girl is evaluated in the office after her second episode of palpitations followed by presyncope in
the past year. Both events occurred during volleyball practice. She has no current symptoms. Her medical
history is significant for a single unprovoked seizure at the age of 7 years. She underwent a complete
neurologic evaluation at that time, which was unrevealing, and has since been seizure free. The girl is
otherwise healthy and takes no medications or supplements. Her family history is significant for a paternal
grandfather with a myocardial infarction at 55 years old, requiring coronary stent placement, and a maternal
uncle who died in a car accident at age 25 years. Her mother has a history of syncope which was diagnosed
as vasovagal. The adolescent’s vital signs and physical examination findings are normal. Electrocardiography
is performed in the office (Figure).
Courtesy of M. Carr
Correct answer is D
PREP Pearl(s)
The clinical presentation of long QT syndrome may include palpitations, presyncope, syncope, seizures,
or sudden death.
Accurate measurement of the QTc on an electrocardiogram (ECG) is vital to the diagnosis of a
prolonged QT interval.
For all children and adolescents with unexplained cardiac symptoms, it is important to obtain a
thorough family history that should include individuals with early or unexplained death (including the
mechanism of death).
Critique
The adolescent in the vignette’s electrocardiogram (ECG) demonstrates significant QT prolongation (QTc ~500
ms). Her symptom history, although infrequent in event number, is concerning given the association with
exercise. Her family history raises red flags for a primary cardiac abnormality (maternal syncope, paternal
grandfather with early myocardial infarction, and maternal uncle who died in a car accident; if he was the
driver there could have been a cardiac event). This adolescent requires urgent cardiology consultation, which
could be accomplished by speaking directly with the consultant and obtaining expert interpretation of the
ECG and clinical guidance. Any individual with newly diagnosed, significant QT prolongation should be
restricted from competitive and high-intensity activity until expert evaluation occurs, due to the risk for
arrhythmia.
Long QT syndrome (LQTS) is a form of cardiac electrical disease, specifically of repolarization. It can be
congenital or acquired, and predisposes the individual to syncope, seizures, and sudden death secondary to
a malignant ventricular arrhythmia (ie, torsades de pointe). Prevalence is estimated to be 1:2,000 individuals,
with a slight female predominance. Long QT syndrome results from a specific genetic mutation in one of the
cardiac ion channels. There are currently 17 different identified mutations; most clinical disease is accounted
for by 3 subtypes, LQT1, LQT2, and LQT3, all of which are inherited in an autosomal dominant pattern. In the
vast majority of LQTS, there is a loss of function of voltage gated potassium channels (LQT1, LQT2); LQT3 has
a gain of function of a sodium channel.
The phenotypic presentation of LQTS varies by subtype, with clinical symptoms typically associated with
specific events or activities:
Episodes of torsades de pointes can be self-resolving, with clinical presentations including episodic
palpitations/tachycardia, presyncope, syncope, or anoxic seizures; or an episode can degenerate into life-
threatening ventricular fibrillation. Long QT syndrome is found to be the etiology in 5% to 10% of cases of
sudden cardiac death and approximately 15% of cases of resuscitated, unexplained cardiac arrest.
As most LQTS mutations are autosomal dominant, it is vital to obtain a thorough family history for children
and adolescents with cardiac symptoms; this history should focus on the following:
Physicians should interpret with caution family descriptions of unexplained early death in relatives from
“heart attacks” or other cardiac etiologies, especially in the absence of known underlying cardiac disease in
the decedent or in the absence of an autopsy diagnosis. This is especially true if there are multiple affected
relatives. Postmortem molecular autopsies, which can identify LQTS mutations, are becoming more common
but are far from routine.
Acquired long QT in the pediatric population is most commonly related to medications. Electrolyte
derangements (eg, hypokalemia and hypocalcemia) can lead to QT prolongation; these are most often
encountered in urgent care settings, emergency departments, and in inpatient settings (particularly among
those requiring intensive care). Children in the intensive care unit are often subject to a combination of QT
prolonging drugs and electrolyte abnormalities. Electrolyte abnormalities and/or administration of QT-
prolonging medications place individuals with genetic LQTS at increased risk for torsades de pointes. A
subset of individuals that appear to have acquired LQT, actually have genetic LQTS that was uncovered by
medications or electrolyte disturbances.
Technically, an exact calculation of the corrected QT requires manual or digitally assisted measurement.
However, estimates can be made as follows:
Calculation of corrected QT using the example ECG (Figure above):
QTc = 500 ms
These intervals can be over- or underestimated without manual measurements. While expert consultation is
required in the presence of any prolonged QT noted on an ECG, this should be expedited in any individual
with a QTc ≥500 ms and a concerning symptom history or family history. The family should be explicitly
counseled about the importance of restriction from competitive and high-intensity activity, whether or not
there are active symptoms, pending further evaluation by a cardiologist.
Evaluation with a Holter monitoring may be indicated as part of this girl’s evaluation, but this is best decided
by the consulting cardiologist. Instituting measures to improve fluid intake is appropriate for adolescents
experiencing presyncopal and syncopal events related to relative dehydration, when the concern for a
primary cardiac etiology is low; however, that is not the case for the girl in the vignette. Similarly, continued
monitoring without further evaluation is not appropriate in this situation, where there is a high suspicion for
a cardiac abnormality.
Suggested Reading(s)
Etheridge SP, Cohen MI. An overview of diagnosis and management strategies for long QT syndrome. J
Innov Card Rhythm Manag. 2017;8(6):2750-2757. doi:10.19102/icrm.2017.080605
Krahn AD, Laksman Z, Sy RW, Postema PG, Ackerman MJ, Wilde AAM, Han HC. Congenital long QT
syndrome. JACC Clin Electrophysiol. 2022;8(5):687-706. doi:10.1016/j.jacep.2022.02.017
Peter Harris J. Cardiac arrhythmias. In: McInerny TK, Adam HM, Campbell DE, DeWitt TG, Foy JM, Kamat
DM, eds. American Academy of Pediatrics Textbook of Pediatric Care. American Academy of Pediatrics;
2021:chap 132. Pediatric Care Online
Wallace E, Howard L, Liu M, O'Brien T, Ward D, Shen S, Prendiville T. Long QT syndrome: genetics and
future perspective. Pediatr Cardiol. 2019;40(7):1419-1430. doi:10.1007/s00246-019-02151-x
Content Domain
Cardiology
An 18-month-old girl diagnosed with left-sided cervical lymphadenitis is admitted to the general pediatrics
inpatient unit. She initially presented to the emergency department 6 days ago with 4 days of fever, an
erythematous truncal rash, and left cervical tenderness. Computed tomography of the neck confirmed a
phlegmon in the left anterior cervical lymph node chain. She was evaluated by an otolaryngologist and given
ampicillin-sulbactam. Her fever and rash have persisted despite 2 days of antibiotic treatment, and her neck
findings are unchanged. Today she developed bilateral nonpurulent conjunctivitis and oropharyngeal
erythema. The remainder of her physical examination findings are normal. Laboratory evaluation
demonstrates an elevated C-reactive protein level, mild transaminitis, hypoalbuminemia, and hyponatremia.
Her complete metabolic profile and complete blood cell count are otherwise normal.
Echocardiography is performed because of a concern for Kawasaki disease. Although the findings were
somewhat limited because of patient movement, this imaging demonstrates a structurally and functionally
normal heart with mild mitral regurgitation and a small pericardial effusion. The dimensions of the proximal
coronary arteries are normal.
B. broaden the antibiotic coverage and have the otolaryngologist reevaluate the child
C. continue the current treatment plan and monitor for additional diagnostic criteria
Correct answer is A
PREP Pearl(s)
Kawasaki disease is a clinical diagnosis that can be supported by laboratory and echocardiographic
data. Diagnosis often requires reevaluation of signs and symptoms.
A high index of suspicion is required for the diagnosis of Kawasaki disease, which may be
misdiagnosed as bacterial lymphadenitis, urinary tract infection, meningitis, a gastroenterologic
condition (including appendicitis), or a systemic bacterial or viral infection.
Intravenous immunoglobulin and oral aspirin are the mainstays of therapy for Kawasaki disease. The
addition of steroid therapy is becoming more common, especially in higher-risk patients.
Critique
The child in the vignette meets the clinical criteria for Kawasaki disease (KD) and has laboratory findings
supportive of the diagnosis. Therefore, the best next step in her management is to begin treatment with
intravenous immunoglobulin (IVIG) and oral aspirin. It would not be appropriate to broaden the antibiotic
coverage and have the otolaryngologist reevaluate the child or to initiate evaluation for an underlying
systemic inflammatory condition.
Kawasaki disease is a medium vessel vasculitis of uncertain origin that primarily affects young children (<5
years of age). Although KD is believed to be caused by a viral pathogen, no specific virus has been identified.
Rash
Maculopapular
Diffuse erythroderma
Erythema multiforme–like
Bilateral, nonexudative (often limbic-sparing) conjunctivitis
Extremity changes
Erythema of palms and soles or induration or edema of hands and feet
Oral changes
Lips: erythema, dryness, cracking
Strawberry tongue
Diffuse erythema of the oropharyngeal mucosa
Cervical lymphadenopathy (least common; often misdiagnosed)
Unilateral (>1.5 cm)
This child meets the criteria for KD: fever for 6 days, rash, nonpurulent conjunctivitis, oropharyngeal
erythema, and lymphadenitis. Lymphadenopathy, typically unilateral and confined to the anterior cervical
chain, is the least commonly seen criterion of KD; however, this finding often leads to misdiagnosis or delay
in the diagnosis. Computed tomography may demonstrate involvement of multiple lymph nodes and/or
retropharyngeal edema or phlegmon formation. The typical clinical features of KD are often ignored, missed,
or attributed to the suspected bacterial illness (eg, the rash may be attributed to antibiotics used to treat
lymphadenitis) because they can present at different times during the acute stage of the disease.
Kawasaki disease is a systemic vasculitis; therefore, multiple organ systems may be involved in the acute
disease process, including the following:
Liver (hepatitis)
Lung (interstitial pneumonitis)
Gastrointestinal tract (abdominal pain, diarrhea, vomiting, or gallbladder hydrops)
Pancreas (pancreatitis)
Urinary tract (urethritis or meatitis)
Musculoskeletal (arthralgia or arthritis)
Lymphatics (lymphadenitis)
Central nervous system (aseptic meningitis, irritability, sensorineural hearing loss, or facial nerve palsy)
Ophthalmologic (anterior uveitis)
Macrophage activation syndrome
Although not diagnostic, several laboratory findings support the diagnosis of KD, including the following:
Echocardiography is diagnostic for KD if there is coronary artery dilation (coronary dimension of >2.5 z
scores) in the presence of typical KD clinical findings. Other cardiovascular manifestations may include the
following:
Myocarditis (elevated troponin I or T levels)
Valvulitis (typically mitral regurgitation)
Pericarditis (clinical symptoms or a pericardial effusion noted on echocardiography)
Aortic root dilation
Peripheral gangrene (secondary to thrombosis of other medium-sized arteries)
Treatment when there are coronary artery aneurysms includes an antiplatelet agent (aspirin or clopidogrel)
for small to moderate aneurysm and anticoagulation (heparin or enoxaparin) for large or giant aneurysms.
Repeat echocardiography is key in the short- and long-term management of KD, regardless of the initial
findings. If there were a primary bacterial cause of this girl’s condition, her continued fever and cervical
lymph node findings could represent a failure of the current antibiotic regimen. However, she has findings
not commonly associated with lymphadenitis, which should lead to a broadening of the differential diagnosis.
Continuation of this girl’s current treatment with monitoring is not appropriate because she meets criteria for
classic KD. For children with findings suggestive of incomplete KD (fever for at least 5 days with 2 or 3 classic
findings or infants ≤6 months of age with unexplained fever for ≥7 days or 3 days of fever and a strong
clinical suspicion of KD), echocardiography and further laboratory evaluation (as noted above) should be
performed, along with continued monitoring of fever and for the development of any new clinical symptoms.
The differential diagnosis of KD includes bacterial and viral infections, systemic inflammatory diseases, and
drug-related reactions. Although it is important to consider these diagnoses, it is also important to begin
treatment for KD when classic criteria are met; initiation of IVIG decreases the risk of coronary artery
aneurysm formation from 25% to approximately 4%. The risk of delayed treatment for a child with KD far
outweighs the risk of treating a child for KD who is subsequently diagnosed with another disease process.
Suggested Reading(s)
Friedman KG, Gauvreau K, Baker A, et al. Primary adjunctive corticosteroid therapy is associated with
improved outcomes for patients with Kawasaki disease with coronary artery aneurysms at diagnosis.
Arch Dis Child. 2021;106(3):247-252. doi:10.1136/archdischild-2020-319810
Jackson MA. Kawasaki disease. In: McInerny TK, Adam HM, Campbell DE, DeWitt TG, Foy JM, Kamat DM,
eds. American Academy of Pediatrics Textbook of Pediatric Care. 2nd ed. American Academy of
Pediatrics; 2017:chap 280. Pediatric Care Online
McCrindle BW, Rowley AH, Newburger JW, et al; American Heart Association Rheumatic Fever,
Endocarditis, and Kawasaki Disease Committee of the Council on Cardiovascular Disease in the Young;
Council on Cardiovascular and Stroke Nursing; Council on Cardiovascular Surgery and Anesthesia; and
Council on Epidemiology and Prevention. Diagnosis, treatment, and long-term management of
Kawasaki disease: a scientific statement for health professionals from the American Heart Association.
Circulation. 2017;135(17):e927-e999. doi:10.1161/CIR.0000000000000484
Selamet Tierney ES, Runeckles K, Tremoulet AH, et al; International Kawasaki Disease Registry.
Variation in pharmacologic management of patients with Kawasaki disease with coronary artery
aneurysms. J Pediatr. 2022;240:164-170.e1. doi:10.1016/j.jpeds.2021.08.072
Son MBF, Newburger JW. Kawasaki disease. Pediatr Rev. 2018;39(2):78-90. doi:10.1542/pir.2016-0182
Content Domain
Cardiology
A 15-year-old boy is being monitored for elevated blood pressure that was incidentally discovered at a health
supervision visit. He is otherwise healthy and plays baseball for the high school team. He takes no
medications or over-the-counter supplements. There is a family history of hypertension in his mother and
several of her relatives. His body mass index is at the 80th percentile for age. Blood pressure measurements
during the past 4 months have all been in the stage 1 hypertension range. The results of basic laboratory
evaluation were unremarkable, and recent echocardiography did not demonstrate left ventricular
hypertrophy. During the past 4 months, he has made lifestyle changes, including dietary changes and
exercising at a gym.
Correct answer is D
PREP Pearl(s)
Pediatric hypertension diagnosis, evaluation, and initial management are within the scope of practice
of the general pediatrician.
Pediatricians should be knowledgeable about commonly used antihypertensive medications, including
their effects and adverse effects.
Clinical practice guidelines exist for the diagnosis, evaluation, and management of pediatric
hypertension.
Critique
On the basis of this adolescent’s duration of blood pressure readings in the stage 1 hypertension range,
despite having implemented appropriate lifestyle changes, pharmacologic treatment should be initiated.
Lifestyle changes are an important component of the long-term management of hypertension.
There are several options for pharmacologic treatment of pediatric hypertension (Table 1). Primary care
physicians should be knowledgeable about commonly used medications, including their effects and adverse
effects.
Typically, a baseline renal function laboratory panel is obtained before the initiation of angiotensin-
converting enzyme inhibitors, angiotensin receptor blockers, and thiazide diuretics.
Recommended antihypertensive medication for children with chronic conditions are listed in Table 2.
Pediatric hypertension is becoming more common; the initial evaluation and management of this condition
are well within the realm of general pediatricians. The American Academy of Pediatrics has published
extensive guidelines for the diagnosis and management of pediatric hypertension (Flynn 2017). They include
specific recommendations for the follow-up of all levels of blood pressure elevation and are intuitive to
follow. The most recent guidelines were modified to reflect the changes in the definition of hypertension in
adults. The adolescent in the vignette had hypertension on a routine examination. An appropriate history
was obtained and physical examination performed (including body mass index) to rule out secondary causes
of hypertension. Other important components of the hypertension history are medication use, a history of
prematurity, and snoring or obstructive sleep apnea. He was appropriately scheduled for follow-up of stage 1
hypertension readings, in addition to a trial of lifestyle modification and basic laboratory screening for
secondary causes. Before pharmacotherapy is started, baseline echocardiography is recommended to assess
for left ventricular hypertrophy, which is considered evidence of end-organ change and the potential need for
more aggressive treatment. Echocardiography can also rule out coarctation of the aorta, which should have
already been assessed for with a detailed physical examination and by obtaining upper and lower extremity
blood pressures.
Simply continuing to follow up blood pressures that have consistently been in the stage 1 hypertension range
for several months is not appropriate. On the basis of the history provided, there is no indication for renal
ultrasonography. Renal ultrasonography is recommended in the evaluation of hypertension for children
younger than 6 years and individuals with laboratory evidence of abnormal renal function or screening
urinalysis. Given this boy’s physical examination findings, personal and family history, normal laboratory
evaluation findings, and blood pressures in the stage 1 hypertensive range, there is no reason to restrict
competitive activity, and in fact, this may be detrimental both medically and psychosocially.
Suggested Reading(s)
Chandar J, Messiah SE, Zilleruelo G, Lipshultz SE, 2016. High blood pressure. In: McInerny TK, Adam
HM, Campbell DE, DeWitt TG, Foy JM, Kamat DM, eds. American Academy of Pediatrics Textbook of
Pediatric Care. 2nd ed. American Academy of Pediatrics; 2017:chap 163. Pediatric Care Online
Flynn JT, Kaelber DC, Baker-Smith CM, et al; Subcommittee on Screening and Management of High
Blood Pressure in Children. Clinical Practice Guideline for Screening and Management of High Blood
Pressure in Children and Adolescents. Pediatrics. 2017;140(3):e20171904. doi:10.1542/peds.2017-1904
Weaver DJ Jr. Hypertension in children and adolescents. Pediatr Rev. 2017;38(8):369-382.
doi:10.1542/pir.2016-0106. PMID: 28765199.
Weaver DJ Jr. Pediatric hypertension: review of updated guidelines. Pediatr Rev. 2019;40(7):354-358.
doi:10.1542/pir.2018-0014
Content Domain
Cardiology
$EEUHYLDWLRQV$&(,DQJLRWHQVLQFRQYHUWLQJHQ]\PH,QKLELWRU$5%DQJLRWHQVLQUHFHSWRU
EORFNHU
&RXUWHV\0&DUU
&KURQLFNLGQH\GLVHDVHDQGSURWHLQXULD $&(,$5%
7\SHRUW\SHGLDEHWHV $&(,$5%
&HUWDLQFRQJHQLWDORUDFTXLUHGKHDUWGLVHDVH $&(,
ZLWKYHQWULFXODUG\VIXQFWLRQ
$EEUHYLDWLRQV$&(,DQJLRWHQVLQFRQYHUWLQJHQ]\PHLQKLELWRU$5%DQJLRWHQVLQUHFHSWRU
EORFNHU
&RXUWHV\0&DUU
AAP PREP 2024 - Question 21/267 Child maltreatment Question 1/2
A 4-week-old girl is brought to the office by her mother for a scheduled weight check. Her birth history is
unremarkable. The mother reported difficulty with formula feeding at her 2-week appointment and today
reports that the infant continues to be fussy while feeding as well as crying intermittently throughout the day
and night. The mother states, “I thought she would be sleeping through the night by now.” The infant lives in
a home with her mother, stepfather, and twin brother.
The infant’s weight has increased appropriately from the last visit. There are skin changes on her left cheek
(Figure). On inquiry, the mother reports no history of trauma.
Reprinted with permission from Visual Diagnosis [CD-ROM]. 3rd ed. American Academy of Pediatrics; 2008.
Correct answer is B
PREP Pearl(s)
Pediatricians who suspect child abuse are mandated by law to make a report to child protective
services; failure to make a report can result in legal consequences for the physician and lethal
consequences for the child.
It is important for the pediatrician to discuss a mandated child protective service report with the
nonoffending caregiver and assist the family with the investigation process, serving as a point of
contact and medical home for coordination of follow-up care.
When a child is suspected of being abused, their siblings should be evaluated for injuries. Twin siblings
are at increased risk.
Critique
The infant in the vignette has a bruise on her left cheek. Developmentally, she is preambulatory and likely
cannot roll over; thus, she would not be expected to sustain an injury to a soft area of her face with no known
trauma history. The bruise pattern resembles a handprint, further heightening the suspicion of abuse.
Pediatricians are mandatory reporters and can be held legally responsible if they fail to report potential child
abuse. The best next step in this infant’s management is to make a report to child protective services (CPS).
A patterned slap mark on the face is a suspicious finding in an ambulatory child (Table 1) and even more
concerning in a preambulatory infant. The TEN-4-FACES rule is outlined as follows:
Torso
Ear
Neck
Younger than 4 years/infant younger than 4 months
Frenulum
Angle of jaw
Cheeks
Eyelids
Subconjunctivae or sclera
Patterned bruise
The TEN-4-FACES rule can guide pediatricians in determining which bruises are more likely to result from
abuse. The infant in the vignette meets criteria for the number 4 and letters C and P. The infant’s bruise can
be categorized as a sentinel injury, a relatively minor injury that is suggestive of child abuse. Children who
experience abuse often experience recurrent trauma; sentinel injuries may be the only indication to
investigate for additional trauma and thereby prevent additional harm.
Suspicion of abuse is all that is required to make a CPS report. Physicians do not need to prove certainty or
intent, and they are not responsible for determining who committed the abuse. Consultation with
subspecialists, including child abuse pediatricians, is encouraged when evaluation and management are
uncertain and further guidance is needed. Depending on the severity of injury or situational safety concerns,
hospitalization may be necessary. The child's medical care must be the priority. Although it may be
uncomfortable to discuss concerns with the nonoffending parent, it is recommended that pediatricians do so
early to provide support for the family and to remind them that physicians have ethical and legal obligations
in cases of suspected abuse. When the pediatrician has concerns that the presenting parent is the alleged
perpetrator, such as in the vignette, it is best to consider patient safety and ask for guidance from CPS
personnel about disclosing a report.
It is important that the pediatrician remains a point of contact and medical home for the affected child,
providing guidance, resources, assistance, and reassurance to the family. Pediatricians may need to work
with investigators to describe medical findings and concerns.
The infant in the vignette has known risk factors and triggers for child abuse (Table 2). Her mother reported
fussiness with feeding and intermittent crying, and she expressed an expectation that her child should be
sleeping through the night. It is common for parents to misunderstand age-appropriate development; it is
vital for the pediatrician to provide anticipatory guidance at each visit. In addition, the infant in the vignette
has a twin sibling. Siblings of abused children are at increased risk for abuse, especially twin siblings, and
should be evaluated for injuries.
Although it is important to provide counseling and anticipatory guidance, including discussing expected
behavior during each visit as children develop, the infant’s safety is the critical factor at this time; therefore,
this would not be the best next step in the management of the infant in the vignette. Laboratory testing for
hematologic disorders might be appropriate in some cases to rule out a medical explanation for bruising;
however, it would not be the best next step for the infant in the vignette with an unexplained bruise. A
bleeding disorder would not explain the infant’s patterned injury. Scheduling a follow-up visit the next day
leaves the infant at risk for additional abuse.
With increased awareness and knowledge about abusive injuries and use of a multidisciplinary approach for
suspected abuse cases, pediatricians will become more comfortable making CPS reports and supporting
families. Physician familiarity with community resources can both assist families during an abuse
investigation and, importantly, facilitate appropriate family support before abuse occurs.
Suggested Reading(s)
American Academy of Pediatrics. Visual Diagnosis of Child Abuse [CD-ROM]. 3rd ed. American Academy
of Pediatrics; 2008.
Anderst JD, Carpenter SL, Abshire TC; Section on Hematology/Oncology and Committee on Child Abuse
and Neglect; Hord J, Crouch G, Gregory Hale G, et al. Evaluation for bleeding disorders in suspected
child abuse. Pediatrics. 2013;131(4):e1314-e1322. doi:10.1542/peds.2013-0195
Christian CW; Committee on Child Abuse and Neglect, American Academy of Pediatrics. The evaluation
of suspected child physical abuse. Pediatrics. 2015;135(5):e1337-e1354. doi:10.1542/peds.2015-0356
Flaherty EG, Stirling J; The Committee on Child Abuse and Neglect. The pediatrician's role in child
maltreatment prevention. Pediatrics. 2010;126(4):833-841. doi:10.1542/peds.2010-2087
Sheets LK, Leach ME, Koszewski IJ, Lessmeier AM, Nugent M, Simpson P. Sentinel injuries in infants
evaluated for child physical abuse. Pediatrics. 2013;131(4):701-707. doi:10.1542/peds.2012-2780
Content Domain
Child maltreatment
A 6-year-old boy is seen for a health supervision visit. There is a note in the chart that his mother has
concerns about her boyfriend’s behavior around her son. Prior to the physical examination, she steps out to
use the restroom. While she is gone, the boy says that someone at home gave him money to keep a secret
about seeing his private area. He then points to his buttocks and says, “He touched me there last night.”
Correct answer is D
PREP Pearl(s)
Disclosure is a process; it may take months or years for children to tell someone they trust, such as
their pediatrician, about their abuse.
Pediatricians should be familiar with effective ways to obtain further details when children disclose
abuse, as this may be necessary if community resources are limited and decisions for management
need to be made.
Critique
The boy in the vignette spontaneously disclosed potential sexual abuse. Using the phrase “what happened
next” is open ended and leaves it up to the boy to speak freely about any additional details that he feels
comfortable sharing.
Many children look to their pediatrician as a trusted adult they can speak to. Although most pediatricians do
not have training in forensic interviewing, they should be informed about how to respond when a child
discloses abuse, as well as the next steps in management. Further questioning by the general pediatrician
may be necessary if there are limited community resources available and decisions need to be made about
referring and reporting. The pediatrician should provide comfort and support for the child if a disclosure is
made. The child should be reassured that they are in a safe environment to continue speaking and that the
pediatrician is there to help take care of their health. Having an established relationship can help the child
feel more comfortable. It is recommended to speak to the child alone, without the parent present. Asking
about nonthreatening topics, such as a favorite color or a pet’s name, prior to asking questions related to the
abuse may help to build rapport and ease discomfort. Language and terminology should be tailored to the
child’s age and development.
The pediatrician should focus on open-ended, non-leading questions to gather further information (Table).
Documentation of the disclosure is vital during an investigation. Quotation marks should be used when
documenting the child’s statements; it is helpful to record the question that was asked in association with the
child’s answer. Because additional questioning by professionals trained in forensic interviewing may follow,
the child should not be pressured to tell the entire story or provide every detail. It is important to gather
information that will guide the medical examination, if possible. The timing of the incident and any current
complaints can help guide testing, treatment, and reporting.
Forensic interviews are typically conducted by trained social workers, child protective service workers, and
law enforcement members. There are numerous forensic training courses for initial certification and
recertification. Child advocacy centers, if present in the community, can facilitate the process for a child and
their family. These centers usually house a multidisciplinary team in a child-friendly atmosphere. The
interviews take place in a closed room with video or audio documentation, if available, or a one-way mirror,
for legal and educational purposes. Interviewers are trained to ask questions appropriate for the child’s age,
development, and abilities. The interviews are structured in stages—introduction and explanation of the
process, gathering details about the abuse, and closure to answer any questions and reassure the child.
Throughout the interview, rapport is maintained, always starting with questions not related to abuse. This
also provides an opportunity to assess language development and understanding. Tools, such as anatomical
drawings and dolls, can be used to confirm or clarify statements. The interview can guide the forensic
medical examination, especially if more details are revealed than what was initially reported.
As evidenced by the 2017 “Me Too” movement, disclosure is a personal process and it can often take years
for survivors to verbalize abuse. Grooming is the term used to describe how perpetrators coerce their victims
over a period of time, developing trust and a relationship that is confusing for a child and becomes a barrier
to disclosing. Tactics used by perpetrators include secrets, gifts, playfulness, manipulation, and sometimes
threats. Children may have a hard time distinguishing boundaries and recognizing abusive acts. They may not
be aware of safe and unsafe touch, and relationships may blur the lines of acceptable behavior. It is not
uncommon for the child to know the perpetrator, such as a family member or family friend. Disclosure may
be delayed for concern of punishment, fear of the perpetrator, or distress about not being believed. Details
of the disclosure may be incomplete and additional information may be disclosed at a later time. Sometimes,
children recant their disclosure, especially if they receive a negative reaction from the people they tell.
The other response choices are incorrect because they are examples of leading and suggestive questions.
The boy stated that the incident occurred the night before; however, asking a question that includes multiple
choices about the number of times should be avoided. It is not appropriate to ask why questions, as this
requires higher-level thinking and can insinuate blame. Although the mother expressed concerns about her
boyfriend, naming him in a question without the boy stating his involvement first is suggestive and not
recommended. In cases of suspected abuse, the pediatrician should consult with child abuse specialists when
possible and remember that a report to authorities is mandated if there is a suspicion of abuse.
Suggested Reading(s)
Chiesa A, Goldson E. Child sexual abuse. Pediatr Rev. 2017;38(3):105-118. doi:10.1542/pir.2016-0113
Faller KC. Interviewing the prepubertal child for possible sexual abuse. In: Reece RM, Christian CW, eds.
Child Abuse: Medical Diagnosis & Management. 3rd ed. American Academy of Pediatrics; 2008:257-
268.
Jenny C, Crawford-Jakubiak JE; Committee on Child Abuse and Neglect; Christian CW, Flaherty EG,
Leventhal JM, Lukefahr JL, Sege RD. The evaluation of children in the primary care setting when sexual
abuse is suspected. Pediatrics. 2013;132(2):e558-e567. doi:10.1542/peds.2013-1741
Content Domain
Child maltreatment
Preferred Questions/Probes
1. General question Open-ended inquiry 1. How are you today?
about the child's 2. How have you been
well-being or salient feeling?
issues; does not assume
abuse may have occurred
2. Open abuse-related Open-ended inquiry that 1. Do you know why I am
question assumes there may be talking to you?
Directive question abuse or trauma 2. I see from your
medical examination
that something may have
happened to you. Tell me
about it as best you can.
3. Invitation or Utterances that invite 1. Can you tell me
invitational question free recall and a narrative everything you can
remember?
2. Tell me all about what
happened, from the
beginning to the end.
4. Facilitative cue Interviewer gesture or ut- 8KKXK DIͤUPDWLYH
Narrative cue terance aimed at encour- 2. Okay
Facilitators aging more narration 3. Anything else?
4. What happened next?
A 24-hour-old neonate is seen in the nursery with his mother, who has concerns about feeding. He was born
vaginally after an unremarkable pregnancy and delivery. He is the mother’s first child. She is attempting to
breastfeed but is having difficulty with latching and expressing breastmilk. Her mother and aunts want her to
stop breastfeeding and feed the neonate formula exclusively because he seems to cry all night and they
never breastfed any of their children. The mother wants to breastfeed on the basis of advice from her
physician, but she does not want to go against her family’s recommendations. The neonate’s father is not
involved.
On physical examination, the neonate has lost 5% of his birth weight. The remainder of his physical
examination findings are normal.
Of the following, the BEST recommendation to provide this neonate’s mother is that she
Correct answer is C
PREP Pearl(s)
Recognizing, exploring, and respecting the sometimes divergent forces in medical decision making is
critical to building a trusting relationship between health care professionals and families.
Practicing complete transparency, demonstrating humility, and, in some cases, consulting an ethics
committee can help achieve more complete health equity in complex cases involving sometimes
divergent forces in medical decision making.
Critique
The mother in the vignette is having difficulty with breastfeeding and is receiving conflicting advice from her
doctor and her family. The best recommendation to provide her at this time is to see a lactation consultant. A
lactation consultant can provide techniques on improving latching, expressing breastmilk, and identifying
feeding cues that may help not only improve breastfeeding but also establish the healthy mother-newborn
bond that is critical to both the mental health of the mother and the social-emotional development of the
newborn. Advising the mother to follow the advice of either her family or her doctor does not allow her to
fully explore her options and denies her agency to make her own decisions. Starting this healthy newborn,
with acceptable weight loss, on formula supplementation before seeing a lactation consultant is not the best
guidance at this time.
When families make decisions regarding the care of their children, they incorporate multiple cultural factors
such as upbringing, living environment, socioeconomic status, family customs, faith background, and peer
influences. These factors affect how families perceive the medical field, understand the causes of disease,
and make treatment decisions. Recognizing, exploring, and respecting these sometimes divergent forces in
medical decision making is critical to building a trusting relationship between health care professionals and
families.
An important aspect of practicing with cultural humility is recognizing cultural diversity: that individuals from
the same culture can have different, and sometimes opposing, viewpoints. Asking open-ended questions and
incorporating family perspectives when defining treatment plans can enhance rapport and improve
adherence. The feeding preference of the mother in the vignette, for example, is different from that of her
family. Encouraging her to explore the available options demonstrates trust in her capacity to make her own
decisions.
Some medical decisions are more easily adaptable to different cultural perspectives than are others.
Circumcision, for example, which is medically recommended or contraindicated in certain situations, is for
most families a decision that is highly personal. Conversely, decisions regarding vaccinations or blood
transfusions may require more complex, culturally sensitive conversations. These conversations become
even more nuanced when an adolescent’s autonomy comes into play. Practicing complete transparency,
demonstrating humility, and, in some cases, consulting an ethics committee can help achieve health equity in
these complex cases.
Suggested Reading(s)
Diekema DS, Leuthner SR, Vizcarrondo FE, eds; Section on Bioethics, American Academy of Pediatrics.
American Academy of Pediatrics Bioethics Resident Curriculum: Case-Based Teaching Guides. Revised
2017. American Academy of Pediatrics: 2017. Accessed September 1, 2023. Bioethics Case Based
Teaching Guides for Resident Training (aap.org)
Hagan JF, Shaw JS, Duncan PM. Promoting family support. In: Hagan JF, Shaw JS, Duncan PM, eds. Bright
Futures: Guidelines for Health Supervision of Infants, Children, and Adolescents. 4th ed. American
Academy of Pediatrics; 2017:41-64.
Olness K, Torjesen K. Providing culturally effective care. In: McInerny TK, Adam HM, Campbell DE,
DeWitt TG, Foy JM, Kamat DM, Baum R, Kelleher KJ, eds. American Academy of Pediatrics Textbook of
Pediatric Care. American Academy of Pediatrics; 2023. Accessed September 1, 2023. Pediatric Care
Online
Content Domain
Communication and Care Coordination
A 7-year-old is admitted to the hospital with a new diagnosis of acute lymphoblastic leukemia. At the time her
chemotherapy is being ordered, her weight is documented in the medical record as 70 kg. Because this
weight seems incorrect, she is reweighed with verification by 2 nurses. Her new weight is 70 lb (31.8 kg), not
70 kg. Review of her medical record reveals that a dose of acetaminophen was administered based on the
incorrect weight; this event was reported through the hospital’s incident reporting system.
Of the following, the events surrounding this girl’s chemotherapy administration are an example of a/an
A. adverse event
B. medical error
C. mistake
D. near miss
Correct answer is D
PREP Pearl(s)
Medication error is the most common type of medical error.
There are several procedural steps during which prevention strategies can be implemented to
decrease the incidence of medication errors: ordering, transcribing, preparing, delivering, and
administering.
Critique
The events surrounding this girl’s chemotherapy administration meet criteria for a near-miss event. The
Institute of Medicine defines medical error as “failure to complete a planned action as intended, or the use of
the wrong plan to achieve an aim.” Medication error, the most common type of medical error, can be an act
of omission (a needed action is not taken) or commission (the wrong action is taken). The Table lists lists the
definitions of several patient safety terms.
There are several procedural steps during which prevention strategies can be implemented to decrease the
incidence of medication errors: ordering, transcribing, preparing, delivering, and administering. Examples of
error prevention practices include the following:
It is also important to ensure appropriate patient education regarding their medications, especially when
starting treatment with a new medication.
In the scenario described in the vignette, the child’s weight was documented incorrectly. This error was
corrected before the administration of any chemotherapy; therefore, regarding her chemotherapy
administration, there was no harm done and the event qualifies as a near miss. In contrast, acetaminophen
was administered at a dose calculated using the incorrect weight, which meets criteria for an adverse event
(defined as harm caused by medical care).
A medical mistake is defined as an error in which the wrong choice is made because of lack of knowledge,
training, information, or poor planning.
It is best practice to disclose medical errors to the family in a timely manner. Institutions and clinician
practices often have specific disclosure protocols defined. Processes used to investigate, analyze, and
address medical errors may include root cause analysis, patient safety/quality assurance conferences (eg,
morbidity and mortality conference), and reporting to a patient safety organization. Root cause analysis looks
at the systems factors involved in an error incident, seeking systems-based prevention strategies to help
prevent a similar outcome in the future. Analysis using a cause-and-effect (Figure.) or fishbone diagram can
help categorize the causes to help identify the relationships that led to the error.
Suggested Reading(s)
Institute of Medicine (US) Committee on Quality of Health Care in America. Kohn LT, Corrigan JM,
Donaldson MS, eds. To Err is Human: Building a Safer Health System. National Academies Press; 2000.
Leonard MS. Patient safety and quality improvement: medical errors and adverse events. Pediatr Rev.
2010;31(4):151-158. doi:10.1542/pir.31-4-151
Neuspiel DR. Medical errors, adverse events, and patient safety. In: McInerny TK, Adam HM, Campbell
DE, DeWitt TG, Foy JM, Kamat DM, eds. American Academy of Pediatrics Textbook of Pediatric Care.
2nd ed. American Academy of Pediatrics; 2017:chap 288. Pediatric Care Online
Content Domain
Communication and Care Coordination
Reprinted with permission from Neuspiel DR. Medical errors, adverse events, and patient
safety. In: McInerny TK, Adam HM, Campbell DE, DeWitt TG, Foy JM, Kamat DM, eds. American
Academy of Pediatrics Textbook of Pediatric Care. 2nd ed. American Academy of Pediatrics;
2017:chap 288.
FIGURE: Cause and effect (fishbone) diagram.
7DEOH'HͤQLWLRQVRI3DWLHQW6DIHW\7HUPV
Term 'HͤQLWLRQ Example
Adverse drug event Adverse event caused by medication Diarrhea caused by amoxicillin
Adverse event An occurrence of injury or harm caused by medical care Laceration caused by wall ophthalmoscope falling on infant
&DXVHDQGHIIHFW ͤVKERQH A method to sort all possible causes of an error into useful After serious adverse event, a team is assembled to conduct root cause analysis
or Ishikawa) diagram categories using a cause-and-effect diagram to assist their discussion
Clinical decision support A system to improve clinical decision-making by providing direct Electronic medical record includes feature that allows the physician to click on
access to evidence-based information “diagnosis” to open a screen that shows recommendations from the latest AAP
clinical practice guideline
Failure modes and effects A quality improvement process used to prospectively identify the To reduce inpatient pediatric prescribing errors, teams analyzed drug delivery
analysis risk of error within a process process to identify potential failures and plan practice changes
Forcing function A limit built into a process that prevents an action with high risk of *DVWURVWRP\WXELQJFDQQRWͤWLQWRLQWUDYHQRXVSRUW
error
Human factors science A discipline that attempts to identify and address safety problems A human factors approach was used to analyze adverse events during pediatric
arising from the interaction between people, technology, and work cardiac surgery and found that communication breakdowns were a major cause
environments
Medical error An act of commission or omission that causes or entails risk of Vaccine given to wrong patient
undesirable patient outcome
Mistake Error resulting from incorrect choice, related to faulty knowledge, Physician prescribes wrong antibiotic for infection
training, or information; a failure of planning
Near miss Error that does not result in AE (ie, “close call”) Physician orders wrong vaccine, but the error is detected by a nurse before it is
administered
Preventable AE Patient harm related to a medical error Caused by lack of follow-up of newborn screening results, an infant with congenital
hypothyroidism remains untreated for months and develops intellectual disability
Redundancy Duplication of key part of process to reduce chance of error Two nurses sign off on all chemotherapy orders before administration
Root cause analysis A process of comprehensive investigation of all underlying After serious medication error, a team of relevant staff is assembled to investigate
causes of an error, emphasizing systems rather than individual possible causes and to recommend solutions to reduce or prevent recurrence of the
performance error
Sentinel event An unexpected occurrence involving actual or potential death or Suicide of patient receiving around-the-clock care; infant abduction or discharge to
serious physical or psychological injury wrong family; hemolytic transfusion reaction involving administration of blood or
blood products with major blood group incompatibilities; surgery on wrong patient or
body part; medication error resulting in major permanent loss of function or death
Serious reportable (never) An event that is unambiguous, serious, and usually preventable Wrong-side surgery; invasive procedure on the wrong patient; death or serious injury
event caused by contaminated drugs or equipment; death caused by electric shock in health
care setting
Six Sigma A quality improvement process that aims to reduce errors to A children’s hospital used a Six Sigma approach to improve the accuracy of weight
approach <3.4 per million events determination for pediatric burn patients to reduce medication and fluid
administration errors
Slip Error resulting from unconscious lapse in automatic task A physician intends to consult a subspecialist about a complex patient, but fails to do
performance caused by distraction; a failure of execution so because of distraction
Trigger tool A measurable signal for detecting likely AEs Using a Global Trigger Tool, investigators at a children’s hospital detected incidence
of harm 2 to 3 times higher than previously published pediatric rates
A 2-year-old boy with a 4-day history of vomiting and diarrhea is admitted to the pediatric intensive care unit.
The boy had poor oral intake and has been unable to keep anything down, with emesis after nearly every
attempt at drinking. He has also had profuse watery diarrhea, low-grade fever, vague abdominal pain, and
decreased urine output. The boy’s vital signs show normotension, tachypnea, and tachycardia. Findings on
physical examination are notable for lethargy, somnolence, dry mucous membranes, doughy skin, and cold
extremities.
Of the following, the physiological parameters that BEST explain this boy’s findings are
A.
Systemic Vascular Resistance Cardiac Preload Cardiac Output
B.
Systemic Vascular Resistance Cardiac Preload Cardiac Output
C.
Systemic Vascular Resistance Cardiac Preload Cardiac Output
D.
Systemic Vascular Resistance Cardiac Preload Cardiac Output
Correct answer is B
PREP Pearl(s)
As a compensatory mechanism for early shock in children, systemic vascular resistance increases to
augment venous return to the right side of the heart.
Hypotension in children with shock is often a late finding and may precipitate organ damage or death.
Physical manifestations of increased systemic vascular resistance (SVR) in children include delayed
capillary refill and cool extremities as blood is shunted from the periphery to the central circulation.
This increased SVR may preserve blood pressure even in low-cardiac-output states.
Critique
The boy in the vignette is experiencing hypovolemic shock as a result of severe dehydration. Thus, his
intravascular volume (cardiac preload) and cardiac output are low. Because the boy has normal blood
pressure, his physiology is consistent with compensated shock (high systemic vascular resistance [SVR]), as
opposed to uncompensated shock (low SVR).
Shock is defined as the lack of adequate oxygen delivery to the end organs. Although the etiology of shock is
multifactorial, the common pathway of all shock includes failure of oxygen delivery. In shock states, the
delivery of oxygen is inadequate to meet the tissue demands. Untreated compensated shock will deteriorate
to uncompensated shock, which can lead to irreversible organ damage and death. Early recognition of shock
is paramount to ensure optimal outcomes in children.
Oxygen delivery is a function of oxygen content (derived from the dissolved oxygen and hemoglobin-bound
oxygen in the blood) and cardiac output. Cardiac output is a function of stroke volume and heart rate. Unlike
in adults, in children cardiac output is highly coupled to preload and afterload. Pediatric hearts are stroke-
volume–limited; thus, any derangement that negatively affects stroke volume will have a substantial impact
on cardiac output. As an early compensatory mechanism for decreases in stroke volume, cardiac preload, or
both, a child’s heart rate increases to preserve cardiac output. As intravascular volume falls and cardiac
preload decreases, SVR increases to augment venous return to the right side of the heart. Blood pressure
early in shock states is preserved (compensated shock) as a result of increased sympathetic nervous system
tone and the increased systemic vascular resistance. Physical manifestations of increased SVR include
normotension/hypertension, delayed capillary refill, and cool extremities as blood is shunted from the
periphery to the central circulation. However, as intravascular volume continues to fall, the compensation
from high SVR is no longer able to further augment preload. At that point, the child becomes acutely
hypotensive as a result of severe low cardiac output (uncompensated shock). Characteristic patterns of SVR,
cardiac preload, and cardiac output are observed in different shock states (Table).
In early septic shock, children become vasodilated as cytokine release causes vascular leak, leading to third
spacing and intravascular volume depletion (warm shock). This low SVR state leads to an initial high cardiac
output state from low afterload. If left untreated, the SVR increases as cardiac output falls, and a state of
vasoconstriction occurs, leading to cold extremities with poor perfusion (cold shock).
In hypovolemic states, as for the boy in the vignette, volume loss leads to an early increase in SVR to preserve
venous return and maintain perfusion to vital organs, even as cardiac output decreases. Thus, children in
compensated hypovolemic shock often have tachycardia with poor perfusion but maintain a normal or
slightly elevated blood pressure.
A high SVR, high cardiac preload, and low cardiac output are suggestive of cardiogenic shock with high
preload due to congestive heart failure.
A low SVR, low cardiac preload, and high cardiac output are suggestive of anaphylaxis where cardiac output is
initially increased as a result of decreased SVR.
A low SVR state would be accompanied by hypotension (not normotension). A low SVR, cardiac preload, and
cardiac output reflect uncompensated shock likely caused by either sepsis or untreated hypovolemic shock.
Suggested Reading(s)
Cornell T, Arutyunyan T, Custer JR. Shock. In: McInerny TK, Adam HM, Campbell DE, Foy JM, Kamat DM,
eds. American Academy of Pediatrics Textbook of Pediatric Care. 2nd ed. American Academy of
Pediatrics; 2023. Accessed September 1, 2023. Pediatric Care Online
McKiernan CA, Lieberman SA. Circulatory shock in children: an overview. Pediatr Rev. 2005;26(12);451-
460. doi:10.1542/pir.26-12-451
Mendelson J. Emergency department management of pediatric shock. Emerg Med Clin North Am.
2018;36(2):427-440. doi:10.1016/j.emc.2017.12.010
Weiss SL, Peters MJ, Alhazzani W, et al. Surviving sepsis campaign international guidelines for the
management of septic shock and sepsis-associated organ dysfunction in children. Intensive Care Med.
2020;46(suppl 1):10-67. doi:10.1007/s00134-019-05878-6
Wheeler DS, Basu RK. Pediatric shock: an overview. Open Pediatr Med J. 2013;7(suppl 1: M2):2-9.
https://openpediatricmedicinejournal.com/contents/volumes/V7/TOPEDJ-7-2/TOPEDJ-7-2.pdf
Content Domain
Critical Care
Abbreviations: BP, blood pressure; HR, heart rate; SVR, systemic vascular resistance.
Adapted from McKiernan CA, Lieberman SA. Circulatory shock in children: an overview. Pediatr Rev. 2005;26(12);453.
AAP PREP 2024 - Question 26/267 Critical Care Question 2/4
A previously healthy 8-month-old girl is brought to the emergency department shortly after falling out of her
high chair. Her mother saw the girl stand in the chair, then fall from approximately 3 feet onto a hardwood
floor, landing head first. The girl appeared dazed and did not cry immediately, but then started to cry
vigorously. Her mother notified the pediatrician, who instructed her to bring the child to the emergency
department.
On physical examination, the girl’s vital signs are normal for age except for a heart rate of 160 beats/min and
a blood pressure of 125/80 mm Hg. There is noticeable swelling over the right side of her head. She becomes
progressively somnolent but wakes when she is examined and to noxious stimulation. Her Glascow Coma
Scale score is 11 (eye opening 3, verbal 3, motor 5). She moves all extremities; however, her left side moves
less than her right. The remainder of the girl’s physical examination findings are normal.
Results of noncontrast computed tomography of the head are shown (Figure 1).
Reprinted with permission from Atabaki SM. Pediatric head injury. Pediatr Rev. 2007;28(6):217.
Figure 1. Computed tomography findings for the girl described in the vignette.
A. concussion
C. epidural hematoma
D. subdural hematoma
Correct answer is C
PREP Pearl(s)
The most common manifestations of intracranial hemorrhages caused by traumatic brain injury in
children are epidural and subdural hematomas.
Epidural hematomas result in a characteristic hyperdense lens-shaped collection of blood visible on
computed tomography; subdural hematomas result in a concave or crescent-shaped hyperdense
lesion.
Children with minor head injury may not require radiographic imaging and can be observed for the
development of symptoms in the emergency department.
Critique
The girl in the vignette sustained blunt head trauma after falling onto a hard surface, resulting in a moderate
traumatic brain injury (TBI). Children with TBI that is moderate (Glascow Coma Scale [GCS] score of 9-13) to
severe (GCS score of 3-8) often have distinct intracranial injuries. The most common manifestations of
intracranial hemorrhage caused by TBI in children are epidural and subdural hematomas. The hyperdense
lens-shaped collection of blood seen on the girl’s computed tomographic (CT) scan is characteristic of an
epidural hematoma (Figure 1).
Reprinted with permission from Atabaki SM. Pediatric head injury. Pediatr Rev. 2007;28(6):217.
Figure 1. Computed tomography of child with traumatic brain injury revealing right sided lens-shaped
hyperdensity consistent with epidural hematoma.
Epidural hematomas result from arterial bleeding that fills the potential space between the dura mater and
the skull; they are often associated with skull fractures. As a result of arterial disruption, epidural
hemorrhages can quickly create a mass effect on the brain, leading to rapid neurologic deterioration.
In contrast, subdural hemorrhage results from disruption of the subarachnoid bridging veins that leads to
accumulation of blood in the virtual space between the dura and arachnoid layer. Subdural hematomas
(Figure 2) have a characteristic concave or crescent-shaped hyperdense appearance on CT. Because of
cerebral compliance, subdural hemorrhage is generally well tolerated in young children and rarely leads to
rapid neurologic deterioration. However, if the hematoma evolves quickly and exceeds the ability to displace
other intracranial components, neurologic deterioration can ensue. The most catastrophic consequence of
any intracranial hemorrhage with cerebral displacement is cerebral herniation.
Reprinted with permission from Atabaki SM. Pediatric head injury. Pediatr Rev. 2007;28(6):217.
Diffuse axonal injury is caused by significant acceleration and deceleration forces. The stretching and tearing
of neurons results in diffuse edema that may not be apparent on initial head imaging. Concussion occurs
when the brain rapidly shifts inside the skull, often from a blow to the head. Concussion symptoms may or
may not include altered mental status, headache, confusion, nausea, dizziness, tinnitus, and alterations in
memory. Although a coexisting concussion is possible for the girl in the vignette, the findings of an epidural
hematoma on CT make concussion unlikely to be the primary cause of her evolving symptoms.
Traumatic brain injury is classified as mild, moderate, or severe, depending on the degree of neurologic
dysfunction as measured by the Glasgow Coma Scale:
Mild—GCS score of 14 to 15
Moderate—GCS score of 9 to 13
Severe—GCS score of 3 to 8
The GCS requires modification for infants to account for child development. The score is the sum of the
points from each of 3 clinical neurological testing domains (Table).
Children with minor head injuries may not require radiographic imaging. Recent clinical guidelines suggest
that children at low to intermediate risk of experiencing intracranial hemorrhage can be observed for 4 to 6
hours in the emergency department and do not require CT imaging if they remain asymptomatic. Guidelines
from the Pediatric Emergency Care Applied Research Network (Schonfeld 2014) recommend judicious
observation of children who are seen in the emergency department within 24 hours of injury and who have
GCS scores of 14 to 15. The Canadian Assessment of Tomography for Childhood Head Injury guideline
contains similar recommendations.
Treatment of more severe head injuries consists of stopping the progression of the primary insult and
preventing or reducing secondary brain injury; this requires early recognition and management. Intracranial
hemorrhage may require surgical intervention to address the primary injury. Neurological protection
measures aim to prevent secondary injury and require attention to normal physiological parameters. In
severe traumatic brain injury, intracranial pressure monitoring with targeted interventions focused on
preservation of normal cerebral perfusion may be required.
Suggested Reading(s)
Gelineau-Morel RN, Zinkus TP, Le Pichon JB. Pediatric head trauma: a review and update. Pediatr Rev.
2019;40(9):468-481. doi:10.1542/pir.2018-0257
Holmes JF, Palchak MJ, MacFarlane T, Kuppermann N. Performance of the pediatric Glasgow Coma
Scale in children with blunt head trauma. Acad Emerg Med. 2005;12(9):814-819.
doi:10.1197/j.aem.2005.04.019
Kochanek PM, Tasker RC, Carney N, et al. Guidelines for the management of pediatric severe traumatic
brain injury: update of the Brain Trauma Foundation guidelines, executive summary. Neurosurgery.
2019;84(6):1169-1178. doi:10.1093/neuros/nyz051
Nigrovic LE, Kuppermann N. Children with minor blunt head trauma presenting to the emergency
department. Pediatrics. 2019;144(6):e20191495. doi:10.1542/peds.2019-1495
Schonfeld D, Bressan S, Da Dalt L, Henien MN, Winnett JA, Nigrovic LE. Pediatric Emergency Care
Applied Research Network head injury clinical prediction rules are reliable in practice. Arch Dis Child.
2014 May;99(5):427-431. doi:10.1136/archdischild-2013-305004
Content Domain
Critical Care
7DEOH*ODVFRZ&RPD6FDOH0RGLͤHGIRU,QIDQWV
Glasgow Coma Scale 1HXURORJLF5HVSRQVH 3RLQWV$ZDUGHG
'RPDLQ
Eye opening Spontaneous eye opening 4
To verbal stimulation 3
To painful stimulation 2
No response 1
Verbal response Coos, babbles 5
Irritable, spontaneous cry 4
Cries to painful stimulation 3
Moans to pain 2
No response 1
Motor response Normal spontaneous movement 6
Withdraws to touch 5
Withdraws to pain 4
Abnormal flexion (decorticate) 3
Abnormal extension (decerebrate) 2
None 1
AAP PREP 2024 - Question 27/267 Critical Care Question 3/4
Four children are admitted to the pediatric intensive care unit with sepsis and acute respiratory failure. They
undergo endotracheal intubation and receive mechanical ventilation with various levels of respiratory
support. Data obtained at 6 hours after intubation are shown for the children:
Child PaO2 (mm Hg) pCO2 (mm Hg) Mean Airway Pressure (mm Hg) SPO2 (%) FiO2 (%)
A 45 50 15 88 50
B 60 50 12 88 50
C 100 50 12 92 50
D 120 50 10 95 50
Of the following, the child at HIGHEST risk of experiencing death as a complication of pediatric acute
respiratory distress syndrome is child
A. A
B. B
C. C
D. D
Correct answer is A
PREP Pearl(s)
Pediatric acute respiratory distress syndrome is a clinical syndrome caused by alveolar disruption and
increased capillary permeability resulting in noncardiogenic pulmonary edema, inflammation, and
hypoxemia.
Pediatric acute respiratory distress syndrome accounts for up to 10% of pediatric intensive care unit
admissions and carries a high mortality risk.
The oxygenation index allows clinicians to determine the progression or resolution of lung disease
from pediatric acute respiratory distress syndrome while taking into account changes in mechanical
ventilator support.
Critique
Pediatric acute respiratory distress syndrome (PARDS) is a clinical syndrome caused by alveolar disruption
and increased capillary permeability resulting in noncardiogenic pulmonary edema and inflammation. It
accounts for up to 10% of all pediatric intensive care unit admissions and is a leading cause of mortality, with
25% of affected children succumbing to their illness.
The severity of PARDS, and thus its mortality risk, is a function of the degree of hypoxemia and oxygenation
failure. The oxygenation index (OI) is a clinical calculation that describes the degree of hypoxemia in relation
to the amount of mechanical ventilator support and FiO2. The oxygenation index is calculated by means of
the following equation:
A higher oxygenation index is associated with a greater severity of lung disease and associated mortality.
Child A in the vignette has the highest calculated OI (16.6) and thus the highest risk of mortality compared
with the other children.
Pediatric acute respiratory distress syndrome is classified as mild (4 ≤ OI < 8), moderate (8 ≤ OI < 16), severe
(OI ≥ 16). Although the primary predictor of mortality is the degree of hypoxemia, other significant predictors
include immunocompromised state, multiorgan system failure, and older age.
When children with PARDS receive mechanical ventilation, oxygenation can be assessed via methods such as
the following:
Only the oxygenation index takes into account the amount of mechanical ventilator support when assessing
the degree of hypoxemia. The OI allows for longitudinal assessment of lung injury, improvement, or both
while taking into account the amount of mechanical ventilator support provided.
Pediatric acute respiratory distress syndrome is not an isolated diagnosis, but rather a clinical syndrome. The
hallmark of PARDS is failure of oxygen diffusion across the alveolar membrane, which results in acute
hypoxemia and noncardiogenic pulmonary edema. Diffuse infiltrates are seen on radiography. Alveolar
epithelial injury may occur as a result of direct injury (eg, acute aspiration) or indirect injury secondary to
systemic inflammation (eg, sepsis). There are consensus-based guidelines addressing the management of
PARDS; the overall management strategy is supportive in nature. Ultimately, the goal is to treat the inciting
cause, provide oxygenation and ventilation support, and avoid secondary ventilator-induced lung injury.
Suggested Reading(s)
Muniraman HK, Song AY, Ramanathan R, et al. Evaluation of oxygen saturation index compared with
oxygenation index in neonates with hypoxemic respiratory failure. JAMA Netw Open.
2019;2(3):e191179. doi:10.1001/jamanetworkopen.2019.1179
Nitu ME, Eigen H. Respiratory failure. Pediatr Rev. 2009; 30(12):470-477. doi:10.1542/pir.30-12-470
Orloff KE, Turner DA, Rehder KJ. The current state of pediatric acute respiratory distress syndrome.
Pediatr Allergy Immunol Pulmonol. 2019;32(2):35-44. doi:10.1089/ped.2019.0999
Trachsel D, McCrindle BW, Nakagawa S, Bohn D. Oxygenation index predicts outcome in children with
acute hypoxemic respiratory failure. Am J Respir Crit Care Med. 2005;172(2):206-211.
doi:10.1164/rccm.200405-625OC
Content Domain
Critical Care
A 17-year-old girl is brought to the emergency department for evaluation of nausea, vomiting, and altered
mental status. The girl was well until yesterday, when she came home from school emotionally distraught
after breaking up with her boyfriend. On physical examination, the girl’s vital signs are a temperature of 36.5
°C, blood pressure of 85/53 mm Hg, a heart rate of 110 beats/min, and a respiratory rate of 14 breaths/min.
She is somnolent but arouses to voice. Her Glascow Coma Score is 13 (eye, 3; verbal, 4; motor, 6). Her pupils
are normal in size and briskly reactive. Her abdomen is diffusely tender without guarding or rebound. The
remainder of the girl’s physical examination findings are normal.
Test Result
B. aspirin
C. glyburide
D. sertraline
Correct answer is A
PREP Pearl(s)
Acute liver failure often presents with nonspecific gastrointestinal symptoms (eg, nausea, malaise),
requiring a high index of suspicion for diagnosis. Unless treated promptly, individuals with fulminant
hepatic failure will progress to develop cerebral edema and death.
Laboratory findings suggestive of acute liver failure include high levels of serum aminotransferases,
hyperbilirubinemia, and evidence of coagulopathy (eg, prolonged prothrombin time).
N-acetylcysteine is a life-saving antidote for acetaminophen toxicity that can prevent the development
of liver failure if administered shortly after the ingestion.
Critique
The girl in the vignette has signs and symptoms consistent with fulminant hepatic failure (nausea, vomiting,
mental status changes, significantly elevated liver transaminase concentrations, and coagulopathy). Of the
response choices, acetaminophen is the substance she most likely ingested. Toxic ingestions of aspirin,
glyburide, or sertraline would not result in hepatic failure.
Acute liver failure often presents with nonspecific gastrointestinal symptoms (eg, nausea, malaise). Left
untreated, the condition will progress. The affected child will develop jaundice, rapid onset of altered mental
status, coma, cerebral edema, and eventual death. Laboratory findings consistent with liver failure include
high serum aminotransferase concentrations (eg, alanine aminotransferase, aspartate aminotransferase),
hyperbilirubinemia, and evidence of coagulopathy (eg, prolonged prothrombin time). Hypoglycemia may also
be present. Hyperammonemia is a late finding; the ammonia level may be significantly elevated and
associated with alterations in mental status. The definitive treatment for fulminant hepatic failure is liver
transplantation.
Acetaminophen (paracetamol) is available as an over-the-counter medication commonly used to treat fever
and pain. Although generally safe when taken according to prescribing guidelines, excessive doses of
acetaminophen can be lethal. The parent drug is metabolized by the liver via 3 main hepatic pathways. Under
normal circumstances, the toxic metabolite N-acetyl-p-benzoquinone imine (NAPQI) reacts with glutathione
and is excreted in the urine. When acetaminophen is ingested in toxic doses, NAPQI production overwhelms
the glutathione capacity; the excess NAPQI accumulates in and destroys the hepatocytes, resulting in hepatic
necrosis and liver failure. N-acetylcysteine (NAC) is a potentially life-saving antidote for acetaminophen
toxicity when administered shortly after the ingestion; if given within 8 hours, the individual has a high
likelihood of full recovery. An antioxidant that reduces the accumulation of toxic metabolites and increases
production of nontoxic metabolites, NAC prevents hepatic injury primarily by restoring hepatic glutathione,
thereby increasing excretion and limiting the hepatic accumulation of NAPQI.
Acetaminophen is the third most commonly intentionally ingested drug among adolescents. In 2016, it was
the medication most commonly used for all intentional self-harm incidents. Acetaminophen is readily
available and relatively inexpensive, making it easily accessible to most children seeking to harm themselves
through drug overdose. Although outcomes of intentional acetaminophen overdose tend to be better in
children than in adults, up to 17% of affected children develop acute hepatic failure. When acute fulminant
hepatic failure is refractory to therapy, liver transplantation is required.
Suggested Reading(s)
Lee WM. Acetaminophen toxicity: a history of serendipity and unintended consequences. Clin Liver Dis
(Hoboken). 2020;16(suppl 1):34-44. doi:10.1002/cld.984
Nadler A, Fein DM. Acetaminophen poisoning. Pediatr Rev. 2018;39(6):316-318. doi:10.1542/pir.2017-
0093
Polson J, Lee WM; American Association for the Study of Liver Disease. AASLD position paper: the
management of acute liver failure. Hepatology. 2005;41(5):1179-1197. doi:10.1002/hep.20703
Shadman KA, Edmonson MB, Coller RJ, et al. US hospital stays in children and adolescents with
acetaminophen poisoning. Hosp Pediatr. 2022;12(2):e60-e67. doi:10.1542/hpeds.2021-005816
Content Domain
Critical Care
A 2-year-old girl is being evaluated for a rash on her leg. Her father first noticed 2 spots a few weeks ago; now
she has several bumps on her left leg (Figure). They do not seem painful. She is otherwise healthy.
Courtesy of Mancini AJ, Krowchuk DP. Pediatric Dermatology: A Quick Reference Guide. 3rd ed. Elk Grove
Village, IL: American Academy of Pediatrics; 2016, p104.
A. cutaneous histoplasmosis
B. molluscum contagiosum
C. varicella-zoster virus
D. verruca vulgaris
Correct answer is B
PREP Pearl(s)
Molluscum contagiosum is a self-limited disease.
To prevent the spread in immunocompromised patients or those with atopic dermatitis, early
treatment of molluscum contagiosum should be considered.
Molluscum contagiosum lesions can become secondarily infected.
Critique
The small, pearly, dome-shaped lesions seen on the girl’s leg are most consistent with molluscum
contagiosum. This painless lesion is commonly seen in otherwise healthy children and can spread, especially
if the child has a compromised immune system or underlying atopic dermatitis. The lesions typically have a
central punctum.
The rash is caused by a poxvirus resulting in the characteristic superficial papules. It is self-limited, but often
takes 6 to 12 months to resolve, with some cases lasting several years. The lesions can become secondarily
infected by skin bacteria, becoming red, swollen, and painful; management of infected lesions may require
topical or systemic antibiotics, depending on the severity.
Inside the molluscum papules is a core of white material. If the lesions are scratched, children can spread the
rash (autoinoculation). If desired, the lesions can be treated with cryotherapy, cantharidin (“beetle juice”), or
curettage. This is usually recommended for children with underlying atopic dermatitis, lesions in the genital
area, or compromised immune systems. Treatment may also be considered for lesions that are present for
several months.
Cutaneous histoplasmosis appears as papules, pustules, and nodules in the setting of systemic
histoplasmosis infection. Varicella (Figure 2) is more vesicular in appearance; the lesions are at various stages,
pruritic, and accompanied by systemic symptoms. Verruca vulgaris (Figure 3) is known as the common wart; it
does not usually present in clusters on the extremity as is shown in the vignette.
Reprinted with permission from Kimberlin DW, Barnett ED, Lynfield R, Sawyer MH, eds.
Red Book: 2021–2024 Report of the Committee on Infectious Diseases. 32nd ed. American Academy of
Pediatrics; 2021.
Suggested Reading(s)
American Academy of Dermatology. Molluscum contagiosum: diagnosis and treatment.
https://www.aad.org/diseases/a-z/molluscum-contagiosum-treatment
Centers for Disease Control and Prevention. Molluscum contagiosum.
https://www.cdc.gov/poxvirus/molluscum-contagiosum/clinical_information.html.
Murray DL, Vodzak J, Immergluck LC. Contagious exanthematous diseases. In: McInerny TK, Adam HM,
Campbell DE, DeWitt TG, Foy JM, Kamat DM, eds. American Academy of Pediatrics Textbook of Pediatric
Care. American Academy of Pediatrics; Accessed September 1, 2023 Pediatric Care Online
Content Domain
Dermatology
A 4-month-old infant is undergoing a routine health supervision visit. She has a skin lesion over her left upper
eyelid that was first noted at age 1 month. The lesion is now raised and has substantially increased. The
infant has normal growth and development for age. She has a 3×3-cm erythematous, raised, well-
demarcated lesion involving the left upper eyelid with mild left ptosis (Figure). The remainder of the physical
examination findings are normal.
Reprinted with permission from Darrow HD, Greene AK, Mancini AJ, Nopper AJ, Section on Dermatology,
Section on Otolaryngology-Head and Neck Surgery, Section on Plastic Surgery. Pediatrics.
2015;136(4):e1085.
Figure. Left upper eyelid findings for the infant described in the vignette.
Of the following, the STRONGEST indication to initiate treatment for this infant’s physical examination finding
is to prevent
A. amblyopia
B. cosmetic disfigurement
D. lymphatic obstruction
Correct answer is A
PREP Pearl(s)
Infantile hemangiomas follow a characteristic course of initial rapid growth, followed by a growth
plateau and, finally, involution.
Periocular hemangiomas are associated with a high risk of deprivation amblyopia owing to visual field
obstruction.
Early ophthalmology consultation and treatment of periocular hemangiomas is important to prevent
permanent vision loss.
Critique
The infant in the vignette has a superficial infantile hemangioma (IH) involving her left upper eyelid
(periocular), as well as left ptosis. Of the response choices, the strongest indication to treat this infant’s
hemangioma is to prevent the development of amblyopia, which can result from obstruction of the field of
vision.
Amblyopia is defined as a functional visual deficit not directly caused by a structural abnormality of the eye.
Amblyopia can occur secondary to the following:
Strabismus (esotropia/exotropia)
Refractive errors (myopia/hyperopia)
Visual deprivation (impaired visual input caused by eyelid tumors or corneal opacities)
Amblyopia is the most common complication of untreated periocular IH, occurring in about 50% of affected
children. Successful treatment of amblyopia, and prevention of irreversible vision loss, depends on early
detection during the critical period of vision development. Rare complications of periocular IH include
retrobulbar extension, astigmatism, and refractive errors. Early ophthalmological evaluation and treatment is
critical for children with periocular IH to prevent permanent blindness.
High-output heart failure can occur in the presence of multiple cutaneous or large visceral hemangiomas,
which are not present in the infant in the vignette. Lymphatic obstruction occurs rarely with deep or mixed
lesions containing lymphatic malformations, but not with superficial IH. Although IH causes temporary (and
occasionally minor permanent) disfigurement, prevention of amblyopia is a more important reason to treat
the infant in the vignette.
Infantile hemangiomas are the most common vascular tumors in infancy, with an incidence of about 5% to
10%. There is an increased prevalence in premature and female infants. Infantile hemangiomas are classified
according to the soft-tissue depth (superficial, deep, or combined) and extent of the lesion (localized,
segmental, or multifocal).
Infantile hemangiomas have a characteristic growth pattern; there is a rapid growth phase during the first 6
months after birth, followed by a plateau for the next 6 months, and finally involution over the next few
years. At birth, these lesions appear as flat bluish macules with telangiectasia and typically transform into the
characteristic bright red lesions by age 1 month. Congenital hemangiomas, in contrast, are fully developed at
birth and do not exhibit the growth phases characteristic of IH.
Infantile hemangiomas are diagnosed clinically, according to their characteristic timeline and physical
appearance. Imaging is indicated if deeper lesions are suspected (eg, airway or intra-abdominal) and in cases
of large segmental hemangiomas associated with syndromes:
PHACES (posterior fossa defects, hemangiomas, arterial anomalies, cardiovascular anomalies, eye
anomalies, sternal raphe)
LUMBAR (lower body hemangiomas, urogenital abnormalities, myelopathy, bony deformities,
arterial/anorectal anomalies, renal anomalies).
The pathogenesis of IH is unclear. It has been postulated that endothelial progenitor cells are upregulated by
angiogenic growth factors and that hypoxia-induced vascular proliferation occurs.
Most infantile hemangiomas involute and completely regress. Treatment is indicated only for high-risk lesions
with the potential for substantial complications. High-risk lesions and their complications are outlined in the
Table.
Oral propranolol is the first-line therapy for high-risk IH, owing to its safety profile and efficacy. Heart rate,
blood pressure, and glucose should be monitored when initiating propranolol, especially in young infants.
Other treatment options for IH include corticosteroids (topical, intralesional, and systemic),
immunomodulators, and laser therapy.
Suggested Reading(s)
Arneja JS, Benson A, Gilardino MS. Hemangiomas. In: McInerny TK, Adam HM, Campbell DE, Foy JM,
Kamat DM, eds. American Academy of Pediatrics Textbook of Pediatric Care. 2nd ed. American
Academy of Pediatrics; 2016:chap 261. Accessed September 1, 2023. Pediatric Care Online
Darrow DH, Greene AK, Mancini AJ, Nopper AJ; Section on Dermatology, Section on Otolaryngology–
Head and Neck Surgery, and Section on Plastic Surgery; et al. Diagnosis and management of infantile
hemangioma. Pediatrics. 2015;136(4):e1060-e1104. doi:10.1542/peds.2015-2485
Jung HL. Update on infantile hemangioma. Clin Exp Pediatr. 2021;64(11):559-572.
doi:10.3345/cep.2020.02061
Loh AR, Chiang MF. Pediatric vision screening. Pediatr Rev. 2018;3 (5):225-234. doi:10.1542/pir.2016-
0191
Nelson LB, Johnson BB, O’Hara M. Amblyopia and strabismus. In: McInerny TK, Adam HM, Campbell DE,
Foy JM, Kamat DM, eds. American Academy of Pediatrics Textbook of Pediatric Care. 2nd ed. American
Academy of Pediatrics; 2016:214. Accessed September 1, 2023. Pediatric Care Online
Content Domain
Dermatology
A 15-year-old adolescent is being evaluated for acne. They tried an over-the-counter salicylic acid wash
without benefit. They has been in good health and take no medications. There are approximately 8 eight
active inflammatory lesions on each cheek and their chin. There are several open comedones (Figure 1), but
no scarring. The forehead, chest, and back are free of acne lesions.
Reprinted with permission from Mancini AJ, Krowchuk DP eds. Pediatric Dermatology: A Quick Reference
Guide. 3rd ed. Elk Grove Village, IL: American Academy of Pediatrics; 2016:68.
Correct answer is C
PREP Pearl(s)
A topical retinoid should be included in the treatment of adolescents who have moderate or severe
acne.
Obstruction within follicles should be addressed, even if blackheads and whiteheads are not observed.
Extensive inflammatory acne (ie, involving the trunk, as well as the face) requires treatment with an
oral antibiotic.
Critique
This patient has moderate inflammatory and comedonal acne involving the face (Figure 2). There are several
inflammatory papules (red arrow) and open comedones (yellow arrow). No scarring is evident, but there are
several resolving inflammatory lesions (blue arrow). These lesions may remain erythematous or violaceous
for months and are often confused with scars. The most appropriate treatment, therefore, is benzoyl
peroxide (BPO)/clindamycin applied to the face each morning and a retinoid (eg, tretinoin or adapalene)
applied at bedtime. Therapy with BPO, BPO/clindamycin, or doxycycline orally combined with BPO topically
would address the inflammatory component of the disease, but not follicular obstruction. In addition,
because the adolescent has moderate acne that is limited to the face with no scarring, and is using no
medication, an attempt to manage the inflammatory component of the disease with topical agents is
reasonable.
Adapted and reprinted with permission from Mancini AJ, Krowchuk DP eds. Pediatric Dermatology: A Quick
Reference Guide. 3rd ed. Elk Grove Village, IL: American Academy of Pediatrics; 2016:68.
Figure 2. Moderate inflammatory and comedonal acne on the face. There are inflammatory papules and
pustules (red arrow), open comedones (yellow arrow), and resolving inflammatory lesions (blue arrow).
Several factors contribute to the development of acne. Key among these are disordered keratinization
(leading to obstruction within pilosebaceous follicles), increased sebum production (which contributes to
obstruction), and inflammation (caused, in large part, by activation of the immune system by the bacterium
Cutibacterium acnes). As obstruction increases, follicles may rupture, contributing to the inflammatory
process. In some patients, the inflammatory process results in scarring. On the face, scars appear as small
pits, whereas on the trunk they are hypopigmented macules.
Treatment plans should be designed to affect as many aspects of disease pathophysiology as possible. The
inflammatory process can be addressed with BPO, antibiotics (topical or oral), and dapsone. Topical retinoids
improve follicular obstruction; oral contraceptives, in girls, lessen the impact of androgens (reducing sebum
production and follicular obstruction). It is important to note that particularly in moderate or severe acne,
follicular obstruction is present and should be addressed therapeutically, even if blackheads (open
comedones) and whiteheads (closed comedones) are not observed.
Guidelines for acne management exist, but treatment should be individualized according to the patient’s
perception of disease severity, past experiences with medications, cost (insurance coverage may vary), and
the ability to adhere to therapy. Suggested treatment plans for mild, moderate, and severe acne are
presented in Table 1, Table 2, and Table 3.
Suggested Reading(s)
Chen C, Williams JV. Acne. In: McInerny TK, Adam HM, Campbell DE, DeWitt TG, Foy JM, Kamat DM, eds.
American Academy of Pediatrics Textbook of Pediatric Care. American Academy of Pediatrics; 2023.
Accessed September 1, 2023. Pediatric Care Online
Eichenfield LF, Krakowski AC, Piggott C, et al; American Acne and Rosacea Society. Evidence-based
recommendations for the diagnosis and treatment of pediatric acne. Pediatrics. 2013;131(suppl
3):S163-S186. doi:10.1542/peds.2013-0490B
Mancini AJ, Krowchuk DP, eds. Pediatric Dermatology: A Quick Reference Guide. 3rd ed. Elk Grove
Village, IL: American Academy of Pediatrics; 2016.
Thiboutot DM, Dréno B, Abanmi A, et al. Practical management of acne for clinicians: an international
consensus from the Global Alliance to Improve Outcomes in Acne. J Am Acad Dermatol. 2018;78(2
suppl 1):S1-S23.e1. doi:10.1016/j.jaad.2017.09.078
Content Domain
Dermatology
The correct answer is: benzoyl peroxide/clindamycin and a retinoid administered topically
View Peer Results
5HSULQWHGZLWKSHUPLVVLRQIURP0DQFLQL$-.URZFKXN'3HGVPediatric Dermatology:
A Quick Reference GuideUGHG(ON*URYH9LOODJH,/$PHULFDQ$FDGHP\RI3HGLDWULFV
5HSULQWHGZLWKSHUPLVVLRQIURP0DQFLQL$-.URZFKXN'3HGVPediatric Dermatology:
A Quick Reference GuideUGHG(ON*URYH9LOODJH,/$PHULFDQ$FDGHP\RI3HGLDWULFV
5HSULQWHGZLWKSHUPLVVLRQIURP0DQFLQL$-.URZFKXN'3HGVPediatric Dermatology:
A Quick Reference GuideUGHG(ON*URYH9LOODJH,/$PHULFDQ$FDGHP\RI3HGLDWULFV
AAP PREP 2024 - Question 32/267 Dermatology Question 4/6
A 15-year-old adolescent boy developed a spot on his chest 3 weeks ago. His mother thought that he might
have ringworm and applied an over-the-counter antifungal cream. However, the lesion persisted, and within
1 to 2 weeks, many new lesions appeared on the trunk. The rash is pruritic but the patient has been
otherwise well. He has normal vital signs. Physical examination findings are unremarkable aside from the
skin. There is a widespread eruption that involves the chest, abdomen, and back. The face, neck, and
extremities are spared. The original lesion is present on the right side of the upper chest (Figure 1).
Courtesy of D Krowchuk
Correct answer is B
PREP Pearl(s)
The lesions of pityriasis rosea are oval, scaling, thin plaques. The long axes of plaques are oriented
parallel to lines of skin stress. The scale is located on the trailing edge of lesions rather than at the
leading edge (border).
On the back, the arrangement of pityriasis rosea lesions along lines of skin stress (and the ribs) may
mimic the appearance of the boughs of a fir tree (ie, the “Christmas-tree” appearance).
In persons of color, pityriasis rosea may have an “inverse” distribution (with lesions concentrated on
the extremities and relative sparing of the trunk); alternatively, lesions may be papules with fewer
plaques.
Critique
The boy in the vignette developed a rash that began as a round patch mimicking tinea corporis. On close
inspection, however, the lesion lacks central clearing and border elevation. Within 2 weeks he developed
numerous lesions on the trunk, many of which are papules. These findings are consistent with a diagnosis of
pityriasis rosea and, accordingly, symptomatic treatment with hydroxyzine is appropriate. If tinea corporis
was suspected, topical terbinafine (for one or a few lesions) or oral griseofulvin (for large or multiple lesions)
could be prescribed. Tinea versicolor is characterized by scaling macules and patches (not papules) located
on the trunk (Figure 2); it may be treated with topical selenium sulfide.
Reprinted with permission from Mancini AJ, Krowchuk DP, eds. Pediatric Dermatology. A Quick Reference
Guide. 3rd ed. American Academy of Pediatrics; 2016:278.
Pityriasis rosea (PR) is a self-limited papulosquamous disorder (ie, lesions are elevated and have scale). It may
be infectious (human herpesvirus types 6 and 7 have been implicated) and usually occurs in the spring and
fall. In as many as 80% of patients, the eruption begins with a herald patch. This lesion is round or oval, often
has central clearing (although in this patient it did not), and exhibits scaling (Figure 3). Within 2 weeks, a more
widespread eruption develops that is composed of erythematous papules and plaques. The long axes of
plaques are oriented parallel to lines of skin stress (Figure 4). On the back, this alignment of lesions may
mimic the boughs of a fir tree (ie, the “Christmas-tree” appearance) (Figure 5).
Courtesy of D Krowchuk
Figure 3. The herald patch of pityriasis rosea is erythematous, round or oval, and has scale (concentrated
at the trailing, not leading edge, of the lesion).
Reprinted with permission from Mancini AJ, Krowchuk DP, eds. Pediatric Dermatology. A Quick Reference
Guide. 3rd ed. American Academy of Pediatrics; 2016:332
Figure 5. On the back, the alignment of pityriasis rosea lesions along lines of skin stress may mimic the
appearance of the boughs of a fir tree (ie,, the “Christmas-tree” appearance).
When examining individual lesions of PR, scale is observed at the trailing edge (ie, more centrally) unlike in
tinea corporis, in which it is located at the leading edge (ie, peripherally) (Figure 4). In persons of color, the
eruption may differ in two ways: it may have an “inverse distribution” with lesions concentrated on the
extremities and relative sparing of the trunk, or it may be primarily papular with fewer plaques, as was the
case for the boy in the vignette (see also Figure 6). New lesions appear for 2 to 3 weeks, and the eruption
resolves in 4 to 8 weeks.
Courtesy of D Krowchuk
Figure 4. The classical lesions of pityriasis rosea are oval thin scaling plaques. The long axes of plaques are
oriented parallel to lines of skin stress. As with the herald patch, scale is located at the trailing edge of
lesions.
Reprinted with permission from Mancini AJ, Krowchuk DP, eds. Pediatric Dermatology. A Quick Reference
Guide. 3rd ed. American Academy of Pediatrics; 2016:332.
Figure 6. A child who has pityriasis rosea. There are numerous small papules.
The diagnosis of PR is made clinically on the basis of the typical appearance and distribution of the rash. In
patients who have a predominantly papular presentation, clinicians should examine the skin carefully for oval
plaques, because finding these lesions helps secure the diagnosis.
Most individuals who have PR require no therapy. If pruritus is significant, a topical corticosteroid or an
emollient containing phenol or menthol (agents that act as counterirritants, masking the perception of
pruritus) may be applied as needed, or a sedating antihistamine (eg, hydroxyzine) may be taken at bedtime.
Judicious sun exposure may reduce pruritus and hasten the resolution of the eruption.
Suggested Reading(s)
Chang HC, Sung CW, Lin MH. The efficacy of oral acyclovir during early course of pityriasis rosea: a
systematic review and meta-analysis. J Dermatolog Treat. 2019;30(3):288-293.
doi:10.1080/09546634.2018.1508820.
Mancini AJ, Krowchuk DP, eds. Pediatric Dermatology. A Quick Reference Guide. 3rd ed. Elk Grove
Village, IL: American Academy of Pediatrics; 2016.
Content Domain
Dermatology
An 8-year-old girl is being evaluated for a rash on her hand that developed 2 months ago. The lesion began
as a “bump” and has become larger, forming a ring with a border that is firm to palpation (Figure 1). Her
parents applied an over-the-counter antifungal cream without benefit.
Courtesy of D. Krowchuk
A. clotrimazole topically
B. griseofulvin orally
C. hydrocortisone 1% topically
D. no intervention
Correct answer is D
PREP Pearl(s)
Granuloma annulare results in rings that may mimic the lesions of tinea corporis.
Granuloma annulare may be differentiated from tinea corporis by the absence of scale , the firmness
of the border, and lesions are often skin colored.
Critique
The girl in this vignette has an annular lesion with an elevated border and central clearing. The lesion has
been treated unsuccessfully with a topical antifungal agent. Although the physical findings are reminiscent of
tinea corporis (Figure 2), the absence of scale supports a diagnosis of granuloma annulare (GA). As a result,
no intervention is required. Clotrimazole is effective in the management of tinea corporis, and griseofulvin
would be indicated in resistant infections or infections characterized by large or numerous lesions. An
emollient and topical corticosteroid would be useful in managing nummular eczema.
Courtesy of D. Krowchuk
Figure 2. Tinea corporis. This lesion on the nose exhibits an erythematous, scaling border (arrow) and
some central clearing. (On the face, tinea corporis is sometimes called tinea faciei.)
Granuloma annulare is a relatively common disorder whose cause is unknown. It usually occurs in school-
aged children and affects girls more often than boys. Although an association with diabetes mellitus has been
suggested in adults, this is not the case in children.
Granuloma annulare is characterized by papules that form a ring; occasionally, the ring is incomplete (Figure
3). Lesions may occur at any body site but commonly involve areas that are prone to trauma like the dorsa of
the hands or feet. The lesions of GA may be skin-colored (Item Figure 3) but often are erythematous (Figure 1)
or violaceous (Figure 4). Most individuals have a single lesion but multiple lesions may occur (Figure 4).
Because the pathology of GA occurs in the dermis, lesions have a “deeper” or “firmer” feel than the border of
a tinea corporis lesion. A subcutaneous form of GA is characterized by multiple subcutaneous nodules with
normal overlying skin. The diagnosis of GA is made clinically and may be confirmed with skin biopsy, although
rarely required.
Courtesy of D. Krowchuk
Figure 3. At times, the ring of granuloma annulare may be incomplete. In this patient, the ring is slightly
erythematous.
Courtesy of D. Krowchuk
Figure 4. Lesions of granuloma annulare often are violaceous in color. Some individuals develop multiple
lesions.
Granuloma annulare often is confused with tinea corporis as discussed previously. It also may mimic
nummular eczema (Figure 5). However, nummular eczema does not exhibit border elevation or central
clearing. In addition, a crust is present in nummular eczema. The lesions of tinea corporis and nummular
eczema are often pruritic. Subcutaneous GA may mimic a soft-tissue malignancy, rheumatoid nodules, or
sarcoidosis.
Reprinted with permission from American Academy of Pediatrics Section on Dermatology. Atopic
dermatitis. In: Mancini AJ, Krowchuk DP, eds. Pediatric Dermatology: A Quick Reference Guide. 3rd ed.
American Academy of Pediatrics; 2016:34.
Figure 5. Nummular eczema is characterized by oval or round lesions that form crust (arrows).
Suggested Reading(s)
American Academy of Pediatrics Section on Dermatology. Atopic dermatitis.In: Mancini AJ, Krowchuk
DP, eds. Pediatric Dermatology: A Quick Reference Guide. 4th ed. Itasca, IL: American Academy of
Pediatrics; 2020:33-46
Piette EW, Rosenbach M. Granuloma annulare. Clinical and histologic variants, epidemiology, and
genetics. J Am Acad Dermatol. 2016;75(3):457-465. doi:10.1016/j.jaad.2015.03.055.
Content Domain
Dermatology
A previously healthy 10-year-old boy is being evaluated in the emergency department for a rash of 1 week’s
duration. Two weeks ago, he saw his pediatrician because of a sore throat and fatigue and was diagnosed
with an Epstein-Barr virus infection. Those symptoms resolved, but he subsequently developed oral and skin
lesions.
The boy’s temperature is 37.5 °C, his blood pressure is 110/80 mm Hg, his heart rate is 120 beats/min, his
respiratory rate is 24 breaths/min, and his oxygen saturation is 99% in room air. He has lost 5% of his body
weight since his last visit 2 weeks ago. He appears moderately uncomfortable owing to pain. In addition to
oral lesions (Figure 1), there are target lesions (Figure 2) and skin erosions (Figure 3) that cover about 9% of
his body surface area. The affected skin sloughs easily. The boy’s skin turgor is decreased, and his mucous
membranes are dry. His examination findings are otherwise unremarkable.
Reprinted with permission from Henderson L, Williams JV. In:McInerny TK, Adam HM, Campbell DE, DeWitt
TG, Foy JM, Kamat DM, eds. American Academy of Pediatrics Textbook of Pediatric Care. American
Academy of Pediatrics; 2016:chap 245.p. 1990
Reprinted with permission from Mancini AJ, Krowchuk DP, eds. Pediatric Dermatology: A Quick Reference
Guide. American Academy of Pediatrics;2021:chap 110
A. Kawasaki disease
C. Stevens-Johnson syndrome
PREP Pearl(s)
Stevens-Johnson syndrome affects <10% of the body surface area, results in skin lesions that are easily
sloughed, and involves more than 2 types of mucous membranes.
Stevens-Johnson syndrome may be caused by medication (eg, sulfonamides) or infection (eg, Epstein-
Barr virus).
Ophthalmology consultation is required for any eye involvement in Stevens-Johnson syndrome.
Critique
The examination findings for the boy in the vignette include ulceration of his mucosal membranes and easy
sloughing of skin lesions. Less than 10% of his body surface area is affected. This constellation of findings is
most consistent with Stevens-Johnson syndrome (SJS). The findings of Kawasaki disease do not usually
include ulceration of the mucosal membranes. Staphylococcal scalded skin syndrome does not usually have
mucosal involvement. The skin lesions of toxic epidermal necrolysis are more extensive than in SJS, typically
affecting more than 30% of the body surface area. Children with 10% to 30% of their body surface area
affected are classified as having SJS/toxic epidermal necrolysis overlap.
Stevens-Johnson syndrome describes a multifactorial process of focal epidermal necrolysis that may be
caused by medication (eg, sulfonamides) or infection (eg, Epstein-Barr virus). Cases of SJS caused by infection
can occur up to 2 weeks later. The skin may initially appear dusky, followed by the appearance of blisters that
slough easily. Usually, more than 2 mucosal membranes are involved (eg, eyes, respiratory tract, esophagus,
anus, genitalia, urethra). Conjunctivitis, if present, is usually purulent. The signs and symptoms of SJS typically
progress over 1 week, followed by re-epithelialization over the next several weeks.
Management of SJS usually requires hospitalization for pain control, topical wound care, and hydration. If SJS
is caused by a medication, immediate discontinuation of the drug is mandatory. Consultation with an
ophthalmologist is important for children with eye involvement. Depending on the clinical scenario,
treatment with steroids, intravenous immunoglobulin, or immunosuppressants may be recommended; this
should occur in consultation with the appropriate subspecialists.
Suggested Reading(s)
American Academy of Pediatrics. Stevens-Johnson syndrome (SJS) and Mycoplasma pneumoniae–
induced rash and mucositis (MIRM). In: Mancini AJ, Krowchuk DP, eds. Pediatric Dermatology: A Quick
Reference Guide. 4th ed. American Academy of Pediatrics;2021:chap 110. Accessed March 7, 2023.
Pediatric Dermatology Online
Henderson L, Williams JV. Drug eruptions, erythema multiforme, Stevens-Johnson syndrome.
In:McInerny TK, Adam HM, Campbell DE, DeWitt TG, Foy JM, Kamat DM, eds. American Academy of
Pediatrics Textbook of Pediatric Care. American Academy of Pediatrics; 2016:chap 245. Accessed March
7, 2023. Pediatric Care Online
Schneider JA, Cohen PR. Stevens-Johnson syndrome and toxic epidermal necrolysis: a concise review
with a comprehensive summary of therapeutic interventions emphasizing supportive measures. Adv
Ther. 2017;34(6):1235-1244. doi:10.1007/s12325-017-0530-y
Content Domain
Dermatology
A 17-year-old is evaluated in the emergency department for a suspected overdose. He was found confused
and vomiting in his bedroom. His parents found a suicide note and several empty bottles of medication in
the child’s bathroom, including melatonin, ibuprofen, aspirin, and vitamin C. There were no other
medications in the home. In the emergency department, the boy is vomiting. His vital signs are as follows:
temperature, 37.9 °C; heart rate, 135 beats/min; respiratory rate, 28 breaths/min; blood pressure, 118/75
mm Hg; and oxygen saturation, 97% in room air.
The remainder of his physical examination findings are unremarkable. An arterial blood gas sample reveals
the following: pH, 7.58; PaO2, 105 mm Hg; PaCO2, 30 mm Hg; and bicarbonate, 22 mEq/L (22 mmol/L).
A. initiation of hemodialysis
B. intravenous N-acetylcysteine
D. oral N-acetylcysteine
Correct answer is C
PREP Pearl(s)
Salicylate toxicity results in a mixed respiratory alkalosis and metabolic acidosis, which eventually
progresses to severe metabolic acidosis if not treated.
The initial treatment of salicylate overdose includes administration of intravenous sodium bicarbonate
to induce alkalemia and achieve a urine pH of 7.5 to 8.0.
Hemodialysis may be used to treat salicylate toxicity; however, hemodialysis is typically reserved for
severe cases or cases refractory to initial treatment.
Critique
For the adolescent in the vignette, the signs and symptoms are most consistent with salicylate toxicity, which
requires urgent intervention. The best next step in this boy’s management is administration of intravenous
sodium bicarbonate. For the other medications he might have ingested (melatonin, ibuprofen, or vitamin C),
which can cause toxicity if ingested in extremely large quantities, there are no known antidotes or treatments
other than supportive care.
Salicylates directly affect the respiratory drive, resulting in tachypnea, which results in a mixed respiratory
alkalosis and metabolic acidosis. If untreated, the condition will progress to severe metabolic acidosis. Initial
treatment is with intravenous sodium bicarbonate, which results in alkalemia that increases the ionized
proportions of salicylate in the blood, thereby reducing its ability to cross the blood-brain barrier. In addition,
alkalinization of the urine inhibits salicylate reabsorption and enhances its renal elimination. Intravenous
sodium bicarbonate should be administered to individuals showing signs and symptoms of salicylate toxicity
to achieve a urine pH of 7.5 to 8.0. Hemodialysis can be used to treat salicylate toxicity; however,
hemodialysis is typically reserved for cases associated with renal failure, persistent central nervous system
alterations or seizures, pulmonary edema, severe or refractory acid-base or electrolyte disturbances, or
coagulopathy. N-acetylcysteine, administered orally or intravenously, is the treatment for acetaminophen
toxicity.
Toxicity resulting from melatonin overdose is exceedingly rare, even when ingested in large quantities.
Symptoms of melatonin overdose are usually mild and may include drowsiness or central nervous system
depression, vomiting, or headache. Because vitamin C is water soluble, it is not stored in body tissues and is
readily excreted in urine. Toxicity from vitamin C overdose is rare; symptoms may include diarrhea and
gastrointestinal bloating. Ibuprofen is a nonsteroidal anti-inflammatory drug (NSAID) that is readily available
over the counter and is present in most homes. Mild to moderate overdose (<400 mg/kg) may result in injury
to gastrointestinal mucosa, including gastritis and gastric bleeding, affect renal blood flow, and inhibit normal
platelet function and its response to vascular injury. Severe NSAID overdose (>400 mg/kg) can result in
hypotension, dysrhythmias, hypothermia, seizures or other neurologic dysfunction, and metabolic acidosis.
There is no antidote for ibuprofen toxicity; treatment is supportive.
Suggested Reading(s)
Fine JS. 2016. Poisoning. In: McInerny TK, Adam HM, Campbell DE, DeWitt TG, Foy JM, Kamat DM, eds.
American Academy of Pediatrics Textbook of Pediatric Care. American Academy of Pediatrics;
2023:chap 369. Accessed September 1, 2023. Pediatric Care Online
Hendrickson RG, McKeown NJ. Acetaminophen. In: Nelson LS, Howland M, Lewin NA, Smith SW,
Goldfrank LR, Hoffman RS. eds. Goldfrank's Toxicologic Emergencies. 11 ed. McGraw Hill; 2019.
Lugassy DM. Salicylates. In: Nelson LS, Howland M, Lewin NA, Smith SW, Goldfrank LR, Hoffman RS,
eds. Goldfrank's Toxicologic Emergencies. 11 ed. McGraw Hill; 2019.
Rakovchik EE, Fein DM. Nonsteroidal anti-inflammatory drug and salicylate poisoning. Pediatr Rev.
2016;37(1):48-50. doi:10.1542/pir.2015-0052
Renny MH, O’Donnell KA, Calello DP. Toxicologic emergencies. In: Shaw KN, Bachur RG, eds. Fleisher &
Ludwigs’ Textbook of Pediatric Emergency Medicine. 8th ed. Wolters Kluwer; 2021:1050-1052.
Content Domain
Emergency Medicine
A previously healthy 2-year-old is evaluated in the emergency department after ingesting an unknown
amount of oven cleaner approximately 30 minutes ago. He was playing at home when his mother found him
crying near a container of oven cleaner she had just used. They were drooling, appeared uncomfortable, and
had blue liquid on their chin.
On physical examination, the child has a heart rate of 130 beats/min, respiratory rate of 25 breaths/min,
blood pressure of 95/60 mm Hg, and oxygen saturation of 97% in room air. They are drooling, and their
perioral skin is erythematous. The patient’s lungs are clear to auscultation with good aeration and no
wheezing or increased work of breathing. The remainder of the physical examination findings are
unremarkable.
Of the following, the BEST next step in this child’s management is to
D. perform endoscopy
Correct answer is D
PREP Pearl(s)
Ingestion of cleaning agents containing strong acids or alkali can lead to esophageal injury. Children
who ingest substances with a pH of less than 2 or greater than 12 should be evaluated with endoscopy
12 to 48 hours after the ingestion.
Administering activated charcoal or neutralizing agents to children who have ingested acid or alkali is
contraindicated because of the risk of further injury.
Contact information for Poison Control (1-800-222-1222) should be provided to parents and caregivers
at health supervision visits for toddlers and young children.
Critique
The child described in the vignette ingested oven cleaner, a common household alkali with a pH of 12 to 14.
Alkali is caustic to tissues and can cause significant esophageal injury when ingested. Of the response
choices, the best next management step is to perform endoscopy in 12 to 48 hours to evaluate the extent of
esophageal injury.
Administration of activated charcoal is contraindicated as it could cause vomiting and further esophageal
injury or aspiration leading to airway injury. A neutralizing agent is contraindicated because it can cause an
exothermic reaction and further injury to the esophagus. Intentional exposure to household cleaning
products as a form of child abuse should be considered in any such situation; however, the history for the
child in the vignette is fitting for the exposure, and they were immediately brought to the emergency
department, making child abuse unlikely. A careful history and physical examination are essential to identify
children who may have been forced to ingest toxic substances.
A common household alkali is sodium hydroxide, which is found in lye and oven cleaners. Injury to the
esophagus after ingestion of sodium hydroxide is caused by liquefactive necrosis, leading to deep or full-
thickness injuries and potentially perforation. Laundry detergent pods, which contain sodium hydroxide, are
a significant risk to young children who are attracted to their bright colors. Recently, teenagers have ingested
laundry pods in response to social media challenges. These highly concentrated detergents, contained in a
water soluble membrane, can cause extensive injury when ingested or ocular exposure occurs. Additional
severe manifestations of alkali ingestion may include, but are not limited to, aspiration leading to pulmonary
edema, coma, and death.
Common household acids include sulfuric acid, nitric acid, and hydrochloric acid, which are found in toilet
bowl and drain cleaners. Ingestion of these corrosive acids can cause coagulation necrosis of the esophagus.
Acids are less likely than alkali to cause full-thickness injuries and perforations.
Treatment of alkali or acid ingestion is largely supportive. The skin and eyes should be decontaminated by
removing clothing, cleaning the skin, and flushing the eyes with water. Endoscopy should be considered 12 to
48 hours after ingestion of substances with a pH less than 2 or greater than 12; external physical examination
findings do not accurately predict the extent of internal injury. Circumferential injuries confer a long-term risk
of strictures; affected children should be referred to a gastroenterologist.
Urgent consultation with Poison Control (1-800-222-1222) is important for treatment and tracking of all
ingestions. The contact information for Poison Control should be provided to parents and caregivers at
health supervision visits for toddlers and young children.
Eighty percent to 90% of ingestions by toddlers and preschool-aged children reported to Poison Control
occur within the home. Household products, including personal care products, cleaning agents, and foreign
bodies, account for the majority of calls but only approximately 2% of fatalities. Ingestions of acids, alkali,
hydrocarbons, and bleach are most likely to require hospitalization. While many household products have
low concentrations of harmful ingredients, furniture polish, drain cleaner, and oven cleaner can cause
significant harm. The Consumer Product Safety Commission enforces the Poison Prevention Packaging Act of
1970, which requires potentially harmful products to have child-resistant packaging; this packaging has
significantly decreased childrens’ exposure to harmful chemicals.
Suggested Reading(s)
Goodbye winter grime, hello safe spring cleaning! Poison Control: National Capital Poison Center. 2022.
Accessed February 28, 2022. https://www.poison.org/articles/spring-cleaning
Household chemical products and their health risk. Cleveland Clinic. Accessed February 28, 2022.
https://my.clevelandclinic.org/health/articles/11397-household-chemical-products-and-their-health-
risk
McKenzie L, Ahir N, Stolz U, Nelson N. Household cleaning product-related injuries treated in US
emergency departments in 1990-2006. Pediatrics. 2010;126(3):509-516. doi:10.1542/peds.2009-3392
O’Donnell KA. Pediatric toxicology: household product ingestions. Pediatr Ann. 2017;46(12):e449-e453.
doi:10.3928/19382359-20171120-04
Ricca RL, Drugas GT. Esophageal caustic injury. In: McInerny TK, Adam HM, Campbell DE, DeWitt TG,
Foy JM, Kamat DM, eds. American Academy of Pediatrics Textbook of Pediatric Care. American
Academy of Pediatrics; 2021:chap 359. Accessed February 28, 2022. Pediatric Care Online
Sheridan DC, Hughes A, Horowitz BZ. Pediatric ingestions: new high-risk household hazards. Pediatr
Rev. 2021;42 1):2-10. doi:10.1542/pir.2019-0309
Content Domain
Emergency Medicine
A 16-year-old-girl is evaluated in the emergency department for a suspected overdose. She was found at
home with altered mental status after attending a “house party” with her friends. In the emergency
department the girl is confused, with minimal response to painful stimuli. Her vital signs include a
temperature of 38.2°C, heart rate of 138 beats/min, respiratory rate of 12 breaths/min, blood pressure of
118/75 mm Hg, and oxygen saturation of 97% in room air. Her physical examination findings are otherwise
unremarkable. Resuscitation efforts are initiated, including stabilization of her airway, breathing, and
circulation.
Correct answer is C
PREP Pearl(s)
Initial stabilization of the airway, breathing, and circulation is the first step in treating a child with
unknown ingestion.
After initial stabilization of a child with an unknown ingestion, it is critically important to perform point-
of-care bedside glucose testing. Hypoglycemia can be a complication of an ingestion (eg, ethanol, β-
blockers, oral hypoglycemics, and salicylates) or an insulin injection.
Consultation with a local Poison Control center is available 24 hours a day by calling 1-800-222-1222.
Critique
The best next step in treatment of the girl in the vignette is to obtain a point-of-care bedside glucose test to
evaluate for hypoglycemia, a rapidly reversible cause of altered mental status. Determining the best
treatment approach for a child with a suspected or unknown ingestion can be difficult. After initial
stabilization of the airway, breathing, and circulation (ABCs), it is critically important to measure the glucose
level. Hypoglycemia can be a complication of an ingestion (eg, ethanol, β-blockers, oral hypoglycemics, and
salicylates) or an insulin injection.
Activated charcoal should only be used to treat children with an intact airway, normal mental status, and
ability to safely swallow (ie, children able to protect their airway) and only if the ingestion was witnessed. It
should be given within 2 hours of the ingestion and should not be administered via nasogastric tube. Serum
electrolyte levels and a urine drug screen may help identify specific causes of poisoning; however, these test
results will not be immediately available and are not the most appropriate next step in the management of
this child.
Dermal
Envenomation
Inhalation
Ocular
Oral/rectal/vaginal
Parenteral
Transplacental
The initial evaluation and treatment of a child with a suspected poisoning and altered mental status should
include the following:
Consultation with Poison Control can be extremely helpful in managing any child with acute poisoning. The
American Association of Poison Control Centers is available 24 hours a day to both the general public and
health care practitioners. Center staff, trained in toxicology, can provide triage, protocol, and treatment
recommendations for poisoned patients. In addition, these centers provide education programs for the
public and health care professionals. Poison Control centers also collect data that are reported to the Toxic
Exposure Surveillance System; this is vital to tracking exposures and poisonings nationally, including
morbidity and mortality, epidemiology, trends, and new or emerging poisonings.
Suggested Reading(s)
Cossey MG, Gambill LK. "Lucy, a 16-Year-Old Girl with Altered Mental Status." In: Pediatric Hospital
Medicine: A Case-Based Educational Guide. American Academy of Pediatrics; 2022.
Fine JS. Poisoning. In: McInerny TK, Adam HM, Campbell DE, DeWitt TG, Foy JM, Kamat DM, eds.
American Academy of Pediatrics Textbook of Pediatric Care. 2nd ed. American Academy of Pediatrics;
2017:1627-1636. Pediatric Care Online
Lumba-Brown A. Drug overdose. In: McInerny TK, Adam HM, Campbell DE, DeWitt TG, Foy JM, Kamat
DM, eds. American Academy of Pediatrics Textbook of Pediatric Care. 2nd ed. American Academy of
Pediatrics; 2017:1627-1636. Pediatric Care Online
Toce MS, Burns MM. The poisoned pediatric patient. Pediatr Rev. 2017;38(5):207-220.
doi:10.1542/pir.2016-0130
Young KD. Ingestions: diagnosis and management. In: Berkowitz CD. Berkowitz’s Pediatrics: A Primary
Care Approach. 6th ed. American Academy of Pediatrics; 2020:591-598.
Content Domain
Emergency Medicine
A 12-year-old girl is brought to the office after falling off her scooter and striking her face on the curb. She did
not lose consciousness. She complains of nasal and right-sided facial pain and denies neck or back pain.
Physical examination reveals swelling and ecchymosis of her right cheek and nasal bridge, and bleeding from
her right naris. Her pupils are equal and reactive to light, extraocular movements are intact bilaterally,
dentition is intact, and there is no bleeding from her mouth. Her nasal bleeding is controlled with 15 minutes
of application of direct pressure, after which, reexamination of the nasal passages reveals a bluish bulging
area of the septum on the right side. Head computed tomography findings are normal. Computed
tomography of her face shows a nondisplaced fracture of the nasal bone.
D. refer her to the emergency department for incision and drainage and nasal packing
Correct answer is D
PREP Pearl(s)
A nasal septal hematoma requires urgent incision (horizontal or L-shaped) and drainage and nasal
packing to prevent avascular necrosis of the nasal septum.
A nasal septal hematoma may appear as a bluish, purplish, or cherry-like bulging tender mass of the
nasal septum.
Common complications from nasal septal hematomas include re-accumulation of the hematoma and
the development of a nasal septal abscess.
Critique
The girl in the vignette has a hematoma of the nasal septum which requires urgent drainage with a horizontal
or L-shaped incision through the mucosa. This procedure is necessary to prevent avascular necrosis of the
nasal septum.
Pressure from bleeding into the nasal septum can cause disruption of the blood supply to the cartilaginous
nasal septum, with a significant potential for complications. This always requires urgent intervention. Neither
bilateral nasal packing and follow-up with an otolaryngologist, nor chemical cauterization and nasal packing is
the best next step in her management, as the girl would still require drainage of the nasal septal hematoma.
Needle aspiration is less efficient than incision and drainage in evacuating a nasal septum hematoma and has
a high risk of reaccumulation of blood; therefore, this is not the preferred method of treatment.
A nasal septal hematoma may occur unilaterally or bilaterally and may or may not be associated with a nasal
fracture. On physical examination, a nasal septal hematoma may appear as a bluish, purplish, or cherry-like
bulging tender mass. The vascular supply to the nasal septal cartilage is provided by the perichondrium. A
nasal septal hematoma can separate the perichondrium from the septal cartilage, disrupting the blood
supply. This can result in avascular necrosis of the nasal septum and lead to septal perforation and nasal and
facial deformities. Drainage should be performed urgently in order to restore blood flow to the septal
cartilage from the perichondrium.
Common complications from nasal septal hematomas include re-accumulation of the hematoma and the
development of a nasal septal abscess. Therefore, treatment should include nasal packing (to decrease the
risk of re-bleeding) and a broad-spectrum antibiotic. Affected children should undergo frequent reevaluation
by an otolaryngologist to monitor for hematoma reaccumulation and signs or symptoms of infection. Other
complications may include cosmetic deformities and spread of infection causing meningitis, intracranial
abscess, orbital cellulitis, or venous sinus thrombosis.
The management of a nasal bone fracture is dependent upon its complexity, location, and displacement.
Displaced, complex injuries with high risk of complication may require emergency consultation with an
otolaryngologist and surgical repair. For isolated, nondisplaced, or minimally displaced nasal fractures,
conservative management with cold compresses, oral analgesia, and nasal decongestants are appropriate.
Suggested Reading(s)
Ginsburg CM. Consultation with the specialist: nasal septal hematoma. Pediatr Rev. 1998;19(4):142-143.
doi:10.1542/pir.19-4-142
Lopez M, Liu J, Hartley B, Myer C. Septal hematoma and abscess after nasal trauma. Clin Pediatr.
2000;39:609-610. doi:10.1177/000992280003901006
Savage RR, Valvich C. Hematoma of the nasal septum. Pediatr Rev. 2006;27(12):478-479.
doi:10.1542/pir.27-12-478
Kirolos H. Epistaxis. Point-of-Care Quick Reference. Pediatric Care Online. American Academy of
Pediatrics; 2020. Accessed September 1, 2023. Pediatric Care Online
Content Domain
Emergency Medicine
The correct answer is: refer her to the emergency department for incision and drainage and nasal packing
A 2-year-old child with no significant birth or medical history is brought to the emergency department by
ambulance for decreased responsiveness. He was in his normal state of health today until he suddenly
became sleepy and less responsive. His mother found 2 open medication bottles in the bathroom, one
containing clonidine tablets and the other containing methylphenidate tablets, which his older brother takes
for attention-deficit/hyperactivity disorder. She is unsure how many tablets are missing from each of the
bottles. On physical examination, the boy appears lethargic and pale. His temperature is 36.5 °C, heart rate is
65 beats/min, respiratory rate is 8 breaths/min, blood pressure is 60/39 mm Hg, and oxygen saturation is
90% in room air. An intravenous fluid bolus with normal saline is initiated.
Correct answer is C
PREP Pearl(s)
The management of any acutely poisoned child begins with stabilization of the airway, breathing, and
circulation.
Clonidine is a central-acting α2-adrenergic agonist; acute poisoning can cause central nervous system
depression, miosis, respiratory depression or apnea, hypotension, and bradycardia.
Naloxone is an effective antidote for clonidine poisoning; very large (up to 10 mg) and repeated doses
may be required.
Critique
The boy in the vignette is exhibiting signs and symptoms of a severe clonidine overdose. The child has
hypotension, bradycardia, bradypnea, and profound central nervous system (CNS) depression. The
management of any acutely poisoned child begins with stabilization of airway, breathing, and circulation
(ABCs). Thus, the best next step in this child’s management is to begin bag-valve mask ventilation while
preparing for intubation to secure the airway and manage breathing.
Although activated charcoal binds clonidine, it should not be administered to a child who has impaired
breathing, poor airway reflexes, or CNS abnormality due to the risk of aspiration. When used to treat children
with stable ABCs and no CNS depression, activated charcoal is most effective if administered within the first
hour of toxin ingestion.
Flumanzenil is the antidote for benzodiazepine poisoning and thus would not be an appropriate treatment
for this child. Cardiopulmonary resuscitation (CPR) is not indicated for this child; however, initiation of chest
compressions and CPR should begin immediately if his heart rate decreases below 60 beats/min with poor
perfusion despite adequate ventilation and oxygenation.
Clonidine is a centrally acting α2-adrenergic agonist that is commonly used in the management of
hypertension; it has recently gained popularity for the treatment of attention-deficit/hyperactivity disorder. It
is an extremely powerful drug and ingestions of very small amounts (“one pill can kill”) can lead to significant
toxicity. Symptoms of clonidine toxicity may include the following:
Bradycardia
CNS depression (ranging from lethargy to coma)
Hypotension
Hypothermia
Miosis
Respiratory depression or apnea
Transient and brief hypertension
The Table outlines a toxicologic differential diagnosis for bradycardia and hypotension.
Naloxone is an effective antidote for clonidine poisoning; very large doses of naloxone (maximum single dose
of 10 mg) may be required. Administration of 0.1 to 0.2 mg/kg of naloxone (up to 10 mg) may reverse CNS
depression and improve the child’s mental status. It is often necessary to give repeat doses of naloxone or
start a continuous infusion due to the long half-life of clonidine.
Suggested Reading(s)
Fine JS. Poisoning. In: McInerny TK, Adam HM, Campbell DE, et al, eds. American Academy of Pediatrics
Textbook of Pediatric Care. American Academy of Pediatrics, 2023. Accessed September 1,
2023. Pediatric Care Online
Mangus CW, Canares TL. Toxic ingestions: initial management. Pediatr Rev. 2018;39(4):219–221.
doi:10.1542/pir.2017-0119
Seger DL, Loden JK. Naloxone reversal of clonidine toxicity: dose, dose, dose. Clin Toxicol. 2018;56:873-
879. doi:10.1080/15563650.2018.1450986
Toce MS, Burns MM. The poisoned pediatric patient. Pediatr Rev. 2017;38(5):207–220.
doi:10.1542/pir.2016-0130
Content Domain
Emergency Medicine
A previously healthy 2-year-old is brought to the emergency department after their mother found them
sleepy and not responsive. They are obtunded and responsive only to painful stimuli. Their vital signs are a
temperature of 36.8 °C, blood pressure of 86/53 mm Hg, a heart rate of 84 beats/min, a respiratory rate of 52
breaths/min, and an oxygen saturation of 94% in room air. The patient’s pupils are equal and reactive to
light. A cardiovascular examination reveals a regular rate and rhythm with no murmur and good peripheral
pulses. Their lungs are clear to auscultation with good aeration; there are no adventitious sounds and no
accessory muscle use. The patient has normal tone and reflexes, and there is no bruising or signs of head
trauma.
Urinalysis Crystals
A. alcohol intoxication
C. postictal state
Correct answer is B
PREP Pearl(s)
Unintentional poisoning is a major cause of morbidity and mortality in the pediatric population,
especially in children aged 1 to 3 years.
The findings of an elevated anion gap metabolic acidosis, hypocalcemia, and urine calcium oxalate
crystals is highly suggestive of ethylene glycol toxicity.
Early recognition of ethylene glycol toxicity and treatment with fomepizole is crucial to prevent
morbidity and mortality.
Critique
The child in the vignette has a mental status change, elevated anion gap metabolic acidosis, mild elevation in
creatinine concentration, hypocalcemia, and crystals (calcium oxalate) in his urine. This constellation of
findings is highly suggestive of ethylene glycol ingestion.
An elevated anion gap reflects the presence of anions unbalanced by cations in the serum. The differential
diagnosis of an elevated anion gap acidosis is suggested by the acronym MUDPILES, which stands for
methanol, urea, diabetic ketoacidosis, propylene glycol/paraldehyde, iron/isoniazid, lactate, ethylene
glycol/ethanol, and salicylates.
Alcohol intoxication also causes mental status change, sleepiness, and anion gap acidosis. However, alcohol
toxicity does not lead to formation of calcium oxalate crystals in the urine. Children with alcohol intoxication
often have hypoglycemia, which is not present in the patient described in the vignette. A child in the postictal
phase after a seizure may be sleepy and difficult to arouse, but this would not explain this child’s profound
anion gap metabolic acidosis and urine findings. Renal tubular acidosis causes a non–anion gap metabolic
acidosis resulting from bicarbonate wasting in the urine, and it would not cause the acute clinical
presentation exhibited by this child.
Ethylene glycol poisoning, a rare occurrence in children, is associated with significant morbidity and mortality,
even in small doses. Ethylene glycol is a sweet-tasting, colorless, odorless liquid found in antifreeze, de-icing
solutions, window cleaners, cosmetics, and paints. It is rapidly absorbed from the gastrointestinal tract.
Symptoms of toxicity develop within 20 to 30 min of ingestion. Metabolic acidosis is caused by accumulation
of glycolic acid, some of which is converted to oxalic acid. The oxalic acid then binds to calcium and forms
calcium oxalate crystals. Ethylene glycol toxicity manifests in stages, with some overlap between them
(Table).
Early recognition and treatment of ethylene glycol toxicity is crucial. The ingestion by children may not be
observed by caregivers. Ethylene glycol poisoning should be suspected in a young child who has mental
status changes, anion gap acidosis, and urine calcium oxalate crystals. Serum ethylene glycol levels should be
sent for confirmation; however, treatment should not be delayed while awaiting test results.
Fomepizole is the targeted treatment for ethylene glycol poisoning. It works by blocking alcohol
dehydrogenase (the first step in ethylene glycol metabolism), thereby preventing formation of toxic
metabolites. Fomepizole is administered intravenously (15 mg/kg bolus over 30 min followed by 10-15 mg/kg
every 12 hours) until the serum level of ethylene glycol is <20 mg/dL. During treatment, children should be
monitored for headache, dizziness, eosinophilia, and transient elevation of liver enzymes. Thiamine and
pyridoxine are used as adjuvant therapies that direct ethylene glycol to less toxic pathways. Hemodialysis is
indicated for extremely high levels of ethylene glycol, severe acidosis, or worsening acute kidney injury.
Unintentional poisoning is a major cause of morbidity and mortality in the pediatric population, especially in
children aged 1 to 3 years. Poison control centers report more than 2 million calls annually, of which about
65% involve toxic ingestions/exposures in children and adolescents. An accidental ingestion or exposure to a
toxic substance should be suspected in a young child with mental status changes, even in the absence of
such history. As most accidental poisoning occurs in the home, anticipatory guidance on prevention is crucial,
starting at the 6-month health supervision visit.
Suggested Reading(s)
Cagil Y, Diaz J, Iskowitz S, Muñiz Crim AJ. Ingested foreign bodies and toxic materials: who needs to be
scoped and when? Pediatr Rev. 2021;42(6):290-301. doi:10.1542/pir.2018-0327
Fine JS. Poisoning. In: McInerny TK, Adam HM, Campbell DE, Foy JM, Kamat DM, eds. American
Academy of Pediatrics Textbook of Pediatric Care. 2nd ed. American Academy of Pediatrics; 2016:chap
369. Accessed September 1, 2023. Pediatric Care Online
McQuade DJ, Dargan PI, Wood DM. Challenges in the diagnosis of ethylene glycol poisoning. Ann Clin
Biochem. 2014;51(pt 2):167-178. doi:10.1177/0004563213506697
Toce MS, Burns MM. The poisoned pediatric patient. Pediatr Rev. 2017;38(5):207-220.
doi:10.1542/pir.2016-0130
Content Domain
Emergency Medicine
A 10-year-old boy is seen in the emergency department after falling approximately 6 ft from a horse, which
subsequently stepped on him. He was wearing a helmet and did not lose consciousness. The boy is crying
and experiencing right-sided abdominal and flank pain. His temperature is 37.5 °C, heart rate is 122
beats/min, respiratory rate is 20 breaths/min, blood pressure is 106/75 mm Hg, and oxygen saturation is 97%
in room air. On physical examination, he has diffuse tenderness to palpation and guarding of his abdomen,
with ecchymosis of the right abdominal wall and flank. His penis appears normal, and his scrotum is
nontender. The remainder of his examination findings are unremarkable. After blood is obtained for initial
laboratory studies and an intravenous fluid bolus is initiated, the boy has a spontaneous void that appears
grossly bloody. Imaging of his abdomen and pelvis is obtained.
Of the following, the MOST appropriate diagnostic imaging modality for this child is
Correct answer is A
PREP Pearl(s)
The gold standard imaging modality for children with blunt abdominal trauma is intravenous contrast-
enhanced computed tomography of the abdomen and pelvis.
A child who has sustained abdominal trauma (including nonaccidental trauma) requires a thorough
multisystem evaluation.
Evaluation of the entire genitourinary system is indicated for children with hematuria (gross hematuria
or microscopic with >50 red blood cells per high-power field) after a traumatic abdominal injury.
Critique
The child in the vignette is hemodynamically stable with signs and symptoms of blunt abdominal trauma. The
gold standard imaging modality for children with blunt abdominal trauma (diagnostic accuracy as high as
98%) is intravenous contrast-enhanced computed tomography (CT) of the abdomen and pelvis. The sensitivity
of the focused assessment with sonography in trauma (FAST) examination is much lower; significant
abdominal injuries can be missed. Currently, there is no role for magnetic resonance imaging of the
abdomen and pelvis in the immediate evaluation of blunt abdominal trauma. Although upright and supine
radiography of the abdomen may be useful in identifying fractures or free air, this modality cannot identify
many of the injuries sustained by children with abdominal trauma.
A child who has sustained abdominal trauma (including nonaccidental trauma) requires a thorough
multisystem evaluation. Laboratory evaluation should include a complete blood cell count, basic metabolic
panel, liver enzyme and lipase levels, and a urinalysis. Imaging studies may include ultrasonography, plain
radiography, and/or computed tomography.
Indications for abdominal and pelvic computed tomography with intravenous contrast include the following:
Evaluation of the entire genitourinary system is indicated for children with hematuria (gross or microscopic
with >50 red blood cells per high-power field) after a traumatic abdominal injury. Depending on the severity
and type of injury suspected, this evaluation may include serial urinalysis and intravenous contrast-enhanced
CT (including delayed-phase imaging to fully evaluate the renal pelvis and ureter). More specialized imaging
studies (eg, CT urography, CT cystography, retrograde pyelography, or retrograde urethrography) should only
be obtained per the recommendation of a urologist.
Suggested Reading(s)
Burbulys DB. Abdominal trauma. In: Berkowitz CD. Berkowitz’s Pediatrics: A Primary Care Approach.
6th ed. American Academy of Pediatrics; 2020:543-548.
Dubowitz H, Finkel MA. Physical abuse and neglect. In: McInerny TK, Adam HM, Campbell DE, DeWitt
TG, Foy JM, Kamat DM, eds. American Academy of Pediatrics Textbook of Pediatric Care. 2nd ed.
American Academy of Pediatrics; 2017:chap 367. Pediatric Care Online
Kovell, RC, Tasian GE, Belfer RA. Genitourinary trauma. In: Bachur RG, Shaw KN, Chamberlain J, Lavelle
J, Nagler J, Shook J. Fleisher & Ludwig's Textbook of Pediatric Emergency Medicine. 8th ed. Wolters
Kluwer Health; 2020:1131-1142.
Mirkinson LJ, Reed PL, Vezzetti R, Cavalier ME. Index of suspicion. Pediatr Rev. 2005;26(5):183-187.
doi:10.1542/pir.26-5-183
Content Domain
Emergency medicine
A 16-month-old girl is seen in the emergency department for evaluation of burns to the right arm. Her
mother was cooking in the kitchen today and was unaware that the patient was behind her. When her
mother opened the oven to retrieve the food, the girl touched the hot oven door with her right hand and
arm. Her vital signs include a temperature of 37.6 °C, heart rate of 135 beats/min, respiratory rate of 24
breaths/min, blood pressure of 95/70 mm Hg, and oxygen saturation of 99% in room air. She is crying and
appears to be in pain. There are burns to her right hand, forearm, and antecubital fossa with blistering
(Figure 1). The remainder of her physical examination findings are unremarkable. The girl’s immunizations
are up to date, and she has appropriately met her developmental milestones. Appropriate analgesics are
administered to the child.
Courtesy of T. Swan
A. contact the local child abuse hotline to report suspected child abuse
B. debride the burns of loose and blistered skin and apply a sterile dressing
C. immediate consultation with consideration for transfer to the nearest burn center
D. wrap the burn with clean, dry, sterile gauze and prescribe a 10-day course of oral antibiotics
Correct answer is C
PREP Pearl(s)
Burns are classified as superficial, partial-thickness, or full-thickness dependent on the layers of skin
involved. Burns are often heterogeneous, with areas of superficial, partial-thickness, and full-thickness
burns.
Burns evolve over the first 24 to 72 hours, so they must be reassessed frequently to accurately assess
the burn depth.
Burns to the oral commissure are high risk for complications; they require specialized attention to
monitor for scarring and contractures which may affect mouth function. Life-threatening bleeding from
the labial artery may occur 1 to 2 weeks after the injury, when the eschar separates from the corner of
the mouth.
Critique
The girl in the vignette has partial-thickness burns to her hand, forearm, and antecubital fossa which cross
joint lines. Because the burns involve the hand and cross joint lines, immediate consultation with
consideration for transfer to the nearest burn center is the next best step in her management. Table 1 lists
the American Burn Association Burn Center Transfer Criteria. Although the child will require debridement of
the loose tissue and large bullae, this should not occur prior to consultation, and if recommended, transfer to
a burn center. Wrapping the burn with clean, dry, sterile gauze and prescribing a 10-day course of oral
antibiotics would not be appropriate, as treatment of burns with prophylactic antibiotics is not
recommended.
It is important for providers to recognize history and physical examination findings that are suspicious for
child abuse. Careful attention must be paid to the reported history of the presenting injury within the context
of the child’s age, physical capabilities, and developmental stage. A reported history of the event that has
inconsistencies, does not adequately explain the physical examination findings, or is not consistent with the
child’s age or developmental stage should raise concern for child abuse. The girl in the vignette has small,
noncontiguous areas of partial-thickness burns that are not circumferential. The history is consistent with the
report of her touching an open, hot stove door and with her capabilities and developmental stage.
Characteristics suggestive of burns caused by abuse include: delayed presentation for care, history not
matching the burn injury, deep or extensive burns, burns in a patterned shape (eg, an iron or cigarette),
“stocking or glove” distribution, bilateral burns of the extremities, or clear demarcation of the burn edges
without splash marks.
Burns may result from contact with hot liquids or objects, electricity, flames, chemicals, or extremely cold
objects or environments. Burns are classified as superficial, partial thickness, or full thickness dependent on
the layers of skin involved (Table 2). It is important to note that burns are often heterogeneous in nature and
may include areas of superficial, partial-thickness and full-thickness burns. Burns also evolve over the first 24
to 72 hours, so they must be reassessed frequently to accurately assess the burn depth.
It is important to assess the total body surface area (TBSA) of a burn. This can be calculated using
commercially available burn charts. Only burns that are partial thickness or full thickness should be included
in the TBSA (superficial burns are not included). Burn management depends on the percentage of TBSA, burn
location, airway involvement, and potential for toxicologic exposure (eg, carbon monoxide, cyanide
poisoning).
Inflicted burns, electrical burns, and chemical burns require special consideration. Children with suspected
inflicted burns must be thoroughly evaluated for other traumatic injuries. Children with burns from high-
voltage electrical current require intensive evaluation for internal injuries and myocardial injury at a burn
center. One common type of electrical burn seen in children is an electrical injury to the tongue or lips that
occurs when children bite into a live electrical cord. Deep burns at the oral commissure, such as those seen in
Figure 2, Figure 3, and Figure 4, are considered high risk for scarring and contractures, which may affect
mouth function; these burns require specialized attention and monitoring. In addition, with this type of burn
there is a risk of severe, life-threatening bleeding from the labial artery; this typically occurs 1 to 2 weeks
later, when the eschar separates from the corner of the mouth. Chemical burns require specific
decontamination procedures depending on the type of chemical (water can actually worsen some types of
chemical burns). Chemical burns require consultation with a burn specialist and a toxicologist.
Courtesy of T. Swan
Courtesy of T. Swan
Suggested Reading(s)
Guidelines for burn patient referral. American Burn Association.
https://ameriburn.org/resources/burnreferral/. Published 2022. Accessed May 23, 2023.
Dubowitz H, Finkel MA. Child physical abuse and neglect. In: McInerny TK, Adam HM, Campbell DE,
DeWitt TG, Foy JM, Kamat DM, eds. American Academy of Pediatrics Textbook of Pediatric Care.
American Academy of Pediatrics; 2021:chap 367. Accessed September 1, 2023. Pediatric Care Online
Klein GL, Herndon DN. Burns. Pediatr Rev. 2004;25(12):411-417. doi:10.1542/pir.25-12-411
Sheridan RL. Burn care for children. Pediatr Rev. 2018;39(6):273-286. doi:10.1542/pir.2016-0179
Content Domain
Emergency Medicine
&KHPLFDO ̽ $OOFKHPLFDOLQMXULHV
LQMXULHV
(OHFWULFDO ̽ $OOKLJKYROWDJH ͡9 ̽ /RZYROWDJH 9
LQMXULHV HOHFWULFDOLQMXULHV HOHFWULFDOLQMXULHV
̽ /LJKWQLQJLQMXU\ VKRXOGUHFHLYH
FRQVXOWDWLRQDQG
FRQVLGHUDWLRQIRU
IROORZXSLQDEXUQ
FHQWHUWRVFUHHQIRU
GHOD\HGV\PSWRPRQVHW
DQGYLVLRQSUREOHPV
$EEUHYLDWLRQ7%6$WRWDOEXUQVXUIDFHDUHD
5HSULQWHG LQSDUW ZLWKSHUPLVVLRQIURP$PHULFDQ%XUQ$VVRFLDWLRQ&RS\ULJKWk
$PHULFDQ%XUQ$VVRFLDWLRQ
7DEOH%XUQ&ODVVLͤFDWLRQDQG&KDUDFWHULVWLFV
Burn /D\HURI6NLQ,QYROYHG &KDUDFWHULVWLFV
&ODVVLͤFDWLRQ
6XSHUͤFLDO ̽ (SLGHUPLVRQO\ ̽ 5HG
IRUPHUO\ͤUVW ̽ 'U\
degree) ̽ 1RHGHPD
̽ 1REXOODHRUEOLVWHULQJ
̽ %ODQFKHVZLWKSUHVVXUH
̽ 6HQVDWH
̽ 3DLQIXO
̽ 5HVROYHVLQWRGD\V
̽ 1RVFDUULQJ
&RXUWHV\RI76ZDQ
AAP PREP 2024 - Question 43/267 Emergency Medicine Question 9/14
A 3-year-old girl is brought to the emergency department after unintentional ingestion of half a bottle of
drain cleaner. Her mother was using an empty water bottle to store the drain cleaner, which she kept under
the kitchen sink. When the mother called Poison Control, they told her the active ingredient in this specific
drain cleaner is sodium hydroxide. After ingestion, the girl immediately began to cry because of mouth pain.
On physical examination in the emergency department, the girl is crying and drooling. Her vital signs include
a temperature of 37.9 °C, heart rate of 98 beats/min, respiratory rate of 22 breaths/min, blood pressure of
105/72 mm Hg, and oxygen saturation of 98% in room air. There is mild erythema of her posterior
oropharynx. The remainder of her examination findings are unremarkable.
Correct answer is C
PREP Pearl(s)
Ingestion of an alkali substance causes liquefactive necrosis, which can lead to full-thickness mucosal injuries
and perforation.
Twenty to forty-five percent of patients with esophageal burns have no evidence of oral injury or oral burns.
Endoscopy, performed 12 to 48 hours after caustic ingestion, is indicated if the child is symptomatic,
intentionally ingested the caustic substance, or ingested a product with a concentration sufficient to produce a
corrosive effect, even in the absence of visible oropharyngeal injury.
Critique
The child in the vignette ingested drain cleaner, which is a caustic substance (pH <2 or >12). Drain cleaners
commonly contain sodium hydroxide, a strong alkali with a pH of 12 to 14. Ingestion of an alkali substance
causes liquefactive necrosis, which can lead to full-thickness mucosal injuries and perforation. Therefore, the
next best step in this child’s management is to obtain an urgent gastroenterology consultation for endoscopy;
endoscopy should be performed with the patient under sedation between 12 to 48 hours after ingestion.
The administration of steroids after caustic ingestion remains controversial and in some cases (grade III
mucosal burns) may be more harmful than beneficial; it is, therefore, not the best next step in the treatment
of this patient. Although ingestion of alkali and acid-containing products cause direct harm to mucosal
surfaces, they are not known to affect serum electrolytes or lactate immediately after ingestion.
Gastrointestinal decontamination with activated charcoal, gastric lavage, whole bowel irrigation, or a
neutralization substance is contraindicated in caustic ingestions because of the risk of further injury. Both the
blind placement of a nasogastric tube and nasal intubation are contraindicated in this situation because of
the risk of perforation of injured laryngeal and/or esophageal structures.
Many household cleaners contain alkali or acidic agents. Alkali substances cause liquefactive necrosis,
producing deep or full-thickness injury that may lead to perforation of the mucosa of the mouth, esophagus,
stomach, intestine, and/or respiratory tract. In contrast, acidic substances cause coagulation necrosis, which
typically results in more superficial damage rather than deep, penetrating burns.
The immediate management of any child with a caustic ingestion should be, when possible, to determine the
agent ingested. The ingredients and the caustic potential of the product can be determined through
consultation with Poison Control (1-800-222-1222). Rapid assessment of the airway and gastrointestinal tract
should be performed. Laryngeal injury may progress rapidly to life-threatening airway obstruction. Soft tissue
neck radiography may be useful in evaluation for swelling in the prevertebral space, free air, or laryngeal
edema; however, this imaging technique is not as sensitive for detecting swelling of or the extent of injury to
the esophagus or trachea, which must be assessed by direct visualization using bronchoscopy and
endoscopy.
Presenting signs and symptoms of caustic ingestion may include the following:
Twenty to forty-five percent of patients with esophageal burns have no evidence of oral injury or burns.
Therefore, endoscopy is indicated 12 to 48 hours after caustic ingestion for any child who meets the following
criteria:
Symptomatic
Intentional ingestion
Ingestion of a potentially toxic product with a concentration sufficient to produce a corrosive effect
Ingestion of an unknown product
Complications of caustic ingestion may include perforation, mediastinitis, hemorrhage secondary to vessel
erosion, tracheoesophageal or aortoesophageal fistula formation, stricture formation, gastric outlet
obstruction, esophageal dysmotility, and esophageal carcinoma.
Suggested Reading(s)
Cagil Y, Diaz J, Iskowitz S, Muñiz Crim AJ. Ingested foreign bodies and toxic materials: who needs to be
scoped and when? Pediatr Rev. 2021;42(6):290-301. doi:10.1542/pir.2018-0327
Lovejoy FH, Woolf AD. Corrosive ingestions. Pediatr Rev. 1995;16:473-474.
doi: https://doi.org/10.1542/pir.16-12-473
O’Donnell KA. Pediatric toxicology: household product ingestions. Pediatr Ann. 2017;46(12):e449-e453.
doi:10.3928/19382359-20171120-04
Park KS. Evaluation and management of caustic injuries from ingestion of acid or alkaline substances.
Clin Endosc. 2014;47(4):301-307. doi:10.5946/ce.2014.47.4.301
Ricca RL, Drugas GT. Esophageal caustic injury. In: McInerny TK, Adam HM, Campbell DE, DeWitt TG,
Foy JM, Kamat DM, eds. American Academy of Pediatrics Textbook of Pediatric Care. American
Academy of Pediatrics; 2023. Accessed September 1, 2023. American Academy of Pediatrics;
2016:2857-2862. Pediatric Care Online
Content Domain
Emergency Medicine
A 14-year-old boy is brought to the emergency department (ED) by ambulance after being involved in an all
terrain vehicle (ATV) crash. While the boy was driving at 35 miles per hour, the ATV hit a tree stump, and the
boy’s chest and abdomen hit the handlebars. The boy is in respiratory distress and is experiencing chest
pain. His temperature is 37.6°C, heart rate is 145 beats/min, respiratory rate is 32 breaths/min, blood
pressure is 70/55 mm Hg, and oxygen saturation is 97% in room air. On physical examination, breath sounds
are absent on the left side, his neck veins are distended, and there is tracheal deviation to the right side. The
remainder of his examination findings are unremarkable.
Correct answer is C
PREP Pearl(s)
The clinical presentation of a tension pneumothorax may include respiratory distress, tachycardia,
hypotension, absent breath sounds on the affected side, tracheal deviation away from the affected
side, and distended neck veins.
The diagnosis of a tension pneumothorax is clinical; imaging and other interventions may delay
treatment.
The treatment of tension pneumothorax is immediate decompression by needle thoracostomy
inserted on top of the rib at the second intercostal space in the midclavicular line.
Critique
The boy in the vignette has signs and symptoms of a life-threatening left side tension pneumothorax;
therefore, the best next management step is to perform needle thoracostomy on his left side.
Tension pneumothorax requires immediate decompression by needle thoracostomy. The presence of free air
in the pleural cavity causes increased thoracic pressure and can severely impair venous return to the right
side of the heart and reduce cardiac output. If not quickly recognized and treated immediately, this can result
in cardiac arrest.
Needle decompression is performed by inserting a 14- to 18-gauge needle catheter into the second
intercostal space at the midclavicular line. It is important that the needle be inserted on top of the rib to
avoid injury to the neurovascular bundle that runs beneath the rib. Insertion of a chest tube (tube
thoracostomy) can be performed after needle decompression is complete.
Suggested Reading(s)
American College of Surgeons. Pediatric trauma. In: Advanced Trauma Life Support: Student Course
Manual. 10th ed. American College of Surgeons; 2018:187-212.
Barbut G, Needleman JP. Pediatric chest pain. Pediatr Rev. 2020;41(9):469-480. doi:10.1542/pir.2019-
0058
Zebrack CM, Bratton SL. Pneumothorax and pneumomediastinum. In: McInerny TK, Adam HM,
Campbell DE, et al. American Academy of Pediatrics Textbook of Pediatric Care. 2nd ed. American
Academy of Pediatrics; 2016:chap 368. Pediatric Care Online
Content Domain
Emergency Medicine
The correct answer is: perform needle thoracostomy on the left side
An 8-month-old girl is brought to the emergency department after she was unintentionally left in a hot car for
an hour. Her father picked her up from daycare but, on arriving home, forgot she was in the car and went
inside without her. Vital signs reveal a rectal temperature of 41.1 °C, a blood pressure of 82/54 mm Hg, a
heart rate of 170 beats/min, a respiratory rate of 22 breaths/min, and an oxygen saturation of 92% in room
air. She is lethargic and moans in response to painful stimuli. Her skin is warm, flushed, and dry. Her physical
examination findings are otherwise normal.
A. administer antipyretics
Correct answer is D
PREP Pearl(s)
Rectal temperature measurement is the most accurate method of measuring core body temperature
in infants and young children.
Children should not be left alone in an enclosed vehicle, even for short durations, owing to the risk of
heat stroke.
Prompt initiation of cooling decreases the morbidity and mortality risk associated with heat stroke.
Cold water immersion (eg, ice bath) is indicated to treat athletes with exertional heat stroke;
evaporation is the preferred method for infants and young children with classic heat stroke.
Critique
The infant in the vignette has classic (nonexertional) heat stroke owing to the rapidly rising temperature in an
enclosed vehicle. Immediate treatment should focus on lowering her core temperature after ensuring a
stable airway, breathing, and circulation. In infants and young children, evaporation is the preferred cooling
method. Evaporative cooling can be accomplished by spraying the infant with tepid water and using fans, or
applying cool wet towels to her skin (the towels should be resoaked and reapplied as needed). Cooling should
be discontinued when her core temperature reaches 38.3 to 38.9 °C (100.9-102 °F) to prevent hypothermia.
Cold water immersion (eg, ice bath) is indicated to treat athletes with exertional heat stroke but is not
recommended for infants and young children owing to the risk of bradycardia. Antipyretics are not effective
for hyperthermia associated with heat stroke, because they act through a different physiologic pathway.
Intravenous fluids are not immediately required for this infant because her cardiovascular status is stable
and, therefore, cooling takes priority.
Hyperthermia is an unregulated elevation of core temperature beyond the hypothalamic set point, usually
occurring when the temperature is ≥41.1 °C (106 °F). Causes of hyperthermia include the following:
A temperature higher than 40 °C (104 °F) and altered mental status differentiate heat stroke from heat
exhaustion. The morbidity of heat stroke is related to multiorgan dysfunction and can be reversed with
prompt initiation of cooling. Risk factors for a poor outcome include the degree and duration of
hyperthermia. The mortality rate of severe, untreated heat stroke is approximately 10%, with a 30% risk of
neurological sequelae. Therefore, prevention must be a high priority.
Normal body temperature ranges between 36.5 °C and 37.5 °C (97.7 °F and 99.5 °F); variations occur
according to age, time of day, and measurement method. Normal temperature in young children is slightly
higher compared with that in older children and adults. Because of normal circadian rhythm, temperature
readings are lower in the early morning and higher in the late afternoon and early evening.
Rectal temperature measurement most closely correlates with core temperature. It is the most accurate
method of temperature measurement in infants and children younger than 3 years. In older children, oral
temperature is acceptable. Although oral temperature may be slightly lower than rectal temperature, the
difference is negligible. Axillary temperature measurement is easy to perform and causes the least
discomfort, but it does not accurately correlate with rectal temperature. Tympanic thermometers are
frequently inaccurate owing to improper positioning, cerumen, or inflammation (eg, acute otitis media); this
method is, therefore, not recommended. Similarly, temporal artery scan thermometers have not been shown
to be accurate and are not recommended, especially in infants and young children.
The accepted standard definition of fever in young infants is a rectal temperature of greater than or equal to
38 °C (100.4 °F). The anterior hypothalamus plays an important role in thermoregulation. In response to
infection or inflammation, cytokines are released and circulate to the hypothalamus, resulting in secretion of
prostaglandin E2 and an increased temperature set point. In response, shivering and vasoconstriction
increase core temperature to the raised set point. Later, vasodilation occurs as a compensatory mechanism
to dissipate heat by means of sweating and evaporation.
Although fever in children is rarely harmful, it provokes anxiety and fear in parents. There are clear benefits
of fever, including inhibition of pathogen growth and an enhanced immune response. However, young
children with fever may be fussy and older children often express discomfort, leading to antipyretic
administration for symptomatic relief.
Caregivers should be counseled about appropriate clothing and hydration for children exposed to high
ambient temperatures, especially in infants and children with special needs who may not be able to
communicate discomfort and thirst. Coaches and athletic trainers should follow available guidelines to
prevent exertional heat stroke, monitor athletes with underlying medical conditions, schedule practices
during cooler times of day, and ensure adequate hydration.
Pediatric practitioners should advise parents and caregivers to never leave a child alone in an enclosed car,
even for only a short period. Even when the ambient temperature is mild, the temperature inside an
enclosed vehicle can climb to dangerous levels within 60 minutes (the peak rate of rise occurs during the first
30 minutes). Caregivers should be encouraged to leave important belongings in the rear seat to serve as a
reminder that the child is present. Newer vehicles may be equipped with rear seat reminder alarms.
Suggested Reading(s)
Adam HM. Physiology and management of fever. In: McInerny TK, Adam HM, Campbell DE, Foy JM,
Kamat DM, eds. American Academy of Pediatrics Textbook of Pediatric Care. 2nd ed. American
Academy of Pediatrics; 2016:chap 53. September 1, 2023. Pediatric Care Online
Epstein Y, Yanovich R. Heatstroke. New Engl J Med. 2019;380(25):2449-2459.
doi:10.1056/NEJMra1810762
Mangus CW, Canares TL. Heat-related illness in children in an era of extreme temperatures. Pediatr
Rev. 2019;40(3):97-107. doi:10.1542/pir.2017-0322
McLaren C, Null J, Quinn J. Heat stress from enclosed vehicles: moderate ambient temperatures cause
significant temperature rise in enclosed vehicles. Pediatrics. 2005;116(1):e109-e112.
doi:10.1542/peds.2004-2368
van der Jagt ÉW. Fever. In: McInerny TK, Adam HM, Campbell DE, Foy JM, Kamat DM, eds. American
Academy of Pediatrics Textbook of Pediatric Care. 2nd ed. American Academy of Pediatrics; 2016:chap
152. September 1, 2023. Pediatric Care Online
Content Domain
Emergency Medicine
A 17-year-old boy is brought to the emergency department after being rescued from a house fire. He was in
the burning house for 30 minutes, trapped in an upstairs room, and was rescued through a window. He
complains of shortness of breath, headache, and difficulty focusing his vision.
On physical examination, the adolescent has a blood pressure of 125/60 mm Hg, a heart rate of 100
beats/min, a respiratory rate of 25 breaths/min, and an oxygen saturation via pulse oximetry of 98% in room
air. He smells strongly of smoke and has burns on his hands and arms but not his face. His mucous
membranes are mildly erythematous. He is coughing, appears to have dyspnea, and is using intercostal and
suprasternal muscles to breathe. His breath sounds are coarse, with scattered crackles but no wheezing. The
remainder of his examination findings are unremarkable.
The boy is treated with oxygen and intravenous fluids and undergoes cardiorespiratory monitoring.
Of the following, the BEST next step in this boy’s evaluation is to obtain
A. blood co-oximetry
C. chest radiography
Correct answer is A
PREP Pearl(s)
Pulse oximetry measures the percentage of hemoglobin binding sites that have oxygen attached; it
does not measure tissue oxygen delivery.
Both intrinsic and extrinsic factors affect the validity of pulse oximetry readings and should be
considered when interpreting results.
Acidosis, hypercarbia, and hyperthermia all decrease the affinity of hemoglobin for oxygen, leading to
lower saturation at a given partial pressure of oxygen (PaO2); alkalosis, hypocarbia, and hypothermia
increase the affinity of hemoglobin for oxygen, leading to greater saturation at a given PaO2.
Critique
The boy described in the vignette has a smoke inhalation injury with a high probability of carbon monoxide
poisoning. He has a headache, vision problems, erythematous mucous membranes, and a pulse oximetry
reading (98%) higher than expected for his degree of respiratory distress. Of the response choices,
measurement of serum carboxyhemoglobin with co-oximetry is the best next step in his evaluation. In the
presence of carboxyhemoglobin, a pulse oximetry reading will be spuriously high because it measures both
the oxyhemoglobin and carboxyhemoglobin concentrations. Co-oximetry measures the total concentration
of hemoglobin, broken out into the concentrations of oxygenated hemoglobin, deoxygenated hemoglobin,
carboxyhemoglobin, and methemoglobin.
Treatment of carbon monoxide poisoning includes administration of 100% oxygen, delivered via high-flow
nasal cannula or non-rebreather mask. Carbon monoxide inhalation is the most common cause of death in
fire and smoke-exposure injuries. Failure to recognize and address carboxyhemoglobinemia in smoke
inhalation increases the risk of death.
Although a capillary blood gas estimates carbon dioxide content accurately, it is a poor measure of
oxygenation. An arterial blood gas would be more helpful in assessing the boy’s circulating oxygen delivery.
Serum electrolyte level measurement would not help with his respiratory management, which is currently the
primary concern for his care. A chest radiograph may be indicated in a smoke inhalation injury, but
addressing carbon monoxide inhalation is a higher priority.
It has been suggested that pulse oximetry is the fifth vital sign and that it should be a part of every physical
examination in which vital signs are recorded. Pulse oximetry is a readily available, noninvasive measure of
peripheral oxygen saturation that is applicable for all ages and in virtually any setting, including those outside
the medical establishment. The technology involves transmission of 2 different wavelengths from a light-
emitting diode across oxygenated tissue (eg, fingertip, toe, ear lobe) to a photodetector. Saturation is
determined by the ratio of oxyhemoglobin to the sum of oxyhemoglobin and reduced hemoglobin as
measured by the 2 wavelengths. The values are valid between 70% and 100% saturation but not considered
reliable at <70% saturation.
A variety of individual and environmental factors affect the measurement of oxygen saturation or the
interpretation of the results. Individuals with anemia may have normal oxygen saturation via pulse oximetry
but poor tissue-level oxygen delivery because of low oxygen-carrying capacity. Individuals with
carboxyhemoglobinemia or methemoglobinemia will have spuriously normal values because the pulse
oximeter cannot distinguish between oxyhemoglobin and these transformed hemoglobins. High altitude
affects the partial pressure of oxygen; a normal saturation at high altitude is 85%. The Table outlines other
factors leading to low or inaccurate pulse oxygen saturation.
Pulse oximetry does not measure the availability or tissue delivery of oxygen, particularly in conditions with
altered tissue delivery (eg, anemia, heart failure, and shock). Direct measurement of PaO2 in arterial blood is
the most reliable tool for determining oxygen available for tissue delivery. There is a direct but nonlinear
relationship between PaO2 and pulse oxygen saturation. The oxygen dissociation curve (ODC; Figure 1) is S-
shaped, with a steep linear rise in saturation between 20 mm Hg and 75 mm Hg oxygen PaO2 . Above 75 mm
Hg, the rise in saturation for each increment increase in PaO2 is much slower. A pulse oximetry reading (pulse
oxygen saturation) of <93% predicts PaO2 <70 mm Hg. Various factors can shift the oxygen dissociation curve
to the right or to the left. The ODC is shifted to the right by increased hydrogen ion (H+) concentration
(acidosis), increased 2,3-diphosphoglycerate (DPG), increased temperature (T°), increased partial pressure of
carbon dioxide (Pco2), or the presence of hemoglobin S (HbS) (sickle cell disease). Decreased H+ (alkalosis),
DPG, T°, Pco2 or the presence of fetal hemoglobin (HbF) shift the curve to the left (upper dotted line). These
factors affect the avidity of the hemoglobin for oxygen and its availability to the tissues (Figure 2).
Hemoglobin alterations (eg, fetal hemoglobin and sickle cell hemoglobin) can also affect affinity of oxygen for
hemoglobin and, therefore, tissue oxygen delivery.
Suggested Reading(s)
Martin GR, Ewer AK, Gaviglio A, et al. Updated strategies for pulse oximetry screening for critical
congenital heart disease. Pediatrics. 2020;146(1):e20191650. doi:10.1542/peds.2019-1650
Nasr VG, DiNardo JA. Pulse oximetry. Pediatr Rev. 2019;40(11):605-608. doi:10.1542/pir.2018-0123
Olsen J, Puri K. Interpretation of oxygen saturation in congenital heart disease: fact and fallacy. Pediatr
Rev. 2022;43(8):436-446. doi:10.1542/pir.2020-005364
Content Domain
Emergency Medicine
Reprinted with permission from Nasr VG, DiNardo JA. Pulse oximetry. Pediatr Rev.
2019;40(11):607.
PREPSA2024_C.indd 212
Reprinted with permission from Transport of oxygen and carbon dioxide in blood and tissue
fluids. Hall JE, Hall ME (eds). Guytan and Hall Textbook of Medical Physiology. 2021. Elsevier. Chap
521. P 524
FIGURE 1: Oxygen dissociation curve.
In the emergency department, the girl has a core temperature of 28 °C, a respiratory rate of 45 breaths/min,
a blood pressure of 85/40 mm Hg, and an oxygen saturation of 85% on 100% FiO2 via non-rebreather face
mask oxygen. She has a Glascow Coma Score of 8. The girl is intubated and admitted to the pediatric
intensive care unit for further treatment.
Of the following, the factor MOST predictive of a good neurologic outcome for this girl is
Correct answer is C
PREP Pearl(s)
Drowning is the most preventable cause of accidental death and injury in children and is the leading
cause of accidental death in children aged 1 to 4 years.
The strongest predictor of a good outcome after drowning is a shorter duration of submersion. Age,
water temperature (other than ice water), and witnessed versus unwitnessed drownings have not been
shown to have an effect on overall outcomes.
Terminology to describe drowning includes fatal or nonfatal drownings with or without morbidity.
Terms such as “near-drowning,” “dry/wet drowning,” and “secondary drowning” or “delayed drowning”
are no longer used.
Critique
The girl in the vignette experienced a drowning event. The factor most predictive of a good neurologic
outcome is her short duration of submersion (less than 5 min). Age, freshwater versus saltwater submersion,
water temperature (other than ice water), and witnessed versus unwitnessed drownings have not been
associated with a positive effect on survival or difference in likelihood of neurologic injury after drowning.
Conversely, failure of return of spontaneous circulation before arrival at the emergency department is
associated with worse outcomes.
There are more than 500,000 drowning events and 1,100 deaths due to drowning in the United States
annually. Drowning is the leading cause of accidental death in children aged 1 to 4 years, and it is second only
to motor vehicle crashes in children younger than age 14 years. Most drownings in young children occur in
swimming pools and often when children are not expected to be near water (eg, unplanned or unsupervised
access to home pools). Compared to coastal cities, inland drowning often occurs in pools or small bodies of
water (eg, ponds, bathtubs).
Drowning begins when the airway falls below the water’s surface. In conscious victims, voluntary breath
holding ensues, but the average maximal breath holding duration is 45 to 60 seconds in healthy adults and
even less time in children. Coughing and laryngospasm occur when small amounts of water enter the airway.
As water penetrates to the alveoli, the child is unable to breathe and gas exchange is compromised, leading
to hypoxia and hypercarbia. Arterial oxygenation declines with as little as 1 to 2 mL/kg of aspirated water’s
entering the lungs. With further decrease in arterial oxygen tension, laryngospasm abates and full aspiration
occurs. Cerebral hypoxemia (as opposed to hypercarbia) induces a loss of consciousness. With water in the
alveoli preventing gas exchange, asphyxia ensues with near-immediate cardiac arrest thereafter. The entire
drowning process from immersion to arrest occurs within seconds to minutes in children.
All drowning events in children should be considered avoidable. Anticipatory guidance and public policy
endeavors should focus on reducing drowning as a public health priority. The American Academy of
Pediatrics endorses a multilayered age- and developmentally targeted safety campaign that provides
anticipatory guidance regarding water safety (Table).
Suggested Reading(s)
McCallin TE, Morgan M, Hart ML, Yusuf S. Epidemiology, prevention, and sequelae of drowning. Pediatr
Rev. 2021;42(3):123-132. doi:10.1542/pir.2019-0150
Quan L, Mack CD, Schiff MA. Association of water temperature and submersion duration and drowning
outcome. Resuscitation. 2014;85(6):790-794. doi:10.1016/j.resuscitation.2014.02.024
Semple-Hess J, Campwala R. Pediatric submersion injuries: emergency care and resuscitation. Pediatr
Emerg Med Pract. 2014;11(6):1-21; quiz 21-2. https://pubmed.ncbi.nlm.nih.gov/25090739/
Suominen P, Baillie C, Korpela R, et al. Impact of age, submersion time and water temperature on
outcome in near-drowning. Resuscitation. 2002;52(3):247. doi:10.1016/s0300-9572(01)00478-6
Szpilman D, Bierens JJ, Handley AJ, Orlowski JP. Drowning. N Engl J Med 2012;366(22):2102-2110.
doi:10.1056/NEJMra1013317
Content Domain
Emergency Medicine
6RXUFH$PHULFDQ$FDGHP\RI3HGLDWULFV1HZV5HOHDVH0D\
AAP PREP 2024 - Question 48/267 Emergency Medicine Question 14/14
A 3-year-old girl is brought to the emergency department after being found with an open and empty bottle of
prenatal gummy vitamins that contained ferrous fumarate. Her mother had purchased the bottle of 120
gummies yesterday; each gummy contained 10 mg of ferrous fumarate. The child is vomiting and
experiencing diffuse abdominal pain. Her temperature is 37.1 °C, heart rate is 165 beats/min, respiratory rate
is 28 breaths/min, blood pressure is 98/63 mm Hg, and oxygen saturation is 97% in room air. Her weight is 13
kg. The remainder of her physical examination findings are unremarkable.
A. deferoxamine
B. dimercaprol
C. flumazenil
D. N-acetylcysteine
Correct answer is A
PREP Pearl(s)
Intravenous deferoxamine is the antidote for iron toxicity (chelation).
Treatment of symptomatic iron toxicity is focused on gastrointestinal decontamination and increased
excretion of iron through chelation with deferoxamine.
Indications for initiating deferoxamine therapy (chelation) for iron toxicity include intractable vomiting,
lethargy or altered mental status, hypotension, metabolic acidosis, toxic appearance, organ failure, or
a serum iron level greater than 500 µg/dL (89.50 µmol/L).
Critique
The child in the vignette has ingested a toxic amount of iron and should be treated with intravenous
deferoxamine, the antidote for iron toxicity (chelation). Dimercaprol is used as part of chelation therapy for
lead poisoning. Flumazenil is the antidote for benzodiazepine toxicity. N-acetylcysteine is the antidote for
acetaminophen toxicity.
Of all the iron salt preparations, ferrous fumarate has the highest elemental iron composition (33%), followed
by ferrous chloride (28%), ferrous sulfate (20%), and ferrous gluconate (12%). The severity of toxic effects is
related to the amount of elemental iron ingested:
Mild: 10 to 20 mg/kg
Moderate: 40 mg/kg
Severe: greater than 60 mg/kg
Although serum iron levels can be useful to help guide management, toxicity from iron overdose is a clinical
diagnosis. Serum iron levels peak 2 to 6 hours after ingestion; monitoring of serum iron levels may be
necessary over this time range. An iron level greater than 500 µg/dL (89.50 µmol/L) is an indication for
chelation therapy with deferoxamine. The total iron-binding capacity is falsely elevated in the setting of iron
poisoning and therefore should not be used in determining the risk of iron toxicity.
The treatment of symptomatic iron toxicity is focused on gastrointestinal decontamination and increased
excretion of iron through chelation with deferoxamine. Activated charcoal is not effective in binding iron salts
and should not be used. To decrease iron absorption, gastrointestinal decontamination with early and
aggressive whole bowel irrigation (WBI) should be performed until a clear rectal effluent is obtained.
Contraindications to WBI include shock, gastrointestinal hemorrhage, ileus, or bowel obstruction. In the case
of iron ingested in tablet form, abdominal radiography should be performed to look for pill fragments; the
absence of pill fragments (as may be the case if the ingested form is liquid or gummy) does not exclude toxic
ingestion.
Chelation therapy with intravenous deferoxamine enhances urinary excretion of iron and produces the
characteristic vin rose urine (pink, orange, or red urine). Indications for initiating deferoxamine therapy
include intractable vomiting, lethargy or altered mental status, hypotension, metabolic acidosis, toxic
appearance, organ failure, or a serum iron level greater than 500 µg/dL (89.50 µmol/L).
Suggested Reading(s)
Fine JS. Poisoning. In: McInerny TK, Adam HM, Campbell DE, et al. American Academy of Pediatrics
Textbook of Pediatric Care. 2nd ed. American Academy of Pediatrics; 2016:chap 369. Pediatric Care
Online
Mangus CW, Canares TL. Toxic ingestions: initial management. Pediatr Rev. 2018;39(4):219-221.
doi:10.1542/pir.2017-0119
Content Domain
Emergency medicine
Timeline after ingestion 1-6 hours 6-24 hours ≥24 hours 4-6 weeks
Courtesy of T. Swan
AAP PREP 2024 - Question 49/267 Endocrinology Question 1/13
A 7-day-old neonate is seen in the office after newborn screening obtained 3 days after birth showed a
thyroid-stimulating hormone level of 300 µIU/mL. The family reports no feeding difficulties or constipation.
On physical examination, the neonate appears vigorous and has regained his birth weight. He has mild facial
jaundice and scleral icterus; there is no hepatomegaly. The remainder of the neonate’s physical examination
findings are unremarkable. Confirmatory laboratory evaluation shows a thyroid-stimulating hormone level of
420 µIU/mL and free thyroxine level of 0.2 ng/dL (reference range, 0.85-1.75).
Of the following, the BEST next step in this neonate’s management is to initiate treatment with
A. desiccated thyroid
B. iodine
C. levothyroxine
D. liothyronine
Correct answer is C
PREP Pearl(s)
An abnormal newborn screening test finding for congenital hypothyroidism should be confirmed via
serum thyroid function testing.
Rapid initiation of levothyroxine treatment in cases of confirmed congenital hypothyroidism results in
normal growth and development of the affected neonate.
Hypothyroidism is not a common cause of weight gain in children. Thyroid function should not be
routinely checked unless other signs or symptoms of hypothyroidism are present.
Critique
The neonate in the vignette has primary congenital hypothyroidism. The best next step in management is to
initiate levothyroxine (LT4) therapy.
Thyroid hormone is critically important for brain development in infants and children up to age 2 years.
Thyroxine (T4) is the major circulating form of thyroid hormone. At target tissues, T4 undergoes deiodination
to triiodothyronine (T3), which is the major active form of thyroid hormone. Because T4-to-T3 conversion is
the major source of thyroid hormone in the brain, levothyroxine (LT4) is the treatment of choice for
congenital hypothyroidism. Levothyroxine is administered once daily. Neonates treated before 14 days after
birth have normal growth and development. Untreated infants with congenital hypothyroidism have
significant risk of severe developmental delays and poor growth.
Desiccated thyroid is produced from porcine or bovine thyroid glands and contains a mixture of T4 and T3.
Some preparations may also contain inactive forms of thyroid hormone (eg, T1, T2, reverse T3). Owing to
variability in the content of preparations, it is not recommended to treat children, especially neonates and
infants, with desiccated thyroid.
Although iodine deficiency may be a cause of congenital hypothyroidism, it is rare in the United States. Iodine
is not a standard treatment for congenital hypothyroidism.
Liothyronine (T3) is not recommended for treatment of infants and children with hypothyroidism. It is
shorter-acting than LT4 and must be administered multiple times per day. Additionally, it offers no benefit
over LT4, given that T4 is so efficiently converted to T3 in the brain. Liothyronine is used after thyroidectomy
for thyroid cancer while the patient is awaiting radioactive iodine therapy (because of its short half-life it is
cleared quickly, a requirement before radioiodine therapy). It also is used in rare cases of enzymatic defects
that affect T4-to-T3 conversion.
Newborn screening programs are designed to detect medical conditions for which early treatment may
drastically improve the prognosis of an affected neonate. Every state screens for congenital hypothyroidism,
but the analyte tested varies. Some states measure thyroid stimulating hormone (TSH), which, if elevated,
suggests primary hypothyroidism (a problem with the thyroid gland itself). Secondary and tertiary congenital
hypothyroidism (originating from the pituitary or hypothalamus, respectively) are not detected on a TSH-
primary screen because the TSH level will not be elevated. Other states measure total thyroxine (T4), which is
low in all forms of hypothyroidism. Neonates with thyroid-binding globulin deficiency will have an abnormal
T4 screening result even though their thyroid function is normal (the total T4 level is low due to low binding
protein levels, but the free T4 level is normal).
If the newborn screening test is obtained too soon after birth, all thyroid function values may be elevated
owing to a normal surge of thyroid hormones. Ideally, newborn screening for congenital hypothyroidism
should be obtained between days 3 to 5 after birth for healthy infants. However, this timing is often
impractical because screening must be performed before hospital discharge by law. Owing to this limitation,
all abnormal thyroid newborn screening tests should be followed up with serum TSH and free T4 testing.
Worldwide, the most common cause of congenital hypothyroidism is iodine deficiency. In the United States,
most cases of congenital hypothyroidism are sporadic and caused by a developmental abnormality of the
thyroid gland (dysgenesis). In a smaller number of neonates, an enzymatic defect in thyroid hormone
production is the cause (dyshormonogenesis). Most of these defects are inherited in an autosomal recessive
manner. Rarely, maternal autoimmune thyroid disease results in transient hypothyroidism that resolves as
maternal autoantibodies are eliminated. Central congenital hypothyroidism is usually caused by congenital
hypothalamic or pituitary malformations.
In the short term, most neonates with congenital hypothyroidism have no abnormal physical examination
findings. Signs suggestive of hypothyroidism include jaundice with direct hyperbilirubinemia, enlarged
anterior fontanelle, umbilical hernia, or goiter.
Acquired hypothyroidism occurs in older children and adolescents and is most commonly caused by
autoimmune thyroid disease (chronic lymphocytic thyroiditis). This inflammatory condition is characterized
by a painless thyroiditis and often a goiter. Signs and symptoms of hypothyroidism in this age group may
include poor growth, fatigue, constipation, cold intolerance, and irregular menses. Hypothyroidism is not a
significant contributor to weight gain in children. Sometimes excess weight will be associated with mild
elevations of TSH, and these levels usually normalize with weight reduction. Children with excess weight gain
or constipation should not be routinely screened for thyroid dysfunction. Thyroid function should be
measured only when other signs or symptoms of thyroid disease are present, especially if there is poor
growth. There is no treatment for the autoimmune component of chronic lymphocytic thyroiditis; the
resulting hypothyroidism is treated with LT4.
Suggested Reading(s)
Alarcon G, Figueredo V, Tarkoff J. Thyroid disorders. Pediatr Rev. 2021;42(11):604-618.
doi:10.1542/pir.2020-001420
Marras V, Casini MR, Pilia S, et al. Thyroid function in obese children and adolescents. Horm Res
Paediatr. 2010;73(3):193-197. doi:10.1159/000284361
Orlowski CC. Hypothyroidism. In: McInerny TK, Adam HM, Campbell DE, et al, eds. American Academy
of Pediatrics Textbook of Pediatric Care. American Academy of Pediatrics. 2023. Accessed September
1, 2023. Pediatric Care Online
Rose SR, Wassner AJ, Wintergerst KA, et al; Section on Endocrinology Executive Committee; Council on
Genetics Executive Committee. Congenital hypothyroidism: screening and management. Pediatrics
2023;151(1):e2022060419. doi:10.1542/peds.2022-060419
Rose SR, Wassner AJ, Wintergerst KA, et al; Section on Endocrinology Executive Committee; Council on
Genetics Executive Committee. Congenital hypothyroidism: screening and management. Pediatrics
2023;151(1):e2022060420. doi:10.1542/peds.2022-060420
Content Domain
Endocrinology
A 2-year-old girl is evaluated in the emergency department for rapid breathing that has lasted for 2 days. She
was evaluated yesterday at a different facility and diagnosed with an upper respiratory infection. Her mother
states that over the past week, the girl has gone through an entire box of diapers owing to frequent
urination.
On physical examination, she has a temperature of 37.1°C, a heart rate of 135 beats/min, a blood pressure of
85/58 mm Hg, a respiratory rate of 45 breaths/min, and an oxygen saturation of 99% in room air. Her weight
is 10.2 kg, which is 2.3 kg less than at a health supervision visit 1 month ago. She appears tired and is taking
rapid, deep breaths. Her mucous membranes appear dry. The remainder of her physical examination
findings are unremarkable.
Of the following, the BEST test(s) to reveal this girl’s diagnosis is (are)
B. chest radiography
Correct answer is A
PREP Pearl(s)
Type 1 diabetes should be considered as a diagnosis in a child with rapid breathing and increased
urine output, especially without other signs of infection.
Young children with new-onset type 1 diabetes frequently experience diabetic ketoacidosis.
Rapid recognition of type 1 diabetes is essential for prompt treatment and prevention of adverse
outcomes.
Critique
The girl in the vignette’s signs and symptoms of polyuria, weight loss, and dehydration are suggestive of new-
onset type 1 diabetes mellitus. Her rapid breathing, a manifestation of respiratory compensation for
metabolic acidosis, makes diabetic ketoacidosis (DKA) likely. Of the response choices, the best test to reveal
this girl’s diagnosis is a basic metabolic panel.
New-onset type 1 diabetes manifests in children in a variety of ways, often with DKA. This life-threatening
condition is defined by elevated blood glucose (>200 mg/dL [11.10 mmol/L]), ketosis, and metabolic acidosis
(serum bicarbonate <15 mEq/L [15 mmol/L] or venous pH <7.3) with a widened anion gap. Owing to insulin
deficiency, the body cannot use glucose for energy production. When the serum glucose level exceeds the
renal threshold (~180 mg/dL [9.99 mmol/L]), more than the renal tubules can reabsorb, glucose is excreted in
the urine, causing osmotic diuresis and polyuria. The increased urine output causes dehydration and
compensatory polydipsia. To provide the body with an alternative energy source, lipolysis increases and
results in production of ketone bodies, predominantly β-hydroxybutyrate. The presence of ketoacids, along
with dehydration, results in metabolic acidosis. To compensate for the metabolic acidosis, children with DKA
may breathe more rapidly and deeply (Kussmaul respirations) to blow off CO 2. In severe cases of DKA, the
increased ketone level results in abdominal pain and vomiting, which worsens the dehydration and acidosis.
The overall energy deficit also results in weight loss and, in some cases, a compensatory polyphagia.
New-onset type 1 diabetes can masquerade as common illnesses. It is essential to recognize diabetes to
prevent delayed diagnosis, DKA, and adverse outcomes. Although the girl in the vignette appears to have
respiratory distress, she does not have other signs or symptoms of infection. Her history and physical
examination findings are most consistent with diabetes. Neither chest radiography nor a respiratory viral
panel would reveal this diagnosis. The girl’s increased urination is due to hyperglycemia. She does not appear
to have dysuria and is afebrile. A urinalysis may reveal glucosuria and ketonuria, but the basic metabolic
panel would quantify the hyperglycemia and acidosis. A urine culture would not reveal her diagnosis.
Suggested Reading(s)
ElSayed NA, Aleppo G, Aroda VR, et al; on behalf of the American Diabetes Association. 14. Children
and adolescents: standards of care in diabetes—2023. Diabetes Care. 2023;46(suppl 1):S230-S253.
doi:10.2337/dc23-S014
Cashen K, Petersen T. Diabetic ketoacidosis. Pediatr Rev. 2019;40(8):412-420. doi:10.1542/pir.2018-
0231
Jackson S, Creo A, Al Nofal A. Management of type 1 diabetes in children in the outpatient setting.
Pediatr Rev. 2022;43(3):160-170. doi:10.1542/pir.2020-001388
Javed A, Schwenk WF II, Tebben P, 2016. Diabetes mellitus. In: McInerny TK, Adam HM, Campbell DE,
Foy JM, Kamat DM, eds. American Academy of Pediatrics Textbook of Pediatric Care. 2nd ed. American
Academy of Pediatrics; 2023. Accessed September 1, 2023. Pediatric Care Online
Content Domain
Endocrinology
A 12-year-old girl is seen for evaluation of absent menstrual periods. Her mother had menarche at age 11
years and her older sister at age 12 years. The girl has no relevant medical history, and results of a review of
systems are unremarkable. She is a dancer and practices 5 days per week. Her mother reports that she has a
good appetite. The girl’s vital signs are normal for age. Her growth curves are shown in Figure 1 and Figure 2.
Her sexual maturity rating is 1 for breast development and pubic hair. The remainder of the girl’s physical
examination findings are unremarkable.
A. anorexia nervosa
C. Mayer-Rokitansky-Küster-Hauser syndrome
D. Turner syndrome
Correct answer is B
PREP Pearl(s)
The first sign of puberty is breast development in girls and testicular growth in boys.
Normal pubertal onset occurs between ages 8 to 13 years in girls and 9 to 14 years in boys.
The most common cause of delayed puberty is constitutional delay of growth and development.
Critique
The girl in the vignette has no signs of puberty. She has short stature compared with her peers and a low
body mass index. She is otherwise healthy, is very physically active, and has normal physical examination
findings and vital signs. At age 12 years, she does not yet have abnormally delayed puberty. The most likely
cause of her lack of pubertal development and short stature is constitutional delay of growth and
development, a normal variant of growth and puberty.
Puberty is defined as the appearance of breast tissue in girls or testicular growth in boys. The normal range
for puberty onset is between ages 8 and 13 years for girls and ages 9 and 14 years for boys. True puberty
should be distinguished from isolated pubic or axillary hair growth (premature adrenarche). Adolescents who
do not show any signs of pubertal development by age 13 years (girls) or 14 years (boys) should undergo
evaluation for primary (gonadal) or secondary (pituitary or hypothalamic) causes of pubertal failure. This
evaluation may include measurement of concentrations of gonadotropins (luteinizing hormone, follicle-
stimulating hormone) and sex steroids (testosterone for boys, estradiol for girls), which normally rise during
puberty. Pubertal development is affected by genetic factors, general health status, and body weight.
Adolescents who have low weight will often have delayed puberty.
Anorexia nervosa can result in delayed puberty owing to low body weight and overall energy deficiency.
Anorexia nervosa is often accompanied by abnormal vital signs (hypothermia, bradycardia), physical signs
such as lanugo, and laboratory indicators of malnutrition. The girl in the vignette is healthy, with no
concerning findings other than her lack of puberty. Additionally, there is not a concern about restrictive
eating or excess exercise.
Mayer-Rokitansy-Küster-Hauser syndrome is characterized by lack of development of the Müllerian
structures (uterus, fallopian tubes, proximal vagina). This syndrome typically manifests in affected girls during
adolescence with primary amenorrhea. The hormone changes of puberty, and therefore secondary sex
characteristics (except menarche), are normal. The girl in the vignette has no signs of puberty, so Mayer-
Rokitansy-Küster-Hauser syndrome is not the most likely diagnosis.
Girls with Turner syndrome (45,X karyotype or variant) typically have abnormal puberty caused by primary
ovarian failure. Classically, there are physical findings that suggest the diagnosis (eg, significant short stature,
increased carrying angle of the elbows, wide-spaced nipples, low-lying hairline). The girl in the vignette does
not have any features suggestive of Turner syndrome.
Suggested Reading(s)
Bakhtiani P, Geffner M. Delayed puberty. Pediatr Rev. 2022;43(8):426-435. doi:10.1542/pir.2020-005291
Harrington J, Palmert MR. An approach to the patient with delayed puberty. J Clin Endocrinol Metab.
2022;107(6):1739-1750. doi:10.1210/clinem/dgac054
Kritzler RK, Long D, Plotnick L. Puberty: normal and abnormal. In: McInerny TK, Adam HM, Campbell
DE, Foy JM, Kamat DM, eds. American Academy of Pediatrics Textbook of Pediatric Care. 2nd ed.
American Academy of Pediatrics; 2023. Accessed September 1, 2023. Pediatric Care Online
Wolf RM, Long D. Pubertal development. Pediatr Rev. 2016;37(7):292-300. doi:10.1542/pir.2015-0065
Content Domain
Endocrinology
12 13 14 15 16 17 18 19 20
Mother’s Stature Father’s Stature cm in
Date Age Weight Stature BMI*
AGE (YEARS) 76
190
74
185 S
72
180 T
70 A
95
175 T
90
68 U
170 R
75 66
165 E
in cm 3 4 5 6 7 8 9 10 11 50
64
160 25 160
62 62
155 10 155
60 5 60
150 150
58
145
56
140 105 230
54
S 135 100 220
T 52
A 130 95 210
50
T 125 90 200
U
48 190
R 120 85
E 95 180
46
115 80
44 170
110 90 75
42 160
105 70
150 W
40 75
100 65 140 E
38 I
95 60 130 G
50
36 90 H
55 120
25 T
34 85 50 110
10
32 80
5
45 100
30
40 90
80 35 35 80
W 70 70
30 30
E 60 60
I 25 25
G 50 50
H 20 20
40 40
T
15 15
30 30
10 10
lb kg AGE (YEARS) kg lb
2 3 4 5 6 7 8 9 10 11 12 13 14 15 16 17 18 19 20
Published May 30, 2000 (modified 11/21/00).
SOURCE: Developed by the National Center for Health Statistics in collaboration with
the National Center for Chronic Disease Prevention and Health Promotion (2000).
http://www.cdc.gov/growthcharts
Courtesy of E. Graber
Figure 2. Body mass index chart for the girl in the vignette.
35
34
33
32
95
31
30
29
BMI 28
90
27 27
26 85 26
25 25
24 75 24
23 23
22 22
50
21 21
20 20
25
19 19
10
18 5
18
17 17
16 16
15 15
14 14
13 13
12 12
2 2
kg/m AGE (YEARS) kg/m
2 3 4 5 6 7 8 9 10 11 12 13 14 15 16 17 18 19 20
Courtesy of E. Graber
AAP PREP 2024 - Question 52/267 Endocrinology Question 4/13
A 16-year-old patient is being evaluated in the office for a 3-week history of fatigue, daily abdominal pain, and
nausea without vomiting. They completed craniospinal radiation therapy for medulloblastoma 1 year ago
and have been taking levothyroxine for the past 6 months for central hypothyroidism. Their temperature is
37.2 °C, heart rate is 102 beats/min, blood pressure is 86/56 mm Hg, and oxygen saturation is 99% in room
air. They appear tired. The remainder of the physical examination findings are unremarkable.
A laboratory evaluation was obtained at 8 AM. Results are shown:
C. start desmopressin
D. start hydrocortisone
Correct answer is D
PREP Pearl(s)
Cortisol is important for normal free-water excretion from the kidney; isolated cortisol deficiency due
to adrenocorticotropic hormone deficiency can result in hyponatremia.
To ensure proper treatment, adrenal insufficiency must be distinguished from the syndrome of
inappropriate antidiuretic hormone secretion when hyponatremia is present.
Individuals who have received treatment for central nervous system tumors should be monitored over
the long term for pituitary hormone deficiencies.
Critique
The patient in the vignette has secondary adrenal insufficiency (adrenocorticotropic hormone [ACTH]
deficiency) due to craniospinal radiation. The signs and symptoms of adrenal insufficiency include fatigue,
abdominal pain, nausea, hypotension, tachycardia, hyponatremia, and a low morning cortisol level. The
treatment for all types of adrenal insufficiency is glucocorticoid replacement. Therefore, the best next step in
this patient’s care is to start hydrocortisone.
Primary adrenal insufficiency is caused by a problem with the adrenal gland itself (eg, Addison disease,
congenital adrenal hyperplasia) and results in not only glucocorticoid deficiency, but also mineralocorticoid
deficiency. Mineralocorticoid deficiency results in laboratory findings associated with salt wasting (low
sodium level, high potassium level).
Secondary and tertiary causes of adrenal insufficiency (eg, hypopituitarism, withdrawal from exogenous
steroid) result in glucocorticoid deficiency, but mineralocorticoid production is minimally affected, because
the renin-angiotensin system is the primary regulator of aldosterone secretion. However, cortisol is
necessary for renal free-water excretion. Cortisol deficiency results in decreased free-water excretion, which
can cause hyponatremia without hyperkalemia. The cause of hyponatremia may be confused with, and must
be distinguished from, the syndrome of inappropriate antidiuretic hormone secretion (SIADH).
The syndrome of inappropriate antidiuretic hormone secretion is associated with certain medications (eg,
vincristine, carbamazepine) and with a triphasic response (first diabetes insipidus [DI], then SIADH, then DI
again) after neurosurgery or head trauma. Increased antidiuretic hormone levels result in increased renal
water reabsorption, decreased serum osmolality, and low serum sodium levels. The treatment of SIADH is
water restriction and treatment of the underlying cause, if possible. The adolescent in the vignette was
treated with cranial radiation a year before presentation. Although SIADH might have been seen soon after
treatment, DI is more likely to occur as a late effect of radiation therapy owing to destruction of the posterior
pituitary gland. Therefore, fluid restriction is not an appropriate therapy for the patient in the vignette.
High-dose cranial radiation therapy can result in multiple pituitary hormone deficiencies. The adolescent in
the vignette has known thyroid-stimulating hormone deficiency (central hypothyroidism) and is receiving
levothyroxine therapy. Hypothyroidism can also result in fluid retention and hyponatremia. However, the
patient in the vignette has a normal free thyroxine level, so an adjustment to the levothyroxine dose is not
indicated.
Desmopressin is a treatment for vasopressin deficiency caused by central diabetes insipidus. Children and
adolescents with DI have increased urine output, extreme thirst, and elevated serum sodium levels. Serum
osmolality increases and urine osmolality is inappropriately low owing to an inability to concentrate urine
without vasopressin stimulation. The patient in the vignette does not have evidence of DI, so treatment with
desmopressin is not recommended.
Suggested Reading(s)
Auron M, Raissouni N. Adrenal insufficiency. Pediatr Rev 2015;36(3):92-103. doi:10.1542/pir.36-3-92
Jones DP. Syndrome of inappropriate secretion of antidiuretic hormone and hyponatremia. Pediatr
Rev. 2018;39(1):27-35. doi:10.1542/pir.2016-0165
Garrahy A, Thompson CJ. Hyponatremia and glucocorticoid deficiency. In: Peri A, Thompson CJ, Verbalis
JG, eds. Disorders of Fluid and Electrolyte Metabolism: Focus on Hyponatremia. Karger; 2019:80-92.
Frontiers of Hormone Research, vol 52.
Speiser PW. Adrenal dysfunction. In: McInerny TK, Adam HM, Campbell DE, DeWitt TG, Foy JM, Kamat
DM, eds. American Academy of Pediatrics Textbook of Pediatric Care. American Academy of Pediatrics;
2021:chap 211. Accessed September 1, 2023. Pediatric Care Online
Content Domain
Endocrinology
A 3-day-old neonate is being evaluated for atypical genitalia. The neonate is feeding well and has regained
birth weight. Physical examination findings are notable for a 1-cm phallic structure, bifid labioscrotal folds,
and palpable gonads in the inguinal canals bilaterally. The remainder of the newborn’s examination findings
are normal.
Pelvic ultrasonography shows gonads in both inguinal canals with features consistent with seminiferous
tubules. A uterus is not definitively identified.
Luteinizing hormone 0.8 mIU/mL (0.8 IU/L) (reference range, 0.02-7 mIU/mL [0.02-7.0 IU/L])
Anti-Müllerian 202 ng/mL (1,442.3 pmol/L) (reference range, birth-6 years: males, 32.77-262.69
hormone ng/mL [234-1875.6 pmol/L]; females, 0.53-7.78 ng/mL [3.78-55.5 pmol/L])
82 ng/dL (2.85 nmol/L) (reference range: males, 75-400 ng/dL [2.6-13.88 nmol/L];
Testosterone
females, 20-64 ng/dL [0.69-2.22 nmol/L])
20 ng/dL (0.69 nmol/L) (reference range: males 5-60 ng/dL [0.17-2.07 nmol/L],
Dihydrotestosterone
females <2-15 ng/dL [<0.07-0.52 nmol/L])
Estradiol <15 pg/mL (4.08 pmol/L) (reference range: <15 pg/mL [4.08 pmol/L])
17-
55 ng/dL (1.66 nmol/L) (reference range: <78 ng/dL [<2.36 nmol/L])
hydroxyprogesterone
Karyotype 46,XY
Correct answer is D
PREP Pearl(s)
Genital development is a complex process that requires intact genetic and hormonal factors, as well as
hormone receptor responsiveness.
Androgen insensitivity syndrome may present in a 46,XY neonate with phenotypic female or atypical
genitalia, palpable testes, and absence of internal female genital structures.
Congenital adrenal hyperplasia due to 21-hydroxylase deficiency is the most common cause of
virilization of a female (46,XX) neonate.
Critique
The neonate in the vignette has atypical genitalia, a 46,XY karyotype, and a normal male hormonal profile for
age. Imaging reveals gonads that are consistent with testes and no discernible internal female structures.
These findings are most consistent with a 46,XY difference of sex development and undervirilization. Of the
response choices, this infant’s most likely diagnosis is partial androgen insensitivity syndrome.
Differences of sex development are conditions in which the genitals do not form normally, which may or may
not result in atypical-appearing genitals at birth. Sexual differentiation begins at the chromosomal level. The
SRY gene on the Y chromosome plays a major role in sex development. A normally functioning SRY gene
results in differentiation of the bipotential gonads into testes, while absence (or nonfunction) of the SRY gene
results in ovary development. By 12 weeks of gestation, the developing fetus is “committed” to a genital path.
Androgen insensitivity syndrome is an X-linked recessive condition that affects the androgen receptor’s ability
to bind and respond to androgens (eg, T). Individuals with complete androgen insensitivity syndrome do not
respond to androgen. Affected 46,XY neonates are born with phenotypic female external genitalia.
Individuals with partial androgen insensitivity syndrome exhibit partial responsiveness to androgen, and
affected 46,XY neonates have varying degrees of incomplete virilization of the external genitalia. The testes of
46,XY individuals with complete or partial androgen insensitivity syndrome produce normal amounts of T and
anti-Müllerian hormone. Normal anti-Müllerian hormone values result in lack of development of the
Müllerian (internal female) structures.
The enzyme 5-ɑ reductase catalyzes the conversion of T to DHT. Genetically male (46,XY) neonates are
undervirilized when there is a lack of DHT present in the prenatal period. When T values rise during
adolescence, affected individuals become more virilized. An increased ratio of T:DHT during times of gonadal
stimulation (either artificially or during puberty) is consistent with 5-ɑ reductase deficiency. Genetic testing
provides a definitive diagnosis. The 46,XY neonate in the vignette is undervirilized, but the normal values of T
and DHT are not consistent with 5-ɑ reductase deficiency.
Mixed gonadal dysgenesis is associated with a mosaic 45,X/46,XY karyotype. It is characterized by gonadal
asymmetry, with a unilateral dysgenetic streak gonad on one side and a functioning testis or ovotestis on the
contralateral side. The neonate in the vignette has a 46,XY karyotype and bilateral palpable gonads with
sonographic features of testes, making mixed gonadal dysgenesis an unlikely diagnosis.
All differences of sex development with testicular tissue located in the abdomen or pelvis (eg, complete or
partial androgen insensitivity syndrome, complete or mixed gonadal dysgenesis, Turner syndrome with Y
chromosomal material present) are associated with an increased risk of gonadoblastoma development. In
these cases, gonadectomy is recommended. However, the optimal timing of gonadectomy is unclear, and
gonadectomy is often performed according to the risk of developing malignancy by adolescence versus
adulthood.
Congenital adrenal hyperplasia (CAH) due to 21-hydroxylase deficiency is the most common cause of
virilization of a female (46,XX) neonate. All states screen for this form of CAH as part of the newborn
screening program. Excess androgen results in variable degrees of virilization of female (46,XX) neonates
depending on the degree of impairment in enzyme activity. Elevated values of the steroid precursor 17-
hydroxyprogesterone is the hallmark biochemical finding in CAH due to 21-hydroxylase deficiency. Nonclassic
CAH is caused by mild impairment of 21-hydroxylase; it may result in premature adrenarche in both males
(46,XY) and females (46,XX) and may mimic polycystic ovarian syndrome in females. It does not cause atypical
genitalia in males or females. Although other (rare) forms of CAH can cause undervirilization of a male (46,XY)
neonate, androgen (including T) values would be low in such cases, making CAH an unlikely diagnosis for the
neonate in the vignette.
Suggested Reading(s)
Diaz A, Diaz EGL. Disorders of sex development. Pediatr Rev. 2021;42(8):414-426. doi:10.1542/pir.2018-
0183
León NY, Reyes AP, Harley VR. A clinical algorithm to diagnose differences of sex development. Lancet
Diabetes Endocrinol. 2019;7(7):560-574. doi:10.1016/S2213-8587(18)30339-5
Loomba-Albrecht LA, Styne DM. Disorders of sex development. In: McInerny TK, Adam HM, Campbell
DE, Foy JM, Kamat DM, eds. American Academy of Pediatrics Textbook of Pediatric Care. 2nd ed.
American Academy of Pediatrics; 2023. Accessed September 1, 2023. Pediatric Care Online
Content Domain
Endocrinology
A 16-year-old adolescent being seen for a routine health supervision reports a recent onset of increased
thirst and urination. The findings of a review of systems are otherwise unremarkable. The adolescent’s vital
signs reveal a blood pressure of 140/92 mm Hg, a heart rate of 85 beats/min, and an oxygen saturation of
100% in room air.
Physical examination findings are notable for a body mass index of 40 kg/m2, striae over the flanks, and dark
skin pigmentation over the neck with skin tags. The remainder of the physical examination findings are
normal.
Laboratory evaluation reveals the following:
Test Result
Of the following, the MOST appropriate initial treatment for this adolescent is
A. empagliflozin
B. insulin
C. liraglutide
D. metformin
Correct answer is D
PREP Pearl(s)
Metformin is the first-line pharmacologic treatment of type 2 diabetes for children and adolescents
who are metabolically stable and have a hemoglobin A1c level <8.5%. Lifestyle change is a critical
aspect of management.
Insulin is indicated for treatment of type 2 diabetes when the hemoglobin A1c is 8.5% or higher, serum
glucose is ≥250 mg/dL (13.9 mmol/L) with symptoms, or ketosis/ketoacidosis is present.
Long-term complications are occurring in young adults diagnosed with type 2 diabetes as adolescents.
Aggressive blood glucose management and treatment of comorbidities are necessary to reduce
complication rates.
Critique
The adolescent in the vignette meets diagnostic criteria for diabetes mellitus, with a random serum glucose
level ≥200 mg/dL (11.1 mmol/L), symptoms of diabetes (polyuria, polydipsia), and a hemoglobin A1c level
≥6.5%. Type 2 diabetes mellitus is the most likely diagnosis, given the adolescent’s severe obesity (body mass
index of 40 kg/m2), acanthosis nigricans (sign of insulin resistance), and hypertension (condition associated
with insulin resistance). There are no signs of acute metabolic decompensation that would indicate diabetic
ketoacidosis or hypoketotic hyperosmolar syndrome. The American Diabetes Association recommends
metformin as the first-line pharmacological treatment of type 2 diabetes for children and adolescents who
are metabolically stable and have a hemoglobin A1c level <8.5%. Lifestyle change is also critical in the
management of type 2 diabetes.
Insulin is indicated for youth with type 2 diabetes when the hemoglobin A1c is 8.5% or higher, serum glucose
is ≥250 mg/dL (13.9 mmol/L) with symptoms, or ketosis/ketoacidosis is present. In the absence of ketosis or
hypoketotic hyperosmolar syndrome, basal insulin alone is indicated at diagnosis. Prandial insulin can be
considered if the hemoglobin A1c goal is not met after optimizing other treatments.
Liraglutide is a glucagon-like peptide (GLP-1) agonist that was approved by the US Food and Drug
Administration (FDA) in 2019 for the treatment of type 2 diabetes in youth aged 10 years or older. Glucagon-
like peptide-1 is an incretin hormone secreted from the ileum and proximal colon that slows gastric
emptying, stimulates satiety centers in the brain, and enhances insulin secretion. Liraglutide is a second-line
treatment that can be added to metformin to reduce hemoglobin A1c levels and facilitate weight loss via its
effect on the brain appetite centers.
Empagliflozin is a sodium-glucose cotransporter-2 inhibitor that lowers the renal threshold for glucose
excretion in the kidney, resulting in lower serum glucose levels via glucosuria. This class of medication was
recently FDA-approved for the treatment of type 2 diabetes in adolescents, but it is not a first-line treatment.
Genital candidiasis is a common side effect, and euglycemic diabetic ketoacidosis has been reported.
The prevalence of type 2 diabetes mellitus in youth is rising. The disease is often aggressive in adolescents,
with one-half of affected adolescents requiring insulin therapy despite maximal lifestyle change and
additional pharmacologic therapies. Complications such as retinopathy, nephropathy, and neuropathy are
occurring in young adults diagnosed with type 2 diabetes as adolescents. Aggressive treatment of
comorbidities (eg, hypertension or hyperlipidemia) is recommended in addition to optimized blood glucose
control. Currently, the only pharmacologic treatments available are insulin, metformin, glucagon-like peptide-
1 agonists, and empagliflozin. Studies of additional pharmacologic treatment options for adolescents with
type 2 diabetes are ongoing.
Suggested Reading(s)
ElSayed NA, Aleppo G, Aroda VR, et al., on behalf of the American Diabetes Association. 14. Children
and adolescents: standards of care in diabetes—2023. Diabetes Care. 2023 Jan 1;46(Suppl 1):S230-
S253. doi:10.2337/dc23-S014
Arslanian S, Bacha F, Grey M, Marcus MD, White NH, Zeitler P. Evaluation and management of youth-
onset type 2 diabetes: a position statement by the American Diabetes Association. Diabetes Care.
2018;41(12):2648-2668. doi:10.2337/dci18-0052
Javed A, Schwenk WF II, Tebben P. Diabetes mellitus. In: McInerny TK, Adam HM, Campbell DE, DeWitt
TG, Foy JM, Kamat DM, eds. American Academy of Pediatrics Textbook of Pediatric Care. American
Academy of Pediatrics; 2023. Accessed September 1, 2023. Pediatric Care Online
Koren D, Levitsky LL. Type 2 diabetes mellitus in childhood and adolescence. Pediatr Rev. 2021;
42(4):167-179. doi:10.1542/pir.2019-0236
TODAY Study Group; Bjornstad P, Drews KL, Caprio S, et al. Long-term complications in youth-onset
type 2 diabetes. N Engl J Med. 2021;385(5):416-426. doi:10.1056/NEJMoa2100165
Content Domain
Endocrinology
A 12-year-old boy is seen in the office for evaluation of severe headaches that have occurred intermittently
over the past month. His headaches are associated with blurry vision, heart palpitations, sweating, and facial
flushing. There is no associated vomiting, photophobia, or phonophobia. The family history is notable for a
grandparent who died of thyroid cancer and an uncle who had a cerebrovascular accident in his mid-30s.
On physical examination, the boy’s vital signs are a temperature of 37.1 °C, a blood pressure of 160/96 mm
Hg, a heart rate of 105 beats/min, and an oxygen saturation of 100% in room air. His body mass index is at
the 50th percentile for age. The remainder of his physical examination findings are unremarkable.
Of the following, the BEST test to confirm this boy’s diagnosis is
A. an aldosterone concentration
Correct answer is B
PREP Pearl(s)
Episodic or persistent hypertension associated with severe headaches, palpitations, and sweating
should raise concern regarding pheochromocytoma.
The initial evaluation for suspected pheochromocytoma should include fractionated plasma
metanephrine or 24-hour urinary metanephrine levels.
Most children with pheochromocytoma have an associated genetic syndrome (eg, multiple endocrine
neoplasia type 2, von Hippel-Lindau disease, neurofibromatosis type 1).
Critique
The boy in the vignette has considerable hypertension (stage 2) documented in the office. The constellation
of hypertension (the likely cause of his headaches and blurry vision), tachycardia, facial flushing, and
palpitations is suggestive of catecholamine excess caused by a pheochromocytoma. The best test to confirm
the diagnosis of pheochromocytoma is fractionated metanephrine concentrations. The boy’s family history of
thyroid cancer and stroke at a young age (which may have been caused by a hypertensive emergency) raises
concern regarding an inherited condition, specifically multiple endocrine neoplasia type 2 (MEN2). Medullary
thyroid cancer and pheochromocytoma are features of MEN2.
Conditions associated with aldosterone excess (eg, primary aldosteronism) cause hypertension owing to
sodium and fluid retention. However, tachycardia, flushing, and sweating are not associated with these
conditions. An elevated midnight salivary cortisol level would be consistent with Cushing syndrome, which
can cause hypertension and facial flushing (plethora). However, Cushing syndrome has an indolent onset, is
accompanied by weight gain and poor growth, and is not associated with palpitations and sweating.
Hyperthyroidism can cause hypertension, tachycardia, palpitations, and sweating. However, in the context of
the boy’s findings and his family history, an inherited condition (eg, MEN2) is more likely.
Pheochromocytoma is a tumor of the adrenal medullary chromaffin cells, which produce catecholamines.
These tumors may be unilateral or bilateral. They can produce any or all types of catecholamines, including
epinephrine, norepinephrine, metanephrine, normetanephrine, and dopamine. Typical signs and symptoms
of pheochromocytoma include episodic or chronic hypertension, tachycardia, severe headache, facial
flushing, and sweating. The initial evaluation for suspected pheochromocytoma should include measurement
of fractionated metanephrine concentrations (via 24-hour urine collection or spot plasma collection). The
pattern of catecholamine elevation can inform the etiology of the tumor (eg, MEN2, neurofibromatosis type 1,
von Hippel-Lindau disease).
Pheochromocytoma in a child is rare and often associated with a genetic syndrome. These syndromes must
be considered in any child diagnosed with pheochromocytoma to avoid potential comorbidities. Multiple
endocrine neoplasia type 2 is characterized by medullary thyroid cancer, pheochromocytoma, and
hyperparathyroidism. It is caused by pathologic variants in the RET proto-oncogene and is inherited in an
autosomal dominant fashion. Medullary thyroid carcinoma is universal in MEN2, so early detection of a RET
pathologic variant is essential when there is a known family history of this condition. Pheochromocytomas
caused by MEN2 typically secrete both metanephrine and normetanephrine.
Von Hippel-Lindau disease is an autosomal dominant condition associated with multiple malignant and
benign tumors, including hemangioblastomas of the retina and central nervous system, renal cell carcinoma,
and pheochromocytoma. Pheochromocytomas associated with von Hippel-Lindau disease typically secrete
normetanephrines, which distinguishes them from pheochromocytomas caused by MEN2.
Other conditions associated with pheochromocytoma include succinate dehydrogenase deficiency (types B
and D) and neurofibromatosis type 1. Pheochromocytomas associated with neurofibromatosis type 1
typically secrete metanephrines.
Suggested Reading(s)
Speiser PW. Adrenal dysfunction. In: McInerny TK, Adam HM, Campbell DE, Foy JM, Kamat DM, eds.
American Academy of Pediatrics Textbook of Pediatric Care. 2nd ed. American Academy of Pediatrics;
2016:chap 211. Accessed September 1, 2023. Pediatric Care Online
Waguespack SG, Rich T, Grubbs E, et al. A current review of the etiology, diagnosis, and treatment of
pediatric pheochromocytoma and paraganglioma. J Clin Endocrinol Metab. 2010;95(5):2023-2037.
doi:10.1210/jc.2009-2830
Weaver DJ. Hypertension in children and adolescents. Pediatr Rev. 2017;38(8):369-382.
doi:10.1542/pir.2016-0106
Content Domain
Endocrinology
A healthy neonate is born after an uncomplicated pregnancy. Noninvasive prenatal testing indicated a 46,XY
karyotype. However, on physical examination the neonate has normal female genitalia, consistent with
prenatal ultrasonographic findings. There are no masses palpable in the labial folds or along the inguinal
canal. The remainder of the neonate’s physical examination findings are normal.
Of the following, the BEST laboratory test to obtain for this neonate is
A. estradiol
B. 17-hydroxyprogesterone
C. karyotype
D. testosterone
Correct answer is C
PREP Pearl(s)
Noninvasive prenatal testing has a low positive predictive value for sex chromosome abnormalities.
A noninvasive prenatal test result that indicates a sex chromosome abnormality must be confirmed
with a postnatal karyotype. Hormonal testing and pelvic imaging of the neonate should be performed
only after a difference of sex development is confirmed.
Management of differences of sex development requires an interprofessional and multidisciplinary
team approach and may include monitoring for development of gonadal tumors.
Critique
The neonate in the vignette may have a difference of sex development (DSD). Noninvasive prenatal testing
(NIPT) revealed a 46,XY karyotype despite the neonate’s having normal-appearing female genitalia. The best
next step in this neonate’s evaluation is to confirm the NIPT result by obtaining a postnatal karyotype.
Noninvasive prenatal testing involves the isolation and sequencing of cell-free fetal DNA from the mother’s
blood. Initially, NIPT was used to screen for trisomy 13, 18, and 21. For families at high risk of having a child
with a chromosomal anomaly, early detection by NIPT provides time for information gathering and genetic
counseling to help with decision making as to whether or not to continue the pregnancy. Diagnosis through
NIPT may also facilitate earlier screening for comorbid conditions associated with the chromosomal
abnormality (eg, heart anomalies in Turner syndrome).
The use of NIPT has expanded to include sex chromosome abnormalities, which has raised ethical concerns.
The positive predictive value of an NIPT result that indicates a sex chromosome abnormality is between 26%
and 50%. Therefore, more times than not, a positive NIPT screening result for a sex chromosome abnormality
is a false-positive. Families may use NIPT results to make decisions about the future of their pregnancy or to
plan for a child’s developmental needs. Noninvasive prenatal testing may also reveal a previously unknown
medical condition in the mother if chromosomal mosaicism is detected. Ethical concerns have arisen
surrounding families “selecting” for a certain sex, because NIPT can provide fetal sex determination as early
as 10 weeks’ gestation.
Differences of sex development are classified according to the sex chromosome complement (46,XX, 46,XY,
or sex chromosomal DSD) and are further subdivided according to hormonal synthesis or action defects.
Figure 1 shows the normal process of fetal sex determination and differentiation. The interplay of the SRY
gene and other transcription factors results in the bipotential gonad becoming a testicle. Without these
genes, an ovary forms. Testes secrete testosterone, resulting in formation of male external genitalia, and
anti-Müllerian hormone, causing regression of female internal genitalia. Lack of testosterone stimulation
results in female external genitalia, and lack of anti-Müllerian hormone allows for continued development of
female internal genitalia. An algorithm for the evaluation of a DSD is outlined in Figure 2.
On identification of a DSD, the neonate should undergo further evaluation and treatment by an
interprofessional and multidisciplinary team including the primary care practitioner, pediatric
endocrinologists, pediatric urologists, geneticists, and social workers or psychologists. In addition to
providing medical care, this team will help families cope with the diagnosis and may assist in shared decision
making regarding a sex of rearing. Some DSDs are associated with development of gonadal tumors (eg, when
the karyotype contains Y material and an internal or undescended gonad is present). Early diagnosis
facilitates the planning and implementation of appropriate screening for tumor development.
Suggested Reading(s)
Diaz A, Lipman Diaz EG. Disorders of sex development. Pediatr Rev. 2021;42(8):414-426.
doi:10.1542/pir.2018-0183
Guo N, Cai M, Lin M, Xue H, Huang H, Xu L. Positive predictive value of noninvasive prenatal testing for
sex chromosome abnormalities. Mol Biol Rep. 2022;49(10):9251-9256. doi:10.1007/s11033-022-07754-x
Johnston M, Warton C, Pertile MD, et al. Ethical issues associated with prenatal screening using non-
invasive prenatal testing for sex chromosome aneuploidy. Prenat Diagn. 2022. doi:10.1002/pd.6217
Lee PA, Nordenström A, Houk CP, et al. Global disorders of sex development update since 2006:
perceptions, approach and care. Horm Res Paediatr. 2016;85(3):158-180. doi:10.1159/000442975
Loomba-Albrecht LA, Styne DM. Disorders of sex development. In: McInerny TK, Adam HM, Campbell
DE, Foy JM, Kamat DM, eds. American Academy of Pediatrics Textbook of Pediatric Care. 2nd ed.
American Academy of Pediatrics; 2016:chap 243. Accessed September 1, 2023. Pediatric Care Online
Content Domain
Endocrinology
A 3-day-old term neonate is being evaluated in the newborn nursery for ambiguous genitalia. The pregnancy
was uncomplicated except for prenatal ultrasonography that revealed the ambiguity. Prenatal genetic testing
demonstrated a fetal 46,XX karyotype. The neonate is feeding well and stooling and urinating normally.
The newborn is vigorous and has normal vital signs for age. The skin is hyperpigmented. Genital examination
reveals a phallic structure measuring 3 cm in length, posterior fusion of the labioscrotal folds, and a single
urogenital orifice. There are no palpable gonads, and the remainder of the physical examination findings are
normal.
Of the following, the test MOST likely to reveal this neonate’s diagnosis is a
A. 17-hydroxypregnenolone level
B. 17-hydroxyprogesterone level
C. cortisol level
D. testosterone level
Correct answer is B
PREP Pearl(s)
Because of the life-threatening nature of an adrenal crisis, the etiology of virilization of a 46,XX neonate
should be presumed to be congenital adrenal hyperplasia due to 21-hydroxylase deficiency until
proven otherwise.
Salt-wasting congenital adrenal hyperplasia is treated with hydrocortisone and fludrocortisone
replacement. Infants with salt-wasting congenital adrenal hyperplasia also require sodium chloride
supplementation.
The initial screening test of choice for congenital adrenal hyperplasia is a 17-hydroxyprogesterone
level.
Critique
The neonate in the vignette has a fetal 46,XX karyotype and virilized external genitalia. Although a postnatal
karyotype test is required to confirm the prenatal genetic test findings, the most likely cause of these findings
is congenital adrenal hyperplasia (CAH) due to 21-hydroxylase deficiency. The diagnostic test of choice is a 17-
hydroxyprogesterone level, which would be elevated in this case of CAH due to 21-hydroxylase deficiency
(Figure). The 17-hydroxyprogesterone level may be elevated in premature infants without CAH. It is therefore
important to use gestational age-appropriate cutoffs when interpreting the findings of this laboratory test,
including on newborn screening, which measures 17-hydroxyprogesterone to screen for CAH.
Congenital adrenal hyperplasia describes multiple conditions in which the formation of adrenal steroids is
disrupted. The most common form is 21-hydroxylase deficiency. This enzyme is concentrated in the zona
glomerulosa and zona fasciculata layers of the adrenal cortex and is, therefore, involved in the eventual
production of aldosterone and cortisol, respectively. However, the enzyme is not present in the zona
reticularis, the androgen-producing layer of the adrenal cortex. When enzyme activity is disrupted, the ability
of the adrenal glands to produce cortisol and aldosterone is reduced. Because of decreased negative
feedback from cortisol, adrenocorticotropic hormone concentrations rise, shunting steroid precursors into
the adrenal androgen pathway and resulting in androgen excess. In female (46,XX) fetuses, the excess
androgens result in variable degrees of genital virilization that may include clitoromegaly, partial or complete
fusion of the labioscrotal folds, and a single urogenital sinus. Male (46,XY) genitalia are generally unaffected.
The high adrenocorticotropic hormone levels also stimulate melanocyte-stimulating hormone receptors,
resulting in skin hyperpigmentation.
Cortisol deficiency can result in an adrenal crisis. Neonates do not typically experience a crisis until at least 5
days after birth owing to the small amount of circulating maternal cortisol that escapes metabolism by the
placenta. Symptoms and signs of a crisis may include lethargy, poor feeding, vomiting, hypoglycemia, and
shock. Older children may report abdominal pain. Aldosterone deficiency results in salt wasting and
potassium retention in the kidney, resulting in the pattern of low sodium and high potassium levels
characteristic of classic salt-wasting CAH (as opposed to nonclassic or simple virilizing CAH).
The treatment for CAH is steroid replacement. Glucocorticoid deficiency is treated with hydrocortisone
replacement, whereas mineralocorticoid deficiency is treated with fludrocortisone. Owing to the relatively low
sodium content of breast milk and infant formula, infants are also treated with sodium chloride
supplementation. Steroid replacement must be balanced to reduce androgen production while avoiding
growth limitation due to iatrogenic Cushing syndrome. During times of illness or injury, affected children are
unable to mount an adrenal stress response, so increased doses (“stress doses”) of hydrocortisone are
required. High-dose glucocorticoids can stimulate mineralocorticoid receptors, so an increase in the
fludrocortisone dose is not needed during times of stress.
Measurement of cortisol is part of the evaluation for adrenal insufficiency. However, without stress, cortisol
concentrations may be normal even in the setting of CAH. A more significant finding is a cortisol
concentration that is not appropriately elevated at times of stress or crisis. In infants, there is no diurnal
variation in cortisol production, so a low concentration does not definitively indicate adrenal insufficiency (eg,
CAH).
Suggested Reading(s)
Diaz A, Diaz EGL. Disorders of sex development. Pediatr Rev. 2021;42(8):414-426. doi:10.1542/pir.2018-
0183
Endocrine disorders manifesting in the newborn period. In: McInerny TK, Adam HM, Campbell DE, Foy
JM, Kamat DM, eds. American Academy of Pediatrics Textbook of Pediatric Care. American Academy of
Pediatrics; 2023. Accessed September 1, 2023, 2023. Pediatric Care Online
Speiser PW, Arlt W, Auchus RJ, et al. Congenital adrenal hyperplasia due to steroid 21-hydroxylase
deficiency: an Endocrine Society clinical practice guideline. J Clin Endocrinol Metab. 2018;103(11):4043–
4088. doi:10.1210/jc.2018-01865
Content Domain
Endocrinology
A 13-year-old boy is seen for a preparticipation examination in preparation for playing baseball. He is
concerned about his small size compared with his peers and fears he will not be able to keep up with his
taller teammates. He has no notable medical history, and findings of a review of systems are unremarkable.
On physical examination, the boy’s vital signs are normal. He is 140 cm tall (<3rd percentile, z score of -2.1),
weighs 30 kg (<3rd percentile), and has a body mass index of 15.3 kg/m2 (3rd percentile) (Figure 1, Figure 2).
His height percentile has not deviated since age 5 years. He has a sexual maturity rating of 1 for pubic hair,
and his testicular volumes are 3 mL bilaterally. The remainder of his physical examination findings are
normal.
Correct answer is C
PREP Pearl(s)
Constitutional delay of growth and puberty is a normal variant that does not usually require any
intervention.
In cases of severe delay of puberty and psychosocial distress, a short course of sex steroid can be
considered to induce puberty.
Critique
The boy in the vignette has constitutional delay of growth and puberty. This normal variant of later-than-
average pubertal onset does not usually require any intervention. The best next treatment step for this boy is
to monitor his growth and pubertal development over time.
Adolescents with constitutional growth delay have late puberty and are otherwise healthy. Their absolute
height percentile is low, but their linear growth velocity is normal; therefore, their height percentile is
maintained over time. Bone age is delayed in comparison with chronological age and projects an adult height
appropriate for the family (Figure 3). There is frequently a family history of delayed puberty.
Reprinted with permission from Bakhtiani P, Geffner M. Delayed puberty. Pediatr Rev. 2022;43(8):428.
Figure 3. “Constitutional delay growth curve showing that at a chronological age of 14 years (green box),
this adolescent’s bone age is delayed at 12 years (red open star) . Mid-parental height is projected ( black
open star) based on growth charts form the Center for Disease Control and Prevention.”.
For boys aged 14 years or older, a short course of testosterone can be considered to induce puberty. This
treatment may help adolescents who are distressed about their lack of growth and sexual development. The
boy in the vignette is too young for testosterone therapy. Girls can be treated with a short course of estradiol
if there are no signs of puberty by age 13 years.
Growth hormone therapy is not indicated for constitutional delay of growth and puberty unless the short
stature is severe (height below the 1st percentile; z score lower than –2.25). Without a hormone deficiency,
height is expected to normalize when the child enters puberty spontaneously.
Other causes of delayed or absent puberty should be ruled out. The investigation may include a bone age
radiograph, a laboratory evaluation for puberty hormones (luteinizing hormone, follicle-stimulating hormone,
estradiol, or testosterone concentrations), prolactin concentration, thyroid function tests, and other testing
guided by history and physical examination findings. A karyotype should be performed for girls aged 13 years
or older with absent puberty to evaluate for Turner syndrome. A karyotype is not indicated for the otherwise
healthy boy in the vignette.
Suggested Reading(s)
Bakhtiani P, Geffner M. Delayed puberty. Pediatr Rev. 2022;43(8):426-435. doi:10.1542/pir.2020-005291
Kritzler RK, Long L, Plotnick L. Puberty: normal and abnormal. In: McInerny TK, Adam HM, Campbell DE,
Foy JM, Kamat DM, eds. American Academy of Pediatrics Textbook of Pediatric Care. 2nd ed. American
Academy of Pediatrics; 2016:chap 185. Accessed September 1, 2023. Pediatric Care Online
Content Domain
Endocrinology
12 13 14 15 16 17 18 19 20
Mother’s Stature Father’s Stature cm in
Date Age Weight Stature BMI*
AGE (YEARS) 76
95
190
74
90
185 S
75
72
180 T
50 70 A
175 T
25 68 U
170 R
10 66
165 E
in cm 3 4 5 6 7 8 9 10 11 5
64
160 160
62 62
155 155
S 60 60
T 150 150
A 58
T 145
U 56
140 105 230
R
54
E 135 100 220
52
130 95 95 210
50
125 90 200
90
48 190
120 85
46 180
115 80
75
44 170
110 75
42 160
105 50 70
150 W
40
100 65 140 E
25
38 I
95 60 130 G
10
36 90 5 H
55 120
T
34 85 50 110
32 80 45 100
30
40 90
80 35 35 80
W 70 70
30 30
E 60 60
I 25 25
G 50 50
H 20 20
40 40
T
15 15
30 30
10 10
lb kg AGE (YEARS) kg lb
2 3 4 5 6 7 8 9 10 11 12 13 14 15 16 17 18 19 20
Published May 30, 2000 (modified 11/21/00).
SOURCE: Developed by the National Center for Health Statistics in collaboration with
the National Center for Chronic Disease Prevention and Health Promotion (2000).
http://www.cdc.gov/growthcharts
2 to 20 years: Boys NAME
Body mass index-for-age percentiles RECORD #
35
34
33
32
31
30
95
29
BMI 28
90
27 27
26 85 26
25 25
75
24 24
23 23
50
22 22
21 21
25
20 20
10
19 19
5
18 18
17 17
16 16
15 15
14 14
13 13
12 12
2 2
kg/m AGE (YEARS) kg/m
2 3 4 5 6 7 8 9 10 11 12 13 14 15 16 17 18 19 20
An 6-month-old infant with a history of prematurity is brought to the emergency department for decreased
movement of an extremity. His parents report that he was playful and moving all extremities that morning.
His father brought him to daycare shortly after his mother left for work; he recalled feeling “stressed out”
because he had to bundle him up and drive through the snowstorm. When his mother picked him up from
daycare later that day, he was still doing well and fell asleep in the car. When she undressed him for a bath,
she noticed swelling of his extremity and became concerned. She called the daycare, which reported no
history of falls or injuries.
The infant was born at 33 weeks’ gestation. All prenatal test results were normal, and the pregnancy was
uncomplicated other than preterm delivery. He was cared for in the neonatal intensive care unit (NICU) for 3
weeks and required oxygen by nasal cannula for 2 days. His NICU course was otherwise unremarkable. He is
exclusively breastfed. He was recently seen by his primary pediatrician for concerns of poor growth.
Laboratory studies reveal a low 25-hydroxyvitamin D level. Radiographic imaging is obtained in the
emergency department.
A. intrasutural bones
Correct answer is B
PREP Pearl(s)
Vitamin D–deficient rickets should be considered in children with risk factors, such as limited sunlight
exposure, exclusive breastfeeding not supplemented with vitamin D, increased melanin in the skin,
and certain conditions and medications that affect vitamin absorption.
For exclusively or partially breastfed infants, the current standard recommended daily intake of
vitamin D is 400 IU beginning by 2 weeks of age.
Vitamin D–deficient rickets is associated with decreased levels of 25-hydroxyvitamin D, calcium, and
phosphorus and an increased parathyroid hormone level.
Critique
The infant in the vignette has risk factors for age, history of prematurity, exclusive intake of breast milk, and
limited sun exposure during the winter months and laboratory findings consistent with vitamin D deficiency.
The finding of metaphyseal cupping of the wrists on radiographic imaging is associated with vitamin D
deficiency.
Vitamin D regulates the absorption of calcium and phosphorus, which is important for bone health. The main
source for vitamin D synthesis is the sun. According to the American Academy of Pediatrics, a serum level of
25-hydroxyvitamin D less than 20 ng/mL (49.92 nmol/L) constitutes vitamin D insufficiency. Risk factors for
vitamin D deficiency include limited sun exposure, increased melanin in the skin, exclusive breastfeeding not
supplemented with vitamin D, and conditions and medications that affect vitamin absorption.
Vitamin D deficiency can result in rickets, which is characterized by impairment of calcification of the growth
plates in infants and children. The peak incidence occurs between the ages of 4 and 12 months. Signs and
symptoms may include poor growth, widened wrists (Figure 1), bowing of the forearms and legs, and
seizures. Laboratory findings may include decreased levels of 25-hydroxyvitamin D, calcium, and phosphorus.
A low calcium level triggers the release of parathyroid hormone (PTH); therefore, PTH levels may be elevated.
Radiographic findings may include cupping of the metaphysis (Figure 2), bony prominence of the ribs at the
costochondral junction (rachitic rosary), osteopenia, and fractures (uncommon).
The current standard recommended daily intake of vitamin D is 400 IU; however, an individual child’s
recommended intake may vary, depending on factors such as prematurity, birth weight, diet, and underlying
medical conditions. Treatment of vitamin D–deficient rickets is with high-dose vitamin D until serum levels
and radiographic findings improve.
Osteogenesis imperfecta is a congenital bone disorder that results in an increased propensity to fracture. It is
associated with specific clinical features and radiographic findings, such as intrasutural bones (wormian
bones). This infant has no findings suggestive of this disorder. Metaphysitis is the radiologic finding seen in
infants with congenital syphilis. This infant has no history or findings consistent with congenital syphilis.
Posterior rib fractures have a high specificity for physical abuse. Intentional trauma is less likely than rickets
in this infant with risk factors and laboratory evidence of an underlying bone disease; however, intentional
trauma should be considered in any infant or child presenting with an unexplained injury. It is important to
consider nonaccidental trauma even in children with a medical condition that makes them more susceptible
to injury.
Suggested Reading(s)
Golden NH, Abrams SA; Committee on Nutrition; Daniels SR, Abrams SA, Corkins MR, et al. Optimizing
bone health in children and adolescents. Pediatrics. 2014;134(4):e1229-e1243. doi:10.1542/peds.2014-
2173
Lauer B, Spector N. Vitamins. Pediatr Rev. 2012;33(8):339-352. doi:10.1542/pir.33-8-339
Lee JY, So TY, Thackray J. A review on vitamin D deficiency treatment in pediatric patients. J Pediatr
Pharmacol Ther. 2013;18(4):277-291. doi:10.5863/1551-6776-18.4.277
Misra M, Pacaud D, Petryk A, Collett-Solberg PF, Kappy M; Drug and Therapeutics Committee of the
Lawson Wilkins Pediatric Endocrine Society. Vitamin D deficiency in children and its management:
review of current knowledge and recommendations. Pediatrics. 2008;122(2):398-417.
doi:10.1542/peds.2007-1894
Wagner CL, Greer FR; Section on Breastfeeding and Committee on Nutrition. Prevention of rickets and
vitamin D deficiency in infants, children, and adolescents. Pediatrics. 2008;122(5):1142-1152.
doi:10.1542/peds.2008-1862
Content Domain
Endocrinology
A 15-year-old boy is seen for a health supervision visit. He has no concerns. His body mass index is noted to
be 30 kg/m2 (>95th percentile). He does not exercise regularly. He reports eating normal portion sizes and
occasionally drinking regular soda.
The boy’s blood pressure is 137/87 mm Hg and his heart rate is 82 beats/min. He is in no distress. Skin
examination reveals the finding shown (Figure). He has a sexual maturity rating of 5. The remainder of his
physical examination findings are unremarkable.
Of the following, the BEST next step in this adolescent’s care is to initiate therapy with
A. atorvastatin
B. insulin
C. liraglutide
D. metformin
Correct answer is D
PREP Pearl(s)
Acanthosis nigricans is a sign of insulin resistance and a risk factor for development of type 2 diabetes.
The first-line treatment of uncomplicated type 2 diabetes is metformin, which sensitizes tissues to
insulin action and decreases hepatic glucose production.
Critique
The adolescent in the vignette meets the diagnostic criteria for diabetes mellitus based on his elevated
hemoglobin A1c level of 7.2% (≥6.5%) and fasting glucose level of 150 mg/dL (≥126 mg/dL). He appears well
and his bicarbonate concentration is normal, so he does not meet criteria for diabetic ketoacidosis (DKA). His
elevated body mass index and presence of acanthosis nigricans make type 2 diabetes the most likely
diagnosis. Of the response choices, initiation of metformin is the best next management step.
Insulin resistance secondary to obesity is a major contributor to the pathophysiology of type 2 diabetes. The
lack of normal insulin action results in a clinical picture similar to type 1 diabetes. Diabetic ketoacidosis is less
common given that high levels of insulin can overcome the resistance to a degree. However, chronic
hyperglycemia can impair β-cell function, which can result in relative insulin deficiency and DKA. Individuals
with type 2 diabetes are also at risk of experiencing a hypoketotic hyperosmolar state. In this condition,
insulin action is effective enough to prevent significant ketone production, but glucose levels rise dramatically
(sometimes >1,000 mg/dL [55.50 mmol/L]), causing electrolyte shifts and dehydration that can result in coma
and death.
Insulin resistance frequently manifests as acanthosis nigricans. Increased insulin levels stimulate epidermal
growth factors that cause hyperpigmentation in the flexor surfaces (eg, neck, axillae, groin). Occasionally, skin
tags are seen.
The first-line treatment for type 2 diabetes without DKA, a hypoketotic hyperosmolar state, or severe
hyperglycemia (glucose level ≥250 mg/dL or hemoglobin A1c ≥8.5%) is metformin. This medication sensitizes
tissues to insulin action and decreases hepatic glucose production. Insulin is indicated if DKA, a hypoketotic
hyperosmolar state, or severe hyperglycemia is present or if treatment with other medications fails. Basal
insulin is initiated before proceeding to basal-bolus therapy. Basal-bolus insulin therapy is more difficult to
manage, and insulin, an anabolic hormone, makes weight loss more difficult.
Liraglutide is a glucagon-like peptide 1 (GLP-1) receptor agonist that is approved as a second-line drug to
treat type 2 diabetes. Glucagon-like peptide 1 agonists function by increasing insulin production, decreasing
gastric emptying (resulting in satiety), and stimulating satiety centers in the brain. Glucagon-like peptide 1
agonists also are indicated in adolescents with obesity when diet and exercise changes are not effective.
Atorvastatin is a statin used to reduce low-density lipoprotein (LDL) cholesterol in those with
hypercholesterolemia. Statin therapy is indicated in adolescents with type 2 diabetes who have an LDL
cholesterol level greater than 130 mg/dL (3.37 mmol/L) despite improved glucose control and healthy diet
and exercise changes. Although the adolescent in the vignette has an elevated LDL cholesterol level (135
mg/dL [3.5 mmol/L]), statin therapy is not indicated until he is given an opportunity to improve his cholesterol
with diet and exercise and his diabetes is treated.
Suggested Reading(s)
Arslanian S, Bacha F, Grey M, Marcus MD, White NH, Zeitler P. Evaluation and management of youth-
onset type 2 diabetes: a position statement by the American Diabetes Association. Diabetes Care.
2018;41(12):2648-2668. doi:10.2337/dci18-0052
ElSayed NA, Aleppo G, Aroda VR, et al; on behalf of the American Diabetes Association. 14. Children
and adolescents: standards of care in diabetes—2023. Diabetes Care. 2023;46(suppl 1):S230-S253.
doi:10.2337/dc23-S014
Hampl SE, Hassink SG, Skinner AC, et al. Clinical practice guideline for the evaluation and treatment of
children and adolescents with obesity. Pediatrics. 2023;151(2):e2022060640. doi:10.1542/peds.2022-
060640
Javed A, Schwenk WF II, Tebben P. Diabetes mellitus. In: In: McInerny TK, Adam HM, Campbell DE, Foy
JM, Kamat DM, eds. American Academy of Pediatrics Textbook of Pediatric Care. 2nd ed. American
Academy of Pediatrics; 2023. Accessed September 1, 2023. Pediatric Care Online
Koren D, Levitsky LL. Type 2 diabetes mellitus in childhood and adolescence. Pediatr Rev.
2021;42(4):167-179. doi:10.1542/pir.2019-0236
Nagpal N, Messito MJ, Katzow M, Gross RS. Obesity in children. Pediatr Rev. 2022;43(11):601-617.
doi:10.1542/pir.2021-005095
Content Domain
Endocrinology
A previously healthy 7-year-old girl is brought to the emergency department for evaluation of acute-onset
upper abdominal pain that radiates to her back. The pain worsens when she eats. The girl appears ill and
uncomfortable. Her temperature is 38 °C, blood pressure is 120/85 mm Hg, heart rate is 120 beats/min, and
respiratory rate is 22 breaths/min. She has diffuse abdominal tenderness without rebound or guarding. The
remainder of her examination findings are unremarkable.
An intravenous line is placed and a blood sample is obtained for laboratory evaluation. The blood sample
appears turbid.
Of the following, the MOST likely laboratory finding for this child is a markedly elevated level of
A. albumin
B. hemoglobin
C. indirect bilirubin
D. triglyceride
Correct answer is D
PREP Pearl(s)
Clinical manifestations of markedly elevated triglyceride levels may include acute pancreatitis,
hepatosplenomegaly, and eruptive xanthomas; the serum may have a turbid or milky appearance.
Management of marked hypertriglyceridemia includes a very low-fat diet, medium chain triglyceride
supplementation, and fat-soluble vitamin supplementation.
Familial chylomicronemia syndrome is caused by a lipoprotein lipase enzyme or cofactor deficiency; it
is associated with markedly elevated triglyceride levels.
Critique
The child in the vignette has acute pancreatitis caused by hypertriglyceridemia. Her most likely laboratory
finding is a markedly elevated triglyceride level, which is responsible for the turbid appearance of her blood
sample. An elevation in albumin, hemoglobin, or indirect bilirubin would not cause a turbid appearance of
the blood. Pancreatitis can be associated with an elevated direct bilirubin level if it is caused by an obstructive
stone. Pancreatitis is not associated with an elevated indirect bilirubin level.
Apolipoproteins
Function as cofactors and inhibitors for enzymes involved in lipid metabolism
Mediate lipoprotein binding to cellular receptors
Wolman disease, a disturbance of intracellular cholesterol metabolism, is caused by lysosomal acid lipase
deficiency, which leads to accumulation of cholesterol esters within the cell. Clinical manifestations include
hepatosplenomegaly, failure to thrive, and steatorrhea. Prognosis is poor, and death often occurs by age 1
year. Cholesterol ester storage disease is a less severe form of lysosomal acid lipase deficiency.
Suggested Reading(s)
Squires JE, Kocoshis SA. Pancreatitis. In: McInerny TK, Adam HM, Campbell DE, DeWitt TG, Foy JM, eds.
American Academy of Pediatrics Textbook of Pediatric Care. American Academy of Pediatrics;
2016:chap 306. Pediatric Care Online
Stanley CA, Bennett MJ. Defects in metabolism of lipids: disorders of mitochondrial fatty acid β-
oxidation. In: Kliegman R, St. Geme J, eds. Nelson Textbook of Pediatrics. 21st ed. Elsevier; 2019.
Valaiyapathi B, Sunil B, Ashraf AP. Approach to hypertriglyceridemia in the pediatric population. Pediatr
Rev. 2017;38(9):424-434. doi:10.1542/pir.2016-0138
Content Domain
Endocrinology
A healthy, pregnant 16-year-old adolescent and her parents are seeking advice about her pregnancy from
the pediatrician, who has known the family since the adolescent’s birth. The adolescent was adopted; her
parents have never disclosed the adoption to her. Additionally, they have told her that the family medical
history is negative, although her biological mother has cystic fibrosis. The pediatrician has strongly suggested
to the parents multiple times that they disclose this information to their daughter. The adolescent states that
she and her parents mutually want to be very involved during her pregnancy and in caring for her child. She
adds that her obstetrician informed her today of a positive cystic fibrosis carrier screening test result,
information she has not provided her parents until now. She asks how she could be a carrier of cystic fibrosis
if her family has no history of any medical issues. Her parents appear clearly uncomfortable.
Of the following, the BEST initial strategy to address this adolescent’s concern is to
A. advise her that the test result could be a false-positive and should be repeated
B. speak to the adolescent privately and inform her of her adoption status and family medical
history
C. speak to the parents privately and advise them to inform her of her adoption status and family
medical history
D. suggest the family discuss this inquiry together rst with honesty and in privacy
Correct answer is D
PREP Pearl(s)
Patients and parents should be allowed, when possible, to discuss sensitive issues privately, among
themselves, before discussing them with the pediatrician.
Fidelity (trustworthiness) and veracity (truthfulness) must be practiced in context and may manifest
differently depending on the situation.
Critique
The adolescent in the vignette is unaware of her biological mother’s medical history, which could significantly
affect her own health and that of her child. Her pediatrician has an obligation to maintain trust in the
relationship by being truthful but also sensitive to the relationship between the adolescent and her parents.
The best initial strategy to address this adolescent’s concern is to acknowledge the family’s feelings and
suggest they discuss her inquiry privately together first, allowing them time and space to be honest with each
other. If the pediatrician speaks with the parents in private first, it may create a sense of secrecy that builds
mistrust between the pediatrician and the adolescent. In this case, if the pediatrician discloses information
privately to the adolescent first, it might increase tension between her and her parents. Advising the
adolescent that the test result could be a false-positive would be a false statement.
Pediatricians are unique in that they may have several “patients”: the child, the parents, and other legal or
emotional caregivers. Balancing these interactions may feel like walking on a tightrope, with the potential for
falling by failing to build equally healthy relationships with all members of the family. Being openly sensitive
to the family while respecting the ethical principles that guide pediatricians’ interactions with families is
paramount to building a safe medical home.
The pediatrician had previously encouraged the adolescent’s parents to disclose to their daughter
information about her adoption status. The pediatrician must respect the parents’ right to decide the best
timing to reveal this information. Such disclosure was, perhaps, not seen by the parents as urgent because it
did not affect their daughter at the time. Now that their daughter is pregnant, such information becomes
more relevant. In parallel, their daughter has developed her own decision-making capacity and sense of self-
identity. The family has also had the opportunity to establish trust with the pediatrician as part of a medical
home that respects autonomy and confidentiality for all members of the family. Now, more than ever, the
pediatrician must represent the truthfulness and trustworthiness that the family has come to depend on and
value.
Trustworthiness, or fidelity, is a key building block for any relationship. Physicians must be trusted to keep
the best interests of their patients as their first priority. Respect, confidentiality, and transparency promote
trustworthiness. For pediatricians, this fidelity can be tested when adolescents’ values, which may be
dynamic, come into conflict with those of their caregivers. The duty of the pediatrician, especially with highly
sensitive topics such as identity and risk-taking behaviors, is to respect the confidentiality of both the
adolescent and the caregivers, while encouraging transparency among all parties to both prevent adverse
health outcomes and promote flourishing.
Truthfulness, or veracity, is also a component of fidelity. Yet, truthfulness may be contextual, framed by
multiple factors such as patient confidentiality and capacity. The decision about what information to disclose,
as well as when and how to disclose it, may vary, not only among families but also within a family. Disclosure
may depend on the specific situation as well as on the adolescent’s growth and development. At times,
physicians need to make quick judgments regarding disclosure decisions.
As with all relationships, the patient-parent-pediatrician relationship is incredibly intricate, but by expressing
vulnerability and humility it can be immensely rewarding for everyone involved.
Suggested Reading(s)
Diekema DS, Leuthner SR, Vizcarrondo FE, eds; Section on Bioethics, American Academy of Pediatrics.
American Academy of Pediatrics Bioethics Resident Curriculum: Case-Based Teaching Guides. Revised
2017. American Academy of Pediatrics: 2017. Accessed September 1, 2023. Bioethics Case Based
Teaching Guides for Resident Training (aap.org)
Friedman RL, Lantos JD. Ethical and legal issues for the primary care physician. In: McInerny TK, Adam
HM, Campbell DE, DeWitt TG, Foy JM, Kamat DM, eds. American Academy of Pediatrics Textbook of
Pediatric Care. American Academy of Pediatrics; 2023 Accessed September 1, 2023. Pediatric Care
Online
Lantos J. The patient-parent-pediatrician relationship: everyday ethics in the office. Pediatr Rev.
2015;36(1):22–30. doi:10.1542/pir.36-1-22
Content Domain
Ethics
An attending pediatrician is being sued by a family for malpractice. A 3-year-old child had been brought to
the clinic with a high fever and rash. She was seen by the resident and sent home with a diagnosis of viral
illness. She was brought to the emergency department the following day because she was obtunded and had
fever and purpura. Appropriate care was rapidly implemented in the emergency department; however, she
was left with persistent hearing loss and loss of fingers on both hands as a result of her meningococcal
infection.
A. resident
Correct answer is B
PREP Pearl(s)
A malpractice claim must meet the 4 criteria of proving duty, breach, causation, and damages.
Pediatricians experience lower rates of malpractice lawsuits and indemnity than other specialists.
There are ethical and legal obligations to inform patients of medical errors.
Critique
In the case presented in the vignette, both the resident and supervising attending physician are liable.
Malpractice law is a subset of law known as tort law. A tort refers to a “civil wrong in which a person has
breached a legal duty with harm caused to another.” If a physician's work is substandard and results in injury
to a patient, the physician can be sued for malpractice. The most common reason for which physicians are
sued is medical negligence. To successfully sue for medical negligence, a patient must prove the following:
In a malpractice case, the plaintiff must show that each of these 4 elements is “more likely than not.”
Duty refers to the requirement that the physician has a duty to the patient. There must be a patient-physician
relationship to meet the criteria of duty. A physician supervising a resident or a nurse practitioner caring for a
patient qualifies as having such a relationship, and the physician is considered to have a legal duty to that
patient.
A breach in responsibility must be proved by the plaintiff. Such proof requires the physician to have acted in
a way that was below the standard of care for the circumstances. Sometimes this standard is obvious, but
typically the standard of care needs to be determined and described to the jury.
There must be proof of causation—that the breach was the direct cause of the alleged injury. Expert
testimony is often required to assist a jury in determining whether there was causation.
Finally, there needs to be proof of either economic or noneconomic damages. Economic damages include the
predicted costs of future care and loss of future income; in pediatrics, this can be determined to be very high.
Nursing staff are, under specific circumstances, able to be sued. The most common situations in which
nurses are liable are medication mistakes, failure to monitor patients, patient falls and pressure injuries. In
this vignette, with a lawsuit alleging a missed diagnosis, there is no evidence that nursing staff would have
any associated responsibility.
Pediatricians experience lower rates of malpractice lawsuits and indemnity than other specialties.
Approximately 3.1% of pediatricians face claims in a given year, compared with 7.4% for all other specialties
and 14.5% for high-risk specialties. Although rates of indemnity payments are overall lower for pediatricians
than other specialists, rates of indemnity payments of greater than $1 million were indistinguishable from
other physicians. Claims for children between the ages of 1 and 12 months and those for incidents that
resulted in permanent injury were more likely to result in an indemnity payment.
Expert witnesses are an essential component of the medical malpractice system. They help juries understand
complex scientific and medical information. A judge must deem an expert witness qualified, based on
sufficient knowledge, education, and experience regarding the issue being contested. In the courtroom, the
reliability and relevance of scientific evidence are assessed using the Frye and Daubert rules. The Frye
standard refers to there being general acceptance among the scientific or medical community regarding the
evidence presented. The Daubert rule lays out the criteria to be used by the court with regard to whether the
statement of scientific evidence has been tested and subjected to peer review and whether there is
information regarding the known error rate.
Medical errors continue to be a prevalent problem. In 2000, the Institute of Medicine estimated that 98,000
deaths per year were the result of hospital errors. There is an ethical and legal obligation to inform patients
of medical errors. Patients nearly always want to know that an error took place, regardless of the severity of
the outcome. Physicians may avoid disclosing errors to their patients for a variety of reasons, including fear
of a malpractice lawsuit. However, there is evidence that transparent communication regarding errors may
decrease the likelihood of legal action. Many states have enacted laws that mandate disclosure of serious
errors to patients; 35 states have laws that prevent apologies from being used as evidence in a malpractice
suit.
Suggested Reading(s)
Fanaroff JM. Medical malpractice/expert testimony/disclosure of errors. Pediatr Rev. 2010;31(3):e24-
e27. doi:10.1542/pir.31-3-e24
Fanaroff J, Turbow R. Neonatology and pediatrics. In: Sanbar SS ed. Medical Malpractice Survival
Guide.Mosby; 2007:353-363.
Institute of Medicine, Committee on Quality of Health Care in America. To Err Is Human: Building a
Safer Health System. National Academies Press; 2000.
Jena AB, Chandra A, Seabury SA. Malpractice risk among US pediatricians. Pediatrics. 2013;131(6):1148-
1154. doi:10.1542/peds.2012-3443
Fanaroff JM, Turbow R. Medical-legal considerations in the care of newborns title case. In: McInerny TK,
Adam HM, Campbell DE, DeWitt TG, Foy JM, Kamat DM, eds. American Academy of Pediatrics;
2021:chap 85. Accessed September 1, 2023. Pediatric Care Online
Content Domain
Ethics
A 17-year-old boy is seen in the office with a friend for follow-up after a hospital admission for seizure-like
activity. He has a history of anorexia nervosa, binge-purge subtype, and generalized anxiety disorder. The
discharge diagnosis was functional neurological disorder. The adolescent was told to take a selective
serotonin reuptake inhibitor daily and follow up with his primary care physician and psychiatrist. He has
been consistently taking his medication since discharge. He has no suicidal ideation or recent self-harm. He
graduated from high school 2 months ago, moved out of his parents’ home, and now lives with a partner and
his partner’s sister. He works to pay his rent.
The boy has lost 4.5 kg since his last office visit 4 months ago. He reports daily purging by vomiting. His vital
signs and physical examination findings are normal. Results of his laboratory evaluation are pending. The
adolescent asks if his parents must be informed about his medical condition and laboratory results. He does
not want his parents to know that he continues to struggle with an eating disorder.
C. his parents must be notified of his condition because he is younger than 18 years
D. his parents must give permission for any nonemergency medical care
Correct answer is B
PREP Pearl(s)
An emancipated minor has the legal right to consent to all medical care without parental permission or
notification. The parent has no responsibility in the medical care of an emancipated minor.
Although laws vary by state, in general, emancipated minors are those who are enlisted in the military
(active duty), living separately from their parents, married, or financially independent.
Providers should be aware of their state laws regarding emancipation, parental permission, and
parental notification of confidential health services.
Critique
The adolescent in the vignette has graduated from high school and is living separately from his parents. He is
considered an emancipated minor, and therefore his parents do not have to be notified regarding his
medical care. Involvement of child protective services is not appropriate and may cause harm in this case.
Adolescence is a time of transition between ages 11 and 21 years when physical, cognitive, social, and
emotional development occurs. Adolescents are legally considered adults at age 18 years in most states. The
parents or legal guardians of minors must consent to the child’s medical care and can access their
confidential medical records. Minors are not subject to the protections of the Health Insurance Portability
and Accountability Act of 1996.
If minors meet certain criteria before the age of majority in their state of residence, they may be legally
emancipated. An emancipated minor has the legal right to consent to all medical care without parental
permission or notification. In this case, the parent has no responsibility in the medical care of the
emancipated minor.
Although laws vary by state, in general, emancipated minors are those who meet at least one of the following
criteria:
Because of the confidential health needs of adolescents, there are exceptions to the requirement for
parental consent for minors. These exceptions include federally mandated services such as those guaranteed
by the Emergency Medical Treatment and Labor Act (EMTALA) and the Title X Family Planning Program of the
Public Health Service Act. The EMTALA requires examination and treatment of all individuals, including
minors, who seek emergency medical care in any federally funded emergency department. The Title X Family
Planning Program provides legal access to confidential reproductive health care services for minors without
parental permission. State laws and programs vary regarding parental notification, but minors in most states
can consent to the following services without parental permission:
When minors receive confidential health services without parental permission, physicians should take care to
avoid potential privacy breaches when documenting in electronic health records and when completing billing
and insurance claims. For example, insurance explanation-of-benefit notifications should not include
sensitive information about health services rendered to adolescents. Numerous health professional
organizations have formal policy statements on the importance of confidentiality protections. Adolescents
who are legal adults, by law, have the same confidentiality protections as older adults.
It is recommended that risk management or legal services be involved in care scenarios in which state laws
are unclear regarding the need for parental permission, parental notification, or emancipation laws.
Suggested Reading(s)
Alderman EM, Breuner CC; Committee on Adolescence. Unique needs of the adolescent. Pediatrics.
2019;144(6):e20193150. doi:10.1542/peds.2019-3150
Hagan JF, Shaw JS, Duncan PM, eds. Bright Futures: Guidelines for Health Supervision of Infants,
Children, and Adolescents. 4th ed. American Academy of Pediatrics; 2017.
Society for Adolescent Health and Medicine; American Academy of Pediatrics. Confidentiality
protections for adolescents and young adults in the health care billing and insurance claims process. J
Adolesc Health. 2016;58(3):374-377. doi:10.1016/j.jadohealth.2015.12.009
Content Domain
Ethics
An 8-year-old girl is admitted to the hospital with fever and dehydration. She appears ill but is responsive.
Her temperature is 37 °C, her blood pressure is 98/62 mm Hg, heart rate is 130 beats/min, and respiratory
rate is 20 breaths/min. Her capillary refill time is 5 seconds. The remainder of her examination findings are
unremarkable.
A normal saline bolus and dextrose are ordered. The plan to place an intravenous line to administer fluid and
medications is discussed with the family. The child’s mother expresses understanding of and agreement with
the plan. The child begins to cry and refuses treatment.
Correct answer is C
PREP Pearl(s)
Adolescents’ decisions regarding their sexual health (treatment of sexually transmitted illnesses,
contraceptive care, and prenatal care) should be honored. Laws regarding confidentiality of this care
are determined at the state level.
Consent and assent are guided by the ethical principles of autonomy, justice, and beneficence.
When a parent or guardian denies consent for medically necessary treatment, input from a team that
may include members of an ethics committee, psychiatrist, chaplain, psychologist, and/or palliative
care specialist should be sought before seeking legal intervention.
Critique
The child in the vignette has significant dehydration, electrolyte derangements, and hypoglycemia. Not
treating her would be harmful, and her mother has given consent for treatment. Oral rehydration is not
sufficient given the severity of her illness. Therefore, the best next management step is to continue with
intravenous therapy despite the child’s lack of assent. If there were no potential harm in delaying treatment,
it would be appropriate to postpone the procedure and elicit help (eg, from child life specialists) to obtain
assent from the child. There is no requirement to obtain consent from both parents when there is lack of
assent from a child.
The parent or guardian of a minor child has the right to give legal consent in most situations. If the parent is
also a minor, they are able to give legal consent for their child unless there is a different predetermined legal
arrangement. In all cases, if withholding emergency interventions could harm the minor child, it is not
necessary to await consent (per the Emergency Medical Treatment and Active Labor Act of 1986).
Consent and assent are guided by the ethical principles of autonomy, justice, and beneficence. Informed
consent includes 3 core elements:
A child gives assent when they express agreement to a procedure or testing. Children are able to give assent
as early as age 7 years, when concrete thinking develops. Information, including an explanation of the testing
or treatment, should be provided to the child in a developmentally appropriate manner, and the child’s
understanding of the procedure or treatment should be assessed. A child’s wish to forgo or delay treatment
or testing should not be honored if this choice would cause them harm, as in the case of the child in the
vignette. However, in such a case, acknowledgment of the child’s feelings and an apology can show the child
respect for their wishes. For example, if a child requires a vaccine and is saying “no,” it is reasonable to
explain, “I know it hurts. We are doing this to help keep you healthy. I am sorry it’s causing you pain.”
Adolescents are considered “mature minors” regarding decisions about their sexual health (sexually
transmitted illness treatment, contraceptive care, prenatal care), mental health, and substance use; parental
consent is generally not required. Levels of confidentiality of care for these issues is determined by state law.
An important goal of mature minor designation is to facilitate adolescents’ ability to seek care and receive
proper treatment in situations when asking their parents for consent may be a barrier.
Emancipated minor adolescents have the legal right to consent to all medical care without parental
permission or notification. Although state laws vary, criteria for emancipation typically include a minor who is
one of the following:
When a parent or guardian denies consent for medically necessary treatment, consultation should be sought
from a designated team that may include members of an ethics committee, psychiatrist, chaplain,
psychologist, and/or palliative care specialist before seeking legal intervention. When agreement on a safe
plan cannot be achieved, the state can mandate treatment. In these cases, the burden to the child should be
low and the likelihood of a positive outcome high. If there is uncertainty in the outcome, the overall prognosis
is poor, or the burden to the child is high, it may be reasonable for the parent or guardian to refuse
treatment.
Suggested Reading(s)
Katz AL, Webb SA: American Academy of Pediatrics Committee on Bioethics; et al. Informed consent in
decision-making in pediatric practice. Pediatrics. 2016;138(2):e20161485. doi:10.1542/peds.2016-1485
Ladd RE, Forman EN. Ethics for the pediatrician: pediatrician/patient/parent relationships. Pediatr Rev.
2010;31(9):e65-e67. doi:10.1542/pir.31-9-e65
Maslyanskaya S, Alderman EM. Confidentiality and consent in the care of the adolescent patient.
Pediatr Rev. 2019;40(10):508–516. doi:10.1542/pir.2018-0040
Wasserman JA, Navin MC, Vercler CJ. Pediatric assent and treating children over objection. Pediatrics.
2019;144(5):e20190382. doi:10.1542/peds.2019-0382
Content Domain
Ethics
A 10-month-old boy with conjunctivitis, rash, and 12 days of fever is admitted to the hospital for treatment of
Kawasaki disease. Echocardiography reveals a moderate proximal right coronary artery aneurysm and a
small to moderate proximal left anterior descending coronary artery aneurysm, no significant mitral
regurgitation, and normal ventricular function. He was treated with intravenous immunoglobulin and
intravenous steroids. Dual antiplatelet therapy was initiated with a combination of aspirin and clopidogrel
because of the size of the aneurysms. On hospital day 3, the pharmacy reaches out with a concern about the
dosing of clopidogrel; the boy has been receiving 1 mg/kg once daily instead of 0.2 mg/kg once daily.
A. change the medication order to the appropriate dose and continue current treatment
C. inform the family of the dosing error and discuss the case with a hematologist
D. inform the nursing staff of the dosing error and ask them to complete the appropriate
documentation
Correct answer is C
PREP Pearl(s)
Medical errors are common in pediatric patients and can be a source of significant morbidity and
mortality.
The cause of medication errors in pediatric patients is multifactorial; the risk of error is increased with
less frequently used drugs and drugs that require compounding, weight-based dosing
(miscalculations), and polypharmacy in medically complex patients.
Full disclosure of medical errors to the families of pediatric patients is expected.
Critique
The best next step in the management of the child in the vignette is to inform the family of the dosing error
and then determine what additional evaluation or change in treatment is necessary to ensure patient safety
and optimize further patient care.
Medical errors are common in all fields of medicine, including pediatrics. Between 1% and 3% of all pediatric
admissions are complicated by a medical error. Medication error, one of the most common forms of medical
error, is estimated to be the 14th leading cause of morbidity and mortality worldwide (World Health
Organization). Some studies suggest that children have a 3-fold higher risk of adverse drug events compared
with adults. There are multiple reasons for this increased risk. Two of the most common issues are the use of
weight-based dosing, which can lead to miscalculations, and the use of adult drug formulations. Medically
complex patients that require polypharmacy are at even higher risk.
Medication errors can occur at any point in the medication use process, including prescribing, preparing,
dispensing, administering, and monitoring. For the child in the vignette, there are likely multiple factors at
play. Clopidogrel is not a commonly used pediatric drug because it is unavailable in a liquid form; therefore,
for use in this child’s age group, it is prepared as a compounded solution. An additional complication is that
there are different recommended daily doses based on age (0.2 mg/kg for children younger than 24 months
and 1 mg/kg for children 24 months and older).
Interventions to reduce medical errors, including medication errors, focus on the following:
Disclosure of medical error, regardless of the outcome, is an ethical and moral obligation for practitioners.
However, the frequency of disclosure is variable among practitioners and institutions and, even when
performed, may be suboptimal. Practitioners often have little formalized training in disclosure and may fear
losing strong bonds with patients, facing blame, and even legal ramifications. These concerns can be
mitigated with the promotion of a culture of patient safety, which includes open discussion of errors, robust
reporting mechanisms, and practitioner training in disclosure. Error-specific interventions are often explored;
however, data suggest that focus is better placed on a higher-level systems approach rather than low-level
fixes to maximize change. Involving pharmacists in inpatient rounds and assigning unit-based pharmacists
reduce medication errors but require additional resources that may not be available in all settings.
Although it would be important to assess this child for evidence of bleeding, simply discontinuing his
pharmacotherapy would not address the root problem. Correcting the dosing error without informing the
family of the error is not appropriate and does not promote the overarching goal of patient safety. Most
institutions have policies in place to facilitate reporting of medical errors; it is important for practitioners to
take responsibility for their own error documentation.
Suggested Reading(s)
Koeck JA, Young NJ, Kontny U, Orlikowsky T, Bassler D, Eisert A. Interventions to reduce medication
dispensing, administration, and monitoring errors in pediatric professional healthcare settings: a
systematic review. Front Pediatr. 2021;9:633064. doi:10.3389/fped.2021.633064
Naseralallah LM, Hussain TA, Jaam M, Pawluk SA. Impact of pharmacist interventions on medication
errors in hospitalized pediatric patients: a systematic review and meta-analysis. Int J Clin Pharm.
2020;42(4):979-994. doi:10.1007/s11096-020-01034-z
Rinke ML, Bundy DG, Velasquez CA, et al. Interventions to reduce pediatric medication errors: a
systematic review. Pediatrics. 2014;134(2):338-360. doi:10.1542/peds.2013-3531
Shaikh SK, Cohen SP. Disclosure of medical errors. Pediatr Rev. 2020;41(1):45-47. doi:10.1542/pir.2018-
0228
Content Domain
Ethics
The correct answer is: inform the family of the dosing error and discuss the case with a hematologist
View Peer Results
AAP PREP 2024 - Question 67/267 Eye/Ear/Nose/Throat Question 1/3
A 12-year-old boy is being evaluated in the clinic for 2 weeks of left ear pain and 1 day of left ear discharge.
He returned from summer camp 1 week ago, where he participated in a variety of water sports. He has
otherwise been well without fever, rhinorrhea, or cough, and has a normal activity level. His vital signs and
physical examination findings, other than the left ear, are normal.
Correct answer is D
PREP Pearl(s)
Otitis externa presents with pain that worsens when traction is applied to the pinna.
Exposure to water, high temperatures, humidity, and injury to the ear canal are predisposing factors
for the development of otitis externa.
Topical combined antibiotic and steroid drops are the treatment of choice for otitis externa.
Critique
The boy in the vignette’s isolated left ear pain and discharge with a history of water exposure is most
consistent with a diagnosis of left otitis externa (OE). Of the response choices, the most likely finding on his
ear examination is pain with traction on the left pinna.
A bulging tympanic membrane with a purulent effusion is an expected finding of acute otitis media, which
usually occurs as a sequela of an upper respiratory infection (URI). The boy in the vignette has no recent
history of URI symptoms. Clear serous fluid behind the tympanic membrane is an expected finding of otitis
media with effusion (OME). Neither OME nor a foreign body in the ear canal is associated with significant pain
or made worse by traction on the pinna.
Otitis externa is an inflammatory process affecting the external auditory canal. It classically presents with
pain and a history of water exposure. Discharge from the ear canal, itching, a sense of fullness, and hearing
loss can also occur. Pain is elicited by traction on the ear pinna upward and away from the head.
The warm, dark, and often moist skin-lined ear canal is predisposed to bacterial or fungal infections.
Cerumen is a lipid-rich, acidic substance that protects the ear canal from water intrusion and bacterial and
fungal growth. Otitis externa occurs when this defense fails or the ear canal epithelium is damaged.
Risk factors for OE include swimming, perspiration, high temperature, humidity, and ear canal trauma (eg,
from cotton-tipped swabs). Systemic skin diseases (eg, atopic dermatitis, psoriasis) also predispose the ear
canal to OE.
The majority of cases of OE are caused by the bacteria Pseudomonas aeruginosa or Staphylococcus aureus.
Approximately 10% have a fungal etiology, most commonly Aspergillus. Fungal OE may result from disruption
of the natural flora of the ear canal secondary to antibiotic treatment of bacterial OE. In comparison to
bacterial OE, fungal OE tends to be less painful and more likely to present with pruritus and a sensation of a
fullness of the ear canal.
The initial treatment of OE consists of clearing debris from the ear canal. Once the canal is free of debris,
topical treatment with combined antibiotic and steroid drops is recommended. The recommended duration
of treatment varies from 3 days past resolution of symptoms up to a 14-day course when presenting
symptoms are severe. Referral to an otolaryngologist may be required when appropriate visualization
equipment is needed or for symptoms persisting beyond 14 days.
Suggested Reading(s)
American Academy of Pediatrics. Prevention of illnesses associated with recreational water use. In:
Kimberlin DW, Barnett ED, Lynfield R, Sawyer MH, eds. Red Book: 2021–2024 Report of the Committee
on Infectious Diseases. 32nd ed. American Academy of Pediatrics; 2021. Accessed February 23, 2022.
Red Book Online
McWilliams CJ, Smith CH, Goldman RD. Acute otitis externa in children. Can Fam Physician.
2012;58(11):1222-1224. https://pubmed.ncbi.nlm.nih.gov/23152458/
Melissa Long; Otitis Externa. Pediatr Rev. 2013; 34 (3): 143–144. doi:10.1542/pir.34.3.143
Sander R. Otitis externa: a practical guide to treatment and prevention. Am Fam Physician.
2001;63(5):927-936, 941-2. https://pubmed.ncbi.nlm.nih.gov/11261868/
Tan TQ. Otitis media and otitis externa. In: McInerny TK, Adam HM, Campbell DE, DeWitt TG, Foy JM,
Kamat DM, eds. American Academy of Pediatrics Textbook of Pediatric Care. American Academy of
Pediatrics; 2021:chap 305. Accessed February 23, 2022. Pediatric Care Online
Content Domain
Eye, Ear, Nose, and Throat
The correct answer is: pain with traction on the left pinna
A fully immunized 3-year-old boy with no significant medical history is seen in the office for a 2-day history of
right ear pain and fever. Physical examination reveals a red and bulging right tympanic membrane. There is a
boggy swelling behind the right ear, over the mastoid bone, that is causing his right pinna to protrude
relative to his left pinna. He is seen urgently by the otolaryngologist; a right myringotomy tube is placed, and
a sample of fluid from behind his tympanic membrane is sent for bacterial culture.
Of the following, the organism MOST likely responsible for this child’s condition is
A. Moraxella catarrhalis
B. Pseudomonas aeruginosa
C. Staphylococcus aureus
D. Streptococcus pneumoniae
Correct answer is D
PREP Pearl(s)
Acute mastoiditis is a complication of acute otitis media.
Acute mastoiditis is commonly caused by Streptococcus pneumoniae despite widespread routine
pneumococcal conjugate vaccination.
Management of acute mastoiditis typically includes urgent myringotomy tube placement and may
require mastoidectomy.
Critique
The child in the vignette has acute mastoiditis. The most common bacterial cause of this condition is
Streptococcus pneumoniae. Although the routine administration of pneumococcal conjugate vaccine has
greatly decreased the incidence of acute otitis media, pneumonia, and meningitis in children, the impact on
the rate of acute mastoiditis has been less dramatic. Streptococcus pneumoniae is the most frequently
cultured organism (most often, but not exclusively, serotypes that are not included in the pneumococcal
conjugate 13-valent vaccine).
Up to half of surgically cultured middle-ear specimens in the setting of acute mastoiditis will have no growth,
likely because of the presence of antibiotics. Another commonly cultured organism in acute mastoiditis is
Streptococcus pyogenes (group A Streptococcus [GAS]). Nontypeable Haemophilus influenzae,
Staphylococcus aureus, and Fusobacterium necrophorum are less commonly (<5% each) seen.
Treatment of acute mastoiditis should include intravenous antibiotics to cover S pneumoniae; if a β-lactam
antibiotic is chosen, it will also be active against GAS. Surgical intervention (simple myringotomy or, in more
severe cases, mastoidectomy) is often beneficial.
Suggested Reading(s)
Hernstadt H, Cheung A, Hurem D, et al. Changing epidemiology and predisposing factors for invasive
pneumococcal disease at two Australian tertiary hospitals. Pediatr Infect Dis J. 2020;39(1):1-6.
doi:10.1097/INF.0000000000002489
Kynion R. Mastoiditis. Pediatr Rev. 2018;39(5):267-269. doi:10.1542/pir.2017-0128
Macaj M, Perdochova L, Jakubikova J. Streptococcus pneumoniae as cause of acute otitis media (AOM)
in Slovak children in the pneumococcal conjugate vaccine era (2008-2019). Vaccine. 2023;41(2):452-459.
doi:10.1016/j.vaccine.2022.11.025
Sapir A, Ziv O, Leibovitz E, et al. Impact of the 13-valent pneumococcal conjugate vaccine (PCV13) on
acute mastoiditis in children in southern Israel: a 12-year retrospective comparative study (2005-2016).
Int J Pediatr Otorhinolaryngol. 2021;140:110485. doi:10.1016/j.ijporl.2020.110485
Tawfik KO, Ishman SL, Tabangin ME, Altaye M, Meinzen-Derr J, Choo DI. Pediatric acute mastoiditis in
the era of pneumococcal vaccination. Laryngoscope. 2018;128(6):1480-1485. doi:10.1002/lary.26968
Content Domain
Eye, Ear, Nose, and Throat
A 2-month-old infant is seen in the office for a health supervision visit. Her physical examination is significant
for the appearance of her left eye (Figure 1). The remainder of her examination findings are normal. She is
referred to an ophthalmologist for further evaluation but her caregivers fail to keep her appointment. After
several missed appointments, the girl is next seen in the office at 18 months of age.
© 2022 American Academy of Ophthalmology. Reprinted with permission from the American Academy of
Ophthalmology
Of the following, the MOST likely outcome of the delay in care for this infant’s condition is a/an
Correct answer is D
PREP Pearl(s)
Early detection and treatment of cataracts is necessary to prevent deprivation amblyopia and vision
loss.
Most congenital cataracts are idiopathic and require surgical intervention.
The red reflex evaluation is vital to identifying vision-threatening conditions in newborns and infants.
Critique
The infant in the vignette has a cataract of the left eye (Figure 1). The delay in treatment will most likely result
in permanent vision loss in the affected eye. A cataract is a clouding or opacity of the lens of the eye. Any
opacity greater than 3 mm is considered visually significant. Visual stimulation of the occipital lobe is required
for normal postnatal vision development. Anything that interferes with a young child’s vision can lead to
deprivation amblyopia and result in an uncorrectable loss of vision. Deprivation amblyopia can occur very
rapidly in a young infant. Amblyopia is more easily treated the earlier it is detected. The current
recommendation is that a unilateral congenital cataract be surgically treated by 6 weeks of age and bilateral
cataracts by 10 weeks of age. In some cases, a cataract may be removed as early as the first week after birth.
Any delay in treatment is likely to result in at least partial vision loss. Treatment may be ineffective by the
time a child reaches early school age. While hyperopia can be associated with anterior polar cataracts, this is
not the most likely outcome of this infant's condition.
Cataracts are the leading cause of blindness in children worldwide. In the United States, childhood blindness
attributable to cataracts ranges from 1 to 6/10,000 cases. About half of all congenital cataracts are idiopathic. The
remainder may have genetic, metabolic, or infectious causes. Unilateral cataracts are more likely to be caused by
primary ocular disorders, whereas bilateral cataracts often have a genetic etiology and are most commonly inherited
in an autosomal dominant pattern. Unilateral cataracts are rarely associated with systemic disease, except for those
associated with intrauterine infections (eg, rubella, cytomegalovirus, toxoplasmosis, syphilis). Congenital rubella
presents with unilateral cataracts in 20% to 30% of cases. Metabolic conditions, such as galactosemia,
hypothyroidism, and hypoglycemia, can result in congenital cataract formation. Unlike cataracts resulting from other
causes, cataracts associated with galactosemia may be reversible once infants are placed on an appropriate lactose-
free infant formula.
Infants with unilateral cataracts as an isolated finding may not require further evaluation. Children with
bilateral cataracts should be evaluated for an underlying cause; this evaluation should include testing for
TORCH infections, which stands for Toxoplasma gondii, other agents (eg, Treponema pallidum, varicella
zoster virus, parvovirus B19, HIV), rubella, cytomegalovirus, and herpes simplex virus). Testing should also
include urine analysis for reducing substances and organic acids, serum electrolytes, glucose, calcium,
phosphorus, plasma amino acids, and genetic sequencing to identify known genetic causes of cataracts.
Routine neonatal and infant eye examination is crucial in identifying infants with cataracts and preserving
vision. This should include observing the red reflex; this evaluation is vital for detecting potentially life-
threatening conditions (eg, retinoblastoma) as well as vision-threatening conditions (eg, cataracts). To assess
the red reflex, a light is shined toward the infant's eyes from a distance of about 18 inches in a darkened
room. The symmetry of the red reflex in terms of color, clarity, and intensity is more significant than the
actual color of the reflex. Children with cancer of the retina (retinoblastoma) will have a white pupillary light
reflex or leukocoria (Figure 2). A notably diminished reflex, a white reflex, dark spots within the reflex, or
asymmetry of the reflexes between the two eyes are indications for urgent referral to an ophthalmologist.
Suggested Reading(s)
Davenport KM, Patel AA. Cataracts. Pediatr Rev. 2011;32(2):82-83. doi:10.1542/pir.32-2-82
de Alba Campomanes AG, Binenbaum G. Eye and vision disorders. In: Gleason CA, Juul SE, eds. Avery's
Diseases of the Newborn. 10th ed. Elsevier; 2018:1536-1557.e4.
Wilson ME. Pediatric cataracts: overview. American Academy of Ophthalmology. November 11, 2015.
Accessed September 1, 2023. https://www.aao.org/disease-review/pediatric-cataracts-overview
Kemper AR, Wasserman R. Vision screening. In: McInerny TK, Adam HM, Campbell DE, DeWitt TG, Foy
JM, Kamat DM, eds. American Academy of Pediatrics; 2023. Accessed September 1, 2023. Pediatric Care
Online
Content Domain
Ear, eye, nose, and throat
The correct answer is: uncorrectable vision loss in the affected eye
A 3-year-old boy is brought to the emergency department with diarrhea and vomiting of 2 days’ duration. He
is refusing to drink and has had decreased urine output for 1 day. The boy’s heart rate is 140 beats/min,
respiratory rate 40 breaths/min, and blood pressure 70/40 mm Hg. He has dry mucous membranes, sunken
eyes, and a capillary refill time of 4 seconds. He is sleepy but arousable. The remainder of his physical
examination findings are unremarkable.
Laboratory data are shown:
pH 7.25
Correct answer is A
PREP Pearl(s)
Metabolic acidosis can be classified as elevated or normal anion gap acidosis; a normal anion gap is 8
mEq/L to 12 mEq/L.
An elevated anion gap acidosis results from overproduction of endogenous acids, ingestion of
exogenous acids, or decreased renal excretion of acids.
Normal anion gap acidosis results from a net loss of bicarbonate (HCO3−) from the gastrointestinal
tract or kidneys.
Critique
The boy in the vignette has clinical findings of severe dehydration. His low pH on capillary blood gas analysis
and low bicarbonate level support a diagnosis of metabolic acidosis with an elevated anion gap. The serum
anion gap is elevated secondary to increased unmeasured anions from lactic acidosis and uremia due to
hypovolemic shock.
Metabolic acidosis is defined as a low serum bicarbonate (HCO3−) concentration. Metabolic acidosis results
from either a net gain in hydrogen ions (H+), decreased renal H+ excretion, or a net loss of HCO3−. The first
step in the evaluation of metabolic acidosis is to calculate the serum anion gap, the mathematical difference
between the major cation and anion concentrations:
Anion gap (mEq/L) = sodium (Na+) − [chloride (Cl−) + bicarbonate (HCO3−)]
A normal anion gap ranges from 8 mEq/L to 12 mEq/L. The anion gap for the boy in the vignette is elevated at
20 mEq/L (136−[104+12]).
Metabolic acidosis is classified as elevated or normal anion gap acidosis (Table). An elevated anion gap
acidosis results from overproduction of endogenous acids (eg, lactic acidosis, ketoacidosis), ingestion of
exogenous acids (eg, methanol, ethylene glycol), or decreased renal excretion of acids (eg, uremia). Normal
anion gap acidosis results from a net loss of HCO3− from the gastrointestinal tract or kidneys. Children with
diarrhea usually have a normal anion gap acidosis owing to loss of HCO3− in stool. However, if diarrhea
results in severe hypovolemia, an elevated anion gap acidosis develops because of increased lactic acid
production and decreased acid excretion by the kidney. The boy in the vignette has a history (poor oral
intake, decreased urine output), physical examination findings (elevated heart rate, dry mucous membranes,
prolonged capillary refill time, sleepiness), and laboratory findings (elevated blood urea nitrogen and
creatinine levels) that indicate severe dehydration, consistent with his elevated anion gap metabolic acidosis.
Another common cause of normal anion gap acidosis is renal tubular acidosis (RTA), which results from renal
losses of bicarbonate (proximal RTA) or a renal tubule acidification defect (distal RTA).
The body’s response to the low pH of metabolic acidosis is tachypnea, hyperpnea, or both, occurring over
minutes to hours and resulting in a low PCO2 (compensatory respiratory alkalosis). Primary respiratory
alkalosis secondary to hyperventilation will have low PCO2, high pH (alkalosis), and a compensatory decrease
in HCO3− by renal excretion. Although the boy in the vignette has tachypnea, he also has an acidic pH, making
primary respiratory alkalosis unlikely.
Suggested Reading(s)
Hsu BS, Lakhani SA, Wilhelm M. Acid-base disorders. Pediatr Rev. 2016;37(9):361-369.
doi:10.1542/pir.2015-0093
Mahajan P, Felt JR. Fluids, electrolytes, and acid-base composition. In: McInerny TK, Adam HM,
Campbell DE, DeWitt TG, Foy JM, Kamat DM, eds. American Academy of Pediatrics Textbook of Pediatric
Care. American Academy of Pediatrics; 2023. Accessed September 1, 2023. Pediatric Care Online
Pelletier J, Gbadegesin R, Staples B. Renal tubular acidosis. Pediatr Rev. 2017;38(11):537-539.
doi:10.1542/pir.2016-0231
Content Domain
Fluids and Electrolytes
ABP Content Specification(s) / Content Area(s)
Understand the pulmonary mechanism for regulating acid-base physiology
Calculate an anion gap
The correct answer is: elevated anion gap acidosis secondary to severe dehydration
View Peer Results
Methanol Diarrhea
Uremia Renal tubular acidosis
Diabetic ketoacidosis Hyperalimentation
Paraldehyde Acetazolamide use
Inborn errors of metabolism Ureterosigmoid fistula
Iron Pancreatic fistula
Isoniazid
Lactic acidosis
Ethylene glycol
Ethanol
Salicylate use
Courtesy of A. Jain
AAP PREP 2024 - Question 71/267 Fluids and Electrolytes Question 2/4
A 4-week-old infant is seen in the office for evaluation of irritability, poor feeding, and an increased number
of wet diapers. He has not had a bowel movement in 5 days. The infant was born at term, after an uneventful
pregnancy, via emergency cesarean delivery owing to placental abruption. He required a 7-day stay in the
neonatal intensive care unit for transient tachypnea of the newborn.
On physical examination, the infant is crying and difficult to console. His temperature is 37.1 °C, his heart
rate is 165 beats/min, and his oxygen saturation is 98% in room air. The infant’s capillary refill time is 3
seconds, and his mucous membranes are dry. He has firm, subcutaneous nodules on his back, trunk, and
arms (Figure).
Reprinted with permission in Arango ML, MD, Shah AN. An infant with rash. Pediatr Rev. 2019;40(3):e11
The infant is sent to the emergency department, where laboratory evaluation reveals the following:
Hematocrit 35%
A. bacterial infection
B. diabetes mellitus
C. nonaccidental trauma
Correct answer is D
PREP Pearl(s)
Symptoms and signs of hypercalcemia include polyuria, dehydration, vomiting, and a change in mental
status.
Subcutaneous fat necrosis should be considered in infants with hypercalcemia who have had a
traumatic birth.
Immobilization is a rare cause of hypercalcemia, but should be considered in children and adolescents
with reduced mobility (eg,cerebral palsy or traumatic brain injury).
Critique
The infant in the vignette’s findings of severe hypercalcemia, history of a traumatic birth (placental
abruption), and a nodular rash (Figure) are consistent with a diagnosis of subcutaneous fat necrosis (SFN) of
the newborn. The infant’s symptoms and signs of irritability, poor feeding, polyuria, and dehydration are
consistent with hypercalcemia. Other symptoms and signs of hypercalcemia may include hypotonia, lethargy,
abdominal pain, and vomiting.
Subcutaneous fat necrosis is a rare, but potentially dangerous, condition associated with birth trauma, birth
asphyxia, and cooling therapy (sometimes referred to as cold panniculitis). Hypercalcemia occurs secondary
to granuloma formation in the damaged adipose tissue. This granulomatous tissue produces excess 1,25-
dihydroxyvitamin D, which results in increased calcium absorption from the gut and increased serum calcium
levels.
The first-line treatment for hypercalcemia is hydration, which increases calcium excretion through the kidney.
Any causative agent (eg, medication) should be removed. When hypercalcemia is refractory, glucocorticoids,
calcitonin, or bisphosphonate therapy can be considered.
Although polyuria and dehydration may suggest diabetes mellitus, the infant in the vignette has a normal
blood glucose level. Additionally, the infant is too young to have autoimmunity associated with type 1
diabetes, and neonatal diabetes, a monogenic form of diabetes, is very rare.
Nonaccidental trauma must be considered in an infant with abnormal skin markings, especially when mental
status changes are present. However, the skin findings and hypercalcemia in the infant in the vignette are not
consistent with nonaccidental trauma.
Sepsis due to bacterial infection must also be considered in an infant with a change in mental status and
dehydration. For the infant in the vignette, it would be appropriate to start antibiotics empirically while
awaiting results of a sepsis evaluation. However, sepsis would not explain the infant’s hypercalcemia or
polyuria.
Suggested Reading(s)
Ahamed I, Jauhar N. Hypercalcemia due to immobilization. J Endocrine Soc. 2021;5(suppl 1):A196.
doi:10.1210/jendso/bvab048.398
Arango ML, Shah AN. Visual diagnosis: an infant with rash and hypercalcemia. Pediatr Rev.
2019;40(3):e11-e13. doi:10.1542/pir.2017-0058
Mancilla E, Levine MA. Hypocalcemia, hypercalcemia, and hypercalciuria. In: McInerny TK, Adam HM,
Campbell DE, DeWitt TG, Foy JM, Kamat DM, eds. American Academy of Pediatrics Textbook of Pediatric
Care. American Academy of Pediatrics; 2023. Accessed September 1, 2023. Pediatric Care Online
Underland L, Markowitz M, Gensure R. Calcium and phosphate hormones: vitamin D, parathyroid
hormone, and fibroblast growth factor 23. Pediatr Rev. 2020;41(1):3-11. doi:10.1542/pir.2018-0065
Content Domain
Fluids and Electrolytes
A 9-year-old is brought to the emergency department for a 1-day history of nausea and headache. Two days
ago, they fell while skateboarding without a helmet. There was no loss of consciousness, vomiting, or visible
bleeding. The patient’s blood pressure is 106/62 mm Hg, heart rate is 78 beats/min, and respiratory rate is 14
breaths/min. They are sleepy but arousable. Their mucous membranes are moist. Capillary refill time is less
than 2 seconds. The patient has no focal neurological signs and the remainder of their examination findings
are unremarkable.
Urine Test
Nitrite Negative
Blood Negative
Protein Negative
A Decreased Decreased
B Decreased Increased
C Increased Normal
D Normal Increased
A. Row A
B. Row B
C. Row C
D. Row D
Correct answer is D
PREP Pearl(s)
Total body water is distributed in the extracellular fluid (one-third) and intracellular fluid
compartments (two-thirds).
An imbalance of water in the different body compartments can occur in various clinical scenarios.
Understanding the fluid shifts between the compartments helps with their management.
In syndrome of inappropriate antidiuretic hormone, extracellular fluid volume is normal or near
normal and the intracellular fluid volume is increased.
Critique
The child in the vignette has syndrome of inappropriate secretion of antidiuretic hormone (SIADH) secondary
to head trauma. His euvolemia, hyponatremia, hypoosmolality, inappropriately concentrated urine (high
urine osmolality and urine specific gravity), and high urine sodium concentration favor the diagnosis of
SIADH. Their hyponatremia is a result of antidiuretic hormone (ADH)–induced water retention and volume
expansion. Activation of secondary mechanisms that cause sodium and water loss in the urine restores
euvolemia. Thus, extracellular fluid (ECF) volume is normal or near normal, but the intracellular fluid (ICF)
volume is increased in SIADH. The increased ICF volume is the rationale for managing hyponatremia caused
by SIADH with fluid restriction and loop diuretics.
Total body water (TBW), composing approximately 60% of the body weight, is distributed in 2 compartments:
the ECF and the ICF. Two-thirds of the TBW is present in the ICF compartment and one-third in the ECF
compartment. The ECF compartment is further divided into interstitial or extravascular fluid (75%) and the
blood volume or intravascular fluid (25%). Hydrostatic pressure, oncotic pressure provided by albumin, and
capillary permeability are responsible for fluid movement between the ECF and ICF compartments and help
to maintain equilibrium.
An imbalance of water in the different body compartments can occur in various clinical scenarios (Table). An
understanding of the fluid shifts between the compartments helps with management.
Suggested Reading(s)
Jain A. Body fluid composition. Pediatr Rev. 2015;36(4):141-152. doi:10.1542/pir.36-4-141
Jones DP. Syndrome of inappropriate secretion of antidiuretic hormone and hyponatremia. Pediatr
Rev. 2018;39(1):27-35. doi:10.1542/pir.2016-0165
Mahajan P, Felt JR. Fluids, electrolytes, and acid-base composition. In: McInerny TK, Adam HM,
Campbell DE, DeWitt TG, Foy JM, Karnat DM, eds. American Academy of Pediatrics Textbook of
Pediatric Care. 2nd ed. American Academy of Pediatrics; 2017:chap 58. Accessed September 1, 2023.
Pediatric Care Online
Content Domain
Fluids and electrolytes
A 16-month-old child is seen in the clinic following 3 days of nonbloody diarrhea and fever. His last urine
output, a small volume 4 hours ago, was visibly concentrated. His weight today is 11.6 kg, which is decreased
from 12.4 kg at his well visit 3 weeks earlier. On physical examination, his blood pressure is 70/55 mm Hg,
heart rate is 135 beats/min, respiratory rate is 25 breaths/min, and temperature is 37.8 °C. He is alert but
appears tired. He has tacky mucous membranes and is not making tears. The remainder of his examination
findings are unremarkable.
A. 4%
B. 6%
C. 8%
D. 10%
Correct answer is B
PREP Pearl(s)
Dehydration is a common complication of illness in young children.
It is important to use physical examination and other objective findings (eg, prior weight) to determine
a child’s degree of dehydration to implement appropriate treatment.
Juice, broth, and sports drinks are not appropriate fluids for the oral rehydration of children with
moderate dehydration.
Critique
The best estimate is that the boy in the vignette is approximately 6% dehydrated. Dehydration due to illness,
particularly from diarrhea, is a common event among young children. Children in this age group are at
increased risk for dehydration for multiple reasons, including the following:
Infants' high body surface area to weight ratio results in higher fluid requirements.
Common childhood illnesses often include fever, vomiting, and diarrhea, each of which leads to excess
fluid loss.
Young children are dependent on others to meet their needs and cannot independently increase their
fluid intake.
Determination of a child’s degree of dehydration, classified as mild (3%-5% volume loss), moderate (6%-9%
volume loss), or severe (≥10% volume loss), is often difficult but is an important guide for management. On
physical examination, children with moderate dehydration can be expected to have dry mucous membranes,
decreased skin turgor, irritability, tachycardia, increased capillary refill time, and decreased urine output and
tear production. If a previous weight is available, calculation of the decrease in the child’s weight makes the
estimate of dehydration more accurate. The following formula is used to calculate a child's degree of
dehydration:
(12.4 − 11.6)
× 100 = 0.8/12.4 × 100 = 6.4% (moderately dehydrated)
12.4
When dehydration occurs, the body attempts to maintain water and electrolyte balance by shifting fluid from the
intracellular fluid (ICF) space to the extracellular fluid (ECF) space and by secreting antidiuretic hormone to decrease
the urinary excretion of fluid. In addition, receptors in the hypothalamus cause signals to be sent that increase thirst
and the desire for salt.
Isotonic (isonatremic) dehydration is the most common type; it results from a net loss of isotonic fluid
containing sodium and potassium. Sodium, the primary ECF cation, is lost from the body and through a shift
into the ICF compartment to balance the body’s potassium loss. Sodium will shift back from the ICF space to
the ECF space during rehydration as potassium levels are restored. The water deficit in dehydration comes
primarily from the ECF compartment; there is minimal net loss of fluid from the ICF space.
The management of moderate to severe dehydration generally involves the administration of 20 mL/kg
intravenous (IV) fluid boluses of normal saline followed by 1 to 2 days of IV hydration with a solution
containing sodium and glucose until the child is able to take adequate oral fluids. The volume of fluid and
electrolytes administered is determined by calculations that include the percentage of dehydration, predicted
additional losses, plus estimated maintenance fluid requirements.
When possible, the preferred method of rehydration for children with moderate, acute dehydration is to
rapidly restore ECF losses and then administer oral rehydration fluid (ORF) therapy. It is important to ensure
that an appropriate ORF is being used. Attempting rehydration with juice, broth, or sports drinks is not
recommended because these fluids do not contain the correct sodium to glucose ratio to promote salt and
water reabsorption in the gastrointestinal tract.
Suggested Reading(s)
Chandramohan G. Management of dehydration in children: fluid and electrolyte therapy. In: Berkowitz
CD, ed. Berkowitz’s Pediatrics: A Primary Care Approach. 6th ed. American Academy of Pediatricians;
2021.
Mahajan P. Dehydration. In: McInerny TK, Adam HM, Campbell DE, et al, eds. American Academy of
Pediatrics Textbook of Pediatric Care. American Academy of Pediatrics; 2016:chap 353. Pediatric Care
Online
Moritz ML, Ayus JC. Misconceptions in the treatment of dehydration in children. Pediatr Rev.
2016;37(7):e29–e31. doi:10.1542/pir.2016-003
Powers KS. Dehydration: isonatremic, hyponatremic, and hypernatremic recognition and management.
Pediatr Rev. 2015;36(7):274–285. doi:10.1542/pir.36-7-274
Content Domain
Fluids and Electrolytese
A 16-year-old adolescent boy is seen for evaluation of a 3-week history of jaundice and fatigue. He was
diagnosed with ulcerative colitis at age 12 years, which has been stable on mesalamine (5-aminosalicylic acid)
without a significant flare for the past several years. He has not been seen by a health care provider in more
than 1 year.
His physical examination reveals a tired-appearing boy. His weight is at the 35th percentile, his height is at
the 30th percentile, and his body mass index is at the 30th percentile for age. He has scleral icterus. His liver
edge is firm and palpable 2 cm below the right costal margin. His spleen is palpable 2 cm below the left costal
margin. There is no abdominal distention or peripheral edema. The remainder of his physical examination
findings are unremarkable.
Laboratory results are shown:
Of the following, the test MOST likely to reveal the etiology of this adolescent’s jaundice is
D. urinalysis
Correct answer is B
PREP Pearl(s)
Conjugated hyperbilirubinemia in older children and adolescents is a concerning finding. The
differential diagnosis is broad and includes hepatic and extrahepatic causes.
The initial evaluation of an adolescent patient with conjugated hyperbilirubinemia should include a
thorough history and physical examination, laboratory studies to assess for transaminitis and hepatic
synthetic function, and liver ultrasonography.
Critique
The adolescent boy in the vignette has conjugated hyperbilirubinemia, with findings suggestive of chronic
hepatobiliary disease (firm liver, splenomegaly, anemia, mild thrombocytopenia) and normal liver synthetic
function (albumin and prothrombin time). Given his history of ulcerative colitis, these findings are consistent
with autoimmune hepatitis and primary sclerosing cholangitis (PSC). Magnetic resonance
cholangiopancreatography (MRCP) images the biliary tree and is the test most likely to reveal the etiology of
this adolescent’s jaundice. Dilated and strictured biliary ducts (“beading”) seen on MRCP are consistent with
PSC.
Although acute hepatitis A infection could cause conjugated hyperbilirubinemia, signs of chronic liver disease
would not be present. Furthermore, detection of hepatitis A IgG implies immunity, either from previous
infection or vaccination. A red blood cell smear would be helpful in determining the presence of hemolysis in
the context of unconjugated hyperbilirubinemia. A urinalysis may detect findings supportive of conjugated
hyperbilirubinemia (urine bilirubin would be positive, urobilinogen negative), but it would not be a useful tool
in evaluating for causes of cholestasis.
Conjugated hyperbilirubinemia is always a concerning finding. The differential diagnosis for conjugated
hyperbilirubinemia in older children and adolescents includes the following:
Autoimmune liver disease and PSC
Benign recurrent intrahepatic cholestasis
Biliary tract obstruction
Choledocholithiasis
Choledochal cyst
Hydrops of the gallbladder
Drug-induced liver injury
Infectious hepatitis
Oncologic disease (tumor, hepatic sinusoidal obstruction syndrome)
Wilson disease
Initial evaluation of an adolescent patient with jaundice should include the following:
History
Onset of jaundice (acute, chronic)
Use of hepatotoxic medications, drugs, supplements (eg, acetaminophen, minocycline)
Previous episodes of jaundice
Other symptoms (fatigue, weight loss, rashes, abdominal distention, abdominal pain, nausea,
fever)
Physical examination
Liver size and texture (soft, firm, hard)
Signs of portal hypertension or cirrhosis (eg, splenomegaly, palmar erythema, spider angioma,
caput medusae, ascites)
Laboratory studies
Serum aminotransferases, conjugated and unconjugated bilirubin levels
Markers of hepatic synthetic function (albumin level, prothrombin time)
Imaging
Ultrasonography of the abdomen
Other studies tailored to the suspected diagnosis
Infectious hepatitis
Hepatitis (A, B, and C)
Epstein-Barr virus, cytomegalovirus
Adenovirus, influenza, varicella, herpes simplex virus, parvovirus, and enterovirus
Autoimmune hepatitis
Antinuclear antibody, anti-smooth muscle antibody, anti-liver kidney microsomal antibody
Liver biopsy
Primary sclerosing cholangitis
Magnetic resonance cholangiopancreatography
Endoscopic retrograde cholangiopancreatography
Liver biopsy
Drug-induced liver injury
Liver biopsy
Wilson disease
Serum ceruloplasmin level
Ophthalmologic examination (Kayser-Fleischer rings)
24-hour urine copper
Liver biopsy
Benign recurrent intrahepatic cholestasis
Liver biopsy
The boy in the vignette was found to have PSC, confirmed by findings on liver biopsy and MRCP. The
treatment of PSC includes immunosuppressive medications (eg, prednisone, azathioprine). Liver transplant
may be indicated for end-stage liver disease.
Suggested Reading(s)
Brumbaugh D, Mack C. Conjugated hyperbilirubinemia in children. Pediatr Rev. 2012;33(7):291-302.
doi:10.1542/pir.33-7-291
Meili-Vergani G, Vergani D, Baumann U, et al. Diagnosis and management of pediatric autoimmune
liver disease: ESPGHAN Hepatology Committee position statement. J Pediatr Gastroenterol Nutr.
2018;66(2):345-360. doi:10.1097/MPG.0000000000001801
Pan DH, Rivas Y. 2016. Jaundice. In: McInerny TK, Adam HM, Campbell DE, DeWitt TG, Foy JM, Kamat
DM, eds. American Academy of Pediatrics Textbook of Pediatric Care. American Academy of Pediatrics;
2023. Accessed September 1, 2023. Pediatric Care Online
Content Domain
Gastroenterology
A 14-month-old boy with a history of failure to thrive that has lasted several months is evaluated in the
emergency department for 1 week of vomiting and upper leg rash and 2 days of swelling of his eyelids and
extremities. His emesis is nonbilious and nonbloody and occurs once daily. The boy’s diet consists of almond
milk (16 oz daily) and small amounts of mashed avocados and bananas. He receives outpatient speech-
language and physical therapy services. Few growth data are available; however, there is a documented
weight loss of 6 oz (0.17 kg) over the past 6 weeks.
On physical examination, the boy is irritable but consolable. His weight is 8.4 kg (4th percentile for age),
length is 71 cm (less than 1st percentile for age), and weight for length is at the 30th percentile for age. His
abdomen is distended but soft, without hepatosplenomegaly. There is periorbital edema and symmetrical
pitting edema of his extremities. Red/orange erosions coalescing into larger plaques are present over his
upper medial thighs and diapered area.
C. outpatient monitoring with daily weight checks and increasing his caloric intake by 50%
D. outpatient monitoring with weekly weight checks and administration of an appetite stimulant
Correct answer is A
PREP Pearl(s)
Children with malnutrition who are at high risk of experiencing refeeding syndrome (eg, kwashiorkor)
should be carefully monitored in an inpatient setting while feedings are increased.
In children at risk of experiencing refeeding syndrome, nutritional support should be introduced at a
reduced rate (40%-50% of estimated caloric needs).
In children at risk of experiencing refeeding syndrome, frequent monitoring of glucose, magnesium,
phosphate, potassium, and sodium concentrations is recommended as feedings are advanced.
Critique
The boy in the vignette has evidence of severe malnutrition with edema (kwashiorkor or protein
malnutrition); he is at high risk of experiencing refeeding syndrome. The best next step in his treatment is
hospitalization and administration of 50% of his estimated calorie needs. Immediate administration of
feedings that meet his total nutritional requirement is more likely to result in refeeding syndrome and is
therefore not recommended. The boy was found to have zinc deficiency leading to acrodermatitis
enteropathica; his rash improved with zinc supplementation. His vomiting resolved with improved nutritional
status, after evaluation with ultrasonography, laboratory testing, and esophagogastroduodenoscopy yielded
normal findings.
Refeeding syndrome—metabolic and clinical changes that may occur when a malnourished patient
undergoes aggressive nutritional rehabilitation—can carry significant risks. Electrolyte abnormalities
(hypophosphatemia, hypokalemia, and hypomagnesemia) can result in cardiac arrhythmias and even death if
electrolytes are not monitored and abnormalities treated carefully. Fluid overload, hyperglycemia, and
thiamine deficiency can also occur. Table 1 lists risk factors for refeeding syndrome, and Table 2 outlines the
clinical manifestations of refeeding syndrome.
To help prevent refeeding syndrome, initial refeeding should be limited to between 40% and 50% of
estimated caloric needs, with slow advancements over 3 to 7 days to a goal of 100% of estimated caloric
needs. Nutrition may be delivered orally, via nasogastric tube, or parenterally.
An interprofessional team, which may include a physician, nurse, dietitian, occupational and/or speech
therapists, social worker, child life specialist, and psychologist, provides optimal monitoring for and
treatment of refeeding syndrome. Outpatient monitoring (even with daily weights) is not recommended for
children and adolescents at high risk of experiencing refeeding syndrome.
Monitoring and treatment of children and adolescents at risk of experiencing refeeding syndrome includes
the following:
Suggested Reading(s)
Da Silva JSV, Seres DS, Sabino K, et al; Parenteral Nutrition Safety and Clinical Practice Committees,
American Society for Parenteral and Enteral Nutrition. ASPEN consensus recommendations for
refeeding syndrome. Nutr Clin Pract. 2020;35(2):179-195. doi:10.1002/ncp.10474
Pulcini CD, Zettle S, Srinath A. Refeeding syndrome. Pediatr Rev. 2016;37(12):516-523.
doi:10.1542/pir.2015-0152
Racine AD. Failure to thrive: pediatric undernutrition. In: McInerny TK, Adam HM, Campbell DE, Foy JM,
Kamat DM, eds. American Academy of Pediatrics Textbook of Pediatric Care. 2nd ed. American
Academy of Pediatrics; 2016:chap 149. Pediatric Care Online
Content Domain
Gastroenterology
Reprinted with permission from Pulcini CD, Zettle S, Srinath A. Refeeding syndrome.
Pediatr Rev. 2016;37(12):517.
Hypophosphatemia* Sudden death, Dyspnea, Weakness, Hemolysis, Confusion, delirium, Metabolic acidosis,
arrhythmia, respiratory failure, myalgia, thrombocytopenia, paresthesias, insulin resistance,
heart failure, impaired rhabdomyolysis leukocyte paralysis, seizures, acute tubular
hypotension, diaphragm dysfunction hallucinations, necrosis
shock function tetany, seizures,
coma
Hypokalemia Arrhythmia Respiratory failure Weakness, Nausea, vomiting, Paralysis Death
rhabdomyolysis, constipation
muscle necrosis
Hypomagnesemia Arrhythmia Weakness Nausea, vomiting, Tremor, tetany, Refractory
diarrhea seizures, altered hypokalemia and
mental status, hypocalcemia,
coma death
Vitamin/thiamine Encephalopathy Lactic acidosis,
deficiency death
Hyperglycemia Hypotension Respiratory failure Weakness, Nausea, vomiting, Paralysis Infection, death
rhabdomyolysis, constipation
muscle necrosis
Fluid overload Heart failure Edema Death
Trace element Arrhythmia, Encephalopathy Metabolic acidosis
deficiency heart failure
*
Most commonly reported mechanism of refeeding syndrome.
Reprinted with permission from Pulcini CD, Zettle S, Srinath A. Refeeding syndrome. Pediatr Rev. 2016;37(12):519.
Hypophosphatemia Mild: 2.3-2.7 mg/dL 0.3-0.6 mmol/kg per day 0.3-0.6 mmol/kg Single dose: 15 mmol/kg (IV) Over 6-12 hours, measure
(0.74-0.87 mmol/L) per day POa Daily: 1.5 mmol/kg (IV) phosphate 2-4 hours after
Moderate: 1.5-2.2 mg/dL 0.08-0.24 mmol/kg infusion completion
(0.48-0.71 mmol/L) IV over 6-12 hoursa
Severe: <1.5 mg/dL (0.48 mmol/L)
Hypomagnesemia Mild/Moderate: 1.0-1.8 mEq/L 0.2 mmol/kg per day 25-50 mg/kg per Single dose: 2,000 mg Over 4 hours
(0.50-0.90 mmol/L) PO dose (0.2-0.4 (16 mEq) (oral)
Severe: <1.0 mEq/L (0.50 mmol/L) mEq/kg per dose)a
Hypokalemia Mild/Moderate: 2.5-3.4 mEq/L 1-2 mmol/kg per day 0.3-0.5 mEq/kg per Single dose: 30 mEq/dose (IV) Over at least 1 hour,
(2.5-3.4 mmol/L) doseb (IV) measure potassium
Severe: <2.5 mEq/L (2.5 mmol/L) 2 hours after infusion
completion
Abbreviations: IV, intravenous; PO, oral.
a
50% less for impaired renal function.
b
Urine output must be 0.5 mL/kg per hour or greater, continuous monitoring recommended while replacing potassium.
Reprinted with permission from Pulcini CD, Zettle S, Srinath A. Refeeding syndrome. Pediatr Rev. 2016;37(12):520.
•
AAP PREP 2024 - Question 76/267 Gastroenterology Question 3/10
An 8-year-old boy with no notable medical history is undergoing evaluation for a 1-week history of
intermittent fatigue and fever, as well as a 2-day history of jaundice. He has taken 2 doses of acetaminophen
according to package directions over the past 2 days; he has taken no other supplements or medications.
The boy appears tired but not in distress. His temperature is 39.8 °C, blood pressure is 105/64 mm Hg, heart
rate is 110 beats/min, respiratory rate is 20 breaths/min, weight is 33.8 kg (90th percentile for age), height is
133 cm (60th percentile for age), and body mass index is 19.4 kg/m2 (90th percentile for age). His sclera are
icteric and his skin is jaundiced. There is bilateral nontender posterior cervical lymphadenopathy. His
abdomen is soft and nondistended, with a soft liver edge palpable 2 cm below the right costal margin and
spleen palpable 3 cm below the left costal margin. The remainder of his physical examination findings are
unremarkable.
ɣ-Glutamyltransferase 20 U/L
Lipase 79 U/L
Abdominal ultrasonography reveals a heterogeneous and enlarged liver (measuring 16 cm) and an enlarged
spleen (measuring 15 cm). The ultrasonographic findings are otherwise normal, including a normal-
appearing biliary tree.
Of the following, the test MOST likely to reveal this boy’s diagnosis is
Correct answer is A
PREP Pearl(s)
Hepatomegaly (liver span >4.5-5 cm in infants or >6-8 cm in older children) can occur because of
infectious etiologies, infiltrative processes, storage disorders, vascular congestion, and biliary
obstruction.
Splenomegaly and hepatomegaly together may occur with advanced liver disease or with infectious,
infiltrative, or storage diseases.
Critique
The most likely diagnosis for the boy described in the vignette, who has acute onset of systemic symptoms,
posterior cervical lymphadenopathy, hepatosplenomegaly, transaminitis, and conjugated hyperbilirubinemia,
is Epstein-Barr virus infection. Of the response choices, the test most likely to reveal this diagnosis is an
antiviral capsid antigen IgM titer, which would be elevated.
An elevated creatinine kinase value in the context of hepatomegaly is associated with glycogen storage
disease; this diagnosis is unlikely in an otherwise healthy boy without hypoglycemia. An elevated mean
corpuscular hemoglobin concentration is seen in hereditary spherocytosis; additional findings include
splenomegaly, hemolysis, and unconjugated hyperbilirubinemia. This boy has conjugated hyperbilirubinemia
and a normal hemoglobin value. Magnetic resonance cholangiopancreatography is a diagnostic test for
biliary tract disease (eg, primary sclerosing cholangitis, choledocholithiasis), an unlikely diagnosis for this boy
with normal alkaline phosphatase and ɣ-glutamyltransferase values.
On physical examination, liver size is assessed using percussion to mark the upper and lower margins. A
normal liver span is 4.5 to 5 cm for a neonate and 6 to 8 cm for children by age 12 years. A liver edge palpable
below the right costal margin (>3.5 cm in newborns and >2 cm in children) raises concern regarding
hepatomegaly. However, displacement of the liver inferiorly (eg, caused by pneumothorax; lung
hyperinflation due to asthma, bronchiolitis, or pneumonitis; congenital diaphragmatic hernia) can result in
similar examination findings.
Causes of hepatomegaly are listed in the Table. The presence of splenomegaly in an infant or child with
hepatomegaly may indicate more advanced liver disease (eg, cirrhosis with portal hypertension). However,
infections and infiltrative, neoplastic, and storage diseases may manifest similarly.
The initial diagnostic evaluation of the infant or child with hepatomegaly should include measurement of liver
enzymes (alanine aminotransferase, aspartate aminotransferase, alkaline phosphatase, ɣ-
glutamyltransferase), fractionated bilirubin, markers of liver function (prothrombin time, albumin, total
protein, ammonia), complete blood count, electrolytes, and glucose. Abdominal ultrasonography should be
performed, with doppler flow if vascular etiologies are suspected. Figure 1 and Figure 2 describe diagnostic
algorithms for neonates and children with hepatomegaly.
Suggested Reading(s)
Adam HM. Hepatomegaly. Point-of-Care Quick Reference. Pediatric Care Online. American Academy of
Pediatrics. 2020. Accessed March 7, 2023. Pediatric Care Online
Ozuah PO, Reznik M. Hepatomegaly. In: McInerny TK, Adam HM, Campbell DE, Foy JM, Kamat DM, eds.
American Academy of Pediatrics Textbook of Pediatric Care. 2nd ed. American Academy of
Pediatrics;2023. Accessed September 1, , 2023. Pediatric Care Online
Wolf AD, Lavine JE. Hepatomegaly in neonates and children. Pediatr Rev. 2000;21(9):303-310.
doi:10.1542/pir.21-9-303
Content Domain
Gastroenterology
Reprinted with permission form Wolf AD, Lavine JE. Hepatomegaly in neonates and children.
Pediatr Rev. 2000;21(9):308.
Reprinted with permission form Wolf AD, Lavine JE. Hepatomegaly in neonates and children.
Pediatr Rev. 2000;21(9):309.
FIGURE 2: Diagnostic algorithm for hepatomegaly in a child.
AAP PREP 2024 - Question 77/267 Gastroenterology Question 4/10
A 7-year-old child is seen for evaluation of an 8-month history of regurgitation, which occurs frequently after
eating. The child regurgitates then either rechews and swallows or spits out stomach contents. The
regurgitation does not occur at night. There is no associated nausea or retching. A 4-week trial of proton
pump inhibitor therapy did not improve his symptoms. The child’s weight, height, and body mass index are at
the 50th percentile for age. His physical examination findings are normal.
Of the following, the BEST treatment option for this child is (a)
B. diaphragmatic breathing
C. gluten-free diet
Correct answer is B
PREP Pearl(s)
Rumination syndrome should be suspected in children with persistent or recurrent regurgitation of
recently ingested food into the mouth with subsequent spitting or remastication and swallowing.
Treatment options for rumination syndrome include diaphragmatic breathing, cognitive behavioral
therapy, and relaxation training.
Children with recurrent regurgitation associated with dysphagia (food sticking sensation), odynophagia
(painful swallowing), vomiting (particularly if forceful or contains biliary contents or blood), weight loss,
abdominal distension, or rectal bleeding should undergo further evaluation.
Critique
The child in the vignette has recurrent regurgitation after eating, with both spitting and chewing and re-
swallowing stomach contents. There is no associated retching or nausea. This constellation of symptoms is
consistent with rumination syndrome, a functional gastroduodenal disorder. Diaphragmatic breathing is the
first-line treatment for rumination syndrome.
The diagnosis of rumination syndrome is determined by clinical history using Rome IV criteria. These criteria
were developed by the Rome Foundation to define functional gastrointestinal disorders in adult and pediatric
patients. The Rome IV criteria for rumination syndrome are as follows:
Diagnostic criteria (must include the presence of both criteria for at least 3 months with
symptom onset at least 6 months before diagnosis; symptoms are not explained by another
medical condition)
Persistent or recurrent regurgitation of recently ingested food into the mouth with
subsequent spitting or remastication and swallowing
Regurgitation is not preceded by retching
Supportive criteria
Children with recurrent regurgitation who are exhibiting concerning signs and/or symptoms should undergo
further evaluation to determine the cause. Concerning signs/symptoms include dysphagia (food sticking
sensation), odynophagia (painful swallowing), vomiting (particularly vomit that is forceful or contains biliary
contents or blood), weight loss, abdominal distension, and rectal bleeding.
The primary treatment for rumination syndrome is diaphragmatic breathing. Adjunctive therapies include
cognitive behavioral therapy (eg, substituting other self-soothing techniques, exposure therapy, developing
strategies to overcome the urge to ruminate), gum chewing, biofeedback, and relaxation training. There are
limited data in favor of recommending pharmacologic treatment of rumination syndrome. Baclofen may be
considered to treat rumination that is refractory to behavioral interventions. Other medications (eg, tricyclic
antidepressants, proton pump inhibitors, histamine-2 receptor antagonists, prokinetics, antiemetics) have
not been found to be effective. Increasing carbonated beverage intake may exacerbate this child’s symptoms.
A gluten-free diet (treatment for celiac disease) is not recommended for the management of rumination
syndrome.
Suggested Reading(s)
Martinez M, Rathod S, Friesen HJ, Rosen JM, Friesen CA, Schurman JV. Rumination syndrome in children
and adolescents: a mini review. Front Pediatr. 2021;9:709326. doi:10.3389/fped.2021.709326
Murray HB, Juarascio AS, Di Lorenzo C, Drossman DA, Thomas JJ. Diagnosis and treatment of
rumination syndrome: a critical review. Am J Gastroenterol. 2019;114(4):562-578.
doi:10.14309/ajg.0000000000000060
Rome IV Criteria, Appendix A: Rome IV diagnostic criteria for FGIDs, B. Gastroduodenal disorders, B4.
Rumination syndrome. Rome Foundation. Accessed September 1, 2023.
https://theromefoundation.org/rome-iv/rome-iv-criteria
Content Domain
Gastroenterology
A 4-year-old girl with Alagille syndrome, chronic liver disease, and cholestasis (currently listed for a liver
transplant) is undergoing follow-up evaluation after surgical repair of a femur fracture 1 week ago. Her
fracture was diagnosed after evaluation of 2 days of progressive right leg pain, abnormal appearance of her
right hip, and refusal to bear weight on her right leg. A radiograph revealed a fracture of the right proximal
femoral diaphysis with posteromedial angulation and diffuse osseous demineralization (Figure 1). She had 2
other leg fractures in the past year.
Courtesy of J. Sullivan
The girl appears thin and jaundiced but in no acute distress. She has scleral icterus, deep-set eyes, and a
small, pointed chin. Abdominal examination is limited by a spica cast. Her extremities are thin and well
perfused, with multiple xanthomas present. The remainder of her physical examination findings are
unremarkable.
Of the following, the MOST likely laboratory finding for this girl is a
Correct answer is D
PREP Pearl(s)
Children with cholestasis are at risk of developing malnutrition owing to several factors, including poor
intake, increased energy expenditure, nutrient maldigestion, and nutrient malabsorption.
Children with cholestasis are at risk of experiencing deficiencies in fat-soluble vitamins (A, D, E, K).
Children with cholestasis should undergo regular evaluation to detect nutritional deficiencies, including
anthropometric measurements, a focused history taking and physical examination, and laboratory
assessments.
Critique
The girl in the vignette has Alagille syndrome with resultant cholestasis (conjugated hyperbilirubinemia as a
result of hepatic dysfunction and reduced bile flow), placing her at risk of experiencing substantial
deficiencies in fat-soluble vitamins (eg, vitamin D). Vitamin D deficiency results in decreased intestinal calcium
absorption, hypocalcemia, and increased parathyroid hormone secretion (Figure 2). Increased parathyroid
hormone values result in bone demineralization owing to calcium and phosphorus release from bone and
phosphaturia (decreased phosphorus reabsorption in the kidney). Initially, with vitamin D deficiency, serum
phosphorus values are low and calcium values are often normal. In a more severe state, both calcium and
phosphorus values are low.
This child is at risk of experiencing deficiencies in fat-soluble vitamins (A, D, E, and K). With vitamin D
deficiency, hypocalcemia is expected rather than an elevated calcium value. As retinol (vitamin A) is a fat-
soluble vitamin, this child would not be expected to have an elevated value. Cyanocobalamin (vitamin B12)
deficiency is not associated with bone disease and is more likely to occur in children with short gut syndrome,
a history of intestinal surgery (eg, gastric bypass, ileal resection in Crohn disease), conditions associated with
ileal inflammation (Crohn disease), and low vitamin B12 intake (eg, individuals following vegetarian or vegan
diets).
Children with chronic liver disease are at high risk of experiencing malnutrition owing to several factors,
including the following:
Poor intake (early satiety, chronic nausea, delayed gastric emptying, organomegaly)
Increased nutrient metabolism (increased metabolic needs, insulin resistance, growth hormone resistance)
Maldigestion (decreased bile flow and resultant fat maldigestion)
Malabsorption (enteropathy from portal hypertension)
Nutritional status must be carefully monitored in children with cholestasis. Anthropometric assessment
(weight, length or height, mid-upper arm circumference, triceps skin fold) should occur at least every 3
months. The child’s access to healthy foods, vitamins, and supplements should also be evaluated.
Macronutrient (fat, protein, carbohydrate) and micronutrient (vitamins, minerals) status should be assessed
regularly. Generally, children with chronic liver disease and cholestasis should consume 130% of calorie
requirements for age. If there is significant cholestasis, fat requirements may be increased and there is an
increased risk of experiencing essential fatty acid deficiency. Protein needs may be increased (130%-150% of
age requirements), and carbohydrates should make up 40% to 60% of total caloric intake. Fat-soluble vitamin
deficiencies are common in children with cholestasis. Prothrombin time and serum vitamin A, D, and E values
should be measured every 3 to 6 months, with supplementation based on the laboratory test results. Iron
and zinc deficiency can also occur in children with cholestasis and should be treated when present. Signs and
symptoms of pertinent nutritional deficiency states are outlined in the Table.
Suggested Reading(s)
Diab L, Krebs NF. Vitamin excess and deficiency. Pediatr Rev. 2018;39(4):161-179. doi:10.1542/pir.2016-
0068
American Academy of Pediatrics. Liver disease. In: Kleinman RE, Greer FR, eds. Pediatric Nutrition. 8th
ed. American Academy of Pediatrics; 2020:chap 43. Accessed March 7, 2023. Pediatric Nutrition Online
Lauer B, Spector N. Vitamins. Pediatr Rev. 2012;33(8):339-352.doi:10.1542/pir.33-8-339
Mouzaki M, Bronsky J, Gupte G, et al. Nutrition support of children with chronic liver diseases: North
American Society for Pediatric Gastroenterology, Hepatology, and Nutrition and the European Society
for Pediatric Gastroenterology, Hepatology, and Nutrition. J Pediatr Gastroenterol Nutr. 2019;69(4):498-
511. doi:10.1097/MPG.0000000000002443
Content Domain
Gastroenterology
An 8-year-old girl is undergoing evaluation for a 6-month history of recurrent abdominal pain. She has had 6
episodes, each lasting 1 to 2 days. The episodes begin with generalized abdominal pain and nausea, followed
by nonbilious, nonbloody emesis. She is completely well between episodes. During one episode, she was
admitted to the hospital overnight for dehydration and treated with intravenous fluids. Findings of
abdominal ultrasonography at that time were normal. She has daily soft stools. The girl has no other notable
medical or surgical history. Her mother has a history of migraine headaches and Barrett esophagus; the
family history is otherwise unremarkable.
Physical examination reveals a healthy-appearing girl with a weight of 35 kg (90th percentile), height of 131
cm (60th percentile), and body mass index of 20 kg/m2 (90th percentile). The remainder of her physical
examination findings are normal.
Of the following, the BEST next step in this girl’s care is to prescribe
A. bisacodyl
B. cyproheptadine
C. dicyclomine
D. lansoprazole
Correct answer is B
PREP Pearl(s)
Chronic and recurrent abdominal pain is common in children; it is most often caused by a functional
gastrointestinal disorder.
Abdominal migraine, a migraine variant, is diagnosed on the basis of clinical criteria; it may be treated
with lifestyle modifications, pharmacologic therapies (abortive and prophylactic), and complementary
therapies.
The presence of concerning symptoms and signs (eg, blood in the stool, weight loss) should prompt
further testing in children with chronic and recurrent abdominal pain.
Critique
The girl in the vignette has a pattern of recurrent abdominal pain and symptoms (nausea, vomiting)
consistent with abdominal migraine. Her mother’s history of migraine headaches supports this diagnosis. Of
the response choices, cyproheptadine, a migraine prophylaxis medication, is the best next management
step.
There are no medications approved by the US Food and Drug Administration for treatment of abdominal
migraine in children. In practice, however, abdominal migraine is treated similarly to migraine headache.
Commonly used treatments include the following:
Lifestyle modifications (eg, ensuring adequate rest, good sleep hygiene, adequate hydration)
Avoiding dietary triggers
Abortive pharmacotherapy (eg, nonsteroidal anti-inflammatory drugs, antiemetic medications,
triptans)
Intravenous hydration
Prophylactic pharmacotherapy (eg, cyproheptadine, propranolol, tricyclic antidepressants)
Complementary therapies (eg, cognitive behavioral therapy, hypnosis)
Chronic and recurrent abdominal pain is common in children; 1 in 10 children visit health care providers for
evaluation of these symptoms. Functional gastrointestinal disorders (eg, irritable bowel syndrome, abdominal
migraine, functional dyspepsia, functional abdominal pain) are the most common causes. Testing is often
unnecessary for the diagnosis and treatment of these disorders. Testing is generally indicated when
concerning symptoms and signs are present (Table 1). Laboratory, endoscopic, and/or radiologic evaluation
should be tailored to the likely diagnoses according to the history and physical examination findings (Table 2).
Suggested Reading(s)
Azmy D, Qualia C. Review of abdominal migraine in children. Gastroenterol Hepatol (N Y).
2020;16(12):632-639. PMCID: PMC8132691
Hyman PE. Chronic and recurrent abdominal pain. Pediatr Rev. 2016;37(9)377-390.
doi:10.1542/pir.2015-0169
Klein J, Koch T. Headache in children. Pediatr Rev. 2020;41(4):159-171. doi:10.1542/pir.2017-0012
Rome IV criteria, Appendix A. Rome IV diagnostic criteria for FGIDs, H. Childhood functional GI
disorders: child/adolescent, H2c. Abdominal migraine. The Rome Foundation. Accessed September 1,
2023. https://theromefoundation.org/rome-iv/rome-iv-criteria
Samras N. Abdominal pain. Pediatric Care Online. American Academy of Pediatrics. Accessed
September 1, 2023. https://publications.aap.org/pediatriccare/article-
abstract/doi/10.1542/aap.ppcqr.396048/1527/Abdominal-Pain?redirectedFrom=fulltext
Content Domain
Gastroenterology
A 10-year-old boy is seen for a health supervision visit. He has a history of biliary atresia and underwent a
Kasai portoenterostomy at 72 days of age. Other than an episode of cholangitis as an infant, he has been well
and growing and developing appropriately. His stools remain normally pigmented.
On physical examination, the boy’s weight is at the 95th percentile, height is at the 65th percentile, and body
mass index is at the 97th percentile for age. He is breathing comfortably in room air. His sclera are anicteric.
His abdomen is soft with well-healed scars in the right upper quadrant, a firm liver edge is palpable at the
right costal margin, and his spleen is palpable 2 cm below the left costal margin. There are erythematous
plaques with scaling on the flexor surface of his arms. The remainder of the boy’s physical examination
findings are normal. His family is concerned about his risk of experiencing gastrointestinal bleeding.
Of the following, the physical examination finding that places the boy at HIGHEST risk of experiencing this
complication is
A. abdominal scarring
B. arm rash
C. obesity
D. palpable spleen
Correct answer is D
PREP Pearl(s)
Physical examination findings suggestive of portal hypertension include splenomegaly, caput medusa,
spider angiomata, telangiectasias, palmar erythema, ascites, jaundice, easy bruising/bleeding,
malnutrition, and mental status changes.
In a child with underlying liver disease, thrombocytopenia (with or without anemia and leukopenia)
should raise suspicion regarding portal hypertension.
Guided by severity and associated complications, the treatment of portal hypertension includes
management of complications (gastrointestinal hemorrhage and ascites), surgical shunts (portal
decompression), and liver transplantation.
Critique
The splenomegaly (palpable spleen) in the boy in the vignette raises concern regarding portal hypertension, a
complication of biliary atresia (BA). Portal hypertension confers an increased risk of experiencing
gastrointestinal bleeding. The examination findings of abdominal scarring (which would be typical after
undergoing portoenterostomy) and arm rash are not suggestive of a complication of his BA. Obesity may
increase the risk for nonalcoholic fatty liver disease, but is not for portal hypertension.
Biliary atresia is a progressive liver disease that presents in infancy. A Kasai portoenterostomy should be
performed, ideally within 90 days of birth, to avoid the need for liver transplantation. Even with a successful
portoenterostomy, children with BA develop liver injury, including fibrosis, cirrhosis, or both. Portal
hypertension may manifest with gastrointestinal bleeding (esophageal and/or gastric varices), ascites, or
hepatic encephalopathy. The most common causes of portal hypertension in children are portal vein
thrombosis and BA.
Physical examination findings associated with portal hypertension include splenomegaly; caput medusa
(engorged superficial periumbilical abdominal vessels); spider angiomata; telangiectasia; palmar erythema;
easy bruising; and bleeding of the nose, gums, or both. Mental status changes (especially sleep-wake
disturbances in young children) can occur in individuals with hepatic encephalopathy. Malnutrition is
common in children with cirrhosis and portal hypertension.
Laboratory findings suggestive of portal hypertension are those seen in hypersplenism: thrombocytopenia,
anemia, and leukopenia. Abdominal ultrasonography with Doppler may demonstrate abnormalities in portal
venous flow (eg, flow reversal in severe portal hypertension). The hepatic venous pressure gradient (the
difference between wedged hepatic venous pressure and free hepatic venous pressure) helps determine the
severity of portal hypertension.
Guided by severity and associated complications, the treatment of portal hypertension includes management
of gastrointestinal hemorrhage (endoscopic intervention, β-blockade), management of ascites, shunt surgery
(portal decompression), and liver transplantation. Surgical shunts are considered when gastrointestinal
bleeding and/or ascites is refractory to medical and endoscopic management. Liver transplantation may be
the best treatment option for children with portal hypertension and severe, progressive liver disease.
Suggested Reading(s)
Chapin C, Bass L. Cirrhosis and portal hypertension in the pediatric population. Clin Liv Dis.
2018;22(4):735-752. doi:10.1016/j.cld.2018.06.007
Shneider BL, Bosch J, de Franchis R, et al; expert panel of the Children's Hospital of Pittsburgh of
UPMC. Portal hypertension in children: expert pediatric opinion on the report of the Baveno V
Consensus Workshop on methodology of diagnosis and therapy in portal hypertension. Pediatr
Transplant. 2012;16(5):426-437. doi:10.1111/j.1399-3046.2012.01652.x
Sriram I, Nicklas D. Biliary atresia. Pediatr Rev. 2022;43(11):659-661. doi:10.1542/pir.2021-005287
Zallen GS, Bliss DW, Curran TJ, Harrison MW, Silen ML. Biliary atresia. Pediatr Rev. 2006;27(7):243-248.
doi:10.1542/pir.27-7-243
Content Domain
Gastroenterology
A 4-year-old boy is seen for evaluation of a 10-month history of constipation. He began to experience
infrequent and painful stooling around the time of toilet training. He has a large-diameter stool every 7 to 10
days in a diaper and refuses to use the toilet. His parents observe him actively withholding stool. Over the
few days before stooling, he experiences abdominal distention and poor appetite. He has taken polyethylene
glycol 3350 intermittently over the last several months, which causes him to have diarrhea. He is otherwise
healthy and urinates in the toilet without difficulty. His growth and development are normal. There is no
history of delayed passage of meconium.
Physical examination reveals a well-nourished child with mild abdominal distention and a large amount of
palpable stool in his lower abdomen. The remainder of his examination findings are normal, including a
normal-appearing anus.
D. manual disimpaction
Correct answer is C
PREP Pearl(s)
Constipation is common in childhood and most often is functional.
Management of constipation should begin with treatment of any fecal impaction followed by medical
(use of osmotic, lubricant, and/or stimulant laxatives) and behavioral therapies to maintain daily
evacuation of soft stools.
Critique
The child in the vignette has history and physical examination findings consistent with functional constipation
(infrequent, large-diameter stools; onset around the time of toilet training; stool-withholding behaviors) and
fecal impaction (diarrhea after administration of an osmotic laxative and palpable stool burden). The best
next step in his care is medical disimpaction, either with high-dose oral polyethylene glycol (PEG) 3350 or a
rectally administered medication (enema or suppository). Generally, an oral medication (eg, PEG 3350) is
preferred; if it is unsuccessful or rejected, daily enemas can be administered. If both oral and rectal
medications are not effective or are refused, hospitalization with nasogastric administration of PEG 3350 is
an appropriate next step.
Manual disimpaction is rarely indicated for functional constipation, but it could be considered if nasogastric
administration of PEG 3350 is not tolerated. This procedure should be performed under anesthesia. Potential
complications include colonic perforation. Although a well-balanced diet is important, a daily fiber
supplement is not indicated to treat fecal impaction caused by functional constipation. Encouraging toilet use
should be a management goal once the impaction is successfully treated and the child is comfortably stooling
regularly.
Constipation, the passage of hard, infrequent, and painful stools, is common during childhood (affecting
approximately 12%-18% of children), with the highest incidence occurring around the time of toilet training.
The most common cause of pediatric constipation is functional. Hyams 2016 outlines the Rome IV diagnostic
criteria for functional constipation.
Anorectal malformation
Celiac disease
Cow milk protein allergy
Cystic fibrosis
Hirschsprung disease
Spinal cord abnormalities
Thyroid disease
Trauma/abuse
Further evaluation of constipation should be pursued when concerning signs and symptoms are present,
such as the following:
Abnormal anal examination findings (eg, abnormal position, absent anal reflex, unusual appearing
fissures/tags)
Bilious emesis
Blood in the stool (without fissure)
Constipation in the first month after birth
Difficulty stooling with thin, ribbon-appearing stools (suggestive of Hirschsprung disease or anal
stenosis)
Failure to thrive
History of delayed passage of meconium (suggestive of cystic fibrosis or Hirschsprung disease)
Sacral dimple or tuft
Severe abdominal distention
Severe abdominal pain
Significant fear of anal inspection (suggestive of abuse)
Treatment of functional constipation includes both medical and behavioral interventions. The first priority is
treatment of any fecal impaction. Table 1 outlines medical therapies for fecal disimpaction. After resolution of
the impaction, maintenance medical and behavioral therapies should be initiated. The goal of medical
therapy is to maintain regular evacuation of the colon with daily, soft stools, allowing the rectal tone to return
to normal. Daily administration of an osmotic laxative is often used as first-line therapy; PEG 3350 is most
commonly used for toddlers and older children and lactulose is used most commonly for infants.
Maintenance-therapy dosing for osmotic (eg, PEG 3350, lactulose, magnesium hydroxide), lubricant (eg,
docusate, mineral oil) and stimulant (eg, senna, bisacodyl) laxatives is listed in Table 2.
Behavioral therapy is important for successful constipation treatment and should include these measures:
Daily monitoring (stool appearance and frequency, presence of soiling, medications taken)
A toileting routine (sitting after meals for 5-10 minutes)
For children who are not already toilet trained, this routine should be delayed until daily,
comfortable stooling is well established
Use of a comfortable toilet (eg, the nurse’s office bathroom if the child is uncomfortable using a
classroom or hallway bathroom)
Use of a toilet seat insert (if appropriate for the child’s size) and a stepstool to keep the child’s feet
supported
Encouraging a well-balanced diet rich in fruits and vegetables
Positive reinforcement and small rewards for following the behavioral and medical care plan
Consideration of psychology and/or physical therapy consultation
Constipation treatment (behavioral and medical) can be difficult and time-consuming. Acknowledgment of
this difficulty is important for successful implementation. It may take months to years before colonic
distention and function improve and medications can be discontinued. Close follow-up and the support of
caregivers is important during treatment. Referral to a pediatric gastroenterologist for further testing and
evaluation may be considered when concerning signs and symptoms are present or if conventional medical
or behavioral therapies fail.
Suggested Reading(s)
Belamarich PF. Constipation. In: McInerny TK, Adam HM, Campbell DE, Foy JM, Kamat DM, eds.
American Academy of Pediatrics Textbook of Pediatric Care. 2nd ed. American Academy of Pediatrics;
2021:chap 134. Accessed September 1, 2023. Pediatric Care Online
Constipation. GIKids, North American Society for Pediatric Gastroenterology, Hepatology and Nutrition.
Accessed September 1, 2023. www.gikids.org/constipation
Hyams JS, Di Lorenzo C, Saps M, Shulman RJ, Staiano A, van Tilburg M. Functional disorders: children
and adolescents. Gastroenterology. 2016;S0016-5085(16)00181-5
LeLeiko NS, Mayer-Brown S, Cerezo C, Plante W. Constipation. Pediatr Rev. 2020;41(8):379-392.
doi:10.1542/pir.2018-0334
Tabbers M, DiLorenzo C, Berger MY, et al; European Society for Pediatric Gastroenterology,
Hepatology, and Nutrition; North American Society for Pediatric Gastroenterology. Evaluation and
treatment of functional constipation in infants and children: evidence-based recommendations from
ESPGHAN and NASPGHAN. J Pediatr Gastroenterol Nutr. 2014;58(2):258-274.
doi:10.1097/MPG.0000000000000266
Content Domain
Gastroenterology
Note that systematic dosage and safety studies on these medications have not been
rigorously performed in the pediatric age range.
Reprinted with permission from LeLeiko NS, Mayer-Brown S, Cerezo C, Plante W.
Constipation. Pediatr Rev 2020;41(8):388.
•
Adapted and reprinted with permission from LeLeiko NS, Mayer-Brown S, Cerezo C, Plante W. Constipation. Pediatr Rev
AAP PREP 2024 - Question 82/267 Gastroenterology Question 9/10
A 16-year-old adolescent boy is seen regarding possible isotretinoin therapy for severe nodular acne. He has
tried multiple topical therapies, but he continues to experience debilitating acne. During the appointment, he
expresses concern about an episode of yellowing of the eyes that occurred several months ago. At that time,
he was seen in a local emergency department. Results of laboratory tests obtained at that visit are shown
below. His eye color returned to normal without intervention a few days later.
γ-Glutamyltransferase 10 U/L
The boy has been otherwise healthy and denies any use of any prescription, over-the counter, or recreational
drugs. He was born at 38 weeks’ gestation and had neonatal jaundice requiring phototherapy for 3 days in
the newborn nursery. He does not recall any other episodes of jaundice.
Physical examination reveals a healthy-appearing adolescent boy with normal growth and development. His
sclera are anicteric, and there is no hepatosplenomegaly. He has nodular acne on his face and back. The
remainder of his physical examination findings are normal.
Of the following, the BEST recommendation regarding the prescription and monitoring of isotretinoin for this
boy is to
A. not prescribe it
Correct answer is D
PREP Pearl(s)
Children and adolescents with jaundice should undergo prompt laboratory testing to identify if the
hyperbilirubinemia is conjugated (direct) or unconjugated (indirect).
Causes of unconjugated (indirect) hyperbilirubinemia include hemolytic disease and disorders of
bilirubin metabolism (eg, Gilbert and Crigler-Najjar syndromes).
Gilbert syndrome is a benign condition associated with mild unconjugated hyperbilirubinemia that
most often occurs during prolonged fasting or acute viral illnesses.
Critique
The adolescent in the vignette had a transient episode of jaundice associated with an elevated level of
unconjugated bilirubin. Common causes of unconjugated hyperbilirubinemia in adolescents include
hemolytic diseases and Gilbert syndrome. Given the adolescent’s normal hemoglobin concentration, Gilbert
syndrome is the most likely cause of his jaundice. Because there is no underlying hepatocellular injury with
Gilbert syndrome, no adjustment is needed to the standard dosing and monitoring protocol for isotretinoin.
Conjugated hyperbilirubinemia (elevated conjugated or direct bilirubin concentration), not seen in this
adolescent, suggests biliary obstruction or liver disease. For individuals with liver disease, hepatotoxic
medications (including isotretinoin) should be avoided or may require a reduced dose with frequent
monitoring of liver function.
The evaluation of children and adolescents with jaundice should include a complete history and physical
examination (Table 1) and laboratory studies (Table 2). Using these tools, it is possible to quickly discern
whether the jaundice is caused by hepatocellular injury or biliary obstruction (conjugated
hyperbilirubinemia), hemolysis, or a disorder of bilirubin metabolism. Conjugated hyperbilirubinemia is
always pathologic and requires further evaluation by a gastroenterologist or hepatologist.
Unconjugated bilirubin, an end product of heme degradation, is transported (bound by albumin) from the
spleen, liver, and bone marrow to the liver for clearance. Once in the hepatocyte, unconjugated bilirubin
undergoes conjugation by bilirubin uridine diphosphate glucuronosyltransferase (BUGT) to form water-
soluble bilirubin conjugates (conjugated bilirubin). The conjugated bilirubin is then exported through the
canalicular membrane as a component of bile for fecal excretion (Figure).
Reprinted with permission from Pan DH, Rivas Y. Jaundice: Newborn to Age 2 Months. Pediatr Rev
(2017);38 (11):500.
Neonatal unconjugated hyperbilirubinemia results from immature BUGT function. For most infants, this is a
benign and self-limiting condition. However, it can develop into severe unconjugated hyperbilirubinemia.
Treatment includes maximizing enteral intake and phototherapy according to American Academy of
Pediatrics guidelines.
Crigler-Najjar syndrome, the congenital absence (type 1) or decreased function (type 2) of BUGT activity,
results in severe neonatal unconjugated hyperbilirubinemia. Treatment options for this rare disorder include
lifetime phototherapy and liver transplant.
Suggested Reading(s)
Kemper AR, Newman TB, Slaughter JL, et al. Clinical practice guideline revision: management of
hyperbilirubinemia in the newborn infant 35 or more weeks of gestation. Pediatrics.
2022;150(3):e2022058859. https://doi.org/10.1542/peds.2022-058859
LiverTox. Clinical and research information on drug-induced liver injury. National Institute of Diabetes
and Digestive and Kidney Diseases. Accessed September 1, 2023.
https://www.ncbi.nlm.nih.gov/books/NBK547852/
Pan DH, Rivas Y. Jaundice. In: McInerny TK, Adam HM, Campbell DE, DeWitt TG, Foy JM, Kamat DM, eds.
American Academy of Pediatrics Textbook of Pediatric Care. American Academy of Pediatrics;
2021:chap 170. Accessed September 1, 2023. Pediatric Care Online
Content Domain
Gastroenterology
View Peer Results Table 1. Important Elements of the History and Physical Examination
in a Patient With Jaundice.
Information Element Specific Element
A 9-day-old, full-term male neonate is seen in the pediatric emergency department for evaluation of
nonbloody, white and yellow-green emesis of 3 days’ duration. He is breastfed and has 2 to 3 yellow, seedy
stools daily. The pregnancy and delivery were uncomplicated. He passed meconium within 24 hours after
birth. The neonate had an unremarkable nursery course and was discharged home 2 days after birth.
A nasogastric (NG) tube is placed in the emergency department owing to concerns about intestinal
obstruction.
Physical examination reveals a fussy but consolable neonate with a NG tube in place. He has a temperature
of 37.2 °C, a heart rate of 140 beats/min, a respiratory rate of 32 breaths/min, and an oxygen saturation of
99% in room air. His physical examination findings are otherwise normal. A radiograph is obtained (Figure 1).
Reprinted with permission from Rich BS, Bornstein E, Dolgin SE. Pediatr Rev. 2022;43(5):268.
A. colonic atresia
B. duodenal atresia
C. jejunal atresia
D. pyloric atresia
Correct answer is B
PREP Pearl(s)
Bilious vomiting in the neonate should prompt immediate evaluation for intestinal obstruction.
Duodenal atresia, a congenital intestinal malformation, may present with bilious or nonbilious emesis,
upper abdominal distention, and an abdominal radiograph with a dilated stomach and proximal
duodenum (“double bubble”).
Critique
The neonate in the vignette has a history of bilious emesis and a radiograph demonstrating a dilated
stomach and proximal duodenum (“double bubble”), findings consistent with duodenal atresia. Pyloric atresia
is an unlikely diagnosis for this neonate, because a single bubble (dilated stomach) on radiograph and
nonbilious emesis are expected with pyloric atresia. The absence of abdominal distention (on physical
examination) and dilated loops of small bowel (on radiograph) make jejunal and colonic atresias unlikely
diagnoses for this neonate. Dilation of the stomach, duodenum, and proximal jejunum can appear as a “triple
bubble” on radiography in jejunal atresia. Figure 2 demonstrates dilated loops of small bowel in a neonate
with ileal atresia.
Reprinted with permission from Rich BS, Bornstein E, Dolgin SE. Intestinal atresias. Pediatr Rev.
2022;43(5):270.
Bilious emesis in a neonate should prompt immediate evaluation for intestinal obstruction. Other signs and
symptoms of neonatal intestinal obstruction include a history of maternal polyhydramnios, failure to pass
meconium by 24 to 48 hours after birth, and abdominal distention. Obstruction of the proximal intestine can
result in nonbilious (if proximal to the ligament of Treitz) or bilious emesis, and a scaphoid or nondistended
abdomen. Obstruction of the large intestine or distal small intestine can present with abdominal distention,
bilious emesis, and failure to pass or delayed passage of meconium. It is important to note that neonates can
present with nonbilious emesis in the case of intestinal obstruction at any level; thus, the presence of emesis
that is nonbilious does not exclude an intestinal obstruction.
Initial evaluation and management of the neonate with bilious emesis includes decompression of the
stomach with a nasogastric tube, stabilization (correction of dehydration, electrolyte abnormalities, or both if
present), and pediatric surgery consultation. Abdominal radiography can be helpful to identify the level of the
obstruction; however, a normal radiographic finding may not rule out an intestinal obstruction. An upper
gastrointestinal series may be indicated (for a neonate in stable condition) to further define the obstruction,
particularly if the stomach is decompressed. A contrast enema may be indicated if there is suspicion of distal
intestinal (ileal or colonic) obstruction. Surgical intervention may be indicated without definitive imaging for
unstable neonates for whom there is a high clinical suspicion of intestinal obstruction.
Duodenal atresia, a congenital malformation of the intestine, occurs in 1:5,000 births. Fifty percent to 60% of
newborns with duodenal atresia have associated congenital anomalies (eg, trisomy 21, congenital cardiac
malformations, imperforate anus, tracheoesophageal fistula, jejunal atresia, renal anomalies). Duodenal
atresia may be suspected prenatally (“double bubble” in the fetal abdomen, polyhydramnios) or diagnosed
after birth in neonates with bilious or nonbilious vomiting and upper abdominal distention.
Treatment of duodenal atresia includes surgical resection of the atretic duodenal segment with anastomosis
of the proximal and distal ends of the duodenum (duodenoduodenostomy). For clinically stable neonates,
resection may occur within the first few days after birth. In such cases, preoperative treatment includes
gastric decompression and parenteral nutrition. If the neonate is in distress, or if malrotation and midgut
volvulus cannot be excluded, urgent surgical intervention is indicated. Parenteral nutrition (or in some cases,
jejunal enteral nutrition) is provided in the postoperative period, and most neonates achieve full oral feeding
within 2 to 3 weeks after surgery. Rarely, an anastomotic stricture can occur, requiring revision.
Gastroesophageal reflux, delayed gastric emptying, or both can be seen in neonates after
duodenoduodenostomy; however, most neonates with isolated duodenal atresia recover without
complication.
Suggested Reading(s)
Burge DM. The management of bilious vomiting in the neonate. Early Hum Dev. 2016;102:41-45.
doi:10.1016/j.earlhumdev.2016.09.002
Juang D, Snyder CL. Neonatal bowel obstruction. Surg Clin N Am. 2012;92(3):685-711, ix-x.
doi:10.1016/j.suc.2012.03.008
Mino J, Monteiro R, Stallion A. Gastrointestinal obstruction. In: McInerny TK, Adam HM, Campbell DE,
DeWitt TG, Foy JM, Kamat DM, eds. American Academy of Pediatrics Textbook of Pediatric Care.
American Academy of Pediatrics; 2021:chap 257. Accessed September 1, 2023. Pediatric Care Online
Rich BS, Bornstein E, Dolgin SE. Intestinal atresias. Pediatr Rev. 2022;43(5):266-274.
doi:10.1542/pir.2021-005177
Shields TM, Lightdale JR. Vomiting in children. Pediatr Rev. 2018;39(7):342-358. doi:10.1542/pir.2017-
0053
Content Domain
Gastroenterology
A 4-month-old, full-term infant is seen for a health supervision visit. The boy’s medical history is significant
for gross motor delay and being slow to feed. He is taking 2 to 3 oz of 20 kcal/oz of formula every 3 to 4
hours. Echocardiography performed in the neonatal period shows a small muscular ventricular septal defect.
His physical examination findings are significant for an alert boy with normal vital signs, a weight 2.5 SDs
below the mean for his age, hypotonia, midface hypoplasia, up-slanted palpebral fissures, epicanthal folds,
and bilateral single transverse palmar creases. The remainder of his examination findings are normal.
A. chest radiography
B. echocardiography
C. observation
Correct answer is D
PREP Pearl(s)
Infants with trisomy 21 can present with feeding issues (eg, gastroesophageal reflux and dysphagia).
In children with trisomy 21, dysphagia can occur in both the oral and pharyngeal phases of swallowing.
Causative factors may include hypotonia, decreased jaw strength, large tongue relative to a small jaw,
and poor tongue control.
A videofluoroscopic swallowing study is the test of choice to evaluate children with trisomy 21 with
dysphagia; this study aids in the identification of laryngeal penetration and tracheal aspiration, in
addition to assessing the oral and pharyngeal phases of the swallowing mechanism.
Critique
The best next step in the management of the boy in the vignette is a videofluoroscopic swallowing study. The
boy has trisomy 21 and is exhibiting feeding issues (slow to feed) and failure to gain weight. Infants with
trisomy 21 often have feeding issues (eg, gastroesophageal reflux and dysphagia) due to multiple factors,
including hypotonia, decreased jaw strength, large tongue relative to a small jaw, and poor tongue control.
Other contributing factors may include heart defects, developmental delay, and upper airway obstruction.
Dysphagia can occur in both the oral and pharyngeal phases.
The indications for a feeding assessment and videofluoroscopic swallowing study include the following:
Slow feeds
Choking episodes with feeds
Marked hypotonia
Being underweight
Recurrent or persistent respiratory symptoms
Desaturation with feeds
Approximately 90% of aspiration occurring in infants with trisomy 21 is silent (ie, without choking or cough),
which can be missed during a feeding assessment. A videofluoroscopic swallowing study is the test of choice
to identify infants with laryngeal penetration and tracheal aspiration and to assess the oral and pharyngeal
phases of swallowing. Prompt management of aspiration can prevent recurrent respiratory symptoms and
infections that result from untreated aspiration.
Although the boy is known to have a small muscular ventricular septal defect, echocardiography is not
indicated at this time because he is not exhibiting signs or symptoms of cardiac failure. In most cases,
muscular ventricular septal defects close spontaneously. Chest radiography would be helpful if the boy had
signs or symptoms of aspiration pneumonia, which this boy does not. In this situation, it is important to
identify the underlying cause of the boy’s feeding issues to optimize his care. Observation alone would not be
appropriate because the boy is exhibiting concerning features, including hypotonia, poor weight gain, and
slow feeding.
Suggested Reading(s)
Bull MJ. Down syndrome. N Engl J Med. 2020;382(24):2344-2352. doi:10.1056/NEJMra1706537
Bull MJ. Down syndrome: managing the child and family. In: McInerny TK, Adam HM, Campbell DE,
DeWitt TG, Foy JM, Kamat DM, eds. American Academy of Pediatrics Textbook of Pediatric Care.
American Academy of Pediatrics; 2023. Pediatric Care Online
Bull MJ, Trotter T, Santoro SL, et al. Health supervision for children and adolescents with Down
syndrome. Pediatrics. 2022;149(5):e2022057010. doi:10.1542/peds.2022-057010
Duan S. Down syndrome. Point-of-Care Quick Reference. Pediatric Care Online. August 20, 2021.
Accessed September 1, 2023. Pediatric Care Online
Narawane A, Eng J, Rappazzo C, et al. Airway protection & patterns of dysphagia in infants with Down
syndrome: videofluoroscopic swallow study findings & correlations. Int J Pediatr Otorhinolaryngol.
2020;132:109908. doi:10.1016/j.ijporl.2020.109908
Content Domain
Genetics
A 2-year-old boy is seen for a health supervision visit. He achieved his motor milestones on time but is not
saying any words yet. Physical examination findings are remarkable for 8 café au lait (CAL) macules that are
each greater than 5 mm. The family history is remarkable for an older sister with multiple CAL macules and a
history of optic pathway glioma; their mother with multiple CAL macules, multiple neurofibromas, and a large
plexiform neurofibroma on her neck; and multiple maternal family members with similar and additional
findings of this same disorder.
Of the following, the MOST likely risk of this boy’s parents having another child with his same disorder is
A. 25%
B. 50%
C. 75%
D. 100%
Correct answer is B
PREP Pearl(s)
Autosomal dominant inheritance is characterized by multiple affected family members in multiple
generations, equal risk in males and females, and a 50% risk of passing on the condition with every
offspring. Examples include neurofibromatosis type 1, achondroplasia, and tuberous sclerosis
complex.
Autosomal recessive inheritance is characterized by unaffected parents of an affected child. Parental
consanguinity increases the risk. Examples include cystic fibrosis and sickle-cell anemia.
X-linked inheritance is characterized by the absence of male-to-male transmission. Some X-linked
dominant disorders have male lethality (eg, incontinentia pigmenti); a few have presentation both in
males and females (eg, X-linked hypophosphatemia). X-linked recessive disorders (eg, Duchenne
muscular dystrophy) are characterized by variable presentation in carrier females and a severe
phenotype in males.
Mitochondrial inheritance is maternally inherited (absence of transmission of the disorder from an
affected male). All children of an affected mother will be affected. Examples include Leber hereditary
optic neuropathy and mitochondrial encephalomyopathy, lactic acidosis, and stroke-like episodes
(MELAS).
Critique
The boy in the vignette and many of his maternal family members have neurofibromatosis type 1 (NF1).
Neurofibromatosis type 1 is inherited in an autosomal dominant pattern with variable expressivity. Each
affected individual has a 50% risk of passing on the disorder with every offspring. An example of an
autosomal dominant disorder pedigree is shown in the Figure.
In the absence of a parent with diagnosed NF1, at least 2 of the following features are required to
make a clinical diagnosis.
Café au lait macules (at least 6; >5-mm prepubertal, >15-mm postpubertal)
Freckling in the axillary or inguinal region
Cutaneous neurofibromas (≥2)
Lisch nodules (≥2) or choroidal abnormalities (≥2)
Plexiform neurofibroma (at least 1)
Optic pathway glioma
Osseous lesion (sphenoid dysplasia, anterolateral tibial bowing, or long bone pseudoarthrosis)
Pathogenic variant in the NF1 gene
In the presence of a parent diagnosed with NF1, 1 of the above features is sufficient to make a clinical
diagnosis.
Suggested Reading(s)
Legius E, Messiaen L, Wolkenstein P, et al; International Consensus Group on Neurofibromatosis
Diagnostic Criteria (I-NF-DC). Revised diagnostic criteria for neurofibromatosis type 1 and Legius
syndrome: an international consensus recommendation. Genet Med. 2021;23(8):1506-1513.
doi:10.1038/s41436-021-01170-5
Nussbaum RL, McInnes RR, Willard HF. Patterns of single gene inheritance. In: Thompson and
Thompson Genetics in Medicine. 8th ed. Elsevier; 2016:107-132.
Saul RA. Fundamentals of genetics and genomics. Medical Genetics in Pediatric Practice. American
Academy of Pediatrics; 2013:1-20.
Content Domain
Genetics
Courtesy of A. Sidhu
FIGURE: Sample autosomal dominant disorder pedigree.
A term neonate is being evaluated in the newborn nursery. Her physical examination findings are remarkable
for hypotonia, bilateral single transverse palmar crease, up-slanted palpebral fissures, epicanthal folds,
midface hypoplasia, and wide gap between the first and second toes. Echocardiography is obtained, which
reveals atrial and ventricular septal defects.
Of the following, the BEST prenatal test to screen for this neonate's underlying diagnosis is
B. first-trimester screen
Correct answer is A
PREP Pearl(s)
Prenatal screening tests include a cell-free DNA test, first-trimester screen, quadruple marker test, and
nuchal translucency measurement.
A cell-free DNA test is a screening test that uses fetal cell-free DNA fragments circulating in maternal
blood to screen for fetal aneuploidies.
The sensitivity and specificity of cell-free DNA screening for common fetal aneuploidies are the highest
among commonly available prenatal screening tests.
Critique
The neonate in the vignette has trisomy 21. The sensitivity and specificity of cell-free DNA screening for
common fetal aneuploidies are the highest among all commonly available prenatal screening tests. The
detection rate for trisomy 21 is greater than 99%.
Prenatal screening tests include a cell-free DNA test, first-trimester screen, quadruple marker test,
integrated, and nuchal translucency (NT) measurement. The cell-free DNA screen can be performed after 9 to
10 weeks of gestation. The test uses cell-free DNA fragments circulating in maternal blood to screen for fetal
aneuploidies. Cell-free DNA in maternal blood consists of both maternal and fetal fragments; the fetal
component (3%-13%), derived from placental trophoblasts, is released into the maternal circulation after
apoptosis.
A first-trimester screen can be performed between 10 and 14 weeks of gestation. The test includes
measurement of NT by ultrasonography (assessment of the fluid-filled space at the fetal neck) and
measurement of serum ꞵ-human chorionic gonadotropin (β-hCG) and pregnancy-associated plasma protein
A (PAPP-A) levels. A risk estimate is calculated using the results of testing and accounting for maternal factors
(age, history of aneuploidy, weight, race, and number of fetuses). Results of a first-trimester screen in the
case of a fetus with trisomy 21 should reveal a decrease in the PAPP-A level and an increase in the β-hCG
level. An increase in NT is associated with fetal aneuploidy and cardiac anomalies. The sensitivity of the first-
trimester screen for the detection of trisomy 21 is 82% to 87%, whereas that of NT alone is 70%. Some testing
laboratories also measure the α-fetoprotein (AFP) level as part of the first-trimester screen; however, this test
would not replace the need for AFP screening in the second trimester (for open neural tube defects).
The quadruple marker test (quad screen) can be performed between 15 and 22 weeks of gestation. Four
maternal serum analytes are measured: hCG, AFP, unconjugated estriol (uE3), and dimeric inhibin A. A risk
estimate is calculated using the serum analyte levels and maternal factors. In the case of a fetus with trisomy
21, expected results include decreased uE3 and AFP levels and increased hCG and inhibin A levels. The
sensitivity of the quad screen for detection of trisomy 21 is 80%.
The integrated screen uses data from the first-trimester screen (NT and PAPP-A) and quadruple marker
screen to obtain a single test result in the second trimester. The detection rate for trisomy 21 is 96%. The
disadvantage of the integrated screen is the requirement for samples at 2 different timepoints in gestation
and, therefore, the inability to obtain results in the first trimester.
Suggested Reading(s)
American College of Obstetricians and Gynecologists’ Committee on Practice Bulletins—Obstetrics;
Committee on Genetics; Society for Maternal-Fetal Medicine. Screening for fetal chromosomal
abnormalities: ACOG Practice Bulletin. Obstet Gynecol. 2020;136(4):e48-e69.
doi:10.1097/AOG.0000000000004084
Campbell DE. Prenatal pediatric visit. In: McInerny TK, Adam HM, Campbell DE, DeWitt TG, Foy JM,
Kamat DM, eds. American Academy of Pediatrics Textbook of Pediatric Care. American Academy of
Pediatrics; 2023. Accessed September 1, 2023. Pediatric Care Online
Nussbaum RL, McInnes RR, Willard HF, eds. Prenatal diagnosis and screening. In: Thompson and
Thompson Genetics in Medicine. 8th ed. Elsevier; 2016:349-368.
Saul RA, ed; AAP Committee on Genetics. Preventable disabilities: the road ahead. In: Medical Genetics
in Pediatric Practice. American Academy of Pediatrics; 2013:349-358.
Content Domain
Genetics
A male neonate, born at term, is seen in the nursery for hypotonia and dysmorphic facial features. On
physical examination, the patient has epicanthal folds, upslanted palpebral fissures, a depressed nasal
bridge, bilateral single transverse palmar crease, and an increased gap between the first and second toes
bilaterally. Chromosome analysis shows an unbalanced translocation of chromosomes 14 and 21 (46,XY, der
[14;21] [q10:q10] +21). His parents ask about the risk of having another child with the same disorder.
Of the following, the MOST appropriate testing recommendation to determine this family’s recurrence risk is
D. no additional testing
Correct answer is A
PREP Pearl(s)
Approximately 95% of cases of trisomy 21 (Down syndrome) are due to a free-standing extra
chromosome 21 that results from meiotic nondisjunction. Parental chromosome analysis is not
indicated because this is not an inherited event (sporadic).
Approximately 4% of cases of trisomy 21 (Down syndrome) result from an unbalanced translocation
between chromosome 21 and another acrocentric chromosome (13, 14, 15, 21, or 22). Parental
chromosome analysis is required to determine inheritance and provide an accurate recurrence risk for
the family.
When a child has a free-standing extra chromosome 21, the recurrence risk is estimated to be 1% or
the age-adjusted risk of the mother (if >1%) at the time of the next pregnancy.
Critique
The newborn in the vignette has trisomy 21 (Down syndrome). His chromosome analysis shows an
unbalanced translocation between chromosomes 14 and 21 as 46,XY, der (14;21) (q10:q10) +21. In this case,
chromosome analysis for both parents is recommended.
In trisomy 21, there is an extra chromosome 21, resulting in a numerical or structural chromosomal
abnormality. Approximately 95% of cases are due to a free-standing extra chromosome 21, resulting from
meiotic nondisjunction (maternal in 90% and paternal in 10%). The scenario in the vignette is an example of a
numerical chromosomal abnormality. Mosaicism for a free-standing extra chromosome 21 has been
identified in approximately 1% of cases. When a child has a free-standing extra chromosome 21, the
recurrence risk (RR) is estimated to be 1% or the age-adjusted risk of the mother (if >1%) at the time of the
next pregnancy. Parental testing is not indicated in this situation because this is not an inherited event
(sporadic).
In the absence of a balanced translocation in either parent, the RR is the same as that of a free-standing
chromosome 21 trisomy. A balanced translocation increases the RR; how much of an increase is dependent
on the sex of the parent and type of translocation. For translocations that involve chromosomes 13, 14, 15, or
22, the RR is 10% to 15% if the mother is the carrier; it is 2% to 5% if the father is the carrier. For
translocations between 2 chromosomes 21, the RR is always 100%. Genetic counseling is provided both
before and after parental testing. The discussion involves possible result outcomes, RR, and options for
testing in a subsequent pregnancy.
The finding of an unbalanced translocation in the neonate in the vignette necessitates chromosome analysis
for both parents to determine the RR. Performing chromosome analysis on 1 parent only would not be
appropriate because the unbalanced translocation could have been inherited from either parent and this
analysis would not provide an accurate RR estimation for the family.
Suggested Reading(s)
Bull MJ. Down syndrome: managing the child and family. In: McInerny TK, Adam HM, Campbell DE,
DeWitt TG, Foy JM, Kamat DM, eds. American Academy of Pediatrics Textbook of Pediatric Care. 2nd ed.
American Academy of Pediatrics; 2017:chap 244. Pediatric Care Online
Bull MJ, Trotter T, Santoro SL, et al. Health supervision for children and adolescents With Down
syndrome. Pediatrics. 2022;149(5):e2022057010. doi:10.1542/peds.2022-057010
Saul RA. Specific genetic conditions. In: Saul RA. Medical Genetics in Pediatric Practice. American
Academy of Pediatrics; 2014;chap 9.
Sheets KB, Crissman BG, Feist CD, et al. Practice guidelines for communicating a prenatal or postnatal
diagnosis of Down syndrome: recommendations of the National Society of Genetic Counselors. J Genet
Counsel. 2011;20(5):432-441. doi:10.1007/s10897-011-9375-8
Content Domain
Genetics
A 16-year-old boy is seen for a health supervision visit. His achievement of developmental milestones was
delayed for speech and communication skills, and he received speech therapy from age 2 to 6 years. He is
currently a high school junior in a regular education classroom with an individualized education plan for
reading and comprehension. The family history is noncontributory. The boy’s weight is at the 40th percentile
and height is at the 98th percentile for age. His physical examination findings are remarkable only for
gynecomastia and sparse body hair. Sexual maturity rating is 3 for pubic hair and 2 for genital development.
A. fragile X syndrome
B. Klinefelter syndrome
C. Marfan syndrome
D. Sotos syndrome
Correct answer is B
PREP Pearl(s)
Klinefelter syndrome is a sex chromosome aneuploidy (47,XXY) seen in males; it is diagnosed with
chromosome analysis.
Clinical features of Klinefelter syndrome include developmental delay, learning disability,
gynecomastia, tall stature, and hypergonadotropic hypogonadism.
Critique
The boy in the vignette’s most likely diagnosis is Klinefelter syndrome given his tall stature, developmental
delay, and abnormal pubertal development. Klinefelter syndrome is a sex chromosome aneuploidy (47,XXY)
seen in males. The diagnosis of Klinefelter syndrome is made by chromosome analysis.
Fragile X syndrome (FXS) can present with developmental delay, however, the tall stature with gynecomastia
and abnormal puberty noted in the child in the vignette are not seen in children with FXS. The child in the
vignette does not have the characteristic body habitus seen in individuals with Marfan syndrome.
Additionally, individuals with Marfan syndrome have normal intellect and normal pubertal development,
unlike this child. While tall stature and developmental delay are seen in individuals with Sotos syndrome, this
child does not have the characteristic facial dysmorphology, macrocephaly, normal upper-to-lower segment
ratio, and normal sexual development seen in Sotos syndrome. The Table compares features found in
individuals with Klinefelter, fragile X, Marfan, and Sotos syndromes.
Suggested Reading(s)
The chromosomal and genomic basis of disease: disorders of the autosomes and sex chromosomes.
In: Willard HF, Nussbaum RL, McInnes RR, eds. Thompson and Thompson Genetics in Medicine. 8th ed.
Elsevier; 2016:75-106.
Jayakar P, Spiliopoulos M. Klinefelter syndrome. In: McInerny TK, Adam HM, Campbell DE, DeWitt TG,
Foy JM, Kamat DM, eds. American Academy of Pediatrics Textbook of Pediatric Care. American
Academy of Pediatrics; 2021:chap 281. Accessed September 1, 2023. Pediatric Care Online
Recognizable patterns of malformation. In: Jones KL, Jones MC, del Campo M, eds. Smith's
Recognizable Patterns of Human Malformation: Expert Consult. 8th ed. Elsevier; 2021:1-928.
Samango-Sprouse C, Song SQ, Lin AE, Powell CM, Gropman AL. Klinefelter syndrome and Turner
syndrome. Pediatr Rev. 2021;42(5):272-274. doi:10.1542/pir.2020-004028
Specific genetic conditions. In: American Academy of Pediatrics Committee on Genetics; Saul RA, eds.
Medical Genetics in Pediatric Practice. American Academy of Pediatrics; 2014:chap 9.
Content Domain
Genetics
A 6-year-old boy is seen for a health supervision visit. His developmental milestones have been achieved on
time. His physical examination findings are remarkable for downward slanting palpebral fissures, mandibular
and malar hypoplasia, micrognathia, notching of the lower eyelid, and small ears bilaterally. The remainder
of his examination findings are normal. The boy wears a bone-anchored hearing aid for conductive hearing
loss. The family history is unremarkable.
Of the following, this boy’s MOST likely diagnosis is
B. Noonan syndrome
C. Pierre-Robin sequence
Correct answer is D
PREP Pearl(s)
Treacher Collins syndrome, a type of mandibulofacial dysostosis, is a developmental disorder of the
mandible and facial bones.
Classic craniofacial features of Treacher Collins syndrome include downward slanting palpebral
fissures, mandibular and malar hypoplasia, micrognathia, notching of the lower eyelid, and ear
anomalies.
Treacher Collins syndrome is inherited in an autosomal dominant and recessive pattern with both
interfamilial and intrafamilial variability.
Critique
The findings of the boy in the vignette are most consistent with a diagnosis of Treacher Collins syndrome
(TCS). Treacher Collins syndrome, a type of mandibulofacial dysostosis, is a developmental disorder of the
mandible and facial bones. The features are symmetric and present at birth. Other examples of dysostosis
include cleidocranial dysostosis, Crouzon syndrome, and Klippel-Feil syndrome.
Treacher Collins syndrome is characterized by the following craniofacial features (Figure 1):
Courtesy of Edgar O. Ledbetter, MD, FAAP. Reprinted with permission from Marion RW, Samanich J. Fascial
dysmorphia. In: McInerny TK, Adam HM, Campbell DE, DeWitt TG, Foy JM, Kamat DM, eds. American
Academy of Pediatrics Textbook of Pediatric Care (Online). 2nd ed. American Academy of Pediatrics;
2017:chap 148.
Eye
Downward slanting palpebral fissures
Coloboma of lower eyelid
Absent or hypoplastic lashes and nasolacrimal ducts
Ear
Microtia or anotia
Absent or hypoplastic external auditory canals
Conductive hearing loss, normal inner ear structures
Jaw
Micrognathia or retrognathia
Midface
Hypoplasia of zygomatic arch and lateral aspect of orbits, leading to characteristic downward
slant to palpebral fissures
Choanal atresia
Facial hair
Growth extends in the preauricular region to the lateral cheekbones
Dental
Tooth agenesis
Enamel opacities
Treacher Collins syndrome is inherited in an autosomal dominant (TCOF1, POLR1D, POLR1B) or autosomal
recessive (POLR1C, POLR1D) pattern. There is variability in expression in both related and unrelated
individuals. Additional evaluation recommended for a child diagnosed with TCS is outlined in the Table.
The 22q11.2 deletion syndrome is characterized by abnormalities of the palate (cleft palate and
velopharyngeal incompetence) and/or heart (tetralogy of Fallot, ventricular septal defect, and truncus
arteriosus), immune defects, characteristic facial features (bulbous nose, micrognathia, hooded eyelids, and
prominent nasal bridge) (Figure 2), and developmental delay.
Reprinted with permission from the American Academy of Pediatrics Committee on Genetics. Lyons MJ.
Specific genetic conditions. In . Saul RA, ed. Medical Genetics in Pediatric Practice. American Academy of
Pediatrics.2013:227
Classic features of Noonan syndrome include congenital heart defect (pulmonary valve stenosis), short
stature, dysmorphic features (down slanted palpebral fissure, epicanthal folds, wide-set eyes, low-set
posteriorly rotated ears, webbed neck, pectus excavatum, and/or carinatum) (Figure 3), and developmental
delay.
Pierre-Robin sequence (Figure 4) is characterized by micrognathia that leads to glossoptosis, with or without
cleft palate. Additional craniofacial and ear anomalies with hearing loss are not seen.
Suggested Reading(s)
Katsanis SH, Jabs EW, Adam MP, et al, eds. Treacher Collins syndrome. In: GeneReviews [Internet].
University of Washington, Seattle; 1993.
Kruk-Marszalek BA, Wójcicki P, Dowgierd K, Śmigiel R. Treacher Collins syndrome: genetics, clinical
features and management. Genes (Basel). 2021;12(9):1392. doi:10.3390/genes12091392
Marion RW, Samanich J. Fascial dysmorphia. In: McInerny TK, Adam HM, Campbell DE, DeWitt TG, Foy
JM, Kamat DM, eds. American Academy of Pediatrics Textbook of Pediatric Care (Online). 2nd ed.
American Academy of Pediatrics; 2017:chap 148. Pediatric Care Online
Content Domain
Genetics
Courtesy of A Sidhu
Reprinted with permission from Evans KN, Sie KC, Hopper RA, Glass RP, Hing AV, Cunningham
ML. Robin sequence: from diagnosis to development of an effective management plan.
Pediatrics. (2011); 127 (5): 942.
FIGURE 4: Craniofacial features of Pierre-Robin sequence. Presurgical
photographs of 3 infants with Robin sequence who all failed prone
positioning and ultimately underwent mandibular distraction osteogenesis
in the first year of life. A, Prior management with an nasopharagneal airway
(a nasopharangeal airway is in place in this photograph). B, Neonatal
distraction. C, Initial management with tracheotomy and later mandibular
distraction osteogenesis.
A, Patient living with Noonan syndrome at (from left to right) 10 days, 6 months, and 2 years
of age. B, Patient living with Noonan syndrome at (from left to right) 4 months and 1, 2, 5, 9, and
21 years of age
Reprinted with permission from Romano AA, Allanson JA, Dahlgren J, et al. Noonan syndrome:
clinical features, diagnosis, and management guidelines. Pediatrics. (2010); 126 (4): 749.
FIGURE 3: Craniofacial features of Noonan syndrome.
AAP PREP 2024 - Question 90/267 Genetics Question 7/13
An 8-year-old boy is seen for a health supervision visit. His developmental milestones were delayed for motor
skills, and he required physical therapy till 4 years of age. Physical examination findings are remarkable only
for macrocephaly, short stature, and rhizomelic (proximal) shortening of extremities. The family history is
significant for his mother and maternal family members with similar findings; the family pedigree is shown in
the Figure.
Of the following, the inheritance pattern seen in the boy’s family is
A. autosomal dominant
B. autosomal recessive
C. mitochondrial
D. X-linked dominant
Correct answer is A
PREP Pearl(s)
Achondroplasia is a skeletal dysplasia inherited in an autosomal dominant pattern.
Clinical features of achondroplasia include disproportionate short stature, rhizomelic shortening of
extremities, macrocephaly, motor delay, frontal bossing with midface retrusion, bowed legs, and
trident configuration of the hands.
Complications seen in achondroplasia include middle ear dysfunction, spinal stenosis, obstructive
sleep apnea, obstructive hydrocephalus (secondary to craniocervical junction compression), and
kyphosis.
Critique
The boy and his affected family members have achondroplasia, which is inherited in an autosomal dominant
pattern. Achondroplasia is a skeletal dysplasia characterized by the following:
Suggested Reading(s)
Legare JM, ed. Achondroplasia. GeneReviews [Internet]. University of Washington, Seattle; 2022.
Accessed September 1, 2023. https://www.ncbi.nlm.nih.gov/books/NBK1152/
Overview of genetics. In: American Academy of Pediatrics Committee on Genetics; Saul RA, eds.
Medical Genetics in Pediatric Practice. American Academy of Pediatrics; 2014:chap 1.
Patterns of single-gene inheritance.. In: Willard HF, Nussbaum RL, McInnes RR, eds. Thompson and
Thompson Genetics in Medicine. 8th ed. Elsevier; 2016:107-132.
Content Domain
Genetics
No surviving males in
disorders with male
lethality
X-linked recessive Affected mother 50% No M>>F Females can be variably Duchenne muscular
affected based on X- dystrophy
Affected father: all inactivation
daughters, none to sons
Abbreviations: F, female; M, male.
Courtesy of A. Sidhu
AAP PREP 2024 - Question 91/267 Genetics Question 8/13
A 5-year-old boy is seen for a health supervision visit. His achievement of language/communication
developmental milestones was delayed, and speech therapy was initiated at 2 years of age. He has been
diagnosed with attention-deficit/hyperactivity disorder (ADHD) and is scheduled to have an evaluation for
autism spectrum disorder. He will start kindergarten with an individualized education plan addressing the
domains of speech, reading, comprehension, and accommodations for his ADHD. The family history is
significant for premature ovarian failure in his mother and Parkinson disease in the maternal grandfather.
The boy’s physical examination findings are remarkable for a long face, prominent forehead, prominent jaw,
and large ears (Figure 1).
Figure 1. Reprinted with permission from the American Academy of Pediatrics Committee on Genetics.
Saul RA, ed. Medical Genetics in Pediatric Practice. 2013:198.
Of the following, the BEST test to identify this boy’s likely diagnosis is
A. chromosome analysis
B. chromosomal microarray
C. methylation analysis
Correct answer is D
PREP Pearl(s)
Trinucleotide repeat analysis is the diagnostic test of choice for conditions caused by expansion of the
normal triplet repeat size.
Trinucleotide repeat disorders include fragile X syndrome, Huntington disease, type 1 myotonic
dystrophy, and Friedreich ataxia.
Fragile X syndrome is an X-linked recessive disorder caused by more than 200 CGG repeats in the
promoter region of the FMR1 gene.
Critique
The best diagnostic test to identify the likely diagnosis of the patient in the vignette is trinucleotide repeat
analysis. His physical examination findings and personal and family history are consistent with the diagnosis
of fragile X syndrome (FXS). Fragile X syndrome occurs due to expansion of trinucleotide CGG repeats in the
promoter region of the FMR1 gene. It is inherited in an X-linked recessive pattern. The clinical features seen in
FXS are outlined in Table 1.
According to the American College of Medical Genetics and Genomics, fragile X analysis (trinucleotide CGG
repeats) and chromosomal microarray testing are recommended as first-line genetic tests for evaluation of a
child with autism spectrum disorder, intellectual disability, or global developmental delay.
The normal CGG repeat size in the FMR1 gene is 5 to 44; the occurrence of more than 200 repeats results in
FXS in male patients and some female patients; male patients are generally more severely affected. Although
individuals with repeat sizes greater than 44 and less than 200 do not have FXS, they are at risk for other
conditions (Table 2). Other common trinucleotide repeat disorders include Huntington disease, type 1
myotonic dystrophy, and Friedreich ataxia.
Chromosome analysis is the diagnostic test of choice for aneuploidy and balanced translocation.
Chromosomal microarray testing detects duplications and deletions as small as 50 kb across the genome.
Methylation analysis is the diagnostic test of choice for an imprinting disorder (eg, Prader-Willi syndrome and
Angelman syndrome). None of these testing methods would detect expansion of a trinucleotide repeat.
Common genetic testing methods are described in Table 3.
Suggested Reading(s)
Adam MP, Mirzaa GM, Pagon RA, et al, eds. GeneReviews® [Internet]. University of Washington,
Seattle; 1993-2022.
Fragile X syndrome. Point-of-Care Quick Reference. Pediatric Care Online. April 8, 2019. Accessed
February 12, 2023. Pediatric Care Online
Nussbaum RL, McInnes RR, Willard HF. The molecular, biochemical, and cellular basis of genetic
disease. In: Nussbaum RL, McInnes RR, Willard HF. Thompson and Thompson Genetics in Medicine. 8th
ed. Elsevier; 2016:215-280.
Saul RA. Fragile X syndrome. In: McInerny TK, Adam HM, Campbell DE, et al. American Academy of
Pediatrics Textbook of Pediatric Care. 2nd ed. American Academy of Pediatrics; 2017:chap 253.
Accessed April 2, 2023. Pediatric Care Online
Saul RA; American Academy of Pediatrics Committee on Genetics. Overview of genetic testing. In:
Medical Genetics in Pediatric Practice. American Academy of Pediatrics; 2014:235-252. Accessed April 2,
2023. Pediatric Care Online
Saul RA; American Academy of Pediatrics Committee on Genetics. Specific genetic conditions. In:
Medical Genetics in Pediatric Practice. American Academy of Pediatrics; 2014:179-234. Accessed April 2,
2023. Pediatric Care Online
Content Domain
Genetics
Behavior Aggression
Anxiety
Attention-deficit/hyperactivity disorder
Autism spectrum disorder
Oppositional defiant disorder
Temper tantrums
Craniofacial Large ears
Long face
Macrocephaly
Prominent forehead
Prominent jaw
Genitourinary Postpubertal macroorchidism
Musculoskeletal Joint hyperextensibility
Pes planus
Neurological Seizures
Courtesy of A. Sidhu
Chromosome analysis • Aneuploidy (eg Trisomy 21, Trisomy 13, • Can detect balanced and • Low resolution (5,000-10,000 kb)
Trisomy 18, Turner syndrome) unbalanced translocation • Need for tissue culture
• Recurrent (≥3) first trimester miscarriages • Genome-wide view • 14 day turnaround time, 5-6 days for
in a couple (looking for balanced newborns
translocation)
Fluorescence in situ hybridization (FISH) • Phenotype consistent with a specific gene • Rapid turnaround time (48 hours) • Probe-specific to target locus,
duplication or deletion (eg 22q11.2 deletion • Reliability of target locus individual probes must be designed
syndrome) and ordered.
• Need for quick results in a critically ill • Highest resolution is 300 kb
patient
Chromosomal microarray • Multiple congenital anomalies not • Whole-genome analysis • Cannot detect balanced
suggestive of a specific syndrome • High resolution (detects translocation or identify location of
• Unexplained non-syndromic developmental duplications and deletions as small duplication
delay as 50 kb) • 14-21 day turnaround time
• Autism spectrum disorder
Methylation analysis • Imprinting disorder (eg Prader-Willi • Specificity to the locus being tested • Southern blot assay is time-
Syndrome, Angelman syndrome) consuming and laborious
• 21 day turnaround time
Trinucleotide repeat analysis • Repeat expansion disorder (eg, fragile-X • Specific to the disorder being tested • 14-21 day turnaround time
syndrome, Huntington disease, type 1 • Additional testing required for large
myotonic dystrophy) repeat sizes
Whole exome sequencing • Not first-line for any indication • Sequences protein-coding regions • Cost, may not be covered by
of all genes to identify disease- insurance
causing change in the proband • 3-4 month turnaround time
Courtesy of A. Sidhu
AAP PREP 2024 - Question 92/267 Genetics Question 9/13
A 3-year-old girl is undergoing a routine health supervision visit. She has a history of abdominal distension
since birth. She has no vomiting, diarrhea, constipation, headaches, or easy fatigability. Her 5-year-old-sibling
has a history of autosomal recessive polycystic kidney disease.
The girl’s weight and height are at the 25th percentile for age. Her blood pressure is 122/82 mm Hg, her heart
rate is 92 beats/min, and her respiratory rate is 16 breaths/min. Her abdomen is distended and soft, with
mild hepatosplenomegaly, normal bowel sounds, and no free fluid. The remainder of her physical
examination findings are unremarkable.
Renal ultrasonography shows bilateral enlarged kidneys, poor corticomedullary differentiation, and multiple
microcysts.
A. a cerebral aneurysm
B. colonic diverticula
C. a hepatic cyst
D. portal brosis
Correct answer is D
PREP Pearl(s)
Neonates with autosomal recessive polycystic kidney disease may exhibit nephromegaly and
respiratory distress.
Children and adolescents with autosomal recessive polycystic kidney disease exhibit hypertension,
urinary tract infection, nephrolithiasis, hematuria, proteinuria, and/or chronic kidney disease.
Autosomal recessive polycystic kidney disease is always associated with portal and interlobular fibrosis
of the liver and dilation of the intrahepatic bile ducts.
Critique
The girl in the vignette has autosomal recessive polycystic kidney disease (ARPKD). Her abdominal distension
(secondary to nephromegaly), hypertension, family history of ARPKD, and renal ultrasonographic findings of
enlarged kidneys with microcysts favor this diagnosis. The most common condition associated with ARPKD in
children is portal fibrosis, which may result in gastroesophageal varices.
Autosomal recessive polycystic kidney disease is a rare genetic disorder with an incidence of 1 in 10,000 to
40,000 live births. One-third of cases present in infancy, one-third present between ages 1 and 20 years, and
the remainder present in adulthood. Infantile forms are detected when prenatal ultrasonography shows
enlarged hyperechoic kidneys. Oligohydramnios may be present in severe cases. At birth, abdominal masses
may be palpated and the neonate may develop respiratory distress owing to diaphragmatic elevation, lung
hypoplasia, or both. Infants can have difficult-to-treat hypertension. Affected children and adolescents exhibit
hypertension, urinary tract infection, nephrolithiasis, hematuria, proteinuria, and/or chronic kidney disease.
Renal ultrasonography in ARPKD shows large echogenic kidneys with poor corticomedullary differentiation
and microcysts (<2 mm in diameter). Macrocysts are seen in autosomal dominant polycystic kidney disease;
they are not usually present in infants with ARPKD.
Children with ARPKD always have associated portal and interlobular fibrosis of the liver and dilation of the
intrahepatic bile ducts. Portal hypertension develops and may result in gastrointestinal bleeding, esophageal
varices, hepatosplenomegaly, and hypersplenism. Anemia, thrombocytopenia, and leukopenia may occur
owing to splenic sequestration. Children with ARPKD are at risk of developing acute cholangitis.
Hepatic cysts, colonic diverticula, and cerebral aneurysms are extrarenal manifestations seen in autosomal
dominant polycystic kidney disease; they are not usually seen in children with ARPKD.
Children with ARPKD should be closely monitored for worsening kidney function, hypertension, and
infections. Periodic endoscopy to monitor for gastroesophageal varices and portal hypertension is
recommended. Once progressive renal insufficiency ensues, dialysis or kidney transplant is required.
Suggested Reading(s)
Benun J, Lewis C. Polycystic kidney disease. Pediatr Rev. 2009;30(10):e78-e79. doi:10.1542/pir.30-10-e78
Janjua HS, Lam KS, Gupta V, Krishna S. Congenital anomalies of the kidneys, collecting system, bladder,
and urethra. Pediatr Rev. 2019;40(12):619-626. doi:10.1542/pir.2018-0242
Content Domain
Genetics
A term male neonate is admitted to the neonatal intensive care unit for hypoglycemia. Shortly after birth, he
required treatment with a dextrose bolus for a blood glucose level of 35 mg/dL (1.94 mmol/L), after which he
had recurrent episodes of hypoglycemia. The neonate’s birth weight was 4.2 kg. The pregnancy was
unremarkable, including no maternal history of gestational diabetes. Physical examination findings include
macroglossia, posterior helical pits, umbilical hernia, and lower extremity asymmetry (left side larger than the
right). The remainder of the physical examination findings are normal.
A. Beckwith-Wiedemann syndrome
C. Silver-Russell syndrome
D. Sotos syndrome
Correct answer is A
PREP Pearl(s)
Beckwith-Wiedemann syndrome is an overgrowth disorder. Cardinal clinical features include
macroglossia, lateralized overgrowth, omphalocele, multifocal and/or bilateral Wilms tumor or
nephroblastomatosis, and hyperinsulinism lasting >1 week.
Suggestive clinical features of Beckwith-Wiedemann syndrome include polyhydramnios,
placentomegaly, birth weight >2 standard deviations above the mean, facial nevus simplex, ear creases
and/or pits, transient hypoglycemia lasting <1 week, typical tumors (neuroblastoma,
rhabdomyosarcoma, unilateral Wilms tumor, hepatoblastoma, adrenocortical carcinoma, or
pheochromocytoma), nephromegaly and/or hepatomegaly, and umbilical hernia and/or diastasis recti.
Children with Beckwith-Wiedemann syndrome and those with isolated hemihyperplasia are at risk for
embryonal tumors. Tumor surveillance for children with Beckwith-Wiedemann syndrome includes
serum α-fetoprotein level every 3 to 4 months until age 4 years and abdominal ultrasonography every
3 to 4 months until age 8 years.
Critique
Of the response choices, the most likely diagnosis for the neonate in the vignette is Beckwith-Wiedemann
syndrome (BWS). This overgrowth disorder has an incidence rate of 1:13,700 births. The diagnosis of BWS can
be made with a BWS methylation study or CDKN1C gene analysis. In the absence of a molecular or gene-
sequencing diagnosis, a clinical diagnosis of BWS can be made using the features defined in the expert
consensus document (Brioude 2018).
The clinical features of BWS are divided into 2 categories, cardinal and suggestive. Points are given for each
feature noted. A score of ≥4 supports a clinical diagnosis of BWS.
Macroglossia
Lateralized overgrowth (one side larger than the other)
Omphalocele
Multifocal and/or bilateral Wilms tumor or nephroblastomatosis
Hyperinsulinism (lasting >1 week and requiring escalated treatment)
Pathology findings: adrenal cortex cytomegaly, placental mesenchymal dysplasia, or pancreatic
adenomatosis
Recommended evaluation at the time of diagnosis and ongoing surveillance for children with BWS are shown
in the Table.
Isolated hemi-hyperplasia can occur. It is important to recognize that children with isolated hemi-hyperplasia
are also at-risk for embryonal tumors and that some cases of isolated hemi-hyperplasia are believed to be on
the BWS spectrum. Therefore, for children with isolated hemi-hyperplasia, current practice guidelines
recommend tumor screening similar to that for BWS.
PTEN hamartoma tumor syndrome (PHTS) is an overgrowth disorder characterized by macrocephaly,
developmental delay, autism spectrum disorder, vascular malformations, and risk for various tumors, both
benign (lipoma, thyroid adenoma, multinodular goiter, hamartomatous intestinal polyps, fibromas, cerebellar
dysplastic gangliocytoma), and malignant (breast cancer, thyroid cancer, endometrial carcinoma, renal cell
carcinoma). Postnatal hypoglycemia, posterior helical pits, umbilical hernia, and macroglossia are not
features of PHTS.
Silver-Russell syndrome (SRS) is an imprinting disorder characterized by both pre- and postnatal growth
delay, limb-length asymmetry, feeding difficulties, and normal head circumference. Other features include
developmental delay, fifth finger clinodactyly, frontal bossing, triangular facies, and skin pigmentary changes.
While lower extremity asymmetry can be seen in SRS, the neonate in the vignette’s other features are more
consistent with BWS.
Sotos syndrome is an overgrowth disorder characterized by dysmorphic facial features (broad forehead,
sparse hair, downslanting palpebral fissures, long and narrow face), overgrowth (height and/or head
circumference), and developmental delay. Sotos syndrome and BWS have several similar clinical features,
however, posterior helical pits, umbilical hernia, lateralized overgrowth, and macroglossia are not features of
Sotos syndrome.
Suggested Reading(s)
Brioude F, Kalish JM, Mussa A, et al. Expert consensus document: Clinical and molecular diagnosis,
screening and management of Beckwith-Wiedemann syndrome: an international consensus
statement. Nat Rev Endocrinol. 2018;14(4):229-249. doi:10.1038/nrendo.2017.166
Pappas JG. The clinical course of an overgrowth syndrome, from diagnosis in infancy through
adulthood: the case of Beckwith-Wiedemann syndrome. Curr Probl Pediatr Adolesc Health Care.
2015;45(4):112-117. doi:10.1016/j.cppeds.2015.03.001
Rosen O, Marion RW, Samanich JM. Common congenital anomalies. In: McInerny TK, Adam HM,
Campbell DE, DeWitt TG, Foy JM, Kamat DM, eds. American Academy of Pediatrics Textbook of Pediatric
Care. American Academy of Pediatrics; 2021:chap 96. Accessed September 1, 2023. Pediatric Care
Online
Shuman C, Beckwith JB, Weksberg R. Beckwith-Wiedemann syndrome. GeneReviews [Internet].
University of Washington, Seattle; 2021. Accessed September 1, 2023.
https://www.ncbi.nlm.nih.gov/books/NBK1394/
Content Domain
Genetics
Courtesy of A. Sidhu
AAP PREP 2024 - Question 94/267 Genetics Question 11/13
A 36-year-old, pregnant woman is seen for a prenatal visit at 18 weeks’ gestation. The anatomy scan is
remarkable for growth restriction and multiple fetal anomalies, including holoprosencephaly, bilateral
postaxial polydactyly of the hands, shortened long bones, cleft lip, and ventricular septal defect.
Of the following, the BEST next step in fetal diagnostic testing would be to perform
Correct answer is B
PREP Pearl(s)
Cell-free DNA testing on a maternal blood sample is the most sensitive and specific screening test for
the common aneuploidies (eg, trisomy 13).
Chorionic villus sampling is a prenatal diagnostic test that is performed between 10 and 13 weeks of
gestation.
Amniocentesis is a diagnostic prenatal genetic test that is typically performed between 15 and 20
weeks of gestation.
Critique
The most likely diagnosis for the fetus in the vignette is trisomy 13. The best next step in diagnostic testing
would be to perform chromosome analysis on an amniocentesis sample.
Amniocentesis is a diagnostic prenatal genetic test that is typically performed between 15 and 20 weeks of
gestation. With ultrasonographic guidance, 15 to 20 mL of amniotic fluid is collected and sent for culture,
chromosome analysis, and testing for many other genetic disorders (eg, detection of neural tube defects by
direct measurement of ɑ-fetoprotein). The risk of miscarriage after amniocentesis is estimated to be 0.11% (1
in 900); the baseline risk of pregnancy loss is 1% to 2% at 15 to 20 weeks’ gestation.
The American College of Obstetricians and Gynecologists recommends that prenatal screening or diagnostic
testing for aneuploidy be offered to all pregnant women. The most common indications for recommending
prenatal diagnostic testing include the following:
Cell-free DNA testing on a maternal blood sample is the most sensitive and specific screening test for the
common aneuploidies (eg, trisomy 13). It can be performed from approximately 9 to 10 weeks’ gestation until
term. It is important to remember that cell-free DNA is a prenatal screening test and cannot be used for
diagnosis. If the results demonstrate an increased risk of trisomy 13, a diagnostic test (eg, chorionic villus
sampling or amniocentesis) will be required to confirm the diagnosis; the appropriate test is determined
based on the gestational age.
Chorionic villus sampling is a prenatal diagnostic test performed between 10 and 13 weeks of gestation. A
transcervical or transabdominal approach is used to obtain placental villi with ultrasonographic guidance.
Diagnostic genetic testing is performed on the sample (eg, chromosome analysis, chromosome microarray,
fluorescence in situ hybridization for a target locus, single-gene analysis, and methylation analysis) based on
specific indications.
Although chromosome analysis on a chorionic villus sample can be used to confirm a diagnosis of trisomy 13,
this test would be inappropriate for this 18 weeks’ gestation fetus because it has passed the time frame for
this method of sample collection.
Fetal magnetic resonance imaging would not be appropriate because it would not confirm this fetus’
suspected diagnosis (trisomy 13).
Suggested Reading(s)
American College of Obstetricians and Gynecologists’ Committee on Practice Bulletins—Obstetrics.
Screening for fetal chromosomal abnormalities: ACOG Practice Bulletin Number 226. Obstet Gynecol.
2020;136(4):348-369. doi:10.1097/AOG.0000000000004084
Nussbaum RL, McInnes RR, Willard HF. Prenatal diagnosis and screening. In: Nussbaum RL, McInnes
RR, Willard HF. Thompson and Thompson Genetics in Medicine. 8th ed. Elsevier; 2016:349-368.
Practice Bulletin No. 162: prenatal diagnostic testing for genetic disorders. Obstet Gynecol.
2016;127(5):e108-e122. doi:10.1097/AOG.0000000000001405
Wolfe DS, Suskin B. Prenatal diagnosis. In: McInerny TK, Adam HM, Campbell DE, DeWitt TG, Foy JM,
Kamat DM, eds. American Academy of Pediatrics Textbook of Pediatric Care. 2nd ed. American
Academy of Pediatrics; 2017:chap 82. Pediatric Care Online
Content Domain
Genetics
A 1-day-old term neonate is seen in the neonatal intensive care unit. He is intubated due to increased work
of breathing. The pregnancy was remarkable only for intrauterine growth restriction. His weight is 2.3 kg,
length is 46 cm, and head circumference is 31 cm. He has a cat-like cry (noted prior to intubation), round
face, hypertelorism, epicanthal folds, and generalized hypotonia. The remainder of his physical examination
findings are unremarkable. Two-dimensional echocardiography showed an atrial septal defect, and renal
ultrasonography showed a horseshoe-shaped kidney. A routine chromosome analysis provides the
diagnosis.
A. chromosome microdeletion
C. single-gene defect
Correct answer is B
PREP Pearl(s)
Gross chromosome deletions can be detected by high-resolution, cytogenetic-banding techniques and
are typically 3 to 5 Mb or larger in size.
Chromosome microdeletions are approximately 1.5 to 4 Mb in size and cannot be detected by
conventional high-resolution cytogenetic banding technique.
Cri du chat syndrome is caused by a partial deletion of the short arm of chromosome 5. Common
clinical features include catlike cry, prenatal and postnatal growth retardation, hypotonia, and
dysmorphic facial features (microcephaly, hypertelorism, epicanthal folds, low-set ears, micrognathia,
and round face).
Critique
The neonate in the vignette has Cri du chat or chromosome 5p deletion (5p-) syndrome. Cri du chat
syndrome is caused by a partial deletion of the short arm of chromosome 5. Gross chromosome deletions
can be detected by high-resolution, cytogenetic-banding techniques and are typically 3 to 5 Mb or larger in
size. Another example of a gross chromosome deletion is Wolf-Hirschorn syndrome, with deletion of the
short arm of chromosome 4 (4p-). The size of the deletion in Cri du chat syndrome can vary among
individuals; the critical region identified is band 5p15. Affected individuals can exhibit phenotypic variability
based on the size and location of the deletion (Table). Conventional chromosome analysis cannot detect
microdeletions, single-gene disorders, or triplet repeat expansion disorders.
The majority (85%) of Cri du chat cases are sporadic; the remaining 15% have a parent with a balanced
translocation. The major causes of death in children with Cri du chat syndrome include respiratory distress
syndrome, pneumonia, and congenital heart defects.
Chromosome microdeletions are approximately 1.5 to 4.0 Mb in size and cannot be detected by conventional
high resolution cytogenetic banding technique. Common examples of chromosomal microdeletions include
22q11.2 and 1p26 deletion syndromes.
Single-gene disorders are caused by pathogenic variants in a single gene. These are identified by sequencing
and deletion/duplication analysis of the gene known to cause the disorder. Common examples include
Marfan syndrome (gene: FBN1) and neurofibromatosis type 1 (gene: NF1).
Triplet repeat expansion disorders are caused by an increase in the number of triplet repeats in a gene.
Common examples include fragile X syndrome (>200 CGG repeats in the FMR1 gene) and myotonic dystrophy
type 1 (>50 CTG repeats in the DMPK gene).
Suggested Reading(s)
The chromosomal and genomic basis of disease: disorders of the autosomes and sex chromosomes.
In: Nussbaum RL, McInnes RR, Willard HF, eds. Thompson and Thompson Genetics in Medicine. 8th ed.
Elsevier Inc; 2016:75-102.
Chromosomal abnormality syndromes identifiable on routine karyotype. In: Jones K, Jones MC, del
Campo M, eds. Smith’s Recognizable Patterns of Human Malformation. 8th ed. Elsevier Inc; 2021:1-70.
National Organization for Rare Disorders Inc. Rare disease database. Accessed September 1, 2023.
https://rarediseases.org/rare-diseases/cri-du-chat-syndrome/
Content Domain
Genetics
A 9-year-old girl is seen for a health supervision visit. Her medical history is remarkable for supravalvular
aortic stenosis, mild intellectual disability, and hypercalcemia. Physical examination findings are remarkable
for joint laxity, soft skin, and facial features of bitemporal narrowing, periorbital fullness, stellate iris pattern,
long philtrum, full lips, and a wide mouth. She is very talkative throughout the visit, and her mother describes
the girl as outgoing, sensitive, and very friendly.
Of the following, the MOST likely diagnosis is
B. Prader-Willi syndrome
C. Turner syndrome
D. Williams syndrome
Correct answer is D
PREP Pearl(s)
Williams syndrome is a contiguous gene deletion syndrome that involves the Williams-Beuren
syndrome critical region at 7q11.23 that includes the elastin gene, ELN.
Contiguous gene deletion syndromes (eg, Williams syndrome and 22q11.2 deletion syndrome) are
inherited in an autosomal dominant pattern.
The diagnostic test of choice for a contiguous gene deletion syndrome (eg, Williams syndrome and
22q11.2 deletion syndrome) is a chromosomal microarray.
Critique
On the basis of the medical history, physical findings, and characteristic personality of the patient in the
vignette, the most likely diagnosis is Williams syndrome (WS). Williams syndrome is a contiguous gene
deletion syndrome that involves the Williams-Beuren syndrome critical region (WBSCR) at 7q11.23 that
includes the elastin gene, ELN. The diagnostic test of choice for a contiguous gene deletion syndrome is a
chromosomal microarray. Williams syndrome can be diagnosed by chromosomal microarray (preferred
modality) or by a targeted fluorescent in situ hybridization test for the WBSCR at 7q11.23. Contiguous gene
deletion syndromes, such as WS, are inherited in an autosomal dominant pattern. Although there are rare
familial occurrences, most cases of WS occur de novo; affected individuals are not known to reproduce. The
clinical features of WS are outlined in the Table.
22q11.2 deletion syndrome is a contiguous gene deletion syndrome inherited in an autosomal dominant
pattern. Common clinical features include cardiac malformation (tetralogy of Fallot and truncus arteriosus),
palatal anomalies (cleft palate and velopharyngeal incompetence), developmental delay, immunodeficiency,
craniofacial features (bulbous nose, micrognathia, and prominent nasal bridge), and cognitive deficits.
22q11.2 deletion syndrome is characterized by hypocalcemia; hypercalcemia is seen in WS. This girl’s
supravalvular aortic stenosis, characteristic personality, hypercalcemia, and physical examination findings
favor a diagnosis of WS.
Prader-Willi syndrome (PWS), an imprinting disorder, occurs due to an abnormality in expression of genes in
the paternally derived PWS region. One of the mechanisms that leads to this abnormality is a contiguous
gene deletion of 15q11.2-q13. Clinical features of PWS include hypotonia, failure to thrive, hypogonadism,
and feeding difficulties in infancy. In early childhood, individuals present with hyperphagia, short stature, and
behavioral concerns. Supravalvular aortic stenosis and hypercalcemia are not seen in PWS.
Turner syndrome is a sex chromosome aneuploidy characterized by congenital lymphedema, webbed neck,
dysmorphic facial features, short stature, gonadal dysgenesis, and renal and cardiac anomalies (bicuspid
aortic valve and coarctation of aorta). Hypercalcemia and the characteristic personality as described in the
vignette are not seen in Turner syndrome.
Suggested Reading(s)
Fanella S, Embree J. Index of suspicion. Pediatr Rev. 2008;29(8):281-287. doi:10.1542/pir.29-8-281
Morris CA. Williams syndrome. In: Adam MP, Mirzaa GM, Pagon RA, et al, eds. GeneReviews®
[Internet]. University of Washington, Seattle; 1993-2022.
Morris CA, Braddock SR; Council on Genetics. Health care supervision for children with Williams
syndrome. Pediatrics. 2020;145(2):e20193761. doi:10.1542/peds.2019-3761
Pereira E, Marion R. Contiguous gene syndromes. Pediatr Rev. 2018;39(1):46-49. doi:10.1542/pir.2016-
0073
Saul RA; American Academy of Pediatrics Committee on Genetics. Specific genetic conditions. In:
Medical Genetics in Pediatric Practice. American Academy of Pediatrics; 2014:179-234.
Content Domain
Genetics
Personality Empathic
Friendly
Very talkative
Anxious, sensitive
Courtesy of A. Sidhu
AAP PREP 2024 - Question 97/267 Hematology Question 1/6
A 6-year-old girl is seen in the emergency department for fever, cough, runny nose, intermittent nonbloody,
nonbilious emesis, and decreased oral intake. There is no diarrhea or urinary symptoms. The girl has sickle
cell trait. She had an episode of jaundice and tachycardia a few years prior. Physical examination reveals a
temperature of 39.4°C, heart rate of 146 beats/min, respiratory rate of 30 breaths/min, blood pressure of
111/45 mm Hg, and oxygen saturation of 100% in room air. The girl is not in significant distress; she looks ill
but nontoxic. Her mucous membranes are moist and there is no scleral icterus. Her lungs are clear to
auscultation bilaterally; there is tachycardia at rest with brisk capillary refill; and her abdomen is soft and not
distended, with palpable splenomegaly 5 fingerbreadths below the left costal margin. There is no palpable
lymphadenopathy, no skin rash, and no neurologic deficits appreciated.
Laboratory data are shown:
Neutrophils 67%
Lymphocytes 23%
Monocytes 10%
Reticulocyte 2.9%
Hemoglobin A 56%
Hemoglobin A2 3%
Hemoglobin S 41%
Of the following, the BEST next step in this child’s management is administration of
A. intravenous antibiotics
D. platelets, 10 mL/kg
Correct answer is C
PREP Pearl(s)
Hereditary spherocytosis is a cause of anemia, jaundice, and splenomegaly.
Antibiotic prophylaxis is required for at least 2 years post-splenectomy for children with hereditary
spherocytosis.
Splenectomy may increase the risk of thrombosis and pulmonary hypertension in children with
hereditary spherocytosis.
Critique
The child in the vignette has severe anemia, mild thrombocytopenia, and splenomegaly. These findings may
be associated with several diagnoses. However, in a child without sickle cell disease, the presence of anemia,
elevated indirect bilirubin level (with or without jaundice), and splenomegaly make a diagnosis of hereditary
spherocytosis very likely. Because this child has severe anemia without adequate reticulocytosis, transfusion
of packed red blood cells (RBCs) is the best next step in her management.
Hereditary spherocytosis (HS) is an inherited, usually autosomal dominant disorder (75%; the remainder are
recessive) of the RBC membrane. Pathogenic variants in genes related to proteins involved in the formation
of RBC membrane structure lead to loss of the lipid bilayer and surface area of the RBC, causing the usually
biconcave RBC to become a sphere. The abnormally shaped RBCs become trapped in the spleen during
circulation, where they are engulfed by macrophages. This leads to splenomegaly and hemolysis, causing
anemia, indirect hyperbilirubinemia, and, in many cases, gallstones. Clinical signs and symptoms may include
pallor and fatigue due to the anemia, and jaundice from the indirect hyperbilirubinemia. Direct
hyperbilirubinemia may occur in the setting of cholelithiasis. In addition to anemia, laboratory findings may
include reticulocytosis (absent in the child in the vignette due to viral myelosuppression from the influenza
infection), elevated mean corpuscular hemoglobin concentration (MCHC), and increased RBC distribution
width. Iron deficiency may alter the expected laboratory parameters (eg, normalize MCHC or lack of
reticulocytosis)
The diagnosis of HS can be made using a combination of history, physical, and laboratory findings. A known
family history, along with spherocytes seen on review of the child’s blood smear (Figure), can support the
diagnosis. A positive osmotic fragility test result can confirm the diagnosis. This test involves incubation of the
child’s blood sample with saline solutions of different concentrations. The RBCs in HS lyse at a higher saline
concentration than normal RBCs. However, results of the osmotic fragility test may be falsely normal in a
newborn due to the presence of fetal hemoglobin. Eosin-5-maleimide flow cytometry has a higher sensitivity
and specificity for HS. Genetic testing is also available to detect pathogenic variants.
The management of HS depends on the disease severity. Most affected individuals are started on daily folic
acid supplementation at the time of diagnosis. A RBC transfusion is indicated in the setting of severe anemia
(eg, aplastic crisis resulting from a parvovirus B19 infection). Individuals with frequent and severe episodes of
anemia and splenomegaly may benefit from splenectomy. As splenectomy renders the individual
immunocompromised, it is imperative to ensure vaccination against encapsulated organisms (eg,
Streptococcus pneumoniae, Neisseria meningitidis, and Haemophilus influenzae) prior to splenectomy. Post
splenectomy, prophylactic antibiotics (eg, oral penicillin) must be taken for at least 2 years, and possibly for
as long as the individual’s lifetime. Febrile episodes in asplenic children require urgent assessment, including
a blood culture, and administration of broad-spectrum empiric antibiotics. Splenectomy may increase the risk
of thrombosis and pulmonary hypertension. In some younger children (<5 years of age) who are transfusion
dependent, a partial splenectomy may be considered to decrease the risk of sepsis.
The girl in the vignette’s signs and symptoms could raise concern for a bacterial infection, however, the
positive test result for influenza infection explains the fever and clinical findings. Because she is not
immunocompromised, intravenous antibiotics are not indicated at this time.
A normal saline bolus is not warranted for the girl in the vignette, as there is no evidence of dehydration on
examination (moist mucous membranes, brisk capillary refill) or laboratory findings. The tachycardia can be
attributed to her anemia and fever, supporting the need for a blood cell transfusion. A normal saline bolus is
not indicated, as this would dilute the blood, worsening the anemia and decreasing oxygen carrying capacity.
A platelet transfusion is not indicated for the girl in the vignette. She has mild thrombocytopenia with no
evidence of bleeding. This child’s thrombocytopenia is most likely the result of viral myelosuppression due to
her influenza infection.
Suggested Reading(s)
Blanc L, Wolfe LC. General considerations of hemolytic diseases, red cell membrane, and enzyme
defects. In: Lanzkowsky P, Lipton J, Fish JD, eds. Lanzkowsky’s Manual of Pediatric Hematology and
Oncology. 6th ed. Elsevier; 2016:134-143.
Mahajan V, Jain SK. Hereditary spherocytosis. NeoReviews. 2016;17(12):e697-e704. doi:10.1542/neo.17-
12-e697
Noronha SA. Acquired and congenital hemolytic anemia. Pediatr Rev. 2016;37(6):235-246.
doi:10.1542/pir.2015-0053
Shah S, Vega R. Hereditary spherocytosis. Pediatr Rev. 2004;25(5):168-172. doi:10.1542/pir.25-5-168
Content Domain
Hematology
Courtesy of A. Appiah-Kubi
FIGURE: Peripheral blood smear of an individual with hereditary spherocytosis.
AAP PREP 2024 - Question 98/267 Hematology Question 2/6
A 15-year-old girl is seen for a routine health supervision visit. She has been doing well and has not had any
significant illness. Menarche was at age 12 years. Her menses occur monthly, last for 8 days, and she uses 8
pads per day. Menstrual flow is heaviest the first 2 days. She eats a well-balanced diet with no food
restrictions. Her vital signs are normal for age. Other than pallor of the conjunctivae, her examination
findings are unremarkable.
Laboratory data are shown:
Neutrophils 49%
Lymphocytes 40%
Monocytes 10%
Eosinophils 1%
Reticulocytes 1%
Unsaturated iron-binding 300 µg/dL (53.7 µmol/L) (reference range, 110-370 µg/dL [19.7-66.2
capacity µmol/L)
Correct answer is C
PREP Pearl(s)
Menorrhagia is a common cause of iron-deficiency anemia.
Menorrhagia requires investigation for an underlying bleeding disorder.
Because ferritin is also a marker of inflammation, the level can sometimes be normal in cases of iron
deficiency and should be interpreted with caution.
Critique
The girl in the vignette has iron-deficiency anemia. This diagnosis is supported by laboratory findings of
microcytic anemia, increased red blood cell distribution width and total iron-binding capacity, as well as low
iron level, low ferritin level, and low percentage iron saturation. The best next step in her management is iron
replacement therapy.
Iron-deficiency anemia may occur in menstruating females due to heavy menses. Menstrual bleeding that
lasts more than 7 days, requires the use of more than 7 pads per day, or has blood loss of more than 80
mL/cycle, is defined as menorrhagia. It may be difficult to quantify menses. Some individuals with heavy
bleeding think their menses are normal, especially if family members have similar bleeding patterns. A
pictorial bleeding assessment calendar (PBAC) can help quantify the bleeding. A score greater than 100 is
indicative of significant blood loss and warrants a bleeding workup with the help of a hematologist. A sample
PBAC can be found at betteryouknow.org – Menstrual chart scoring system.
Once iron-deficiency anemia is identified, it is important to both replenish the iron and determine and
address the underlying cause. The girl in the vignette should have a bleeding evaluation and be considered
for treatment to stop her heavy menses. Oral contraceptives may be an appropriate treatment to initiate
after evaluation, however, this treatment is not the best next step in her management. Because there is no
current menstrual bleeding that needs to be stopped, it is more appropriate to first treat her with iron.
A normal saline bolus is not needed for the girl in the vignette as she is hemodynamically stable. It is
important to note that, when required, administration of a normal saline bolus to a child with anemia should
be done with caution as hemodilution will worsen the anemia. An asymptomatic individual with anemia
typically does not require administration of packed red blood cells. It is more important to administer iron.
When assessing a child for iron deficiency, it is important to not only obtain iron studies (ie, iron, total iron-
binding capacity, unsaturated iron-binding capacity) but also a ferritin level, which is an indicator of iron
stores. Because ferritin is also a marker of inflammation, the level can sometimes be normal in cases of iron
deficiency and should be interpreted with caution. A stool guaiac test should be considered for children with
anemia, to assess for gastrointestinal blood loss. Other causes of iron-deficiency anemia include inadequate
dietary intake or malabsorption; the history and physical examination can help determine whether further
evaluation to investigate for those causes may be appropriate.
Iron replacement therapy can be administered orally or intravenously. Oral iron therapy can be provided at a
dose of 3 mg/kg by mouth once daily. It is important to separate the time of oral iron administration from the
intake of any milk or dairy products by at least 1 hour, as such products prevent the absorption of iron. It
takes several months to replenish iron stores (about 3 months of oral iron therapy). By about 1 week of oral
iron therapy, there should be an increase in the reticulocyte count. The rise in hemoglobin occurs later.
Intravenous iron may be administered if there is severe anemia but hemodynamic stability. After an initial
dose of intravenous iron, treatment may continue with oral iron. Close follow-up is required to ensure there
is no further need for intravenous iron. Iron supplementation should be continued until iron stores are
replenished (evidenced by a normal ferritin level), which may take several months depending on the iron
administration method (intravenous vs oral).
Suggested Reading(s)
Centers for Disease Control and Prevention. Heavy menstrual bleeding. Accessed September 1, 2023.
https://www.cdc.gov/ncbddd/blooddisorders/women/menorrhagia.html
Graham RA, Davis JA, Corrales-Medina FF. The adolescent with menorrhagia: diagnostic approach to a
suspected bleeding disorder. Pediatr Rev. 2018;39(12):588-600. doi:10.1542/pir.2017-0105
Krishnamurti L. Iron-deficiency anemia. In: McInerny TK, Adam HM, Campbell DE, DeWitt TG, Foy JM,
Kamat DM, eds. American Academy of Pediatrics Textbook of Pediatric Care. 2nd ed. American
Academy of Pediatrics; 2023. Accessed September 1, 2023.
https://publications.aap.org/pediatriccare/book/348/chapter/5782173/Iron-Deficiency-Anemia?
searchresult=1
Wolfe, LC. Extracorpuscular hemolytic anemia. In: Lanzkowsky P, Lipton J, Fish JD. Lanzkowsky’s Manual
of Pediatric Hematology and Oncology. 6th ed. Elsevier Inc; 2016:159-165.
Watt MB. Vaginal bleeding. Point of Care Quick Reference. Pediatric Care Online. American Academy of
Pediatrics; 2020. Accessed September 1, 2022. Pediatric Care Online
Content Domain
Hematology
A 9-month-old, fully immunized boy is seen for a health supervision visit. He has had 3 episodes of
pneumonia requiring admission for intravenous antibiotics. He is currently well, and his parents have no
acute concerns. His growth is appropriate, and his physical examination findings are unremarkable.
After consultation with infectious diseases and immunology specialists, laboratory tests are obtained to
further investigate the cause of his recurrent pulmonary infections.
Neutrophils 65%
Lymphocytes 30%
Monocytes 5%
Reticulocytes 1.5%
Of the following, the BEST next step in this child’s management is to prescribe
A. corticosteroids
C. prophylactic antimicrobials
Correct answer is C
PREP Pearl(s)
Children with frequent or recurrent infections should be evaluated for a primary immunodeficiency.
Chronic granulomatous disease is a primary immunodeficiency disease most often inherited in an X-
linked (rarely autosomal recessive) pattern. Males are almost exclusively affected.
Individuals with chronic granulomatous disease require prophylactic antimicrobials to help prevent
serious infections.
Critique
The boy in the vignette has had several episodes of pneumonia requiring hospitalization, which has
prompted an investigation for possible immunodeficiency. Based on the nitroblue tetrazolium (NBT)
reduction test results, his most likely diagnosis is chronic granulomatous disease (CGD). His normal
neutrophil count makes a disorder of neutropenia less likely, and his IgG level is normal as well.
Chronic granulomatous disease is a primary immunodeficiency disease that is most often inherited in an X-
linked (rarely autosomal recessive) pattern. Males are almost exclusively affected. The disorder presents early
in childhood with recurrent bacterial (eg, Staphylococcus aureus and Burkholderia cepacia complex) and/or
fungal (eg, Aspergillus fumigatus) infections. Bowel and/or pulmonary inflammation may occur in children
with CGD. In addition to recurrent infections, individuals with CGD may develop an inflammatory bowel
disorder similar to Crohn disease (with granulomas).
Chronic granulomatous disease is a qualitative white blood cell disorder in which myeloid cells are unable to
phagocytose due to a defect in the oxidative burst. A defect of the nicotinamide adenine dinucleotide
phosphate (NADPH) oxidase enzyme leads to a lack of free radicals needed for killing certain bacteria and
fungi. The diagnosis is confirmed by demonstrating a lack of NADPH oxidase activity (indicative of an inability
to produce the reactive oxygen species that lead to phagocytosis). For a normal NBT test result, the yellow
reagent changes to blue when the NBT reacts with and forms a precipitate with the reactive oxygen species
(oxidative burst). In CGD, the reagent remains yellow. In the case of an X-linked carrier, there may be 2 result
populations (both colors). Flow cytometry or genetic testing for common pathologic variants may also be
used to determine the diagnosis.
Once the diagnosis of CGD is confirmed, prophylactic antimicrobials (eg, sulfamethoxazole-trimethoprim for
bacterial infections and itraconazole for fungal infections) should be prescribed to help prevent serious
illness. Whenever an infection is suspected, appropriate cultures should be obtained and empiric
antimicrobials given immediately. In individuals with inflammatory conditions (eg, colitis), steroid-sparing
agents should be considered to avoid increased immunosuppression. Cellular therapies (eg, hematopoietic
stem cell transplantation and gene therapy) are being explored as treatment options.
Corticosteroids are a potential treatment option for individuals with CGD inflammatory bowel disease. The
child in the vignette has no current gastrointestinal symptoms.
Subcutaneous gamma globulin may be a treatment of choice for a child with a primary immunodeficiency
with low IgG levels, which this child does not have.
Growth colony-stimulating factor is not a treatment option for CGD. The defect in CGD is qualitative;
therefore, increasing the amount of neutrophils will not overcome the problem or lead to an improved
outcome.
Suggested Reading(s)
Gennery AR. Progress in treating chronic granulomatous disease. Br J Haematol. 2021;192(2):251-264.
doi:10.1111/bjh.16939
Goldman DL. 2016. Recurrent infections. In: In: McInerny TK, Adam HM, Campbell DE, DeWitt TG, Foy
JM, Kamat DM, eds. American Academy of Pediatrics Textbook of Pediatric Care. American Academy of
Pediatrics; 2023. Accessed September 1, 2023. Pediatric Care Online
Content Domain
Hematology
ABP Content Specification(s) / Content Area(s)
Recognize the clinical presentation of chronic granulomatous disease
Plan and interpret the results of laboratory evaluation in a patient with chronic granulomatous disease
A 3-year-old with sickle cell disease, type SS, is seen in the emergency department for a febrile illness. Her
parents state that her baseline hemoglobin is 8 to 9 g/dL (80-90 g/L). She seems tired but is arousable. Her
vital signs include a temperature of 38.9 °C, heart rate of 140 beats/min, respiratory rate of 25 breaths/min,
blood pressure of 100/60 mm Hg, and oxygen saturation by pulse oximetry of 100% in room air. On physical
examination, she has conjunctival pallor with no scleral icterus. Her lungs are clear to auscultation bilaterally
with good aeration, and a systolic ejection murmur is heard at the left sternal border. Her abdomen is soft,
with a palpable spleen 7 cm below the left costal margin. The remainder of her examination findings are
normal. Intravenous access is obtained, blood is drawn for testing, and empiric antibiotics are administered.
Packed red blood cells are prepared for administration.
Laboratory data are shown:
Reticulocytes 1.6%
Of the following, the MOST appropriate volume of packed red blood cells to administer is
A. 6 mL/kg
B. 10 mL/kg
C. 15 mL/kg
D. 20 mL/kg
Correct answer is A
PREP Pearl(s)
Infection is a major cause of morbidity and mortality in individuals living with sickle cell disease;
prompt administration of empiric antibiotics is required for every febrile illness.
Parvovirus infection causes maturation arrest of red blood cell precursors, leading to severe anemia
with reticulocytopenia. Parvovirus infection may cause an acute splenic sequestration crisis in a child
with sickle cell disease.
In the setting of an acute splenic sequestration, red blood cell transfusions should be administered in
small aliquots (3-6 mL/kg) to prevent hyperviscosity, which may increase stroke risk.
Critique
The girl in the vignette should receive a transfusion of 6 mL/kg of packed red blood cells (RBCs). This child,
with sickle cell disease, has an acute splenic sequestration crisis (enlargement of the spleen with a decrease
in the hemoglobin level from baseline and thrombocytopenia), fever, and an aplastic crisis (severe anemia,
reticulocytopenia, and sometimes thrombocytopenia). Each of these conditions is a medical emergency for
an individual living with sickle cell disease. Infection leading to bacteremia and sepsis has a significant risk of
death; therefore, empiric broad-spectrum antibiotics (eg, ceftriaxone) must be administered in a timely
fashion. Her other concerning conditions can then be addressed.
The most likely cause of this girl’s aplastic crisis is a parvovirus infection, which causes an arrest of
maturation of RBC precursors. For a definitive diagnosis, parvovirus titers and/or polymerase chain reaction
testing may be performed; the results will take several days to return.
A transfusion of packed RBCs is the best next intervention to prevent hemodynamic instability. The volume of
blood to be administered must be carefully calculated to avoid reaching a level of hemoglobin that will lead
to hyperviscosity and increase the risk of stroke. The volume administered must take into consideration the
increase in hemoglobin associated with release of the RBCs sequestered in the spleen as the circulating
blood volume is replaced. Therefore, transfusions should be administered in small aliquots (3-6 mL/kg), with
close monitoring of the hemoglobin level and frequent examination of the spleen to ensure it is not
enlarging. Although this child has an aplastic crisis and will most likely require multiple transfusions, it is
safest to administer the RBCs in small aliquots.
An RBC transfusion of 6 mL/kg will result in an approximately 2 g/dL (20 g/L) increase in hemoglobin. If this
aliquot successfully reverses the sequestration, the hemoglobin will increase even further; if not, another
small aliquot may be administered. Higher transfusion volumes (a 20 mL/kg transfusion may result in a 6.0 to
7.0 g/dL [60 to 70-g/L] increase in the hemoglobin level) place the child at significant risk for hyperviscosity if
the sequestration process is reversed.
Suggested Reading(s)
Kwiatkowski JL. Hemoglobinopathies. In: Lanzkowsky P, Lipton J, Fish JD. Lanzkowsky’s Manual of
Pediatric Hematology and Oncology. 6th ed. Elsevier Inc; 2016:166-185.
McCavit TL. Sickle cell disease. Pediatr Rev. 2012;33(5):195-206. doi:10.1542/pir.33-5-195
Content Domain
Hematology
A 13-year-old girl with no significant medical history is evaluated for nonbloody, nonbilious emesis and
abdominal pain. The patient has no history of recent travel or known sick contacts. Menarche occurred at 11
years of age, and her last menses was 1 week ago. Her menses last 5 to 7 days, during which she uses 3 to 4
pads per day. She denies joint or extremity pain. Her vital signs are as follows: temperature, 39.4 ℃; heart
rate, 120 beats/min; respiratory rate, 30 breaths/min; blood pressure, 90/50 mm Hg; and oxygen saturation,
100% in room air.
She looks tired and in mild discomfort but otherwise has a nontoxic appearance. Her sclera are anicteric,
mucous membranes are moist, and her neck is supple with bilateral posterior cervical lymphadenopathy (2 ×
2 cm, mobile, nontender). Her lungs are clear to auscultation, cardiovascular examination reveals
tachycardia, and her abdomen is soft with mild tenderness to palpation. Organomegaly was not assessed
because of discomfort. She has full range of motion of her extremities, no focal neurologic deficits were
appreciated, and no rashes were noted. Intravenous fluids are administered.
Neutrophils 40%
Lymphocytes 52%
Monocytes 8%
Reticulocyte 0.5%
Abdominal sonography reveals mild hepatosplenomegaly with normal echotexture and bile ducts with
normal caliber.
On the basis of the available results, further laboratory tests were performed. The results are shown:
Epstein-Barr virus viral capsid antigen IgG enzyme 32.8 U/mL (reference range, ≤17.9
immunoassay U/mL)
Epstein-Barr nuclear antigen IgG enzyme immunoassay <3 U/mL (reference range, ≤17.9 U/mL)
Epstein-Barr virus early antigen antibody enzyme immunoassay 20 U/mL (reference range, <8.9 U/mL)
Of the following, the BEST next step in this patient’s management is administration of
A. antibiotics
B. intravenous iron
D. platelets
Correct answer is A
PREP Pearl(s)
Viral infections may cause myelosuppression, resulting in a transient pancytopenia.
Investigation of the bone marrow (aspirate and biopsy) is needed to diagnose most causes of
pancytopenia.
Critique
The girl in the vignette has pancytopenia, defined as a decrease in all 3 blood cell lines. The differential
diagnosis of pancytopenia is broad, including, but not limited to, infection, medication, and genetic
abnormalities associated with bone marrow failure. The most likely cause of this girl’s pancytopenia is viral
myelosuppression due to Epstein-Barr virus (EBV) infection. Her clinical signs and symptoms (fever,
hypotension, and neutropenia) are suggestive of sepsis. In addition to urgent administration of intravenous
fluids to address her hemodynamic status, the best next step in this immunocompromised girl’s
management is administration of empiric antibiotics due to her significant risk of secondary bacterial sepsis.
Even if this girl has iron deficiency anemia, intravenous iron administration would not be an appropriate
immediate intervention for this girl with pancytopenia and evidence of sepsis. Although she might benefit
from administration of packed red blood cells, she does not have severe anemia; therefore, this treatment
can be considered after she receives empiric antibiotics. Platelet administration is not warranted at this time
because the girl does not have severe thrombocytopenia or any signs or symptoms of bleeding.
Viral infections (eg, EBV) may cause transient viral myelosuppression of the bone marrow, leading to
pancytopenia. Treatment for this is supportive care; the bone marrow is expected to recover with time.
Hemophagocytic lymphohistiocytosis (HLH) is another cause of pancytopenia. The diagnostic criteria for HLH
include an identified genetic pathogenic variant or 5 of the following 8 criteria:
Cytopenias in 2 of the 3 blood cell lines
Elevated ferritin level
Elevated soluble CD25 (interleukin 2 receptor)
Fever
Hemophagocytosis in the spleen, bone marrow, or lymph node without evidence of malignancy
Hypertriglyceridemia and/or hypofibrinogenemia
Low or absent natural killer (NK) cell activity
Splenomegaly
Secondary HLH may occur as a result of an EBV infection. Therefore, children with EBV infection should be
closely monitored and repeat laboratory testing should be considered to ensure complete recovery.
Treatment for HLH secondary to EBV infection may include rituximab (anti-CD20 monoclonal antibody),
anakinra (interleukin 1 receptor antagonist), and/or dexamethasone. Primary HLH usually presents before 18
months of age and may be due to a genetic pathogenic variant. When a diagnosis of HLH is made, brain
magnetic resonance imaging and a lumbar puncture should be performed to assess for central nervous
system disease for which intrathecal treatment is indicated.
Leukemia causes pancytopenia due to bone marrow infiltration with abnormal blast cells. A bone marrow
investigation is the diagnostic test of choice to confirm leukemia; however, peripheral blood flow cytometry
can be helpful to assess for abnormal cells while waiting for a bone marrow investigation to be performed.
Moderate
<1,500/μL <40 × 103/µL <10 g/dL (<100 <50 × 103/µL
aplastic 25-50
(<1.50 × 109/L) (<40 × 109/L ) g/L) (<50 × 109/L)
anemia
Severe aplastic <500/μL (<0.50 <20 × 103/µL <10 g/dL (<100 <20 × 103/µL
<25
anemia × 109/L) (<20 × 109/L) g/dL) (<20 × 109/L)
Very severe
<200/μL (<0.20 <20 × 103/µL <10 g/dL (<100 <20 × 103/µL
aplastic <20
× 109/L) (<20 × 109/L) g/dL) (<20 × 109/L)
anemia
Courtesy of A. Appiah-Kubi
Once a diagnosis of aplastic anemia is made, it is important to determine whether there is any underlying
genetic predisposition.
Suggested Reading(s)
Kaplan JA. Leukemia in children. Pediatr Rev. 2019;40(7):319-331. doi:10.1542/pir.2018-0192
Noronha SA. Aplastic and hypoplastic anemias. Pediatr Rev. 2018;39(12):601-611. doi:10.1542/pir.2017-
0250
Sharma R, Nalepa G. Evaluation and management of chronic pancytopenia. Pediatr Rev.
2016;37(3):101-111; quiz 112-113. doi:10.1542/pir.2014-0087
Content Domain
Hematology
An 18-month-old child is seen for follow-up for a Staphylococcus aureus skin infection. Their medical history
includes an episode of periodontal disease treated 6 months ago, and they had umbilical cord separation at
4 weeks of age. With their current and previous infection, no pus was visualized.
Of the following, the test MOST likely to confirm this child’s diagnosis is
A. ow cytometry
Correct answer is A
PREP Pearl(s)
Recurrent infections with neutrophilia without pus should prompt investigation for a leukocyte
adhesion defect.
Children with a leukocyte adhesion defect require prophylactic antibiotics and prompt treatment of
infections.
Critique
The child in the vignette has a current staphylococcal skin infection and a history of delayed umbilical cord
separation and periodontal disease, which should raise concern for an underlying immune dysfunction
disorder. Their history and clinical findings are consistent with a qualitative leukocyte disorder; children with
these conditions present with recurrent infections of the skin, lung, and mucous membranes.
Leukocytes, once activated by a pathogen, migrate to the site of infection, where adhesion is needed for
chemotaxis. Appropriate adhesion does not occur in children with a leukocyte adhesion disorder. Children
affected by these autosomal recessive disorders have pathogenic variants in CD18, the common chain of β2-
integrin, which is required for adhesion and subsequent chemotaxis. This condition leads to impairment of all
downstream activity. There are 3 defined leukocyte adhesion deficiency syndromes (Table).
Total complement activity (CH50 assay) testing is used to detect complement system defects or deficiencies.
Affected individuals usually present at an older age with recurrent meningococcal infections.
The nitroblue tetrazolium reduction test is used to diagnose chronic granulomatous disease. Affected
individuals can present at a young age with recurrent infections; however, delayed umbilical cord separation,
omphalitis, and periodontal disease are not typically seen in this disorder.
Serum immunoglobulin levels may be used to diagnose humoral immune deficiencies (eg, X-linked
agammaglobulinemia or common variable immunodeficiency). These conditions typically present with
recurrent sinopulmonary infections with encapsulated organisms.
Suggested Reading(s)
Bonilla MA, Menell JS. Disorders of white blood cells. In: Lanzkowsky P, Lipton J, Fish JD. Lanzkowsky’s
Manual of Pediatric Hematology and Oncology. 6th ed. Elsevier Inc; 2016:209-238.
Goldman DL. Recurrent infections. In: McInerny TK, Adam HM, Campbell DE, DeWitt TG, Foy JM, Kamat
DM, eds. American Academy of Pediatrics Textbook of Pediatric Care. 2nd ed. American Academy of
Pediatrics; 2017:chap 187. Pediatric Care Online
Sivathanu S, Sampath S, Sridhar I. Case 1: recurrent omphalitis and nonhealing ulcers in a 7-month-old
girl. Pediatr Rev. 2016;37(11):491-493. doi:10.1542/pir.2016-0017
Content Domain
Hematology
Clinical findings Delayed umbilical cord separation Less frequent infections than LAD I Significant bleeding (including intracranial
Omphalitis Skin, lung, and periodontal infections hemorrhage)
Recurrent infections (skin, respiratory, bowel, Neutrophilia (very high leukocyte counts with Some severe infections
perirectal, periodontal, sepsis) infection) Delayed umbilical cord separation
Neutrophilia without pus formation (very high Short stature Omphalitis
leukocyte counts with infection) Intellectual disability Recurrent infections (skin, respiratory, bowel,
Bombay red blood cell phenotype perirectal, periodontal, sepsis)
Neutrophilia without pus formation (very high
leukocyte counts with infection)
Defect β2-integrin family is deficient or defective Guanosine 5′-diphospho-β-L-fucose transporter Defective activation of all β-integrins
Lack of neutrophil adherence to endothelium I defect Lack of integrin-dependent platelet aggregation
Diagnostic test Flow cytometry with lack of CD11b or CD18 Flow cytometry for lack of CD15 Abnormal platelet aggregation studies
Molecular genetic testing for specific variants
Treatment Prophylactic antibiotics (eg, trimethoprim- Prophylactic antibiotics (eg, trimethoprim- Hematopoietic stem cell transplant
sulfamethoxazole) sulfamethoxazole)
Antibiotic mouthwash Prompt treatment of infections
Prompt treatment of infections Consider a trial of fucose supplementation
Hematopoietic stem cell transplant considered
for severe cases
Courtesy of A. Appiah-Kubi
AAP PREP 2024 - Question 103/267 Infectious Diseases Question 1/30
A 5-week-old infant born at 39 weeks’ gestation is evaluated in the emergency department for parental
concern of tactile fever. The infant has been feeding normally and has had no vomiting, diarrhea, cough, or
congestion. There are school-aged siblings at home.
His vital signs are a rectal temperature of 38.6 °C, heart rate of 110 beats/min, respiratory rate of 36
breaths/min, and oxygen saturation of 98% in room air. The infant is alert, with moist mucous membranes
and normal findings.
Of the following, the BEST next step in the management of this infant is to
A. administer ibuprofen, 10 mg/kg orally, and recheck the temperature after 1 hour
C. obtain a urinalysis, blood culture, absolute neutrophil count, and procalcitonin level
D. obtain a urinalysis, blood culture, complete blood cell count, and cerebrospinal fluid analysis
Correct answer is C
PREP Pearl(s)
Escherichia coli is the most common cause of bacteremia in children between 29 and 60 days of age.
C-reactive protein and procalcitonin are better predictors of invasive bacterial infection risk in infants
aged 8 to 60 days than white blood cell and/or band counts.
Ibuprofen should not be administered to infants younger than 6 months. It should be used with
caution in patients with significant dehydration; prostaglandin inhibition can alter renal perfusion and
result in acute kidney injury.
Critique
The best next step in the management of the infant in the vignette is to obtain a urinalysis, blood culture,
absolute neutrophil count, and procalcitonin level. The management of fever in the young infant has been
studied for decades. Recommendations have changed based on epidemiologic factors, advances in testing
and treatment, and evolving research. The 2021 Clinical Practice Guideline: Evaluation and Management of
Well-Appearing Febrile Infants 8 to 60 Days Old (Pantell) divides these infants into 3 age categories: 8 to 21
days, 22 to 28 days, and 29 to 60 days. Recent studies show that Escherichia coli has emerged as the most
common organism to cause bacteremia in the 29- to 60-day age group, which includes the infant described in
the vignette. This finding significantly impacts the choice of testing recommended to identify infants at higher
risk for invasive bacterial infections (IBI) (eg, bacteremia, meningitis).
The Figure shows the algorithm outlined in the 2021 clinical guideline recommendations for the evaluation
and management of a 29- to 60-day old well-appearing infant with fever of 38.0 °C or higher. A urinalysis
remains an important test to obtain in this age group due to the significant risk of urinary tract infection. In
the setting of Escherichia coli emergence, for more accurate identification of infants at higher risk for IBI,
recent studies support the value of the inflammatory markers (IMs) C-reactive protein ([CRP], produced in the
liver), procalcitonin ([PCT], produced primarily by the thyroid), absolute neutrophil count (ANC), and fever.
Current recommendations for testing in this age group are to obtain a PCT level in conjunction with either a
CRP or ANC level. If PCT testing is not available, both a CRP and an ANC should be obtained, and a
temperature higher than 38.5 °C is considered abnormal. Procalcitonin is one of the most accurate IMs to
help identify infants at risk for IBI, but it can be normal in some cases. For this algorithm, abnormal IMs
include a temperature higher than 38.5 °C, CRP level greater than 2.0 mg/dL (20 mg/L), procalcitonin level
greater than 0.5 ng/mL, and an ANC greater than 4,000/µL (4.0 x 109/L) when used in conjunction with the
procalcitonin level or greater than 5,200/µL (5.2 x 109/L) when the procalcitonin level in unavailable. The ANC
level in the algorithm is presented as 2 distinct ANC values, dependent on the availability of a procalcitonin
level, representing the current state of evidence. The algorithm acknowledges that ANC is useful due to broad
availability and timely results. The ANC should be used in combination with other clinical and laboratory
predictors for risk stratification due to its high sensitivity but low specificity.
a
Key action statement references are shown in parentheses.
b
If available, procalcitonin (PCT) should be obtained along with an absolute neutrophil count (ANC). If PCT is
unavailable, ANC and C-reactive protein (CRP) should be obtained, and a temperature >38.5 °C is considered
abnormal. PCT is considered abnormal at >0.5 ng/mL; CRP is considered abnormal at >20 mg/L; ANC is
considered abnormal at >4,000/𝜇L when used in conjunction with PCT or >5,200/𝜇L when PCT is unavailable
Reprinted with permission from Pantell RH, Roberts KB, Adams WG, et al; Subcommittee on Febrile Infants.
Clinical practice guideline: evaluation and management of well-appearing febrile infants 8 to 60 days old.
Pediatrics. 2021;148(2):e2021052228.
Figure. 2021 Clinical guideline recommendations for the evaluation and management of a 29-60 day old well-
appearing infant with fever.
If any IM result is abnormal, a lumbar puncture should be considered. Cerebrospinal fluid (CSF) should be
sent for cell count, Gram stain, glucose, protein, and bacterial culture. Enterovirus polymerase chain reaction
(PCR) testing (if available) is recommended if CSF pleocytosis is present and during periods of increased local
enterovirus prevalence.
Although uncommon in this age group, testing for herpes simplex virus (HSV) should be considered when
there is a maternal history of genital HSV lesions and in infants with vesicles, seizures, hypothermia, mucous
membrane ulcers, CSF pleocytosis in the absence of a positive Gram stain result, leukopenia,
thrombocytopenia, or elevated alanine aminotransferase levels. Recommended HSV studies include the
following: surface swabs of mouth, nasopharynx, conjunctivae, and anus for HSV culture (if available) or PCR
assay; testing for alanine aminotransferase levels; a CSF PCR; and a blood PCR.
If CSF testing is indicated but unobtainable or uninterpretable, there are insufficient data to make a specific
treatment recommendation. Treatment options include the following:
Observe without treatment for a period of time and, depending on infant clinical condition, repeat
lumbar puncture and/or laboratory markers
Begin empirical antimicrobial agents and reassess in 24 hours on the basis of infant response and
results of blood culture
If CSF is bloody or antimicrobial agents have previously been started, analysis by multiplex PCR can
add additional information
Consult with a pediatric infectious disease specialist
Most 29- to 60-day-old infants with negative IM and urinalysis results may be observed at home. However,
hospital observation is an option for infants when there are barriers to follow-up. Infants may be managed at
home if the parent and clinician agree that all of the following are present:
Reliable phone and transportation resources
Parent willingness to observe and communicate changes in condition
Agreement to have the infant reevaluated within 24 hours
If CSF is positive for enterovirus, clinicians may withhold or discontinue antimicrobial agents and discharge at
24 hours, provided they meet other criteria for observation at home.
Infants in this age group do not require a lumbar puncture with cerebrospinal fluid analysis if they have
normal IMs; therefore, this procedure would not be included in the best next step in the management of this
infant.
The infant in the vignette does not have respiratory symptoms and has a normal pulmonary examination
with a normal oxygen saturation; therefore, testing for respiratory viruses is not indicated.
Fever in infants and children often causes significant parental anxiety due to potential associated dangers.
The pediatrician can play an important role by educating families about what a fever represents, why it may
be helpful, and reasonable goals for treatment. Encouraging parents and caregivers to focus on their child’s
comfort rather than treating a specific temperature can help alleviate fever phobia. It is important to review
appropriate dosing of antipyretic medications at each well-child visit to help prevent supratherapeutic dosing
and potential adverse effects.
The infant in the vignette is well-appearing, not fussy or irritable, and feeding well. If this infant were to
require antipyretics, the appropriate choice would be acetaminophen; data are insufficient to support the
safety and efficacy of ibuprofen in children younger than 6 months. It is important to discuss potential
adverse effects of ibuprofen (eg, gastritis, acute kidney injury) with parents and caregivers. Children should
take ibuprofen with food to decrease the risk of gastritis. Ibuprofen should be avoided in children with
dehydration. Prostaglandin synthesis is an important mechanism to maintain blood flow to the kidneys in
children with dehydration; therefore, prostaglandin inhibitors such as ibuprofen can interfere with this
system and lead to renal dysfunction. Ibuprofen should only be administered to children when good
hydration can be ensured.
Suggested Reading(s)
Milcent K, Faesch S, Gras-Le Guen C. Use of procalcitonin assays to predict serious bacterial infection in
young febrile infants. JAMA Pediatr. 2016(1):62-69. doi:10.1001/jamapediatrics.2015.3210.
Pantell RH, Roberts KB, Adams WG, et al; Subcommittee on Febrile Infants. Clinical practice guideline:
evaluation and management of well-appearing febrile infants 8 to 60 days old. Pediatrics.
2021;148(2):e2021052228. doi:10.1542/peds.2021-052228.
Sullivan JE, Farrar HC; Section on Pharmacology and Clinical Therapeutics, Committee on Drugs. Clinical
report: fever and antipyretic use in children. Pediatrics. 2011;127(3):580-587. doi:10.1542/peds.2010-
3852.
Content Domain
Infectious Diseases
A 6-year-old unimmunized girl is brought to the office after sustaining a fall at home, hitting her face on the
backyard coffee table. She did not lose consciousness and is acting appropriately. Her dentition is intact, and
there are no loose or broken teeth. She has a small gingival abrasion near her upper frenulum and a 1 cm
linear shallow laceration of her upper lip which crosses the vermillion border. She has no tenderness to
palpation or swelling of her nose or cheek.
Of the following, the MOST appropriate step in management of this girl’s condition is to
A. anesthetize and suture the upper lip laceration; administer tetanus vaccine
B. anesthetize and suture the upper lip laceration; administer tetanus vaccine and tetanus
immune globulin
C. gently irrigate the wounds and allow them to close by secondary intention
Correct answer is B
PREP Pearl(s)
When evaluating any wound, a review of the child’s tetanus immunization status is important.
An unimmunized child with any wound other than a clean, minor wound requires administration of
tetanus vaccination and tetanus immune globulin.
For lacerations involving the vermillion border of the lip, the first suture should be placed to exactly
align the edges of the vermillion border.
Critique
The girl in the vignette has a small, simple, linear laceration of the upper lip that crosses the vermillion
border. This laceration can be repaired in the office or emergency department by appropriately anesthetizing
the wound and placing sutures to close the laceration. When evaluating any wound, review of the child’s
tetanus immunization status is important. Because this girl is unimmunized, she also requires tetanus
vaccination and administration of tetanus immune globulin. Administration of the tetanus vaccine alone for
any wound that is not clean and minor is not sufficient to provide protection from disease in an
unimmunized individual. Table 1 shows recommendations for tetanus prophylaxis in routine wound care.
The primary goals of wound closure are to restore function, achieve hemostasis, and optimize cosmesis.
Placement of sutures is considered the standard method for wound closure. Table 2 lists the suggested
suture type based on the location of a wound, as well as duration of time to leave sutures in place. Extra care
must be taken when suturing a laceration that crosses the vermillion border. The first suture should be
placed at the vermillion border to establish accurate alignment; the remainder of the sutures placed should
ensure that all vermillion border edges are appropriately aligned. Allowing the wound to heal by secondary
intention would leave a large scar and would not achieve the best cosmetic outcome. The girl does not have
any evidence of dental or facial injury; the wound appears to be isolated to the soft tissue of the lip, therefore
imaging is not indicated. Imaging studies (eg, radiographs, computed tomography) are rarely indicated for
children with simple lacerations or wounds.
Prophylactic antibiotics should not be routinely prescribed for wounds and lacerations. A few special
circumstances (eg, bites from dogs, cats, or humans, or puncture wounds through the rubber sole of a shoe)
warrant prophylactic antibiotics. Table 3 details suggested antibiotic choices in these circumstances.
Suggested Reading(s)
American Academy of Pediatrics. Wound care and tetanus prophylaxis. In. Kimberlin DW, Barnett ED,
Lynfield R, Sawyer MH, eds. Red Book: 2021–2024 Report of the Committee on Infectious Diseases.
32nd ed. American Academy of Pediatrics; 2021. Red Book Online.
Black KD, Cico SJ, Caglar D. Wound management. Pediatr Rev. 2015;36(5):207-216. doi:10.1542/pir.36-5-
207
Lease JG. Office care of wounds. Pediatr Rev. 1992;13(7):257-261. doi:10.1542/pir.13-7-257
Parlin LS. Repair of lip lacerations. Pediatr Rev. 1997;18(3):101-102. doi:10.1542/pir.18-3-101
Schmitt B. Sutured wound care. Pediatric Care Advice. Pediatric Care Online. 2022. Accessed
September 1, 2022. Pediatric Care Online
Content Domain
Infectious Diseases
Table 1. Tetanus Prophylaxis in Routine Wound Management. Table 2. Suture Type, Appropriate Location, and Time to Removal.
History of Clean, Minor Wounds All Other Wounds* Location Suture Type Suture Size Duration of Sutures
Tetanus Toxoid
(Doses) Face Monofilament, fast-absorbing 5.0 or 6.0 4–5 days
(fast-absorbing gut), or
DTaP, Tdap, or Td TIG DTaP, Tdap, or Td TIG nonabsorbing (nylon,
polypropylene)
Fewer than 3 Yes No Yes Yes
or unknown Subcutaneous Monofilament absorbable 4.0 or 5.0 N/A
(deep) (plain or chromic catgut) or
3 or more No if <10 years since No No if <5 years since No
polyfilament absorbable
last tetanus- containing last tetanus- containing
(polyglactin)
vaccine dose. vaccine dose.
Trunk Nonabsorbing (nylon, 4.0 or 5.0 7–10 days
Yes if ≥10 years since No Yes if ≥5 years since No
polypropylene)
last tetanus- containing last tetanus- containing
vaccine dose. vaccine dose. Extremities Nonabsorbing (nylon, 4.0 or 5.0 10–14 days
polypropylene)
Abbreviations, DTaP, diphtheria and tetanus toxoids with pertussis; Tdap, tetanus toxoid,
reduced diphtheria toxoid, and acellular pertussis, adsorbed; Td, tetanus and diphtheria Reprinted with permission from Black KD, Cico SJ, Caglar D. Wound management. Pediatr
toxoids (adult type); TIG, tetanus immune globulin. Rev. 2015;36(5):212.
*
Such as, but not limited to, wounds contaminated with dirt, feces, soil, and saliva; puncture
wounds; avulsions; and wounds resulting from missiles, crushing, burns, and frostbite.
Note: DTaP is used for children <7 years of age. Tdap is preferred to Td for underimmunized
children 7 years of age or older who have not received Tdap previously. Table 3. Common Wound-related Antibiotics.
Antibiotic Dosing Common Use
Amoxicillin-clavulanate 90 mg/kg divided Bites, intraoral wounds,
twice a day grossly contaminated
wounds, wounds with
devitalized tissue
Ciprofloxacin 10–20 mg/kg Puncture wounds
divided twice a day through shoes
(Pseudomonas), hand
bites (cat, human),
penicillin-allergic
patients
Clindamycin 20–40 mg/kg Hand bites (cat,
divided every 6–8 human), penicillin-
hours allergic patients
Trimethoprim/ 8–10 mg/kg/day of Hand bites (cat,
sulfamethoxazole suspension trimethoprim divided human), penicillin-
(40/200 per 5 mL) every 12 hours allergic patients
AAP PREP 2024 - Question 105/267 Infectious Diseases Question 3/30
A 16-day-old female neonate born at 33 weeks’ gestation is brought to the emergency department by
ambulance after she had a generalized tonic-clonic seizure. She has had 2 days of poor feeding, fussiness,
and increased sleeping. In the emergency department, her temperature is 38.5 °C, heart rate is 160
beats/min, respiratory rate is 45 breaths/min, blood pressure is 50/30 mm Hg, and oxygen saturation is 98%
in room air. The neonate is somnolent. Her physical examination findings are otherwise unremarkable. The
neonate’s mother was noted to be positive for group B Streptococcus and received 2 doses of antibiotics
more than 2 hours before the infant was delivered vaginally. The pregnancy, delivery, and postpartum period
were otherwise uncomplicated.
D. urgent electroencephalography
Correct answer is B
PREP Pearl(s)
Group B Streptococcus and Escherichia coli are the most likely cause of meningitis in neonates
younger than 1 month.
Ampicillin should be paired with a third-generation cephalosporin (eg, cefotaxime), as opposed to
gentamicin, for empiric treatment of neonates with suspected meningitis born to women who received
prepartum antibiotic prophylaxis for group B Streptococcus infection.
A lumbar puncture should be performed on all neonates younger than 1 month with concerns for
sepsis/meningitis, preferably prior to antibiotic infusion.
Critique
The neonate in the vignette, with a fever, poor feeding, somnolence, and a generalized tonic-clonic seizure,
most likely has meningitis. Therefore, the best next step in her management is to perform a lumbar puncture
for cerebrospinal fluid cell count and culture.
The epidemiology of meningitis in neonates and children in the United States has been greatly impacted over
the past decades by advances in vaccines and management of maternal group B Streptococcus (GBS)
colonization. Historically, childhood meningitis was primarily due to infection with Haemophilus influenzae
type B, Streptococcus pneumoniae, Neisseria meningitidis, GBS, Escherichia coli, and Listeria monocytogenes.
Routine vaccination of infants with conjugate H influenzae type B vaccine starting in 1990 and the
pneumococcal vaccine (PCV7 in 2000, PCV13 in 2012) have resulted in a dramatic decrease in cases of
meningitis in children.
Even with routine administration of the PCV13 vaccine, S pneumoniae remains the most common cause of
bacterial meningitis in children over the age of 1 month. Group B Streptococcus and E coli remain the most
common cause of neonatal meningitis.
Approximately one-third of women are colonized with Streptococcus agalactiae (ie, GBS). Since 1996, there
has been routine screening of pregnant women for colonization with this organism late in pregnancy and
guidelines for treatment of GBS-positive women with antibiotics within 4 hours before delivery. This practice
has resulted in a decreased rate of early-onset GBS infection (in infants <7 days of age) from 1.7 cases per
1,000 live births (1993) to 0.22 cases per 1,000 live births (2016). The incidence of late-onset GBS disease,
(between the ages of 7 and 90 days) has not been impacted in the same way by these guidelines. Late-onset
disease is still seen in approximately 0.28/1,000 infants in the United States. The incidence is highest in
premature infants and infants born to women with GBS colonization.
The routine administration of antibiotic prophylaxis to women with GBS colonization has been identified as a
likely contributor to the increased prevalence of antibiotic-resistant E coli, primarily affecting preterm and
low-birth-weight infants. It is therefore recommended that infants suspected of having meningitis that were
born to women that received GBS prophylaxis should receive empiric treatment with ampicillin paired with a
third-generation cephalosporin (eg, cefotaxime), as opposed to gentamicin.
Unless there is a specific contraindication, such as cardiopulmonary instability, a lumbar puncture should be
performed on all neonates younger than 1 month with suspected sepsis/meningitis. Ideally, this should be
performed prior to the infusion of antibiotics. However, in cases where an infant is not clinically stable or in
the event of a traumatic lumbar puncture, antibiotics should not be delayed.
Neither head computed tomography nor electroencephalography is recommended as the initial intervention
for an infant that presents with severe illness suggestive of meningitis.
Suggested Reading(s)
Kim KS. Neonatal bacterial meningitis. NeoReviews. 2015;16(9):e535-e543. doi:10.1542/neo.16-9-e535
Randis TM, Baker JA, Ratner AJ. Group B streptococcal infections. Pediatr Rev. 2017; 38(6):254-262.
doi:10.1542/pir.2016-0127
Swanson D. Meningitis. Pediatr Rev. 2015;36(12):514-526. doi:10.1542/pir.36-12-514
Weinberg GA, Buchanan AM. Meningitis. In: McInerny TK, Adam HM, Campbell DE, DeWitt TG, Foy JM,
Kamat DM, eds. American Academy of Pediatrics Textbook of Pediatric Care. American Academy of
Pediatrics; 2021:chap 289. Accessed September 1, 2023. Pediatric Care Online
Content Domain
Infectious Diseases
The correct answer is: lumbar puncture for cerebrospinal fluid cell count and culture
View Peer Results
AAP PREP 2024 - Question 106/267 Infectious Diseases Question 4/30
A 10-year-old, previously healthy girl is seen in the emergency department for sudden-onset blindness in her
left eye. Urgent ophthalmologic consultation is obtained. Her fundoscopic examination (Figure) is suggestive
of toxoplasmosis.
Courtesy of S. Gauthreaux
Of the following, the MOST likely finding in the girl’s medical or personal history is
D. she recently traveled and sampled unfamiliar foods, including soft cheeses
Correct answer is A
PREP Pearl(s)
Toxoplasmosis infection can reactivate in older children, particularly in the eye.
Toxoplasmosis in healthy hosts can present as an undifferentiated febrile illness, often with
lymphadenopathy that self-resolves.
When treatment for toxoplasmosis is indicated, a prolonged multidrug treatment course is usually
required.
Critique
The girl in the vignette, with sudden-onset unilateral blindness, has reactivation of a toxoplasmosis infection.
Ocular toxoplasmosis can occur after either a congenital or childhood infection (often presenting as an
undefined febrile illness with lymphadenopathy). Toxoplasmosis can be transmitted through eating raw or
undercooked meat (the tissue cysts can infect most mammals) or through contact with the feces of felines
(toxoplasmosis’ definitive hosts). The organism spreads to muscle and neural tissue, typically causing a mild
or subclinical, self-limited illness in a healthy host.
Toxoplasmosis infection presents the greatest risk to the developing fetus; intrauterine infection can cause
stillbirth, hepatosplenomegaly, deafness, blindness, brain calcifications, and hydrocephalus. The congenital
presentation can be mild and may not be recognized. When toxoplasmosis is diagnosed in a pregnant
woman or a newborn, treatment is recommended. Affected children and adults with severe
immunosuppression (eg, AIDS) can develop serious illness with brain and eye lesions, requiring prolonged
multidrug antibiotic treatment.
Treatment of ocular toxoplasmosis is typically a prolonged (1 year or more) course of dual antibiotic therapy
(eg, pyrimethamine and sulfadiazine) together with a course of steroids.
Eating soft cheeses is a risk factor for brucellosis infection, and scratches from cats (particularly kittens) are a
risk factor for Bartonella henselae infection (cat-scratch fever). Neither is associated with toxoplasmosis.
Strabismus is not a characteristic finding of toxoplasmosis and strabismus surgery is not a risk factor for
retinal infection.
Suggested Reading(s)
Abdel-Haq N, Chearskul P, Rafee Y, Rafee BI. Parasitic infections. In: McInerny TK, Adam HM, Campbell
DE, DeWitt TG, Foy JM, Kamat DM, eds. American Academy of Pediatrics Textbook of Pediatric Care.
American Academy of Pediatrics; 2021:chap 308. Accessed September 1, 2023. Pediatric Care Online
Dubey JP, Murata FHA, Cerqueira-Cézar CK, Kwok OCH, Villena I. Congenital toxoplasmosis in humans:
an update of worldwide rate of congenital infections. Parasitology. 2021;148(12):1406-1416.
doi:10.1017/S0031182021001013
Khan K, Khan W. Congenital toxoplasmosis: An overview of the neurological and ocular manifestations.
Parasitol Int. 2018;67(6):715-721. doi:10.1016/j.parint.2018.07.004
Penner J, Hernstadt H, Burns JE, Randell P, Lyall H. Stop, think SCORTCH: rethinking the traditional
'TORCH' screen in an era of re-emerging syphilis. Arch Dis Child. 2021;106(2):117-124.
doi:10.1136/archdischild-2020-318841
Peyron F, L'ollivier C, Mandelbrot L, Wallon M, Piarroux R, et al. Maternal and congenital toxoplasmosis:
diagnosis and treatment recommendations of a French multidisciplinary working group. Pathogens.
2019;8(1):24. doi:10.3390/pathogens8010024
Content Domain
Infectious Diseases
The correct answer is: her mother had an undefined febrile illness late in pregnancy
View Peer Results
AAP PREP 2024 - Question 107/267 Infectious Diseases Question 5/30
A 3-year-old girl is brought to the office for evaluation of nasal congestion, nasal discharge, and cough of 12
days’ duration. She is having trouble sleeping owing to nasal stuffiness and worsening cough. She is
tolerating oral fluids well, but her solid food intake has decreased. The girl’s mother is worried that the
symptoms are not improving. She was previously healthy and her immunizations are up to date. On physical
examination, the girl is afebrile; she appears nontoxic and in no distress. Her nasal mucosa is erythematous
with enlarged turbinates and clear discharge. The remainder of her physical examination findings are
normal.
b. allergic rhinitis
Correct answer is A
PREP Pearl(s)
Acute bacterial sinusitis should be considered in young children with upper respiratory infection
symptoms that are prolonged (>10 days) and persistent (without improvement).
Conditions predisposing a person to sinusitis include nasal anatomical abnormalities and
immunodeficiencies.
Routine imaging is not indicated for the diagnosis of sinusitis and should be obtained only when
complications are suspected.
Critique
The girl in the vignette has history and physical examination findings consistent with a diagnosis of acute
bacterial sinusitis (12 days of worsening upper respiratory infection [URI] symptoms). Most URIs last for 7 to
10 days. Sinusitis should be considered when URI symptoms last more than 10 days and persist without
improvement.
Although allergic rhinitis and nasal foreign bodies can present with similar symptoms and predispose to
sinusitis, the girl in the vignette does not have specific history or physical examination findings suggestive of
these conditions. Allergic rhinitis triggered by environmental allergens usually has a seasonal pattern, with
increased symptom occurrence during the spring and fall. Indoor allergens and irritants (eg, tobacco
exposure, dust mites, mold) may trigger intermittent symptoms throughout the year. Physical examination
findings associated with allergic rhinitis include dark areas under the eyes (allergic shiners); creases at the
base of the nose from rubbing (allergic salute); pale, boggy nasal turbinates; and a cobblestone appearance
of the posterior pharynx.
Nasal foreign bodies occur most often in children aged 3 to 5 years. There is commonly no history of foreign
body insertion, as it is often not observed. The foreign body may be visualized on physical examination if
present anteriorly. Unilateral purulent and/or foul-smelling discharge may occur.
The American Academy of Pediatrics’ clinical practice guideline recommends considering a diagnosis of acute
bacterial sinusitis in children with URI and one of the criteria listed in the Table.
The symptoms of sinusitis vary by age owing to the progression of sinus development. The ethmoid and
maxillary sinuses, although small, are present at birth. The sphenoid and frontal sinuses develop later
(around age 5 and 10-12 years, respectively). Younger children with sinusitis usually have prolonged and
persistent URI symptoms. Older children typically experience more severe or worsening URI symptoms. The
classic presentation of headache, facial swelling, and sinus tenderness is more often seen in older children.
Sinusitis is a clinical diagnosis, and imaging is not indicated unless orbital or intracranial extension is
suspected. Complications of acute sinusitis include periorbital or orbital cellulitis, subperiosteal abscess,
meningitis, brain abscess, subdural or epidural empyema, and venous thrombosis.
A viral URI progresses to acute bacterial sinusitis in 5% to 10% of cases. The thickening of secretions and
ciliary dysfunction from a URI blocks the sinus opening, leading to oxygen depletion. Negative pressure
created in the sinus cavity results in the introduction of nasopharyngeal bacteria. Risk factors for the
development of sinusitis include anatomical abnormalities (eg, deviated nasal septum, cleft palate, foreign
body, nasal polyps), allergic rhinitis (resulting in the breakdown of defense mechanisms), immotile cilia
syndrome, cystic fibrosis, and immunodeficiencies.
The most common bacterial pathogens associated with sinusitis are Streptococcus pneumoniae, nontypeable
Haemophilus influenzae, and Moraxella catarrhalis. There is strong evidence for antibiotic treatment when
severe or worsening symptoms are present. Children with persistent symptoms may be treated with
antibiotics or observed for 3 additional days with initiation of therapy if there is no symptom improvement.
The recommended first-line treatment is amoxicillin or amoxicillin-clavulanate, with dosing based on local
resistance patterns.
Recurrent sinusitis is defined as more than 2 to 3 discrete episodes per year. Chronic sinusitis is defined as
symptoms lasting for more than 3 months. Conditions predisposing a person to recurrent and chronic
sinusitis include allergic rhinitis, anatomical abnormalities, tobacco smoke exposure, immotile cilia syndrome,
cystic fibrosis, and immunodeficiencies. In addition to antibiotic treatment, referral to an
otorhinolaryngologist and management of the predisposing condition is warranted for recurrent or chronic
sinusitis.
Suggested Reading(s)
DeMuri G, Wald ER. Acute bacterial sinusitis in children. Pediatr Rev. 2013;34(10):429-437.
doi:10.1542/pir.34-10-429
Tan TQ. Sinusitis. In: McInerny TK, Adam HM, Campbell DE, DeWitt TG, Foy JM, Kamat DM, eds.
American Academy of Pediatrics Textbook of Pediatric Care. American Academy of Pediatrics;
2021:chap 331. Accessed September 1, 2023. Pediatric Care Online.
Wald ER, Applegate KE, Bordley C, et al; American Academy of Pediatrics. Clinical practice guideline for
the diagnosis and management of acute bacterial sinusitis in children aged 1 to 18 years. Pediatrics.
2013;132(1):e262-e280. doi:10.1542/peds.2013-1071
Content Domain
Infectious Diseases
The correct answer is: acute bacterial sinusitis Table. Clinical Presentation and Criteria for the Diagnosis of Acute Bacterial
Sinusitis.
View Peer Results Persistent Symptoms
• Nasal discharge/congestion and/or cough for ≥10 days without improvement
Severe Symptoms
• Temperature ≥38.5°C with purulent rhinorrhea for at least 3 days
Worsening Symptoms
• Worsening of nasal congestion or rhinorrhea, cough, and fever after a 3- to 4-day
period of improved symptoms
AAP PREP 2024 - Question 108/267 Infectious Diseases Question 6/30
A 17-year-old girl is seen in the emergency department for fever, headache, and pain in her ankles, wrists,
and fingers of 1 day’s duration. She has a temperature of 40 °C, a heart rate of 110 beats/min, a respiratory
rate of 25 breaths/min, a blood pressure of 90/65 mm Hg, and an oxygen saturation of 100% in room air. She
appears toxic. There is a nonblanchable rash over both hands and lower legs (Figure) and moderate swelling
of both ankles and her right hand metacarpal joints.
The girl is fully immunized, including 2 doses of meningococcal B vaccination. She does not recall a recent tick
bite. She had unprotected sexual intercourse with 2 male partners 1 month ago.
Neutrophils 75%
Lymphocytes 15%
Monocytes 5%
Eosinophils 5%
Cerebrospinal fluid
Protein 35 mg/dL
Of the following, the BEST test to confirm this adolescent’s diagnosis is for
Correct answer is A
PREP Pearl(s)
Neisseria gonorrhoeae is the second most common cause of sexually transmitted infection in the
United States. The clinical spectrum includes asymptomatic (most common), local mucosal infection
(urethritis, cervicitis, pharyngitis), disseminated (pelvic inflammatory disease, disseminated gonococcal
infection), and neonatal (gonococcal ophthalmia neonatorum) infection.
Disseminated gonococcal infection is a rare complication (< 3%) of untreated Neisseria gonorrheae
infection; it is characterized by fever, arthritis, tenosynovitis, and extremity rash.
Nucleic acid amplification testing is the preferred test for Neisseria gonorrhoeae; treatment is with
parenteral ceftriaxone.
Critique
The sexually active girl in the vignette has fever, extremity rash (pustules, papules), and polyarthritis, which
are suggestive of disseminated gonococcal infection (DGI), also known as arthritis–dermatitis syndrome.
Features of this syndrome include a triad of tenosynovitis, dermatitis, and arthritis. A nucleic acid
amplification test (NAAT) performed on a urine or mucosal site (eg, urethra, cervix, pharynx, rectum) sample
yields Neisseria gonorrhoeae DNA in 80% of cases of DGI.
Meningococcemia should be considered in an adolescent patient with fever and a petechial rash. Neisseria
meningitidis DNA in the cerebrospinal fluid (CSF) would confirm a diagnosis of meningococcal meningitis,
which may be associated with meningococcemia. However, the findings of the CSF analysis for the girl in the
vignette is not suggestive of meningitis. Furthermore, she has received meningococcal ACYW-135 and B
vaccination, lowering her risk of N meningitidis infection.
Rickettsia rickettsiae antibodies in the serum would confirm a diagnosis of Rocky Mountain spotted fever
(RMSF). Rocky Mountain spotted fever presents with fever, headache, and myalgias. A rash is present in 80%
to 95% of cases on days 3 to 5 of the illness. The hallmark rash of RMSF consists of blanching erythematous
macules that become petechial over time. The rash appears first on the wrists, spreads to the trunk, and then
develops on the palms and soles. The rash location and absence of thrombocytopenia, hyponatremia, and
elevated level of liver enzymes makes RMSF an unlikely diagnosis for the girl in the vignette.
Treponema pallidum antibodies in the serum would confirm a diagnosis of secondary syphilis. Secondary
syphilis presents with constitutional symptoms (fever, headache, myalgias), lymph node enlargement
(cervical, axillary, inguinal, femoral), and rash (classically, a diffuse symmetric maculopapular rash over the
trunk and extremities, including palms and soles). Polyarthritis is not a finding of syphilis, making it an
unlikely diagnosis for the adolescent in the vignette.
Gonococcal infection, or gonorrhea, is caused by N gonorrhoeae, a gram-negative diplococcus. It is the
second most common sexually transmitted infection in the United States (Chlamydia trachomatis is the most
common). N gonorrhoeae primarily infects the columnar or transitional epithelial cells in the human genital
tract. The highest incidence is in females aged 14 to 19 years and males aged 20 to 24 years. Risk factors
include multiple sexual partners, unprotected sex, and cervical ectropion (in females).
Neisseria gonorrhoeae infection results in a spectrum of clinical manifestations, depending on local versus
systemic involvement and age.
Asymptomatic (87% of cases in females, 95% of cases in males)
Local mucosal (cervicitis, urethritis): The most common presentation of symptomatic N gonorrhoeae
infection is acute cervicitis (females) or urethritis (males). Symptomatic infection is more common in
males than females. Symptomatic females may present with vaginal discharge, dysuria, abdominal
pain, or intermenstrual bleeding. The cervix may appear normal or erythematous, edematous, and
friable, with or without mucopurulent discharge. Urethritis in males presents as dysuria with mucoid or
mucopurulent discharge. Pharyngitis is seen in 3% to 7% of infected heterosexual males and in 10% to
20% of infected heterosexual females and men who have sex with men (MSM). Vaginitis is the most
common clinical manifestation in prepubertal girls. N gonorrhoeae should be considered in
prepubertal girls with vaginal discharge, even when suspicion of sexual abuse is low.
Disseminated disease (pelvic inflammatory disease, DGI)
Pelvic inflammatory disease (PID) is the most common complication of N gonorrhoeae
(10% to 20%). Upper genital tract inflammation can result in endometritis, salpingitis,
oophoritis, perihepatitis (Fitz-Hugh-Curtis disease), or a tubo-ovarian abscess. Although N
gonorrhoeae may be the initial sexually transmitted infection, PID itself is a polymicrobial
(aerobes, anaerobes) infection.
Disseminated gonococcal infection is an acute systemic infection caused by N
gonorrhoeae. It is rare, occurring in <3% of infected individuals. The classic findings are
arthritis, tenosynovitis, and dermatitis, which result from intermittent bloodstream
infection. A pustular or vesiculopustular rash typically occurs on the extremities. A tender
necrotic pustule on a distal extremity (mainly the hands) is suggestive of DGI. Migratory
polyarthritis and tenosynovitis affecting small joints are early signs. Blood culture is
positive in less than one-half of cases of DGI; skin culture is positive in 10% to 15% of
cases. This low positivity rate is due to the fastidious nature of N gonorrhoeae, which
makes it difficult to isolate. Isolation of N gonorrhoeae requires transport in a humid and
carbon dioxide–rich environment and culture on chocolate blood agar. N gonorrhoeae
can be isolated from mucosal sites in about 80% of DGI cases.
Neonatal (gonococcal ophthalmia neonatorum): Neonatal infection is acquired from exposure to an infected
birth canal. Ophthalmia neonatorum caused by N gonorrhoeae manifests 2 to 5 days after birth; it is
characterized by marked bilateral eyelid edema, chemosis, and copious eye purulent discharge.
The diagnosis of gonococcal infection is confirmed by detection of the organism in a sample of urine or from
a mucosal site (eg, urethra, vagina, cervix, rectum, pharynx). Nucleic acid amplification testing is the ideal
method. Alternative tests include the following:
Gram stain of discharge revealing gram-negative diplococci, a test that is technician dependent and has low
sensitivity; a negative Gram stain finding does not rule out N gonorrhoeae infection.
Growth in culture, in which specimens from sterile sites (eg, blood, CSF, and synovial fluid) are grown on
nonselective chocolate agar; specimens from nonsterile sites (eg, pharynx, urethra, vagina, cervix) are plated
on a selective agar to inhibit growth of nonpathogenic Neisseria species; culture growth allows for antibiotic
susceptibility testing, which is not possible with NAAT.
Suggested Reading(s)
American Academy of Pediatrics. Gonococcal infections. In: Kimberlin DW, Barnett ED, Lynfield R,
Sawyer MH, eds. Red Book: 2021–2024 Report of the Committee on Infectious Diseases. 32nd ed.
American Academy of Pediatrics; 2021. Accessed September 1, 2023. Red Book Online
Hsu KK, Rice PA, Lieberman JM. Neisseria gonorrhoeae. In: Long SS, Pickering LK, Prober CG, eds.
Principles and Practice of Pediatric Infectious Diseases. 4th ed. Elsevier; 2012:741-748.
Joffe A. Sexually transmitted infections. In: McInerny TK, Adam HM, Campbell DE, DeWitt TG, Foy JM,
Kamat DM, eds. American Academy of Pediatrics Textbook of Pediatric Care. American Academy of
Pediatrics; 2023. Accessed September 1, 2023. Pediatric Care Online
Content Domain
Infectious Diseases
An 8-year-old girl with a history of well-controlled, moderate, persistent asthma is admitted to the pediatric
ward for an asthma exacerbation. She is fully immunized, including the influenza and COVID-19 vaccines. She
began to experience fever, runny nose, and cough 4 days ago. She was brought to the emergency
department this morning for worsening shortness of breath. In the emergency department, the girl’s
temperature was 39°C and her oxygen saturation was 87% in room air. Lung examination revealed diffuse
wheezing without focal findings. Oxygen supplementation by nasal cannula, oral prednisolone, and albuterol
by inhalation were administered. She was admitted to the hospital for further treatment. The results of rapid
testing of nasopharyngeal secretions were positive for influenza and negative for COVID-19.
Of the following, the BEST next step in this child’s management is to administer
A. ampicillin
B. nirmatrelvir-ritonavir
C. no additional medication
D. oseltamivir
Correct answer is D
PREP Pearl(s)
All children requiring hospitalization for influenza virus infection should be treated with antiviral
medication (eg, oseltamivir) regardless of the interval from symptom onset.
Children with influenza virus not requiring hospitalization should be offered antiviral medication if they
are within 2 days of symptom onset or at any interval if they have high-risk household contacts.
Critique
The best next step in management of the child in the vignette is to administer oseltamivir. Specific antiviral
therapy (eg, neuraminidase inhibitors) is underused in the pediatric setting. Although most effective when
given early in the course of influenza virus infection (preferably within 2 days of symptom onset), a
neuraminidase inhibitor is indicated for any patient requiring hospitalization. It is also indicated for any
infected individual with a high-risk household contact (eg, family members <6 months of age or >65 years of
age).
Three neuraminidase inhibitors have been approved for use in children: oseltamivir (capsule or a
suspension), peramivir (single intravenous dose), and zanamivir (dry-powdered inhaler). Zanamivir is not
recommended for children younger than 7 years or those with asthma (due to the risk of worsening
bronchospasm). Ampicillin is not indicated for this child with no evidence of pneumonia or other bacterial
infection. Nirmatrelvir-ritonavir is an oral antiviral combination therapy that is only indicated for outpatient
use in nonhypoxic patients older than 12 years with evidence of COVID-19. Giving no specific antiviral therapy
would not be the best management for this child who required hospitalization for treatment of an influenza
virus infection.
Suggested Reading(s)
American Academy of Pediatrics. Influenza. In: Kimberlin DW, Barnett ED, Lynfield R, Sawyer MH, eds.
Red Book: 2021-2024 Report of the Committee on Infectious Diseases. 32nd ed. American Academy of
Pediatrics; 2021. Red Book Online
Campbell AP, Tokars JI, Reynolds S, et al. Influenza antiviral treatment and length of stay. Pediatrics.
2021;148(4):e2021050417. doi:10.1542/peds.2021-050417
Mehta K, Morris SK, Bettinger JA, et al. Antiviral use in Canadian children hospitalized for influenza.
Pediatrics. 2021;148(4):e2020049672. doi:10.1542/peds.2020-049672
Moodley A, Bradley JS, Kimberlin DW. Antiviral treatment of childhood influenza: an update. Curr Opin
Pediatr. 2018;30(3):438-447. doi:10.1097/MOP.0000000000000618
Content Domain
Infectious Diseases
A 6-month-old girl is brought to the emergency department for 2 days of fever and an expanding rash over
her thigh. The rash first appeared as an erythematous patch, which developed a central area of black
discoloration within 12 hours. She was seen at an urgent care center yesterday and was started on a course
of oral clindamycin.
On physical examination, the girl is irritable. She has a temperature of 40 °C, a heart rate of 110 beats/min, a
respiratory rate of 30 breaths/min, and an oxygen saturation of 100% in room air.
There is a large, annular skin lesion with a purple to black center surrounded by an erythematous halo on
her left thigh (Figure). The lesion is indurated and tender. The remainder of her physical examination findings
are unremarkable.
Courtesy of A. Noor
Neutrophils 2%
Lymphocytes 68%
Monocytes 25%
Eosinophils 5%
Of the following, the MOST appropriate antibiotic choice for this girl’s condition is
A. cefepime
B. ceftriaxone
C. tobramycin
D. trimethoprim-sulfamethoxazole
Correct answer is A
PREP Pearl(s)
Ecthyma gangrenosum, a skin lesion associated with Pseudomonas sepsis, begins as a painless macule
and rapidly evolves into a gangrenous ulcer with violaceous borders.
First-line anti-Pseudomonas antibiotics include ceftazidime, cefepime, piperacillin-tazobactam,
carbapenems, aztreonam, and quinolones. Aminoglycosides are used as an adjunct but never as
monotherapy.
Critique
The girl in the vignette has signs of sepsis (fever, tachycardia, irritability), neutropenia, and an ecthyma
gangrenosum lesion on her left thigh, a skin lesion commonly associated with Pseudomonas infection. Of the
response choices, both cefepime and tobramycin are effective against Pseudomonas. However, in light of the
girl’s elevated creatinine level, tobramycin is contraindicated; aminoglycosides are commonly used as an
adjunct but never as monotherapy. Therefore, cefepime is the most appropriate antibiotic choice for this
girl’s condition.
The airway of a child with cystic fibrosis is highly susceptible to Pseudomonas infection. Chronic
Pseudomonas infection of the airway is associated with pulmonary function decline and decreased survival.
Acute pulmonary exacerbation and endobronchial disease are common. The Table outlines the types of
Pseudomonas infections in various hosts.
Early recognition of Pseudomonas infection and prompt initiation of antibiotics are essential. First-line
antipseudomonal agents include ceftazidime, cefepime, piperacillin-tazobactam, carbapenems, aztreonam,
and quinolones. Carbapenem-resistant Pseudomonas infections are treated with cephalosporins or
carbapenems combined with a beta-lactamase inhibitor (eg, ceftazidime-avibactam, ceftolozane-tazobactam,
or imipenem/cilastatin-relebactam).
The physician can consider using combination empiric antibiotics in children with serious infections, such as
sepsis, while awaiting antibiotic sensitivity data. If ecthyma gangrenosum with expanding necrosis occurs
despite antibiotics, surgical débridement is necessary. Children may develop resistance while receiving
antibiotics.
Suggested Reading(s)
American Academy of Pediatrics. Pseudomonas aeruginosa infections. In: Kimberlin DW, Barnett ED,
Lynfield R, Sawyer MH, eds. Red Book: 2021–2024 Report of the Committee on Infectious Diseases.
32nd ed. American Academy of Pediatrics; 2021. Accessed September 1, 2023. Red Book Online
Burns JL. Pseudomonas species and related organisms. In: Long SS, Pickering LK, Prober CG. Principles
and Practice of Pediatric Infectious Diseases. 4th ed. Elsevier; 2012:841.
Fisher RG, Chen C, Williams JV. Bacterial skin infections. In: McInerny TK, Adam HM, Campbell DE,
DeWitt TG, Foy JM, Kamat DM, eds. American Academy of Pediatrics Textbook of Pediatric Care.
American Academy of Pediatrics; 2023. Accessed September 1, 2023. Pediatric Care Online
Content Domain
Infectious Diseases
Courtesy of A. Noor
AAP PREP 2024 - Question 111/267 Infectious Diseases Question 9/30
A 7-year-old child is undergoing evaluation in the emergency department for a 5-day history of fever and
abdominal pain. He is stooling 2 to 3 times per day, without mucus or blood, but is able to tolerate oral fluids.
His family returned from Pakistan 1 week ago. His medical history is notable only for anaphylaxis to cefdinir
at age 4 years. The boy appears tired. He has a temperature of 39 °C, a blood pressure of 90/60 mm Hg, a
heart rate of 90 beats/min, and a respiratory rate of 25 breaths/min. There is lower abdominal tenderness.
The remainder of his physical examination findings are unremarkable.
Neutrophils 79%
Lymphocytes 5%
Eosinophils 5%
Blood culture (preliminary Gram-negative rods: lactose negative, oxidase negative, hydrogen sulfide
report) producing
Of the following, the BEST empiric antibiotic treatment for this child is
A. azithromycin
B. ciprofloxacin
C. clindamycin
D. vancomycin
Correct answer is A
PREP Pearl(s)
Fever and abdominal pain in children occurring within 2 weeks after a trip to South Asia, Southeast
Asia, or sub-Saharan Africa should raise suspicion of enteric fever (typhoid) caused by Salmonella
typhi.
Empiric antibiotic options for enteric fever (typhoid) presenting within 2 weeks after a trip to Pakistan
are azithromycin (for uncomplicated illness) or a carbapenem (for complicated illness).
Empiric antibiotic options for enteric fever (typhoid) in a child returning from a region outside Pakistan
are azithromycin (for uncomplicated illness) and ceftriaxone (for complicated illness).
Critique
The most likely diagnosis for the child described in the vignette is enteric fever (typhoid) caused by a
Salmonella infection. His presentation with fever, abdominal pain, and a blood culture growing gram-negative
rods after a trip to Pakistan is consistent with this diagnosis. The incidence of enteric fever is highest in South
Asia (India, Pakistan, and Bangladesh), followed by southeast Asia and sub-Saharan Africa.
Of the response choices, azithromycin is the best empiric antibiotic treatment for this child, who has a history
of a type I hypersensitivity reaction to an oral third-generation cephalosporin. Empiric treatment of suspected
enteric fever is appropriate for a child with a history of travel to an endemic region within the previous 2
weeks. The recommended antibiotic is dependent on regional antibiotic-resistant patterns (and the child’s
allergy history). Empiric antibiotic options for a child with suspected enteric fever after returning from a
region outside Pakistan are azithromycin (for an uncomplicated illness) and ceftriaxone (for a complicated
illness). A complicated infection is characterized by systemic toxicity, depressed consciousness, prolonged
fever, organ system dysfunction, or any other indication for hospitalization. Ceftriaxone-resistant Salmonella
strains have been increasingly identified in Pakistan. These drug-resistant strains have also shown resistance
to other common first-line antibiotics (eg, ampicillin, trimethoprim sulfamethoxazole, and ciprofloxacin), but
at this time they remain susceptible to azithromycin (for uncomplicated illness) and carbapenem (for
complicated illness). Vancomycin and clindamycin do not have activity against Salmonella (a gram-negative
organism).
Salmonella is a non–lactose-fermenting gram-negative rod that primarily infects small intestinal epithelial
cells. Salmonella infections are broadly divided into typhoidal and nontyphoidal forms. The vast majority of
Salmonella cases in the United States are nontyphoidal.
Salmonella typhi and Salmonella paratyphii (A, B) cause enteric fever (typhoid), a severe illness with
protracted bacteremia. Most enteric fever infections in the United States are acquired during travel to
endemic regions outside the United States. Humans are the only reservoirs, and transmission is fecal-oral.
The incubation period is 7 to 14 days. Common clinical manifestations include fever, malaise, myalgias,
headache, and abdominal pain. Diarrhea (common in children) or constipation may be early symptoms. Rose
spots (a blanching erythematous rash on the trunk) are less common (1.5%). Complications (eg, typhoid
intestinal perforation) may occur (rarely) in untreated cases in the third week of illness.
Nontyphoidal Salmonella serotypes are among the most common causes of acute gastroenteritis. There are
more than 2,600 nontyphoidal Salmonella serotypes, 5 of which are responsible for 45% of Salmonella
infections in the United States: enteritidis, Newport, Typhimurium, Javiana, and Salmonella enterica serotype
4,5,12:i:–. The principal reservoirs of nontyphoidal Salmonella are birds, mammals, reptiles, and amphibians.
Transmission is through contaminated food (vegetables, produce, poultry, eggs, dairy products, and beef).
The most common transmission sources in the United States are chicken and eggs. The highest incidence is
in children younger than age 5 years.
The primary clinical presentation of nontyphoidal Salmonella is acute gastroenteritis with fever, frequent
diarrhea, and abdominal cramping. The illness is typically self-limiting in otherwise healthy,
immunocompetent children and does not require antibiotic treatment. Treatment with antibiotics does not
shorten the illness duration and, in fact, prolongs shedding. Less common presentations of nontyphoidal
Salmonella infection (eg, bacteremia, severe disease, and focal infections) are seen in infants,
immunocompromised children, and children infected with virulent serotypes (eg, Dublin, Choleraesuis,
Panama, or Poona). Children with sickle cell disease are at increased risk of experiencing Salmonella
osteomyelitis owing to impaired splenic function and bone infarctions. When treatment is indicated (infants
who are younger than 3 months, are immunocompromised, have sickle cell disease, or have bacteremia),
ceftriaxone is the best empiric antibiotic choice for nontyphoidal Salmonella.
Blood culture is the primary method of diagnosing enteric fever. Nontyphoidal acute gastroenteritis is
diagnosed with stool culture or multiplex polymerase chain reaction platforms. Nontyphoidal extraintestinal
infections are diagnosed by means of a culture of the infected site (eg, blood, cerebral spinal fluid, synovial
fluid).
Children with typhoidal Salmonella should be excluded from school and child care during active infection.
Three negative stool culture results at least 48 hours apart are required for return to school/child care, given
the easy transmission. Children with nontyphoidal Salmonella enteritis can return to school or child care
facilities when stools are contained within the diaper, toilet-trained children stay continent, and stool
frequency is no more than 2 stools per day above the usual number.
Suggested Reading(s)
American Academy of Pediatrics. Salmonella infections. In: Kimberlin DW, Barnett ED, Lynfield R,
Sawyer MH, eds. Red Book: 2021-2024 Report of the Committee on Infectious Diseases. 32nd ed.
American Academy of Pediatrics; 2021. Accessed Marh 7, 2023. Red Book Online
Ulshen MH. Diarrhea and steatorrhea. In: McInerny TK, Adam HM, Campbell DE, Foy JM, Kamat DM,
eds. American Academy of Pediatrics Textbook of Pediatric Care. American Academy of Pediatrics;
2023. Accessed September 1, 2023. Pediatric Care Online
Worsena CR, Miller AS, King MA. Salmonella infections. Pediatr Rev. 2019;40(10):543-545.
doi:10.1542/pir.2017-0198
Content Domain
Infectious Diseases
A fully immunized 7-year-old girl is seen in the office 2 days after stepping on a nail while playing in her yard.
She was wearing tennis shoes at the time of the injury. The nail punctured her heel and produced mild
bleeding. Her mother washed her foot well with soap and water and covered it with a clean bandage. Today,
her heel appears red and swollen. She is having difficulty walking due to heel pain.
An antibiotic with coverage for methicillin-resistant Staphylococcus aureus is prescribed, and follow-up is
scheduled for tomorrow.
A. amoxicillin-clavulanic acid
B. levo oxacin
Correct answer is B
PREP Pearl(s)
Puncture wounds through a rubber-soled shoe convey a significant risk of Pseudomonas infection.
Treatment with an oral fluoroquinolone is appropriate in children of any age when it is the only oral
option available for treatment of a specific pathogen.
Tetanus vaccination should be given to any child with a significant wound whose most recent tetanus-
toxoid–containing vaccine was administered more than 5 years ago.
Critique
Puncture wounds through rubber-soled shoes present a specific risk for Pseudomonas infection, which can
lead to osteomyelitis. Other non-bite wound infections are more commonly caused by Staphylococcus
aureus. An early wound infection can be managed safely in the outpatient setting with oral antibiotics,
provided close follow-up is possible. While there is concern about the potential for prolonged use of
fluoroquinolones in young children causing tendon damage, this should not hinder its use when a
fluoroquinolone is the only oral treatment option (eg, Pseudomonas infection) or the drug of choice (eg,
anthrax).
Amoxicillin-clavulanic acid would be indicated in the setting of a dog or cat bite to provide coverage for
Pasteurella, but it would not have added benefit for a puncture wound to the foot.
This child is fully vaccinated for her age, so she would have received her most recent tetanus vaccine within
the past 3 years. She would not benefit from administration of tetanus immunoglobulin, which should be
given to any child with a significant (not clean or minor) wound who has not received at least 3 doses of a
tetanus-toxoid–containing vaccine. She would also not benefit from administration of a tetanus vaccine,
which should be given to any child with a significant wound whose most recent tetanus-toxoid–containing
vaccine was administered more than 5 years ago.
Suggested Reading(s)
Jacobs RF, McCarthy RE, Elser JM. Pseudomonas osteochondritis complicating puncture wounds of the
foot in children: a 10-year evaluation. J Infect Dis. 1989;160(4):657-661. doi:10.1093/infdis/160.4.657
Truong DH, La Fontaine J, Malone M, Wukich DK, Davis KE, Lavery LA. A comparison of pathogens in
skin and soft tissue infections and pedal osteomyelitis in puncture wound injuries affecting the foot. J
Am Podiatr Med Assoc. 2020:20-206. doi:10.7547/20-206
Volk A, Zebda M, Abdelgawad AA. Plantar and pedal puncture wounds in children: a case series study
from a single level I trauma center. Pediatr Emerg Care. 2017;33(11):724-729.
doi:10.1097/PEC.0000000000000615
Content Domain
Infectious Diseases
A 6-year-old is being evaluated in the clinic for a 2-day history of sore throat, fever, and headache. He has not
had congestion or cough. On physical examination, his temperature is 38.5 °C. His uvula is midline, there are
bilateral tonsillar exudates, and he has tender anterior cervical lymph nodes. The remainder of his
examination ndings are normal. A point-of-care rapid test is performed that con rms the diagnosis. On
review of the boy’s allergies, before the physician prescribes antibiotic treatment, the family reports that the
boy had an anaphylactic reaction to amoxicillin.
Of the following, the BEST treatment for this boy’s condition is a 10-day course of
A. azithromycin
B. cefdinir
C. clindamycin
D. trimethoprim-sulfamethoxazole
Correct answer is C
PREP Pearl(s)
Streptococcus pyogenes pharyngitis is associated with acute rheumatic fever; treatment with a 10-day
course of amoxicillin is recommended within 9 days of symptom onset.
Superficial skin manifestations of Streptococcus pyogenes (eg, impetigo) can be treated with topical or
systemic antibiotics; 24 hours of treatment is necessary before the child can return to school/day care.
Children who have had acute rheumatic fever may develop symptom recurrence. Penicillin prophylaxis
is recommended for 5 years if there is no evidence of carditis and for 10 years or until age 21 years
(whichever is later) if there is carditis that improves.
Critique
The child in the vignette has symptoms and signs consistent with group A streptococcal (GAS) pharyngitis.
The diagnosis is best confirmed with a point-of-care rapid antigen test or culture performed on a throat swab
specimen. The typical treatment is a 10-day course of oral amoxicillin (50 mg/kg/dose once per day).
However, because this child has a history of an anaphylactic reaction to amoxicillin, of the response choices,
the best treatment option is clindamycin. Azithromycin is an appropriate treatment in a child allergic to
amoxicillin or penicillin, but with a treatment duration of 5 days (12 mg/kg/dose once on day 1, then 6
mg/kg/day once per day for 4 days). There is a risk of cross reaction to cephalosporins in a child with an
anaphylactic amoxicillin allergy. A 1st-generation cephalosporin (eg, cephalexin) can be considered to treat
GAS in cases where amoxicillin is contraindicated. Cefdinir is a 3rd-generation cephalosporin, which provides
much broader coverage than is needed.
Pharyngitis is the most common manifestation of a Streptococcus pyogenes infection. The highest incidence
is in children aged 5 to 15 years; infection is most prevalent during the winter. Symptoms and signs of
streptococcal pharyngitis may include sore throat, vomiting, headache, fever, tender anterior cervical chain
lymphadenopathy, and tonsillar and posterior pharyngeal exudates. Cough and congestion make the
diagnosis of streptococcal pharyngitis less likely.
For children older than 3 years, a throat-swab rapid antigen test is recommended to confirm the diagnosis. If
the result of the rapid test is negative, a throat culture should be performed. The US Food and Drug
Administration has approved a few nucleic acid amplification tests as stand-alone tests, not requiring culture
confirmation of a negative rapid test. For children younger than 3 years, testing is not recommended given
the low risk of complications of disease, but it can be considered for a symptomatic child with an infected
close household contact.
Treatment of streptococcal pharyngitis should be started within 9 days of illness onset to decrease the risk of
rheumatic fever. Low-dose (50 mg/kg/day) amoxicillin given orally once per day for 10 days or one dose of
intramuscular benzathine penicillin G is recommended. Repeat testing after completing appropriate therapy
is not routinely recommended.
Acute rheumatic fever (ARF) is a leading cause of death worldwide, especially in low-resource countries.
Symptoms typically begin 2 to 4 weeks after a streptococcal pharyngitis infection. There is evidence that ARF
can also occur after a streptococcal skin infection. The diagnosis of ARF is made by using the Jones criteria (2
major or 1 major and 2 minor) (Table).
If acute rheumatic fever is suspected, echocardiography should be obtained; if the initial findings are normal,
this study should be repeated owing to the ongoing risk of carditis. The treatment of ARF is mainly
supportive. On the basis of signs and symptoms, treatment may also include fluid restriction, medications to
treat cardiac dysfunction, corticosteroids, or a combination of these. Arthritis is treated with aspirin or
naproxen for several weeks, which is then tapered. Chorea does not always require treatment, but if
necessary (eg, in the case of disabling symptoms), valproic acid, carbamazepine, or corticosteroids may be
considered. Treatment of ARF should be started only after the diagnosis is confirmed to avoid diagnostic
uncertainty resulting from masking of symptoms.
Children who have had ARF are at increased risk of experiencing symptom recurrence. Penicillin prophylaxis
is recommended for 5 years if there is no evidence of carditis; for 10 years or until age 21 (whichever is later)
if there is carditis that improves; or until at least age 40 years (often for life) if there are persistent signs of
carditis.
Scarlet fever, another complication of streptococcal pharyngitis or skin infection, is mediated by an exotoxin.
Scarlet fever presents with an erythematous, blanching, sandpaper-textured rash that appears on the upper
chest and neck 1 to 2 days after the start of symptoms. Nonblanching pinpoint petechiae are often present.
Other symptoms and signs may include strawberry tongue and circumoral pallor; skin desquamation may
occur later in the course.
Streptococcal skin infections may be superficial (impetigo) or deep (eg, cellulitis). Staphylococcus aureus is
the most common cause (80%) of impetigo. Impetigo most commonly affects children aged 2 to 5 years on
exposed areas of the skin (eg, face, extremities). The rash initially appears as erythematous papules, which
quickly evolve into vesicles that rupture, resulting in a yellow “honey-crusted” appearance. Localized impetigo
infections can be treated with a topical antibiotic (eg, mupirocin). For diffuse infections, a first-generation
cephalosporin (eg, cephalexin) should be considered to treat both methicillin-sensitive S aureus and S
pyogenes. In areas with high rates of methicillin resistance, antibiotic choice should be based on local
antibiograms. Children can return to day care or school 24 hours after beginning treatment.
Suggested Reading(s)
American Academy of Pediatrics. Group A streptococcal infections. In: Kimberlin DW, Barnett ED,
Lynfield R, Sawyer MH, eds. Red Book: 2021-2024 Report of the Committee on Infectious Diseases.
32nd ed. American Academy of Pediatrics; 2021:chap 130. Red Book Online
Centers for Disease Control and Prevention. Group A streptococcal (GAS) disease: for clinicians.
Accessed December 2, 2022. https://www.cdc.gov/groupastrep/diseases-hcp/index.html
Dietrich ML, Steele RW. Group A Streptococcus. Pediatr Rev. 2018; 39(8):379-391. doi:10.1542/pir.2017-
0207
Gersony WM, Starc TJ. Rheumatic fever. In: McInerny TK, Adam HM, Campbell DE, Foy JM, Kamat DM,
eds. American Academy of Pediatrics Textbook of Pediatric Care. 2nd ed. American Academy of
Pediatrics; 2023. Accessed September 1, 2023. Pediatric Care Online
Gewitz MH, Baltimore RS, Tani LY, et al. Revision of the Jones criteria for the diagnosis of acute
rheumatic fever in the era of Doppler echocardiography: a scientific statement from the American
Heart Association. Circulation. 2015;131:1806-1818. doi:10.1161/CIR.0000000000000205
Content Domain
Infectious Diseases
Subcutaneous nodules
A 10-year-old boy is diagnosed with acute lymphoblastic leukemia. His induction chemotherapy will include
vincristine, prednisolone, asparaginase, and imatinib (a tyrosine kinase inhibitor). He will require
antimicrobial prophylaxis against Pneumocystis pneumonia. His medical history is significant for an episode
of Stevens-Johnson syndrome at age 5 years after he underwent treatment with a sulfonamide agent for a
skin infection.
Of the following, the BEST antimicrobial prophylaxis option for this boy is
A. atovaquone, orally
B. dapsone, orally
C. pentamidine, aerosolized
D. trimethoprim-sulfamethoxazole, orally
Correct answer is A
PREP Pearl(s)
Pneumocystis pneumonia (PCP) is an opportunistic infection caused by Pneumocystis jirovecii. Risk
factors for PCP include cancer (lymphoproliferative disorders, receiving intensive chemotherapy), cell-
mediated immune deficiencies (eg, severe combined immunodeficiency), immunosuppression after an
organ transplant, and HIV with a CD4 count <200 cells/μL.
Antibiotic prophylaxis is highly effective in preventing Pneumocystis pneumonia. Trimethoprim-
sulfamethoxazole is the antibiotic of choice, administered 3 consecutive days per week to reduce the
risk of toxicity associated with daily dosing.
Atovaquone is a safe option and as effective in preventing Pneumocystis pneumonia as dapsone.
Dapsone should be avoided in a child with a serious reaction (eg, Stevens-Johnson syndrome, toxic
epidermal necrolysis, anaphylaxis) to trimethoprim-sulfamethoxazole. Aerosolized pentamidine is
reserved for children who cannot tolerate other agents.
Critique
The boy in the vignette, with acute lymphoblastic leukemia, is at high risk of developing Pneumocystis
pneumonia (PCP) while receiving chemotherapy. Twenty-two percent of children receiving ≥3
chemotherapeutic agents develop PCP compared with 5% of children receiving <3 agents. Antibiotic
prophylaxis against PCP is highly effective. Trimethoprim-sulfamethoxazole, a sulfonamide, is the antibiotic of
choice for PCP prophylaxis (administered 3 consecutive days per week to reduce the risk of toxicity
associated with daily dosing). However, this boy developed Stevens-Johnson syndrome after receiving a
sulfonamide, so both trimethoprim-sulfamethoxazole and dapsone are contraindicated. Dapsone should be
avoided in a child with a serious reaction (eg, Stevens-Johnson syndrome, toxic epidermal necrolysis,
anaphylaxis) to trimethoprim-sulfamethoxazole. In this case, atovaquone is the best next option. Outcomes
with atovaquone are similar to those seen with dapsone. It is safe and effective but considerably more
expensive than trimethoprim-sulfamethoxazole and dapsone. Aerosolized pentamidine (300 mg once per
month) can be used for PCP prophylaxis in children aged >5 years who can cooperate with nebulization.
Aerosolized pentamidine, however, is less effective than the other 3 agents.
Pneumocystis pneumonia is an opportunistic fungal infection caused by Pneumocystis jirovecii (formerly
known as Pneumocystis carinii). Human-to-human transmission of P jirovecii occurs through an airborne
route. Individuals with normal immune systems experience asymptomatic infection (colonization) and serve
as a pool for transmission. Most healthy children acquire the infection by age 2 years. Children at risk of PCP
include those with cancer (lymphoproliferative disorders, receiving intensive chemotherapy), cell-mediated
immune deficiencies (eg, severe combined immunodeficiency), immunosuppression after an organ
transplant, and HIV with a CD4 count <200 cells/μL.
Whether the course of PCP is abrupt or indolent depends on the T cell defect. An abrupt onset with rapid
progression over a few days is seen in children with non-HIV causes of T cell suppression. In contrast, an
indolent onset with a slowly progressive course over 3 weeks or more is observed in children with HIV
infection. Signs and symptoms of PCP in children and adolescents include fever, cough, tachypnea, and
dyspnea. Hypoxia is characteristic of PCP pneumonia. Most children with PCP have an arterial partial
pressure of oxygen less than 70 mm Hg and an alveolar-arterial gradient >30.
The diagnosis of PCP is made by detecting P jirovecii, in pulmonary secretions or other specimens, using a
polymerase chain reaction (PCR) assay. The least invasive method of obtaining a specimen is induced sputum
(after inhalation of 3% hypertonic saline). However, this method requires cooperation and has a low
sensitivity (20% to 40%). The ideal method of specimen collection is bronchoalveolar lavage (75% to 95%
yield). If clinical suspicion for PCP infection is high and the bronchoalveolar lavage is negative, a
transbronchial biopsy should be attempted (87% to 95% yield).
Suggested Reading(s)
American Academy of Pediatrics. Pneumocystis jirovecii infections. In: Kimberlin DW, Barnett ED,
Lynfield R, Sawyer MH, eds. Red Book: 2021–2024 Report of the Committee on Infectious Diseases.
32nd ed. American Academy of Pediatrics; 2021. Accessed September 1, 2023. Red Book Online
Gigliotti F, Wright TW. Pneumocystis jirovecii. In: Long SS, Pickering LK, Prober CG. Principles and
Practice of Pediatric Infectious Diseases. 4th ed. Elsevier; 2012:1230-1233.
Light M. Pneumonia. In: McInerny TK, Adam HM, Campbell DE, DeWitt TG, Foy JM, Kamat DM, eds.
American Academy of Pediatrics Textbook of Pediatric Care. American Academy of Pediatrics; 2023.
Accessed September 1, 2023. Pediatric Care Online
Content Domain
Infectious Diseases
A healthy, unvaccinated 10-year-old boy is seen for jaw swelling of 2 days’ duration. A few days ago he had a
temperature of 38 °C, headache, and myalgias, which have since resolved. On physical examination, his vital
signs are within normal limits. There is bilateral jaw swelling (Figure 1) that is mildly tender to palpation,
without erythema, fluctuance, or warmth. The Stensen ducts are erythematous (Figure 2). The remainder of
his physical examination findings are normal.
Courtesy of P Wehrle.
Reprinted with permission from Kimberlin DW, Barnett ED, Lynfield R, Sawyer MH, eds. Red Book: 2021–
2024 Report of the Committee on Infectious Diseases. 32nd ed. American Academy of Pediatrics; 2021.
Reprinted with permission from Kimberlin DW, Barnett ED, Lynfield R, Sawyer MH, eds. Red Book: 2021–
2024 Report of the Committee on Infectious Diseases. 32nd ed. American Academy of Pediatrics; 2021.
B. obtain radiography
C. prescribe antibiotics
D. refer to an otolaryngologist
Correct answer is A
PREP Pearl(s)
Mumps infection should be managed with supportive care.
The mumps vaccine significantly reduces the risk of experiencing complications of mumps infection.
Mumps orchitis usually presents as testicular pain and swelling, fever, nausea, and vomiting; about
one-half of affected men will experience testicular atrophy. Sterility is not a complication of mumps
orchitis.
Critique
The child in the vignette most likely has viral parotitis, which is best managed with supportive care. If he had
overlying erythema and warmth, or if his Stensen duct was exuding purulent secretions, antibiotics would be
indicated. Ultrasonography would be appropriate if there were findings suggestive of an abscess (eg,
fluctuance). Radiography is not indicated for a child with this boy’s findings. Referral to an otolaryngologist
would be appropriate if his findings raised concern regarding an abscess or if he was experiencing recurrent
symptoms.
The most common cause of parotitis is a viral infection. It may occur unilaterally or bilaterally. Before a
vaccine was available, mumps was one of the most common causes of viral parotitis. Although mumps
outbreaks are currently rare, they still occur, and providers must consider mumps in the differential
diagnosis of parotitis. Children with mumps may experience prodromal symptoms of low-grade fever,
headache, and myalgias.
Complications of mumps infection are more common in adults and unvaccinated individuals. The most
common complication in postpubertal men is orchitis, which usually presents with testicular pain and
swelling, fever, nausea, and vomiting; about one-half of affected men experience testicular atrophy. Sterility
is not a complication of mumps orchitis. Postpubertal women may experience oophoritis and mastitis. Other
complications include aseptic meningitis, pancreatitis, and sensorineural hearing loss (transient or
permanent). The mumps vaccine significantly reduces the risk of experiencing complications of mumps
infection.
The treatment of viral parotitis is largely supportive. If the cause is determined to be mumps, the affected
individual should be isolated for 5 days following the start of parotid gland swelling. Classmates who are not
immune to mumps may need to be excluded for at least 26 days. Unimmunized close contacts should receive
mumps vaccine; this will not offer protection against the current exposure but will be protective in the event
of future exposures. When an outbreak of mumps occurs, local public health authorities may determine that
additional mumps vaccine doses are warranted for fully vaccinated individuals deemed to be at high risk.
Suggested Reading(s)
American Academy of Pediatrics. Mumps. In: Kimberlin DW, Barnett ED, Lynfield R, Sawyer MH, eds.
Red Book: 2021–2024 Report of the Committee on Infectious Diseases. 32nd ed. American Academy of
Pediatrics; 2021. Accessed September 1, 2023. Red Book Online
Chase L, Lye CG. Parotitis. In: Gershel JC, Rauch DA, eds. Caring for the Hospitalized Child: A Handbook
of Inpatient Pediatrics. 2nd ed. American Academy of Pediatrics; 2018:129-134.
Marlow M, Haber P, Hickman C, Patel M. Mumps. In: Hall E, Wodi AP, Hamborsky J, Morelli V, Schillie S,
eds. Epidemiology and Prevention of Vaccine-Preventable Diseases. 14th ed. Public Health Foundation;
2021:chap 15. Accessed September 1, 2023.
https://www.cdc.gov/vaccines/pubs/pinkbook/mumps.html
Content Domain
Infectious Diseases
A 2-year-old girl who was recently adopted with her sibling is seen for a health supervision visit. She is
growing and developing normally. The birth mother of both children has a history of hepatitis C virus (HCV)
infection, and the girl’s sibling was diagnosed with HCV infection as a toddler. The remainder of the family
history is unknown.
The girl’s physical examination findings are normal. The adoptive parents are concerned about whether she
has an HCV infection.
C. reassure the family that testing for hepatitis C virus infection would have already occurred
Correct answer is B
PREP Pearl(s)
All infants with perinatal hepatitis C virus (HCV) exposure should be screened for HCV infection.
Options for hepatitis C virus HCV screening include anti-HCV antibody testing at or after age 18 months
(preferred) or HCV-ribonucleic-acid–(RNA) testing after age 2 months.
Children with chronic hepatitis C virus infection should be referred to a pediatric gastroenterologist or
infectious disease specialist for monitoring and therapy; highly effective therapies are available for
children older than age 3 year.
Critique
The girl in the vignette is at risk of experiencing vertically acquired chronic hepatitis C virus (HCV) infection,
because she was born to a mother with a history of HCV infection. The best next management step is to
screen her for HCV infection with anti-HCV antibody testing, which should be obtained at or after age 18
months. If positive, she should be referred to a pediatric gastroenterologist or infectious disease specialist for
monitoring and treatment of chronic HCV infection. The girl in the vignette is older than 18 months, so
screening may have already occurred; however, screening rates in infants exposed to HCV remain low (23%)
and information regarding this child’s history is incomplete. Therefore, reassurance that testing must have
already occurred would not be appropriate. Exclusion from out-of-home child care is not recommended,
even if HCV is detected. This girl has not been diagnosed with HCV infection, and therefore referral to a
pediatric infectious disease specialist is not indicated.
Hepatitis C is a single-stranded ribonucleic-acid (RNA) virus that can cause persistent infection (chronic HCV)
in children and adults. Although most children with chronic HCV are asymptomatic, liver failure due to HCV
infection is one of the leading indications for liver transplantation among adults. Pediatric HCV infection most
often occurs as a result of maternal-fetal transmission, occurring in 5% to 6% of HCV-exposed infants.
Transmission risk increases with a higher maternal viral load, maternal coinfection with HIV, internal fetal
monitoring, prolonged rupture of membranes, and vaginal lacerations. Up to 25% to 50% of infected infants
spontaneously clear the HCV infection by age 4 years.
Infants born to mothers with HCV infection should be screened for HCV. Screening options include HCV-RNA
testing, performed as early as age 2 months, or anti-HCV antibody testing at or after age 18 months. Because
of the higher cost of the HCV-RNA test, lack of approved therapy in children younger than 3 years, and the
possibility of spontaneous HCV clearance, anti-HCV antibody testing is preferred for screening. However,
when concern arises for symptomatic liver disease, limited ability to follow up, significant distress about the
potential diagnosis, or a combination of these, HCV-RNA testing can be performed as early as age 2 months.
Children diagnosed with HCV infection should be referred to a specialist in pediatric gastroenterology or
infectious diseases for further monitoring and therapy. Highly successful direct-acting antiviral regimens are
available for children as young as age 3 years. Treatment options vary depending on genotype and previous
treatment.
Suggested Reading(s)
American Academy of Pediatrics. Hepatitis C. In: Kimberlin DW, Barnett ED, Lynfield R, Sawyer MH, eds.
Red Book: 2021–2024 Report of the Committee on Infectious Diseases. 32nd ed. American Academy of
Pediatrics; 2021. Accessed June 20, 2022. Red Book Online
Jhaveri R. We need a new national strategy for hepatitis C virus screening. Pediatrics.
2020;145(3):e20193060. doi:10.1542/peds.2019-3060
Leung DH, Squires JE, Jhaveri R, et al. Hepatitis C in 2020: A North American Society for Pediatric
Gastroenterology, Hepatology, and Nutrition position paper. J Pediatr Gastroenterol Nutr.
2020;71(3):407-417. doi:10.1097/MPG.0000000000002814
Lopata SM, McNeer E, Dudley JA, et al. Hepatitis C testing among perinatally exposed infants.
Pediatrics. 2020;145(3):e20192482. doi:10.1542/peds.2019-2482
Recommendations for HCV testing of perinatally exposed children and siblings of children with HCV
infection. American Association for the Study of Liver Diseases. Accessed September 14, 2022, 2022.
https://www.hcvguidelines.org/unique-populations/children
Content Domain
Infectious Diseases
A previously healthy, underimmunized toddler, who is a recent immigrant from the United Kingdom, spent
30 minutes in the office waiting room yesterday prior to her appointment. On physical examination, she was
febrile with bilateral conjunctivitis, nasal congestion, a cough, and a maculopapular rash. Her
underimmunized status prompted testing for measles, which was positive. Treatment with vitamin A and
supportive care were recommended. There were several children in the waiting room at the same time as
this child.
Of the following fully immunized children who were in the waiting room, the child for whom treatment with
intravenous immune globulin is MOST recommended is the
B. 8-year-old who received intravenous immune globulin for Kawasaki disease 3 months ago
Correct answer is A
PREP Pearl(s)
Severely immunocompromised children should receive intravenous immune globulin within 6 days of
measles exposure due to their high vulnerability and ineligibility for live virus vaccination.
Vaccination with measles-mumps-rubella vaccine can be administered to children 6-to-11 months of
age when exposures occur; this dose will not count toward the primary vaccine series.
Immune globulin can be administered to protect pregnant adolescents and young adults exposed to
measles for whom vaccination is not recommended.
Critique
The fully immunized 6-year-old girl who underwent a bone marrow transplant 6 months ago should receive
intravenous immune globulin (IVIG) within 6 days of measles exposure because she is highly vulnerable to
measles and is not eligible for live virus vaccination. Other individuals eligible to receive IVIG to prevent or
decrease the severity of measles after an exposure include pregnant women with no evidence of immunity to
measles, infants and children with severe primary immunodeficiencies, those who have received a solid
organ transplant, and those with uncontrolled HIV with severe immunosuppression.
To help prevent measles infection, the healthy 9-month-old infant should receive a measles-mumps-rubella
(MMR) vaccine within 72 hours of exposure; this dose will not count toward his primary series but will provide
protection in the short term. The healthy 18-month-old girl should be given her second dose of MMR within
72 hours of exposure, provided she received the first dose more than 28 days ago. An additional dose at 4-to-
6 years of age would no longer be required. The boy that received IVIG for Kawasaki disease is fully
immunized against measles, therefore, he does not need either IVIG or MMR administration. Of note,
children should not receive any live vaccines for 11 months following high-dose IVIG administration.
Suggested Reading(s)
American Academy of Pediatrics. Measles. In: Kimberlin DW, Barnett ED, Lynfield R, Sawyer MH, eds.
Red Book: 2021–2024 Report of the Committee on Infectious Diseases. 32nd ed. American Academy of
Pediatrics; 2021. Accessed September 1, 2022. Red Book Online
Arciuolo RJ, Jablonski RR, Zucker JR, Rosen JB. Effectiveness of measles vaccination and immune
globulin post-exposure prophylaxis in an outbreak setting—New York City, 2013. Clin Infect Dis.
2017;65(11):1843-1847. doi:10.1093/cid/cix639
Kohlmaier B, Holzmann H, Stiasny K, et al. Effectiveness and safety of an intravenous immune globulin
(IVIG) preparation in post-exposure prophylaxis (PEP) against measles in infants. Front Pediatr.
2021;9:762793. doi:10.3389/fped.2021.762793
Nguyen THT, Nguyen TV, Luong QC, Ho TV, Faes C, Hens N. Understanding the transmission dynamics
of a large-scale measles outbreak in Southern Vietnam. Int J Infect Dis. 2022;122:1009-1017.
doi:10.1016/j.ijid.2022.07.055
Content Domain
Infectious Diseases
A 10-year-old girl is seen in the emergency department the day after returning from summer camp. She
reports having had fever, myalgia, and headache during the previous week, with a recently worsening
headache and a stiff neck, which prompted her parents to bring her to the emergency department.
Evaluation reveals aseptic meningitis, and cerebrospinal fluid test results are positive for West Nile virus.
Supportive management is recommended, and the patient is discharged to home. The girl’s family asks how
they can protect the patient’s younger sister from the disease.
C. have the sister avoid contact with birds and bird feeders
D. not allow the sisters to share a bedroom while the patient is symptomatic
Correct answer is A
PREP Pearl(s)
West Nile virus is spread through the bite of an infected mosquito; human to human transmission is
extremely rare.
Treatment for West Nile virus is supportive care.
The most effective protection from mosquito-transmitted infection is to apply a DEET (N,N-diethyl-
meta-toluamide)–based insect repellent before outdoor play.
Critique
The most effective intervention to protect the patient’s sister from infection with West Nile virus is to apply a
DEET (N,N-diethyl-meta-toluamide)–based insect repellent before outdoor play. West Nile virus is an
arbovirus that is spread by the bite of the Culex mosquito. Decreasing the risk of mosquito bites is the
primary means of preventing West Nile virus infection. Humans are considered a “dead end” host for the
virus. Transmission by organ donation or from mother to infant has been documented; however, these
methods of transmission are exceedingly rare.
West Nile virus is amplified in the bird population. Mosquitoes become infected by feeding on infected birds
and then pass the virus to humans through their bite. The virus is not generally transmitted to humans
through direct bird contact. West Nile virus is not passed by airborne or contact routes from person to
person; therefore, not sharing a bedroom with an infected individual or improving hand hygiene practices
would not be effective in preventing transmission.
Suggested Reading(s)
American Academy of Pediatrics. West Nile virus. In: Kimberlin DW, Brady MT, Jackson MA, Long SS,
eds. Red Book: 2018-2021 Report of the Committee on Infectious Diseases. 31st ed. American Academy
of Pediatrics; 2018. Accessed December 7, 2021. Red Book Online
Colpitts TM, Conway MJ, Montgomery RR, Fikrig E. West Nile virus: biology, transmission, and human
infection. Clin Microbiol Rev. 2012;25(4):635-648. doi:10.1128/CMR.00045-12
Rochlin I, Faraji A, Healy K, Andreadis TG. West Nile virus mosquito vectors in North America. J Med
Entomol. 2019;56(6):1475-1490. doi:10.1093/jme/tjz146
Content Domain
Infectious diseases
The correct answer is: apply a DEET (N,N-diethyl-meta-toluamide)–based insect repellent before outdoor play
A 3-year-old girl is evaluated for right facial swelling for 1 day (Figure 1). The family has recently moved to the
United States from China, and her immunization history is unknown. Her illness was preceded by 3 days of
tactile fevers, along with decreased appetite and activity. On physical examination, her temperature is 38.5
°C, heart rate is 90 beats/min, and respiratory rate is 20 breaths/min. There is swelling and erythema at the
angle of the left mandible and preauricular area with mild tenderness to palpation. Oral examination (Figure
2) reveals swelling and erythema of the Stensen duct; parotid massage elicits clear discharge from the duct.
Of the following, the MOST likely cause of this child’s signs and symptoms is
B. mumps paramyxovirus
C. Sjögren syndrome
D. Staphylococcus aureus
Correct answer is B
PREP Pearl(s)
Children with bacterial parotitis are generally ill-appearing, with high temperature and unilateral
parotid swelling, erythema, warmth, and tenderness. Children with viral parotitis are generally less ill-
appearing, have little warmth and erythema over the parotid gland, and may have bilateral
involvement.
Inflammatory syndromes (eg, Sjögren syndrome) should be considered when assessing a child with
recurrent parotid swelling.
Embryologic branchial cleft anomalies can lead to swelling of the parotid and preauricular areas.
Critique
The presentation of the girl in the vignette is consistent with viral parotitis. Mumps paramyxovirus, Epstein-
Barr virus, human herpesvirus 6, parainfluenza virus, and HIV are the most common causes. Viral parotitis
most often occurs in children between the ages of 3 and 10 years. The clinical presentation often begins with
several days of prodromal symptoms (eg, fever and malaise) before the onset of unilateral parotid gland
swelling (Figure 1). These symptoms can progress, after several days, to bilateral involvement. Physical
examination findings include swelling over the involved parotid gland that is mildly tender to palpation but
lacks significant erythema or warmth. On oral examination, the Stensen duct (Figure 2; located in the buccal
mucosa adjacent to the upper second molar) shows redness and clear secretions.
It is important to consider anatomical abnormalities in the differential diagnosis for children with swelling
that affects the neck and preauricular area. Branchial cleft anomalies (BCAs) are the second most common
lesions of the head and neck in children; thyroglossal duct cysts are the most common. Congenital BCAs
result from incomplete involution of branchial cleft structures during embryogenesis, which generally occurs
by 7 weeks’ gestation. Anomalies of the first branchial cleft result from incomplete fusion of the ventral part
of the first and second arches. Facial nerve migration and development of the parotid gland occur at the
same time and, therefore, can be involved. Branchial cleft anomalies often initially present as smooth,
fluctuant nodules located between the external auditory canal and submandibular region; when infected,
swelling, pain, and purulent discharge may be seen. Associated sinuses or fistulas may lead to openings at
the external auditory canal, middle ear cleft, postauricular region, and the angle of the mandible. There is no
connection to the Stensen duct.
Sjögren syndrome is an autoimmune disorder that causes chronic, recurrent bilateral parotitis and dryness of
the eyes and mouth in the pediatric population. Associated laboratory findings include positive antinuclear
antibody, positive rheumatoid factor, and hypergammaglobulinemia. Although rare in the pediatric
population, primary Sjögren syndrome must be considered in a child with recurrent parotitis because prompt
referral to a pediatric rheumatologist is important for treatment and monitoring for complications.
Bacterial infection of the parotid gland is most often seen in children younger than 2 months or older than 10
years. The most common cause is Staphylococcus aureus; streptococci, gram-negative bacilli, and anaerobes
may also be seen. The clinical presentation of bacterial parotitis is acute unilateral swelling of the parotid
gland with significant erythema, warmth, and tenderness to palpation. Examination of the Stensen duct
reveals inflammation and purulent secretions. Affected children are usually ill-appearing with a high
temperature.
Suggested Reading(s)
American Academy of Pediatrics. Mumps. In: Kimberlin DW, Barnett ED, Lynfield R, Sawyer MH, eds.
Red Book: 2021–2024 Report of the Committee on Infectious Diseases. 32nd ed. American Academy of
Pediatrics; 2021. Red Book Online
Baszis K, Toib D, Cooper M, French A, White A. Recurrent parotitis as a presentation of primary
pediatric Sjögren syndrome. Pediatrics. 2012;129(1e179-e182. doi:10.1542/peds.2011-0716
Chase L, Lye CG. Neck Masses. In: Gershel, JC, Rauch DA, eds. Caring for the Hospitalized Child: A
Handbook of Inpatient Pediatrics. 2nd ed. American Academy of Pediatrics; 2018:chap 19:151-158.
Hall J, Hays JF, Sulo KS, David J. Sudden onset of a unilateral erythematous preauricular mass in a
preterm infant. Neoreviews. 2021;22(2):e136-e140. doi:10.1542/neo.22-2-e136
Content Domain
Infectious Diseases
A 15-year-old boy with no significant medical history, other than a penicillin allergy (hives), is seen in the
office for evaluation of an injury. The boy was involved in an altercation at school the day before
presentation. He punched another classmate in the mouth with a closed fist, sustaining a large abrasion of
the dorsum of his hand over his fourth and fifth metacarpophalangeal joints. On physical examination, the
boy is afebrile. The injured hand is red, swollen, and painful. The remainder of his examination findings are
unremarkable.
Of the following, the MOST appropriate oral therapy for this adolescent’s condition is
A. amoxicillin-clavulanate
B. cephalexin
C. ciprofloxacin
Correct answer is D
The most common bacteria isolated from cultures from human bite wounds are Staphylococcus
aureus, Streptococcus pyogenes, Eikenella corrodens, and Fusobacterium species.
The first-line therapy for human bite wound infections is amoxicillin-clavulanate.
For children with a penicillin allergy, the recommended alternative treatment for human bite wound
infections is extended-spectrum cephalosporin or trimethoprim-sulfamethoxazole plus clindamycin.
The adolescent in the vignette has findings consistent with cellulitis of his hand after punching another
person in the mouth with a closed fist. “Fight bite” occurs when an individual strikes another person in the
mouth and sustains a break in the skin on contact with the teeth. Antibiotic treatment should be initiated.
The first-line therapy for infections caused by a human bite wound is amoxicillin-clavulanate; however, the
boy in the vignette has a penicillin allergy. The recommended alternative treatment in the case of penicillin
allergy is either extended-spectrum cephalosporin or trimethoprim-sulfamethoxazole plus clindamycin.
Although first-generation cephalosporins, such as cephalexin, provide adequate coverage for gram-positive
bacteria, they lack activity against anaerobic organisms (eg, Eikenalla corrodens). Fluoroquinolones (eg,
ciprofloxacin) have good in vitro activity against organisms that cause infection from human bite wounds;
however, clinical data that demonstrate efficacy and safety are lacking. Additionally, current age restrictions
for fluoroquinolones limit their use in children; they should not be considered first-line therapy for bite
infections in children.
As with all wounds, human bite wound management includes thorough cleansing and irrigation with copious
amounts of water or saline. The history and physical examination should evaluate for the location, extension,
and severity of wound, the possibility of a retained foreign body, as well as the potential for injury to
underlying structures (eg, blood vessels, nerves, organs, tendons, and bones). Tetanus immunization status
should be determined, and tetanus prophylaxis should be administered if indicated. Obvious dirt and foreign
bodies should be removed, and necrotic tissue should be debrided. Table 1 outlines the general
management of bite wounds.
Wound infections caused by a human bite are typically polymicrobial, with mixed anaerobic and aerobic
bacteria. The most common bacteria isolated from human bite wounds are Staphylococcus aureus,
Streptococcus pyogenes, Eikenella corrodens, and Fusobacterium species. Table 2 lists bacteria commonly
found in dog, cat, and human bite wounds. It is important to consider other types of infections that may be
transmitted from a human bite, such as the following:
Cytomegalovirus
Hepatitis B
Hepatitis C
Herpes simplex virus
HIV
Syphilis
Although limited data exist regarding the initiation of prophylactic antibiotics after sustaining an animal or
human bite, there is a general consensus that indications for empiric antibiotic therapy include the following:
Suggested Reading(s)
American Academy of Pediatrics. Bite wounds. In: Kimberlin DW, Brady MT, Jackson MA, Long SS, eds.
Red Book: 2021-2024 Report of the Committee on Infectious Diseases. 32nd ed. American Academy of
Pediatrics; 2021. Accessed December 7, 2022. Red Book Online
Black KD, Cico SJ, Caglar D. Wound management. Pediatr Rev. 2015;36(5):207-216. doi:10.1542/pir.36-5-
207
Bula-Rudas FJ, Olcott JL. Human and animal bites. Pediatr Rev. 2018;39(10):490-500.
doi:10.1542/pir.2017-0212
Herendeen NE, Szilagyi PG. Animal and human bites. In: McInerny TK, Adam HM, Campbell DE, DeWitt
TG, Foy JM, Kamat DM, eds. American Academy of Pediatrics Textbook of Pediatric Care. American
Academy of Pediatrics; 2021:chap 215. Pediatric Care Online
Content Domain
Infectious Diseases
Reprinted with permission from Bula-Rudas FJ, Olcott JL. Human and animal bites. Pediatr
Rev. 2018;39(10):493.
AAP PREP 2024 - Question 121/267 Infectious Diseases Question 19/30
A 3-year-old boy is seen in the emergency department for a 2-day history of back pain and low-grade fevers.
Today he is unable to stand or sit; his mother carried him into the emergency department. The family
recently traveled to visit relatives at their dairy farm, where the boy tried homemade cheeses, interacted with
dairy cows, and played with the farm cats and kittens. Magnetic resonance imaging of the spine
demonstrates spondylodiscitis (vertebral osteomyelitis extending to the disk space of L3-L5). A computed
tomography–guided biopsy of the lesion is performed by the interventional radiologist after the consulting
orthopedic surgeon determined that an open biopsy would present significant risk. The boy is given
intravenous vancomycin and ceftriaxone pending culture results. Pathologic test results suggest acute
osteomyelitis; the fungal, bacterial, and mycobacterial culture results are all negative. Results of a cell-free
DNA next-generation sequencing test are positive for Kingella kingae, at which time his antibiotic treatment is
appropriately narrowed.
Of the following, the MOST significant risk factor for this patient’s condition is
C. exposure to kittens
D. male sex
Correct answer is A
PREP Pearl(s)
Kingella kingae bone and joint infections are common in very young children.
Treatment of Kingella kingae is amoxicillin or a cephalosporin.
Cell-free DNA next-generation sequencing is a noninvasive, but expensive, method of diagnosing
difficult-to-sample or culture-negative organisms that can cause osteomyelitis.
Critique
Of the risk factors in the vignette, only age is a risk factor for Kingella kingae vertebral osteomyelitis. Kingella
kingae is a fastidious gram-negative bacillus that is difficult to culture. It colonizes the throat and is thought to
spread to the bone and joints by intermittent bacteremia. Although this organism can cause culture-negative
osteomyelitis and septic arthritis at any age, these conditions most commonly occur in children between 3
months and 4 years of age. In this age group, these infections are thought to occur almost as commonly as
Staphylococcus aureus bone and joint infections. A cephalosporin or a penicillin is used to treat K kingae
infections.
Consumption of unpasteurized milk or cheese is associated with spinal osteomyelitis due to brucellosis. Close
kitten exposure is associated with Bartonella henselae spinal osteomyelitis. In children, male sex is not a risk
factor for any particular microbial cause of spinal osteomyelitis.
Because K kingae is difficult to culture and is a common cause of spinal osteomyelitis in this boy’s age group,
cell-free DNA (cfDNA) next-generation sequencing is an appropriate diagnostic test. This method of detecting
deep-seated infections by means of a blood sample is performed in a commercial laboratory and is generally
much more expensive than conventional testing. The test can be used to identify most bacterial, fungal, and
parasitic causes of disease as well as many viral causes (eg, adenovirus and the herpesviruses). Although
cfDNA next-generation sequencing can provide a simple, noninvasive microbial diagnosis for a lesion seen on
imaging, it would not be useful to diagnose oncologic or rheumatologic causes; its use should be carefully
considered.
Suggested Reading(s)
Dayer R, Alzahrani MM, Saran N, et al. Spinal infections in children: a multicentre retrospective study.
Bone Joint J. 2018;100-B(4):542-548. doi:10.1302/0301-620X.100B4.BJJ-2017-1080.R1
Principi N, Esposito S. Infectious discitis and spondylodiscitis in children.Int J Mol Sci. 2016;17(4):539.
doi:10.3390/ijms17040539
Samara E, Lutz N, Zambelli P-Y. Kingella kingae spinal infections in children. Children. 2022;9(5):705.
doi:10.3390/children9050705
Wong M, Williams N, Cooper C. Systematic review of Kingella kingae musculoskeletal infection in
children: epidemiology, impact and management strategies Pediatric Health Med Ther.2020;11:73-84.
doi:10.2147/PHMT.S217475
Content Domain
Infectious Diseases
A 5-year-old boy is seen in the emergency department after he was bitten by a raccoon. He was playing with
the raccoon in his backyard in Georgia, when it bit him on the forearm and subsequently ran into the woods
behind the boy’s home. His parents looked for the animal but were unable to capture it. On physical
examination, he has 2 small puncture wounds to his right forearm that are not bleeding and do not appear
erythematous. The wound is thoroughly cleansed and irrigated.
Of the following, the BEST next step in this boy’s management is
Correct answer is B
PREP Pearl(s)
Bats, raccoons, skunks, foxes, and other mammalian carnivores should be regarded as rabid, and a bite
requires administration of both rabies immune globulin and the rabies vaccine series.
Rabies immune globulin should be infiltrated into all areas in and around the wound; any remaining immune
globulin should be administered intramuscularly in a site distant from the vaccine administration.
The rabies vaccine should be administered intramuscularly to children in the deltoid or anterolateral thigh,
with the first dose administered on the same day as RIG, followed by doses on days 3, 7, and 14.
Critique
The boy in the vignette was bitten by a wild racoon, which should be considered rabid unless the geographic
area is known to be rabies free or the animal is proven to be rabies negative by laboratory testing. In this
case, the family was unable to capture the animal for euthanasia and testing. According to the Centers for
Disease Control and Prevention, rabies has been found in raccoons throughout the southern and eastern
United States. The boy requires administration of rabies immune globulin (RIG) and rabies vaccine series.
Administration of RIG alone or the rabies vaccine series alone is not sufficient to provide protection against
the development of rabies in this child. The Table shows the recommendations for rabies postexposure
prophylaxis. Discharge home after placement of a sterile dressing and parental reassurance would not be the
appropriate treatment for a child who has sustained a rabies exposure.
The care for any type of bite wound involves thorough examination of the area, cleansing with soap and
water, and determining: 1) tetanus immunization status and the need for tetanus prophylaxis if appropriate;
2) the need for rabies prophylaxis; and 3) the need for antibiotic therapy or prophylaxis.
Bats, raccoons, skunks, foxes, coyotes, and other mammalian carnivores are the major carriers of rabies in
the United States. Unless the animal is captured and proven to not have rabies by laboratory evaluation, it
should be regarded as rabid, and rabies prophylaxis should be initiated. Bat exposures require special
consideration; a bat bite, exposure to bat fluids (eg, urine), a bat found in an enclosed room where a child is
sleeping, or a bat noted to be in close proximity to a child should all be considered a rabies exposure, even if
no obvious bite wound is visible.
Bites from dogs, cats, ferrets, and other rodents or lagomorphs (mice, squirrels, rats, rabbits) have a lower
risk for rabies transmission and should be considered on an individual basis. If the animal appears healthy
and can be captured and observed/quarantined for 10 days, then prophylaxis is only indicated if the animal
develops signs or symptoms of rabies. Rabies prophylaxis should be initiated when an animal is known or
suspected to be rabid, or cannot be captured and observed. Consultation with the local health department
can be helpful in determining the risk of rabies in specific geographical areas or in particular animals.
It is important to administer RIG and the rabies vaccine series appropriately. One dose of rabies immune
globulin should be infiltrated into all areas in and around the wound; any remaining immune globulin should
be administered intramuscularly in a site distant from the vaccine administration site. Four doses of rabies
vaccine should be administered. The first dose should be administered on the same day as the RIG, followed
by doses on days 3, 7, and 14 after the initial dose. Rabies vaccine should be administered intramuscularly to
children in either the deltoid or anterolateral thigh. The rabies vaccine should not be administered in the
gluteal area. Rabies immune globulin can be administered up to 7 days after the first dose of rabies vaccine.
Suggested Reading(s)
American Academy of Pediatrics. Rabies. In. Kimberlin DW, Barnett ED, Lynfield R, Sawyer MH, eds. Red
Book: 2021–2024 Report of the Committee on Infectious Diseases. 32nd ed. American Academy of
Pediatrics; 2021. Red Book Online
Bula-Rudas FJ, Olcott JL. Human and animal bites. Pediatr Rev. 2018;39(10):490-500.
doi:10.1542/pir.2017-0212
Centers for Disease Control and Prevention. Wild Animals. Updated April 6, 2020. Accessed September
1, 2023. https://www.cdc.gov/rabies/location/usa/surveillance/wild_animals.html
Herendeen NE, Szilagyi PG. Animal and human bites. In: McInerny TK, Adam HM, Campbell DE, DeWitt
TG, Foy JM, Kamat DM, eds. American Academy of Pediatrics Textbook of Pediatric Care. American
Academy of Pediatrics; 2021:chap 215. Accessed September 1, 2023. Pediatric Care Online
Rapoport M. Animal bites: assessing risk for rabies and providing treatment. Pediatr Rev.
1997;18(4):142-143. doi:10.1542/pir.18-4-142
Content Domain
Infectious Diseases
A 12-year-old girl with type 1 diabetes was admitted to the hospital for acute pyelonephritis with
dehydration. She was treated with intravenous fluids and antibiotics. Her condition initially improved, but on
hospital day 4 she developed frequent and severe diarrhea. Her vital signs today include a temperature of
39.5 °C, heart rate of 132 beats/min, respiratory rate of 28 breaths/min, blood pressure of 92/51 mm Hg, and
oxygen saturation of 99% in room air. Physical examination findings are unremarkable. An abdominal
radiograph is obtained (Figure).
Courtesy of M. Anderson
Figure. Abdominal radiograph on hospital day 4 from the patient described in the vignette.
Urine
Leukocyte esterase ++ 0
Nitrites + 0
Blood
Stool
Of the following, the BEST next step in this patient’s management is administration of
A. intravenous metronidazole
B. intravenous vancomycin
C. oral vancomycin
D. rectal vancomycin
Correct answer is C
PREP Pearl(s)
Oral vancomycin is one of the most effective treatments for Clostridioides difficile; it is the first-line
treatment for severe disease.
Fulminant Clostridioides difficile disease may require both medical and surgical intervention.
Critique
The child in the vignette has severe Clostridium difficile colitis as evidenced by fever, elevated white blood cell
count, elevated/increasing creatinine level, and a low albumin level. Her repeat urinalysis suggests that her
pyelonephritis is resolving. The best next step in her management is to administer oral vancomycin.
Clostridium difficile colitis can be categorized as mild-moderate, severe, or fulminant. This girl does not meet
the criteria for mild or moderate disease (based on her dehydration and abnormal laboratory findings) or
fulminant disease (shock or toxic megacolon). Evidence of severe ileus on abdominal radiography places an
infected individual at significant risk of fulminant disease with rapid decompensation.
For mild to moderate disease, the first-line treatment recommendation is oral metronidazole or oral
vancomycin. For severe disease, oral vancomycin is the drug of choice. When severe ileus or toxic megacolon
is present, rectal vancomycin with or without intravenous metronidazole would be the treatment of choice;
surgical intervention may be considered.
Intravenous metronidazole is not as effective as oral metronidazole for C difficile, and neither agent is as
effective as oral vancomycin. Intravenous vancomycin does not provide good penetration to the gut and is
never recommended for C difficile. Fidaxomycin, a newer and expensive medication, may be just as effective
for severe disease as vancomycin, but there is no evidence that it is more effective. On the basis of good
evidence that fidaxomicin may be better at preventing relapse than oral vancomycin, it is recommended in
the setting of frequently recurring disease.
When C difficile becomes fulminant, a subtotal colectomy may be required. A diverting loop ileostomy can
provide a mechanism for antegrade colonic enemas with vancomycin and allow preservation of the colon.
There are also reports of successful use of urgent fecal transplants to treat fulminant disease; however, this
is only sparsely available for the pediatric population.
Suggested Reading(s)
Adelman MW, Woodworth MH, Shaffer VO, Martin GS, Kraft CS. Critical care management of the
patient with Clostridium difficile. Crit Care Med. 2021;49(1):127-139.
doi:10.1097/CCM.0000000000004739
Edwards PT, Thurm CW, Hall M, et al. Clostridioides difficile infection in hospitalized pediatric patients:
comparisons of epidemiology, testing, and treatment from 2013 to 2019. J Pediatr. 2023;252:111-
116.e1. doi:10.1016/j.jpeds.2022.08.030
Hanson RM, Wagner AJ. A novel surgical treatment for pediatric fulminant Clostridium difficile colitis.
Pediatrics. Published online September 30, 2021. doi:10.1542/peds.2021-051365
Miranda-Katz M, Parmar D, Dang R, Alabaster A, Greenhow TL. Epidemiology and risk factors for
community associated Clostridioides difficile in children. J Pediatr. 2020;221:99-106.
doi:10.1016/j.jpeds.2020.02.005
Wolf J, Kalocsai K, Fortuny C, et al. Safety and efficacy of fidaxomicin and vancomycin in children and
adolescents with Clostridioides (Clostridium) difficile infection: a phase 3 multicenter, randomized,
single-blind clinical trial (SUNSHINE). Clin Infect Dis. 2020;71(10):2581-2588. doi:10.1093/cid/ciz1149
Content Domain
Infectious Diseases
A 20-month-old child is seen in the office for follow-up after a consultation visit with an otolaryngologist. The
girl was referred to the otolaryngologist after her fourth lifetime occurrence of acute otitis media; these
episodes have all occurred during the last 12 months, including 2 within the past 6 months. The
otolaryngologist has recommended placement of pressure equalization tubes. The parents would like advice
regarding the risks and benefits of the procedure.
Of the following, the MOST likely outcome of this procedure is (a)
Correct answer is C
PREP Pearl(s)
Pressure equalization tubes improve hearing for 1 to 3 months following insertion.
Pressure equalization tube insertion does not improve language acquisition in otherwise
developmentally typical children.
Postinsertion otorrhea is the most common complication of pressure equalization tube insertion.
Persistent perforation of the tympanic membrane is rare.
Critique
The most accurate statement regarding the outcome of tympanic membrane pressure equalization tube
(PET) insertion is improvement in hearing for 1 to 3 months after the procedure. Myringotomy with PET
insertion is the most common outpatient surgical procedure performed in children in the US. In 2016,
approximately 667,000 children younger than 15 years underwent this procedure.
The American Academy of Otolaryngology practice guideline recommends that PET insertion should be
offered to children with bilateral otitis media with effusion present for 3 months or longer with documented
hearing loss. The American Academy of Pediatrics clinical practice guideline for the diagnosis and
management of acute otitis media (AOM) states that children with recurrent AOM, defined as 3 episodes in 6
months or 4 episodes in a year with at least 1 episode occurring in the past 6 months, may be offered PET
insertion. The child in the vignette meets these criteria.
Many studies have sought evidence in support of specific benefits of this procedure. Evidence has not
supported the hypothesis that PET insertion will result in a decreased incidence of AOM, decreased middle
ear effusions and therefore improved hearing, improved speech acquisition, and less time sick with AOM. In
otherwise developmentally typical children, studies have found no significant improvement in speech
acquisition for either preschool or young school-age children with PET placement compared with watchful
waiting. Improved hearing has only been demonstrated for 1 to 3 months following the procedure; studies
failed to show improvement in hearing 12 to 24 months following PET placement compared with hearing in
children who did not have PET placement. However, PET placement or myringotomy paired with
adenoidectomy showed a more robust improvement in hearing 12 to 24 months after the procedure.
The complications of PET placement are minimal, with the most common being postinsertion otorrhea.
Persistent tympanic membrane perforation following PET extrusion occurs in approximately 3% of cases.
Factors that increase the likelihood of persistent perforation include young age, long-term tube type (as
opposed to short-term type), higher number of PET placements, and recurrent AOM as the indication for
tube placement.
Suggested Reading(s)
Hoberman A, Preciado D, Paradise JL, et al. Tympanostomy tubes or medical management for
recurrent acute otitis media. N Engl J Med. 2021;384(19):1789-1799. doi:10.1056/NEJMoa2027278
Paradise JL, Feldman HM, Campbell TF, et al. Effect of early or delayed insertion of tympanostomy
tubes for persistent otitis media on developmental outcomes at the age of three years. N Engl J Med.
2001;344(16):1179-1187. doi:10.1056/NEJM200104193441601
Rovers MM, Straatman H, Ingels K, van der Wilt G-J, van den Broek P, Zielhuis GA. The effect of
ventilation tubes on language development in infants with otitis media with effusion: a randomized
trial. Pediatrics. 2000;106(3):e42. doi:10.1542/peds.106.3.e42
Steele DW, Adam GP, Di M, Halladay CH, Balk EM, Trikalinos TA. Effectiveness of tympanostomy tubes
for otitis media: a meta-analysis. Pediatrics. 2017;139(6):e20170125. doi:10.1542/peds.2017-0125
Content Domain
Infectious Diseases
The correct answer is: improved hearing for 1 to 3 months after the procedure
A 16-year-old boy is seen in the office for evaluation of fever and headache of 1 week’s duration. His
symptoms started 9 days after returning from a mission trip to Ghana, West Africa. He did not receive
antimalarial chemoprophylaxis during the trip. On physical examination, he appears tired. His vital signs are
a temperature of 38 °C, a heart rate of 90 beats/min, a respiratory rate of 25 breaths/min, and an oxygen
saturation of 100% in room air.
The remainder of his physical examination findings are unremarkable.
Neutrophils 49%
Lymphocytes 28%
Monocytes 17%
Eosinophils 6%
B. atovaquone/azithromycin
C. atovaquone/proguanil
D. primaquine phosphate
Correct answer is A
PREP Pearl(s)
Fever in a returning traveler should raise suspicion for malaria. Malaria is endemic in Africa, the Indian
subcontinent, Southeast Asia, and Central and South America.
The most prevalent species of malaria by geographic region are Plasmodium falciparum in Africa,
Plasmodium vivax in the Indian subcontinent and Central America, Plasmodium ovale in West Africa,
and Plasmodium knowlesi in southeast Asia. Plasmodium malariae has a wide distribution.
The first-line treatment for uncomplicated chloroquine-resistant malaria is artemether/lumefantrine.
The second-line option is atovaquone/proguanil. The first-line option for severe malaria is intravenous
artesunate.
Critique
The adolescent boy in the vignette has fever and 3% parasitemia on a thick blood smear after returning from
Ghana, West Africa, a malaria endemic region. Malaria is the most likely diagnosis.
Empiric treatment of malaria is based on the type of Plasmodium species, drug resistance patterns, and
clinical status. Plasmodium falciparum is the predominant species in West Africa, with high activity
throughout the year. In West Africa, Plasmodium falciparum is chloroquine resistant. This boy has
uncomplicated malaria, with 3% parasitemia (complicated is ≥5%), and lack of other symptoms and signs
other than fever and headache. The first-line treatment in this case is oral artemether/lumefantrine. It has a
favorable safety profile, is effective against all stages of Plasmodium, and is cleared rapidly from the body.
Atovaquone/proguanil is an alternate treatment option for P falciparum species that are chloroquine-
resistant (or have an unknown resistance pattern), if oral artemether/lumefantrine is unavailable. The first-
line treatment for severe malaria is intravenous artesunate.
Primaquine phosphate is indicated as an adjunctive treatment for Plasmodium vivax and Plasmodium ovale
infection to eliminate the liver stage (hypnozoite). A quantitative glucose-6-phosphate dehydrogenase test to
confirm normal enzyme activity should be performed before primaquine is prescribed.
Atovaquone/azithromycin is the treatment for Babesia species, a protozoan that infects red blood cells.
Babesia is a tick-borne disease endemic to the northeastern and upper midwestern United States.
Fever in a returning traveler requires evaluation according to travel geography, exposure, and use of
chemoprophylaxis. Malaria (77%) and typhoid fever (18%) are the most common acute, potentially life-
threatening infections seen in Western travelers.
The malaria parasite, Plasmodium, is transmitted through the bite of the female Anopheles mosquito. The 5
Plasmodium species responsible for malaria worldwide are the following:
P falciparum (severe malaria)
P malariae (chronic)
Symptoms of malaria are nonspecific; a high index of suspicion is required for symptomatic individuals
returning from endemic regions. The most common symptoms are paroxysms of fever, chills, headache,
malaise, myalgias, arthralgias (eg, back pain), gastrointestinal symptoms (eg, nausea, vomiting, diarrhea), and
respiratory symptoms (eg, cough) lasting for 10 to 12 days. The classic pattern of fever occurring every 2 days
(for P falciparum, P vivax, and P ovale) and every 3 days (for P malariae) is rarely observed. The signs and
symptoms of malaria are caused by lysis of red blood cells with release of toxic substances (eg, hemozoin,
glucose phosphate isomerase).
To meet criteria for the diagnosis of complicated malaria, an individual must have ≥ 5% parasitemia, alone or
in combination with any of the following:
Impaired consciousness/coma (cerebral malaria)
Severe anemia [hemoglobin <7.0 g/dL (70 g/L)]
Acute kidney injury
Acute respiratory distress syndrome
Circulatory collapse or shock
Disseminated intravascular coagulation
Acidosis
Hypoglycemia
Jaundice
The clinical presentation of malaria may differ by Plasmodium species. P falciparum is associated with severe
malaria, including cerebral malaria, which can be fatal if untreated. P vivax and P ovale may cause a
persistent hepatic (hypnozoite) stage of infection that can result in relapsing disease. Pfalciparum and P
malariae can persist as a low-density parasitemia with recurrence of symptoms (recrudescence).
The diagnosis of malaria requires the following:
Microscopic blood smear examination (thick and thin smear), and/or
Rapid diagnostic testing (immunochromatography)
A thick blood smear preparation is a sensitive test, but it cannot differentiate among Plasmodium species.
The accuracy of the blood smear reading depends on the skill of the microscopist. Once the organism is
detected, parasite density is estimated on the basis of the percentage of infected red blood cells. If the initial
test result is negative, 3 additional smears should be checked (one every 12 to 24 hours). Polymerase chain
reaction testing is available through reference laboratories; this is helpful for confirmation, but treatment
should not be delayed pending the results.
Malaria is preventable with mosquito avoidance (eg, avoiding being outside at dawn and dusk, wearing long-
sleeved shirts and full-length trousers, using mosquito repellent), and taking chemoprophylaxis (eg,
atovaquone/proguanil, mefloquine, doxycycline).
Suggested Reading(s)
Abdel-Haq N, Chearskul P, Yasen Rafee Y, Asmar BI. Parasitic infections. In: McInerny TK, Adam HM,
Campbell DE, Foy JM, Kamat DM, eds. American Academy of Pediatrics Textbook of Pediatric Care. 2nd
ed. American Academy of Pediatrics; 2016:chap 308. Accessed September 1, 2023. Pediatric Care
Online
American Academy of Pediatrics. Malaria. In: Kimberlin DW, Barnett ED, Lynfield R, Sawyer MH, eds.
Red Book: 2021-2024 Report of the Committee on Infectious Diseases. 32nd ed. American Academy of
Pediatrics; 2021:493-503. Accessed September 1, 2023. Red Book Online
Rajagopalan A, Ilboudo CM. Malaria. Pediatr Rev. 2019;40(3):151-153. doi:10.1542/pir.2017-0225
Content Domain
Infectious Diseases
Courtesy of A. Noor
AAP PREP 2024 - Question 126/267 Infectious Diseases Question 24/30
A 4-month-old boy is seen in the office for 2 days of fever and decreased feeding. He seems fussier than
usual, is taking 3 oz of formula per feeding rather than his usual 6 oz, and had 3 loose stools yesterday. He
has had no vomiting and had 6 wet diapers in the past 24 hours. His parents have treated the fever with
acetominophen; the last dose was given more than 6 hours ago. In the office, the boy is alert and active. His
temperature is 39.2°C. He is an uncircumcised male with no significant findings on physical examination.
After cleaning the perineum, a bag urine specimen is obtained with the following dipstick results:
pH 6.5
Blood Trace
Ketones Negative
Nitrites Positive
Leukocytes 1+
Protein Negative
Glucose Negative
D. treat with oral cephalexin and order renal and bladder ultrasonography
Correct answer is B
PREP Pearl(s)
Escherichia coli remains the most common cause of urinary tract infection in infants and children.
Risk factors, such as age, temperature, length of fever, and, for boys, circumcision status, should be
considered by the pediatrician when making a decision about evaluating a patient for a urinary tract
infection.
Voiding cystourethrography should not be routinely performed after the first febrile urinary tract
infection.
Critique
For the infant in the vignette, who has leukocytes and nitrites noted on dipstick analysis of the bag urine
specimen, the best next step is to obtain a culture via urethral catheterization or suprapubic aspiration. This
culture should be performed before treatment is implemented and imaging studies obtained. Because the
infant appears well, hospital admission for intravenous antibiotic treatment is not indicated. It is reasonable
to treat him with an oral antibiotic with close outpatient monitoring for improvement in symptoms. Renal and
bladder ultrasonography is currently recommended for children younger than 24 months after their first
febrile urinary tract infection (UTI) but would not be performed before obtaining an appropriate urine culture
to document a true UTI. Voiding cystourethrography should not routinely be performed on otherwise healthy
children after their first UTI. In a febrile infant, in whom the chance of renal involvement is high, a third-
generation cephalosporin (eg, cefdinir) would be an appropriate antibiotic choice. For an afebrile infant, the
use of a first-generation cephalosporin (eg, cephalexin) would be appropriate.
Urinary tract infection is one of the most common causes of bacterial infection in children. Approximately 7%
of infants and children younger than 24 months with fever will have a UTI. Children older than 24 months
with UTI are often able to communicate to caregivers that they have urinary symptoms (eg, painful urination,
suprapubic discomfort, urinary frequency, and urinary hesitancy). For children in this age group who have
urinary symptoms, the overall risk of a urinary tract infection is approximately 8%. Bacteria associated with
the gastrointestinal tract are the most common pathogens associated with UTI in children: Escheria coli
(54%-67%), Klebsiella (6%-7%), Proteus (5%-12%), Enterococcus (3%-9%), and Pseudomonas (2%-6%).
Infants and children aged 2 to 24 months who have a UTI often have fever but no other symptoms or physical
examination findings. The probability of UTI in infants is determined by the presence or absence of 1 or more
of several risk factors (Table); practitioners should consider these when deciding whether to obtain a urine
sample. For infants younger than 2 months, UTI should be considered for every febrile patient, even those
with symptoms consistent with bronchiolitis; studies have shown the risk of concurrent UTI in infants with
respiratory syncytial virus to be 3% to 7%.
The method of obtaining the urine specimen is critical in the diagnosis of a UTI. For infants and children who
are not toilet trained, urethral catheterization and suprapubic aspiration are the gold standards for obtaining
an uncontaminated specimen for urinalysis and culture. The disadvantage of these procedures is that they
are painful and uncomfortable for the child. An alternative method is a multistep approach: (1) a urine bag is
applied to a clean perineum to collect urine, which is analyzed by urine dipstick; (2) if the results of the urine
dipstick analysis are normal, no further testing is needed; and (3) If the results of the urine dipstick analysis
show leukocytes and/or nitrites, a urethral catheterization or suprapubic aspiration should be performed to
obtain a specimen for culture; antibiotic therapy can be started while awaiting results. A bag specimen should
not be sent for urine culture due to the high likelihood of contamination.
For toilet-trained children, the preferred urine collection method is a midstream clean catch. When using this
technique, it is important to determine whether a sample is adequate by noting the presence of squamous
epithelial cells. If urinalysis with microscopy demonstrates 5 or more epithelial cells per high-power field, the
clinician should obtain another urine sample, if clinically indicated. Newer bladder stimulation techniques,
such as the Quick-Wee method (Kaufman 2017), may provide a noninvasive method to obtain a clean catch
urine sample for children who are not toilet trained.
Suggested Reading(s)
Balighian E, Burke M. Urinary tract infections in children. Pediatr Rev. 2018;39(1):3-10.
doi:10.1542/pir.2017-0007
Kaufman J, Fitzpatrick P, Tosif F, et al. Faster clean catch urine collection (Quick-Wee method) from
infants: randomised controlled trial. BMJ. 2017;357:j1341. doi:10.1136/bmj.j1341
Kaufman J. Temple-Smith M, Sanci L. Urinary tract infection in children: an overview of diagnosis and
management. BMJ Paediatr Open. 2019;3(1):e000487. doi:10.1136/bmjpo-2019-000487
Mattoo TK, Shaikh N, Nelson, C. Contemporary management of urinary tract infection in children.
Pediatrics. 2021;147(2):1-12. doi:10.1542/peds.2020-012138
Uwaezuoke S. Ayuk A. Muoneke U. Urinary tract infection in children: a review of the established
practice guidelines. EMJ Microbiol Infect Dis. 2020;1(1):57-65. doi:10.33590/emjmicrobiolinfectdis/20-
00001
Content Domain
Infectious Diseases
A 9-month-old is undergoing evaluation for a new-onset rash. He was evaluated in the emergency
department 3 days ago for a simple febrile seizure. He was receiving amoxicillin for an otitis media when the
seizure occurred. He has had no fever today. In the office, he appears well. His temperature is 37.5 °C, his
blood pressure is 95/70 mm Hg, his heart rate is 90 beats/min, his respiratory rate is 25 breaths/min, and his
oxygen saturation is 100% in room air. His physical examination findings are significant only for a
maculopapular rash over the trunk and extremities (Figure).
Courtesy of A. Noor
Neutrophils 50%
Lymphocytes 40%
Monocytes 10%
Cerebrospinal Fluid
Of the following, the MOST likely positive finding on the boy’s meningitis/encephalitis polymerase chain
reaction panel is
A. enterovirus
C. human herpesvirus 6
D. parechovirus
Correct answer is C
PREP Pearl(s)
Human herpesvirus 6 is ubiquitous; almost all children are infected by age 2 years. The peak incidence
is in infants aged 6 to 9 months.
In healthy infants and young children, human herpesvirus 6 infection is benign and self-limiting. The
spectrum of clinical manifestation includes nonspecific febrile illness, roseola, febrile seizures, and
rarely meningitis-encephalitis. Viral relapse in immunocompromised children can result in severe
disease.
Classically, roseola presents with high-grade fever lasting 3 to 5 days, followed by an abrupt cessation
of fever with simultaneous appearance of a maculopapular rash.
Critique
The meningitis/encephalitis polymerase chain reaction panel is most likely to detect human herpesvirus 6
(HHV-6) in the boy described in the vignette. He had a simple febrile seizure and later appearance of a
maculopapular rash, consistent with a primary human herpesvirus 6 (HHV-6) infection, also known as
roseola. About 15% of children between ages 6 and 18 months who have primary HHV-6 B infection develop
febrile seizures. As seen in the boy in the vignette, findings of cerebrospinal fluid analysis in a child with a
febrile seizure associated with a HHV-6 infection are usually normal. A lumbar puncture would not usually be
recommended in the assessment of a child with a simple febrile seizure, but it is recommended for a child
who is receiving antibiotics at the time of the seizure, because signs of meningitis could be masked.
Human herpesvirus 6 is an enveloped, double-stranded DNA virus that primarily infects lymphocytes. It is
closely related to the herpesviridae family and similarly establishes latency. Human herpesvirus 6 has 2
distinct subgroups. Variant A infection is often asymptomatic; variant B is responsible for roseola.
The HHV-6 virus is ubiquitous; almost all children are infected with it by age 2 years. The majority of primary
infections occur in infants aged 6 to 9 months, coinciding with the loss of protective maternal antibodies.
Human herpesvirus 6 causes a variety of clinical manifestations in infants, young children, and
immunocompromised children:
Febrile illness without rash: About 20% of nonspecific febrile illnesses presenting to the emergency
department are caused by HHV-6 primary infection. The characteristic presentation is an abrupt onset
of high-grade fever associated with irritability and nonspecific upper viral symptoms (eg, rhinorrhea).
Febrile illness with rash (roseola): About 25% of infants and young children develop this classic
presentation. It is characterized by an abrupt onset of high-grade fever (40 °C), lasting for 3 to 5 days,
followed by a sudden cessation of fever and appearance of a maculopapular rash. The rash initially
appears on the face and trunk and later spreads to the extremities. It lasts for hours to up to 2 days.
Febrile seizures: Primary HHV-6 infection is associated with febrile seizures in 10% to 40% of cases.
Rarely, HHV-6 has been associated with meningitis and encephalitis in otherwise healthy children.
Reactivation of HHV-6: After the primary infection, HHV-6 remains latent and can be detected in
various bodily fluids such as saliva, blood, and cerebrospinal fluid. In immunocompromised children
(eg, those with T-cell defects, those undergoing hematopoietic stem cell transplants), reactivation of
HHV-6 may result in severe disease. In the early phase after stem cell transplantation, HHV-6 has been
associated with fever, rash, encephalitis, pneumonitis, and hepatitis.
Laboratory confirmation of HHV-6 infection is seldom required, because it is generally a benign and self-
limiting condition. The meningitis/encephalitis multiplex polymerase chain reaction cerebrospinal fluid panels
approved by the US Food and Drug Administration include HHV-6. These panels detect HHV-6 in
approximately 7% of cases; however, this result should be interpreted with caution. A positive test could
indicate an acute primary infection, a previous infection, or a chromosomal integration. One percent of the
population carries a HHV-6 chromosomal integration in their germline, which is vertically transmitted to their
offspring. Chromosomally integrated HHV-6 can be diagnosed via these factors:
In healthy infants and young children, the management of HHV-6 infection is supportive. Antiviral therapy
may benefit immunocompromised children with severe disease. Ganciclovir or foscarnet is recommended to
treat HHV-6 encephalitis in children who have received hematopoietic stem cell or solid organ transplants.
The illness in the child in the vignette is much less likely to have been caused by an enterovirus, a
parechovirus, or a herpes simplex virus infection. The rash associated with enterovirus and parechovirus
infections can be maculopapular, vesicular, petechial, or purpuric; it is typically concurrent with the fever and
lasts a few days after the fever has disappeared. The rash associated with herpes simplex virus infection is
primarily vesicular.
Suggested Reading(s)
American Academy of Pediatrics. Human herpesvirus 6 (including roseola) and 7. In: Kimberlin DW,
Barnett ED, Lynfield R, Sawyer MH, eds. Red Book: 2021-2024 Report of the Committee on Infectious
Diseases. 32nd ed. American Academy of Pediatrics; 2021. Accessed September 1, 2023. Red Book
Online
Caserta MT. Human herpesviruses 6 and 7 (roseola, exanthem subitum). In: Long SS, Prober CG, Fisher
M, eds. Principles and Practice of Pediatric Infectious Diseases. 5th ed. Elsevier; 2018:1081-1088.
Watson JR, Mejias A. Human herpesvirus-6 and human herpesvirus-7 infections. In: McInerny TK, Adam
HM, Campbell DE, Foy JM, Kamat DM, eds. American Academy of Pediatrics Textbook of Pediatric Care.
2nd ed. American Academy of Pediatrics; 2016:chap 267. Accessed March 7, 2023. Pediatric Care
Online
Content Domain
Infectious Diseases
A 14-year-old girl, a recent immigrant, is seen in the emergency department after having a first-time seizure
on the school bus this morning. She has been a healthy child with normal findings on preimmigration and
postimmigration health screenings and up-to-date vaccinations. There is no known family history of seizures.
In the emergency department, she is fully awake. Her vital signs and physical examination findings (including
neurologic examination) are normal. Results of a complete blood cell count and basic metabolic profile are
normal. Head computed tomography (Figure 1) is performed. Subsequent brain magnetic resonance imaging
(Figure 2) establishes the diagnosis.
Reprinted with permission from Cafferty R, Howard C, Kaila R. Pediatr Rev. 2019;40(3):139.
Figure 1. Head computed tomography findings for the girl in the vignette.
Reprinted with permission from Cafferty R, Howard C, Kaila R. Pediatr Rev. 2019;40(3):139
Figure 2. Brain magnetic resonance imaging findings for the girl in the vignette.
Of the following, this child’s emigration location and history MOST likely include
B. Kazakhstan, where several family members were treated for pulmonary tuberculosis
Correct answer is D
PREP Pearl(s)
Neurocysticercosis is a common cause of epilepsy worldwide.
Imaging (computed tomography and magnetic resonance imaging) is the most important diagnostic
testing used to diagnose neurocysticercosis.
Critique
The child in the vignette has neurocysticercosis (NCC), as evidenced by the calcified lesion in her right
occipital lobe (Figure 1) as well as the active inflammatory lesion in her left frontal lobe (Figure 2). The World
Health Organization estimates that approximately 30% of epilepsy cases in low-resource countries are due to
NCC. Neurocysticercosis is caused by the ingestion of Taenia solium (pork tapeworm) eggs, which are shed in
the stool of tapeworm-infected humans. Ingested eggs can travel to the muscles and brain, where they form
cysts (cysticerci and neurocysticerci). Infection with the tapeworm itself (taeniasis) comes from eating
undercooked pork in which larvae have encysted. Free-roaming pigs (a common rearing system in many
parts of the world, including Uganda) that have access to human feces are the primary reservoir of T solium.
Reprinted with permission from Cafferty R, Howard C, Kaila R. Pediatr Rev. 2019;40(3):139
Reprinted with permission from Cafferty R, Howard C, Kaila R. Pediatr Rev. 2019;40(3):139
Figure 2. Left frontal lobe findings on magnetic resonance imaging of the girl in the vignette.
Ingestion of raw beef (eg, steak tartare, a dish eaten in France) would put the child at increased risk for
toxoplasmosis but not NCC. Although a child exposed to family members with pulmonary tuberculosis may
develop tuberculous meningitis, this would not present as isolated seizures in an otherwise healthy child. A
high rate of open defecation greatly increases the risk of typhoid fever and other diarrheal illness. However,
in a majority muslim country (eg, Pakistan), pigs are not generally kept for food. Without the presence of pigs,
there would not be a high risk of NCC.
Management of NCC consists of long-term treatment with antiepileptic drugs, as well as short-term
treatment with anthelmintics in conjunction with steroids (to prevent exacerbation of inflammation while
parasites are dying). Less commonly, neurocysticerci may be located in areas of the brain that induce
headache or neurologic findings other than seizures, and surgery, ventricular shunting, or other interventions
may be required. The diagnosis of neurocysticercosis can be confirmed with serologic testing, but such
testing is not highly sensitive; children presenting with seizures and imaging findings suggestive of NCC are
often treated empirically when there is a history of likely exposure.
Suggested Reading(s)
Abdel-Haq N, Chearskul P, Rafee Y, Asmar BI. Parasitic infections. In: McInerny TK, Adam HM, Campbell
DE, DeWitt TG, Foy JM, Kamat DM, eds. American Academy of Pediatrics Textbook of Pediatric Care.
2nd ed. American Academy of Pediatrics; 2017:chap 308. Pediatric Care Online
Cafferty R, Howard C, Kaila R. Case 1: new-onset seizures in a 16-year-old girl recently emigrated from
African. Pediatr Rev. 2019;40(3):138-141. doi:10.1542/pir.2017-0252
de Oliveira RS, Viana DC, Colli BO, Rajshekhar V, Salomão JF. Pediatric neurocysticercosis. Childs Nerv
Syst. 2018;34(10):1957-1965. doi:10.1007/s00381-018-3889-4
Garcia HH, Gonzalez AE, Gilman RH. Taenia solium cysticercosis and its impact in neurological disease.
Clin Microbiol Rev. 2020;33(3):e00085-19. doi:10.1128/CMR.00085-19
White AC, Coyle CM, Rajshekhar V, et al. Diagnosis and treatment of neurocysticercosis: 2017 clinical
practices guidelines by the Infectious Diseases Society of America (IDSA) and the American Society of
Tropical Medicine and Hygiene (ASTMH). Clin Infect Dis. 2018;66(8):e49-e75. doi:10.1093/cid/cix1084
Content Domain
Infectious Diseases
A 3-year-old with short bowel syndrome is brought to the emergency department for evaluation of 1 day of
fever. The boy requires total parenteral nutrition delivered through a single-lumen central venous catheter.
His vital signs are a temperature of 38.8 °C, a heart rate of 120 beats/min, a respiratory rate of 25
breaths/min, and an oxygen saturation of 100% in room air.He appears well. His physical examination
findings are remarkable only for copious rhinorrhea.
Laboratory results are shown:
Test Result
Neutrophils 79%
Lymphocytes 15%
Monocytes 4%
Eosinophils 2%
Correct answer is D
PREP Pearl(s)
Coagulase-negative staphylococci are biofilm-producing, low-virulence organisms responsible for most
central line–associated bloodstream infections.
A single blood culture isolation of coagulase-negative Staphylococcus, particularly in an otherwise
healthy child, suggests contamination. Repeated growth of the same species on blood culture
represents a true infection, particularly in children with indwelling medical devices or
immunosuppression.
Intravenous vancomycin is the recommended empiric treatment for coagulase-negative
Staphylococcus, to cover for both methicillin-resistant and sensitive species while awaiting antibiotic
susceptibilities.
Critique
The child in the vignette has a fever and findings suggestive of a viral infection (rhinorrhea, detection of
rhinovirus/enterovirus). However, the presence of a central venous catheter and growth of coagulase-
negative, gram-positive cocci in clusters on the blood culture raises concern regarding a central line-
associated bloodstream infection (CLABSI). It necessitates a repeat blood culture and empiric antibiotic
therapy. Vancomycin is an appropriate antibiotic to treat this child for CLABSI from a coagulase-negative
Staphylococcus species (CoNS). Central venous catheter removal is not necessary for a child with CoNS
CLABSI who is hemodynamically stable.
Intravascular central line catheters are prone to infection from biofilm-producing bacteria. Staphylococcus
epidermidis, a coagulase-negative Staphylococcus species, is the most common cause (70%) of CLABSI.
Intravenous vancomycin treats both methicillin-sensitive and methicillin-resistant S epidermidis.
Coagulase-negative Staphylococcus species are frequently isolated on blood culture. Staphylococcus
epidermidis infections are often indolent (low virulence), and differentiation of a true infection from blood
culture contamination can be challenging. In an otherwise healthy child, a blood culture positive for a CoNS
species typically represents contamination due to the introduction of the organism from the skin into the
blood culture bottle during venipuncture. In a child with an indwelling medical device, a single CoNS isolation
typically suggests contamination, whereas repeated positive results represent a true infection. It is
recommended to obtain equal-volume blood cultures from the central line and a peripheral vein around the
same time. In a true central venous line infection, the blood culture from the catheter becomes positive 2 or
more hours before the peripheral culture. A quantitative blood culture drawn from the catheter growing ≥5-
fold colony forming units compared to the peripheral culture is also indicative of a CLABSI.
Coagulase-negative Staphylococcus species are a common inhabitant of human skin. These organisms can be
introduced into the catheter during insertion or manipulation. The duration of catheter placement is directly
related to the risk of infection. Coagulase-negative Staphylococcus species produce a proteinaceous slime
(capsular polysaccharide) and adhesins that enhance adherence to the catheter surface. This biofilm protects
the organism from host defenses and antimicrobial agents. Other frequently isolated CoNS include
Staphylococcus capitis, Staphylococcus hominis, Staphylococcus warneri, and Staphylococcus haemolyticus.
Coagulase-negative Staphylococcus species are less virulent than other organisms that cause catheter-
associated line infections, such as Staphylococcus aureus and gram-negative organisms (eg,
Enterobacteriaceae, Pseudomonas). Infection with CoNS typically presents with fevers alone or with
inflammation at the catheter exit site.
Management of CLABSI caused by CoNS depends on the clinical status of the child, immunocompetence,
presence of exit site infection, and vascular access.
If there is a growth of CoNS on ≥2 blood cultures, systemic antibiotics (eg, vancomycin) to treat a
CLABSI should be administered. Intravenous vancomycin is the antibiotic of choice. Alternate options
include ceftaroline, daptomycin, and linezolid. If the results of repeat blood cultures become negative,
systemic antibiotics should be continued for a total of 10 to 14 days.
Catheter salvage can be attempted when persistent bacterial growth occurs in a hemodynamically
stable child with a combination of antibiotic lock and systemic therapy. In antibiotic lock therapy, an
antibiotic solution is instilled and allowed to dwell in the catheter for a time to maximize exposure to
surface bacteria.
Catheter removal should be considered if a child is hemodynamically unstable, has clinical
deterioration, or has persistent bacterial growth despite adequate antibiotic treatment. Catheter
removal is recommended for organisms other than CoNS, for an antibiotic-susceptible
Enterobacteriaceae, or when there is evidence of infection of an implanted device's tunnel or
subcutaneous pocket (inflammation of the skin at the exit site).
Suggested Reading(s)
American Academy of Pediatrics. Coagulase-negative staphylococcal infections. In: Kimberlin DW,
Barnett ED, Lynfield R, Sawyer MH, eds. Red Book: 2021-2024 Report of the Committee on Infectious
Diseases. 32nd ed. American Academy of Pediatrics; 2021. September 1, 2023. Red Book Online
Ford WJH, Bundy DG, Oyeku S, et al. Central venous catheter salvage in ambulatory central line–
associated bloodstream infections. Pediatrics. 2021;148(6):e2020042069. doi:10.1542/peds.2020-
042069
Sammons JS, Gerber JS. Clinical syndromes of device-associated infections. In: Long SS, Prober CG,
Fischer M, eds. Principles and Practice of Pediatric Infectious Diseases. 5th ed. Elsevier; 2018:600-612.
Content Domain
Infectious diseases
A healthy 6-month-old infant is undergoing evaluation of 1 day of nonbilious, nonbloody vomiting followed
by nonbloody diarrhea for the past 2 days. She had a fever of 38.5 °C on the first day of illness that
subsequently resolved. The infant continues to have good intake and urine output. She received her routine
6-month vaccines, including the rotavirus vaccine, 1 week before her symptom onset. She attends day care
but has otherwise had no infectious exposures, including travel, undercooked foods, or untreated water
sources. The infant has been growing appropriately. Her vital signs are within normal limits, and she appears
well-hydrated and playful. Her physical examination findings are unremarkable.
Of the following, the BEST next step in this infant’s care is to obtain
A. a blood culture
Correct answer is D
PREP Pearl(s)
In the absence of concerning signs and symptoms, a child with a clinical presentation consistent with
viral gastroenteritis need not undergo further diagnostic testing.
Rotavirus can be detected in the stool for at least 10 days after vaccination.
Critique
The infant in the vignette most likely has viral gastroenteritis. She is well hydrated and her fever has resolved,
so no testing is indicated at this time. If she was febrile, it would be appropriate to obtain a urinalysis and
urine culture to assess for a urinary tract infection. If she was still febrile and was moderately or severely ill,
requiring hospitalization, a blood culture would be appropriate. Concerning symptoms such as bloody
diarrhea, prolonged diarrhea, severe dehydration, or a known exposure to a bacterial intestinal pathogen (eg,
Salmonella) should prompt additional testing, including stool testing for bacterial and/or parasitic pathogens.
Viral gastroenteritis is common in infants and children. Before the introduction of the vaccine, rotavirus was
the most common cause of pediatric, community-acquired gastroenteritis. Rotavirus usually presents with
vomiting for 1 to 2 days, followed by diarrhea. High fevers occur in up to one-third of affected children. The
entire illness usually lasts 3 to 7 days. Some children experience profound dehydration, especially those who
have immunodeficiencies. Rarely, seizures may occur.
Rotavirus is transmitted via the fecal-oral route. The virus can survive for months on fomites. Rarely, it can be
passed through contaminated food and water. In the United States, infections are most commonly seen in
odd years in the late winter and early spring.
Testing for rotavirus in a fully vaccinated child is of low yield. Additionally, the virus can be detected for at
least 10 days after receiving the vaccine, and this child received a rotavirus vaccine 1 week before symptom
onset. Immunoassays and polymerase chain reaction tests may detect rotavirus, but also may detect other
viruses that may or may not be causing symptoms.
Suggested Reading(s)
American Academy of Pediatrics. Rotavirus infections. In: Kimberlin DW, Barnett ED, Lynfield R, Sawyer
MH, eds. Red Book: 2021–2024 Report of the Committee on Infectious Diseases. 32nd ed. American
Academy of Pediatrics; 2021:chap 347 Accessed November 14, 2023.
Cortese MM, Haber P. Rotavirus. In: Hall E, Wodi AP, Hamborsky J, Morelli V, Schillie S, eds.
Epidemiology and Prevention of Vaccine-Preventable Diseases. 14th ed. Public Health Foundation;
2021:chap 19. www.cdc.gov/vaccines/pubs/pinkbook/rota.html
Content Domain
Infectious Diseases
A 5-year-old, fully immunized child is seen in the office for 6 days of green nasal discharge and 2 days of
swelling of the left eye. He has a history of pollen allergy. There is no history of trauma, insect bite, or
previous skin infection. The boy does not appear to be toxic. His temperature is 38 °C, his heart rate is 90
beats/min, his respiratory rate is 20 breaths/min, and his oxygen saturation is 100% in room air. His left
upper and lower eyelids are edematous and erythematous (Figure). His conjunctivae are clear, without
discharge, and he can move his eyes without pain. He has left facial swelling and mucopurulent nasal
discharge. The remainder of his physical examination findings are unremarkable.
Courtesy of A. Noor
A. amoxicillin-clavulanate
B. diphenhydramine
C. loratadine
D. trimethoprim-sulfamethoxazole
Correct answer is A
PREP Pearl(s)
Preseptal (periorbital) and orbital cellulitis are bacterial infections of the eyelid and orbit, respectively.
Preseptal cellulitis occurs anterior to, and orbital cellulitis occurs posterior to, the orbital septum.
Features common to both preseptal and orbital cellulitis include unilateral erythematous and tender
eyelid swelling with systemic signs of infection (eg, fever). Impairment of extraocular eye movements,
proptosis, chemosis, and blurred vision occur with orbital cellulitis.
The first-line antibiotic choice to treat preseptal cellulitis in the setting of sinusitis in a well-appearing
child who is able to tolerate oral intake is amoxicillin-clavulanate.
Critique
The child in the vignette has preseptal (periorbital) cellulitis associated with sinusitis. His unilateral eyelid
swelling, erythema, and fever suggest an eyelid infection. His mucopurulent nasal discharge and left facial
swelling are signs of acute bacterial sinusitis. The child's intact extraocular movements, nontoxic appearance,
low-grade fever, and normal vital signs make orbital cellulitis an unlikely diagnosis.
Most cases of preseptal cellulitis in the setting of sinusitis are caused by typical sinus flora (eg, Streptococcus
pneumoniae, nontypeable Haemophilus influenzae, Moraxella). Of the response choices, amoxicillin-
clavulanate provides the best coverage for these organisms and is therefore the preferred treatment option
for this child.
Preseptal cellulitis in the setting of trauma or an insect bite is typically caused by Staphylococcus aureus.
Trimethoprim-sulfamethoxazole would be an appropriate treatment for a S aureus infection; however, the
boy has no such history.
An allergic reaction can result in eyelid swelling. However, an allergic reaction typically involves both eyes and
is associated with pruritus and serous (not mucopurulent) nasal discharge. The boy’s fever also makes an
allergic reaction an unlikely diagnosis. Diphenhydramine (first-generation H1 blocker) and loratadine (second-
generation H1 blocker) would be appropriate treatments for allergic eyelid swelling.
Preseptal and orbital cellulitis are bacterial infections of the eyelid and orbit, respectively. The orbital septum,
an extension of the periosteum, is a thin connective tissue membrane that serves as a physical barrier
separating the superficial eyelid tissue from the deep orbital structures. Infection anterior to the orbital
septum causes preseptal cellulitis. Infection posterior to the orbital septum causes orbital cellulitis, with or
without an orbital or subperiosteal abscess.
Preseptal cellulitis occurs 3 times more often than orbital cellulitis; it most commonly occurs in children
younger than age 5 years. Preseptal cellulitis occurs in the following settings:
Orbital cellulitis occurs in all age groups, with a median age of 6.8 years. Sinusitis, especially ethmoiditis, is
the most common predisposing factor. Orbital cellulitis signs and symptoms include the following:
Sudden onset of eye swelling and erythema in the setting of an upper respiratory tract infection
Systemic signs of infection (eg, fever)
Proptosis (eye globe displacement anteriorly and downward)
Chemosis (edema of the bulbar conjunctiva)
Impaired extraocular movements
Most cases of orbital cellulitis are associated with a subperiosteal abscess. In young children, the abscess
extends from ethmoiditis; in adolescents, it extends from frontal sinusitis. When orbital cellulitis is suspected,
computed tomography (CT) of the orbits and sinuses should be obtained to delineate the extent of infection
and enable evaluation for the presence of a drainable abscess collection. Magnetic resonance imaging should
be considered in a child with CT-confirmed orbital cellulitis who is suspected of having intracranial
complications (eg, abscess, subdural empyema, venous thrombosis).
A child with preseptal cellulitis in the setting of sinusitis, who appears well and is able to tolerate oral intake,
can be treated orally with an antibiotic active against β-lactamase–producing organisms (eg, amoxicillin-
clavulanate or a third-generation cephalosporin [eg, cefixime, cefpodoxime]). Children who are unable to
tolerate oral intake can be treated with intravenous ceftriaxone or ampicillin-sulbactam. The treatment
duration is 10 to 14 days.
A child with preseptal cellulitis in the setting of local skin trauma or an insect bite should be treated with an
oral antibiotic active against S aureus. Coverage for methicillin-resistant S aureus (eg, trimethoprim-
sulfamethoxazole or clindamycin) should be considered according to local prevalence and any history of
methicillin-resistant S aureus infection in the child. The treatment duration is 10 to 14 days.
Orbital cellulitis is treated with intravenous antibiotics. Empiric antibiotics should cover sinus flora (S
pneumoniae, nontypeable H influenzae, Moraxella), S aureus, Streptococcus pyogenes, and anaerobes
[Bacteroides, Prevotella, Fusobacterium]). A combination of ampicillin-sulbactam plus vancomycin or
ceftriaxone plus vancomycin is appropriate treatment for an immunocompetent child. Intravenous antibiotics
are continued until there is improvement in the eye findings, followed by a 2-week course of oral antibiotics.
Suggested Reading(s)
Baiu I, Melendez E. Periorbital and orbital cellulitis. JAMA. 2020;323(2):196.
doi:10.1001/jama.2019.18211
Wald ER. Preseptal and orbital cellulitis. In: McInerny TK, Adam HM, Campbell DE, Foy JM, Kamat DM,
eds. American Academy of Pediatrics Textbook of Pediatric Care. 2nd ed. American Academy of
Pediatrics; 2016:chap 319. Accessed March 8, 2023. Pediatric Care Online
Wald ER. Preseptal and orbital infections. In: Long SS, Prober CG, Fischer M, eds. Principles and Practice
of Pediatric Infectious Diseases. 5th ed. Elsevier; 2017:517-522.
Content Domain
Infectious Diseases
A 28-month-old infant, born preterm, is admitted to the hospital for fever and ill appearance. As a neonate,
she had necrotizing enterocolitis that required extensive bowel resection; she now receives most of her
nutrition intravenously by means of a single-lumen, tunneled central catheter. She was diagnosed with acute
suppurative otitis media 7 days ago and is at day 7 of a 10-day course of amoxicillin-clavulanic acid. Central
catheter and peripheral blood cultures are obtained at the time of admission; 12 hours later her central
catheter culture is growing Candida.
Of the following, the BEST next step for this girl’s management is to administer intravenous
A. amphotericin B deoxycholate
B. caspofungin
C. liposomal amphotericin B
D. voriconazole
Correct answer is B
PREP Pearl(s)
Candidal bloodstream infections associated with a central venous catheter cannot be effectively
treated with antifungal drugs alone; central catheter removal should occur as soon as is feasible.
Echinocandins (eg, caspofungin and micafungin) are recommended for empiric treatment of candidal
bloodstream infections.
Preterm neonates; children receiving chemotherapy, antibiotics, or parenteral nutrition; as well as
those with central venous catheters are at increased risk of invasive candidal infections.
Critique
The best next step in management of the child in the vignette, with a central catheter culture result positive
for Candida, is to administer caspofungin. Additionally, a request for a surgical consultation for prompt
catheter removal should be placed.
This child has several significant risk factors for candidemia, including a central venous catheter, parenteral
nutrition, and current antibiotic treatment. Other common risk factors include neutropenia, receipt of
chemotherapy, and prematurity.
The Infectious Disease Society of America recommends an echinocandin (eg, caspofungin or micafungin) as
the first-line empiric treatment of candidal bloodstream infections. Outside a few specific situations (eg,
treatment of neonatal urinary candidal infection), amphotericin deoxycholate is rarely recommended for
treatment in children. Liposomal amphotericin, a newer formulation, is less nephrotoxic than amphotericin
deoxycholate and has a better safety profile. Liposomal amphotericin could be used in this scenario;
however, when the infecting organism is known to be a Candida species, liposomal amphotericin B would
have no advantage over an echinocandin and carries more risk. Voriconazole could be considered for
treatment of a child in stable condition who is known to be infected with a likely susceptible Candida species
(such as Candida albicans) but would not be an appropriate first-line empiric treatment for this child.
Unlike many bacterial infections, which can be cleared from a central venous catheter with antibiotic
treatment, a candidal bloodstream infection should prompt central venous catheter removal as soon as
feasible. Attempts to clear Candida from a central venous catheter with medical treatment carry a high risk of
failure and a worsening clinical course.
Suggested Reading(s)
American Academy of Pediatrics. Candidiasis. In: Kimberlin DW, Barnett ED, Lynfield R, Sawyer MH. Red
Book: 2021-2024 Report of the Committee on Infectious Diseases. 32nd ed. American Academy of
Pediatrics; 2021:246-252. Pediatric Care Online
Åttman E, Syrjänen J, Lyytikäinen O, Ollgren J, Sinisalo M, Vuento R, Mattila E, Huttunen R. Healthcare-
associated blood stream infections in hematological patients in Finland during the years 2006-2016.
Eur J Haematol. 2021;107(3):311-317. doi:10.1111/ejh.13663
Khairat SM, Sayed AM, Nabih M, Soliman NS, Hassan YM. Prevalence of Candida blood stream
infections among children in tertiary care hospita: detection of species and antifungal susceptibility.
Infect Drug Resist. 2019;12:2409-2416. doi:10.2147/IDR.S196972
Pappas PG, Kauffman CA, Andes DR, et al. Clinical practice guidelines for the management of
candidiasis: 2016 update by the Infectious Disease Society of America. Clin Infect Dis. 2016;62(4):e1-
e50. doi:10.1093/cid/civ933
Zakhem AE, Istambouli R, Jabbour J-F, Hindy JR, Gharamti A, Kanj SS. Diagnosis and management of
invasive Candida infections in critically ill patients. Semin Respir Crit Care Med. 2022;43(1):46-59.
doi:10.1055/s-0041-1741009
Content Domain
Infectious Disease
A 3-year-old girl is brought to the emergency department by ambulance after suffering a cardiac arrest as a
result of a drowning accident in the backyard pool. The child was submersed for an unknown amount of time
and was pulseless when pulled from the water. A bystander started cardiopulmonary resuscitation (CPR)
immediately. When emergency medical services arrived 5 minutes later, bag-mask ventilation was initiated.
Attempts at vascular access were unsuccessful at the scene. In the emergency department, the child is
unresponsive, pulseless, and hypothermic. Active CPR is continued. The decision is made to place an
intraosseous needle in the proximal tibia.
Of the following, the complication MOST likely avoided by performing the procedure in the correct
anatomical location is
Correct answer is D
PREP Pearl(s)
Intraosseous access can be used in an emergency resuscitation as an alternative to intravenous
access.
Growth plate damage and stunted growth can be avoided by ensuring that the intraosseous needle
insertion avoids the epiphyseal plate.
Complications of intraosseous needle insertion include fracture, soft-tissue infection, osteomyelitis,
and fat embolism.
Critique
The girl in the vignette, who is in cardiac arrest, requires vascular access, and the decision is made to place an
intraosseous (IO) needle in her proximal tibia. To enhance the likelihood of successful IO placement with low
risk of complications, the needle should be placed in the diaphysis, 2 finger breadths (1-2 cm) below the tibial
tuberosity (Figure). More proximal placement, in the epiphysis, could damage the epiphyseal growth plate
and result in stunted bone growth. None of the other potential complication options depends on the location
of the IO needle.
Courtesy of J. Kane
Figure. Schematic of the anatomy of the tibia and correct location of intraosseous needle placement.
Vascular access during pediatric resuscitation is critical for an optimal outcome. The ideal venous access is a
short, large-bore catheter for rapid administration of fluids and medications. Obtaining timely, stable
vascular access is a high priority. National guidelines recommend limiting intravenous (IV) access attempts to
2, lasting no more than 120 seconds and, if unsuccessful, prioritizing IO access.
Stable vascular access can be difficult to obtain in children with poor perfusion, hypothermia, or active
cardiac arrest. In an emergency resuscitation, IO access can be used as an alternative to IV access,
particularly when IV access is difficult to obtain or attempts have failed. Intraosseous needle placement is a
safe and quick procedure allowing for the efficient administration of resuscitation fluids and medications.
Intraosseous access can be easily accomplished by trained providers with few complications. In most cases,
results of laboratory evaluation of blood drawn from an IO catheter are comparable with those of blood
obtained via venous draw.
There are several potential complications of IO access. If the needle is not fully inserted into the medullary
space, injection may not be successful or may require high pressure. This situation should be suspected if the
initial flush with saline is not easily performed. Extravasation of fluid occurs when the needle is inserted too
far into the bone and exits through the posterior cortex; this is independent of the IO placement site or
patient anatomy. Extravasation can also occur if an IO needle is placed in a fractured bone, or a bone that
was previously accessed with an IO needle. Extravasation of crystalloid fluids can result in compartment
syndrome, which could compromise the viability of the extremity. Epiphyseal plate necrosis may occur if the
IO insertion site is near the epiphyseal plate. Other potential complications include fracture, soft-tissue
infection, osteomyelitis, and fat embolism; none of these complications is specifically related to the location
of IO needle placement.
Suggested Reading(s)
Davis AL, Carcillo JA, Aneja RK, et al. The American College of Critical Care Medicine clinical practice
parameters for hemodynamic support of pediatric and neonatal septic shock: executive summary.
Pediatr Crit Care Med. 2017;18(9):884-890. doi:10.1097/PCC.0000000000001259
Fiorito BA, Mirza F, Doran TM, et al. Intraosseous access in the setting of pediatric critical care
transport. Pediatr Crit Care Med. 2005;6(1):50-53. doi:10.1097/01.PCC.0000149137.96577.A6
Kandil SB, Mahajan PV, Faustino EVS. Vascular access in critically ill children. Pediatrics. 2020;145(suppl
3):S296-S297. doi:10.1542/peds.2019-3474O
van der Jagt ÉW. Pediatric cardiopulmonary resuscitation. In: McInerny TK, Adam HM, Campbell DE, Foy
JM, Kamat DM, eds. American Academy of Pediatrics Textbook of Pediatric Care. 2nd ed. American
Academy of Pediatrics; 2016:Appendix A. Accessed September 1, 2023. Pediatric Care Online
Content Domain
Medical Procedures
At 9 AM, a 3-year-old girl fell down the front steps at home and landed on her right arm. She did not hit her
head, but she cried immediately. Her mother tried to comfort her and offered apple juice immediately after
the incident. The girl took some of the drink but continued to cry when her arm was moved, so her mother
brought her to the emergency department for evaluation at 11 AM.
On physical examination, the girl is afebrile and her vital signs are a heart rate of 140 beats/min, a respiratory
rate of 30 breaths/min, and a blood pressure of 85/55 mm Hg. She makes ample tears while crying. Her
tonsils are 1+ and do not approximate one another; her mother reports that she snores at night but has no
apnea. There is point tenderness and a small deformity of her right forearm. The remainder of her physical
examination findings are normal.
A radiograph of the right arm reveals a displaced closed fracture of the right distal radius.
An orthopedic surgeon is consulted for a closed forearm fracture reduction in the emergency department.
Of the following, the BEST next treatment step for this girl is to
Correct answer is B
PREP Pearl(s)
Children and adolescents who are healthy and have no significant comorbid conditions are generally
considered appropriate candidates for minimal, moderate, or deep sedation.
The minimum preoperative fasting interval is 2 hours for clear liquids (eg, pulp-free juices or water), 4
hours for human milk, and 6 hours for non-human milk or solid foods.
The most common serious complications of procedural sedation include airway compromise,
hypoventilation, airway obstruction, laryngospasm, hypoxemia, and apnea.
Critique
The girl in the vignette requires procedural sedation. She ingested apple juice (clear liquid) 2 hours ago. The
American Society of Anesthesiologists practice guidelines for minimum preoperative fasting intervals
recommend the following:
2 hours for clear liquids (eg, pulp-free juices or water)
4 hours for human milk
6 hours for non-human milk or solid foods
On the basis of these recommendations, the girl requires a minimum of 2 hours of fasting time, which she
achieved by the time she arrived at the emergency department. Therefore, the best next management step is
to proceed with moderate sedation now. Performing the procedure without sedation would be medically
inappropriate. Delaying procedural sedation for 6 hours after clear fluid intake is unnecessary.
Anesthesiology consultation for sedation management is not indicated, given the girl’s reassuring medical
history and physical examination findings.
The number of procedures performed with sedation in children by nonanesthesiologists outside the
traditional operating room setting has increased in the past several decades. Procedural sedation is
performed in physicians’ offices, dental offices, imaging facilities, emergency departments, and other
settings. Physicians performing procedural sedation must be well versed in the requirements for the
provision of safe sedation. Guidelines for the monitoring and management of children during and after
procedural sedation have been published and endorsed by the American Academy of Pediatrics (Coté et al,
2019).
Children and adolescents who are healthy and have no significant comorbid conditions are generally
considered appropriate candidates for minimal, moderate, or deep sedation. Risk factors that may impact
safe sedation, such as high body mass index or loose teeth, should be noted by the sedation specialist, but
they may not necessarily exclude a child from procedural sedation outside the operating room. However,
children with severe systemic disease, acute or severe illness, special health care needs, anatomical airway
abnormalities, moderate-to-severe tonsillar hypertrophy or significant obstructive sleep apnea may require
referral to anesthesiology even for procedures involving minimal or moderate sedation.
Regardless of the procedural setting, the sedation provider must have facilities, personnel, and equipment
immediately available to manage emergency and rescue situations. The most common serious complications
of sedation include airway compromise, hypoventilation, airway obstruction, laryngospasm, hypoxemia, and
apnea. Hemodynamic instability can ensue from the pharmacologic effects or as a complication of sedation.
In rare circumstances, cardiopulmonary arrest may occur, usually resulting from failure to recognize and
address respiratory compromise.
When considering the use of procedural sedation, practitioners must obtain a relevant history, including a
detailed review of systems to identify any risk factors for sedation-related complications, and perform a
focused physical examination. At a minimum, history of sedation and anesthesia events, symptoms of
obstructive sleep apnea, and enteral intake time all must be considered when determining sedation
candidacy.
Suggested Reading(s)
American Society of Anesthesiologists Committee. Practice guidelines for preoperative fasting and the
use of pharmacologic agents to reduce the risk of pulmonary aspiration: application to healthy patients
undergoing elective procedures: an updated report by the American Society of Anesthesiologists
Committee on Standards and Practice Parameters. Anesthesiology. 2011;114(3):495-511.
doi:10.1097/ALN.0b013e3181fcbfd9
Committee on Drugs, Section on Anesthesiology, American Academy of Pediatrics. Guidelines for the
elective use of conscious sedation, deep sedation, and general anesthesia in pediatric patients.
Pediatrics. 1985;76(2):317-321. doi:10.1542/peds.76.2.317
Coté CJ, Wilson S; American Academy of Pediatrics; American Academy of Pediatric Dentistry.
Guidelines for monitoring and management of pediatric patients before, during, and after sedation for
diagnostic and therapeutic procedures. Pediatrics. 2019;143(6):e20191000. doi:10.1542/peds.2019-
1000
Malviya S, Voepel-Lewis T, Tait AR. Adverse events and risk factors associated with the sedation of
children by nonanesthesiologists. Anesth Analg. 1997;85(6):1207-1213. doi:10.1097/00000539-
199712000-00005
Content Domain
Medical procedures
A 14-year-old boy is undergoing evaluation for sleep difficulties. His mother reports that the 15-year-old girl
who lived next door and attended the boy’s high school died by suicide a few weeks ago. The school
counselor has met with students as a group, but not individually. The mother has avoided discussion about
the girl with her son and his 9-year-old brother because her own father died by suicide and this has been a
difficult time for her as well. In private, the boy admits that he feels sad and is having problems sleeping but
reports no suicidal thoughts or plans to hurt himself. He reports no substance use. The results of
standardized screening instruments for depression and anxiety are negative.
A. ask his mother about access to firearms in the home and, if they are present, educate her
about their safe storage
B. complete a safety contract with him in the office and recommend regular meetings with the
school counselor
C. provide reassurance that he is exhibiting symptoms of a grief response and does not currently
have risk factors for suicide
D. recommend that his mother avoid discussing the event with her sons, so as not to increase their
risk for suicide
Correct answer is A
PREP Pearl(s)
Risk factors for suicide include depression, other psychiatric disorders, family history of suicide, and
recent exposure to suicide.
Neither avoidance of discussion of suicide nor completion of a safety contract decreases the risk of
suicide. Safety plans are more effective than safety contracts.
Practitioners play a key role in screening for depression and anxiety, assessing for suicide, and
providing appropriate management.
Critique
The boy in the vignette has risk factors for suicide, including a family history of death by suicide and recent
exposure to suicidal behavior. Firearms and suffocation are the leading causes of suicide in adolescents.
Access to lethal methods (eg, firearms) is an additional risk factor. Therefore, of the response choices, asking
about access to firearms and, if present, providing education about their safe storage is the best next
management step.
Providing reassurance alone would not be appropriate in this situation. Although the boy’s symptoms of a
grief response to his neighbor’s death are expected and appropriate, he also has risk factors for suicide that
must be recognized. It is not recommended that the boy’s mother avoid discussing the neighbor’s suicide.
Rates of suicide are known to increase among adolescents after a peer suicide (eg, contagion), but parents’
and physicians’ avoiding discussion of suicide does not decrease suicidal thoughts or behavior. Known
protective factors against suicide include having supportive relationships with parents and other family
members and a sense of connectedness.
Suicide is the second leading cause of death in adolescents aged 15 to 19 years and the eighth leading cause
of death in children aged 5 to 11 years. Girls consider and attempt suicide more frequently, but boys more
often die by suicide. Sexual minority youth have twice the risk of suicidal ideation.
Most teens who attempt suicide have a history of depression, anxiety, or other psychiatric diagnosis. Thus,
screening for mood disorders (eg, depression) and substance use is indicated. Other risk factors for suicide in
adolescents include the following:
Past physical or sexual abuse
Bullying
An impaired parent-child relationship
School difficulties
Substance use disorders
Posttraumatic stress disorder
History of aggression, impulsivity, or severe anger
In adolescents, pathologic internet usage (>5 hours a day), which may include playing video games, is
associated with increased rates of depression and suicidality. Internet use for suicide-related searches is also
associated with an increased risk of dying by suicide.
Children who die by suicide have many of the same risk factors for suicide as adolescents, with some
differences. In a multistate population-based qualitative study, younger children who died by suicide were
more likely to have the following factors:
Experienced bullying
A parent with depression
Other adverse exposures (eg, parental divorce, family discord)
A diagnosis of attention-deficit/hyperactivity disorder
Younger children were less likely to have a diagnosis of depression and more often had a negative event on
the day of death. Hanging or suffocation was the most frequent method, commonly occurring in the child’s
bedroom.
Safety contracts (agreements between the adolescent and clinician that they will not harm themselves) have
not been shown to be effective for adolescents who have past suicide attempts or continued suicidal
ideation. In contrast, safety plans have been shown to be effective. The components of a safety plan include
the following:
In addition to screening for depression, anxiety, and other psychiatric conditions, practitioners are often the
first to identify and assess children and adolescents at risk of suicide. Practitioners should be familiar with
resources available for urgent situations (such as emergency department information) and for referral to
mental health care practitioners when appropriate.
Suggested Reading(s)
Maslow GR, Dunlap K, Chung RJ. Depression and suicide in children and adolescents. Pediatr Rev.
2015;36(7):299-308. doi:10.1542/pir.36-7-299
Ruch DA, Heck KM, Sheftall AH, et al. Characteristics and precipitating circumstances of suicide among
children aged 5 to 11 years in the United States, 2013-2017. JAMA Netw Open. 2021;4(7):e2115683.
doi:10.1001/jamanetworkopen.2021.15683
Shain B; Committee on Adolescence; et al. Suicide and suicide attempts in adolescents. Pediatrics.
2016;138(1):e20161420. doi:10.1542/peds.2016-1420
Walter HJ, DeMaso DR. Psychiatric emergencies: suicidality, agitation, psychosis, and disaster exposure.
In: McInerny TK, Adam HM, Campbell DE, Foy JM, Kamat DM, eds. American Academy of Pediatrics
Textbook of Pediatric Care. American Academy of Pediatrics; 2017:chap 370. Accessed September 1,
2022. Pediatric Care Online
Content Domain
Mental health
The correct answer is: ask his mother about access to firearms in the home and, if they are present, educate
her about their safe storage
An 8-year-old girl is in the office for a health supervision visit. She arrived crying and in distress; her mother
explains that there was a stray dog in the parking lot that upset her. One year ago, the girl was bitten by a
dog in their neighborhood and since then has had a fear of dogs. She insists on taking a different route when
walking to school to avoid their neighbor’s dog, and she refuses to go to her grandmother’s home because
there is a dog there. The girl’s father had a similar fear of bees as a child, and he has reassured the girl and
her mother that she just needs time to “outgrow her fear.”
B. provide reassurance that this common fear will resolve on its own
C. recommend that the grandmother send her dog to a neighbor’s house when the girl visits
D. suggest that the family adopt a puppy so the girl can be exposed to dogs safely
Correct answer is A
PREP Pearl(s)
Anxiety disorders, including phobias, are underidentified and undertreated; screening should occur in
the primary medical home with referral for cognitive behavioral therapy when appropriate.
Phobias, a type of anxiety disorder, can be distinguished from specific fears by their duration, severity,
and impact on functioning.
Accommodation and avoidance of the feared object or situation is not recommended for children with
phobias.
Caregivers should be mindful of the potential contribution of their own fears and anxieties to their
children’s fears.
Critique
Of the response choices, the best next step in this girl’s care is to provide psychoeducation and refer her for
cognitive behavioral therapy. The girl in the vignette is exhibiting behavior consistent with a phobia of dogs. A
phobia is a type of anxiety disorder; it is defined by the American Psychiatric Association as intense and
excessive fear of a specific object or situation that occurs most days for a period of at least 6 months.
Individuals with a phobia may experience immediate anxiety that is out of proportion to the actual danger of
the specific object or situation. Avoidance of the object or situation can limit day-to-day functioning. Phobias
can be classified under the following categories:
Natural/Environmental (eg, nature, heights, the dark, thunder, lightning)
Injury (eg, fear of dentists or needles)
Animals (eg, dogs, snakes, insects)
Situations (eg, enclosed spaces [claustrophobia], flying)
Other (eg, clowns, loud noises)
Risk factors for the development of an anxiety disorder in children include, but are not limited to, a family
history of anxiety; increased stress/trauma resulting from housing instability, food insecurity, or both; and
other social determinants of health. These factors may limit a child’s ability to cope with adverse events and
develop resilience.
When assessing a child for a phobia, it may be helpful to gather information from several sources, including
the child, parents, and teachers. Common comorbidities include other anxiety disorders, attention-
deficit/hyperactivity disorder, and depression.
Specific fears that occur during childhood can be differentiated from phobias, as they usually occur at specific
developmental stages and usually subside with reassurance from parents or caregivers. Infants and toddlers
may experience stranger anxiety. Separation anxiety is frequently experienced by toddlers and preschool-
aged children. Common fears among children of preschool age include monsters and the dark. School-aged
children may fear injury or failure. Specific fears do not have the same duration as phobias.
Providing reassurance that the girl’s fear of dogs will resolve on its own is not appropriate, given the long
duration of her phobia and its impact on her functioning. Accommodating the girl’s phobia by having the
grandmother’s dog go to a neighbor’s home is not recommended, as it encourages development of strategies
to avoid exposure and inhibits progress and potential recovery.
Pediatricians play a crucial role in monitoring and screening for anxiety disorders in the medical home.
Anxiety disorders are underidentified and, therefore, undertreated. Given the tendency for anxiety disorders
to run in families, parents should be educated about the importance of not contributing to their child’s fear
with their own anxiety or accommodating their child to avoid exposure themselves.
Suggested Reading(s)
American Psychiatric Association, DSM-5 Task Force. Anxiety disorders. In: Diagnostic and Statistical
Manual of Mental Disorders: DSM-5. 5th ed. American Psychiatric Publishing; 2013.
doi:10.1176/appi.books.9780890425596
Benun J, Lewis C, Siegel M. Fears and phobias. Pediatr Rev. 2008;29(7):250-251. doi:10.1542/pir.29-7-
250
Doyle MM. Anxiety disorders in children. Pediatr Rev. 2022;43(11):618-630. doi:10.1542/pir.2020-
001198
Wissow LS. Anxiety. In: McInerny TK, Adam HM, Campbell DE, Foy JM, Kamat DM, eds. American
Academy of Pediatrics Textbook of Pediatric Care. 2nd ed. American Academy of Pediatrics; 2016:chap
129. Accessed September 1, 2023. Pediatric Care Online
Content Domain
Mental health
A 13-year-old girl with a history of well-controlled anxiety is seen in the office for follow-up after a hospital
admission for seizure-like episodes. Results of neuroimaging and electroencephalography were negative. Her
parents express frustration that no cause of the episodes was identified. They plan on keeping her out of
school until “she is back to normal.” The discharging physician recommended that the girl begin cognitive
behavioral therapy, but the parents have not accessed therapy because they do not understand how this will
help alleviate the episodes.
Since discharge, the girl has been complaining of leg pain and is requesting assistance with walking. She
appears comfortable during most of the visit, but she cries out in pain when standing up. Her physical
examination findings are otherwise normal.
Of the following, the BEST next step in this girl’s care is to
Correct answer is B
PREP Pearl(s)
Mental disorders, including anxiety, depression, and posttraumatic stress disorder, frequently coexist
with conversion disorder.
Pursuing further evaluation and testing after establishment of a diagnosis of conversion disorder is not
recommended and may delay the recovery process.
Education about the diagnosis of conversion disorder and collaboration with patients and their families
regarding treatment (with a focus on improving functioning) is crucial to successful management.
Critique
The girl in the vignette is exhibiting symptoms consistent with a conversion disorder (also known as a
functional neurologic symptom disorder). Conversion disorder is defined as atypical neurologic or physiologic
symptoms (changes in voluntary motor or sensory function) that interfere with daily functioning for which
there is no clear organic or psychiatric pathology, and the symptoms are not better explained by another
medical or mental disorder. The best next step in her management is to discuss the purpose of cognitive
behavioral therapy, so that her parents can better understand its importance in her care. She would also
benefit from physical therapy for her leg pain.
Symptoms of conversion disorder may include nonepileptic seizures, gait changes, fainting, and visual
changes. Individuals with conversion disorder can have and alternate among multiple symptoms. Each
symptom can be considered a subtype (eg, functional seizures, functional movement disorder, persistent
postural perceptual dizziness) with varying frequency and intensity. The results of evaluation of these
symptoms (eg, ophthalmologic examination, electroencephalography, vital signs, imaging) are negative.
The differential diagnosis of conversion disorder includes neurological diseases, factitious disorder,
malingering, depressive disorders, and panic disorder. Conversion disorder should not be considered a
diagnosis of exclusion. Early identification and treatment can avoid unnecessary testing and consultative
evaluations. Treatment for functional disorders includes a return to regular physical activity and cognitive
behavioral therapy. Aerobic exercise should be started, initially for a short duration, and increased to a goal
of at least 30 minutes per day. Resumption of academic activities and return to the school environment are
important aspects of the recovery plan. Cognitive behavioral therapy helps to improve self-regulation
through building coping skills and relaxation techniques, as well as by changing an individual’s response to
and thought process about stressors. For functional disorders with motor involvement, there is evidence
supporting participation in physical therapy or rehabilitative services to further facilitate recovery. Prolonged
inactivity, as in using a wheelchair, would inhibit her recovery.
Conversion disorders occur more frequently in females and in those who have experienced abuse or trauma.
Other risk factors include family dysfunction and bullying. For many individuals there is no identifiable event
or trigger; a documented stressor is not required to make the diagnosis. Meeting with the girl in the vignette
privately would not further her management or support recovery, as her history is consistent with conversion
disorder. Psychiatric disorders (eg, anxiety, depression, posttraumatic stress disorder, and somatic symptom
disorder) frequently coexist; a diagnosis of conversion disorder can be made in a child or adolescent with the
diagnosis of another mental disorder. The girl in the vignette is known to have anxiety disorder; however, her
acute symptoms are most likely secondary to a conversion disorder, and changing or increasing the dose of
her anxiety medication would not address these symptoms.
Readmission to the hospital for additional evaluation is not recommended for the girl in the vignette, as this
may contribute to increased stress for the family, interfere with the family’s understanding of the girl’s
diagnosis, and delay her recovery process. Delivery of the diagnosis and collaboration between the physician,
care team, patient, and family is critical in facilitating the recovery process. Education and reassurance help
the family understand and accept that this diagnosis is definitive and that further evaluation is not indicated.
This allows the focus to shift to improving the adolescent’s functioning by resumption of daily activities with
the support of recommended therapies and interventions.
Suggested Reading(s)
American Psychiatric Association. Diagnostic and Statistical Manual of Mental Disorders: DSM-5. 5th ed.
American Psychiatric Association; 2013.
Espay AJ, Aybek S, Carson A, et al. Current concepts in diagnosis and treatment of functional
neurological disorders. JAMA Neurol. 2018;75(9):1132-1141. doi:10.1001/jamaneurol.2018.1264
Hallett M, Aybek S, Dworetzky BA, McWhirter L, Staab JP, Stone J. Functional neurological disorder: new
subtypes and shared mechanisms. Lancet Neurol. 2022;21(6):537-550. doi:10.1016/S1474-
4422(21)00422-1
Prazar GE. Conversion reactions and hysteria. In: McInerny TK, Adam HM, Campbell DE, DeWitt TG, Foy
JM, Kamat DM, eds. American Academy of Pediatrics Textbook of Pediatric Care. American Academy of
Pediatrics; 2021:chap 237. Accessed September 14, 2022. Pediatric Care Online
Sunde KE, Hilliker DR, Fischer PR. Understanding and managing adolescents with conversion and
functional disorders. Pediatr Rev. 2020;41(12):630-641. doi:10.1542/pir.2019-0042
Content Domain
Mental Health
A 13-year-old girl is seen for a preparticipation physical examination before going to sleepaway camp for the
first time. Her mother reports that in the last year the girl has complained of frequent headaches, which have
occurred daily for the past week. She reports no visual changes, nausea, or vomiting. During the school year,
the girl’s mother noticed the headaches occurred more frequently during the school week, especially on days
when the girl had a test or a presentation to deliver to her class. When asked if she is excited about camp,
the girl admits that she is worried about something happening to her family while she is gone and is nervous
about being outside with “all the bugs.”
Her physical examination findings, including vital signs and neurological examination, are normal. A
symptom ratings scale completed by the girl confirms her diagnosis.
Of the following, the BEST next step in this girl’s treatment is to
A. prescribe a selective serotonin reuptake inhibitor and schedule follow-up in one week
B. provide education about her diagnosis and resources for cognitive behavioral therapy
C. recommend that she wait until next summer to attend sleepaway camp
D. suggest that she participate in a local nature program to overcome her fear of insects
Correct answer is B
PREP Pearl(s)
Anxiety disorders are the most common psychiatric diagnoses in children; the several types (eg,
separation, social, phobias) vary in age of onset.
Cognitive behavioral therapy is the first-line treatment for anxiety disorders in children and
adolescents.
Moderate to severe anxiety disorders may require medication (selective serotonin reuptake inhibitors)
in addition to cognitive behavioral therapy.
Critique
The girl in the vignette is expressing anxiety surrounding everyday events and has experienced frequent
headaches for more than 6 months; these symptoms are consistent with a generalized anxiety disorder. Of
the response choices, the best next step in her treatment is to provide education about the diagnosis and
resources for cognitive behavioral therapy (CBT). Cognitive behavioral therapy is the first-line treatment for
children with anxiety disorders.
Anxiety disorders are the most common psychiatric disorders in children and adolescents, affecting 7% of
children, with a median age of onset of 11 years. The frequency of individual types of anxiety disorders varies
according to age. Separation anxiety disorders are more common in preschool- and school-aged children.
Separation anxiety can be a normal process for children as they enter school for the first time. A separation
anxiety disorder occurs when a child experiences significant fear or worry that something bad will happen to
their caregiver or loved one while they are apart, leading to avoidance of separation. Symptoms of a disorder
must be present for at least 4 weeks and may include physical complaints on school mornings that are not
present at other times. Children with a separation anxiety disorder may also require a parent to be present
to fall asleep or may be resistant to being left with a babysitter or other family member, behaviors that limit
the development of independence and cause significant family dysfunction. It is not recommended that the
girl in the vignette stay at home and not attend summer camp. Similarly, it is not recommended that a child
with separation anxiety and school refusal be allowed to stay home from school.
Social anxiety most commonly occurs during adolescence and manifests as severe anxiety in social situations.
Children and adolescents experiencing social anxiety have a fear of drawing attention to or embarrassing
themselves in social situations. Examples include not wanting to speak in class or order at a restaurant. The
girl in the vignette has a component of social anxiety (headaches before a class presentation) as part of her
generalized anxiety disorder.
Specific phobias can occur at any age and are the most common form of anxiety disorder. Specific phobias
are frequently underidentified; affected people tend to avoid exposure, which can limit the phobia’s impact
and need for treatment. For diagnosis, the phobia must be present for at least 6 months and result in an
impairment in functioning. Although the girl in the vignette may have a fear of insects, she is also
experiencing anxiety with everyday activities and in multiple areas; this is most consistent with a generalized
anxiety disorder. Exposure to a specific fear or phobia can be part of the CBT plan, but her attendance at a
nature program will not address the other aspects of her anxiety.
Cognitive behavioral therapy should be offered as first-line treatment for generalized anxiety, social anxiety,
specific phobias, and separation anxiety for children and adolescents aged 6 to 18 years. Therapy involves
addressing thought processes surrounding anxiety-provoking situations, developing problem-solving skills
and coping strategies, and changing behavior. Therapy can also include gradual exposure to a specific phobia
or feared situation. For many children and adolescents with anxiety, CBT alone is effective. Medication,
specifically a selective serotonin reuptake inhibitor, is indicated for people with moderate to severe anxiety
that does not improve with CBT. Combination treatment (CBT and a selective serotonin reuptake inhibitor)
may be more effective in the short term than either modality alone.
Pediatricians should routinely screen for and provide resources for interventions, including CBT, when
appropriate. Given the frequency of anxiety disorders and shortage of behavioral health subspecialists,
children with moderate to severe anxiety may need to start using medication while seeking a CBT therapist.
Suggested Reading(s)
Bagnell AL. Anxiety and separation disorders. Pediatr Rev. 2011;32(10):440-445. doi:10.1542/pir.32-10-
440
Doyle MM. Anxiety disorders in children. Pediatr Rev. 2022;43(11):618–630. doi.org/10.1542/pir.2020-
001198
Walter HJ, Bukstein OG, Abright AR, et al. Clinical practice guideline for the assessment and treatment
of children and adolescents with anxiety disorders. J Am Acad Child Adolesc Psychiatry.
2020;59(10):1107-1124. doi:10.1016/j.jaac.2020.05.005
Wissow LS. Anxiety. In: McInerny TK, Adam HM, Campbell DE, DeWitt TG, Foy JM, Kamat DM, eds.
American Academy of Pediatrics Textbook of Pediatric Care. American Academy of Pediatrics;
2021:chap 129. Accessed September 16, 2022. Pediatric Care Online.
Content Domain
Mental Health
A 15-year-old is seen in the office for evaluation of new-onset auditory hallucinations. Their mother reports
that over the past 6 months the adolescent has become irritable, is sleeping more, is no longer interested in
extracurricular activities, and spends most of his time in his room. They still attends school full time, and
their grades have not declined. In private, the adolescent reports no history of trauma, bullying, substance
use, or physical or emotional abuse. During the interview, they become tearful and admit to feeling scared
because for the past 2 months they have heard a voice saying they are “worthless” and that everything would
be better if the adolescent was not around. The adolescent is otherwise able to stay on topic during the
interview and answers all questions appropriately. The adolescent insists, “I know the voice is not real—I just
want it to stop.” Their physical examination findings are unremarkable.
Of the following, the BEST next step in this adolescent’s care is to
B. obtain a comprehensive metabolic panel, complete blood cell count, and urine toxicology
screening
C. provide resources for cognitive behavioral therapy and social skills programming
Correct answer is A
PREP Pearl(s)
Psychosis is defined as the inability to distinguish reality from nonreality. Features include delusions,
hallucinations, disorganized thinking, and negative symptoms.
Second-generation antipsychotic medications are the first-line treatment for psychotic disorders,
including very-early-onset schizophrenia.
Psychiatric disorders (including major depressive disorder, anxiety, and obsessive-compulsive
disorder) can present with psychotic experiences that are a symptom of the underlying disorder.
Critique
The adolescent in the vignette is experiencing auditory hallucinations as a symptom of a major depressive
disorder. These symptoms are sometimes referred to as psychotic experiences and can also be seen in other
psychiatric conditions (eg, anxiety and obsessive-compulsive disorder). Given the content of the adolescent’s
hallucinations, suicide screening via a standardized instrument should take priority. If there is concern
regarding the adolescent’s safety, they should be referred to the emergency department for evaluation. The
other response choices may be appropriate but only after screening for suicidal ideation, plan, and intent.
Psychosis is defined as the inability to distinguish between reality and nonreality. Features of psychotic
disorders as defined by Diagnostic and Statistical Manual of Mental Disorders, 5th edition, include delusions,
hallucinations, disorganized thinking/speech or motor behavior (eg, catatonia), and negative symptoms.
Delusions can be further classified as bizarre (eg, an outside force is controlling one’s thoughts and actions)
or nonbizarre (thoughts are plausible, but have no supporting evidence). Hallucinations occur more
frequently than do delusions among children and adolescents diagnosed with psychosis. Hallucinations are
vivid perceptions that can involve any of the senses (sight, sound, smell, touch). Individuals with disorganized
thinking may be unable to stay on topic, have difficulty answering questions, or have incoherent speech.
Disorganized motor behaviors may include catatonia, ranging from a decreased response to environmental
stimuli to increased/excessive motor movement (eg, catatonic excitement). The adolescent in the vignette is
not exhibiting disorganized thinking or motor behaviors and appears to have insight into his auditory
hallucinations. Negative symptoms include decreased emotional expression, lack of speech, anhedonia, and
difficulty forming relationships. The adoelscent in the vignette is not experiencing negative symptoms.
Laboratory studies may be considered after a detailed medical history—including birth history, family
psychiatric history, history of trauma or abuse, and substance use—is obtained. This evaluation may include
a comprehensive metabolic panel, complete blood cell count, thyroid studies, HIV screening, antinuclear
antibodies test, and a urine toxicology screen. Urine toxicology screening should be considered because
some substances (including hallucinogens and cannabis) can cause psychotic experiences that would be
classified as a drug-induced psychotic disorder.
The prevalence of schizophrenia of early onset (before age 18 years) has been reported to be 0.5%. Very-
early-onset schizophrenia (onset of symptoms before age 13 years) has a prevalence of 1 per 10,000 children.
Prodromal symptoms of schizophrenia may include social withdrawal, worsening academic performance,
unusual behavior, and poor personal hygiene. The etiology of schizophrenia is multifactorial, with both
genetic and environmental risk factors. Environmental risk factors include prenatal substance exposure or
maternal infection, and advanced paternal age. Childhood trauma resulting from emotional or physical
abuse, domestic violence, and bullying can increase the risk of psychotic experiences and psychotic
disorders, especially when genetic risk is present.
Suggested Reading(s)
Abidi S. Psychosis in children and youth: focus on early-onset schizophrenia. Pediatr Rev. 2013
ul;34(7):296-305. doi:10.1542/pir.34-7-296
Hua LL; Committee on Adolescence. Collaborative care in the identification and management of
psychosis in adolescents and young adults. Pediatrics. 2021;147(6):e2021051486.
doi:10.1542/peds.2021-051486
McClellan J. Psychosis in children and adolescents. J Am Acad Child Adolesc Psychiatry. 2018;57(5):308-
312. doi:10.1016/j.jaac.2018.01.021
Walter HJ, DeMaso DR. Psychiatric emergencies: suicidality, agitation, psychosis, and disaster exposure.
In: McInerny TK, Adam HM, Campbell DE, Foy JM, Kamat DM, eds. American Academy of Pediatrics
Textbook of Pediatric Care. 2nd ed. American Academy of Pediatrics; 2023. Accessed September 1,
2023. Pediatric Care Online
Content Domain
Mental health
The correct answer is: inquire about thoughts of self-harm/suicidal ideation, plan, and intent
View Peer Results
AAP PREP 2024 - Question 140/267 Mental Health Question 6/6
A 10-year-old boy with attention-deficit/hyperactivity disorder-combined type and tics arrives 20 minutes late
for his routine follow-up visit. His mother apologizes and explains that they were late because the boy had to
perform his “leaving the house ritual.” Whenever they leave home, the boy must tap every door 10 times and
check that it is locked. He must complete the process in a particular order and, if interrupted, becomes very
upset and insists on starting from the beginning. Consequently, the family has been staying home more
frequently. In addition, the boy’s bedtime has recently become later because he repeats the same process
before going to sleep. The boy admits that he frequently worries about whether the doors are locked when
he is at home, at school, or in public, and checking the doors makes him feel better temporarily. He insists
that he is “keeping his family safe.” Before the current ritual, the boy had another bedtime routine that had
to be followed precisely. He has always been particular about his belongings being touched or moved. The
boy’s physical examination findings are unremarkable.
Of the following, the BEST next step in this boy’s care management is to
A. advise the family to accommodate the boy’s ritual and ensure they allow time for it
C. provide reassurance that these behaviors are common in children with attention-
deficit/hyperactivity disorder
D. refer him for cognitive behavioral therapy that incorporates exposure and response
prevention
Correct answer is D
PREP Pearl(s)
Cognitive behavioral therapy with exposure and response prevention is the first-line treatment for mild
to moderate obsessive-compulsive disorder.
Selective serotonin reuptake inhibitors should be considered for children with moderate to severe
obsessive-compulsive disorder.
It is important to advise parents and caregivers to not accommodate a child’s compulsive behavior.
Accommodation decreases the likelihood of remission of childhood obsessive-compulsive disorder.
Critique
The boy in the vignette has obsessive-compulsive disorder (OCD). The best next step in his care is referral for
cognitive behavioral therapy with exposure and response prevention. The boy is exhibiting recurrent and
persistent thoughts, worries, and urges resulting in repetitive behaviors or mental acts aimed at reducing
distress. These obsessions and compulsions are time consuming and distressing and cause significant
functional impairment (eg, work, social). To diagnose OCD, the symptoms cannot be better explained by
another mental disorder (eg, generalized anxiety disorder, major depressive disorder, eating disorder) and
are not the result of substances such as medications or illicit drugs. Symptoms of OCD may fall under several
themes, including cleaning, forbidden thoughts (sexual or religious), symmetry (repeating, ordering,
counting), and hoarding. In contrast to adults with OCD, children may not recognize that these recurring
obsessions and behaviors are excessive or unreasonable.
The first line of treatment for mild to moderate OCD in children is cognitive behavioral therapy. Cognitive
behavioral therapy that incorporates exposure and response prevention is the most efficacious. This therapy
involves exposing the individual to a trigger for their obsessive thoughts with focused practice in refraining
from engaging in the associated repetitive or compulsive behavior.
Selective serotonin reuptake inhibitors are effective for the treatment of children with moderate to severe
OCD. Prescribers and families need to be aware of potential side effects of these medications, including
suicidal ideation and development of behavioral activation or mania. The dose should be low initially and
increased slowly. The physician should refer the boy in the vignette for cognitive behavioral therapy before
considering treatment with a selective serotonin reuptake inhibitor.
Obsessive-compulsive disorder occurs in 1% to 3% of children and adolescents; the peak age of diagnosis is
between the ages of 9 and 10 years. Many children have symptoms that go unrecognized for several years.
The child may not be able to communicate their feeling of distress nor identify its cause, and the impairment
may be seen only at home. Comorbid conditions (eg, attention-deficit/hyperactivity disorder, Tourette
syndrome, and autism spectrum disorder) are frequently seen in childhood-onset OCD; these occur more
often in boys. This boy’s behaviors are better explained by OCD than by his attention-deficit/hyperactivity
disorder, which is a separate comorbid diagnosis.
There is an increased prevalence of OCD in first-degree relatives of children diagnosed with OCD; however,
the condition occurs sporadically for many children. Concordance in monozygotic twins is not 100%,
indicating a combination of genetic and nongenetic etiologic factors. The rate of remission is significantly
higher in cases of childhood-onset OCD than in cases of adult-onset OCD.
Many children with OCD experience increased irritability and meltdowns; this often leads parents and
caregivers to accommodate the child’s compulsions, which reinforces the OCD behaviors. The
accommodations required are dependent on the obsession. For the boy in the vignette, accommodations
include providing enough time for him to complete his ritual and staying home to avoid the process. Advising
the boy’s mother to accommodate his compulsive behaviors would not effectively treat his OCD and may
result in a worse outcome and decreased likelihood of remission. It is important to advise parents and
caregivers to not make accommodations and to limit verbal reassurance regarding the compulsive behavior.
Active involvement of parents/caregivers in cognitive behavioral therapy is encouraged.
Pediatricians play an active role in the identification and coordination of care for multiple mental health
conditions, including OCD. Administering a generalized mental health screening tool is encouraged when
concerns are brought up by parents or if concerning behavioral changes are reported (eg, decreased interest
in friends and activities, decline in school performance).
Suggested Reading(s)
American Psychiatric Association. Obsessive-compulsive disorder. In: The Diagnostic and Statistical
Manual of Mental Disorders. 5th ed. American Psychiatric Publishing; 2013: 260-263.
Geller DA, Homayoun S, Johnson G. Developmental considerations in obsessive compulsive disorder:
comparing pediatric and adult-onset cases. Front Psychiatry. 2021;12:678538.
doi:10.3389/fpsyt.2021.678538
Sarvet B. Childhood obsessive-compulsive disorder. Pediatr Rev. 2013;34(1):19-27. doi:10.1542/pir.34-
1-19
Adams H. Tics. Point of Care Quick Reference. Pediatric Care Online. American Academy of Pediatrics;
2022. Pediatric Care Online
Content Domain
Mental health
The correct answer is: refer him for cognitive behavioral therapy that incorporates exposure and response
prevention
View Peer Results
AAP PREP 2024 - Question 141/267 Musculoskeletal Question 1/5
A 14-year-old boy is seen for intermittent back pain that worsens by the end of the school day. He is on the
football team and performs daily stretching exercises but feels that his hamstrings are tight and affecting his
performance. His father thinks the boy’s symptoms are caused by poor posture, excessive use of the
computer for school, and playing video games at night. His parents constantly tell him to sit up and not
slouch.
On physical examination, the boy’s shoulders appear rounded and his head is leaning forward. With the
Adams forward bend test, there is a smooth, increased forward curvature of the thoracic spine that resolves
as he stands up.
A. kyphosis
B. lordosis
C. scoliosis
D. spondylitis
Correct answer is A
PREP Pearl(s)
There are 3 types of kyphosis: postural, structural, and congenital.
Treatment of kyphosis typically includes physical therapy and pain management; moderate to severe
cases may require bracing or surgery.
Critique
The boy in the vignette has clinical symptoms and signs consistent with kyphosis, an excessive forward
curvature of the thoracic spine. The boy's pain is likely the result of sitting for most of the school day.
Kyphosis most commonly occurs in the lower thoracic area. The vertebral bodies and intervertebral disks
play a role in the development of spinal abnormalities. There are 3 types of kyphosis: postural, structural, and
congenital. The features of these kyphosis types are compared in the Table. Kyphosis can also result from an
injury, infection, tumor, osteoporosis, and increasing age.
The Adams forward bend test is commonly used to screen for scoliosis. This test can also assist with the
diagnosis of kyphosis and help to differentiate between the postural and structural forms. As expected with
postural kyphosis, this adolescent’s forward curvature corrects as he stands upright. Tight hamstrings can be
a symptom of kyphosis. Slouching is a common concern expressed by parents of children with kyphosis.
When kyphosis is suspected, a standing lateral radiograph is recommended for further evaluation. Treatment
typically includes physical therapy and pain management. Progression is monitored with follow-up imaging.
For moderate and severe deformities, bracing and surgery may be indicated.
The boy in the vignette has clinical and diagnostic features consistent with kyphosis. Lordosis is an increase in
the backward, rather than forward, curvature of the spine. Scoliosis is a lateral curvature of the spine.
Spondylitis is an inflammation in the joints of the spine that results in stiffness.
Suggested Reading(s)
Anderson JT. Spinal deformities. In: McInerny TK, Adam HM, Campbell DE, DeWitt TG, Foy JM, Kamat
DM, eds. American Academy of Pediatrics Textbook of Pediatric Care. 2nd ed. American Academy of
Pediatrics; 2017:chap 333. Pediatric Care Online
Brenner JS, Smith DV. Back pain. In: McInerny TK, Adam HM, Campbell DE, DeWitt TG, Foy JM, Kamat
DM, eds. American Academy of Pediatrics Textbook of Pediatric Care. 2nd ed. American Academy of
Pediatrics; 2017:chap 131. Pediatric Care Online
Lam JC, Mukhdomi T. Kyphosis. In: StatPearls. StatPearls Publishing; 2022. Updated August 2022.
Kyphosis - PubMed (nih.gov)
Lamb M, Brenner JS. Back pain in children and adolescents. Pediatr Rev. 2020;41(11):557-569.
doi:10.1542/pir.2019-0051
Sarwark JF, LaBella CR. Kyphosis. In: Sarwark JF, LaBella CR. Pediatric Orthopaedics and Sports Injuries:
A Quick Reference Guide. 3rd ed. American Academy of Pediatrics; 2021:chap 12.
Content Domain
Musculoskeletal
Curvature >50° Anterior wedging of vertebral body >5° over 3 One or more vertebral bodies fail to
Normal vertebral and disc anatomy consecutive levels develop (failure of formation)
OR
Two or more vertebral bodies fail to
separate (failure of segmentation)
Growth and progression Associated with growth spurt Can progress quickly during growth spurt Progression until growth stops; can be
Wedging occurs during growth rapid during growth spurt
Physical examination findings Flexible and smooth curve Rigid and tender curve Rigid curve; curvature depends on
Curvature corrects when spine Curvature does not correct when spine straightens severity and type of deformity
straightens
Imaging Standing lateral radiograph of the Standing lateral radiograph of spine Standing lateral radiograph of spine
spine may be considered Wedging of vertebral body >5° in ≥3 consecutive vertebrae Magnetic resonance imaging, if
No wedging of anterior vertebral body Abnormal end plate neurologic abnormalities are present
A 12-year-old is seen for evaluation of right knee pain of 1 month’s duration. She has had no known trauma. The pain
worsens with weight bearing and improves with rest. There have been no associated fever, rash, or joint findings
(swelling, warmth, or erythema). On physical examination, the girl’s vital signs are normal. Her height is at the 50th
percentile, weight is at the 95th percentile, and body mass index is at the 97th percentile, consistent with previous
measurements. She has a limping gait. When lying supine on the table, she holds the right leg externally rotated.
Findings on examination of both knees are normal. She has limited range of motion of the right hip. Flexion
examination of the hips reveals findings similar to those shown in Figure 1. The remainder of her physical examination
findings are unremarkable.
Reprinted with permission from Karkenny AJ, Tauberg BM, Otsuka NY. Pediatr Rev. 2018;39(9):456.
Figure 1. Right and left leg hip flexion maneuver on examination of the girl in the vignette.
Of the following, the BEST next step in this girl’s evaluation is to obtain
Correct answer is B
PREP Pearl(s)
Slipped capital femoral epiphysis may present with knee or hip pain, limp, and external rotation of the hip on
flexion.
The evaluation of suspected slipped capital femoral epiphysis should begin with bilateral hip radiographs; serum
blood urea nitrogen, creatinine, thyrotropin, and free thyroxine levels may be considered in children younger
than 10 years or whose weight is less than the 50th percentile.
The definitive management of slipped capital femoral epiphysis is screw fixation. Complications of slipped capital
femoral epiphysis may include osteonecrosis, osteoarthritis, impingement, and leg-length discrepancy.
Critique
The girl in the vignette has knee pain and physical examination findings remarkable for external rotation of the right hip
with flexion, holding her right leg externally rotated when supine, and a limping gait. These findings are indicative of a
slipped capital femoral epiphysis (SCFE) of the right hip. The most appropriate next step in management is to obtain
bilateral hip radiographs. Although her signs and symptoms are unilateral, radiographs of both hips should be obtained
for comparison purposes.
Slipped capital femoral epiphysis occurs when the proximal femoral epiphysis is displaced because of weakness of the
physis. It may occur unilaterally or bilaterally; affected children may be at risk for a contralateral slip. Children with SCFE
often have obesity and are typically between 10 and 14 years of age. They commonly present with pain in the hip or
knee of varying intensity and duration. Bilateral pain may indicate bilateral SCFE; however, bilateral SCFE may also
manifest with unilateral pain. Physical examination findings include decreased range of motion of the affected hip on
abduction, flexion, and internal rotation. There may be a leg-length discrepancy. Weight-bearing ability may be an
indicator of stability: children who are unable to bear weight are more likely to have an unstable SCFE, which confers a
higher risk of complications.
Evaluation of a suspected SCFE should include frog-leg lateral and anteroposterior radiographs of both hips (Figure 2).
Magnetic resonance imaging should not be obtained before plain radiography, but it may be used to evaluate for an
early slip if radiographs appear normal but there is a high clinical suspicion of SCFE, to aid in evaluation of the
contralateral hip if a child is at risk of experiencing slip, or to further assess an unstable slip.
Reprinted with permission from Karkenny AJ, Tauberg BM, Otsuka NY. Pediatric hip disorders: slipped capital femoral epiphy
disease. Pediatr Rev. 2018;39(9):457.
Figure 2. A. Left-sided slipped capital femoral epiphysis (SCFE). Klein lines are drawn on both sides, illustrating its intersectio
right side while being lateral to it on the side with the SCFE. B. Left-sided SCFE with crescent-shaped area of increased densi
Slipped capital femoral epiphysis may be associated with renal osteodystrophy or thyroid dysfunction. Laboratory
evaluation (serum blood urea nitrogen, creatinine, thyrotropin, and free thyroxine levels) should be considered for
affected children younger than 10 years or with a weight less than the 50th percentile.
Surgical fixation is the definitive treatment for SCFE; a screw is placed from the metaphysis through the physis into the
epiphysis. In cases of unilateral SCFE, fixation of the contralateral hip may be indicated in children younger than 10
years, children with extreme obesity or metabolic abnormality, or children who may not be able to reliably return for
follow-up care.
Complications of SCFE include osteonecrosis, osteoarthritis, nerve impingement, and leg-length discrepancy. Causes of
leg-length discrepancy are shown in the Table.
Suggested Reading(s)
Janow G, Ilowite NT. Limp. In: McInerny TK, Adam HM, Campbell DE, Foy JM, Kamat DM, eds. American Academy
of Pediatrics Textbook of Pediatric Care. American Academy of Pediatrics; 2023. Accessed September 1, 2023.
Pediatric Care Online
Karkenny AJ, Tauberg BM, Otsuka NY. Pediatric hip disorders: slipped capital femoral epiphysis and Legg-Calvé-
Perthes disease. Pediatr Rev. 2018;39(9):454-463. doi:10.1542/pir.2017-0197
Sarwark JF, LaBella CR. Limb-length discrepancy. In: Sarwark JF, LaBella CR, eds. Pediatric Orthopaedics and
Sports Injuries: A Quick Reference Guide. 3rd ed. American Academy of Pediatrics; 2021:chap 60. Accessed
September 1, 2023. Pediatric Orthopaedics Online
Content Domain
Musculoskeletal
A previously healthy 16-year-old adolescent girl is evaluated in the office after receiving a positive result on a
scoliosis screening performed at school. Her menarche occurred at age 11 years. Findings of a review of
systems and family history are unremarkable. The adolescent’s weight and height are at the 50th percentile
for age. Physical examination reveals a sexual maturity rating of 5 and a slightly elevated right shoulder; on
the Adams forward bend test, the angle of her trunk rotation (measured with a scoliometer) is 7°. The
remainder of her physical examination findings are unremarkable. An anteroposterior radiograph of her
spine reveals a 15° Cobb angle and Risser stage 5. She is concerned about further progression of her
scoliosis.
Correct answer is A
PREP Pearl(s)
Adolescents who are skeletally mature and have a Cobb angle of 10 to <20° have a negligible risk of
scoliosis progression.
Although acquired idiopathic scoliosis is the most common form of scoliosis, other causes should be
considered (eg, genetic, neuromuscular).
Scoliosis in infants and young children may progress rapidly and may be difficult to treat; affected
children should be referred early to an orthopedic surgeon.
Critique
Scoliosis is defined as a lateral curvature of the spine. The adolescent in the vignette can be reassured that
her risk of scoliosis progression is negligible because she has a Cobb angle of 15° (between 10° and 20°) and
a Risser stage of 5, indicating that she is skeletally mature. There is little evidence to support the use of
physical therapy, vitamin D supplementation, maintaining good posture, and not wearing a heavy backpack
in limiting the progression of scoliosis. Because this adolescent’s scoliosis is unlikely to progress, bracing is
not indicated.
Idiopathic scoliosis can be classified by age: infantile idiopathic scoliosis (3 years or younger); juvenile
idiopathic scoliosis (4-9 years); and adolescent idiopathic scoliosis (10 years or older), the most common form
of idiopathic scoliosis. Scoliosis in infants and young children may progress rapidly and be difficult to treat;
affected children should be referred early to an orthopedic surgeon.
Physical examination for scoliosis should include inspection for asymmetry of the chest or back, including the
iliac crests. The child should then, if able, bend forward with their knees kept straight (the Adams forward
bend test); this assessment can highlight spinal asymmetry that is not otherwise evident. While the child is
flexed forward, the examiner can use a scoliometer to measure the angle of trunk rotation (Figure 1).
Children with scoliosis should be assessed for sexual maturity. Recommendations for proceeding to
radiography based on the scoliometer reading and sexual maturity for children with adolescent idiopathic
scoliosis are outlined in Table 1.
Radiography for children with scoliosis should be performed with the child erect if possible and should
include the entire spine on a single film for measurement of a Cobb angle (Figure 2). The radiographs are also
used to estimate skeletal maturity (Risser staging) and thus the risk of progression (Table 2). The Cobb angle
and Risser stage together are used to determine further management, which may include observation,
bracing, or surgery. Lateral radiographs should also be obtained to assess for conditions such as
spondylolisthesis or spondylolysis.
Suggested Reading(s)
Anderson JT. Spinal deformities. In: McInerny TK, Adam HM, Campbell DE, DeWitt TG, Foy JM, Kamat
DM, eds. American Academy of Pediatrics Textbook of Pediatric Care. American Academy of Pediatrics;
2017:chap 333. Accessed September 1, 2023. Pediatric Care Online
Furdock R, Brouillet K, Luhmann S. Organ system anomalies associated with congenital scoliosis: a
retrospective study of 305 patients. J Pediatr Ortho. 2019;39(3):e190-e194.
doi:10.1097/BPO.0000000000001279.
Phillips WA. Scoliosis management for primary care practitioners. Pediatr Rev. 2021;42(9):475-485.
doi:10.1542/pir.2019-0007.
Content Domain
Musculoskeletal
A 5-year-old boy with no significant medical or family history is seen in the office for a follow-up visit. He was
seen 8 weeks ago for a limp that started 1 day after tee-ball practice. He had no pain and was able to bear
weight. There was no history of fever, trauma, recent travel, or medication use. His physical examination
revealed mild decreased range of motion of his right hip and a subtle limp; examination findings were
otherwise normal. Recommended treatment included rest, ibuprofen as needed, and follow-up in 2 weeks.
Today, his mother reports that his symptoms have worsened and he has pain in his right leg. He is unable to
bear weight on the leg. He has pain and decreased internal rotation of his right hip, and his right leg is
shorter than the left. His skin does not feel warm, and there is no swelling or redness. Radiography of the
hips and pelvis (Figure) is performed.
Reprinted with permission from Herman MJ, Martinek M. The limping child. Pediatr Rev. 2015;36(5):192.
Of the following, the BEST next step in this child’s management is to refer him to a pediatric
A. emergency department
B. oncologist
C. orthopedic surgeon
D. rheumatologist
Correct answer is C
PREP Pearl(s)
Referral to a pediatric orthopedic surgeon is important for short-term and long-term management of
Legg-Calvé-Perthes disease. The goals of treatment are to slow the progression of disease and
minimize the risk for osteoarthritis.
Legg-Calvé-Perthes disease (avascular necrosis of the femoral head) has 4 phases: vascular insult,
fragmentation, reossification, and remodeling.
The history and physical examination can guide the pediatrician in the assessment and management
of a patient presenting with a limp.
Critique
The history, physical examination, and radiographic findings for the child in the vignette are consistent with
avascular necrosis of the right femoral head (Legg-Calvé-Perthes disease). The best next step for this child is a
referral to a pediatric orthopedic surgeon.
The clinical presentation of Legg-Calvé-Perthes disease often begins with a painless limp that may be present
for a few months before the child is brought for medical evaluation. Symptoms typically occur in children
between the ages of 4 and 8 years, and the disease is more common in boys. Pain can develop as the disease
progresses, which may occur at the hip itself or be referred to nearby sites, such as the knee. Physical
examination may reveal decreased range of motion with internal rotation and abduction of the hip (assessed
with passive movement of the legs). There may be a leg-length discrepancy and/or muscle atrophy of the
affected limb. When avascular necrosis of the hip is suspected, radiographs should be performed that include
an anteroposterior view of the pelvis and frog-leg lateral views of both hips. Findings consistent with this
diagnosis may range from a flattened appearance to lateral displacement of the femoral head, depending on
the stage of disease.
The etiology of Legg-Calvé-Perthes disease is unknown. The disease has 4 phases. Phase 1 is initial vascular
insult, which has a variety of potential causes (eg, trauma and steroid use). This interruption of the blood
supply to the femoral head of the hip results in necrosis and affects growth. Phase 2 is fragmentation in
which the dead bone is absorbed by the body and the femoral head starts to have a flattened appearance.
Phase 3 is reossification, in which the femoral head starts to remodel and takes on a round appearance
again. Phase 4 is remodeling, which continues until growth stops.
Referral to a pediatric orthopedic surgeon is important for short-term and long-term management. The goals
of treatment are to slow the progression of the disease and minimize the risk of osteoarthritis. Initial
treatment may include nonsteroidal anti-inflammatory medications, restricted activity, and physical therapy.
Surgery may be indicated for older patients who do not respond to conservative treatment.
The differential diagnosis of a limp in childhood is lengthy. To narrow this list, it is important to take a
detailed history that includes the onset of symptoms, progression, aggravating and alleviating factors, quality
of pain (if present), referred pain, history of trauma, recent illness, systemic symptoms, medications, and
family history. The physical examination should include a general inspection of the skin for signs that may
indicate infectious, rheumatologic, and neurologic causes. The musculoskeletal examination should assess
the back, pelvis, and bilateral lower extremities. Depending on the history and physical examination findings,
imaging or laboratory testing may be indicated. Radiographs can reveal bony abnormalities, including those
associated with trauma, infections, tumors, and congenital conditions. In certain cases, computed
tomography, magnetic resonance imaging, or ultrasonography can be helpful. Laboratory testing can confirm
or exclude causes, such as infections and rheumatologic conditions.
Although infection, autoimmune conditions, and malignancy should be considered in the differential
diagnosis for hip pain and limp, this boy’s history, physical examination, and radiographic findings are most
consistent with Legg-Calvé-Perthes disease. A referral to a pediatric rheumatologist or oncologist is not
warranted at this time. Although this condition should be addressed expeditiously, it does not require
referral to an emergency department.
Suggested Reading(s)
American Academy of Orthopaedic Surgeons. Perthes disease. October 2019. Accessed July 10, 2022.
https://orthoinfo.aaos.org/en/diseases--conditions/perthes-disease
Herman MJ, Martinek M. The limping child. Pediatr Rev. 2015;36(5):184-197. doi:10.1542/pir.36-5-184
Janow G, Ilowite NT. Limp. In: McInerny TK, Adam HM, Campbell DE, DeWitt TG, Foy JM, Kamat DM, eds.
American Academy of Pediatrics Textbook of Pediatric Care. American Academy of Pediatrics; 2023.
Accessed September 1, 2023. Pediatric Care Online
Karkenny AJ, Tauberg BM, Otsuka NY. Pediatric hip disorders: slipped capital femoral epiphysis and
Legg-Calvé-Perthes disease. Pediatr Rev. 2018;39(9):454-463. doi:10.1542/pir.2017-0197
Limp. Point-of-Care Quick Reference. Pediatric Care Online. American Academy of Pediatrics. May 11,
2015. Accessed September 1, 2023. Pediatric Care Online
Content Domain
Musculoskeletal
A previously healthy 9-month-old infant is brought to the clinic for recurrent episodes of head-tilting, during
which she becomes fussy and appears pale. At times she tilts her head to the left and other times to the
right. Events always resolve spontaneously within 24 hours. Her family history is significant only for maternal
migraines. Between episodes she is asymptomatic with good oral intake. She is not having frequent emesis
and has been growing well. On physical examination, the infant is well-appearing and in no distress. Her
head is symmetrical with no evidence of plagiocephaly. She has normal muscle tone and strength, and is able
to stand with assistance. The remainder of her physical examination findings are normal.
A. a cerebellar mass
B. breath-holding spells
C. paroxysmal torticollis
D. Sandifer syndrome
Correct answer is C
PREP Pearl(s)
Treatment for congenital torticollis should ideally be initiated within the first month after birth.
Delay in treatment of congenital torticollis can lead to asymmetry of the face, mandible and ear; untreated
torticollis can result in cervical spine dysmorphism as early as 8 months of age.
Paroxysmal torticollis is episodic; episodes resolve over hours to days, can alternate the side affected, and are
associated with symptoms such as irritability, fatigue, pallor, and ataxia.
Critique
The infant in the vignette most likely has paroxysmal torticollis, which, unlike congenital torticollis, is episodic.
The episodes resolve after hours to days, may alternate the side affected, and are associated with symptoms
such as irritability, fatigue, pallor, and ataxia. Paroxysmal torticollis affects infants and young children, and
typically resolves by the age of 5 years. There seems to be familial occurrences. Affected children have an
increased risk for developing migraines. The cause is unknown.
A cerebellar mass could lead to torticollis, but the episodic nature of this infant’s events and her otherwise
normal physical examination makes this diagnosis less likely. Breath-holding spells are typically brought on
when an infant is upset, resulting in crying leading to vagal stimulation and a brief loss of consciousness with
return to normal within a few seconds. The infant's episodes are not brought on by things upsetting to the
infant and are associated with other symptoms for a more prolonged period making this a less likely
diagnosis. Sandifer syndrome can be episodic, but it is associated with symptoms of gastroesophageal reflux,
which this infant does not have.
The more common form of torticollis in infants is congenital torticollis. This is caused by shortening and
secondary tightening of the sternocleidomastoid muscle; associated fibrosis is visible on ultrasonography. On
physical examination, there may be a mass palpable within the sternocleidomastoid muscle. The shortened
muscle results in ipsilateral cervical lateral flexion and contralateral cervical rotation. Other physical
examination findings may include positional plagiocephaly, with flattening of the contralateral side of the
posterior skull. If not treated early, this leads to asymmetry of the face, mandible, and ear (Figure). Untreated
congenital torticollis can lead to cervical spine dysmorphism and limited cervical motion by age 8 months.
Reprinted with permission from Kuo AA, Tritasavit S, Graham JM. Congenital muscular torticollis and
positional plagiocephaly. Pediatr Rev. 2014;35(2):81.
Ideally, treatment of congenital torticollis should be initiated within the first month after birth. Early
treatment typically allows for the shortest treatment course (4 to 8 weeks). Long-term complications are
unlikely if treatment is initiated by 3 months of age. Treatment begins with an extensive evaluation by a
physical therapist, who will define a plan that includes the following:
Exercise to encourage stretching and strengthening of the sternocleidomastoid muscle, including both
active and passive ranges of motion;
Education to improve active symmetrical movement, eg, prone play or “tummy time”;
Environmental adaptations, eg, placing interesting toys and objects on the side opposite of the head
tilt;
Caregiving adaptations, eg, positioning while feeding to promote turning to the desired side.
Physical therapy visits typically occur weekly or biweekly to evaluate progress and reinforce the treatment
plan.
Children who have torticollis that does not improve with appropriate physical therapy should be evaluated
for other causes of head tilt. Diagnostic considerations include congenital scoliosis, abnormal tone, visual
impairment, hemivertebrae, Klippel-Feil syndrome, central nervous system tumor, and gastroesophageal
reflux with Sandifer syndrome.
Suggested Reading(s)
Ozuah PO, Skae CC. Torticollis. In: McInerny TK, Adam HM, Campbell DE, DeWitt TG, Foy JM, Kamat DM,
eds. American Academy of Pediatrics Textbook of Pediatric Care. American Academy of Pediatrics;
2021:chap 203. Accessed September 1, 2023. Pediatric Care Online
Sargent B, Kaplan SL, Coulter C, Baker C. Congenital muscular torticollis: bridging the gap between
research and clinical practice. Pediatrics. 2019;144(2):e20190582. doi:10.1542/peds.2019-0582
Content Domain
Musculoskeletal Disorder
A 46-hour-old neonate has not yet stooled and has had only 1 small void. He was born at 39 weeks’ gestation
via cesarean section to a 32-year-old, gravida 1 para 0 woman. She had an uncomplicated pregnancy with no
significant findings on prenatal testing. Her group B β-hemolytic Streptococcus test result was positive, but
she did not undergo labor before her cesarean section. The neonate had been breastfeeding well every 2 to
3 hours, but this morning his mother noted that he was fussy while trying to feed. His birth weight was 3,520
g, length was 52 cm, and head circumference was 36 cm. His physical examination findings are notable for a
soft, nontender, distended abdomen with decreased bowel sounds. There is a small, closed dimple noted
within the intergluteal cleft. The remainder of his examination findings are normal. During the examination,
the infant has a large amount of nonbilious emesis.
Of the following the MOST likely cause of this infant’s findings is
A. cystic brosis
C. pyloric stenosis
Correct answer is A
PREP Pearl(s)
Delayed passage of meconium and meconium ileus occurs in approximately 20% of children with cystic
fibrosis.
Evaluation and sweat testing should be performed at a Cystic Fibrosis Foundation–accredited care
center within 72 hours of a confirmed positive cystic fibrosis screening test result.
Meconium ileus can present with abdominal distention and poor feeding. In approximately 30% of
cases, meconium ileus will lead to perforation and peritonitis.
Critique
The most likely cause of this neonate’s delayed passage of meconium and minimal urination is meconium
ileus secondary to cystic fibrosis. Delayed passage of meconium or meconium ileus occurs in approximately
20% of neonates with cystic fibrosis. Thick gastrointestinal secretions attach to the intestinal mucosa and
cause bowel obstruction. Meconium ileus can present with abdominal distention and poor feeding. In
approximately 30% of cases, meconium ileus will lead to perforation and peritonitis. The neonate in the
vignette had a large meconium plug that caused extraluminal bladder outlet obstruction, leading to his
minimal urine output.
In the United States, all state newborn screening programs include cystic fibrosis. Testing for immunoreactive
trypsinogen is performed, and if the level is elevated, a confirmatory genetic test for cystic fibrosis
transmembrane conductance regulator (CFTR) pathogenic variants is done. If this test result is positive for 2
CFTR deletions, the neonate should be referred to a Cystic Fibrosis Foundation–accredited care center for
definitive evaluation and sweat testing performed within 72 hours of the positive result. These sites can be
found at http://www.cff.org/ccd/.
Posterior urethral valves affect approximately 1 in 8,000 infants assigned as male at birth. The embryologic
mechanism is not fully understood, but the developmental disruption likely occurs between 9 and 14 weeks’
gestation. The valves obstruct the lumen of the posterior urethra, leading to urethral obstruction. The
obstruction can cause increased pressure all along the urinary tract, leading to bladder and renal damage.
Severe posterior urethral valves often cause oligohydramnios, which can be seen on prenatal
ultrasonography. Affected newborns are often born with respiratory distress, intrauterine growth restriction,
lethargy, and poor feeding. Voiding cystourethrography will show a thickened and trabeculated bladder with
diverticuli and severe vesicoureteral reflux (Figure 1). Delayed passage of meconium is not a sequela of
posterior urethral valves.
Reprinted with permission from Dove DE, Smith ML. Obstructive uropathy and vesticoureteral reflux. In:
McInerny TK, Adam HM, Campbell DE, DeWitt TG, Foy JM, Kamat DM, eds. American Academy of Pediatrics
Textbook of Pediatric Care (Online). 2nd ed. American Academy of Pediatrics; 2017:chap 299
Pyloric stenosis affects approximately 0.2% to 0.4% of children. It occurs approximately 6 times more
frequently in males compared with females; fIrst-born males are most often affected. Infants with pyloric
stenosis often present with projectile, nonbilious emesis after feeds. The diagnosis of pyloric stenosis is made
with ultrasonography; a positive ultrasonography result shows pyloric muscle thickness greater than 3 mm
and the length of the channel is 15 mm or more (Figure 2). The mean age of presentation of hypertrophic
pyloric stenosis is 6 weeks (typically between 2 weeks and 3 months of age). Pyloric stenosis has not been
described in the early newborn period and is not associated with delayed passage of meconium or minimal
urination.
Reprinted with permission from American Academy of Pedaitrics. Point-of-Care Quick Reference. Pediatric
Care Online. 2022.
The neonate in the vignette has minimal urination and delayed passage of meconium, so it is reasonable to
consider a neurogenic cause, such as spina bifida (Figure 3). This neonate has a small, closed dimple present
in the intergluteal crease. This dimple is most likely a coccygeal pit, making the diagnosis of spina bifida
occulta unlikely. Additionally, if spina bifida occulta were present, it would be unlikely to cause the bowel and
bladder symptoms seen in this newborn.
Reprinted with permission from Dove DE, Smith ML. Neural tube defects. In: McInerny TK, Adam HM,
Campbell DE, DeWitt TG, Foy JM, Kamat DM, eds. American Academy of Pediatrics Textbook of Pediatric
Care (Online). 2nd ed. American Academy of Pediatrics; 2017:chap 296.
Suggested Reading(s)
Dickinson KM, Collaco JM. Cystic fibrosis. Pediatr Rev. 2021;42(2):55-67. doi:10.1542/pir.2019-0212
FIshberg SE, Landau EH, Divdevani M, et al. Posterior urethral valves: prenatal, neonatal and long-term
management. NeoReviews. 2018;19(12):e753-e761. doi:10.1542/neo.19-12-e753
Lahiri T, Sullivan J, Sartorelli K, Murphy JJ. Delayed presentation of meconium ileus in an infant with
cystic fibrosis. Pediatrics. 2020;146(4):e20193717. doi:10.1542/peds.2019-3717
Reddy P, Subramanian, S, Murki S, et al. Neonate with urinary retention. Neoreviews. 2021;22(7):e470-
e472. doi:10.1542/neo.22-7-e470
Rich BS, Dolgin SE. Hypertrophic pyloric stenosis. Pediatr Rev. 2021;42(10):539-545.
doi:10.1542/pir.2020-003277
Content Domain
Neonatology
A 48-hour-old newborn has neonatal abstinence scores of 9 and 12 (recorded 2 hours apart) with poor
feeding, nasal congestion, inconsolable crying, jitteriness, increased tone, loose stools, tachypnea, and
sweating. The newborn’s mother has a history of chronic hip pain following an injury while playing softball in
school approximately 5 years ago.
A. acetaminophen
B. ampicillin
C. morphine
D. swaddling
Correct answer is C
PREP Pearl(s)
Newborns with prenatal exposure to prescribed or illicit drugs should be monitored for neonatal
abstinence syndrome using a validated scoring system.
In addition to supportive care (eg, swaddling and minimal stimulation), newborns with neonatal
abstinence syndrome may require pharmacologic therapy.
Newborns with neonatal abstinence syndrome are at increased risk for admission to the neonatal
intensive care unit, preterm birth, low birth weight, hyperbilirubinemia, feeding intolerance with failure
to thrive, and microcephaly.
Critique
The neonate’s history and physical examination findings are consistent with neonatal abstinence syndrome
(NAS). The best next step in the newborn’s management is treatment with morphine. Newborns with prenatal
exposure to opiates, both prescribed and illicit, are at increased risk for NAS after birth. Although prenatal
exposure to other drugs (eg, benzodiazepines, amphetamines, cocaine, and barbiturates) can result in NAS, it
occurs most often in opioid-exposed newborns. Most neonates with NAS present with withdrawal symptoms
within 3 days after birth; however, if the mother has used longer-acting opiates (eg, methadone) symptoms
may be delayed for 1 to 2 weeks.
The American Academy of Pediatrics recommends that newborns prenatally exposed to short-acting opioids
be observed in the hospital for at least 3 days and those exposed to long acting opioids for 5 to 7 days.
Neonatal abstinence syndrome manifests with multisystem involvement, including neurologic, respiratory,
gastrointestinal, and other regulatory systems. Clinical signs and symptoms may include abnormal tone,
high-pitched cry, temperature instability, sneezing, nasal congestion, tachypnea, poor feeding, poor suck,
difficulty sleeping, inconsolability, and seizures. Several NAS scoring tools are available to assess the severity
of NAS symptoms. The most used tool is the Modified Finnegan Neonatal Abstinence Scoring System (Table).
Nonpharmacologic treatment, such as swaddling, minimal stimulation, and calm interactions, is an important
first step in the management of neonates with NAS. If NAS scores are higher than 8, as they are for this
neonate, pharmacologic therapy is indicated. The first-line treatment options are morphine or methadone.
Alternative medications include phenobarbital, buprenorphine, and clonidine. Acetaminophen is not an
appropriate medication to treat NAS. Although this neonate has some symptoms that could be seen in
sepsis, the most likely diagnosis is NAS; therefore, ampicillin is not currently indicated.
The recent opioid epidemic has resulted in increased opioid use in pregnancy (prescription and illicit), in turn
resulting in an increased incidence of NAS. Timely and appropriate antenatal management of the pregnant
mother are crucial for the prevention of NAS. Neonates with NAS are at increased risk for admission to the
neonatal intensive care unit, preterm birth, low birth weight, hyperbilirubinemia, feeding intolerance with
failure to thrive, and microcephaly. Although screening drug tests are available for both mother and
newborn, there is no gold standard practice for identifying prenatal substance use. Urine drug testing of the
mother and newborn is the most commonly used screening test, but this only detects substances within
hours to several days after consumption. Meconium is produced in the second trimester; a meconium drug
screening test reflects drug exposure that occurred during the last several weeks to months preceding
delivery. A positive meconium drug screen result usually indicates long-term drug use.
Suggested Reading(s)
Bailey NA, Diaz-Barbosa M. Effect of maternal substance abuse on the fetus, neonate, and child.
Pediatr Rev. 2018;39(11):550-559. doi:1542/pir.2017-0201
Giri P, Roth P. Treatment of neonatal opioid withdrawal. Pediatr Rev. 2021;42(9):522-525.
doi:10.1542/pir.2020-002519
Hudak ML, Tan RC, Committee on Drugs, Committee on Fetus and Newborn, et al. Neonatal drug
withdrawal. Pediatrics. 2012;129(2):e540-e560. doi:10.1542/peds.2011-3212
Nawaz N, Hester M, Oji-Mmuo CN, Gomez E, Allen AM. Risk factors associated with perinatal relapse to
opioid use disorder. Neoreviews. 2022;23(5):e291-e299. doi:10.1542/neo.23-5-e291
Oji-Mmuo CN, Corr TE, Doheny KKI. Addictive disorders in women: the impact of maternal substance
use on the fetus and newborn. Neoreviews. 2017;18(10):e576-e586. https://doi.org/10.1542/neo.18-10-
e576
Content Domain
Neonatology
Adapted and reprinted with permission from Giri P, Roth P. Treatment of neonatal opioid
withdrawal. Pediatr Rev. 2021;42(9):523
AAP PREP 2024 - Question 148/267 Neonatology Question 3/13
A 4-hour-old male neonate is evaluated in the newborn nursery. He was born vaginally at 39 weeks’ gestation
with Apgar scores of 9 and 9 at 1 and 5 minutes, respectively. He was able to breastfeed for several minutes
and voided once. Physical examination reveals a well-appearing neonate with the ndings shown in the
Figure. The stretched penile length is 3 cm and both testes are palpable in the scrotum. The remainder of the
physical examination ndings are normal.
Reprinted with permission from Wu WJ, Gitlin JS. The male genital system. Pediatr Rev. 2020;4(3):103.
C. hypopituitarism
D. isolated hypospadias
Correct answer is D
PREP Pearl(s)
Parents should consider the medical benefits and risks as well as their cultural beliefs when making an
informed decision regarding circumcision for their male neonate.
Circumcision should be performed by trained personnel, and adequate analgesia should be provided
using topical anesthetics or nerve blocks.
Neonatal circumcision is contraindicated in the presence of structural abnormalities of the penis (eg,
hypospadias, micropenis) or a family history of a bleeding disorder.
Critique
The physical examination findings of the neonate in the vignette are consistent with coronal hypospadias
(urethral meatus located on the ventral surface of the corona, abnormal prepuce creating a dorsal hood). The
most likely diagnosis is isolated hypospadias. There is no cryptorchidism, micropenis, or other features to
suggest a disorder of sex development (eg, 5-alpha reductase deficiency, congenital adrenal hyperplasia) or
hypopituitarism.
Hypospadias is defined as an abnormally located urethral meatus. It is caused by failure of fusion of the
urethral folds, which usually occurs during the 8th week of gestation. Associated physical examination
findings commonly include chordee (ventral curvature of the penis) and a dorsal hood (excess foreskin
dorsally). Hypospadias is classified as perineal, scrotal, penoscrotal, penile, coronal, or glandular, depending
on the location of the urethral meatus.
Except for the most distal hypospadias, the foreskin is used in reconstructive surgery. Therefore, when
hypospadias is present, circumcision should be delayed in the newborn period. Neonatal circumcision is also
contraindicated when other structural abnormalities of the penis are present, such as webbed penis, buried
penis, or micropenis.
Excessive bleeding from venipuncture sites or a family history of a bleeding disorder (eg, hemophilia,
thrombocytopenia) require further evaluation and management before circumcision is performed.
The American Academy of Pediatrics does not recommend universal routine neonatal male circumcision but
endorses that the procedure’s health benefits (decreased incidence of neonatal and young infant urinary
tract infections, decreased risk of sexually transmitted infections, decreased risk of penile cancer) outweigh
its risks (bleeding, infection). Parents should consider the medical benefits and risks as well as their cultural
beliefs when making an informed decision regarding circumcision for their male neonate.
Circumcisions should be performed by trained personnel, and adequate pain control should be provided.
Topical anesthetic creams, dorsal nerve blocks, and subcutaneous ring blocks are commonly used to control
pain for neonatal circumcision.
Hypospadias is usually an isolated defect. However, if cryptorchidism is also present, karyotype testing
should be performed to evaluate for a disorder of sex development. If no testes are palpable, the possibility
of female (46,XX) virilization due to congenital adrenal hyperplasia should be considered; in such a case,
electrolyte and 17-hydroxyprogesterone levels should be obtained.
The enzyme 5-alpha reductase converts testosterone to the more potent dihydrotestosterone (DHT), which is
responsible for differentiation of the male external genitalia. Inadequate levels of DHT in 5-alpha reductase
deficiency causes ambiguous genitalia with failure of fusion of the labioscrotal folds and underdevelopment
of the penis. Neonates with hypopituitarism may present with recurrent hypoglycemia. Male neonates can
have a micropenis (stretched penile length of less than 2 standard deviations below the mean, or 2 cm in a
term neonate) and cryptorchidism due to gonadotropin deficiency. The neonate in the vignette has a normal
stretched penile length and bilaterally descended testes, making congenital adrenal hyperplasia, 5-alpha
reductase deficiency, and hypopituitarism unlikely diagnoses.
Suggested Reading(s)
American Academy of Pediatrics Task Force on Circumcision. Circumcision policy statement. Pediatrics.
2012;130(3):585-586. doi:10.1542/peds.2012-1989
Freedman AL. The circumcision decision. In: McInerny TK, Adam HM, Campbell DE, DeWitt TG, Foy JM,
Kamat DM, eds. American Academy of Pediatrics Textbook of Pediatric Care. American Academy of
Pediatrics; 2023. Accessed September 1, 2023. Pediatric Care Online
Wu WJ, Gitlin JS. The male genital system. Pediatr Rev. 2020;4(3):101-111. doi:10.1542/pir.2017-0316
Content Domain
Neonatology
A 37-weeks’ gestation female neonate is admitted to the newborn nursery. Her birth weight is 2020 g (second
percentile), length is 45 cm (second percentile), and head circumference is 33 cm (third percentile). Her
physical examination findings are otherwise unremarkable. A complete blood cell count is obtained, with
normal findings other than a platelet count of 65.0 × 103/µL (65.0 x 109/L). The newborn's mother received
early and regular prenatal care, has no history of hypertension or diabetes, and is a nonsmoker. Except for a
flulike illness in the first trimester, the pregnancy was unremarkable.
A. chromosomal disorder
B. congenital hypothyroidism
C. congenital infection
D. preeclampsia
Correct answer is C
PREP Pearl(s)
Small for gestational age neonates have higher morbidity and mortality than those that are
appropriate for gestational age.
Small for gestational age neonates are at increased risk for metabolic syndrome as adults.
Newborns who are small for gestational age with no known cause should have screening for
cytomegalovirus and hearing screen performed. Early detection and intervention are important to
prevent sequelae of hearing loss.
Critique
The neonate in the vignette’s growth parameters place her in the growth category of symmetric small for
gestational age (SGA), with all growth indexes below the 10th percentile. This is distinguished from
asymmetrical SGA, where the weight is proportionately less (below the 10th percentile) than length and head
circumference. This finding, in the context of the maternal history of a flulike illness in the first trimester and
the neonate’s low platelet count, makes a congenital infection the most likely cause. One congenital infection
to consider in this scenario is cytomegalovirus (CMV) infection. Screening for CMV should be performed on
newborns with unexplained SGA, as well as hearing screening; early detection and intervention are important
to identify and prevent sequelae of hearing loss.
Although chromosomal disorders are a cause of symmetric SGA, the mother’s history of flulike illness and no
findings of dysmorphology in the newborn make this unlikely. Other causes of symmetric SGA include
constitutional factors, metabolic disorders, and uterine abnormalities. The most common cause of
asymmetric SGA is placental insufficiency secondary to preeclampsia, chronic maternal disease (eg,
hypertension), or advanced maternal age. Neonates with congenital hypothyroidism are usually appropriate
for gestational age and have a normal platelet count.
Small for gestational age neonates are not only at increased risk for complications in utero and postnatally
but also at increased risk for long-term morbidity (eg, metabolic syndrome). Evidence suggests that adults
who experienced severe growth restriction in utero have a significantly increased incidence of hypertension,
insulin resistance, and type 2 diabetes. Small for gestational age neonates at all gestational ages have higher
neonatal morbidity and mortality when compared with appropriate for gestational age neonates. Recent
evidence suggests that severe SGA is associated with an increased risk of childhood mortality beyond the
neonatal period, mainly from infection and neurologic disease.
Suggested Reading(s)
American Academy of Pediatrics. Neonatology for Primary Care. American Academy of Pediatrics; 2020.
American Academy of Pediatrics. PCEP Book 1: Maternal and Fetal Evaluation and Immediate Newborn
Care. American Academy of Pediatrics; 2021.
Ludvigsson JF, Lu D, Hammarström L, Cnattingius S, Fang F. Small for gestational age and risk of
childhood mortality: a Swedish population study. PLoS Med. 2018;15(12):e1002717.
doi:10.1371/journal.pmed.1002717
Nafday SM, Abnormalities of fetal growth.In: McInerny TK, Adam HM, Campbell DE, DeWitt TG, Foy JM,
Kamat DM, eds. American Academy of Pediatrics Textbook of Pediatric Care. 2nd ed. American
Academy of Pediatrics; 2017:chap 98. Pediatric Care Online
Content Domain
Neonatology
A neonate born at 40 weeks’ gestation, delivered by emergency cesarean section for umbilical cord prolapse,
is being resuscitated. He was placed under the warmer and has a secure airway in place. Despite positive
pressure ventilation and appropriate ventilation corrective maneuvers, his heart rate remains at 52
beats/min. Chest compressions are performed, and after 1 minute his heart rate is 48 beats/min.
Correct answer is B
PREP Pearl(s)
When positive pressure ventilation does not improve a neonate’s cardiorespiratory status, the MR
SOPA corrective steps should be applied (Mask adjustment, Reposition the head and neck, Suction the
mouth and nose, Open the mouth, Pressure increase, and Alternative airway).
If a neonate is not responding to chest compressions, evaluation of the quality of the corrective steps
can be undertaken using the mnemonic CARDIO (Chest movement, Airway, Rate, Depth, and Inspired
Oxygen).
Critique
The newborn in the vignette, with a perinatal event of umbilical cord prolapse, requires cardiopulmonary
resuscitation. The best next step in this neonate’s management is to check that the fraction of inspired
oxygen is 100%. When a neonate does not respond to positive pressure ventilation (PPV) through a secured
airway, evaluation with ventilation corrective steps should be undertaken. These steps are known by the
acronym MR SOPA; they include Mask adjustment, Reposition the head and neck, Suction the mouth and
nose, Open the mouth, Pressure increase, and Alternative airway.
This neonate’s heart rate did not improve with either PPV or the ventilation corrective steps; therefore, chest
compressions were performed for 1 minute, after which the heart rate remained less than 60 beats/min. The
best next step is to make sure the fraction of inspired oxygen is 100%.
Suctioning of the nose and mouth are unlikely to improve the cardiorespiratory status of an intubated infant.
Epinephrine administration and umbilical catheter placement may be needed later in the care of this neonate
but are not the best next steps in his management. While continuing to administer chest compressions and
coordinated ventilation, the team should quickly assess the quality of ventilation and compressions using the
mnemonic CARDIO to ask the 5 questions below (Table).
Suggested Reading(s)
Hainstock LM, Raval GR. Neonatal resuscitation. Pediatr Rev. 2020;41(3):155-158. doi:10.1542/pir.2018-
0203
Solevåg AL, Cheung P-Y, Schmölzer GM. Chest compressions and ventilation in delivery room
resuscitation. Neoreviews. 2014;15(9):e396–e402. doi:10.1542/neo.15-9-e396
Weiner GM, Zaichkin J, eds. Textbook of Neonatal Resuscitation. 8th ed. American Academy of
Pediatrics; 2021.
Weiner GM, Zaichkin J. Updates for the Neonatal Resuscitation Program and resuscitation guidelines.
Neoreviews. 2022;23(4):e238-e249. doi:10.1542/neo.23-4-e238
Content Domain
Neonatology
The correct answer is: check that the fraction of inspired oxygen is 100%
Table. Questions to Ask When the Heart Rate Is Not Improving With
Appropriate Compressions and Ventilation (Mnemonic CARDIO).
1. Chest movement: Is the chest moving with each breath?
2. Airway: Is the airway secured with an endotracheal tube or laryngeal mask?
3. Rate: Are 3 compressions coordinated with 1 ventilation being delivered every 2
seconds?
4. Depth: Is the depth of compressions one-third of the anteroposterior diameter of
the chest?
5. Inspired oxygen: Is 100% oxygen being administered through the positive pressure
ventilation device?
Reprinted with permission from Weiner GM, Zaichkin J, eds. Textbook of Neonatal
Resuscitation. 8th ed. American Academy of Pediatrics; 2021.
AAP PREP 2024 - Question 151/267 Neonatology Question 6/13
A 4-day-old girl born at 36 weeks’ gestation is seen in the emergency department for worsening jaundice.
Maternal blood type is B positive, and her group B Streptococcus screen results were negative. This is the
mother's first child. The neonate was discharged home 48 hours after birth. Her birth weight was 2,750 g; the
discharge weight is unknown. Her total bilirubin level at discharge was 11 mg/dL (188.14 µmol/L). She has
been breastfeeding every 2 hours at home. Her last stool was in the hospital just before discharge, and her
last wet diaper was approximately 10 hours ago. Her weight in the emergency department is 12% below birth
weight. On physical examination, the neonate is sleepy but arousable. She is jaundiced, her anterior
fontanelle is depressed, and she has dry mucous membranes. The remainder of her examination findings
are normal.
Results of laboratory tests obtained in the emergency department are shown:
B. discontinue breastfeeding
Correct answer is C
PREP Pearl(s)
Late preterm neonates are at increased risk for physiologic jaundice compared with those born at
term, especially if they are exclusively breastfeeding.
Breastfeeding support, including lactation consultation, should be provided to breastfeeding mothers
to prevent neonatal dehydration and weight loss, which can worsen jaundice.
Late preterm neonates are at risk for neurotoxicity at a lower bilirubin level compared with term
neonates.
Critique
The best next step in the management of the neonate in the vignette is to obtain a lactation consultation.
There are several physiologic factors that place this late preterm infant at increased risk of jaundice, including
red blood cell life span (85 days in newborns compared with 120 days in adults), hemoglobin concentration,
immaturity of the conjugation and excretion bilirubin enzyme pathway in the liver, and increased
enterohepatic circulation. This neonate’s weight loss, lack of stooling since discharge, and minimal urine
output suggest that there is delayed production of breast milk, which can lead to dehydration and
breastfeeding jaundice and increases the risk of significant hyperbilirubinemia.
Rehydration and phototherapy (based on the 2022 American Academy of Pediatrics hyperbilirubinemia
guidelines) are critical steps in the initial management of this neonate. It is important to provide lactation
support to help the mother with successful breastfeeding and pumping (if indicated) and facilitate adequate
production of milk.
There is no indication for this dyad to discontinue breastfeeding. Bottle feeding with pumped breast milk or
formula supplementation may be recommended until the neonate demonstrates weight gain. There is no
evidence of a hemolytic process; therefore, intravenous immunoglobulin is not indicated. Exchange
transfusion is not appropriate for this neonate; she does not have hemolytic disease, and her total bilirubin
level is well below the level at which exchange transfusion is indicated.
Term neonates’ bilirubin levels peak at approximately 5 days of age, whereas late preterm neonates’ bilirubin
levels peak at approximately 7 days of age. Jaundice lasts longer in preterm compared with term neonates.
Late preterm neonates are at risk for neurotoxicity at a lower bilirubin level compared with term neonates.
Suggested Reading(s)
Anderson NB, Calkins KL. Neonatal indirect hyperbilirubinemia. Neoreviews. 2020;21(11):e749–e760.
doi:10.1542/neo.21-11-e749
Kemper AR, Newman TB, Slaughter JL, et al. Clinical practice guideline revision: management of
hyperbilirubinemia in the newborn infant 35 or more weeks of gestation. Pediatrics.
2022;150(3):e2022058859. doi:10.1542/peds.2022-058859
Pan DH, Rivas Y. Jaundice: newborn to age 2 months. Pediatr Rev. 2017;38(11):499-510.
doi:10.1542/pir.2015-0132
Pillai A, Pandita A, Osiovich H, Manhas D. Pathogenesis and management of indirect
hyperbilirubinemia in preterm neonates less than 35 weeks: moving toward a standardized approach.
Neoreviews. 2020;21(5):e298-e307. doi:10.1542/neo.21-5-e298
Slaughter JL, Kemper AR, Newman TB. Technical report: diagnosis and management of
hyperbilirubinemia in the newborn infant 35 or more weeks of gestation. Pediatrics.
2022;150(3):e2022058865. doi:10.1542/peds.2022-058865
Content Domain
Neonatology
In the delivery room, at 5 minutes after birth, a 28 weeks’ gestation newborn who is wrapped in a
polyethylene bag is receiving continuous positive pressure ventilation of 5 cm H2O with a fraction of inspired
oxygen of 70%. He is pink and well perfused, with a heart rate of 135 beats/min and flexion of all extremities.
Correct answer is D
PREP Pearl(s)
Premature newborns require thermal, cardiorespiratory, and metabolic support in the delivery room.
Premature newborns frequently need resuscitation for transition at birth.
The current Neonatal Resuscitation Program guidelines recommend initiation of resuscitation of
preterm newborns (<35 weeks’ gestational age) with oxygen at 21% to 30% fraction of inspired oxygen
using a pulse oximeter and oxygen blender to maintain oxygen saturation within the target range.
Critique
Preterm infants, especially those born at less than 32 weeks’ gestation, usually need assistance with
transition at birth and are more likely to require resuscitation. Because of their anatomical and physiologic
immaturity, they need not only cardiorespiratory support but also thermal and metabolic support. This
neonate, who has been appropriately wrapped in a polyethene bag to prevent heat loss, has a stable
cardiorespiratory status. The best next step in his management is to wean the fraction of inspired oxygen
using a target saturation measured by pulse oximetry. Sensors should be placed on the right hand or wrist to
assess the preductal saturation.
During newborn resuscitation, the need to correct low oxygen saturation must be balanced against the risks
of exposure to high levels of oxygen. Exposure to high levels of oxygen when perfusion has been restored
can result in reperfusion injury, especially in preterm neonates, due to the transition from a low-oxygen fetal
environment. The current Neonatal Resuscitation Program guidelines recommend initiation of resuscitation
of preterm newborns (<35 weeks’ gestational age) with administration of oxygen at 21% to 30% fraction of
inspired oxygen using a pulse oximeter and oxygen blender to maintain oxygen saturation within the target
range (Table). Placing an umbilical venous catheter, obtaining an arterial blood gas measurement, and
obtaining a blood glucose level are important aspects of care, but those procedures can take place once the
inspired fraction of oxygen has been weaned.
Oxygen saturation
Time Point Target
1 min 60%-65%
2 min 65%-70%
3 min 70%-75%
4 min 75%-80%
5 min 80%-85%
10 min 85%-95%
Reprinted with permission from Weiner GM, Zaichkin J. Textbook of Neonatal Resuscitation. 8th ed. American
Academy of Pediatrics; 2021.
Suggested Reading(s)
Jackson K, Harrington JW. SGA and VLBW infants: outcomes and care. Pediatr Rev. 2018;39(7):375-377.
doi:10.1542/pir.2017-0091
Nafday SM, Long CM. Respiratory distress and breathing issues in the newborn. In: McInerny TK, Adam
HM, Campbell DE, DeWitt TG, Foy JM, Kamat DM, eds. American Academy of Pediatrics Textbook of
Pediatric Care. American Academy of Pediatrics; 2022:chap 100. Accessed September 2022. Pediatric
Care Online
Weiner GM, Zaichkin J. Textbook of Neonatal Resuscitation. 8th ed. American Academy of Pediatrics;
2021.
Weiner GM, Zaichkin J. Updates for the Neonatal Resuscitation Program and resuscitation guidelines.
Neoreviews. 2022;23(4):e238-e249. doi:10.1542/neo.23-4-e238
Content Domain
Neonatology
A newborn is delivered by emergency cesarean section at 38 weeks’ gestation because of umbilical cord
prolapse and fetal distress. Her Apgar scores are shown:
1 min 1
5 min 1 1 2
10 min 2 1 2
The neonate is receiving respiratory support through continuous positive pressure of 6 cm H2O, with a
fractionated inspired oxygen of 25%. At 30 minutes after birth, her oxygen saturation is 97%, and an arterial
blood gas shows a pH of 6.89 with a base deficit of 25. The newborn is lethargic with generalized hypotonia
and a weak suck. She is admitted to the neonatal intensive care unit for therapeutic hypothermia.
C. tachycardia
D. thrombocytosis
Correct answer is A
PREP Pearl(s)
Therapeutic hypothermia is the standard of care for newborns born at 36 weeks’ gestation or later with
moderate or severe hypoxic ischemic encephalopathy to decrease adverse long-term
neurodevelopmental sequelae.
Newborns with hypoxic ischemic encephalopathy may have multiorgan dysfunction.
Critique
The neonate in the vignette is most likely to have an elevated creatinine level. The neonate experienced
umbilical cord prolapse with fetal distress and now has perinatal acidosis and clinical findings consistent with
moderate encephalopathy; these findings support a diagnosis of hypoxic ischemic encephalopathy (HIE).
Therapeutic hypothermia (whole body or selective head cooling) is now the standard of care for newborns
born at 36 weeks’ gestation or later with moderate or severe HIE to decrease adverse long-term
neurodevelopmental sequelae. Therapeutic hypothermia should be initiated within 6 hours of birth.
Neonates with HIE may have multiorgan dysfunction due to preferential blood flow to the vital organs (brain
and heart), at the expense of nonvital organs, during periods of hypoxia (diving reflex). This dysfunction leads
to hypoxemia and hypoperfusion of the nonvital organs. The resulting organ dysfunction may include
cardiopulmonary effects (hypotension, myocardial dysfunction, or pulmonary hypertension), renal effects
(acute kidney injury), hepatic effects (elevated liver enzymes or coagulopathy), and thrombocytopenia
(decreased production and increased destruction). Acute kidney injury leads to oliguria (urine output <1
mL/kg/h) or anuria, electrolyte abnormalities, and elevated blood urea nitrogen and creatinine levels. In most
cases, renal function improves in a few days.
This neonate is undergoing therapeutic hypothermia, which leads to bradycardia rather than tachycardia.
Neonates with HIE are more likely to have thrombocytopenia than thrombocytosis; transient
thrombocytopenia can also occur in patients undergoing therapeutic hypothermia. Neonates with HIE
receiving drugs that are cleared through the kidneys will usually have a decreased rate of drug clearance
rather than increased due to acute kidney injury; drug dosing adjustments may be needed. Therapeutic
hypothermia can also decrease drug clearance rates by slowing the kinetics of the drug metabolism
pathways.
Suggested Reading(s)
Bhagat I, Sarkar S. Multiple organ dysfunction during therapeutic cooling of asphyxiated infants.
Neoreviews. 2019;20(11):e653-e660. doi:10.1542/neo.20-11-e653
Chalak LF. Perinatal asphyxia in the delivery room: initial management and current cooling guidelines.
Neoreviews. 2016;17(8):e463-e470. doi:10.1542/neo.17-8-e463
Committee on Fetus and Newborn. Hypothermia and neonatal encephalopathy. Pediatrics.
2014;133(6):1146-1150. doi:10.1542/peds.2014-0899
Perez JMR, Feldman A, Alpan G. Treating hypoxic ischemic encephalopathy with hypothermia.
Neoreviews. 2015;16(7):e413-e419. doi:10.1542/neo.16-7-e413
Polglase GR, Ong T, Hillman NH. Cardiovascular alterations and multiorgan dysfunction after birth
asphyxia. Clin Perinatol. 2016;43(3):469-483. doi:10.1016/j.clp.2016.04.006
Content Domain
Neonatology
A neonate born at 41 weeks’ gestation is delivered precipitously. There is meconium-stained amniotic fluid.
Routine delivery room care is provided. Thirty minutes after birth, the neonate developed respiratory distress
with grunting, retractions, and an oxygen saturation of 88% in room air. A chest radiograph is obtained
(Figure 1). Oxygen and intravenous fluid support are provided.
Reprinted with permission from Warren JB, Anderson JM. Pediatr Rev. 2010;31(12):492.
C. prostaglandin infusion
D. renal ultrasonography
Correct answer is B
PREP Pearl(s)
There is no indication for tracheal suctioning of newborns (vigorous or nonvigorous) with meconium-
stained amniotic fluid.
Newborns with meconium aspiration syndrome are at risk for pneumothorax, persistent pulmonary
hypertension, and hemodynamic instability.
The decreased incidence of meconium-stained amniotic fluid is in large part due to the practice of
ensuring delivery by 41 weeks’ gestation.
Critique
The neonate’s respiratory distress, history of meconium-stained amniotic fluid (MSAF), and chest
radiographic findings of patchy atelectasis with areas of overinflation (air trapping), with no other explanation
for his condition, are consistent with a diagnosis of meconium aspiration syndrome (MAS). The presentation
of MAS in neonates varies from mild respiratory distress to severe and life-threatening illness. Neonates with
MAS are at increased risk of pneumothorax, which may require placement of a chest tube. Affected neonates
may develop significant persistent pulmonary hypertension with hemodynamic instability. Routine
laryngoscopy with tracheal suction, immediately after delivery, is no longer recommended for nonvigorous
newborns with MSAF; studies have found no benefit in the prevention of MAS, and this practice may delay
resuscitation.
Neonates with mild MAS require some level of respiratory support for several days in a neonatal intensive
care unit. More severe cases require mechanical ventilation and may require inhaled nitric oxide for
pulmonary vasodilatation and hemodynamic support. Infants with severe MAS may need extracorporeal
membrane oxygenation (ECMO) support. The risk of MAS markedly increases as gestational age advances
beyond 39 weeks. The incidence of MSAF has decreased, in a large part, due to the practice of ensuring
delivery by 41 weeks’ gestation.
Transient tachypnea of the newborn (TTN) occurs when transient pulmonary edema results from delayed
clearance of fetal lung fluid. In most instances, the respiratory distress resolves within a few hours generally
requiring oxygen support for 6 to 24 hours. Affected neonates rarely require mechanical ventilation. Chest
radiographic findings include prominent perihilar streaking, increased interstitial markings, fluid in the
interlobar fissures, and mild cardiomegaly (Figure 2). The differential diagnosis for TTN includes respiratory
distress syndrome (RDS), which results from surfactant deficiency. This syndrome usually occurs in preterm
neonates but can occur in term newborns. The treatment for RDS is surfactant replacement.
Reprinted with permission from Warren JB, Anderson JM. Newborn respiratory disorders. Pediatr Rev.
2010;31(12):491.
Figure 2. Transient tachypnea of the newborn. Note the increased interstitial markings and fluid in the
interlobar fissure on the right (arrow).
Treatment with a prostaglandin infusion should be considered when a ductal-dependent congenital heart
condition is suspected or diagnosed. Newborns with MAS do not routinely need renal ultrasonography
because there is no association with renal anomalies. Renal ultrasonography would be indicated in a
newborn with suspected pulmonary hypoplasia. Fetal urine contributes to amniotic fluid production, which is
needed for fetal lung development.
Suggested Reading(s)
Hagen E, Chu A, Lew C. Transient tachypnea of the newborn. Neoreviews. 2017;18(3):e141-e148.
doi:10.1542/neo.18-3-e141
Meerkov M, Weiner G. Management of the meconium-stained newborn. Neoreviews. 2016;17(8):e471-
e477. doi:10.1542/neo.17-8-e471
Reuter S, Moser C, Baack M. Respiratory distress in the newborn. Pediatr Rev. 2014;35(10):417-429.
doi:10.1542/pir.35-10-417
Warren JB, Anderson JM. Newborn respiratory disorders. Pediatr Rev. 2010;31(12):487-496.
doi:10.1542/pir.31-12-487
Weiner GM, Zaichkin J. Updates for the neonatal resuscitation program and resuscitation guidelines.
Neoreviews. 2022;23(4):e238-e249. doi:10.1542/neo.23-4-e238
Content Domain
Neonatology
The mother of a child in the practice is pregnant with a 30-weeks’-gestation fetus diagnosed this week with
congenital diaphragmatic hernia. Preparations are being made for her to deliver at a nearby tertiary care
center with a specialty team to care for the baby. She asks about the long-term prognosis for the infant.
Of the following, the factor MOST associated with this child’s risk of neurodevelopmental problems at age 2
years is a/an
Correct answer is D
PREP Pearl(s)
Most congenital diaphragmatic hernias are diagnosed via prenatal ultrasonography. On diagnosis,
arrangements should be made for delivery in a tertiary care center.
Congenital diaphragmatic hernias diagnosed before 25 weeks’ gestation and those associated with
transposition of the liver into the thoracic cavity confer a very high mortality risk.
Children born with congenital diaphragmatic hernia require carefully coordinated multidisciplinary
follow-up to monitor for and manage the potential complications and comorbidities.
Critique
The fetus described in the vignette has congenital diaphragmatic hernia (CDH). Of the response choices,
thoracic transposition of the liver is most associated with neurodevelopmental problems at age 2 years in
children with CDH. Long-term follow-up studies have shown that neonates born with CDH are at greatest risk
of experiencing neurodevelopmental problems at age 2 years if
The hernia is right-sided, particularly if the liver has transposed into the chest
Extracorporeal membrane oxygenation is needed early in care
A synthetic patch was required for the repair
The child’s supplemental oxygen requirement persists at age 30 days
Associated skeletal and chest wall anomalies are common with CDH. They contribute to overall morbidity but
not to neurodevelopmental prognosis. The current standard of care for neonates with CDH is stabilization,
gentle ventilation to protect the hypoplastic lungs from barotrauma, and delayed elective diaphragmatic
hernia repair. Because of initiation of this standard of care (in the 1990s), overall mortality and morbidity
resulting from CDH decreased significantly (Figure).
Congenital diaphragmatic hernia occurs in about 1:3,000 live births; 75% to 85% are on the left side. The most
common associated anomalies are gut malrotation; omphalocele; congenital heart defects; trisomy 13, 18,
and 21; and structural chest wall deformities. Most cases of CDH are diagnosed via prenatal ultrasonography;
prenatal magnetic resonance imaging is sometimes used to confirm and delineate the extent of the
herniation. Prenatal diagnosis allows for obstetric and delivery management in a tertiary care center that is
capable of meeting the complex needs of neonates with CDH.
The management of CDH includes immediate intubation, low-pressure ventilation, gut decompression to
minimize expansion of the bowel within the thoracic cavity, placement of intravenous lines for fluid and
nutritional management, and delayed repair of the diaphragmatic defect. Recent evidence supports a trial of
spontaneous breathing with supplemental oxygen or continuous positive airway pressure as needed, rather
than immediate intubation. Delayed intubation for neonates with CDH who are not able to support
respiration independently does not appear to increase morbidity.
For the neonate with CDH not diagnosed prenatally, immediate recognition of respiratory distress and
intervention to promote adequate ventilation is critical. Chest radiography should be performed to confirm
the diagnosis. The gut should be decompressed to minimize intrathoracic expansion and help prevent
barotrauma to the hypoplastic lungs. Prompt transfer to a tertiary care center is indicated for ongoing
management and evaluation for other anomalies and comorbidities. The lack of prenatal preparation triggers
a highly stressful environment for the neonate, family, and caregivers, and it increases the risk of adverse
outcomes.
In utero repair of the diaphragmatic defect has been attempted with variable success. Because of the high
complication rate, in utero repair should be considered only for fetuses who have a high risk of experiencing
early mortality and for whom the potential benefit for survival outweighs the risk of intervention.
The overall mortality risk is about 50% in fetuses diagnosed with CDH at or before 25 weeks’ gestation;
herniation of the liver into the thorax and a low lung-to-head ratio portend a much worse outcome. If the
ratio of lung size (contralateral to the defect) to head size is less than 0.6, mortality approaches 100%; if the
ratio is more than 1.35, survival is almost 100%.
Risk factors for poor survival in neonates diagnosed with CDH later than 25 weeks’ gestation include the
following:
A prediction scale for mortality in this population, the Congenital Diaphragmatic Hernia Study Group rule,
includes both postnatal and prenatal factors. The score predicts low, medium, or high risk of death in the
newborn period. Factors considered include the following:
Using one point for the presence of each factor, a higher score and the presence of more predictive factors is
associated with a lower chance of survival (Table).
All infants with CDH benefit from long-term follow-up by a coordinated multidisciplinary team to monitor
pulmonary and neurocognitive development and to manage associated anomalies, feeding difficulties, and
hearing problems. Lung function is low in all infants with CDH; it often normalizes by age 2 years. Affected
children are at increased risk of developing asthma, recurrent respiratory infections, and ongoing pulmonary
hypertension. All infants with left-sided CDH have gastroesophageal reflux, which is associated with feeding
problems and poor growth and complicates respiratory management. Most children with CDH have learning
disabilities at school age. One-third of children with CDH have conductive hearing loss, and about 3% have
sensorineural hearing loss (consistent with the incidence in children who received care in a neonatal
intensive care unit).
Suggested Reading(s)
American Academy of Pediatrics. Postdischarge follow-up of infants with congenital diaphragmatic
hernia. In: Neonatal Care: A Compendium of AAP Clinical Practice Guidelines and Policies. American
Academy of Pediatrics; 2019:311-316.
Cochius-den Otter SCA, Erdem O, van Rosmalen J, et al. Validation of a prediction rule for mortality in
congenital diaphragmatic hernia. Pediatrics. 2020;145(4):e20192379. doi:10.1542/peds.2019-2379
Cochius-den Otter SCM, Horn-Oudshoorn E, Allegaert K, et al. Routine intubation in newborns with
congenital diaphragmatic hernia. Pediatrics. 2020;146(4):e20201258. doi:10.1542/peds.2020-1258
Deprest JA, Nicolaides KH, Benachi A, et al. Randomized trial of fetal surgery for severe left
diaphragmatic hernia. N Engl J Med. 2021;385(2):107-118. doi:10.1056/NEJMoo2027030
Gurson SC, Campbell DE. Health and developmental outcomes of selected medically complex
neonates. In Campbell DE, ed. Neonatology for Primary Care. American Academy of Pediatrics;
2021:chap 42. Neonatology for Primary Care
Content Domain
Neonatology
Abbreviations: E1-E5, era 1 to era 5; O:E, observed to expected. Gupta VS, Harting MT, Lally PA, et al. Has survival improved for congenital diaphragmatic hernia? A 25-year review of over 5000 patients
from the CDH study group. Pediatrics. 2021;147(3_MeetingAbstract):939-940.
Figure: (A) Average mortality of centers by year. Data points represent mean + standard error of the mean. Overall mortality by era (E1-E5) is listed and separated
by grey hash lines. (B) Fit plot of O:E mortality ratio using 1995-2000 as a reference and adjusted for birth weight, Apgar at 5 minutes, presence of a major cardiac
anomaly, prenatal diagnosis, and center volume. O:E ratio of 1 is denoted by the grey hash line.
A 40 weeks’ gestation neonate is seen in the newborn nursery. The pregnancy and delivery were
uncomplicated; his Apgar scores were 8 at 1 minute and 9 at 5 minutes. Thirty hours after birth the neonate
is very jittery and has a generalized tonic-clonic seizure. The seizure responds to treatment with an
antiepileptic medication. Evaluation of the seizure is undertaken.
A. ibuprofen
B. levothyroxine
C. paroxetine
D. propylthiouracil
Correct answer is C
PREP Pearl(s)
The use of selective serotonin reuptake inhibitors to treat depression during pregnancy has increased.
Maternal use of selective serotonin reuptake inhibitors during pregnancy can result in neonatal
abstinence syndrome and encephalopathy.
Maternal medication use during pregnancy should be considered in the differential diagnosis of
unexplained newborn seizures.
Critique
The use of selective serotonin reuptake inhibitors (SSRIs) during pregnancy has increased due to increasing
recognition and treatment of depression. Selective serotonin reuptake inhibitors can cross the placenta;
there is evidence that their use in the third trimester of pregnancy can result in symptoms and signs of
neonatal abstinence syndrome (NAS), including seizures due to neurologic dysfunction. Maternal SSRI use
should be considered as a cause of unexplained newborn seizures. Signs and symptoms of neonatal SSRI
withdrawal are most commonly reported with the prenatal use of paroxetine and fluoxetine. The withdrawal
effects are due to the cholinergic and serotonin-induced actions, which regulate cardiovascular, respiratory,
and cerebral functions.
Neonatal encephalopathy, a clinical syndrome of neurologic dysfunction, can range from mild irritability and
feeding difficulties to seizures and coma. The most common cause of neonatal encephalopathy in the first 24
hours after birth is hypoxic ischemic encephalopathy (HIE). The differential diagnosis of neonatal
encephalopathy is broad, including infection, vascular/perfusion-related, toxins/medication-related,
metabolic, epileptic, and genetic (Table).
Ibuprofen, propylthiouracil, and levothyroxine do not cause neonatal encephalopathy or seizures. Ibuprofen
use in later stages of pregnancy can result in premature closure of the ductus arteriosus and has been
associated with idiopathic persistent pulmonary hypertension in the newborn.
Suggested Reading(s)
Ayed M, Embaireeg A, Ayed A. A full-term infant with seizures. NeoReviews. 2021;22(10):e696-e698.
doi:10.1542/neo.22-10-e696
Bondi DS, Khan OA, Hageman J. Pharmacology review: maternal selective serotonin reuptake inhibitor
use and neurologic effects on the neonate. NeoReviews. 2016;17(7):e356-e366. doi:10.1542/neo.17-7-
e356
Rodriguez-Kovacs J, Tien SH, Plunkett BA. Selective serotonin reuptake inhibitor use in pregnancy:
repercussions on the oblivious passenger. NeoReviews. 2018;19(3):e160-e169. doi:10.1542/neo.19-3-
e160
Russ JB, Simmons R, Glass HC. Neonatal encephalopathy: beyond hypoxic-ischemic encephalopathy.
NeoReviews. 2021;22(3):e148-e162. doi:10.1542/neo.22-3-e148
Content Domain
Neonatology
At the time of delivery, a 41 weeks’ gestation newborn was noted to have meconium-stained amniotic fluid
and was covered with thick meconium. The neonate was vigorous and crying, with a normal tone. Apgar
scores were 8 (1 off each for color and tone) at 1 minute and 9 (1 off for color) at 5 minutes after birth.
Routine newborn care was provided. One hour after delivery, the newborn developed respiratory distress
with tachypnea, subcostal retractions, and an oxygen saturation of 85% in room air. When nasal cannula flow
of 2 L/min is provided with inspired oxygen of 70%, the oxygen saturation improves to 89%. A chest
radiograph is obtained (Figure).
Reprinted with permission from Stein A, Molero H, Hess D, Luquette M, Pitt MB. Pediatr Rev.
2020;41(11):e43
Of the following, the BEST next step in the management of this newborn is
B. endotracheal intubation
C. intravenous antibiotics
D. needle thoracentesis
Correct answer is D
PREP Pearl(s)
One percent of well, term newborns will develop non–clinically significant spontaneous pneumothorax
which resolves without treatment. Newborns with lung disease and those receiving positive-pressure
ventilation are at risk for clinically significant pneumothorax.
Pneumothorax should be considered in a newborn receiving positive-pressure ventilation who has
sudden clinical deterioration with cardiorespiratory compromise.
Needle thoracentesis/aspiration may be urgently needed when there is cardiorespiratory compromise
from a tension pneumothorax.
Critique
Of the response choices, the best next step in the management of the neonate in the vignette is needle
thoracentesis. The neonate’s chest radiograph shows a tension pneumothorax, with collapse of the right lung
and the heart shifted toward the left side; therefore, the air in the right chest must be evacuated urgently in
order to prevent further respiratory compromise. The fastest way to do this is through needle thoracentesis,
immediately relieving the tension pneumothorax and improving ventilation. This will help stabilize the
neonate’s condition, allowing time to place a chest tube and determine the ventilatory support needed, which
may range from noninvasive support to endotracheal intubation and mechanical ventilation.
With a history of meconium-stained amniotic fluid, this neonate most likely has meconium aspiration
syndrome (MAS), resulting in partial blockage of the airways leading to a pneumothorax. While placement of
a chest tube will be needed for continued evacuation of the pneumothorax, this should be undertaken after
the neonate’s respiratory status has been stabilized.
The neonate in the vignette, with meconium aspiration, is at risk for aspiration pneumonia. While antibiotics
may be considered, they would not be administered before the newborn’s condition is stabilized.
Approximately 1% of well, term newborns will develop non–clinically significant spontaneous pneumothorax
which resolves without treatment. The newborns most at risk for clinically significant pneumothorax include
those with the following:
A pneumothorax should be considered in any neonate receiving positive-pressure ventilation who has
sudden clinical deterioration. The DOPE mnemonic (Displacement of endotracheal tube, Obstruction of
airway/endotracheal tube, Pneumothorax, Equipment failure) is a helpful tool for assessing an intubated
neonate for inadequate ventilation. If a pneumothorax is suspected, it may be necessary to quickly evacuate
the air prior to obtaining a chest radiograph. Transillumination of the neonate’s chest may be helpful, but if
the infant’s clinical condition is unstable or deteriorating, insertion of a 20- or 18-gauge intravenous catheter
into the infant’s chest and evacuating a pneumothorax should not be delayed. The procedure is performed,
on the side of the pneumothorax, by inserting the needle in the second intercostal space in the midclavicular
line (Leibel and Vachharajani, 2014). In an emergency situation, if the needle does not aspirate air, the
procedure should be performed on the opposite side as well.
Suggested Reading(s)
Leibel S, Vachharajani A. Needle aspiration of the pneumothorax. NeoReviews. 2014;15(4):e163-e165.
doi:10.1542/neo.15-4-e163
Respiratory distress. In: Sinkin RA, Chisholm CA, eds. PCEP Book 3: Neonatal Care Book. 4th ed.
American ACademy of Pediatrics; 2021:69-119.
Stein A, Molero H, Hess D, Luquette M, Pitt MB. Visual diagnosis: tension pneumothorax with evolving
cysts in an infant. Pediatr Rev. 2020;41(11):e42-e45. doi:10.1542/pir.2018-0062
Weiner GM, Zaichkin J; Updates for the Neonatal Resuscitation Program and resuscitation guidelines.
NeoReviews. 2022;23(4):e238-e249. doi:10.1542/neo.23-4-e238
Content Domain
Neonatology
A newborn is being evaluated in the normal neonatal nursery. He is noted to have an extra digit on the radial
aspect of his left hand. His vital signs are within normal limits, and the remainder of his physical examination
findings are normal.
A. CHARGE syndrome
B. Noonan syndrome
C. trisomy 18
D. VACTERL association
Correct answer is D
PREP Pearl(s)
Referral to a pediatric hand surgeon is indicated for postaxial (ulnar) polydactyly with bone, ligament,
or joint involvement and for all preaxial (radial) digits.
Conditions that have associated polydactyly include trisomy 13, Meckel syndrome, Diamond-Blackfan
anemia, Fanconi anemia, and VACTERL association.
Critique
Among the response choices, VACTERL association—vertebral defects, anal atresia, cardiac defects,
tracheoesophageal fistula, renal anomalies, and limb abnormalities—is the only one that includes
polydactyly. Neonates with polydactyly account for 1% to 2% of all newborns with a congenital abnormality;
of those, 10% involve the hands. The extra digit is most often composed of skin and soft tissue; only rarely is
there a functioning additional digit.
A preaxial (radial) digit, an additional digit on the same side as the thumb, is most often the result of a
spontaneous genetic variant. In some cases there is a split thumb, where neither digit is fully formed or fully
functional. A postaxial (ulnar) digit, or additional digit adjacent to the 5th digit, has many forms of postaxial
polydactyly with various inheritance patterns; many are spontaneous variants.
The first step in the evaluation of polydactyly is to obtain a plain radiograph to evaluate for any bone within
the digit. All preaxial digits and postaxial digits with bone, ligament, or joint involvement require evaluation
by a pediatric hand surgeon to ensure optimal function. Postaxial digits with no bone formation (only skin
and soft tissue) can be treated in the clinic or office shortly after discharge from the newborn nursery by
using a sharp excision or suture ligation technique with local anesthesia. Suture ligation involves tying a
suture tightly around the additional digit to induce necrosis of the portion distal to the suture, followed by
autoamputation within approximately 10 to 20 days. An important consideration when selecting a technique
is that suture ligation has a higher risk of developing a painful neuroma compared with sharp excision. For
those requiring surgery (eg, preaxial digit, bone or ligament involvement), the procedure should be
performed before the child develops a pincer grasp to optimize fine motor development. After removal of the
less-developed digit, the surgeon will attach/reattach ligaments to the remaining digit in an attempt to leave
the child with a fully functional digit.
Several conditions have associated polydactyly, including trisomy 13, Meckel syndrome, Diamond-Blackfan
anemia, Fanconi anemia, and VACTERL association.
Suggested Reading(s)
Children with congenital hand anomalies and malformations. Healthychildren.org. Accessed February
22, 2023. https://www.healthychildren.org/English/health-issues/conditions/Cleft-
Craniofacial/Pages/Children-with-Congenital-Hand-Anomalies-Malformations.aspx
Rosen O, Marion RW, Samanich JM. Common congenital anomalies. In: McInerny TK, Adam HM,
Campbell DE, Foy JM, Kamat DM, eds. American Academy of Pediatrics Textbook of Pediatric Care. 2nd
ed. American Academy of Pediatrics; 2016:chap 96. Accessed November 15, 2023. Pediatric Care Online
Content Domain
Neonatology
A term neonate was born via vaginal delivery to a 30-year-old gravida 1, para 0 woman after an
uncomplicated pregnancy. His Apgar scores were 8 and 9 at 1 and 5 minutes, respectively. His initial physical
examination findings were normal. Twelve hours after birth, he develops episodes of apnea accompanied by
twitching in the right face and arm and is transferred to the neonatal intensive care unit.
C. perform electroencephalography
Correct answer is B
PREP Pearl(s)
Identification of the cause of neonatal seizures is crucial for appropriate treatment. Etiology is the most
important determinant of prognosis.
Most neonatal seizures are classified as acute provoked. Metabolic derangements (eg, hypoglycemia,
electrolyte abnormalities) are an important reversible minority of them. Hypoxic-ischemic
encephalopathy and stroke represent the majority of acute provoked neonatal seizures.
A high index of suspicion for neonatal seizures is crucial, owing to the often subtle nature of the clinical
manifestations.
Critique
The neonate in the vignette is experiencing right focal clonic seizures. His initial evaluation should focus on
rapid identification and correction of reversible causes that, if left untreated, could cause brain injury.
Hypoglycemia is a common treatable metabolic derangement encountered in the neonatal period. Blood
glucose measurement should be the first bedside diagnostic test performed in a neonate with new-onset
seizures. Neonates born to mothers with diabetes are recognized to be at increased risk of developing
hypoglycemia. However, other etiologies of hypoglycemia (eg, inborn errors of metabolism) also occur in the
neonatal period. Neonates with new-onset seizures should also be urgently evaluated for electrolyte
abnormalities that can cause seizures (eg, hyponatremia, hypocalcemia, hypomagnesemia).
Neonatal seizures are common, occurring in 1 to 5 per 1,000 live term births. The causes of neonatal seizures
can be classified as either acute provoked seizures or early-onset epilepsy (in the setting of a genetic
syndrome, brain malformation, or both). The underlying etiology of neonatal seizures is the most important
determinant of prognosis and guides evaluation and management.
Most neonatal seizures are classified as acute provoked. Of these, 70% are due to hypoxic-ischemic
encephalopathy or stroke (ischemic or hemorrhagic). Metabolic derangements (eg, hypoglycemia) account for
an important minority of acute provoked seizures and should be rapidly identified and treated.
Meningoencephalitis (bacterial or viral) should be considered in the neonatal period; early treatment is
critical.
Neonatal seizures are often clinically subtle, requiring a high index of suspicion. Owing to the unique
attributes of the immature neonatal brain, seizures are initially focal. Seizures are classified broadly into 2
clinical categories: electroclinical seizures and electrographic-only seizures. Electroclinical seizures can be
further subdivided into the subtypes motor (eg, automatisms, clonic, myoclonic, tonic, epileptic spasms) and
nonmotor (eg, autonomic, behavioral arrest). Electroencephalography (EEG) plays a critical role in the
diagnosis and detection of electrographic-only seizures, as well as in monitoring of treatment response.
Neonatal seizures are a neurological emergency. They require that clinical assessment, diagnostic
evaluations, and treatment occur simultaneously. Each of the response choices is an important component of
the evaluation of neonatal seizures. They are often performed in a rapid stepwise fashion in concert with
treatment initiation. Rapid bedside blood glucose testing is readily available, and the treatment of
hypoglycemia is urgent and easily implemented. Therefore, blood glucose measurement should be
performed before the other diagnostic testing options listed, if seizure is suspected. Electroencephalography
monitoring is a crucial study for the definitive diagnosis and management of seizures, and it should be
initiated as soon as possible. However, EEG is not available in all clinical settings. Amplitude-integrated EEG
can be performed when conventional EEG is unavailable. Head ultrasonography is a bedside imaging
modality that can provide immediate structural data. Sepsis evaluation, including blood, urine, and
cerebrospinal fluid culture, and empiric antimicrobial coverage is warranted in neonates experiencing
seizures.
Suggested Reading(s)
Glass HC, Shellhaas RA. Acute symptomatic seizures in neonates. Semin Pediatr Neurol.
2019;32:100768. doi:10.1016/j.spen.2019.08.004
Leone B, Yozawitz E. The newborn with neurologic findings. In: McInerny TK, Adam HM, Campbell DE,
DeWitt TG, Foy JM, Kamat DM, eds. American Academy of Pediatrics Textbook of Pediatric Care.
American Academy of Pediatrics; 2021:chap 107. Accessed September 1, 2023. Pediatric Care Online.
Olson D. Neonatal seizures. NeoReviews. 2012;13(4):e213-e223. doi:10.1542/neo.13-4-e213
Pavel AM, Rennie JM, de Vries LS, et al. Neonatal seizure management: is the timing of treatment
critical? J Pediatr. 2022;243:61-68.e2. doi:10.1016/j.jpeds.2021.09.058
Pressler RM, Cilio MR, Mizrahi EM, et al. The ILAE classification of seizures and the epilepsies:
modification for seizures in the neonate. Position paper by the ILAE Task Force on Neonatal Seizures.
Epilepsia. 2021;62(3):615-628. doi:10.1111/epi.16815
Content Domain
Neurology
A 3-year-old boy is evaluated in the emergency department for progressive difficulty with walking. Nine days
ago he developed fever, cough, and congestion; a nasal swab polymerase chain reaction test was positive for
severe acute respiratory syndrome coronavirus-2 (SARS-CoV-2). His symptoms were improving until today,
when he developed slowed speech and an inability to sit or walk without assistance. When sitting, his parents
report, he sways from side to side and has difficulty reaching for toys.
The boy’s vital signs are normal for age. On physical examination, he is drowsy but easily arousable. His
speech has a slow, scanning quality that is not typical for him. Cranial nerve, motor, and sensory examination
findings are normal when he is examined while in his mother’s lap. His reflexes are 2+ and symmetric
throughout, and his toes are downgoing to plantar stimulation. On coordination testing, he has difficulty
reaching for objects and demonstrates significant dysmetria. He is unsteady when he sits independently
(titubation). He is unable to stand or walk without assistance, and he has an unsteady, wide-based gait with a
lurching quality.
Results of a complete blood cell count, comprehensive metabolic panel, and urine and serum toxicology
screenings are normal. Findings on magnetic resonance imaging of the brain with and without contrast are
normal. Cerebrospinal fluid protein, glucose, and cell counts are normal..
Of the following, this boy’s MOST likely diagnosis is
C. Guillain-Barré syndrome
D. vestibular migraine
Correct answer is A
PREP Pearl(s)
Acute ataxia is defined as uncoordinated, voluntary motor movements that are present for less than 7
days. The initial evaluation of acute ataxia is focused on serious or life-threatening causes that fall into
4 broad categories: infectious or inflammatory, ingestion, neoplasm, and stroke.
Acute cerebellar ataxia is a postinfectious, self-limited condition characterized by the acute onset of
truncal and gait ataxia lasting hours to a few weeks. It is most commonly seen in young children.
Acute cerebellitis is a rare condition representing the severe end of the postinfectious acute cerebellar
ataxia spectrum. Affected children experience acute ataxia, headache, vomiting, and lethargy due to
severe cerebellar edema.
Critique
The most likely diagnosis for the boy in the vignette is acute cerebellar ataxia (ACA), a self-limited,
postinfectious syndrome characterized by rapid onset of ataxia with abnormalities in gait, speech, posture,
and coordination of voluntary movements. This condition is most common in children younger than 6 years.
It can occur after any viral illness, including severe acute respiratory syndrome coronavirus-2 (SARS-CoV-2)
infection. Findings of laboratory evaluation and neuroimaging studies are typically normal, and treatment is
supportive.
Ataxia refers to uncoordinated, clumsy, voluntary motor movements. Although ataxia can result from
pathology at various levels of the nervous system, it commonly occurs in the setting of cerebellar
dysfunction. Acute ataxia (present for <7 days) is an uncommon pediatric finding seen in a wide range of
conditions, from life-threatening to benign and self-limited. Select causes may include genetic, infectious,
neoplastic, psychogenic, traumatic or vascular conditions. The history, including the timeline and progression
of symptom onset, and physical examination findings guide the selection of appropriate diagnostic studies as
well as management (Figure). The initial evaluation of acute ataxia is aimed at identifying or ruling out serious
and/or life-threatening causes. These fall into 4 broad categories:
Infectious or inflammatory
Ingestion
Neoplasm
Stroke
Ataxia caused by ACA is typically most prominent in the morning and improves over the course of hours to
days. Children present with truncal and gait ataxia, often with refusal to walk. Systemic signs and symptoms
are usually absent in ACA; their presence warrants further evaluation. Acute cerebellitis is a rare condition
representing the severe end of the postinfectious ACA spectrum. Affected children experience acute ataxia,
headache, vomiting, and lethargy due to severe cerebellar edema.
Acute disseminated encephalomyelitis is a post infectious demyelinating disorder of the central nervous
system, preferentially affecting the white matter. The clinical presentation varies according to the areas
involved and may include encephalopathy, seizures, hemiparesis, ataxia, cranial neuropathies, myelopathies,
or a combination of these, often following a prodrome of fever, headache, vomiting, and meningismus. Brain
and spine magnetic resonance imaging demonstrates bilateral, asymmetric, and ill-defined hyperintensities
in the deep and subcortical white matter. Expert consensus recommends treatment of acute disseminated
encephalomyelitis with intravenous methylprednisolone in addition to targeted symptomatic treatment and
supportive care.
Suggested Reading(s)
Caffarelli M, Kimia AA, Torres AR. Acute ataxia in children: a review of the differential diagnosis and
evaluation in the emergency department. Pediatr Neurol. 2016;65:14-30.
doi:10.1016/j.pediatrneurol.2016.08.025
Overby P. Ataxia. In: McInerny TK, Adam HM, Campbell DE, Foy JM, Kamat DM, eds. American Academy
of Pediatrics Textbook of Pediatric Care. 2nd ed. American Academy of Pediatrics; 2016:chap 130.
Accessed September 1, 2023. Pediatric Care Online
Overby P, Kapklein M, Jacobson RI. Acute ataxia in children. Pediatr Rev. 2019;40(7):332-343.
doi:10.1542/pir.2017-0223
Content Domain
Neurology
Courtesy of J. Goldstein
Abbreviations: ADEM, acute disseminated encephalomyelitis; CSF cerebrospinal fluid.
FIGURE: Approach to evaluation of a child with acute ataxia.
AAP PREP 2024 - Question 161/267 Neurology Question 3/12
A 17-year-old adolescent girl is brought to the emergency department for evaluation of progressive
weakness. For the past year, she has been unable to fully close her eyes. By the end of the day, her eyelids
often droop and obscure her vision. She is having difficulty chewing and swallowing her dinner each night
and has lost about 4.5 kg over the last 3 months.
On neurological examination, the girl is awake, alert, and oriented. Her pupils are equal, round, and reactive
to light, and her extraocular movements are intact. She has significant ptosis, is unable to completely close
her eyes, and cannot resist attempts to open her eyelids. Her face is symmetric, but her mouth hangs open
and she is unable to completely close it. She cannot fully smile, and when she attempts to, her mouth
appears to have a snarling expression. She has normal muscle tone and 4/5 strength in her upper and lower
extremities. With repetitive testing, her strength deteriorates to 3/5 and she has increasing difficulty
performing the examination maneuvers. Her deep tendon reflexes are 2+ throughout, with toes downgoing
to plantar stimulation. Sensation is intact to light touch, pinprick, and vibration. Coordination and gait test
findings are normal. The remainder of her physical examination findings are unremarkable.
An ice pack placed on her eyelids for 2 minutes results in transient improvement in her ptosis.
A. brain and spine magnetic resonance imaging with and without contrast
Correct answer is B
PREP Pearl(s)
Autoimmune myasthenia gravis presents with fatigable muscle weakness involving the ocular, bulbar,
skeletal, and/or respiratory muscles. Autoantibodies against the postsynaptic acetylcholine receptors
and associated proteins disrupt neuromuscular junction transmission.
The diagnosis of autoimmune myasthenia gravis is supported by bedside tests (eg, ice-pack test),
serum antibody testing, and/or electrophysiology studies (electromyography with nerve conduction
studies and repetitive nerve stimulation).
Treatment of autoimmune myasthenia gravis has 4 components: symptomatic treatment with
acetylcholinesterase inhibitors, immunotherapy (glucocorticoids, steroid-sparing agents),
immunomodulating therapy (immunoglobulin, plasma exchange) for exacerbations, and surgical
thymectomy (for select individuals).
Critique
The adolescent girl in the vignette has significant fatigable, generalized muscle weakness with preserved
tone, reflexes, and sensation. Fatigable muscle weakness is the hallmark of neuromuscular junction
disorders, such as juvenile autoimmune myasthenia gravis (MG). Clinical bedside testing when ptosis is
present includes the ice pack test. When a neuromuscular junction disorder is present, an ice pack placed on
the eyelid for 2 minutes results in immediate and transient improvement in ptosis; this is due to improved
neuromuscular transmission at lower temperatures.
Electromyography (EMG) with nerve conduction studies (NCS) and repetitive nerve stimulation (RNS) are the
neurophysiologic studies of choice to confirm a suspected diagnosis of MG. Electromyographic findings are
frequently normal, and EMG is performed to rule out alternate causes of neuromuscular weakness. Single-
fiber EMG is the most sensitive electrophysiologic test for MG but is technically difficult and not widely
available. Nerve conduction studies with RNS demonstrate impairment in neuromuscular junction
transmission with a reduction in the compound muscle action potential (decremental response). Serum
testing for acetylcholine receptor antibodies is also indicated when MG is suspected. Antibodies are found in
up to 90% of individuals who have generalized symptoms. Serum antibody testing is often the first-line test
for suspected MG.
Although the clinical course of MG can be variable, the disease generally has 3 stages:
Stage 1: Active
Usually the first 5 to 7 years
Characterized by fluctuating symptoms with exacerbations and remissions
Highest risk for myasthenic crisis
Stage 2: Stable
Symptoms remain and are more stable
Exacerbations can occur, often when triggered by infection or medication changes
Stage 3: Remission
Relative reduction in symptoms and long-term remission can occur
Potential to wean off immunomodulating therapy
When a child or adolescent has fatigable, generalized muscle weakness, the most likely etiology is a
peripheral nervous system disorder, usually affecting the neuromuscular junction. Brain and spine magnetic
resonance imaging with and without contrast is the study of choice to evaluate the central nervous system,
so it is not the preferred test for diagnosing a likely peripheral nervous system disorder. Hypothyroidism and
hyperthyroidism can result in neuromuscular symptoms, such as generalized weakness, owing to an
underlying myopathy. Thyroid function tests are appropriate in the evaluation of an acquired myopathy.
However, the signs and symptoms in the girl in the vignette are not suggestive of myopathy. Neuromuscular
ultrasonography is an adjunctive tool to EMG and NCS in the diagnostic evaluation of a suspected myopathy
or neuropathy, but not for a suspected neuromuscular junction disorder.
Suggested Reading(s)
Narayanaswami P, Sanders DB, Wolfe G, et al. International consensus guidance for management of
myasthenia gravis: 2020 update. Neurology. 2021;96(3):114-122. doi:10.1212/wnl.0000000000011124
O’Connell K, Ramdas S, Palace J. Management of juvenile myasthenia gravis. Front Neurol. 2020;11:743.
doi:10.3389/fneur.2020.00743
Ozuah PO, Reznik M. Fatigue and weakness. In: McInerny TK, Adam HM, Campbell DE, DeWitt TG, Foy
JM, Kamat DM, eds. American Academy of Pediatrics Textbook of Pediatric Care. American Academy of
Pediatrics; 2023. Accessed September 1, 2023. Pediatric Care Online
Peragallo JH. Pediatric myasthenia gravis. Semin Pediatr Neurol. 2017;24(2):116-121.
doi:10.1016/j.spen.2017.04.003
Content Domain
Neurology
An 8-year-old boy is brought to the office for evaluation of abnormal movements. Over the past month, he
has developed random-appearing restless movements of his arms and trunk. The movements are now
occurring almost continuously when he is awake and disappear when he is asleep. He has also become more
irritable and has frequent mood swings.
His physical examination findings are significant for mild hypotonia with normal strength and reflexes. He
has continuous movements of his limbs. When asked to grip the examiner’s hand, he repeatedly grips,
partially releases, then grips again. The remainder of his physical examination findings are unremarkable.
Brain magnetic resonance imaging results are normal.
C. Sydenham chorea
D. Tourette syndrome
Correct answer is C
PREP Pearl(s)
Chorea is a hyperkinetic, irregular, involuntary, dance-like movement that appears as a continuous
flow of random muscle contractions. Chorea can result from a primary genetic disorder or secondary
to an acquired condition.
Sydenham chorea is the most common form of acquired chorea in childhood. It is characterized by an
insidious onset of chorea, hypotonia, and emotional lability occurring 1 to 8 months after a group A
streptococcal infection.
Critique
The subacute onset of chorea with accompanying hypotonia and emotional lability experienced by the boy in
the vignette is most consistent with Sydenham chorea (SC). Sydenham chorea, also known as rheumatic
chorea, is the most common acquired form of chorea in childhood. It is a sequela of group A streptococcal
infection that occurs most often in children aged 5 to 13 years. Sydenham chorea is one of the major clinical
manifestations of acute rheumatic fever, but unlike some other features that can appear within 21 days of
the inciting infection, SC usually has a more insidious onset, occurring 1 to 8 months after infection.
Treatment includes oral penicillin for 10 days (or a single intramuscular dose), followed by antibiotic
prophylaxis against streptococcal infections. The incidence of SC has declined dramatically in countries where
group A streptococcal infections are routinely treated with antibiotics.
Chorea is a hyperkinetic, irregular, involuntary, dance-like movement that appears as a continuous flow of
random muscle contractions (Video 1). Mild chorea can be difficult to distinguish from restlessness. Chorea
can result from a primary genetic disorder or secondary to an acquired condition (Table). The prognosis
depends on the underlying etiology.
Childhood-onset hereditary chorea is a broad term encompassing genetically defined chorea disorders. The
autosomal dominant disorder NKX2-1 chorea (previously known as benign hereditary chorea) manifests in
infancy with hypotonia followed by development of nonprogressive, generalized chorea. Hypothyroidism and
neonatal respiratory distress or frequent pulmonary infections are associated with NKX2-1 chorea, which is
also known as brain-thyroid-lung disease. Treatment is symptomatic.
Chorea is also associated with ADCY-5–related dyskinesia. This disorder manifests in infancy or early
childhood with fluctuating hyperkinetic movements. Treatment is symptomatic.
Benign shuddering attacks of infancy are benign paroxysmal nonepileptic movements characterized by brief
shudders or shivers lasting a few seconds.The movements frequently occur with excitement or stress or
during feeding. This condition is self-limited and spontaneously resolves.
Suggested Reading(s)
Brandsma R, van Egmond ME, Tijssen MAJ; Groningen Movement Disorder Expertise Centre. Diagnostic
approach to paediatric movement disorders: a clinical practice guide. Dev Med Child Neurol.
2021;63(3):252-258. doi:10.1111/dmcn.14721
Kruer MC. Pediatric movement disorders. Pediatr Rev. 2015;36(3):104-115. doi:10.1542/pir.36-3-104
Lahiri S, Sanyahumbi A. Acute rheumatic fever. Pediatr Rev. 2021;42(5):221-232. doi:10.1542/pir.2019-
0288
King RM. Tics. In: McInerny TK, Adam HM, Campbell DE, DeWitt TG, Foy JM, Kamat DM, eds. American
Academy of Pediatrics Textbook of Pediatric Care; American Academy of Pediatrics; 2023. Accessed
September 1, 2023. Pediatric Care Online
Content Domain
Neurology
A neonate’s state newborn screening test result is positive for spinal muscular atrophy. She has been well
and has normal physical examination findings, including muscle tone, strength, and reflexes. Molecular
genetic testing is performed; the findings demonstrate 0 copies of the SMN1 gene and 2 copies of the SMN2
gene, confirming a diagnosis of spinal muscular atrophy.
A. deflazacort
B. fordadistrogene movaparvovec
C. onasemnogene abeparvovec
D. risdiplam
Correct answer is C
PREP Pearl(s)
State newborn screening programs include testing for spinal muscular atrophy, which allows for early
diagnosis and treatment.
Three disease-modifying therapies available for spinal muscular atrophy (onasemnogene abeparvovec,
nusinersen, and risdiplam) improve neurodevelopmental outcomes, reduce the likelihood of ventilator
dependency, and improve nutrition and respiratory function.
Spinal muscular atrophy is classified according to age of onset and gross motor milestone attainment.
Critique
The neonate in the vignette has presymptomatic, genetically confirmed spinal muscular atrophy (SMA) and is
eligible for disease-modifying gene therapy with onasemnogene abeparvovec. This therapy, administered as
a single intravenous infusion, delivers an intact SMN1 gene via a genetically engineered adeno-associated
virus type 9 (AAV9) vector. In 2022, data from the phase III SPR1NT trial of presymptomatic children treated
with onasemnogene abeparvovec demonstrated improved developmental outcomes, nutrition, respiratory
function, and ventilator-free survival.
Before the availability of genetic testing, SMA was classified according to age of onset and gross motor
milestone attainment (Table). Spinal muscular atrophy is inherited in an autosomal recessive manner.
Pathologic variants in the SMN1 gene on chromosome 5q11-13.2, most commonly a deletion of exon 7, cause
a lack of function of the SMN1 gene, resulting in progressive motor neuron degeneration and the clinical
features of the disorder.
A second gene, SMN2, is nearly identical to SMN1, but due to a C-to-T transition at exon 7, the majority of
SMN protein produced by the SMN2 gene is truncated and nonfunctional. Children with SMA have 0 copies of
the SMN1 gene, leaving the SMN2 gene responsible for all SMN protein production, which can only partially
compensate. Disease severity loosely correlates with SMN2 copy number, which can vary from 0 to 8.
Generally, individuals with 3 or more SMN2 copies have a milder disease phenotype, but this is not absolute.
Although uncommon, there are non-5q forms of SMA for which available disease-modifying treatments are
not effective.
On December 23, 2016, the US Food and Drug Administration approved the first disease modifying therapy
(nusinersen) for the treatment of SMA. Since then, 2 additional disease-modifying therapies have become
available, risdiplam and onasemnogene abeparvovec. Presymptomatic diagnosis has allowed for rapid
initiation of disease-modifying treatment, often before the onset of clinical symptoms, transforming the
clinical presentation and course of infants and children with SMA. Early diagnosis and treatment is critical for
optimizing neurodevelopmental outcomes and long-term prognosis. In July 2018, because of the availability
of targeted therapies with demonstrated improved outcomes when initiated presymptomatically, the federal
government added SMA to the recommended uniform screening panel in statewide newborn screening
programs.
Two therapies, nusinersen and risdiplam, enhance production of the SMN protein by targeting the SMN2
gene. Nusinersen is approved for the treatment of presymptomatic SMA from age 3 days through adulthood
and is administered intrathecally as a series of 4 loading doses followed by maintenance dosing every 4
months. Risdiplam is a daily oral therapy approved for children aged 2 years or older who weigh at least 20
kg; it is not available for presymptomatic treatment of neonates identified by newborn screening. Clinical
trials are under way to evaluate the safety and efficacy of combination therapy. Data from trials in which
investigators examined presymptomatic treatment using both nusinersen and onasemnogene abeparvovec
demonstrated improved neurodevelopmental outcomes, improved nutrition and respiratory function, and
reduced likelihood of ventilator dependency.
Fordadistrogene movaparvovec is an investigational gene therapy for the treatment of Duchenne muscular
dystrophy (DMD) using an AAV9 vector to deliver a shortened version of the human dystrophin gene (mini-
dystrophin). Deflazacort is a steroid used as an alternative to prednisone in DMD treatment. Glucocorticoid
treatment is beneficial for preserving muscle strength and motor and pulmonary function; reducing the risk
of scoliosis; and prolonging the time before loss of independent ambulation in children with DMD. Neither
therapy is indicated in the treatment of SMA.
Suggested Reading(s)
Al-Zaidy SA, Mendell JR. From clinical trials to clinical practice: practical considerations for gene
replacement therapy in SMA Type 1. Pediatr Neurol. 2019;100:3-11.
doi:10.1016/j.pediatrneurol.2019.06.007
Nicolau S, Waldrop MA, Connolly AM, Mendell JR. Spinal muscular atrophy. Semin Pediatr Neurol.
2021;37:100878. doi:10.1016/j.spen.2021.100878
Prior TW, Leach ME, Finanger E. Spinal muscular atrophy. February 24, 2000 (updated December 3,
2020). In: Adam MP, Everman DB, Mirzaa GM, et al, eds. GeneReviews (Internet). Seattle (WA):
University of Washington, Seattle; 1993-2022.
Spiro AJ. Hypotonia. In: McInerny TK, Adam HM, Campbell DE, Foy JM, Kamat DM, eds. American
Academy of Pediatrics Textbook of Pediatric Care. 2nd ed. American Academy of Pediatrics; 2016:chap
167. Accessed September 1, 2023. Pediatric Care Online
Strauss KA, Farrar MA, Muntoni F, et al. Onasemnogene abeparvovec for presymptomatic infants with
three copies of SMN2 at risk for spinal muscular atrophy: the Phase III SPR1NT trial. Nat Med.
2022;28(7):1390-1397. doi:10.1038/s41591-022-01867-3>
Content Domain
Neurology
Courtesy of J. Goldstein
AAP PREP 2024 - Question 164/267 Neurology Question 6/12
A 3-year-old boy is brought to the emergency department for refusing to walk. Thirty minutes before arrival
he looked pale, became fussy, and clung to his parent wanting to be held. He then had several episodes of
nonbloody, nonbilious emesis. When his parent placed him on the floor to walk, he cried and had difficulty
taking independent steps, which appeared unbalanced and wobbly; then he immediately lay down on the
floor.
On physical examination, the boy is afebrile and his vital signs are normal. He initially appears comfortable,
then begins crying and develops horizontal nystagmus. Findings of the remainder of his cranial nerve
examination findings, including fundoscopy, is normal. He has normal strength, tone, reflexes, and sensation.
His coordination is difficult to assess because he does not want to reach for objects. He has a wide-based,
wobbly gait and takes only a few steps before lying down, crying to be held. The remainder of his
examination findings are normal.
Results of a complete blood cell count, comprehensive metabolic panel, and serum and urine toxicology
screens are unremarkable. Findings of a head computed tomography scan also are normal.
Within an hour, the boy returns to his neurological baseline and has normal physical examination findings. Of
the following, this boy’s MOST likely diagnosis is
D. vestibular migraine
Correct answer is C
PREP Pearl(s)
Nystagmus is a regular, rhythmic, involuntary oscillation of the eyes. Nystagmus can be classified as
physiologic versus pathologic, congenital versus acquired, or secondary to an ophthalmological versus
a neurological condition.
Benign paroxysmal vertigo is a pediatric migraine variant that occurs in young children. It is
characterized by short episodes of vertigo, nystagmus, vomiting, pallor, fearfulness, and ataxia.
The evaluation and management of nystagmus are guided by the type and the clinical context.
Critique
The boy in the vignette experienced a self-limited episode of vertigo characterized by nystagmus, difficulty
ambulating, ataxic gait, vomiting, and clinginess. This episode is most consistent with benign paroxysmal
vertigo (BPV), a pediatric migraine variant.
Children with BPV are typically aged 3 to 5 years and have a family history of migraine headache. Episodes
last from a few seconds to hours and consist of dizziness, pallor, nausea, vomiting, difficulty ambulating, and
nystagmus. Episodes of BPV follow a relapsing and remitting course that can occur in clusters lasting for
several days. The condition spontaneously resolves around age 5 years. Up to one-third of affected children
later develop migraine headaches. Treatment is often unnecessary owing to the brief nature of the attacks.
However, if the attacks are prolonged or frequent, symptomatic treatment with antiemetics or a daily dose of
a migraine-preventive medication (eg, topiramate) may be considered.
Benign paroxysmal vertigo is categorized in the International Classification of Headache Disorders-3 (ICHD-3)
under “episodic syndromes that may be associated with migraine.” The ICHD-3 diagnostic criteria for BPV are
as follows:
At least 5 attacks
Vertigo, without warning, maximal at onset and resolving spontaneously after minutes to hours
without loss of consciousness
At least one of the following associated signs or symptoms:
Nystagmus
Ataxia
Vomiting
Pallor
Fearfulness
Normal neurological examination findings and audiometric and vestibular functions between
attacks
Not attributed to another condition
Alternate diagnostic considerations are generally ruled out on the basis of the history and physical
examination findings. Acute cerebellar ataxia is a self-limited, postinfectious disorder characterized by acute
onset of ataxia and cerebellar dysfunction. The treatment of acute cerebellar ataxia is supportive, and the
condition resolves within days to weeks.
Vestibular migraine is a separate entity from BPV that develops later in childhood or adulthood. The presence
of migraine headache is needed to meet criteria for vestibular migraine, a feature that is less prominent or
absent in BPV.
Nystagmus refers to a regular, rhythmic, involuntary oscillation of the eyes. There are several ways to
categorize nystagmus, including the following:
Physiologic versus pathologic
Congenital versus acquired
Secondary to an ophthalmological versus a neurological condition
The differential diagnosis and evaluation of nystagmus are guided by the type of oscillation and the clinical
context (including history and physical examination findings). The management is tailored toward the
underlying etiology.
Suggested Reading(s)
Garone G, Suppiej A, Vanacore N, et al. Characteristics of acute nystagmus in the pediatric emergency
department. Pediatrics. 2020;146(2):e20200484. doi:10.1542/peds.2020-0484
Gelfand AA. Episodic syndromes that may be associated with migraine: a.k.a. "the childhood periodic
syndromes.” Headache. 2015;55(10):1358-1364. doi:10.1111/head.12624
IHD Classification IHD-3: 1. Migraine. 1.6.2 Benign paroxysmal vertigo. International Headache Society.
Accessed September 1, 2023. https://ichd-3.org/1-migraine/1-6-episodic-syndromes-that-may-be-
associated-with-migraine/1-6-2-benign-paroxysmal-
vertigo/#:~:text=Description%3A,spontaneously%2C%20in%20otherwise%20healthy%20children.
Loh AR, Chiang MF. Pediatric vision screening. Pediatr Rev. 2018;39(5):225-234. doi:10.1542/pir.2016-
0191
Rivera R, Sellinger C. Dizziness and vertigo. In: McInerny TK, Adam HM, Campbell DE, Foy JM, Kamat
DM, eds. American Academy of Pediatrics Textbook of Pediatric Care. 2nd ed. American Academy of
Pediatrics; 2016:chap 140. Accessed September 1, 2023. Pediatric Care Online
Content Domain
Neurology
An 18-month-old girl is evaluated for motor delay. She was born at 39 weeks’ gestation via normal
spontaneous vaginal delivery after an uncomplicated pregnancy. She had an unremarkable neonatal course.
Around age 6 months, her parents noticed that she preferred to use her left hand and often held her right
hand clenched in a fist at her side. She rolled at age 5 months, sat at age 9 months, and started cruising
along furniture at age 13 months but with a tendency to drag her right foot. She has taken steps holding her
parents’ hands but is not yet walking independently. She preferentially reaches for objects with her left hand,
and if forced to use her right hand she will transfer the object to her left hand as soon as possible.
Her physical examination findings are notable for right upper and lower extremity hypertonia and a tendency
to hold her right fist clenched. Her deep tendon reflexes are 3+ in the right upper and lower extremities. She
prefers to stand on the ball of her right foot, although she can place her full foot flat on the ground with
encouragement. When she walks with assistance, she circumducts her right leg. The remainder of her
physical examination findings are normal.
Brain magnetic resonance imaging reveals a T2 and fluid-attenuated inversion recovery hypointense ovoid
lesion in the left posterior limb of the internal capsule, consistent with a remote ischemic injury.
A. dyskinetic
B. spastic diplegia
C. spastic hemiplegia
D. spastic quadriplegia
Correct answer is C
PREP Pearl(s)
Cerebral palsy is a nonprogressive condition of heterogeneous etiology, characterized by abnormal
movement, motor tone, or posture that results in activity limitation. It is classified according to primary
motor disturbance, distribution of motor involvement, additional motor impairments, and associated
conditions.
Identification of the underlying etiology for cerebral palsy, when possible, is important. If no etiology is
identified, conditions that mimic cerebral palsy should be considered, as some have targeted
treatment options.
Care of the child with cerebral palsy is multidisciplinary, tailored to the individual child, and aimed at
providing support that will enable the child to reach full developmental potential. Medical, educational,
rehabilitative, family, and community support are required for the child’s optimal outcome.
Critique
The girl in the vignette has a spastic right hemiparesis secondary to a remote ischemic event, resulting in a
static change in muscle tone. These findings are consistent with a diagnosis of cerebral palsy (CP) in a spastic
hemiplegic pattern. Cerebral palsy, the most common motor disability in childhood, was defined by
Rosenbaum 2006 as “a group of permanent disorders of the development of movement and posture, causing
activity limitation, that are attributed to nonprogressive disturbances that occurred in the developing fetal or
infant brain. The motor disorders of CP are often accompanied by disturbances of sensation, perception,
cognition, communication and behavior; by epilepsy; and by secondary musculoskeletal problems.”
Cerebral palsy can be accompanied by abnormalities in cognition, vision, hearing, behavior, and the
musculoskeletal system (eg, hip displacement). Pain and comorbid conditions (eg, epilepsy or sleep
disorders) can also be associated with CP. Although children with more severe motor disability are at greater
risk, all children with CP should be screened routinely for comorbid conditions to ensure adequate
developmental support.
The girl in the vignette has a spastic hemiparesis; the pattern of her motor involvement is unilateral. Spastic
diplegia, commonly seen in premature infants with periventricular leukomalacia, describes a pattern of
spasticity in the bilateral lower extremities, sparing the arms. Spastic quadriplegia describes spasticity
involving all four limbs. Dyskinetic cerebral palsy is characterized by abnormal movements or posturing (eg,
dystonia, athetosis, chorea) and can result from insults to the basal ganglia (eg, kernicterus).
The causes of CP are heterogeneous and encompass a wide range of conditions impacting the developing
fetal or infant brain (Table 2). The underlying etiology can be multifactorial and cannot always be determined.
A stepwise investigation to attempt to identify the underlying etiology of CP is warranted and can include
neuroimaging (brain magnetic resonance imaging) and genetic and metabolic testing. Conditions that mimic
cerebral palsy should be considered when an underlying etiology is not found; disease-targeted treatment
options are available for some of these conditions. Risk factors for CP are shown in Table 3.
Although CP is nonprogressive, the clinical manifestations, musculoskeletal findings, and required support
can evolve over time as maturation of the nervous system occurs. Historically, the diagnosis was not made
until age 18 months to 2 years of age to confirm the nonprogressive nature of the condition. However, this
practice has changed; there may be early signs in infants at risk of experiencing CP, and an earlier diagnosis
allows for more rapid access to resources and support. Typically, the diagnosis is made over several visits,
which include serial neurological examinations, developmental assessments, and hearing and vision
screening. When the diagnosis of CP is suspected, it is important to educate the family regarding the
definition of cerebral palsy and their child’s specific features; families may come to the conversation with a
specific preconceived understanding of what the diagnosis means.
Treatment of CP is tailored to the individual child and focused on helping them reach their full developmental
potential and maximize independence, emotional and physical well-being, and communication abilities. Care
is typically provided by a multidisciplinary team, which may include, in addition to the primary care
practitioner, a pediatric neurologist, a pediatric physical medicine and rehabilitation specialist, therapy
services, and a developmental-behavioral pediatrician. Close coordination with the school to access early
intervention services and establish an optimal individual education plan is crucial for creating a supportive
educational environment. Treatment may also include medications (eg, oral anti-spasticity medications,
intrathecal or intramuscular injections) and/or surgical procedures (eg, selective dorsal rhizotomy), in
conjunction with therapeutic services (eg, physical therapy, occupational therapy, speech therapy).
Suggested Reading(s)
Murphy N. Cerebral palsy. In: McInerny TK, Adam HM, Campbell DE, Foy JM, Kamat DM, eds. American
Academy of Pediatrics Textbook of Pediatric Care. 2nd ed. American Academy of Pediatrics; 2016:chap
226. Accessed September 1, 2023. Pediatric Care Online
Noritz G, Davidson L, Steingass K; Council on Children with Disabilities, The American Academy for
Cerebral Palsy and Developmental Medicine. Providing a primary care medical home for children and
youth with cerebral palsy. Pediatrics. 2022;150(6):e2022060055. 10.1542/peds.2022-060055
Pearson TS, Pons R, Ghaoui R, Sue CM. Genetic mimics of cerebral palsy. Mov Disord. 2019;34(5):625-
636. doi:10.1002/mds.27655
Rosenbaum P, Paneth N, Leviton A, et al. A report: the definition and classification of cerebral palsy.
Dev Med Child Neurol Suppl. 2007;109:8-14.
Sheu J, Cohen D, Sousa T, Pham K. Cerebral palsy: current concepts and practices in musculoskeletal
care. Pediatr Rev. 2022;43(10):572-581. doi:10.1542/pir.2022-005657
Content Domain
Neurology
A 14-year-old girl with no notable medical history is brought to the emergency department by ambulance for
a seizure. When she went to bed the evening before, the girl was in her usual state of health. She has not had
any recent illnesses, new medications, or known ingestions. This morning, the girl’s mother heard a thud in
her daughter’s bedroom and found her having a generalized tonic-clonic seizure. The seizure aborted
spontaneously en route to the emergency department after at least 10 minutes.
In the emergency department, the girl appears postictal. She has normal vital signs. Oxygen is applied via
nasal cannula and intravenous access is obtained. A bedside glucose level, complete blood cell count, venous
blood gas, comprehensive metabolic panel, and serum toxicology screen are obtained. Fifteen minutes after
arrival in the emergency department, the girl begins to have another generalized tonic-clonic seizure.
Of the following, the BEST next step in this girl’s care is to administer intravenous
A. levetiracetam, 20 mg/kg
C. phenobarbital, 15 mg/kg
Correct answer is B
PREP Pearl(s)
Generalized convulsive status epilepticus is defined as either a continuous seizure lasting longer than 5
min or as 2 or more seizures without recovery of consciousness in between.
The management of status epilepticus includes seizure termination, prevention of recurrence, and
treatment of any complications.
The acute management of status epilepticus is focused on termination of the seizure; the American
Epilepsy Society and Neurocritical Care Society guidelines are summarized at
https://www.aesnet.org/clinical-care/clinical-guidance/guidelines.
Critique
The girl in the vignette is in convulsive status epilepticus (SE), a neurological emergency. Of the response
choices, administration of intravenous lorazepam (0.1 mg/kg, max: 4 mg/dose), a benzodiazepine, is the best
next treatment step. Intravenous levetiracetam (60 mg/kg, max: 4500 mg/dose) is a second-line therapy for
treating generalized convulsive SE. Intravenous valproic acid (40 mg/kg, max: 3000 mg/dose), another second-
line therapy, should be avoided in adolescents of child-bearing potential owing to its teratogenicity.
Intravenous phenobarbital (20 mg/kg; max: 800mg/dose) is an alternate therapy option if a benzodiazepine is
not available.
Generalized convulsive SE is defined as either a continuous seizure lasting longer than 5 minutes as 2 or
more seizures without recovery of consciousness in between. Status epilepticus results from failure of the
normal mechanisms that terminate a seizure. Long-term effects can occur owing to alteration of neuronal
networks, neuronal injury, and cell death. Historically, SE was defined as a seizure lasting longer than 30 to 60
min. For clinical practice, the definition of SE was shortened to 5 min. This change was prompted by evidence
that seizures lasting longer than 5 min are less likely to terminate spontaneously, and the longer they persist,
the more they become refractory to medication. Prompt recognition, stabilization, and treatment is critical
for seizure termination.
The American Epilepsy Society and Neurocritical Care Society have published guidelines for SE management
that focus on 3 simultaneous goals:
Acute SE management is divided into 4 phases (Figure). The initial management of SE is focused on
termination of the seizure. Practitioners should proactively prepare for each of the next treatment phases to
facilitate rapid progression through the phases, if needed. Simultaneous assessment of airway, breathing,
circulation, and disability is an important aspect of acute management, with intubation for airway protection
if medication causes sedation that leads to compromise. It is important to note that cessation of clinical
seizure activity does not confirm cessation of electrical seizure activity. Prompt initiation of continuous
electroencephalography monitoring for nonconvulsive seizures or SE is an important part of the ongoing
management.
Once SE termination is achieved, management is focused on identification of the underlying etiology,
prevention of recurrence, and management of complications. Additional diagnostic testing may include
neuroimaging, continuous electroencephalography monitoring, lumbar puncture, and/or additional
laboratory evaluation. Complications of SE may include respiratory failure, hyperpyrexia, cardiac arrhythmias,
electrolyte abnormalities, rhabdomyolysis, multiorgan failure, or intracranial hypertension. The risk of
subsequent development of epilepsy depends on the underlying etiology.
Suggested Reading(s)
Fine A, Wirrell E. Seizures in children. Pediatr Rev. 2020;41(7):321–347. doi:10.1542/pir.2019-0134
Glauser T, Shinnar S, Gloss D, et al. Evidence-based guideline: treatment of convulsive status
epilepticus in children and adults: report of the Guideline Committee of the American Epilepsy Society.
Epilepsy Currents. 2016;16(1):48-61. doi:10.5698/1535-7597-16.1.48
Lyttle M, Pereira M. Levetiracetam versus phenytoin for second-line treatment of paediatric convulsive
status epilepticus (EcLiPSE): a multicentre, open-label, randomized trial. Lancet. 2019;393(10186):2125-
2134. doi:10.1016/S0140-6736(19)30724-X
Roddy S, McBride M, 2016. Status epilepticus. In: McInerny TK, Adam HM, Campbell DE, DeWitt TG, Foy
JM, eds. American Academy of Pediatrics Textbook of Pediatric Care. 2nd ed. American Academy of
Pediatrics; 2016:chap 374. Accessed March 23, 2023. Pediatric Care Online
Content Domain
Neurology
A 3-year-old is evaluated for abdominal pain and constipation. Her parents were unable to induce a bowel
movement with an enema. They report that she is now experiencing back pain and is refusing to walk. The
girl is afebrile and has normal vital signs. She appears to be in mild distress. She is unable or unwilling to
walk. On physical examination, pain is elicited on palpation of her midline lower back. The remainder of her
examination findings are unremarkable.
Of the following, the BEST next step in this child’s management is referral to
A. a gastroenterologist
B. an oncologist
C. an orthopedic surgeon
Correct answer is D
PREP Pearl(s)
Constipation, back pain, and refusal or inability to ambulate should prompt investigation for spinal cord
compression.
Magnetic resonance imaging is the imaging of choice to assess for spinal cord compression.
Dexamethasone should be administered promptly to decrease inflammation if spinal cord compression is
suspected or identified.
Critique
The child in the vignette has constipation, back pain, and refusal/inability to ambulate. This constellation of
symptoms raises concern for a neurologic issue (eg, spinal cord compression).
When spinal cord compression is suspected, urgent evaluation and intervention are necessary to avoid
morbidity and mortality. Physical examination and imaging are the key components of evaluation for
suspected spinal cord compression. There may be no overt physical findings; with severe compression, there
may be weakness, sensory deficits, sphincter dysfunction (bowel or bladder), and loss of deep tendon
reflexes.
The imaging of choice to evaluate for spinal compression is magnetic resonance imaging (Figure). For a young
child, this may require sedation, leading to potential delay in treatment. Therefore, if there is high suspicion
for spinal cord compression and imaging may be delayed, steroids (dexamethasone) should be administered
before obtaining imaging to decrease inflammation/edema.
Courtesy of J. Amodio
Figure . Magnetic resonance imaging showing an intradural mass causing spinal cord (SC) compression at
cervicothoracic junction.
Sarcoma is the most common oncologic cause of spinal cord compression, followed by neuroblastoma.
Compression can also be caused by metastatic central nervous system tumors, leukemia, or lymphoma.
If an oncologic cause of spinal cord compression is identified, chemotherapy must be initiated immediately.
For radiosensitive cancers, radiation therapy may be administered. If there is a poor response to these
treatments or significant clinical worsening, urgent surgical decompression (laminectomy or laminotomy)
may be required.
Evaluation by a gastroenterologist may be required for a child with chronic abdominal pain and constipation,
and evaluation by an orthopedic surgeon may be needed for a child with back pain. However, the
combination of back pain, inability or refusal to ambulate, and constipation raise concern for spinal cord
compression, which is a medical emergency that requires urgent evaluation in the emergency department.
An oncologist will be needed to determine the treatment plan if an oncologic cause of spinal cord
compression is identified, but immediate evaluation should take place in the emergency department.
Suggested Reading(s)
Freedman JL, Rheingold SR. Management of oncologic emergencies. In: Lanzkowsky P, Lipton J, Fish JD,
eds. Lanzkowsky’s Manual of Pediatric Hematology and Oncology. 6th ed. Elsevier; 2016:614-615.
Sharma R, Mer J, Lion A, Vik TA. Clinical presentation, evaluation, and management of neuroblastoma.
Pediatr Rev. 2018;39(4):194-203. doi:10.1542/pir.2017-0087
Content Domain
Neurology
A 16-year-old boy is seen in the clinic for evaluation of a right foot-drop present when he awoke this morning.
He denies leg trauma. He denies pain or discomfort in his right leg or back. Yesterday evening, he sat
awkwardly on a hard floor while watching a movie with friends. He has had no previous problems with his
leg, but reports that 1 year ago, during a camping trip, he awoke with difficulty moving his left wrist, which he
attributed to a funny sleeping position. This symptom slowly resolved over a few weeks without treatment.
The boy’s father reports having had similar symptoms, which resolve without intervention and have been
attributed to trauma or poor positioning.
The adolescent’s neurological examination findings are notable for a right foot-drop, with inability to dorsiflex
and evert the foot. Sensation is diminished to pinprick and light touch from the dorsum of his right foot to
the anterolateral aspect of his right leg. His deep tendon reflexes are normal. He walks with a steppage gait
(lifting the right foot higher than normal) and is unable to walk on his right heel. The remainder of his
physical examination findings are normal.
Electromyography and nerve conduction studies are performed. The results are consistent with a
demyelinating distal polyneuropathy and a superimposed mononeuropathy of the right peroneal nerve.
Correct answer is C
PREP Pearl(s)
Peripheral neuropathies encompass a wide range of acquired and hereditary conditions, with acute,
subacute, and chronic forms. Accurate diagnosis relies on identification of the pattern of nerve
involvement, time course, age of onset, family history, exposures (toxic, metabolic, environmental),
and other medical conditions.
The cornerstone of treatment of peripheral neuropathy is supportive care with early rehabilitation
services (eg, physical therapy, orthotics), with more targeted therapy (if available) aimed at the
underlying etiology.
Critique
The boy in the vignette is experiencing foot-drop due to an acute painless mononeuropathy of the right
peroneal nerve superimposed on a chronic distal demyelinating neuropathy. His prior episode of wrist drop
suggests a predisposition to pressure palsies. His clinical presentation, neurodiagnostic findings, and family
history strongly suggest a genetic etiology of his neuropathy, such as hereditary neuropathy with
predisposition to pressure palsies. The cornerstone of treatment for any neuropathy is supportive care and
early rehabilitation services, with more targeted therapy (if available) aimed at the underlying etiology. This
boy would benefit from physical therapy and an ankle-foot orthosis.
If there is pain accompanying the neuropathy, a neuropathic pain medication (eg, gabapentin) can be
prescribed. The boy in the vignette does not have pain, so this type of medication is not indicated.
Neuropathies caused by compression can often be managed conservatively, but if acute entrapment occurs
(eg, postoperatively) or symptoms are persistent, surgery is indicated to relieve the compression. There is no
indication for an adolescent with peripheral neuropathy to be non-weight bearing. Treatment should focus
on maintaining range of motion, building strength, and preventing contractures.
Peripheral neuropathies are conditions affecting the axons, the myelin sheath, or Schwann cells of the
peripheral nervous system. They are relatively uncommon in children. Peripheral neuropathies encompass a
wide range of acquired and hereditary conditions, with acute, subacute, and chronic forms. When peripheral
neuropathy is suspected, a stepwise approach to diagnosis is needed. The Table outlines the history, physical
examination, and diagnostic testing approach to making an accurate diagnosis of peripheral neuropathy,
which will allow for appropriate management. In addition to electromyography and nerve conduction studies,
magnetic resonance imaging of affected sites and/or neuromuscular ultrasonography can provide supportive
diagnostic information and guide further evaluation.
Suggested Reading(s)
Brennan K, Shy M. Genetic peripheral neuropathies. In: Swaiman K, Ashwal S, Ferriero D, Schor N,
Finkel R, et al; eds. Swaiman's Pediatric Neurology. 6th ed. Elsevier; 2018:chap 141. Accessed March 8,
2023. doi:10.1016/B978-0-323-37101-8.00141-7
Homnick D. Pediatric physical examination: interpretation of findings. In: McInerny TK, Adam HM,
Campbell DE, DeWitt TG, Foy JM, eds. American Academy of Pediatrics Textbook of Pediatric Care.
American Academy of Pediatrics; 2016:chap 16. Pediatric Care Online
Rabie M, Ashwal S, Nevo Y. Inflammatory neuropathies. In: Swaiman K, Ashwal S, Ferriero D, Schor N,
Finkel R, et al; eds. Swaiman's Pediatric Neurology. 6th ed. Elsevier; 2018:chapter 143. Accessed March
8th, 2023. doi.org/10.1016/B978-0-323-37101-8.00143-0
Wilmshurst J, Ouvrier R. Hereditary peripheral neuropathies of childhood: an overview for clinicians.
Neuromuscul Disord. 2011;21(11):763-775. doi:10.1016/j.nmd.2011.05.013
Content Domain
Neurology
A 15-year-old adolescent is seen for evaluation of difficulty concentrating and excessive daytime sleepiness.
For the past 2 years, she has had difficulty staying awake throughout the day despite sleeping 10 hours each
night. Her symptoms have progressed to the point that she frequently falls asleep at school and takes several
brief naps on weekends. When laughing, she experiences facial weakness, slurred speech, and sometimes
head drop. Today, her physical examination findings are normal. Polysomnography with multiple sleep
latency demonstrates sleep-onset rapid eye movement on 4 of the nap studies.
Of the following, the MOST likely finding on this adolescent’s cerebrospinal fluid analysis is a
Correct answer is C
PREP Pearl(s)
Excessive daytime sleepiness is the core symptom of narcolepsy, and it may be accompanied by
cataplexy, hypnagogic hallucinations, sleep paralysis, or a combination of these.
Narcolepsy is diagnosed by polysomnography with a multiple sleep latency test; cerebrospinal fluid
analysis will demonstrate absent or low hypocretin levels in 90% of affected individuals.
Management of narcolepsy is directed at symptom relief and includes medication, education, and
lifestyle modification.
Critique
Of the response choices, the most likely finding in this girl’s cerebrospinal fluid analysis is a low hypocretin
level. The adolescent in the vignette has narcolepsy, a chronic sleep disorder. The primary symptom of
narcolepsy is excessive daytime sleepiness. Affected children and adolescents typically experience constant
drowsiness with repeated napping or sleep attacks, most often in boring or monotonous situations. This
adolescent is also experiencing cataplexy (sudden loss of muscle tone provoked by strong emotions), which is
a unique feature of narcolepsy.
Polysomnography with a multiple sleep latency test should be performed when narcolepsy is suspected.
Findings consistent with narcolepsy include sleep onset in equal to or less than 8 min and 2+ naps with sleep-
onset rapid-eye-movement periods.
When the results of polysomnography with a multiple sleep latency test are difficult to interpret or
confounded (eg, by underlying comorbid conditions or medications), a cerebrospinal fluid (CSF) analysis is
helpful for making the diagnosis of narcolepsy. A low CSF hypocretin-1 value (<110 pg/mL) is diagnostic and
differentiates narcolepsy from alternate conditions (eg, idiopathic hypersomnia, sleep apnea, restless leg
syndrome, insomnia). It is important to note that up to 10% of individuals with narcolepsy have normal CSF
hypocretin-1 levels.
Hypocretin is a neuropeptide hormone secreted by the hypothalamus that increases wakefulness by
suppressing rapid-eye-movement sleep. Hypocretin deficiency is postulated to be caused by an autoimmune-
mediated attack on the hypocretin neurons. Hypocretin deficiency can also occur with acquired conditions
affecting the hypothalamic periventricular regions, including demyelinating disorders (eg, neuromyelitis
optica-spectrum disorders), trauma, inflammation, or infection.
Leptin, a hormone released by adipose tissue, plays a role in regulating hunger and satiety. Serum leptin
levels rise during sleep, and there is evidence of reduced serum (not CSF) leptin levels in adults with
narcolepsy, hinting at a complex interplay of several hormones in the disorder. However, leptin level
measurement is not part of the diagnostic workup of narcolepsy.
All individuals with narcolepsy experience excessive daytime sleepiness. The other features are not uniformly
present and can develop over time. In children and adolescents, narcolepsy may present with behavioral
changes (eg, hyperactivity, aggression, poor concentration, “laziness,” or depression). The variability in
presentation of narcolepsy makes a detailed sleep history an important step in the assessment of concerns
regarding attention, hyperactivity, or behavior change.
Management of narcolepsy is directed at symptom relief and has 3 components: medication, education, and
lifestyle modification. Excessive sleepiness is treated with stimulants (eg, modafinil, methylphenidate,
amphetamine). Cataplexy is treated with imipramine, fluoxetine, or venlafaxine. Sodium oxybate (ɣ-
hydroxybutyrate) is a nonstimulant, non–wake-promoting medication that is effective in treating daytime
sleepiness and cataplexy in children aged 7 years and older. Education of families and teachers is important
so that excessive daytime sleepiness is not mistaken for lack of motivation, inattention, or laziness. Planned
daytime naps can help mitigate excessive daytime sleepiness and reduce the overall dose of stimulants
required.
Suggested Reading(s)
Bhargava S. Diagnosis and management of common sleep problems in children. Pediatr Rev.
2011;32(3):91-99. doi:10.1542/pir.32-3-91
Blackwell JE, Kingshott RN, Weighall AR, Elphick HE, Nash H. Paediatric narcolepsy: a review of diagnosis
and management. Arch Dis Child. 2022; 107(1):7-11. doi:10.1136/archdischild-2020-320671
Maski K, Trotti LM, Kotagal S, et al. Treatment of central disorders of hypersomnolence: an American
Academy of Sleep Medicine clinical practice guideline. J Clin Sleep Med. 2021;17(9):1881-93.
doi:10.5664/jcsm.9326
Splaingard ML, May A. Sleep disturbances (nonspecific). In: McInerny TK, Adam HM, Campbell DE,
DeWitt TG, Foy JM, eds. American Academy of Pediatrics Textbook of Pediatric Care. American
Academy of Pediatrics; 2016:chap 194. Pediatric Care Online
Content Domain
Neurology
A 16-year-old adolescent is undergoing evaluation for a 3-year history of headaches that are bifrontal and
pulsatile with associated photophobia, phonophobia, and nausea. When severe, the headaches are
sometimes associated with vomiting. The adolescent reports no preceding aura, accompanying focal
neurological deficit, initiation with valsalva maneuver, or awakening from sleep owing to headache pain. They
have not identified any headache triggers or patterns. The headaches occur at least 4 times per week, last for
hours, and result in school absence at least once weekly. The adolescent takes appropriate doses of
nonsteroidal anti-inflammatory medications every time they get a headache, which results in modest relief.
Sleep is the most effective method of alleviating the headache. Their physical examination findings, including
fundoscopic findings, are normal. Counseling is provided regarding lifestyle and behavioral factors and
modifiable headache triggers.
Correct answer is C
PREP Pearl(s)
Migraine management includes 3 components: lifestyle modifications, abortive therapies, and
preventive therapies.
Frequent use of over-the-counter analgesics (more than 14 days per month) for treatment of migraine
headaches increases the risk of experiencing medication overuse headache.
For migraine headaches, initiation of a daily preventive medication, along with lifestyle modifications,
can reduce disability, medication overuse, and progression to chronic migraine.
Critique
The adolescent described in the vignette has episodic migraine without aura, which is impacting their
activities of daily living and causing them to miss school days. Additionally, the use of over-the-counter
analgesics more than 14 days per month increases their risk of experiencing medication overuse headache.
Initiation of a daily preventive medication (eg, amitriptyline) is the best next management step to prevent
disability (missed school days), medication overuse, and progression to chronic migraine. Although the
addition of a triptan, acetaminophen, or antiemetic (eg, ondansetron) may provide additional acute migraine
treatment, it will not reduce the overall headache burden and potential disability.
Migraine is the most common primary headache disorder in adolescents. The International Classification of
Headache Disorders (ICHD)-3 criteria for diagnosis of migraine without and with aura are listed in Table 1.
Management of migraine is tailored to each individual and necessitates shared decision making. In 2019, the
American Academy of Neurology, the American Academy of Pediatrics, and the Child Neurology Society
together published guidelines on the acute and preventive treatment of migraine in children and adolescents
(reaffirmed in 2022).
Migraine management has 3 components: abortive therapy, preventive therapy, and lifestyle modifications.
Abortive therapy: immediate treatment to alleviate migraine pain and associated symptoms; options
are summarized in Table 2.
Rest
Hydration
Nonsteroidal anti-inflammatory medications or acetaminophen (both are first-line)
Triptan medications alone or in combination with a nonsteroidal anti-inflammatory medication
(for refractory cases)
Antiemetics may be used as an adjunct for significant nausea and/or vomiting
Preventive therapy: treatment options aimed at reducing overall migraine burden. Preventive
medications should be considered when quality of life is impacted by headache (eg, excessive time
away from school, missed home and social activities). Preventive therapy options are summarized in
Table 3. There are 4 main classes of preventive medications:
Antihistamines (eg, cyproheptadine)
Antiseizure medications (eg, topiramate)
Antihypertensives (eg, propranolol)
Antidepressants/anxiolytics (eg, amitriptyline)
Screening for each potential trigger can help open a conversation to guide lifestyle changes.
Headachereliefguide.com is an evidence-based resource to support children and adolescents in adapting
lifestyle modifications beneficial for migraine management.
Complementary therapies are increasingly recommended in the treatment of pediatric migraine. These
therapies include the following:
Vitamin supplements (eg, riboflavin, melatonin, magnesium, CoQ10)
Herbal supplements (eg, feverfew)
Psychological interventions (eg, biofeedback, cognitive behavioral therapy)
Physical therapies (eg, acupuncture, massage)
Vitamin or herbal supplements can be used as preventive or adjunctive therapies and are generally well
tolerated.
Adolescents should be counseled regarding the critical importance of lifestyle modifications for their
headache management. Each visit should include counseling about trigger identification, ongoing lifestyle
modifications, and opportunities for additional changes. The efficacy of the abortive therapy plan should be
assessed to allow for ongoing optimization. When initiating preventive medications, it is important that the
physician set expectations regarding time to effect. Most individuals will experience a gradual reduction in
headache frequency and severity over several weeks. The ongoing need for preventive medication should be
reassessed, because lifestyle modifications may mitigate the need for preventive treatment.
Suggested Reading(s)
Gladstein J. Headache. In: McInerny TK, Adam HM, Campbell DE, Foy JM, Kamat DM, eds. American
Academy of Pediatrics Textbook of Pediatric Care. American Academy of Pediatrics; 2016:chap 157.
Accessed September 1, 2023. Pediatric Care Online
Klein J, Koch T. Headache in children. Pediatr Rev. 2020;41(4):159-171. doi:10.1542/pir.2017-0012
Oskoui M, Pringsheim T, Billinghurst L, et al. Practice guideline update summary: pharmacologic
treatment for pediatric migraine prevention: report of the Guideline Development, Dissemination, and
Implementation Subcommittee of the American Academy of Neurology and the American Headache
Society. Neurology. 2019;93(11):500-509. doi:10.1212/WNL.0000000000008105
Oskoui M, Pringsheim T, Holler-Managan Y, et al. Practice guideline update summary: acute treatment
of migraine in children and adolescents: report of the Guideline Development, Dissemination, and
Implementation Subcommittee of the American Academy of Neurology and the American Headache
Society. Neurology. 2019;93(11):487-499. doi:10.1212/WNL.0000000000008095
Powers SW, Coffey CS, Chamberlain LA, et al; CHAMP Investigators. Trial of amitriptyline, topiramate
and placebo for pediatric migraine. New Engl J Med. 2017;376(2):115-124. doi:10.1056/NEJMoa1610384
Content Domain
Neurology
Ibuprofen 10 mg/kg - max 800 mg Q6h GI upset, bleeding, kidney dysfunction First line
Naproxen 10-20 mg/kg - max 500 mg Q12h GI upset, bleeding, kidney dysfunction First line, longer period of action
Acetaminophen 15 mg/kg - max 1,000 mg Q8h or 3,000 total Liver dysfunction First line, especially in patients with contraindication
mg/day or sensitivity to nonsteroidal anti-inflammatory drugs
Ketorolac 0.5 mg/kg PO or IV - max 10 mg PO Q6h or 30 GI upset, bleeding, kidney dysfunction Often used first line in ED as part of a “migraine
mg IV Q6h cocktail” with fluids and antiemetic agent
Metoclopramide 0.2 mg/kg PO or IV - max 10 mg Q6h Somnolence, extrapyramidal adverse effects Diphenhydramine can be used for pretreatment to
prevent extrapyramidal effects
Prochlorperazine 0.15 mg/kg PO or IV - max 10 mg Q6h Somnolence, extrapyramidal adverse effects, dizziness Diphenhydramine can be used for pretreatment to
prevent extrapyramidal effects
Valproic acid 15 mg/kg IV - max 1,000 mg Q12h Somnolence, GI upset; avoid in patients with hepatic Typically used second line in ED if initial cocktail is
dysfunction and pregnant patients ineffective
Triptans
Almotriptan 6.25 or 12.5 mg - max 25 mg/day Fatigue, somnolence, flushing, chest pain, paresthesia; Use after or in conjunction with analgesic agents; do
do not use in patients with arrhythmia, coronary artery not use >2 days per week
disease, stroke, hemiplegic migraine, migraine with
brainstem aura, poorly controlled hypertension, use
of ergot derivative within previous 24 h; serotonin
syndrome risk when using in patients taking SSRIs
Zolmitriptan Nasal 5 mg, 2.5 or 5 mg ODT - max 10 mg/day Same as above Same as above
Abbreviations: ED, emergency department; GI, gastrointestinal; IV, intravenous; max, maximum; ODT, oral disintegrating tablet; PO, orally; Q, every; SSRI, selective serotonin reuptake
inhibitor.
Adapted and reprinted with permission from Klein J, Oakley C. Migraine and headaches in children. In: Johnston MV, Adams HP, Fatemi A, eds. Neurobiology of Disease. 2nd ed. Oxford, NY:
Oxford University Press; 2016:543-544.
Table 3. Common Preventative Therapy Options
Medication* Dosing Range Adverse Effects; Warnings Favorable Qualities
Antihistamines
Cyproheptadine 2-4 mg PO QHS - max 8 mg Q8-12h Increased appetite, weight gain, somnolence In general, is well tolerated
Antidepressants/anxiolytics
Amitriptyline 10 mg PO QHS - max 50 mg BID Somnolence, dizziness, overdose may cause cardiotoxicity, risk Can help sleep initiation, may stabilize mood in
of suicidal ideation, must be weaned high dose
Nortriptyline 25 mg PO QHS - max 50 mg BID Same as above Same as above; may be helpful for chronic
widespread pain
Duloxetine 20 mg PO QHS - max 80 mg daily Gastrointestinal upset, risk of suicidal ideation; can lead to May be helpful or chronic widespread pain, may
serotonin syndome or dystonia if used with metoclopramide; improve anxiety
can lead to hyponatremia, SIADH, hypotension, serotonin
syndome if used with prochlorperazine; must be weaned
Venlafaxine 37.5 mg PO daily - max 150 mg daily Constipation, dry mouth, risk of suicidal ideation, can lead to Same as above, can be helpful for dizziness
extended serotonin syndome or dystonia if used with metoclopramide,
release must be weaned
Antiepileptics
Topiramate 15 mg PO QHS - max 100 mg BID but typical Cognitive dysfunction, paresthesia, weight loss, kidney stones, Especially useful in overweight/obese patients;
max dose used for headache is 50 mg BID decreased perspiration, metabolic acidosis cna be used for dual purposes in patients who also
have epilepsy
Zonisamide 25 mg PO QHS - max 50 mg BID Contraindicated in pateints with a sulfa allergy In general adverse effects are similar to but less
severe than those for topiramate
Acetazolamide 10 mg/kg per day divided BID-TID or 250 mg Paresthesia, urinary frequency, metabolic acidosis, electrolyte Useful in some patients with hemiplegic migraine
BID - max 4,000 mg/day derangement (hyponatremia, hypokalemia)
Divalproex 10 mg/kg per day PO (usually divided BID) Teratogenicity, weight gain, hair and skin changes, tremor, liver Can be used for dual purposes in patients who
- max 40 mg/kg per day divided BID or 750 dysfunction, requires laboratory monitoring (particularly liver also have epilepsy, mood stabilizer
mg BID function tests and platelets)
Gabapentin 10 mg/kg per day divided TID - max 35 mg/kg Weight gain, somnolence, easy bruising, caution if renal May help with neuropathic pain, helps with
per day divided TID or 900 mg TID impairment sleep initiation, can be used for dual purposes in
patients who also have epilepsy
Antihypertensives
Propranolol 1 mg/kg per day PO (daily or divided BID) Bradycardia, hypotension, decreased exercise tolerance, erectile In general, is well tolerated
- max 4 mg/kg per day dysfunction, contraindicated in patients with poorly controlled
asthma, diabetes; caution in depression as can worsen mood
Verapamil 4 mg/kg per day PO daily - max 8 mg/kg per Hypotension, constipation, leg edema Useful in some patients with hemiplegic migraine
extended day or 480 mg daily
release
*Providers should be aware that many of these medications are not Food and Drug Administration (FDA) approved for migraine or headache treatment in the pediatric age group.
Abbreviations: BID, twice daily; max, maximum; ODT, oral disintegrating tablet; PO, orally; QHS, every night at bed time; Q8-12h, every 8 to 12 hours; SIADH, syndrome of inappropriate
antidiuretic hormone secretion; TID, three times daily.
Reprinted with permission from Klein J, Oakley C. Migraine and headaches in children. In: Johnston MV, Adams HP, Fatemi A, eds. Neurobiology of Disease. 2nd ed. Oxford, NY: Oxford
University Press; 2016:544-545.
AAP PREP 2024 - Question 171/267 Nutrition Question 1/6
An 8-year-old boy with autism is seen in the office. He developed a rash 1 week ago, and today he had
bleeding gums and is refusing to walk, stating that “it hurts.” The boy has not had prior issues with excessive
bleeding or any orthopedic issues. His diet is quite restrictive, limited to macaroni & cheese and chocolate
milk. His growth has been normal. The boy’s only medication is nightly guanfacine. The boy’s height and
weight are at the 75th percentile for age. He appears distressed and sits stiffly on the bed. He has a diffuse,
nonblanching rash (Figure). The remainder of his physical examination findings are unremarkable. Complete
blood count results are normal.
Figure. Reprinted with permission from Nastro A, Rosenwasser N, Daniels SP, et al. Pediatrics, 2019;144 (3):
e20182824.
Of the following, the BEST treatment for this boy’s acute condition is to
B. administer vitamin C
C. administer vitamin K
Correct answer is B
PREP Pearl(s)
Scurvy can present with dramatic, subacute-onset disability due to bone pain.
Scurvy can be seen in children whose diets include normal caloric intake with minimal fresh fruits and
vegetables.
Scurvy is easily treated with vitamin C supplementation.
Critique
The boy in the vignette has scurvy (vitamin C deficiency) due to his restricted diet. The treatment for scurvy is
administration of vitamin C. In the past, scurvy was most commonly seen in sailors who often went many
months without access to fresh fruits and vegetables. So many foods in the United States are now fortified
with vitamin C that scurvy is quite rare in the general population. However, this deficiency can still be seen in
children whose diets are significantly limited either by behavioral or social issues. Scurvy can present with
dramatic, subacute-onset disability due to bone pain. Signs and symptoms of scurvy may include:
Fatigue
Easy bruising/petechiae
Gum bleeding
Bone pain
Myalgias
Perifollicular hemorrhages
Corkscrew hairs
Anemia
Poor wound healing
Untreated scurvy can have multi-organ complications (eg, jaundice, edema, neuropathy, seizures, hemolysis)
and can lead to death.
The treatment for symptomatic scurvy is intravenous or oral high-dose vitamin C 3 times daily for 1 week,
then daily vitamin C for 1 month. After that, if the child cannot or will not take a regular diet, a multivitamin
should be administered.
Blood levels of vitamin C increase quickly with supplementation, therefore a vitamin C level (usually sent to
an outside laboratory) should ideally be drawn before supplementation begins. Treatment should commence
immediately upon suspecting the diagnosis of scurvy (once the blood sample has been obtained for a vitamin
C level); it is not appropriate to wait for the vitamin C level result. High-dose vitamin C is a very low-risk and
inexpensive therapy.
Treatment with fresh frozen plasma would be recommended in the setting of acute bleeding due to vitamin K
deficiency, disseminated intravascular coagulation, or a specific factor deficiency. The child in the vignette’s
gum-bleeding does not meet that level of urgency. Administration of vitamin K would be indicated in the
setting of presumed or known vitamin K deficiency, such as a neonate who did not receive vitamin K
prophylaxis, or a child with liver failure or a fat-absorption deficiency.
A high-protein, high-calorie diet would not address this child’s symptomatic scurvy. The boy had been
consuming a diet that was providing adequate calories, as evidenced by his normal growth. Both milk and
cheese are protein sources, so likely he had adequate protein intake as well. However, this form of dietary
management would not address the boy’s vitamin C deficiency.
Suggested Reading(s)
Ceglie G, Macchiarulo G, Marchili MR, et al. Scurvy: still a threat in the well-fed first world? Arch Dis
Child. 2019;104(4):381-383. doi:10.1136/archdischild-2018-315496
Kothari P, Tate A, Adewumi A, Kinlin LM, Ritwik P. The risk for scurvy in children with
neurodevelopmental disorders. Spec Care Dentist. 2020;40(3):251-259. doi:10.1111/scd.12459
Likhitweerawong N, Boonchooduang N, Morakote W, Louthrenoo O. Scurvy mimicking as systemic
lupus erythematosus. BMJ Case Rep. 2021;14(6):e242958. doi:10.1136/bcr-2021-242958
Racine Ad. Failure to thrive: pediatric undernutrition. In: McInerny TK, Adam HM, Campbell DE, DeWitt
TG, Foy JM, Kamat DM, eds. American Academy of Pediatrics Textbook of Pediatric Care. American
Academy of Pediatrics; 2023. Accessed September 1, 2023. Pediatric Care Online
Content Domain
Nutrition
A 12-year-old is undergoing a health supervision visit. She has been healthy overall and has not had concerns
regarding repeat infections. She was recently diagnosed with celiac disease and started a gluten-free diet.
Her parents report that for the past few weeks, she has had nausea and a decreased appetite and has felt
weak. They ask advice about additional diet changes or nutritional supplements that might help her. The
girl’s physical examination findings are normal. There are no changes in skin pigmentation or signs of
alopecia. Results of a complete blood cell count are normal for age.
Of the following, the nutritional deficiency MOST likely to explain this girl’s current symptoms is
A. copper
B. iron
C. magnesium
D. zinc
Correct answer is C
PREP Pearl(s)
Signs and symptoms of magnesium deficiency may include nausea, vomiting, and decreased energy or
weakness. More significant deficiencies are associated with muscle contractions or cramping, seizures,
numbness or tingling, mood instability, and, in severe cases, arrhythmias.
Signs and symptoms of copper deficiency may include hypopigmentation, anemia, connective tissue
disorders, and ataxia.
Zinc deficiency can lead to diarrhea, decreased growth, decreased appetite, and loss of the senses of
taste and smell. More severe deficiency can also lead to alopecia, skin manifestations (eg, perioral and
perirectal rash in infants), and frequent infections.
Critique
Of the response choices, magnesium is the nutritional deficiency most likely to explain this girl’s current
symptoms. A well-rounded diet, rich in nutrients and dietary minerals (eg, zinc, copper, and magnesium) is
important at all ages. Every 5 years, the US Department of Agriculture and US Department of Health and
Human Services publish Dietary Guidelines for Americans, a report of dietary recommendations based on the
most current data. The Table outlines the recommended daily intake of zinc, copper, and magnesium
according to age.
Magnesium is found in foods such as leafy greens, legumes, nuts, seeds, and whole grains. Processed grains
may lack this vital nutrient unless fortified (eg, fortified cereals). Only about 1% of the body's magnesium is
found in the blood, making the diagnosis of magnesium deficiency somewhat challenging. Most is stored in
the bone (50%-60%), with additional stores in the soft tissues. Urine magnesium levels, especially after
intravenous supplementation, may be helpful in identifying a deficiency (although this is controversial).
Symptoms of magnesium deficiency may include nausea, vomiting, decreased energy and weakness, as seen
in the girl in the vignette. More significant deficiencies are associated with muscle contractions or cramping,
seizures, numbness or tingling, mood instability, and, in severe cases, arrhythmias. Individuals at increased
risk of experiencing magnesium deficiency include those with the following factors:
Increased urinary excretion secondary to loop diuretics (eg, furosemide)
Diabetes with poor glucose control, leading to glucosuria and increased urination (more
common in type 2 diabetes or insulin resistance)
Long-term (>12 months) treatment with proton pump inhibitors (decreased absorption, not
always improved with supplementation)
Malabsorption (eg, Crohn or celiac disease)
Excess magnesium intake, such as from laxatives with magnesium carbonate, chloride, or gluconate can lead
to diarrhea, owing to an inability to absorb the magnesium salts. Toxicity has been rarely reported with doses
of 5,000 mg/day; signs and symptoms include hypotension, flushing, urinary retention, lethargy progressing
to weakness, arrhythmia, and cardiac arrest.
Copper is found in foods such as shellfish, seeds and nuts, chocolate, and whole grains. Tap water and other
beverages are another source of copper, but with a wide content range (0.005 mg/L-1 mg/L). A large portion
of the body's copper is stored in skeletal muscle. The body achieves homeostasis by regulating absorption of
copper, as well as regulation by the liver of the amount excreted through bile. Symptoms of copper deficiency
include hypopigmentation, anemia, connective tissue disorders, and ataxia. Typical diets are rich in foods
with sufficient copper, and therefore deficiency is rare; however, children with certain conditions are at risk.
Celiac disease may cause impaired copper absorption, which may be improved with a gluten-free diet and
supplementation. A rare genetic disorder, Menkes disease causes impaired copper absorption, which leads
to failure to thrive, impaired cognition, and seizures. These children often have “kinky” hair. Treatment of this
disorder with subcutaneous copper supplementation within the first few weeks after birth leads to decreased
morbidity and mortality.
Signs and symptoms of copper toxicity include liver damage, abdominal pain, nausea, vomiting, and diarrhea.
Toxicity is uncommon other than in children with specific genetic disorders (eg, Wilson disease) or those with
excess exposure to copper from environmental factors (eg, copper leaching into drinking water from copper
plumbing).
Iron deficiency would present with anemia. Although weakness can be seen in individuals with iron deficiency
anemia, the girl in the vignette has a normal complete blood cell count.
Zinc is present in foods such as meat, fish, beans, whole grains, and fortified cereals. Beginning at age 6
months, infants should start receiving foods rich in zinc. This is especially important for those who are
breastfed. At the time of birth, breast milk is rich in zinc; however, the level of zinc decreases over the
following 6 months, regardless of maternal intake. Zinc is important for growth and immune function. Zinc
deficiency can lead to diarrhea, decreased growth, decreased appetite, and loss of the senses of taste and
smell. More severe deficiency can lead to alopecia, skin manifestations (eg, perioral and perirectal rash in
infants), and frequent infections.
Some conditions place children at increased risk of developing zinc deficiency, including genetic disorders
that result in a child’s having decreased absorption of zinc or a mother’s having low levels of zinc in her breast
milk. Children consuming a vegetarian diet can be at increased risk of experiencing zinc deficiency; the family
should meet with a nutritionist to review their diet and consider supplementation with a multivitamin, if
indicated.
Excessive zinc supplementation can be associated with decreased absorption of other nutrients (eg, copper
and magnesium) as well as toxicity. Symptoms of zinc toxicity include nausea, vomiting, dizziness, and
headache. Long-term exposure to high levels of zinc can result in lower levels of high-density cholesterol.
Suggested Reading(s)
Copper: fact sheet for health professionals. National Institutes of Health, Office of Dietary
Supplements. Accessed March 23, 2023. https://ods.od.nih.gov/factsheets/Copper-HealthProfessional/
Dietary Guidelines for Americans 2020-2025. US Department of Agriculture. Accessed February 22,
2023. https://www.dietaryguidelines.gov/sites/default/files/2020-
12/Dietary_Guidelines_for_Americans_2020-2025.pdf
Kleinman RE, Greer FR, eds. Pediatric Nutrition. American Academy of Pediatrics; 2020. Accessed
February 22, 2023. Pediatric Nutrition Online
Magnesium: fact sheet for health professionals. National Institutes of Health, Office of Dietary
Supplements. Accessed March 23, 2023. https://ods.od.nih.gov/factsheets/Magnesium-
HealthProfessional/
Zinc: fact sheet for health professionals. National Institutes of Health, Office of Dietary Supplements.
Accessed March 23, 2023. https://ods.od.nih.gov/factsheets/Zinc-HealthProfessional/
Content Domain
Nutrition
An 8-year-old boy with autism is seen in the emergency department for evaluation of fatigue. For the past 3
days, his energy level and appetite have been reduced, and he has appeared pale. Today, he is sleepier than
usual. The boy has a severely limited diet owing to his food preferences. He typically eats cheese snack
crackers, peanut butter sandwiches on white bread, and cinnamon toaster pastries. He drinks apple juice
and water.
On physical examination, the boy is afebrile, his blood pressure is 95/47 mm Hg, his heart rate is 100
beats/min, and his respiratory rate is 16 breaths/min. His weight is 51 kg, his height is 149 cm, and his body
mass index is 23 kg/m2. He is interactive but appears fatigued. His mucous membranes are dry. He has
decreased deep tendon reflexes. His skin is pale and there is a 3-cm ecchymosis on his left shoulder. The
remainder of his physical examination findings are unremarkable.
Of the following, the MOST likely nutrient deficiency causing this boy’s findings is
A. folate
B. vitamin B12
C. vitamin C
D. vitamin D
Correct answer is B
PREP Pearl(s)
Water-soluble vitamins (B complex, vitamin C) are absorbed in the small intestine and renally excreted.
Fat-soluble vitamins (A, D, E, K) are chylomicron dependent for absorption and stored in the liver and
adipose tissue.
Children with oral aversion, restrictive diets, fat malabsorption, and intestinal inflammation are at risk
of experiencing vitamin deficiencies.
Critique
The boy described in the vignette has a very limited diet and is at risk of experiencing nutritional deficiencies.
His pallor, fatigue, decreased deep tendon reflexes, macrocytic anemia, pancytopenia, and lack of animal
protein in his diet suggest vitamin B12 (cobalamin) deficiency.
Although folate deficiency can also cause macrocytic anemia, neurologic manifestations are not common. In
addition, many wheat-containing snack foods, which this boy eats, are fortified with folate. Vitamin C
deficiency can cause fatigue and easy bruising. However, the boy in the vignette drinks apple juice, which is
typically fortified with vitamin C, and he does not exhibit other suggestive findings (eg, bleeding gums).
Vitamin D deficiency is unlikely given the boy’s clinical presentation and normal calcium, phosphorus, and
alkaline phosphatase values.
Vitamin B12 is a water-soluble vitamin. Water-soluble vitamins (B complex, C) are easily absorbed in the small
intestine, metabolized, and then used or renally excreted. In contrast, fat-soluble vitamins (A, D, E, K) require
chylomicrons for intestinal absorption and are stored in the liver and adipose tissue. Table 1 reviews the
absorption, storage, and metabolism of water-soluble and fat-soluble vitamins.
It is important for practitioners to be aware of how various medical conditions can place children at risk of
experiencing nutritional deficiencies, as seen in the following examples:
Oral aversion or a restrictive diet increases the risk of experiencing a variety of nutritional
deficiencies (Table 2).
Difficulty with fat digestion (eg, pancreatic insufficiency or cholestasis) increases the risk of
experiencing fat-soluble vitamin deficiency.
Intestinal inflammation or having had an intestinal resection increases the risk of experiencing
nutritional deficiencies, determined by the region affected (eg, vitamin B12 deficiency in children
with ileal resection or severe ileal Crohn disease).
Suggested Reading(s)
Abali EE, Cline SD, Franklin DS, Viselli SM. Micronutrients: vitamins. In: Abali EE, Cline SD, Franklin DS,
Viselli SM, eds. Lippincott Illustrated Reviews: Biochemistry. 8th ed. Wolters Kluwer Health; 2021.
Diab L, Krebs NF. Vitamin excess and deficiency. Pediatr Rev. 2018;39(4):161-179. doi:10.1542/pir.2016-
0068
Sethuraman U. Vitamins. Pediatr Rev. 2006;27(2):44-55. doi:10.1542/pir.27-2-44
DeWitt TG. Vitamin D inadequacy. Point-of-Care Quick Reference. Pediatric Care Online. American
Academy of Pediatrics. December 29, 2020. Accessed September 1, 2023. Pediatric Care Online
Content Domain
Nutrition
Vitamin A (Retinol) Absorbed and esterified in intestinal Stored in the liver Hydrolyzed to retinol and transported out Blindness, follicular hyperkeratosis, impaired
cells to form retinyl esters; as retinyl esters of the liver with retinol binding protein growth, keratomalacia, nyctalopia, photophobia
chylomicron dependent
Vitamin D Absorbed in the small intestine, Stored in adipose Dietary vitamin D (D2 and D3) converted to Poor growth, rickets, tetany
chylomicron dependent; also tissue primarily 25-OH-D3 in the liver; then is metabolized
absorbed by skin to 1, 25-diOH-D3 in the kidney
Vitamin E Absorbed in the small intestine, Stored in liver Alpha-tocopherol binds to transfer protein Ataxia, double vision, hemolytic anemia, loss
(alpha-tocopherol) chylomicron dependent and low-density lipoprotein and secreted of deep tendon reflexes, muscle weakness,
into bloodstream sensory loss
Vitamin K Absorbed in jejunum; also produced Limited storage Metabolized in the liver to allow for Ecchymosis, hemorrhage, mucosal bleeding,
by intestinal bacteria (used rapidly) production of clotting factors petechiae, purpura
Water-Soluble Vitamins
B1 (thiamine) Absorbed in the small intestine Limited storage Converted by thiamine diphosphokinase Beriberi, fatigue, cardiac failure, polyneuritis,
enzyme to active form (thiamine Wernicke encephalopathy
pyrophosphate)
B2 (riboflavin) Absorbed in the proximal small Limited storage Enterocyte phosphatases convert dietary Blurring of vision, cheilosis
intestine riboflavin to free riboflavin
B3 (niacin) Absorbed easily in the small Limited storage Metabolized to NAD, NADPH Pellagra (rash, diarrhea, stomatitis, glossitis,
intestine (some in stomach) mental status changes)
B6 (pyridoxine) Absorbed rapidly by small intestine Limited storage Metabolized to pyridoxal phosphate Irritability, seizures, sensory ataxia
B9 (folate) Absorbed in the proximal small Limited storage Metabolized by dihydrofolate reductase to Macrocytic anemia, occasional pancytopenia,
intestine tetrahydrofolic acid irritability, failure to gain weight
B12 (cobalamin) Absorbed in the ileum when bound Stored in the liver Metabolized into coenzyme forms Ataxia, loss of deep tendon reflexes, macrocytic
by intrinsic factor anemia, occasional pancytopenia, sensory loss
C (ascorbic acid) Absorbed in the small intestine Limited storage Acts as a cofactor for many enzymes and Bone pain, joint swelling, mucosal bleeding,
hormones petechiae, poor wound healing, purpura, scurvy
NAD, nicotinamide adenine dinucleotide; NADPH, nicotinamide adenine dinucleotide phosphate.
chiectasis should be considered.
A 2-month-old infant is undergoing a routine health supervision visit. He was born at 33 weeks’ gestation with
a birth weight of 3.96 lbs (1,800 g). He stayed in the neonatal intensive care unit for 12 days, requiring
positive airway pressure and oxygen support for the first 3 days after birth and phototherapy for
hyperbilirubinemia. He was fed cow milk–based premature infant formula via nasogastric tube for the first
few days and then orally. His discharge weight was 3.83 lbs (1,740 g). The infant’s mother recently switched
him to a soy protein-based formula owing to fussiness and “spitting up.” His weight today is 7.05 lbs (3,200 g),
and his physical examination findings are normal. The mother is counseled regarding the use of soy protein–
based formula for this infant.
Of the following, the BEST information to provide this infant’s mother is that the use of this formula
Correct answer is A
PREP Pearl(s)
Premature infants require transitional or enriched formulas to provide adequate calories for growth
and minerals (calcium, magnesium, phosphorus) to prevent osteopenia.
The indications for soy protein–based formulas are galactosemia, congenital lactase deficiency, and
family preference for a vegan diet.
Soy protein–based formulas are not recommended for management of colic, gastroesophageal reflux,
or milk protein allergy.
Critique
Of the response choices, the best information to provide the mother of the preterm infant in the vignette is
that soy protein–based formula does not provide adequate minerals. Preterm infants have increased
requirements for calcium and phosphorus that are not met with a soy protein–based formula. Soy formulas
also contain increased concentrations of aluminum, which competes with calcium for absorption, leading to
further bone demineralization and contributing to osteopenia of prematurity.
This infant has had adequate weight gain (20-30 g/day) since discharge from the neonatal intensive care unit,
but he should remain on transitional formula at this time. Preterm infants are generally discharged from the
neonatal intensive care unit on enriched or transitional formulas (22 kcal/oz). In addition to a higher caloric
value, these formulas provide additional calcium, magnesium, and phosphorus, which is required to prevent
osteopenia. Transitional formulas are recommended for neonates born at 34 weeks’ gestation or earlier or
who have a discharge weight of more than 3.96 lbs (1,800 g), and are continued until age 6 to 9 months.
Standard soy protein–based formulas contain only 19 to 20 kcal/oz, which is less than what is recommended
for this infant. There is no evidence that soy protein–based formulas help with symptoms of colic or reflux.
Concern has been raised that high concentrations of phytoestrogens and isoflavones in soy protein–based
formulas can lead to estrogen-related adverse effects, but the evidence is inconclusive.
Nutritional requirements for healthy term and preterm infants are shown in the Table. These requirements
may be increased with specific medical conditions.
The American Academy of Pediatrics recommends exclusive breastfeeding for 6 months after birth, followed
by the introduction of other foods with continued breastfeeding until at least age 1 year. Breastfeeding
provides health benefits to all infants, including those born prematurely, by protecting against infectious
diseases and reducing the risk of sudden infant death syndrome and future obesity. Parents may be unable
or elect not to breastfeed or may prefer a combination of breastfeeding and formula supplementation. There
are rare medical contraindications to breastfeeding (eg, galactosemia, maternal HIV infection).
The Infant Formula Act of 1980 ensures quality control of formulas by the US Food and Drug Administration.
Standard infant formulas have a caloric density of 19 to 20 kcal/oz and are available in powder, liquid
concentrate, and ready-to-feed formulations. Standard infant formulas contain cow milk as the protein
source, lactose as the carbohydrate, and a blend of vegetable oils as the fat source. As of 2002, most
standard formulas also contain long-chain polyunsaturated fatty acids.
The carbohydrate sources of soy protein–based formulas include glucose polymers, corn syrup, maltodextrin,
and sucrose. The fat source is a combination of vegetable oils. Indications for the use of soy formulas include
galactosemia, congenital lactase deficiency, and family preference for a vegan diet. Soy formulas are not
recommended for cow milk protein allergy owing to cross reactivity with soy in 5% to 15% of cases. Infants
with cow milk protein allergy can be treated with partially or extensively hydrolyzed cow milk protein
formulas (short-chain peptides and free amino acids) or elemental formulas (free amino acids).
Suggested Reading(s)
Milbrandt TP. Standard infant formula and formula feeding—cow milk protein formulas. Pediatr Rev.
2017;38(5):239-240. doi:10.1542/pir.2016-0211
Milbrandt TP. Specialized infant formulas. Pediatr Rev. 2017;38(5):241-242. doi:10.1542/pir.2016-0212
Stellwagen LM, Kim JH, Hurst NM. Optimizing nutrition for the preterm, very low-birth-weight infant
after discharge from neonatal intensive care. In: McInerny TK, Adam HM, Campbell DE, Foy JM, Kamat
DM, eds. American Academy of Pediatrics Textbook of Pediatric Care. 2nd ed. American Academy of
Pediatrics; 2016:chap 88. Accessed September 1, 2023. Pediatric Care Online
Content Domain
Nutrition
Courtesy of S. Naganathan
AAP PREP 2024 - Question 175/267 Nutrition Question 5/6
A 3-month-old infant is seen in the office for a health supervision visit. The family has been struggling
financially. They have been unable to afford infant formula and have instead been giving the infant whole
cow milk.
Of the following, this infant’s diet puts them at greatest risk for
B. kidney injury
D. vitamin D toxicity
Correct answer is A
PREP Pearl(s)
For families that are unable to or choose not to breastfeed, a cow milk protein (or soy-based protein
for a vegetarian diet) infant formula is considered an appropriate substitute.
The provision of pasteurized cow milk to infants younger than 12 months can result in iron deficiency
anemia due to poor absorption of iron and possible intestinal blood loss.
The level of iron is low in both human and cow milk but is highly bioavailable from human milk,
whereas bovine milk proteins are efficient inhibitors of iron absorption.
Critique
The infant in the vignette, being fed whole cow milk, is at greatest risk for iron deficiency anemia. The
American Academy of Pediatrics (AAP) recommends that infants exclusively breastfeed for the first 6 months
after birth and then continue breastfeeding with the addition of complementary foods for the second 6
months. For families that are unable to or choose not to breastfeed, a cow milk protein (or soy-based protein
for a vegetarian diet) infant formula is considered an appropriate substitute.
Cow milk protein formula typically contains approximately 40% more protein than breast milk, has a whey-
casein ratio that is more casein dominant (6-7 times the amount in human milk), and has higher
concentrations of the amino acids threonine, phenylamine, valine, and methionine. There are no known
clinical effects of this amino acid composition difference. The higher levels of casein in cow milk make it
harder to digest. Recently, infant formula products have become available with a whey-casein ratio that more
closely mimics human milk. Iron, a crucial component of cow milk protein formulas, is present at a
concentration of 12 mg/L.
The AAP recommends that pasteurized cow milk should only be provided to children after 12 months of age.
The provision of cow milk to infants has been associated with a number of nutritional deficiencies; it is most
strongly associated with iron deficiency anemia. The cause of this anemia is multifactorial. The level of iron is
low in both human and cow milk but is highly bioavailable from human milk, whereas bovine milk proteins
are efficient inhibitors of iron absorption. Approximately 50% of the available iron is absorbed from human
milk as opposed to 10% from cow milk. In addition, infant ingestion of pasteurized whole cow milk is
associated with occult intestinal blood loss; young infants can have nutritionally significant loss. The
significance and amount of occult intestinal blood loss decreases as an infant approaches 12 months of age.
Protein provides approximately 7% of the calories in human milk and 20% of the calories in cow milk.
Compared with human milk, whole cow milk has a lower content of zinc, niacin, and vitamins E and C. It also
has 3 times higher amounts of sodium and potassium, 4 times the amount of calcium, and 6 times the
amount of phosphorus. The higher protein, sodium, potassium, and phosphorus ratios increase the renal
solute load. However, there is no evidence that this causes kidney injury in healthy term infants.
The calorie content of whole cow milk is approximately equivalent to that in infant formula and human breast
milk. Cow milk does not cause vitamin D toxicity in infants.
Suggested Reading(s)
Hall RT, Carroll RE. Infant feeding. Pediatr Rev. 2000;21(6): 191–200. doi:10.1542/pir.21-6-191
Jiang T, Jeter JM, Nelson SE, et al. Intestinal blood loss during cow milk feeding in older infants:
quantitative measurements. Arch Pediatr Adolesc Med. 2000;154(7):673–678.
doi:10.1001/archpedi.154.7.673
Kleinman RE, Greer FR. Formula feeding of term infants. In: Kleinman RE, Greer FR, eds. Pediatric
Nutrition. 8th ed. American Academy of Pediatrics; 2020.
Leung AK, Sauve RS. Whole cow's milk in infancy. Paediatr Child Health. 2003;8(7):419-21.
doi:10.1093/pch/8.7.419
Meek JY, Noble L; Section on Breastfeeding. Policy statement: breastfeeding and the use of human
milk. Pediatrics. 2022;150(1):e2022057988. doi:10.1542/peds.2022-057988
Content Domain
Nutrition
A 7-day-old girl born at 28 weeks’ gestation with a birth weight of 1,250 g is transitioning from parenteral to
enteral nutrition using maternal breast milk with human milk fortifier. To maintain optimal growth, the
nutritionist is recommending preparation of the fortified breast milk to 24 kcal/oz.
Of the following, the condition this neonate should be MOST closely monitored for is
A. hypocalcemia
B. hypokalemia
C. hyponatremia
D. hypophosphatemia
Correct answer is C
PREP Pearl(s)
Preterm neonates have a higher energy requirement than those born at term.
The use of feedings supplemented with human milk fortifier can put a neonate at risk for
hyponatremia. Feedings concentrated to 27 kcal/kg/oz place the neonate at risk for hypercalcemia and
hyperphosphatemia.
Liquid human milk fortifier is preferred over the powdered form because of the greater risk of
bacterial contamination of powdered forms.
Critique
The use of a human milk fortifier (HMF) to increase the caloric value of breast milk puts the neonate in the
vignette at risk for hyponatremia due to the low sodium content of these feedings coupled with the increased
urine sodium losses seen in preterm neonates. For neonates who require high concentration feedings (>27
kcal/oz), there is an increased risk of hypercalcemia and hyperphosphatemia due to the mineral content of
this preparation. Potassium levels are not significantly affected by the addition of HMF. During the transition
to enteral feeds, routine monitoring of electrolytes is recommended until normal levels are stable and the
neonate is no longer receiving intravenous fluids or oral electrolyte supplements.
Term neonates require 100 to 120 kcal/kg/d for optimal growth, whereas preterm neonates require 100 to
150 kcal/kg/d. Healthy neonates require 40 to 60 kcal/kg/d to maintain their basal metabolic rate. Feeding,
digestion, elimination, and absorption require an additional 30 to 50 kcal/kg/d. Thermoregulation requires a
significant amount of energy; this requirement is even higher for preterm neonates with inadequate
subcutaneous fat stores. Those with conditions that increase metabolic demands (eg, lung disease,
congenital heart disease, and sepsis) have higher energy needs.
For preterm neonates, appropriate growth is critical for optimal neurodevelopmental outcomes. When
preterm neonates are ready for enteral feedings, it is essential to ensure that optimal calories, protein,
micronutrients, and electrolytes are provided. Growth charts designed specifically for preterm infants are
helpful to monitor for optimal growth (Figure).
Reprinted with permission from Fenton TR, Kim JH. A systematic review and meta-analysis to revise the
Fenton growth chart for preterm infants. BMC Pediatrics. 2013,(13)59:7.
Current evidence supports the use of liquid HMF over powdered HMF because of the greater risk of bacterial
contamination of powdered products. Both human milk–derived and bovine milk–derived forms of liquid
HMF are available; evidence is currently insufficient to recommend one form over the other.
Suggested Reading(s)
Ben XM. Nutritional management of newborn infants: practical guidelines. World J Gastroenterol.
2008;14(40):61336139. doi:10.3748/wjg.14.6133
Carlson SJ, Ziegler EE. Feeding: NICU Handbook. University of Iowa Stead Family Children’s Hospital;
May 18, 2022. https://uihc.org/childrens/educational-resources/feeding-nicu-handbook
Schultz EV, Wagner CL. Powdered to liquid human milk fortifiers in the preterm infant. Neoreviews.
2021;22(6):e360-e369. doi:10.1542/neo.22-6-e360
Content Domain
Nutrition
A 5-year-old girl undergoing treatment for acute lymphoblastic leukemia is seen in the emergency
department for a new-onset febrile upper respiratory tract infection with cough and runny nose. Her vital
signs include an oral temperature of 39 °C, heart rate of 122 beats/min, respiratory rate of 32 breaths/min,
blood pressure of 110/70 mm Hg, and oxygen saturation of 95% in room air. On physical examination, she is
awake, alert, and in no apparent distress. She has alopecia, conjunctival pallor, moist mucous membranes,
and no oral sores or lesions. Her lungs are clear to auscultation bilaterally, there is a 3/6 systolic ejection
murmur, and her abdomen is soft, nontender, and nondistended with no palpable organomegaly. She has
no focal neurologic deficits, rashes, or tenderness or erythema at the site of her central venous catheter.
Neutrophils 26.5%
Lymphocytes 70%
Monocytes 3.5%
Of the following, the BEST next step in this child’s management is to administer
A. empiric antibiotics
B. uid resuscitation
D. platelets
PREP Pearl(s)
Children with febrile neutropenia who are undergoing treatment for cancer require prompt treatment
with empiric antibiotics, including Pseudomonas coverage.
Prophylactic antimicrobials should be administered to pediatric patients with cancer to help prevent
infections.
A rectal temperature should not be taken in a child undergoing cancer treatment because of the risk of
bacterial seeding into the bloodstream.
Critique
For the child in the vignette, the best next step in management is to administer empiric antibiotics. The girl
has febrile neutropenia (absolute neutrophil count [ANC] <500/μL [0.50 ×109/L]) and an oral temperature
higher than 38°C, which is an oncologic emergency. Rectal temperatures should not be taken in children
undergoing cancer treatment because of the risk of bacterial seeding into the bloodstream. Infection is a
major cause of morbidity and mortality in children with cancer; therefore, prompt diagnosis and
management are of utmost importance. Blood cultures should be obtained peripherally and from the central
venous catheter (CVC) before antibiotic administration. A urine culture should be obtained when clinically
indicated. Broad-spectrum intravenous antibiotics, including Pseudomonas coverage, should be
administered urgently because of the risk of an overwhelming bacterial infection. Appropriate antibiotic
therapy should be narrowed as soon as possible to treat suspected or culture-positive infections. The
duration of treatment should be guided by the confirmed or suspected infection and extended if needed
until the ANC improves. Empiric antifungal therapy should be considered in the presence of certain risk
factors (eg, active graft-vs-host disease, persistent fever, or concern for sinus infection).
Children undergoing cancer treatment are at risk of infections due to an immunocompromised state that
results from both the illness and the therapy. Disruption of the mucosal barrier, suppression of B and T cells,
and the presence of a CVC are all risk factors for infection.
Children undergoing cancer treatment are maintained with prophylactic antimicrobials because of their risk
of infection; the timing, type, and duration of need for this prophylaxis are determined by the
immunosuppressive nature of their therapy. For example, individuals undergoing treatment for acute
myeloid leukemia (AML) receive highly immunosuppressive therapy, and inpatient administration of
antibiotic prophylaxis may be recommended until the neutrophils recover. Additional methods of infection
prophylaxis include antibiotic or ethanol lock of the CVC and chlorhexidine baths. Pneumocystis jiroveci
pneumonia prophylaxis is given to all children receiving active anticancer therapy during treatment and for
several months after treatment until T-cell immunity improves. Antiviral and antifungal prophylaxis is
administered to children with certain high-risk conditions (eg, AML or hematopoietic stem cell
transplantation). The Table lists the risk factors and prophylaxis for each type.
The child in the vignette does not currently require fluid resuscitation because there is no evidence of
dehydration or hemodynamic instability on physical examination. Packed red blood cells will need to be
administered; however, this is not as urgent as the need for antibiotics because she is hemodynamically
stable. A platelet transfusion is not indicated; there are no bleeding signs or symptoms or need for invasive
procedures.
Suggested Reading(s)
Argawal AK, Feusner J. Supportive care for patients with cancer. In: Lanzkowsky P, Lipton J, Fish JD, eds.
Lanzkowsky’s Manual of Pediatric Hematology and Oncology. 6th ed. Elsevier Inc; 2016:620-655.
Lighter-Fisher J, Stanley K, Phillips M, Pham V, Klejmont LM. Preventing infections in children with
cancer. Pediatr Rev. 2016;37(6):247-258. doi:10.1542/pir.2015-0059
Content Domain
Oncology
Abbreviations: ALL, acute lymphoid leukemia; AML, acute myeloid leukemia; CVC, central
venous catheter; HSCT, hematopoietic stem cell transplantation; HSV, herpes simplex
virus; VZV, varicella zoster virus.
Courtesy of A. Appiah-Kubi
AAP PREP 2024 - Question 178/267 Oncology Question 2/4
A previously healthy 3-year-old child is seen in the emergency department for 2 days of abdominal pain and
nonbloody, nonbilious emesis. He is fatigued and has decreased oral intake. He appears tired but nontoxic.
He has a temperature of 37 °C, a heart rate of 122 beats/min, a respiratory rate of 22 breaths/min, blood
pressure of 110/70 mm Hg, and oxygen saturation of 100% in room air. He is moaning and holding his
abdomen. His mucous membranes are moist, lungs are clear to auscultation bilaterally, and his abdomen is
distended but soft, with some guarding and no rebound tenderness. There are no focal neurologic deficits.
The remainder of his examination findings are normal. Abdominal radiography (Figure) reveals a moderate
stool burden and a right upper quadrant round mass measuring 3.7 × 3.1 cm with amorphous calcifications.
A. antihypertensive
B. enema
D. pain medication
Correct answer is D
PREP Pearl(s)
Neuroblastoma is the most common extracranial solid tumor in children.
The presentation of neuroblastoma varies broadly from an asymptomatic incidental finding to a
medical emergency, such as spinal cord compression.
Abdominal sonography is the initial imaging study of choice for suspected neuroblastoma.
Critique
The boy in the vignette has a radiographic finding of a circular abdominal mass with calcification, which is
concerning for neuroblastoma. It is clear that the child is currently in pain, as evidenced by his moaning and
holding of his abdomen. He is currently clinically stable, therefore, the important and best next step is to
relieve the child’s pain. Once his pain is addressed, the additional evaluation required to diagnose his
condition can proceed. The boy's tachycardia and hypertension are most likely due to pain and discomfort;
pain management should ameliorate those conditions as well.
Neuroblastoma is the most common extracranial solid tumor in children and the most common cancer
during infancy. Ninety percent of cases occur in children younger than 5 years of age. The tumor occurs in
neural crest cells of the sympathetic ganglia or adrenal glands, leading to a mass in the abdomen (most
commonly in the adrenal glands), chest, neck, or paraspinal region. Neuroblastoma may be detected in an
asymptomatic child as an incidental finding of a mass on physical examination or radiography. The signs and
symptoms of neuroblastoma may be nonspecific (eg, fatigue, fever, irritability) or more specific (eg,
constipation, pain), or medical emergencies (eg, spinal cord compression). The Table lists some signs and
symptoms in relation to the anatomical site.
Due to involvement of the sympathetic system, the following paraneoplastic syndromes may occur:
Kerner-Morrison syndrome: intractable watery diarrhea which can lead to failure to thrive and
hypokalemia, caused by secretion of vasoactive intestinal peptide
Opsoclonus myoclonus ataxia syndrome: involuntary random rapid eye movements, jerky movements
of extremities, and developmental delays; it is believed to be immune mediated
Abdominal sonography
Initial imaging of choice
to locate a tumor
will show calcifications
Magnetic resonance imaging
abdomen, pelvis, chest, spine
to evaluate for primary masses and metastasis
Computed tomography
abdomen, pelvis, chest
to evaluate for primary masses or metastasis
I123-meta-iodobezylguanidine (MIBG) scintigraphy
detects bony metastasis and soft tissue disease
90% of tumors have uptake
118F-flourodeoxyglocose positron emission tomography (FDG-PET)
Detects primary tumor and metastasis
used for monitoring if tumor is not MIBG avid
Bone marrow
Bilateral aspirates and biopsies
Biopsy of tumor
open biopsy is preferred to ensure adequate tissue for all testing.
This child does not require treatment with an antihypertensive. Though the blood pressure is elevated, it is
most likely due to pain and should improve once the pain resolves. If hypertension persists despite adequate
pain control, antihypertensive medication may be required. Hypertension may occur in neuroblastoma if the
renal vasculature is compromised, though it is not a common finding.
Despite the presence of a moderate stool burden on radiography, an enema is not required at the present
time. His constipation can be addressed at a later time.
A normal saline bolus is not needed, as there are no findings to support dehydration. The boy’s tachycardia is
most likely a result of his pain.
Suggested Reading(s)
Allen-Rhoades W, Whittle SB, Rainusso N. Pediatric solid tumors of infancy: an overview. Pediatr Rev.
2018;39(2):57-67. doi:10.1542/pir.2017-0057
Hackney L, Welch JG, Schwartz CL. Cancers in childhood. In: McInerny TK, Adam HM, Campbell DE,
DeWitt TG, Foy JM, Kamat DM, eds. American Academy of Pediatrics Textbook of Pediatric Care.
American Academy of Pediatrics; 2021:chap 225. Accessed September 1, 2023. Pediatric Care Online
Park JR, Bagatell R. Neuroblastoma. In: Lanzkowsky P, Lipton J, Fish JD. Lanzkowsky’s Manual of
Pediatric Hematology and Oncology. 6th ed. Elsevier Inc; 2016:473-490.
Sharma R, Mer J, Lion A, Vik TA. Clinical presentation, evaluation, and management of neuroblastoma.
Pediatr Rev. 2018;39(4):194-203. doi:10.1542/pir.2017-0087
Content Domain
Oncology
Courtesy of A. Appiah-Kubi.
AAP PREP 2024 - Question 179/267 Oncology Question 3/4
A 15-year-old boy is seen for right proximal humerus swelling that began approximately 6 months ago,
shortly after he received “birthday punches” at that site. The swelling remained unchanged for several
months after his birthday but recently began increasing in size. He denies any limitation in range of motion
but has some soreness in the area. There has been no change in the overlying skin, weight loss, shooting
pain down his arm, or pain in any other area of the body. He has been afebrile.
On physical examination, the boy is in no apparent distress. Vital signs are normal for age, and his body mass
index is at the greater than 95th percentile for age. There is mild gynecomastia and a café au lait spot on the
left chest. A soft mass (approximately 3 × 5 cm) is palpable on his right upper extremity, with mild tenderness
and no erythema. There is no palpable lymphadenopathy or organomegaly. His sexual maturity rating is 5;
testes are descended with no palpable mass. The remainder of his examination findings are unremarkable.
Of the following, the BEST next step in this boy’s management is to obtain
A. computed tomography
C. plain radiography
Correct answer is C
PREP Pearl(s)
Pain and swelling associated with a palpable mass in an extremity should prompt investigation for a
bone tumor.
The initial imaging of choice to evaluate for a bone tumor is plain radiography.
The treatment of choice for a malignant bone tumor is neoadjuvant chemotherapy followed by limb
salvage surgery.
Critique
The child in the vignette has a mass on his proximal humerus that has been present for several months and
is now increasing in size. This finding raises concern for a bone malignancy. The initial imaging of choice to
evaluate for a bone tumor is plain radiography. If bony changes are noted, a decision can be made regarding
the need for and choice of additional imaging, such as magnetic resonance imaging (MRI), computed
tomography (CT), or positron emission tomography (PET).
Figure 1 demonstrates a lesion within the right humeral metaphysis with internal sclerosis and unmineralized
bone in the soft tissues compatible with osteosarcoma. Findings on radiographic imaging of bone tumors
may include a Codman triangle, due to the rapid formation of new bone, lifting the cortex; a sclerotic,
destructive lesion with a sunburst appearance due to demineralization of the bone (osteosarcoma); or a
periosteal reaction, leading to an onion skin appearance (Ewing sarcoma).
Courtesy of J. Amodio
Figure 1. A lesion within the right humeral metaphysis with internal sclerosis and unmineralized bone in
the soft tissues compatible with osteosarcoma.
The most common malignant bone tumors in children are osteosarcoma and Ewing sarcoma. The clinical
signs and symptoms may be subtle, leading to a delay in diagnosis. Associated signs and symptoms may
include the following:
Pathologic fracture + ++
Fever ++
An urgent referral for biopsy of the lesion should take place when a malignant bone tumor is suspected. The
biopsy should be performed by a physician with specific expertise (eg, oncologic orthopedic surgeon). Once
pathology is determined, a pediatric oncologist will perform a staging workup, which includes chest CT to
evaluate for lung nodules, MRI of the nearby joint(s) to assess the soft tissue and neighboring structures, and
whole-body fluorodeoxyglucose PET. In the case of Ewing sarcoma, bilateral bone marrow biopsy specimens
may be obtained to evaluate for bone metastasis. Figure 2 shows the most common primary tumor sites for
osteosarcoma and Ewing sarcoma.
Additional investigations and procedures, in anticipation of therapy, include complete blood cell count,
comprehensive metabolic panel, urinalysis and assessment of renal function, echocardiography,
audiography, and fertility preservation.
Osteosarcoma may be associated with some genetic predisposition syndromes, including retinoblastoma
(tumor suppressor RB1 mutation), Diamond-Blackfan anemia, Werner syndrome, Rothmund-Thompson
syndrome, and Li-Fraumeni syndrome (tumor suppressor TP53 mutation).
The treatment of malignant bone tumors includes neoadjuvant (before surgery) chemotherapy, limb salvage
surgery, and then adjuvant chemotherapy. The use of radiation therapy is dependent on the radiosensitivity
of the tumor. Ewing sarcoma is radiosensitive; radiation therapy may be used as neoadjuvant therapy before
surgical resection or as adjuvant therapy if there are positive margins postoperatively or for lesions that are
not resectable. Osteosarcoma is not very radiosensitive; therefore, radiation therapy is reserved for palliative
symptom control for metastatic disease.
It is important to be mindful of the late effects of the chemotherapy and radiation therapy and ensure
appropriate post-treatment monitoring.
Suggested Reading(s)
Federman N, Van dyne EA, Bernthal N. Malignant bone tumors. In: Lanzkowsky P, Lipton J, Fish JD.
Lanzkowsky’s Manual of Pediatric Hematology and Oncology. 6th ed. Elsevier Inc; 2016:542-543.
Gereige R, Kumar M. Bone lesions: benign and malignant. Pediatr Rev. 2010;31(9):355-362; quiz 363.
doi:10.1542/pir.31-9-355
Hackney L, Welch JG, Schwartz CL. 2016. Cancers in childhood. In: McInerny TK, Adam HM, Campbell
DE, DeWitt TG, Foy JM, Kamat DM, eds. American Academy of Pediatrics Textbook of Pediatric Care.
American Academy of Pediatrics; 2021:chap 348. Pediatric Care Online
Self C, MacQuarrie KL, Cost CR. Osteosarcoma/Ewing sarcoma. Pediatr Rev. 2022;43(5):256-265.
doi:10.1542/pir.2021-005065
Content Domain
Oncology
Reprinted with permission from Self C, MacQuarrie KL, Cost CR. Osteosarcoma/Ewing
sarcoma. Pediatr Rev. 2022;43(5):258.
FIGURE 2: Common primary tumor sites for osteosarcoma and Ewing
sarcoma.
AAP PREP 2024 - Question 180/267 Oncology Question 4/4
An 8-year-old is found to have hearing loss after undergoing treatment for medulloblastoma. Their treatment
consisted of surgery, radiation therapy, and adjuvant chemotherapy including vincristine, cisplatin, and
lomustine.
Of the following, the treatment that MOST likely caused the child’s newly identified condition is
A. cisplatin
B. lomustine
C. radiation
D. vincristine
Correct answer is A
PREP Pearl(s)
It is important to counsel families about the potential immediate and late effects of chemotherapeutic
agents.
All childhood cancer survivors must be followed long term in a survivorship clinic to monitor for late
effects of chemotherapy or other therapies.
Critique
The child in the vignette underwent treatment for a brain tumor (medulloblastoma). Of the therapies the
child received, the most likely cause of their hearing loss is exposure to cisplatin. Hearing loss is a known side
effect of cisplatin, therefore, children’s hearing should be assessed prior to treatment and reassessed at
several time points during and after treatment. Other side effects of cisplatin include delayed nausea and
vomiting, cytopenias (or myelosuppression), and electrolyte abnormalities (eg, hypomagnesemia).
Common side effects of lomustine include nausea, vomiting, and cytopenias. It may also cause mouth sores,
abnormal liver function, or later-onset pulmonary fibrosis. The side effects of radiation therapy (Table 1)
depend on the area irradiated. For cranial radiation, concerning side effects may include the development of
learning disabilities, growth impairment, or hypothalamic dysfunction.
Vincristine may cause peripheral neuropathies, presenting as numbness and tingling, foot drop, or jaw pain;
it can also cause constipation.
Most chemotherapy and other childhood cancer therapies have immediate and late effects. It is therefore
important to be knowledgeable about what side effects can be expected and offer appropriate monitoring.
All childhood cancer survivors must be followed long term in a survivorship clinic.
Table 2 lists some of the known adverse effects of various chemotherapeutic agents.
Suggested Reading(s)
Cancers in childhood. Point-of-Care Quick References. Pediatric Care Online. April 8, 2019. Accessed
September 1, 2023. Pediatric Care Online
Brinkman TM, Gurney JG. Early and often: the need for comprehensive discussion of treatment-
induced cancer late effects. Pediatrics. 2020;145(5):e20200498. doi:10.1542/peds.2020-0498
Content Domain
Oncology
Abdomen/pelvis Hypoplasia (including scoliosis) Examination of area, radiograph of spine during puberty
Liver (if in field) Liver function tests
Kidneys (if in field) Serum creatinine, urinalysis protein (24-hr collection for creatinine, protein)
Gonads (if in field) Pubertal, menstrual, and fertility history, LH, follicle-stimulating hormone, estradiol or
testosterone levels during puberty and if fertility is doubtful, semen analysis
Gastrointestinal tract Nutritional history
Extremities Hypoplasia Examination of area
For all patients, consider radiographs of bones every 5 to 10 years after ≥35-Gy radiation (risk for secondary malignancy). Examine skin for abnormal pigmented nevi (risk for second
malignancy).
Abbreviations: ECG, electrocardiogram; LH, luteinizing hormone; T, thyroxine; TSH, thyroid-stimulating hormone.
Reprinted with permission from Cancers in childhood. Point-of-Care Quick References. Pediatric Care Online. American Academy of Pediatrics.
Table 2. Late Effects of Chemotherapy.
System Affected Chemotherapeutic Agent Potential Effects Monitoring Recommendations
Central and peripheral nervous • ●Intrathecal chemotherapy • ●Cognitive dysfunction • ●Neurocognitive evaluation
• ●High doses of methotrexate or cytosine arabinoside • ●Leukoencephalopathy • ●Neurologic evaluation
• ●Seizure
• ●Hemiplegia
• ●Neuropsychiatric change
• ●Platinum-based treatment (cisplatin, carboplatin) • ●Peripheral neuropathy • ●Physical examination
• ●Hearing loss • ●Audiogram
• ●Vincristine • ●Peripheral neuropathy • ●Physical examination
• ●Corticosteroids • ●Cataracts • ●Eye examination
Respiratory • ●BCNU, CCNU • ●Pulmonary fibrosis • ●Pulmonary function tests with DLCO
• ●Busulfan • ●Chest radiography
• ●Bleomycin
Cardiac • ●Anthracyclines (daunomycin, doxorubicin, idarubicin) • ●Cardiomyopathy • ●Echocardiography
• ●Congestive heart failure • ●Electrocardiography
• ●Arrhythmias
Renal • ●Heavy metals (cisplatin, carboplatin) • ●Glomerular or tubular injury • ●Serum BUN, creatinine, electrolyte
• ●Ifosfamide • insufficiency levels
• ●High doses of methotrexate • ●Blood pressure
• ●Urinalysis
Genitourinary • ●Cyclophosphamide • ●Bladder fibrosis* • ●History
• ●Ifosfamide • ●Dysfunctional voiding • ●Urinalysis
• ●Bladder malignancy
Reproductive • ●Alkylating agents (cyclophosphamide, ifosfamide) • ●Hypogonadism, infertility, • ●History of pubertal development,
• ●Heavy metals (cisplatin, carboplatin) • ●Early menopause menstrual cycles, and sexual function
• ●LH, FSH, estradiol levels
• ●Testosterone, semen analysis (if
sexually mature)
Hepatic • ●Methotrexate • ●Liver dysfunction • ●Liver function tests
• ●6-Mercaptopurine (6MP) • ●Total bilirubin level (fractionated)
• ●Thioguanine (6TG)
Skeletal • ●Methotrexate • ●Osteopenia, osteoporosis • ●Bone density evaluation (eg, DEXA
• ●Corticosteroids • ●Avascular necrosis scan)
• ●History
• ●Physical examination,
• ●Radiographs (if indicated)
Psychosocial • ●Any cancer experience • ●Depression, anxiety, post traumatic • ●Clinical interview
stress, social withdrawal/isolation
• ●Limitations in health care and
insurance
Hematologic • ●Alkylating agents (cyclophosphamide, ifosfamide) • ●Secondary AML • ●Complete blood count
• ●Heavy metals (cisplatin, carboplatin) • ●Myelodysplasia
• ● Epipodophyllotoxins (etoposide)
• ● Anthracyclines (epibucin, mitoxantrone)
Abbreviations: AML, acute myeloid leukemia; BCNU, carmustine; BUN. blood urea nitrogen; CCNU, lomustine; DLCO, diffusing capacity for carbon monoxide; FSH, follicle-stimulating
hormone; LH, luteinizing hormone.
*
In patients treated for hemorrhagic cystitis
Adapted and reprinted with permission from Meck MM, Leary M, Sills RH. Late effects in survivors of childhood cancer. Pediatr Rev. 2006:27(7):259.
AAP PREP 2024 - Question 181/267 Palliative Medicine Question 1/4
A 34-year-old primigravida woman is seen in the labor and delivery triage unit in active labor at 23 weeks’
gestation, with delivery imminent. The neonatal team meets with her to discuss the delivery and
resuscitation plan as well as the possible long- and short-term complications of prematurity (including the
risk of neurodevelopmental impairment). The neonatal team communicates with the woman that, given the
risk of complications and poor prognosis for an infant born at 23 weeks’ gestation, they would not
recommend resuscitation but would provide resuscitation if she desired. After answering all her questions,
the neonatal team asks the woman what her wishes are for the resuscitation and care of her newborn. The
woman states that she would like her child to be resuscitated, including intubation and mechanical
ventilation, but if her child develops complications that the medical team deems catastrophic, then she does
not want her child to go through more medical interventions. She hopes the neonatal team would help her
navigate such a situation if it were to occur. The team acknowledges her decision and reassures her they will
keep her updated and provide guidance in decisions as needed.
Of the following, the concept of patient care BEST demonstrated in this situation is
A. informative
B. paternalistic
C. scientific
D. shared decision-making
Correct answer is D
PREP Pearl(s)
Periviable birth is defined as delivery occurring between 20 0/7 and 25 6/7 weeks’ gestation.
For a birth occurring at less than 21 6/7 weeks’ gestation, current guidelines do not recommend
neonatal assessment for resuscitation; for a birth between 22 0/7 and 23 6/7 weeks’ gestation,
neonatal assessment for resuscitation may be considered; and for a birth between 24 0/7 and 25 6/7
weeks’ gestation, neonatal assessment for resuscitation is recommended.
The periviable gestational age range of 22 0/7 to 23 6/7 weeks is what most would consider the ”gray
zone,” during which a process of shared decision-making involving the parent(s), caregivers, and
medical team is recommended.
Critique
The concept of patient care best demonstrated in this difficult situation is shared decision-making.
The woman in the vignette is about to experience a periviable birth, defined as a delivery between 20 0/7 and
25 6/7 weeks’ gestation. Consensus guidelines, based on current outcome-based studies, do not recommend
neonatal resuscitation assessment for a birth occurring at less than 21 6/7 weeks’ gestation. For a birth
between 22 0/7 and 23 6/7 weeks’ gestation, physicians may consider neonatal assessment for resuscitation,
and for a birth between 24 0/7 and 25 6/7 weeks’ gestation, neonatal assessment for resuscitation is
recommended.
The periviable gestational age range of 22 0/7 to 23 6/7 weeks defines what most would consider the “gray
zone,” when shared decision-making between the parents and medical team regarding resuscitation is
recommended. This care model was demonstrated in the vignette; the woman was informed and
participated in a discussion about the possible complications of neonatal assessment and resuscitation, and
she communicated with the team her wishes for resuscitation and continuing care of her newborn. Although
her desire is to optimize her newborn’s chance of survival, she is realistic and clearly states that she needs
the medical team’s expertise to help minimize the newborn’s pain and provide comfort care if complications
arise. This process of shared decision-making considers parental values, medical team values, and
appropriate newborn care that minimizes undue pain. The traditional model of patient care has been a
paternalistic approach, in which the physician tells the patient what care will be provided. This approach,
which does not consider the patient’s view, has become largely obsolete. Scientific and informative models of
patient care are also known as technical models. In a technical model, the physician provides the patient with
the diagnosis, treatment options, potential risks, and outcomes, and the patient decides what to choose
without discussion about the patient’s goals and values.
Suggested Reading(s)
American College of Obstetricians and Gynecologists; Society for Maternal-Fetal Medicine. Obstetric
care consensus No. 6: periviable birth. Obstet Gynecol. 2017;130(4):e187-e199.
doi:10.1097/AOG.0000000000002352
Boland L, Graham ID, Légaré F, et al. Barriers and facilitators of pediatric shared decision-making: a
systematic review. Implement Sci. 2019;14(1):7. doi:10.1186/s13012-018-0851-5
Chervenak FA, McCullough LB. Professional ethics and decision making in perinatology. Semin
Perinatol. 2022;46(3):151520. doi:10.1016/j.semperi.2021.151520
Soltys F, Philpott-Streiff SE, Fuzzell L, Politi MC. The importance of shared decision-making in the
neonatal intensive care unit. J Perinatol. 2020;40(3):504-509. doi:10.1038/s41372-019-0507-6
Sullivan A, Cummings C. Historical perspectives: shared decision making in the NICU. Neoreviews.
2020;21(4):e217-e225. doi:10.1542/neo.21-4-e217
Nafday, SM. Abnormalities in fetal growth. In: McInerny TK, Adam HM, Campbell DE, DeWitt TG, Foy JM,
Kamat DM, eds. American Academy of Pediatrics; 2021:chap 98. Accessed September 1, 2023. Pediatric
Care Online
Content Domain
Palliative Medicine
A 10-year-old girl is seen for a routine health supervision visit before the new school year. Her mother
reports that the girl’s grandmother, to whom she was very close, passed away suddenly 7 months ago. After
the event, the girl finished the school year online after refusing to go to school so she could be home with
her family. Over the summer, the girl did not express any interest in seeing her friends and did not want to
participate in activities she previously enjoyed. The girl reports that she is unable to stop thinking about her
grandmother and expresses how much she misses her.
The mother states that, after the death, her 5-year-old son initially kept asking when the grandmother would
be coming back, but stopped asking after a few weeks and is “back to normal.” She expected the same from
her daughter and asks what can be done to help her.
Of the following, the BEST next step in this girl’s care is to
A. advise the mother to avoid talking about the grandmother in the girl’s presence
B. allow her to continue online schooling until she feels ready to return to school
C. explain that the girl is in the “depression” stage of grief and will soon reach acceptance
D. refer the girl for counseling because she is exhibiting complicated bereavement
Correct answer is D
PREP Pearl(s)
A child’s understanding of death varies according to age and developmental level and is influenced by
family, cultural, and religious factors.
The stages of grief (anger, denial, bargaining, depression, acceptance) are not sequential or cyclical.
When a child experiences the death of a family member or loved one, information and resources for
support (including referral for grief counseling) should be provided as appropriate (eg, a child that is
exhibiting complicated grief or bereavement).
Critique
The girl in the vignette is exhibiting signs of complicated bereavement (grief), including continued behavioral
concerns (lack of interest in friends and activities previously enjoyed), preoccupying thoughts about the
deceased, and school phobia more than 3 to 6 months after the death. The best next step in her care is a
referral for counseling.
When a child experiences the death of a family member or loved one, information and resources for support
(including referral for grief counseling) should be provided as appropriate (eg, a child who is exhibiting
complicated grief or bereavement). Risk factors for complicated bereavement include having a high level of
attachment to the deceased, a sudden or unexpected death, and prior experiences with traumatic loss.
Preschool-aged children may manifest complicated bereavement as separation anxiety and behavioral
regression. Adolescents may engage in high-risk behaviors (eg, substance use, school delinquency),
experience somatic symptoms, or withdraw from friends and activities. Although children may exhibit
behavioral changes and disinterest in previously enjoyed activities after the death of a family member or
loved one, most will adjust and return to normal functioning. For those experiencing complicated grief or
bereavement, counseling is indicated.
A child’s understanding of death varies according to age and developmental level and is influenced by family,
cultural, and religious factors. Infants and toddlers may sense the emotions of others around them and react
by withdrawing and exhibiting decreased activity and responsiveness. Preschool-aged children (2 to 6 years)
typically do not understand that death is permanent and may repeatedly ask when the loved one is coming
back. They may also think that they have caused or contributed to the death. By around age 6 years, children
understand that death is permanent and irreversible. Adolescents are developing abstract thinking and begin
to understand that death is inevitable for everyone. Regardless of age and developmental level, children and
adolescents vary in their response to the loss of a loved one.
The psychiatrist Elizabeth Kubler-Ross defined 5 stages of grief: denial, anger, bargaining, depression, and
acceptance. The stages of grief are not sequential or cyclical; an individual may move in and out of these
stages in a variety of ways, with most (but not all) reaching acceptance. Grief is best understood as an
ongoing process that may vary in intensity depending on multiple factors, including new and ongoing life
events (eg, anniversary of the loss, birthdays, holidays). Although it may be helpful for the family to
understand that the girl in the vignette is most likely in the depression stage of grief, it is important for them
to know that whether or when she will reach acceptance cannot be determined.
Parents and caregivers may struggle with talking to their children about a death; they are frequently grieving
themselves. Children should be encouraged to communicate their thoughts and feelings, because this will
help them to process their grief. Adults should not avoid talking about death and grief, nor attempt to
distract their children from thinking about the deceased person. Advising the mother of the girl in the
vignette to avoid talking about the grandmother would not be helpful, because it does not support the
processing of her grief. Rather than helping her move through the grieving process, allowing the girl to
continue online schooling may exacerbate the girl’s symptoms of depression.
Suggested Reading(s)
Helping your school-age child cope with death. Pediatr Patient Educ. 2021.
doi:10.1542/peo_document238
Linebarger JS, Sahler OJ, Egan KA. Coping with death. Pediatr Rev. 2009;30(9):350-5; quiz 356.
doi:10.1542/pir.30-9-350
Schonfeld DJ, Demaria T; Committee on Psychosocial Aspects of Child and Family Health, Disaster
Preparedness Advisory Council. Supporting the grieving child and family. Pediatrics.
2016;138(3):e20162147. doi:10.1542/peds.2016-2147
Serwint JR, Bostwick S, Burke AE, et al. The AAP resilience in the face of grief and loss curriculum.
Pediatrics. 2016;138(5):e20160791. doi:10.1542/peds.2016-0791
Walter HJ, DeMaso DR. Psychiatric emergencies in pediatrics. Point-of-Care Quick Reference. Pediatric
Care Online. 2023. Accessed September 1, 2023. Pediatric Care Online
Content Domain
Palliative Medicine
A 12-year-old boy with multiple complex medical issues is admitted to the pediatric intensive care unit (PICU)
for his third episode of pneumonia in 3 months. He was born via emergency cesarean delivery owing to
placental abruption, which resulted in profound perinatal hypoxemia and hypoxic encephalopathy. Sequelae
include spastic quadraplegia, shunted hydrocephalus, and chronic respiratory failure requiring tracheostomy
and mechanical ventilation. The boy has been admitted to the PICU with increasing frequency over the past
year for lower respiratory tract infections. He is currently requiring significantly higher ventilator support
above baseline, and his condition continues to deteriorate. He has an active do-not-attempt-resuscitation
order, and the medical team believes he will not survive this illness. The family requests withdrawal of all
technological support.
Of the following, the BEST next step in this boy’s care is to
B. file for protective custody of the boy and continue all current therapies
D. turn o the mechanical ventilator and administer morphine for air hunger
Correct answer is D
PREP Pearl(s)
End-of-life care includes the provision of medications to ensure the patient’s comfort after removal of
technological support.
Rarely, appropriate interventions may hasten death; however, if the intent was to provide symptomatic
relief, the intervention may be ethically justified under the doctrine of double effect.
The goals of end-of-life care should include administration of therapies that will increase comfort and
removal or cessation of therapies that offer no clinical benefit.
Critique
The boy in the vignette has chronic underlying medical conditions with an increasing frequency of
hospitalization and now has an acute clinical deterioration with low likelihood of survival. The parents are
requesting end-of-life care and withdrawal of supportive technology. The goals of end-of-life care should
focus on the administration of therapies that will increase comfort and removal of therapies that offer no
clinical benefit. Given the boy’s overall condition and alignment with the family’s goals, it would be
appropriate for the medical team to limit any further interventions and turn off the ventilator.
End-of-life care does not equate to withdrawal of all medical support. Patients who undergo removal of
mechanical ventilator support often require treatment of symptoms to ensure comfort at the end of life.
Many require administration of medications to treat air hunger, pain, or general discomfort.
Many clinicians express discomfort with providing medications that may hasten the onset of death,
particularly the use of opioids for the treatment of air hunger. The relevant ethical construct requires
knowledge of the doctrine of double effect. Dying children who are removed from the ventilator frequently
have pain or dyspnea at the end of life. Pharmacologic interventions intended to provide symptomatic relief
may also hasten death in rare cases. The doctrine of double effect justifies using appropriate medications
and doses to relieve symptoms, even if doing so will precipitate an otherwise inevitable and imminent death.
There is an ethical distinction between the intended effect of an action (eg, administration of medication that
will provide pain relief) from the foreseeable but unintended negative effect (respiratory depression as a
known side effect of the opioid). To be ethically justified, a number of criteria must exist simultaneously:
Administering rocuronium, a paralytic agent, before decannulating the tracheostomy tube of the boy in the
vignette (thereby removing support) would violate the doctrine of double effect. This option offers no relief of
symptoms and would precipitate death solely as a means to an end. Given the alignment between the
family’s wishes and the prognosis of the child, there is no indication for an ethics committee consultation.
Obtaining protective custody to continue to provide medical care against the wishes of this family would be
inappropriate in this situation, in which the clinical prognosis is clear and the medical team is aligned.
Suggested Reading(s)
Basu RK. End-of-life care in pediatrics: ethics, controversies, and optimizing the quality of death.
Pediatr Clin North Am. 2013;60(3):725-739. doi:10.1016/j.pcl.2013.02.009
Evans AM, Jonas M, Lantos J. Pediatric palliative care in a pandemic: role obligations, moral distress,
and the care you can give. Pediatrics. 2020;146(1):e20201163. doi:10.1542/peds.2020-1163
Moynihan KM, Jansen MA, Liaw SN, Alexander PMA, Truog RD. An ethical claim for providing medical
recommendations in pediatric intensive care. Pediatr Crit Care Med. 2018;19(8):e433-e437.
doi:10.1097/PCC.0000000000001591
Okun AL. Palliative, end-of-life, and bereavement care. In: McInerny TK, Adam HM, Campbell DE, Foy
JM, Kamat DM, eds. American Academy of Pediatrics Textbook of Pediatric Care. 2nd ed. American
Academy of Pediatrics; 2016:chap 67. Accessed September 1, 2023. Pediatric Care Online
Opel DJ, Olson ME. Ethics for the pediatrician: bioethics education and resources. Pediatr Rev.
2012;33(8):370-373. doi:10.1542/pir.33-8-370
Content Domain
Palliative Medicine
The correct answer is: turn off the mechanical ventilator and administer morphine for air hunger
A 7-year-old boy with intellectual disability is brought to the office for evaluation of recent stomach aches and
difficulty sleeping. He has been waking up in the middle of the night crying for his grandmother, who died 2
months ago. He frequently asks, “When is Grandma coming back?” His mother read online that most children
his age understand that death is permanent. The boy’s constant asking about his grandmother has made it
more difficult for his mother to grieve. The boy’s physical examination findings are normal.
Of the following, the BEST next step in this boy’s care is to
A. advise his mother to allow the boy to sleep in her room until he is coping better
B. advise his mother to change the subject or distract him when he mentions his grandmother
C. diagnose him with prolonged grief disorder and refer him to a grief counselor
D. explain that the boy may not understand the nality of death
Correct answer is D
PREP Pearl(s)
A child’s understanding of death depends on their developmental level, cultural and religious
background, and past experiences with death. Parents and caregivers should allow children to discuss
and ask questions about a deceased loved one.
After the death of a loved one, practitioners can provide immediate support to families as well as
longitudinal support during the grieving process.
Critique
The boy described in the vignette is having difficulty coping with his grandmother’s death. Of the response
choices, the best next step in his care is to explain that he may not understand the finality of death. A child’s
understanding of death depends on their developmental level, cultural and religious background, and past
experiences with death. Given the boy’s age and intellectual disability, he is likely functioning at a preschool-
age level. Children between the ages of 2 and 6 years may think death is temporary and that the deceased
person is sleeping or away and will come back. It is common for young children to repeatedly ask about the
deceased. Parents should be patient with their child during this time.
Preschool-aged children may have magical thinking and feel guilt or fear that their thoughts or actions
caused the death. It is important for parents, caregivers, and the practitioners to provide reassurance to the
child that their thoughts or actions did not contribute to the death.
Children younger than 2 years may not verbally express grief or ask questions. They may experience changes
in their mood or appetite or be more withdrawn in response to the emotions and behaviors of others around
them. Children with typical development and cognition begin around age 6 years to understand that death is
final. Children at this age may be curious about how death occurs. School-aged children start to understand
that death is universal and that they too will die someday. Some adolescents, as they develop abstract
thinking, become existential in their processing of death, whereas others may engage in high-risk behaviors
to deny the possibility of their own death.
Prolonged grief disorder (previously known as complicated bereavement) is diagnosed when a child or
adolescent exhibits impairments for longer than 6 months after the death of a loved one. Preschoolers may
exhibit stranger anxiety and disruptive behaviors. School-aged children may exhibit declining academic
performance, show less interest in seeing friends or participating in previously enjoyed activities, or
experience physical complaints without organic cause. Adolescents may engage in high-risk behaviors,
withdraw from friends and activities, or experience physical complaints. In addition to assessment for new
onset of anxiety or a mood disorder, referral for grief counseling is recommended for school-aged children
and adolescents with prolonged grief disorder. The boy described in the vignette has been exhibiting physical
complaints for 2 months, which does not meet criteria for this diagnosis.
Recommending that the boy sleep in his parents’ room would not be appropriate. After the death of a loved
one, parents and caregivers should maintain as much of the child’s daily routine as possible. Keeping
bedtime, mealtimes, and expectations regarding chores and homework consistent helps provide stability.
Sleeping in his parents’ room may cause further life disruption. This approach also does not help his mother
to better understand how the boy is processing his grandmother’s death.
Although parents too are grieving and may wish to protect their children from suffering, they should be
encouraged to allow the child to speak about and ask questions regarding the deceased. Older children may
want to be included in discussions about memorial services or attend the funeral, when appropriate.
Practitioners play the important role of providing longitudinal support for families after the death of a child,
family member, or loved one. Practitioners can help identify feelings of guilt or fear about the death and
provide reassurance. They can also identify signs of depression, anxiety, or prolonged grief and provide
information regarding community resources for support and grief counseling.
Suggested Reading(s)
Arthur JD. Helping children cope with divorce, death, and deployment. Pediatr Rev. 2020;41(2):93-95.
doi:10.1542/pir.2018-0215
Linebarger JS, Sahler OJZ, Egan KA. Coping with death. Pediatr Rev. 2009;30(9):350-355.
doi:10.1542/pir.30-9-350
Schonfeld DJ, Demaria T; Committee on Psychosocial Aspects of Child and Family Health, Disaster
Preparedness Advisory Council. Supporting the grieving child and family. Pediatrics.
2016;138(3):e20162147. doi:10.1542/peds.2016-2147
Content Domain
Palliative medicine
The correct answer is: explain that the boy may not understand the finality of death
View Peer Results
AAP PREP 2024 - Question 185/267 Pharmacology Question 1/5
A 17-year-old adolescent is seen in the office for evaluation of morning clumsiness. She describes episodes of
her hands jerking, making it hard to brush her teeth, and brief staring episodes that her parents attribute to
inattentiveness. An electroencephalogram is obtained that shows generalized epileptiform discharges,
confirming a diagnosis of idiopathic generalized epilepsy. Levetiracetam is initiated, but is subsequently
switched to topiramate because of significant irritability. She has good seizure control on topiramate without
side effects.
A. B2 (riboflavin)
B. B6 (pyridoxine)
C. B9 (folic acid)
D. B12 (cobalamin)
Correct answer is C
PREP Pearl(s)
Individuals capable of becoming pregnant who are starting antiseizure medication therapy should be
counseled about the teratogenic risks.
The Centers for Disease Control and Prevention recommends vitamin B9 (folic acid) supplementation
for all people capable of becoming pregnant to lower the risk of birth defects, including neural tube
defects.
The US Food and Drug Administration recommends avoidance of valproic acid in people capable of
becoming pregnant, owing to the high risk of congenital malformations.
Critique
The adolescent in the vignette is of childbearing potential and taking daily topiramate, which has known
teratogenicity. The Centers for Disease Control and Prevention (CDC) recommends that all individuals
capable of becoming pregnant take 0.4 mg of folic acid (vitamin B9) daily to lower the risk of birth defects,
including neural tube defects (CDC Website: Folic Acid). Neurologic guidelines recommend folic acid for
individuals taking antiseizure medications, while capable of becoming pregnant, to decrease the risk of
neural tube defects and neurodevelopmental disorders (such as autism spectrum disorder or intellectual
disability). However, data regarding the optimal dose are lacking; recommended doses range from 0.4 to 5
mg daily.
Teratogenicity must be considered when prescribing an antiseizure medication for people capable of
becoming pregnant. Some antiseizure medications carry a teratogenic risk higher than that of others, and the
risk is unknown for many of the newer medications. Prenatal exposure to valproic acid carries a high risk of
congenital malformations. As a result, the US Food and Drug Administration recommends avoidance of
valproic acid in people capable of becoming pregnant. Phenytoin, phenobarbital, and topiramate carry a
moderate risk of causing congenital malformations. Levetiracetam, lamotrigine, and oxcarbazepine carry the
lowest risk. Individuals capable of becoming pregnant should be counseled on these risks when taking
antiseizure medications for any indication (eg, topiramate for migraine prevention or weight loss). Birth
control options may need to be discussed with the adolescent because estrogen-containing contraceptives
and some antiseizure medications may have negative interactions with each other.
The other response choices are indicated for a variety of neurologic conditions, but they would not be
indicated for the adolescent in the vignette. Vitamin B2 (riboflavin) is used for pediatric migraine headache
prevention; it has excellent tolerability and a low side effect profile. However, evidence of its efficacy is
inconclusive. Vitamin B6 (pyridoxine) is the treatment of choice for pyridoxine-dependent epilepsy, which this
adolescent does not have. Vitamin B6 may be used with levetiracetam to reduce the side effect of irritability.
Toxic doses can result in peripheral neuropathy. Vitamin B12 (cobalamin) deficiency can occur in individuals
who are following a vegan or vegetarian diet and can result in subacute combined degeneration of the spinal
cord.
Suggested Reading(s)
Asadi-Pooya AA. High dose folic acid supplementation in women with epilepsy: are we sure it is safe?
Seizure. 2015;27:51-53. doi:10.1016/j.seizure.2015.02.030
Blotière P-O, Raguideau F, Weill A, et al. Risks of 23 specific malformations associated with prenatal
exposure to 10 antiepileptic drugs. Neurology. 2019;93(2):e167-e180.
doi:10.1212/wnl.0000000000007696
Diab L, Krebs NF. Vitamin excess and deficiency. Pediatr Rev. 2018;39(4):161-179. doi:10.1542/pir.2016-
0068
Folic acid. Centers for Disease Control and Prevention. Accessed September 1, 2023.
https://www.cdc.gov/ncbddd/folicacid/index.html
Kim H, Faught E, Thurman DJ, Fishman J, Kalilani L. Antiepileptic drug treatment patterns in women of
childbearing age with epilepsy. JAMA Neurol. 2019;76(7):783-790. doi:10.1001/jamaneurol.2019.0447
Teratogenesis, perinatal, and neurodevelopmental outcomes: A practice guideline update for women
with epilepsy of childbearing potential: report of the Guidelines Subcommittee of the AAN, in full
collaboration with the American Epilepsy Society and the Society for Maternal-Fetal Medicine.
American Academy of Neurology. Accessed September 1, 2023. https://www.aan.com/siteassets/home-
page/policy-and-guidelines/guidelines/guidelines-and-measures-open-for-public-comment/epilepsy-
and-pregnancy_draft-public-comment-manuscript.pdf
Content Domain
Pharmacology
A 4-month-old infant is brought to the emergency department for concerns about dehydration. She has been
congested for the past 3 days. Her oral intake has decreased and today she has had only 1 damp diaper.
Her temperature is 37.4 °C, her heart rate is 190 beats/min, her respiratory rate is 55 breaths/min, and her
oxygen saturation is 94% in room air. She is awake and alert, sitting in her mother’s lap. She has nasal
congestion. Pulmonary examination reveals mild retractions; diffuse, scattered rhonchi; no wheezing; and
good air entry throughout. Her capillary refill time is 4 seconds. The remainder of her examination findings
are unremarkable. Intravenous hydration with isotonic fluid is ordered.
Of the following, the BEST next step in this child’s care is to prescribe
A. a topical anesthetic
B. albuterol
C. ibuprofen
D. oxygen
Correct answer is A
PREP Pearl(s)
Children of all ages undergoing painful procedures should receive appropriate pain management.
Neonates and infants who undergo painful procedures without adequate pain management may
develop sleep disturbances, difficulty bonding with caregivers, an increased stress response, and
increased pain perception in response to repeated noxious stimuli.
Appropriate methods of procedural pain management vary according to age and developmental stage.
Critique
The infant in the vignette has signs and symptoms of dehydration (prolonged capillary refill time, tachycardia,
decreased urine output) and requires intravenous fluids. Managing pain during intravenous catheter
insertion, a painful procedure, is important. Of the response choices, a topical anesthetic is the best next step
in her care. These should be used with caution in infants <3 months of age owing to the risk of
methemoglobinemia. Instructions vary by manufacturer.
Ibuprofen may help control pain during painful procedures, but this infant is too young (age <6 months).
Supplemental oxygen is not indicated for this child, who has mild retractions and an oxygen saturation of
>90% in room air. She has only mild retractions, no wheezing, and good lung aeration, so treatment with
albuterol is not indicated.
Pain management should be considered for children of all ages during painful procedures. The appropriate
management varies according to age and developmental stage.
Neonates feel pain as early as 25 weeks’ gestation. Neonates born prematurely are at increased risk of
experiencing long-term sequelae if pain is not managed well because of their developing brain and exposure
to recurrent painful interventions. Sequelae include sleep disturbances, difficulty bonding with caregivers, an
increased stress response, and increased pain perception in response to repeated noxious stimuli. Infants as
young as 6 months who have experienced pain in the medical setting display anticipation, fear, and
avoidance of painful procedures. Nonpharmacological pain management strategies for neonates and infants
include positioning (eg, swaddling, skin-to-skin contact), sucking, and breastfeeding.
Pain management is especially important for toddler-aged children. Toddlers explore their world using
physical touch and sensory stimulation, which places them at increased risk of experiencing trauma resulting
from painful procedures. They are not able to understand why a painful intervention is required but are likely
to retain the memory of painful procedures long term. Nonpharmacologic pain management strategies for
toddlers include using positions of comfort (eg, sitting on a caregiver's lap in a chest-to-chest position),
comfort objects (eg, stuffed animal or blanket), and distraction (eg, light-up toys, electronic games, videos). It
is important to avoid an overwhelming environment for toddlers. Identifying one person (a caregiver or
medical professional) to be the voice the child hears during the procedure helps minimize stimulation.
Supplies should be set up before the child enters the room and placed to the side, so as not to be a focal
point. If the child has undergone procedures in the past, it is helpful to ask caregivers what has worked well
to ease the experience for the child.
Preschoolers are developing the ability to differentiate between pretend and reality. It is important to ensure
they understand that painful procedures are not punishment. Their ability to understand time is still
developing. As the procedure is discussed, the child may not understand when it will start and the expected
duration of the pain. Appropriate pain-control interventions for preschoolers include positions of comfort (eg,
sitting on or with a caregiver and using a pillow as support under the arm) and the one-voice method.
Keeping discussions simple and avoiding use of medical terms is important at this age.
School-aged children are concrete in their thinking. Preparing a child for a painful procedure with pictures,
videos, and examples of equipment they can see and touch, as well as explaining that pain-control therapies
(eg, topical anesthetics) will be used, can help decrease anxiety. Positions of comfort, comfort objects, and
methods that have previously made them feel safe also help decrease stress.
Adolescents vary in their response to painful procedures. Those who have had previous painful experiences
may feel more stress than expected for their age. In response to this stress, adolescents may regress to a
prior developmental stage. It is important to use easy-to-understand language to discuss what to expect and
how methods to decrease pain work.
Moderate sedation provided by trained practitioners should be considered for more invasive or painful
procedures.
Suggested Reading(s)
Committee on Fetus and Newborn and Section on Anesthesiology and Pain Medicine. Prevention and
management of procedural pain in the neonate: an update. Pediatrics. 2016;137(2):e20154271.
doi:10.1542/peds.2015-4271
Kennedy RM, Luhmann J, Zempsky WT. Clinical implications of unmanaged needle-insertion pain and
distress in children. Pediatrics. 2008;122(suppl 3):S130-S133. doi:10.1542/peds.2008-1055e
Uman LS, Birnie KA, Noel M, et al. Psychological interventions for needle-related procedural pain and
distress in children and adolescents. Cochrane Database Syst Rev. 2013;(10):CD005179.
doi:10.1002/14651858.CD005179.pub3
Content Domain
Pharmacology
A fully immunized 4-year-old child is seen in the office because of bilateral ear pain that has lasted for 3 days.
On physical examination, he appears well. His vital signs are a temperature of 39 °C, a heart rate of 90
beats/min, a respiratory rate of 20 breaths/min, and an oxygen saturation of 100% in room air.
The conjunctiva of both eyes are injected and have purulent discharge, and both tympanic membranes are
erythematous, opacified, and bulging. The remainder of the child’s physical examination findings are normal.
Of the following, the mechanism of action of the MOST appropriate antibiotic to treat this child’s infection is
Correct answer is C
PREP Pearl(s)
Beta-lactam antibiotics include penicillins, cephalosporins, carbapenems, and monobactam.
Beta-lactam antibiotics inhibit cell wall synthesis by binding to penicillin-binding protein, a
transpeptidase enzyme; transpeptidation is an essential step in cross-linking peptides to form
peptidoglycan, which provides stability to the bacterial cell wall. Inhibiting the transpeptidase enzyme
results in autolysis of the bacterial cell wall.
When treating a child with an oral antibiotic for a suspected Haemophilus influenzae infection with
unknown susceptibility, the recommended first-line antibiotic is amoxicillin-clavulanic acid.
Critique
The child in the vignette has bilateral acute otitis media and purulent conjunctivitis. His fever (temperature
≥39 °C) and otalgia of greater than or equal to 48 hours suggest severe disease. With this constellation of
signs and symptoms, Haemophilus influenzae is the most likely causative organism. About 30% of H
influenzae species produce a β-lactamase enzyme. The most appropriate antibiotic to treat this child's
infection is a β-lactam agent combined with a β-lactamase inhibitor (eg, amoxicillin–clavulanic acid).
Beta-lactam antibiotics include penicillins (eg, amoxicillin), cephalosporins, carbapenems, and monobactam
(Table). The common structural feature shared by all β-lactam antibiotics is a nitro-containing 4-membered
ring (1 nitro, 3 carbons). Beta-lactam antibiotics work by inhibiting bacterial cell wall synthesis. The bacterial
cell wall provides structural stability. Gram-positive bacteria cell walls have a thick peptidoglycan component
(≥10 layers), whereas gram-negative bacteria have a thin peptidoglycan component (2-3 layers).
Beta-lactam antibiotics target penicillin-binding protein (PBP), a transpeptidase enzyme. Transpeptidase is
responsible for the last step in peptidoglycan synthesis, the cross-linking of peptides to form peptidoglycan.
Beta-lactam antibiotics interrupt this terminal transpeptidation process by acylating the PBP transpeptidase,
resulting in bacterial cell death by cell lysis.
High-dose amoxicillin, a β-lactam antibiotic, is the recommended initial therapy to treat otitis media caused
by non-β-lactamase–producing Streptococcus pneumoniae or H influenzae species. However, a quarter of H
influenzae species make β-lactamases, rendering amoxicillin therapy alone ineffective. Clavulanic acid, a β-
lactamase inhibitor, binds and inhibits β-lactamases, allowing concurrently administered amoxicillin to be
active against H influenzae. Therefore, when treating a child with an oral antibiotic for a suspected H
influenzae infection with unknown susceptibility, the recommended first-line antibiotic is amoxicillin-
clavulanic acid.
Antibiotic classes inhibiting RNA-dependent bacterial protein synthesis by binding to the 50 S ribosomal
subunit include macrolides (eg, azithromycin), clindamycin, and linezolid. Antibiotic classes inhibiting RNA-
dependent bacterial protein synthesis by binding to the 30 ribosomal subunit include aminoglycosides (eg,
gentamicin) and tetracyclines (eg, doxycycline).
Suggested Reading(s)
American Academy of Pediatrics. Principles of appropriate use of antimicrobial therapy for upper
respiratory tract infections. In: Kimberlin DW, Barnett ED, Lynfield R, Sawyer MH, eds. Red Book: 2021-
2024 Report of the Committee on Infectious Diseases. 32nd ed. American Academy of Pediatrics;
2021:873-875. Accessed September 1, 2023. Red Book Online
Congeni B, Di Pentima C. Antimicrobial therapy. In: McInerny TK, Adam HM, Campbell DE, Foy JM,
Kamat DM, eds. American Academy of Pediatrics Textbook of Pediatric Care. American Academy of
Pediatrics; 2023. Accessed September 1, 2023. Pediatric Care Online
Molloy L, Barron S, Khan N, Abrass E, Ang J, Abdel-Haq N. Oral β-lactam antibiotics for pediatric otitis
media, rhinosinusitis, and pneumonia. J Pediatr Health Care. 2020;34(3):291-300.
doi:10.1016/j.pedhc.2019.11.001
Content Domain
Pharmacology
The correct answer is: inhibition of the transpeptidase enzyme with a β-lactamase inhibitor
4th generation
Cefepime (IV, IM)
5th generation
Cetaroline (IV, IM)
β-lactam plus β-lactamase inhibitor Amoxicillin-clavulanic acid (oral) Ceftazidime-avibactam Meropenem-vaborbactam —
Piperacillin-tazobactam (IV, IM) Ceftolozane-tazobactam Imipenem-cilastin-relebactam
Ampicillin-sulbactam (IV, IM)
Abbreviations: IM, intramuscular; IV, intravenous.
Courtesy of A. Noor
AAP PREP 2024 - Question 188/267 Pharmacology Question 4/5
A 14-year-old is seen in the office for evaluation of a pruritic rash he has had on his lower legs since this
morning. He was evaluated at an urgent care center 2 days ago for sore throat and fever. The result of a
rapid streptococcal antigen test was negative. He was empirically treated with amoxicillin while awaiting
throat culture results.
He appears well. His temperature is 38.4 °C, his blood pressure is 100/75 mm Hg, his heart rate is 110
beats/min, his respiratory rate is 25 breaths/min, and his oxygen saturation is 100% in room air. He has
bilateral 3+ tonsils with exudates, cervical lymphadenopathy, and a maculopapular rash over his bilateral
lower extremities (Figure). The remainder of his physical examination findings are unremarkable.
Courtesy of A. Noor
Neutrophils 19%
Laboratory Test Result
Lymphocytes 45%
Monocytes 10%
25%
Atypical lymphocytes
1%
Eosinophils
Of the following, the MOST likely cause of this adolescent’s rash is a/an
B. IgE-mediated reaction
Correct answer is A
PREP Pearl(s)
Amoxicillin administered to a child with an Epstein-Barr virus infection (as well as many other viral
infections) may cause a maculopapular ampicillin rash; this is not a true hypersensitivity reaction.
The 4 types of hypersensitivity reactions are type I (IgE mediated), type II (antibody-mediated
cytotoxicity), type III (antigen-antibody complex), and type IV (delayed hypersensitivity).
Antibiotic therapy is a risk factor for Clostridioides difficile infection. Clinical manifestations of C difficile
infection range from mild diarrhea to severe pseudomembranous colitis and toxic megacolon.
Critique
The adolescent in the vignette has exudative tonsillitis, cervical lymphadenopathy, fever, transaminitis, and
25% atypical lymphocytosis, all of which suggest an acute Epstein-Barr virus infection. About 30% of children
who receive amoxicillin during an Epstein-Barr virus infection develop a generalized maculopapular rash. This
antibiotic-induced rash, seen with many viral illnesses, is not a true hypersensitivity reaction. The
pathogenesis is not fully understood, but a drug-virus interaction has been proposed.
Antibiotic reactions are classified as type A or type B. Type A reactions (80%) are predictable, dose-
dependent, and a consequence of the mechanism of action (eg, antibiotic-associated diarrhea owing to the
change in gastrointestinal microbiota). Clostridioides difficile infection is not a type A reaction. Type B
reactions (20%) are hypersensitivity reactions. Table 1 outlines the 4 types of hypersensitivity reactions.
The timing of this adolescent’s rash onset (2 days after exposure) makes an IgE-mediated or T-cell–mediated
delayed-type reaction unlikely. His lack of arthritis makes an immune complex reaction unlikely.
Antibiotic therapy is a risk factor for a C difficile intestinal infection. Antibiotics clear the normal
gastrointestinal microbiome and create an environment suitable for C difficile overgrowth. Production of 2
exotoxins (A and B) leads to a spectrum of clinical manifestations. Infants develop asymptomatic C difficile
colonization, because they lack the receptors that produce clinical disease; treatment is often not necessary.
Children and adolescents with CDI may develop mild to severe symptoms. Mild disease is characterized by
watery diarrhea, low-grade fever, and abdominal pain. Severe forms of CDI include pseudomembranous
colitis (diarrhea with mucus, abdominal pain, fever, systemic toxicity) and toxic megacolon (acute colonic
dilation). Host factors (eg, immunodeficiency, inflammatory bowel disease) may predispose a person to
severe disease.
Management of CDI includes discontinuation of the offending antibiotic. Mild CDI typically responds to
discontinuation of antibiotics alone. If symptoms persist, treatment with oral vancomycin is indicated. The
first-line treatment for moderate to severe CDI is oral or rectal vancomycin. Intravenous metronidazole may
be added as an adjunctive treatment. Recurrent CDI is treated with pulse doses followed by a prolonged
taper of oral vancomycin or fidaxomicin.
Table 2 outlines specific adverse effects associated with specific antibiotic classes.
Suggested Reading(s)
American Academy of Pediatrics. Adverse reactions to antimicrobial agents. In: Bradley JS, Nelson JD,
Kimberlin DW, eds. Nelson's Pediatric Antimicrobial Therapy. 21st ed. American Academy of Pediatrics;
2015:203-209.
American Academy of Pediatrics. Clostridioides difficile (formerly Clostridium difficile). In: Kimberlin
DW, Barnett ED, Lynfield R, Sawyer MH, eds. Red Book: 2021-2024 Report of the Committee on
Infectious Diseases. 32nd ed. American Academy of Pediatrics; 2021. September 1, 2023. Red Book
Online
Norton AE, Konvinse K, Phillips EJ, Broyles AD. Antibiotic allergy in pediatrics. Pediatrics.
2018;141(5):e20172497. doi:10.1542/peds.2017-2497
Content Domain
Pharmacology
Courtesy of A. Noor
Aminoglycosides Nephrotoxicity
Ototoxicity
Tetracyclines Dental staining (age <8 years; doxycycline has lower risk)
Deposition in growing bones
Gastrointestinal disturbances
Photosensitivity
Vancomycin Nephrotoxicity
Ototoxicity
Phlebitis
Vancomycin flushing syndrome
Courtesy of A. Noor
AAP PREP 2024 - Question 189/267 Pharmacology Question 5/5
A 1-month-old infant is seen in the office for a follow-up visit. He was born at 36 weeks’ gestation with a birth
weight at the third percentile and head circumference well below the third percentile for gestational age. A
urine polymerase chain reaction test result was positive for cytomegalovirus (CMV), and a diagnosis of
symptomatic congenital CMV was made. Head ultrasonography, dilated eye examination, and initial hearing
screen findings were all normal. A 6-month course of oral valganciclovir was started for the prevention of
CMV-related hearing loss.
Of the following, the BEST serial test to perform while the infant is receiving this treatment is
Correct answer is A
PREP Pearl(s)
A 6-month course of oral valganciclovir is indicated for treatment of symptomatic congenital
cytomegalovirus infections diagnosed before 1 month of age. Studies have documented better hearing
outcomes and a modest improvement in developmental outcomes with this treatment.
Children receiving valganciclovir require routine complete blood cell count monitoring due to the risk
of clinically significant cytopenias.
Critique
Neonates with symptomatic congenital cytomegalovirus (CMV) in the first month after birth benefit
from treatment with an antiviral medication targeting CMV. Better hearing outcomes, as well as a
modest improvement in developmental outcomes, have been documented when this
recommendation is followed. Currently, the only oral option for treatment of CMV is valganciclovir. This
medication is generally well tolerated but can cause severe neutropenia as well as anemia and
thrombocytopenia. Complete blood cell (CBC) monitoring is recommended while this therapy is
ongoing.
Renal or hepatic injury with valganciclovir is uncommon and does not warrant routine laboratory
screening. Symptom-based screening is not recommended for children receiving valganciclovir
because of the risk of complications resulting from cytopenias.
Infants with symptomatic congenital CMV are often referred to an infectious disease specialist for
follow-up care, This care includes CBC monitoring, medication dose adjustment for weight, and
ensuring that needed referrals are placed and followed through (eg, early developmental intervention
and audiology). However, as this treatment becomes more commonplace, primary care pediatricians
may become skilled and comfortable performing this level of management.
Suggested Reading(s)
Kimberlin DW, Jester PM, Sánchez PJ, et al. Valganciclovir for symptomatic congenital cytomegalovirus
disease. N Engl J Med. 2015;372(10):933-943. doi:10.1056/NEJMoa1404599
Marsico C, Kimberlin DW. Congenital cytomegalovirus infection: advances and challenges in diagnosis,
prevention and treatment. Ital J Pediatr. 2017;43(1):38. doi:10.1186/s13052-017-0358-8
Nicloux M, Peterman L, Magny J-F. Outcome and management of newborns with congenital
cytomegalovirus infection. Arch Pediatr. 2020;27(3):160-165. doi:10.1016/j.arcped.2020.01.006
Content Domain
Pharmacology
A 1-day old neonate is evaluated in the nursery. He was born to a 25-year-old primigravida woman at 39
weeks’ gestation via vaginal delivery. His mother has a history of colonization with group B Streptococcus and
had an intrapartum fever. He is feeding well and has good urine output.
His weight is 3.2 kg, heart rate is 150 beats/min, and respiratory rate is 72 breaths/min. He is vigorous and
alert. The remainder of his physical examination findings are normal.
An intravenous line is placed, blood cultures are drawn, and empiric antibiotics are started.
Laboratory results are shown:
Of the following, the BEST interpretation of this neonate’s laboratory results is that his
Correct answer is C
PREP Pearl(s)
The serum creatinine concentration at birth is reflective of the circulating maternal creatinine
concentration.
The average glomerular filtration rate of a term neonate is 39 mL/min/1.73 m2 (range, 17-60
mL/min/1.73 m2) in the first week after birth and approaches a normal adult rate (120 mL/min/1.73 m2)
at age 2 years.
The serum bicarbonate concentration is lower and the serum potassium and phosphorus
concentrations are higher in neonates than in older children and adults.
Critique
The neonate in the vignette underwent laboratory testing as part of the evaluation for suspected neonatal
sepsis. He has normal serum concentrations of creatinine, potassium, bicarbonate, and phosphorus for age.
The normal range of phosphorus (4.5-9.0 mg/dL [1.5-2.9 mmol/L]) is higher in neonates than in children and
adults (2.4-4.4 mg/dL [0.8-1.4 mmol/L]) because of increased reabsorption in the proximal tubules and
reduced glomerular filtration rate (GFR). The normal ranges of serum creatinine and electrolyte
concentrations for neonates differ from those for children and adults because their glomerular and tubular
function changes over time.
The serum creatinine concentration at birth is typically reflective of the circulating maternal creatinine
concentration (0.8-1.0 mg/dL [70.7-88.4 µmol/L]). Creatinine concentrations gradually decline over 1 to 2
weeks to typical neonatal values (0.2-0.4 mg/dL [17.7-35.4 µmol/L]). In preterm neonates, the decline in
serum creatinine concentrations is slower, taking up to 2 months to reach the normal range for chronologic
age.
The serum creatinine concentration is commonly used to estimate GFR and assess renal function. In the early
postnatal period, there is a decrease in peripheral vascular resistance that leads to an increase in blood flow
through the kidneys. This increased blood flow results in enlargement of the glomeruli and tubules and a
progressive improvement in GFR. Healthy term neonates have an average GFR of 39 mL/min/1.73 m2 (range,
17-60 mL/min/1.73 m2) in the first week after birth. The GFR approaches 50% of the typical adult value by age
3 months. At age 2 years, the GFR is close to the normal adult value of 120 mL/min/1.73 m2. Preterm
neonates have a lower GFR compared with term neonates and older children because renal embryogenesis is
not complete until 35 weeks of gestation.
Normal serum bicarbonate concentrations in neonates (18-22 mEq/L [18-22 mmol/L]) are lower than in older
children and adults (22-26 mEq/L [22-26 mmol/L]) owing to lower renal proximal tubular reabsorption of
bicarbonate. Neonates also have a higher normal range of potassium (3.7-5.9 mEq/L [3.7-5.9 mmol/L]) as
compared with children and adults (3.5-5.1 mEq/L [3.5-5.1 mmol/L]). Potassium excretion from the distal
tubule is decreased owing to aldosterone insensitivity and reduced GFR. The renal tubular handling of
electrolytes improves with age, taking longer to reach normal adult concentrations in preterm infants.
Neonates have poor urine-concentrating ability because their renal tubules are immature and lack
responsiveness to antidiuretic hormone. Their urine specific gravity is typically 1.010 or lower. Renal sodium
reabsorption is also decreased in the neonatal period. Both the increased sodium losses and the production
of dilute urine put the neonate at increased risk of dehydration.
Suggested Reading(s)
Selewski DT, Symons JM. Acute kidney injury. Pediatr Rev. 2014;35(1):30-41. doi:10.1542/pir.35-1-30.
Soghier L. Formulas and reference range values. In: McInerny TK, Adam HM, Campbell DE, DeWitt TG,
Foy JM, Kamat DM, eds. American Academy of Pediatrics Textbook of Pediatric Care. 2nd ed. American
Academy of Pediatrics; 2016:Appendix C. Accessed September 1, 2023. Pediatric Care Online
Content Domain
Physiology
A 5-day-old neonate is undergoing a routine health supervision visit. He was born to a 30-year-old
primigravida mother at 39 weeks’ gestation via normal vaginal delivery. Prenatal ultrasonography showed an
absent left kidney and a normal amount of amniotic fluid. Renal ultrasonography performed 2 days after
birth showed a normal right kidney with normal corticomedullary differentiation and no hydronephrosis. The
left kidney was not visualized.
The neonate is breastfed and voids multiple times per day. He passed the newborn hearing screening test.
His vital signs and physical examination findings are normal. He is circumcised.
Of the following, the condition MOST likely to be seen in this neonate is
Correct answer is A
PREP Pearl(s)
Unilateral renal agenesis is usually detected on prenatal ultrasonography and occasionally during the
evaluation of a urinary tract infection.
Children with a solitary kidney should undergo regular monitoring for compensatory hypertrophy (via
renal ultrasonography), proteinuria, and hypertension.
Critique
The neonate in the vignette has unilateral renal agenesis and a normal contralateral kidney. In this scenario,
the contralateral kidney is expected to undergo compensatory hypertrophy.
Renal agenesis is the congenital absence of a kidney due to lack of initiation of embryonic development. The
reported incidence of unilateral renal agenesis is 1 in 1,000 to 3,000 live births. It is more common in males
and occurs predominantly on the left side. Unilateral renal agenesis is usually detected on prenatal
ultrasonography and occasionally during the evaluation of a urinary tract infection. One-third of children with
renal agenesis have associated extrarenal malformations of the heart, genitals, bone, or gastrointestinal
tract. The associated findings of hearing loss or abnormal hearing test result, branchial cyst, preauricular pits,
and renal agenesis is suggestive of branchiootorenal syndrome.
Ultrasonography is the preferred initial imaging modality to confirm the absence of one kidney, detect
associated abnormalities of the contralateral kidney and urinary tract, and measure the size of the solitary
kidney. Abnormal echogenicity and/or decreased size of the solitary kidney may suggest renal dysplasia.
Vesicoureteral reflux can be associated with a solitary kidney, but it is usually low grade. Hydronephrosis of a
solitary kidney warrants a voiding cystourethrogram to evaluate for vesicoureteral reflux or a renal scan to
detect ureteropelvic junction obstruction. When the contralateral kidney is normal, it is expected to undergo
compensatory renal hypertrophy (kidney size at or above the 50th percentile for age) to account for the
reduced nephron number.
Children with a solitary kidney should undergo regular monitoring for the following:
Compensatory hypertrophy (via renal ultrasonography)
Proteinuria
Hypertension
The solitary kidney can experience hyperfiltration injury, resulting in proteinuria, hypertension, and reduced
glomerular filtration rate. Children who have obesity, hypertension, or proteinuria, as well as those without
compensatory renal hypertrophy, are at risk of experiencing progression to chronic kidney disease with
solitary kidney.
Hypertension associated with a solitary kidney can occur in adulthood (not in the neonate) owing to
hyperfiltration injury and obesity; this is not caused by renal artery stenosis. Potter syndrome or sequence
(oligohydramnios, facial and limb abnormalities, and pulmonary hypoplasia) is associated with bilateral renal
agenesis. Bilateral renal agenesis is almost universally fatal at or shortly after birth owing to severe
pulmonary hypoplasia. The neonate in the vignette has no hydronephrosis of the contralateral kidney;
therefore, he is unlikely to have ureteropelvic junction obstruction.
Suggested Reading(s)
Janjua HS, Lam SK, Gupta V, Krishna S. Congenital anomalies of the kidneys, collecting system, bladder,
and urethra. Pediatr Rev. 2019;40(12):619-626. doi:10.1542/pir.2018-0242
Wolfe DS, Suskin B. Prenatal diagnosis. In: McInerny TK, Adam HM, Campbell DE, DeWitt TG, Foy JM,
Kamat DM, eds. American Academy of Pediatrics Textbook of Pediatric Care. 2nd ed. American
Academy of Pediatrics; 2016:chap 82. Accessed November 17, 2023. Pediatric Care Online
Content Domain
Physiology
A 9-month-old infant is seen in the office for a health supervision visit. He is gaining weight and achieving his
developmental milestones appropriately. The infant is up to date with vaccinations. He does not have any
signs or symptoms of illness. The family will be visiting Nigeria in 2 weeks.
Of the following, the BEST approach to protecting this infant against measles is to
Response choice Administer at this office visit Administer beginning at age 12 months
A. Row A
B. Row B
C. Row C
D. Row D
Correct answer is C
PREP Pearl(s)
A single dose of the measles, mumps, and rubella vaccine should be administered to infants 6 to 12
months of age before traveling anywhere outside the United States or during a community outbreak.
If the measles, mumps, and rubella vaccine is administered before age 12 months, it is not counted as
part of the routine 2-dose series owing to a suboptimal humoral immune response; the routine 2-dose
series at ages 12 to 15 months and 4 to 6 years should be completed (with at least 28 days between
the first and second doses).
Measles immune globulin should be considered for vulnerable children (unvaccinated or
immunocompromised) within 6 days of an exposure.
Critique
The 9-month-old infant in the vignette will be traveling to Nigeria in 2 weeks. The measles, mumps, and
rubella (MMR) vaccine should be administered to infants as young as age 6 months before traveling
anywhere outside the United States or during a community outbreak. When the MMR vaccine is administered
before age 12 months, it is not counted toward the routine 2-dose series. The 9-month-old infant in the
vignette should receive an MMR vaccine at this visit and will require the routine 2-dose series at age 12 to 15
months and age 4 to 6 years.
Administration of measles immune globulin should be considered for vulnerable children (unvaccinated or
immunocompromised) within 6 days of an exposure. Measles immune globulin is not readily available; there
is only one formulation in the United States. After administration of measles immune globulin, there should
be an interval of 6 months before administration of measles vaccine, owing to the risk of interference from
passive antibodies.
The MMR vaccine is a highly effective live attenuated virus vaccine. Routinely, the first dose of the MMR
vaccine is administered at age 12 to 15 months (93% efficacy) and the second dose at age 4 to 6 years (>99%
efficacy). The second dose ensures an adequate immune response. There must be at least 28 days between
the first and second doses. Administration of the MMR vaccine to infants younger than age 12 months results
in suboptimal immunity owing to an immature immune system as well as interference from preexisting
maternal measles antibodies. Most infants lose maternal measles antibodies by age 9 to 12 months.
Infants younger than age 12 months mount suboptimal humoral immunity to the MMR vaccine, which can be
overcome by the administration of the 2-dose series after age 12 months. Response to the MMR vaccine at
age 12 months is better (>92% efficacy) when it is also administered at age 9 months (80%). The second dose
in the routine series is not a booster but induces immunity in a small group (7% to 8%) of children who do not
respond to the first dose in the series.
The first MMR vaccine was widely distributed in 1968, and by 1981 the single-dose MMR vaccination had
reduced measles cases by 80%. A measles outbreak in vaccinated children in 1989 led to the
recommendation for a second dose of the MMR vaccine in all children. The Americas were declared measles
free in 2000 after 12 months with zero reported cases. This was achieved by pairing a safe and effective
vaccine with an efficient vaccination program.
However, in recent years, several measles outbreaks have occurred in the United States, usually because of
measles cases imported into pockets of unvaccinated people who are susceptible to this highly contagious
virus. Since the start of the COVID-19 pandemic, there have been increasing numbers of measles outbreaks
worldwide (In 2021-2022, 19,000 cases of measles were reported in Nigeria). Forty countries worldwide have
delayed enforcement of measles vaccine compliance owing to the COVID-19 pandemic.
Suggested Reading(s)
American Academy of Pediatrics. International travel. In: Kimberlin DW, Barnett ED, Lynfield R, Sawyer
MH, eds. Red Book: 2021–2024 Report of the Committee on Infectious Diseases. 32nd ed. American
Academy of Pediatrics; 2021. Accessed September 1, 2023. Red Book Online
Drutz J. Measles. Pediatr Rev. 2016;37(5):220-221. doi:10.1542/pir.2015-0117
Humiston SG, Atkinson WL, Rand CM, Szilagyi PG, et al. Immunizations. In: McInerny TK, Adam HM,
Campbell DE, DeWitt TG, Foy JM, Kamat DM, eds. American Academy of Pediatrics Textbook of Pediatric
Care. American Academy of Pediatrics; 2023. Accessed September 1, 2023. Pediatric Care Online
Kroger AT, Mawle AC, Pickering LK, Orenstein WA. Active immunization. In: Long SS, Pickering LK,
Prober CG. Principles and Practice of Pediatric Infectious Diseases. 4th ed. Elsevier; 2012:44-68.
doi:10.1016/B978-1-4377-2702-9.00006-4
Content Domain
Preventative Pediatrics
A previously healthy, 3-month-old infant born at term is brought to the emergency department after a
cyanotic episode at home. While her mother was holding her, the infant became limp with a bluish color
around her mouth. The episode lasted about 30 seconds. There were no abnormal movements or shaking.
The infant is now acting normally. She has not had any recent choking, cough, congestion, fever, or changes
in activity. The infant lives with her parents and does not attend daycare. No family members are ill.
Physical examination reveals a well-appearing infant. Her temperature is 37.2 °C, heart rate is 150 beats/min,
respiratory rate is 30 breaths/min, and oxygen saturation has been 98% to 100% in room air over the past 4
hours in the emergency department. Her weight, length, and head circumference are at the 50th percentile
for age. There is no bruising. The remainder of her physical examination findings are normal.
Of the following, the BEST next step in this infant’s management is to
B. obtain an electroencephalogram
Correct answer is C
PREP Pearl(s)
Infants evaluated for a brief resolved unexplained event (BRUE) who meet lower-risk criteria should be
discharged home with follow-up in 24 hours; cardiopulmonary resuscitation training should be offered
to the parents.
Laboratory evaluation and imaging should be limited for infants with brief resolved unexplained event
(BRUE) who meet lower-risk criteria.
A thorough history and physical examination, including a social history, should be obtained in infants
presenting with a brief resolved unexplained event or brief resolved unexplained event (BRUE).
Critique
The infant in the vignette had a brief resolved unexplained event (BRUE). Of the response choices, the best
next management step is to offer her parents cardiopulmonary resuscitation (CPR) training.
The first BRUE guideline, published in 2016, defined a BRUE as a brief (<1 minute) event occuring in infants
under 1 year of age that includes 1 or more of the following:
Cyanosis or pallor
Absent, decreased or irregular breathing
Hypertonia or hypotonia
Altered level of responsiveness
There must be no other explanation for the event (eg, cough, fever, choking, or trauma). Brief resolved
unexplained event replaced the term, apparent life-threatening event; it is a more specific, objective term
describing an event that has resolved.
Affected infants can be classified as being at lower- or higher-risk for recurrent events. A higher-risk infant is
one that does not meet lower-risk criteria.
The event described for the infant in the vignette meets the BRUE definition because she is under 1 year of
age, her episode lasted <1 minute, and the episode was associated with cyanosis. She meets lower-risk
criteria because she is older than 2 months of age, was born at term, has not had a prior event, and did not
require CPR.
For an infant determined to have had a BRUE that meets lower-risk criteria, further evaluation can safely be
limited. Evaluation plans should be made using shared decision making with the parents. These infants
should be monitored with pulse oximetry for 1 to 4 hours prior to discharge home, and follow-up should
occur within 24 hours. Testing for pertussis may be considered if the infant is not vaccinated, has a known
exposure, or there is an increased disease prevalence in the community. Electrocardiography is indicated if
there is a first-degree relative with a history of arrhythmia or sudden unexplained death. Cardiopulmonary
resuscitation training for the parents has been shown to decrease parental anxiety and should be offered.
Laboratory testing (eg, complete blood cell count, metabolic panel, lactic acid, blood culture, or blood gas)
and imaging (eg, chest radiography, brain magnetic resonance imaging, or computed tomography) should
not routinely be obtained in lower-risk infants with BRUE. A detailed history of the event should be obtained,
as well as a social history that includes mental health disorders, child protective services involvement,
substance use, domestic violence, and the caregivers’ expectations of the infant. Infants with a BRUE who are
lower-risk and have no social risk factors have a <0.3% risk of abusive head trauma. Additional evaluation for
non-accidental trauma should be considered if there are inconsistencies in the history of the event, the event
does not make sense for the child’s development level, or there are social risk factors.
An electroencephalogram (EEG) is not indicated for infants with a BRUE who are lower-risk and appear well,
such as the girl in the vignette. While a seizure can have a similar clinical presentation to a BRUE, a routine
EEG may not capture an abnormality. If an infant with a lower-risk BRUE does have epilepsy, waiting until a
second episode before beginning an antiepileptic medication would not change the long-term outcome.
Infants who are at high risk for underlying disorders or do not meet the lower-risk criteria require additional
evaluation; the specific evaluation should be determined based on history and physical examination findings.
The management of higher-risk infants with BRUE is summarized in the Figure.
Merritt JL 2nd, Quinonez RA, Bonkowsky JL. A framework for evaluation of the higher-risk infant after a
brief resolved unexplained event. Pediatrics. 2019;144(2):e20184101.
Admission to the hospital is not indicated as the infant appears well. Admission for monitoring has not been
shown to improve outcomes in lower-risk infants. Starting a proton-pump inhibitor is not indicated as the
infant does not have symptoms of gastroesophageal reflux disease such as frequent spit ups, changes in
respiratory status related to feeds or spit ups, or poor weight gain.
Suggested Reading(s)
Behnam-Terneus M, Clemente M. SIDS, BRUE, and safe sleep guidelines. Pediatr Rev. 2019;40(9):443-
455. doi:10.1542/pir.2017-0259
Merritt JL 2nd, Quinonez RA, Bonkowsky JL. A framework for evaluation of the higher-risk infant after a
brief resolved unexplained event. Pediatrics. 2019;144(2):e20184101. doi:10.1542/peds.2018-4101
Rafei K, Blaisdell CJ. Apparent life-threatening events. In: McInerny TK, Adam HM, Campbell DE, DeWitt
TG, Foy JM, Kamat DM, eds. American Academy of Pediatrics Textbook of Pediatric Care. American
Academy of Pediatrics; 2023. Accessed September 1, 2023. Pediatric Care Online
Tieder JS, Bonkowsky JL, Etzel RA, et al; Subcommittee on Apparent Life-threatening Events. Brief
resolved unexplained events (formerly apparent life-threatening events) and evaluation of lower-risk
Infants. Pediatrics. 2016;137(5):e20160590. doi:10.1542/peds.2016-0590
Content Domain
Preventative Pediatrics
A term infant girl is being seen for a routine health supervision visit. When placed in a prone position, she
lifts her head and chest and rests her forearms on the examination table. When pulled to sit, she has some
unsteadiness of her head and mild lag. She coos and smiles in response to her parents.
Correct answer is A
PREP Pearl(s)
Developmental surveillance should be performed at every health supervision visit with attention paid
to parental concerns. Developmental screening using a validated tool should occur at the 9-, 18-, and
30-month visits.
Early detection of developmental delays and referral for appropriate services improves outcomes.
Developmental milestones for infants born prematurely should be assessed at the infants’ corrected
age until a chronological age of 24 months.
Critique
The infant in the vignette is exhibiting developmental milestones typical of a 2-month-old. In addition to
lifting her head and chest when prone, smiling socially, and cooing, a 2-month-old infant should be able to
retain a rattle placed in the hand. An infant is typically able to visually follow a person across the room at age
3 months, turn the head in the direction of a voice at age 4 months, and transfer objects from hand to mouth
to hand at age 5 months.
There is a typical pattern of progression for infants and children in the gross motor, fine motor, language,
cognitive, and social-emotional developmental domains. During the 1st month after birth, infants lift the chin
when prone, keep the hands fisted, learn to look at faces, discriminate the mother’s voice, and make throaty
noises. At age 6 weeks, most infants have a social smile. By age 2 months, they have reciprocal smiles and will
coo.
By age 3 months, most infants prop themselves on their forearms when prone, bat at objects, visually follow
a person moving across a room, and vocalize when talked to. By 4 months, infants start rolling over from
front to back, have no head lag when pulled to sit, play with a rattle, orient to voice, and smile spontaneously.
Most infants roll from back to front, transfer objects from their hand to mouth to hand, look for dropped
objects, and begin to respond to their name by age 5 months. Table 1 outlines typical milestones observed
from age 1 through 5 months.
Milestones provide a framework for assessing and monitoring development during health supervision visits.
The American Academy of Pediatrics recommends developmental surveillance at every health supervision
visit and developmental screening using a validated tool at the 9-, 18-, and 30-month visits.
Children can have global developmental delay (delayed milestones in more than 1 domain) or isolated delay
in 1 domain. Not attaining key milestones in the timeframe expected (“red flags”) should raise concern and
prompt the child’s early referral for evaluation. Parental concerns are highly predictive of true developmental
delays and should not be dismissed. Timely detection of delays facilitates formal evaluation and institution of
therapeutic services to optimize outcomes. Developmental assessment of infants born prematurely should
be based on their corrected age until they reach a chronological age of 24 months, by which time most
children will have caught up to their peers. There are many causes of developmental delay, including birth
factors, infection, intrinsic factors, social factors, and toxins. Table 2 outlines some recognized causes of
developmental delay.
Suggested Reading(s)
Bright Futures guidelines and pocket guide. American Academy of Pediatrics. Accessed December 12,
2022. https://www.aap.org/en/practice-management/bright-futures/bright-futures-materials-and-
tools/bright-futures-guidelines-and-pocket-guide/
CDC’s developmental milestones. Centers for Disease Control and Prevention. Accessed September 1,
2023. https://www.cdc.gov/ncbddd/actearly/milestones/index.html
Scharf RJ, Scharf GJ, Stroustrup A. Developmental milestones. Pediatr Rev. 2016;37(1):25-37.
doi:10.1542/pir.2014-0103
Zubler JM, Wiggins LD, Macias MM, et al. Evidence-informed milestones for developmental surveillance
tools. Pediatrics. 2022;149(3):e2021052138. doi:10.1542/peds.2021-052138
Content Domain
Preventative Pediatrics
A 2-year-old girl is seen in the office for a health supervision visit. The social history reveals that her mother is
an artist who works with stained glass, the family is currently renovating their rural home (built in 1968), and
they enjoy collecting antique metal toys, like the one the child is holding and chewing on in the examination
room. Developmental screening identifies that the girl has an expressive language delay. Point-of-care
capillary sample lead testing reveals a lead level of 4.3 μg/dL (0.21 µmol/L).
Of the following, the BEST next step in this girl’s management is to
A. counsel the family about possible lead exposures and repeat the point-of-care test in 3
months
Correct answer is B
PREP Pearl(s)
Physicians should perform risk assessments or screenings for lead toxicity in children ages 12 and 24
months, based on local and Medicaid requirements and regional prevalence.
The reference value for an elevated blood lead level has been changed from 5 μg/dL (0.24 µmol/L) to
3.5 μg/dL (0.17 µmol/L), as of October 2021.
Elevated point-of-care lead test results should be confirmed by a venous blood lead sample.
Critique
The girl in the vignette, with a point-of-care (POC) capillary sample lead level of 4.3 μg/dL (0.21 µmol/L),
should have confirmatory testing with a venous blood sample (Table). In October 2021, the Centers for
Disease Control and Prevention revised the reference for blood lead levels in children from 5 μg/dL (0.24
µmol/L) to 3.5 μg/dL (0.17 µmol/L), based on evidence that any level of lead in the blood can have a significant
impact on children’s learning and development. An abnormal point-of-care lead level should be confirmed by
a venous blood lead sample. Further patient management should be based on the venous blood lead level. If
the confirmatory venous blood lead level is equal to or greater than 3.5 μg/dL (0.17 µmol/L), the test result
should be reported to the local or state health department; results of a capillary sample should not be
reported.
Children under the age of 36 months are at especially high risk for lead toxicity from exposure in their
environment. Evidence has shown that blood lead levels increase quickly in children between 6 and 12
months of age, peak between ages 18 and 36 months, and then decrease. This pattern results from a
combination of young children’s increasing mobility and their exploring of the environment with normal
mouthing behaviors. At ages 12 and 24 months, the American Academy of Pediatrics preventive health care
periodicity schedule for lead screening recommends that pediatricians “perform risk assessments or
screenings as appropriate, based on universal screening requirements for patients with Medicaid or in high-
prevalence areas” (Suggested Reading 2).
Anticipatory guidance is critically important to help families identify and address potential sources of lead in
their environment. Physicians should ask about the age and location of the home, drinking water sources,
jobs and hobbies that may have a risk of lead exposure, and the possibility of any consumer products, foods,
cosmetics, jewelry, toys, ceramics, or traditional medicines the child may have contact with. In this girl’s case,
there are several sources of potential lead exposure:
Based on the value obtained on her POC testing, a confirmatory venous blood lead sample is recommended
within 3 months. A repeat POC test would not be appropriate follow-up.
While iron-deficiency anemia has been linked to increased susceptibility for lead toxicity, evidence has not
supported that iron supplementation is efficacious in lowering blood lead concentrations in children. This
child should be assessed for iron-deficiency anemia and, if present, recommendations for management
should be based on those results. An elevated point-of-care lead test alone should not prompt treatment
with a multivitamin with iron.
Suggested Reading(s)
American Academy of Pediatrics. Council on Environmental Health; Lanphear BP, Lowry JA, Ahdoot S,
et al. Prevention of childhood lead toxicity. Pediatrics. 2016;138(1):e20161493. doi:10.1542/peds.2016-
1493
American Academy of Pediatrics. Preventive care/periodicity schedule. Updated June 21, 2022.
Accessed September 1, 2023. https://www.aap.org/periodicityschedule
Centers for Disease Control and Prevention. Childhood lead poisoning prevention. Updated September
16, 2022. Accessed September 1, 2023. https://www.cdc.gov/nceh/lead/default.htm
Woolf AD, Brown MJ. Old adversary, new challenges: childhood lead exposure and testing. Pediatrics.
2022;149(5):e2021055944. doi:10.1542/peds.2021-055944
Weitzman M. Lead poisoning. In: McInerny TK, Adam HM, Campbell DE, DeWitt TG, Foy JM, Kamat DM,
eds. Textbook of Pediatric Care Online. American Academy of Pediatrics; 2023. Accessed September 1,
2023. Pediatric Care Online
Content Domain
Preventative Pediatrics
Source: Centers for Disease Control and Prevention; National Center for Environmental
Health.
AAP PREP 2024 - Question 196/267 Preventative Pediatrics Question 5/
27
A 17-year-old adolescent with a history of systemic lupus erythematosus is seen for evaluation of 3 days of
dysuria. She asks to be screened for sexually transmitted infections. She has had 3 lifetime sexual partners
with 1 new partner in the past 3 months. She engages in vaginal and oral sex and has not used condoms
consistently. She was diagnosed with gonorrhea 4 months ago; her vaginal symptoms completely resolved 2
weeks after treatment. Her partners were notified and also treated. She reports no current vaginal discharge,
abdominal pain, or rash. The result of an HIV screening test was negative 4 months ago. Her only current
medication is hydroxychloroquine.
The adolescent’s physical examination findings are normal, including vital signs and pelvic examination
findings.
Samples are obtained for laboratory evaluation:
Vaginal and oral swabs for chlamydia, gonorrhea, and trichomonas nucleic acid amplification
testing
Serum for HIV antigen/antibody concentrations and syphilis serology
Clean-catch urine for urinalysis, urine culture, and pregnancy test
C. perform urine nucleic acid amplification testing for chlamydia, gonorrhea, and trichomonas
Correct answer is B
PREP Pearl(s)
Providers should confidentially speak with all adolescents and young adults about sexual behaviors to
provide optimal sexually transmitted infection prevention education and perform screening on
samples from all involved sites.
The Centers for Disease Control and Prevention recommends pre-exposure HIV prophylaxis for any
adolescent who asks for it.
The Centers for Disease Control and Prevention offers guidance for counseling on, prescribing, and
managing antiretroviral medications for pre-exposure HIV prophylaxis.
Critique
The adolescent in the vignette is at high risk of experiencing sexually transmitted infections (STIs). Of the
response choices, the best next step in her treatment is to discuss and offer HIV pre-exposure prophylaxis
(PrEP). Treatment may begin as soon as the current laboratory results, in addition to baseline tests obtained
specifically for PrEP, are available.
Dysuria in the context of a recent STI and unprotected sexual intercourse increases the likelihood of an STI as
the cause of this adolescent’s current symptoms. However, she does not require urine nucleic acid
amplification testing for chlamydia, gonorrhea, and trichomonas because a vaginal swab has already been
collected for this testing. In addition, urine testing for STIs must be on a “dirty” urine specimen (eg, a
specimen obtained with no cleansing wipe used beforehand). The urine specimen provided by this
adolescent was obtained via clean catch.
It may be appropriate to empirically treat the adolescent in the vignette for chlamydia and gonorrhea. The
2021 Centers for Disease Control and Prevention (CDC) STI treatment guidelines recommend treatment for
Chlamydia trachomatis infection with doxycycline 100 mg twice per day for 7 days. Azithromycin 1 g orally in
a single dose is an alternative regimen but is not recommended as first-line therapy. Ceftriaxone 500 mg
intramuscularly is a recommended treatment for gonococcal infections.
A clean-catch urine sample was obtained for urinalysis and urine culture from the adolescent in the vignette
to evaluate for a bacterial urinary tract infection (UTI). If the urinalysis is concerning for a UTI (eg, presence of
leukocyte esterase, nitrites, white blood cells, or a combination of these), antibiotics should be initiated.
However, ciprofloxacin is not recommended as a first-line treatment for an uncomplicated UTI. In addition,
fluoroquinolones are contraindicated for this adolescent because of their interaction with
hydroxychloroquine.
Adolescents and young adults carry the highest risk of STI acquisition. All 50 states allow minors to give
consent for sexual and reproductive health care (specific rules vary by state). It is imperative for pediatric
providers to spend time privately and discuss confidentiality with each adolescent at every office visit.
Adolescents should be asked about sexual behaviors and provided appropriate education about pregnancy
and STI prevention. Providers should ask about sexual contact or interest in sexual intercourse clearly and
directly (eg, “When was the last time you had sex?”). Vague questions (eg, “Are you sexually active?”) should be
avoided. Inquiry about all types of sexual contact (eg, vaginal, anal, oral, hand-genital) is essential to obtain
STI testing from all involved sites and to provide appropriate STI prevention education. Sexually transmitted
infections can occur with any type of sexual practice. Testing for HIV is recommended for all adolescents aged
15 years or older.
Pre-exposure prophylaxis with antiretroviral medication is indicated for people without HIV infection who are
at risk of exposure through sex or injected drug use. The CDC PrEP guidelines include extensive algorithms
and flowcharts to assist providers in counseling on, prescribing, and monitoring PrEP medications. The 2021
guidelines recommend that PrEP be discussed with all sexually active adults and adolescents and prescribed
for any adolescent who asks for it.
Three medications are approved by the US Food and Drug Administration for PrEP for individuals weighing at
least 35 kg. Dosing and side effects of these medications are outlined in the Table 1.
The CDC recommends that renal function, hepatitis B serology, and 4th-generation HIV antigen/antibody
testing results be assessed before initiation of oral PrEP (Table 2). It is appropriate to prescribe PrEP on the
day of the office visit while awaiting these laboratory results. All patients should have a confirmed negative
HIV test finding within 7 days of being prescribed PrEP. The antiretrovirals used for PrEP offer only partial
treatment for HIV infection, making viral resistance possible if the patient has acquired HIV infection before
starting PrEP. If there is concern about acute HIV infection, a viral load (HIV-1 quantitative RNA) should be
obtained. Daily adherence to PrEP medications is critical for maximum prevention benefit, and providers
should counsel and monitor adolescents accordingly. However, studies have shown that HIV risk reduction
efficacy remains over 90% with up to 3 missed doses per week.
Suggested Reading(s)
American Academy of Pediatrics. STIs during preventive health care of adolescents. In: Kimberlin DW,
Barnett ED, Lynfield R, Sawyer MH, eds. Red Book: 2021-2024 Report of the Committee on Infectious
Diseases. 32nd ed. American Academy of Pediatrics; 2021. Accessed September 1, 2023. Red Book
Online
Brundrett ME. Human immunodeficiency virus preexposure prophylaxis in adolescents and young
adults. Pediatr Rev. 2022;43(1):28-36. doi:10.1542/pir.2020-002048
Emmanuel PJ, Mansfield J, Siberry GK. Human immunodeficiency virus infection: an update for
pediatricians. Pediatr Rev. 2022;43(6):335-346. doi:10.1542/pir.2020-001644
Pre-exposure prophylaxis. Centers for Disease Control and Prevention. Accessed September 1, 2023.
https://www.cdc.gov/hiv/risk/prep/index.html
Preexposure prophylaxis for the prevention of HIV infection in the United States—2021 update: a
clinical practice guideline. US Public Health Service, Centers for Disease Control and Prevention.
Accessed September 1, 2023. https://www.cdc.gov/hiv/pdf/risk/prep/cdc-hiv-prep-guidelines-2021.pdf
Content Domain
Preventative Pediatrics
Table 1. Medications Approved by the US Food and Drug Table 2. Timing of Oral PrEP-Associated Laboratory Tests.
Administration for HIV Pre-exposure Prophylaxis. Test Screening/ Q3 Q6 Months Q 12 When
Generic Name Dosing Frequency Common Patient Baseline Months Months Stopping
Side Effects Population Visit PrEP
HIV Test X* X X*
Cabotegravir 600 mg Once every Injection site Adults with
intramuscular 2 months reactions significant eCrCl X If age If age <50 X
gluteal injection (pain, renal disease ≥50 or or eCrCL
induration) or difficulty eCrCL <90 ≥90 mL/
adhering to mL/min min at
oral regimen at PrEP PrEP
Emtricitabine/ 200 mg/25 mg Once per Diarrhea Men and initiation initiation
tenofovir day transgender Syphilis X MSM/ X MSM/
alafenamide women only; TGW TGW
preferred over
F/TDF for Gonorrhea X MSM/ X MSM/
mildly TGW TGW
decreased Chlamydia X MSM/ X MSM/
renal TGW TGW
function or
bone density Lipid panel (for X X
concern emtricitabine/
tenofovir
Emtricitabine/ 200 mg/300 mg Once per Headache Any adult or alafenamide
tenofovir day Weight loss adolescent only)
disoproxil Abdominal with normal
fumarate pain renal function Hepatitis B X
(eCrCl>60 serology
mL/min) Hepatitis C MSM, TGW, MSM,
Abbreviations: eCrCl, estimated creatinine clearance; F/TDF, emtricitabine/tenofovir serology and PWID TGW, and
disoproxil fumarate. only PWID only
Courtesy of R. Savage Abbreviations: eCrCL, estimated creatinine clearance rate; F/TAF, emtricitabine
coformulated with tenofovir alafenam; MSM, men who have sex with men; PrEP,
preexposure prophylaxis; PWID, persons who inject drugs; TGW, transgender women.
*
Assess for acute HIV infection.
Source: Preexposure prophylaxis for the prevention of HIV infection in the United
States—2021 update: a clinical practice guideline. US Public Health Service, Centers for
Disease Control and Prevention.
AAP PREP 2024 - Question 197/267 Preventative Pediatrics Question 6/27
A recently adopted 12-month-old infant is seen for a health supervision visit. She was born via uncomplicated
vaginal delivery at 39 weeks’ gestation after an unremarkable pregnancy. Her birth weight and length were
3.5 kg and 49 cm, respectively. No other growth measurements are available. At this visit, her weight and
length are 7 kg and 64 cm, respectively.
Of the following, the BEST description of this infant’s growth is that she is
Correct answer is D
PREP Pearl(s)
In the first year after birth, healthy, term infants typically grow about 25 cm in length and 13 cm in head
circumference.
In the first year, appropriate weight gain is about 30 g per day for the first 3 months after birth, about 15 g per
day for the following 3 months, and about 10 g per day for the subsequent 6 months.
Measurement of the upper-to-lower segment ratio (typically 1.7:1 in infants and 1:1 in adults) can help identify
causes of abnormal growth.
Critique
Given the available growth data, the infant in the vignette can be best described as underweight and short for
age. Appropriate weight gain for a healthy term infant in the first year is about 30 g per day for the first 3
months after birth, about 15 g per day for the following 3 months, and about 10 g per day for the subsequent
6 months. In the first year after birth, length typically increases by about 25 cm and head circumference by
about 13 cm. This infant, therefore, should weigh about 9 kg and measure about 74 cm in length at age 12
months.
Monitoring growth is a critical part of infant health supervision. Growth measurements for infants younger
than 2 years should be plotted on standardized growth charts published by the World Health Organization
(available at cdc.gov/growthcharts/who_charts.htm). Further evaluation may be indicated for measurements
below or above 2 standard deviations from the mean or growth patterns that cross percentile lines on the
growth curve.
Numerous factors influence growth velocity (eg, gestational age, nutrition, metabolic conditions, genetic
factors). Certain conditions (eg, preterm birth, achondroplasia, and trisomy 21) have established growth
patterns; condition-specific growth charts can assist clinicians in appropriately assessing growth rates.
Once abnormal growth is identified, further evaluation, including referral to subspecialists, may be necessary
to identify the cause and facilitate appropriate management. More detailed growth measurements (eg,
upper-to-lower segment ratio) may help to differentiate among causes of abnormal growth. To calculate the
upper-to-lower segment ratio, the lower segment is measured from the floor to the top of the symphysis
pubis, and the upper segment is determined by subtracting the lower segment from the total height. Term
infants typically have an upper-to-lower segment ratio of about 1.7:1, which decreases to about 1:1 by
adulthood. This ratio is increased in children with short-limb skeletal dysplasias, preserved in older children
with constitutional tall stature, and decreased in children with some lysosomal storage disorders and older
children with Marfan and Klinefelter syndromes.
Suggested Reading(s)
Bamba V, Kelly A. Assessment of growth. In: Kliegman RM, St Geme JW, Blum NJ, Shah SS, Tasker RC,
Wilson KM, eds. Nelson Textbook of Pediatrics. 21st ed. Elsevier; 2020:151-156.
Braun LR, Marino R. Disorders of growth and stature. Pediatr Rev. 2017;38(7):293-304.
doi:10.1542/pir.2016-0178
DiMaggio DM, Cox A, Porto AF. Updates in infant nutrition. Pediatr Rev. 2017;38(10):449-462. doi:
10.1542/pir.2016-0239
Langston SJ, Krakow D, Chu A. Revisiting skeletal dysplasias in the newborn. Neoreviews.
2021;22(4):e216-e229. doi: 10.1542/neo.22-4-e216
Content Domain
Preventative Pediatrics
A 13-year-old is undergoing a routine health supervision visit. She has no medical diagnoses. Her mother is
concerned that the girl no longer wants to play on the school soccer team, after having been very involved
for the past 4 years. She spends hours alone in her room and is no longer interested in family time after
dinner. During her private and confidential interview, the girl reports that she is no longer interested in
sports. She enjoys watching videos online and connecting with friends over social media. She can name only
one friend at school but reports having many friends online. Her vital signs and physical examination findings
are normal. Her body mass index is at the 86th percentile for age and sex.
C. reassure her mother that these behaviors reflect normal adolescent social development
Correct answer is A
PREP Pearl(s)
Identity formation, a normal part of adolescent social development, may lead to changes in peer
relationships and interests.
Pediatric practitioners should screen all children aged 12 years or older for depression and suicide risk
at every health supervision visit.
Pediatric practitioners should initiate management of depression, which may include providing
resources for behavioral health support (eg, counseling, psychotherapy) and prescribing medication.
Critique
The 13-year-old in the vignette is in the early adolescent stage of development. Identity formation is
occurring, which may lead to changing interests and social environments. She has become more engaged in
online peer interactions and less engaged with family relationships. Although these behavioral changes may
be developmentally normal, they could be an indicator of mental health concerns. The best next step in care
is to administer a screening tool for depression and suicide risk in a confidential manner. Reassurance
without this screening would not be appropriate.
The girl’s body mass index is in the overweight category (≥85th to <95th percentile); therefore,
recommendations regarding healthy lifestyle should be discussed. However, there are many opportunities
for physical activity other than joining a school sports team, which the girl has indicated no longer interests
her. Healthy social media use and screen time should also be discussed. Referral to a mental health
professional may be necessary, depending on the results of the behavioral and mental health screening.
Adolescence is a time of rapid physical, social, emotional, and cognitive development. Peer interactions
become increasingly important and are vital to social development and identity formation. It is also common
during adolescence for mental health problems to emerge. The prevalence of depressive disorders increases
markedly during the adolescent years, especially in girls. In 2018, suicide became the second leading cause of
death in US youth aged 10 to 24 years. The American Academy of Pediatrics, the American Academy of Child
and Adolescent Psychiatrists, and the Children’s Hospital Association declared a national state of emergency
in children’s mental health in October 2021. Risk and protective factors for adolescent depressive disorders
are outlined in the Table.
The American Academy of Pediatrics recommends universal depression and suicide risk screening with an
age-appropriate validated screening tool at all health supervision visits for children aged 12 years or older.
Children aged 8 to 11 years should be screened for suicide risk when clinically indicated. Because depression
is generally first identified and treated by the primary care pediatric practitioner, the Guidelines for
Adolescent Depression in Primary Care (GLAD-PC) were developed to educate practitioners on best practices.
A toolkit is available at http://www.gladpc.org.
Suggested Reading(s)
American Academy of Pediatrics. Blueprint for youth suicide prevention. Blueprint. Accessed February
19, 2023. Suicide: Blueprint for Youth Suicide Prevention (aap.org)
American Academy of Pediatrics. Suicide prevention: signs and safety planning. Pediatric Patient
Education. In: McInerny TK, Adam HM, Campbell DE, DeWitt TG, Foy JM, Kamat DM, eds. American
Academy of Pediatrics; 2021:chap 288. Accessed September 1, 2023. Pediatric Care Online
Hagan JF, Shaw JS, Duncan PM. Adolescence visits: 11 through 21 years. In: Hagan JF, Shaw JS, Duncan
PM, eds. Bright Futures: Guidelines for Health Supervision of Infants, Children, and Adolescents. 4th
ed. American Academy of Pediatrics; 2017.
Hagan JF, Shaw JS, Duncan PM. Early adolescence: 11 through 14 year visits. In: Hagan JF, Shaw JS,
Duncan PM, eds. Bright Futures: Guidelines for Health Supervision of Infants, Children, and
Adolescents. 4th ed. American Academy of Pediatrics; 2017.
Sherman P, Torga AP. Suicide prevention in adolescence. Pediatr Rev. 2022;43(6):356-359.
doi:10.1542/pir.2021-005163
Zuckerbrot RA, Cheung A, Jensen PS, Stein REK, Laraque D; GLAD-PC Steering Group. Guidelines for
Adolescent Depression in Primary Care (GLAD-PC): part I. practice preparation, identification,
assessment, and initial management. Pediatrics. 2018;141(3):e20174081. doi:10.1542/peds.2017-4081
Content Domain
Preventative Pediatrics
A fully immunized 2-year-old child is seen by the pediatrician before a 1-month trip to Nigeria in December.
The child has a history of congenital encephalocele, which was repaired successfully without evidence of
ongoing cerebrospinal fluid leak on the child’s last cisternographic study.
Of the following, the MOST appropriate vaccine recommendation before the trip is
A. meningococcal B
B. meningococcal ACWY
C. pneumococcal 13
D. pneumococcal 23
Correct answer is B
PREP Pearl(s)
Meningococcal ACWY vaccine is routinely administered at ages 11 to 12 years, with a booster dose at
age 16 years. Currently, there are 3 quadrivalent conjugate vaccines licensed in the United States.
Early meningococcal ACWY vaccination is recommended in special situations for infants and young
children at high risk of developing meningococcal infection (eg, asplenia, HIV infection, persistent
complement deficiency, use of complement inhibitors [eculizumab, ravulizumab]).
A single meningococcal ACWY vaccine dose is recommended for children ≥ 2 years traveling to an
endemic region if they are too young to have received the vaccine or have not completed the
recommended immunization series. This additional vaccination does not replace the routine
recommended immunization series.
The meningococcal B vaccine is not currently recommended for routine immunization. Administration
is recommended for children ≥ 10 with risk factors including asplenia (functional or anatomical),
persistent complement deficiency, or use of complement inhibitors (eculizumab, ravulizumab).
Critique
Nigeria is located in the “meningitis belt” of sub-Saharan Africa. This region is hyperendemic (persistent high
levels of disease) with periodic epidemics (widespread regional disease) that can lead to 1,000 cases per
100,000 population. The risk of meningococcal meningitis from serogroup A is highest during the dry season
(December through June). Of the response choices, the child in the vignette will benefit most from a
meningococcal ACWY vaccine before travel. This additional meningococcal vaccination does not replace the
routine recommended immunization series for this child.
The most common serotype responsible for meningococcal meningitis in the United States and Europe is B,
followed by C, W, Y, and nongroupable meningococci. The minimum age for which meningococcal B
vaccination is approved is 10 years; it is recommended for children who have asplenia (functional or
anatomical) or persistent complement deficiency, as well as those who are using complement inhibitors (eg,
eculizumab, ravulizumab).
The conjugate 13-valent or 15-valent pneumococcal vaccine is a routine childhood vaccination, administered
in 4 doses at ages 2, 4, 6, and 12 to 15 months. This child is fully immunized, so he would have received all
recommended doses of this vaccine. The pneumococcal polysaccharide 23-valent vaccine is indicated in
situations that place a child at high risk of developing invasive pneumococcal disease. Indications include
chronic heart and lung disease, diabetes mellitus, cochlear implant, sickle-cell disease, congenital and
acquired immunodeficiencies, malignancies, immunosuppressive therapy, solid organ transplant, chronic
liver disease, and cerebrospinal fluid leak. The child in the vignette had a congenital encephalocele but does
not have an ongoing cerebrospinal fluid leak.
Vaccination is the cornerstone of preventing meningococcal infection. After the addition of the quadrivalent
(ACWY) conjugated meningococcal vaccine to the recommended adolescent immunization schedule, the
incidence of meningococcal disease decreased from 0.40 to 0.14 per 100,000 cases in the United States from
2006 to 2015. Currently, 3 quadrivalent conjugate vaccines are approved in the United States to protect
against serotypes ACWY and 2 recombinant protein vaccines are approved to protect against serotype B.
Routine Immunization
Meningococcal ACWY vaccine: administered at age 11 to 12 years with a booster dose at age 16 years
Meningococcal B vaccine: not currently recommended for routine immunization; however, can be
administered between ages 16 and 23 years as a college entry requirement or on the basis of risk and
shared clinical decision-making
Available formulations are administered as a 2-dose series; the interval between doses is
formulation specific
Special Situations
Early meningococcal ACWY vaccination is recommended for infants aged ≥2 months and young
children (younger than 10 years) at increased risk. Risk factors include underlying medical conditions
such as asplenia (functional or anatomical), HIV infection, or persistent complement deficiency, as well
as use of complement inhibitors (eculizumab, ravulizumab).
A single meningococcal ACWY vaccine dose is recommended for children traveling to countries in the
African meningitis belt or during Hajj if they are too young to have received the vaccine or have not
completed the recommended immunization series. This additional vaccination does not replace the
routine recommended immunization series.
Meningococcal B vaccine is recommended for children ≥ 10 who have asplenia (functional or
anatomical) or persistent complement deficiency, as well as those who use complement inhibitors (eg,
eculizumab, ravulizumab).
Suggested Reading(s)
American Academy of Pediatrics. Meningococcal infections. In: Kimberlin DW, Barnett ED, Lynfield R,
Sawyer MH, eds. Red Book: 2021-2024 Report of the Committee on Infectious Diseases. 32nd ed.
American Academy of Pediatrics; 2021. Accessed September 1, 2023. Red Book Online.
Caserta MT. Meningococcemia. In: McInerny TK, Adam HM, Campbell DE, Foy JM, Kamat DM, eds.
American Academy of Pediatrics Textbook of Pediatric Care. American Academy of Pediatrics; 2023.
Accessed September 1, 2023. Pediatric Care Online
Vaz LE. Meningococcal disease. Pediatr Rev. 2017;38(4):158-169. doi:10.1542/pir.2016-0131
Content Domain
Preventative Pediatrics
ABP Content Specification(s) / Content Area(s)
Know which serotypes are included in the meningococcal vaccine
Know the indications and schedule for the meningococcal vaccine
A 14-year-old is seen in the office with his mother to discuss concerns about his weight. He recently started in
a new school and is being bullied for his size and his inability to keep up with his peers in gym class. His body
mass index is 41 kg/m2. The boy’s mother asks whether bariatric surgery might be a reasonable option for
her son.
Of the following, the MOST accurate information the physician can provide is that
A. Adolescents with a body mass index of 40 kg/m2 or greater should have a comorbid condition
to be considered for bariatric surgery.
B. Laparoscopic adjustable gastric band surgery is the procedure of choice in the adolescent
population.
C. Lifestyle changes and watchful waiting provide similar outcomes to bariatric surgery in
adolescents with severe obesity.
D. Optimal bariatric surgery outcomes for pediatric patients occur in multidisciplinary clinic
settings.
Correct answer is D
PREP Pearl(s)
Severe obesity in children and adolescents is categorized as a body mass index of 35 kg/m2 or higher.
Bariatric surgery should be considered an evidence-based treatment option for children and
adolescents with severe obesity.
Pediatricians should be able to identify patients with severe obesity who meet criteria for bariatric
surgery.
Critique
The American Academy of Pediatrics 2019 Policy Statement, Pediatric Metabolic and Bariatric Surgery:
Evidence, Barriers, and Best Practices, addresses the emerging role of bariatric surgery in the management of
children and adolescents with significantly elevated body mass indexes (BMIs). Evidence supports the safety
and efficacy of bariatric surgery for pediatric patients with elevated BMIs when performed within a
multidisciplinary clinic model that uses pediatric experts in obesity, adolescent health, behavioral health,
nutrition, and exercise science, in addition to surgeons with specific expertise.
Preferred surgical modalities in this age group include the vertical sleeve gastrectomy and the Roux-en-Y
gastric bypass. The laparoscopic adjustable gastric band has shown limited long-term success, with higher-
than-expected complications, in the adult population and disappointing outcomes in the pediatric
population. It is currently only approved by the US Food and Drug Administration for patients 18 years and
older.
Anticipatory guidance regarding lifestyle modification remains the key initial intervention pediatricians should
provide to individuals at risk for obesity and those with mild to moderate elevations in their BMIs. However,
current evidence indicates that those with severely elevated BMIs (≥35 kg/m2) are unlikely to achieve clinically
significant decreases in weight and BMI with lifestyle-based weight management programs alone and that
watchful waiting may result in a higher BMI and an increased incidence of comorbid conditions (eg, type 2
diabetes mellitus and hypertension).
Current indications for bariatric surgery in children and adolescents are outlined in the Table. For individuals
with a BMI of 35 kg/m2 or higher but less than 40 kg/m2, a comorbid condition (eg, hypertension and type 2
diabetes) should be present. For those with a BMI of 40 kg/m2 or higher, indications for bariatric surgery do
not include a comorbid condition.
Suggested Reading(s)
Armstrong SC, Bolling CF, Michalsky MP, et al. Section on Obesity, Section on Surgery. Pediatric
metabolic and bariatric surgery: evidence, barriers, and best practices. Pediatrics.
2019;144(6):e20193223. doi:10.1542/peds.2019-3223
Cuda S, Censani M. Progress in pediatric obesity: new and advanced therapies. Curr Opin Pediatr.
2022;34:407-413. doi:10.1097/MOP.0000000000001150
Daniels SR, Hassink SG. Committee on Nutrition. The role of the pediatrician in primary prevention of
obesity. Pediatrics. 2015;136(1):e275-e292. doi:10.1542/peds.2015-1558
Hampl SE, Hassink SE, Skinner AC, et al. Clinical practice guideline for the evaluation and treatment of
children and adolescents with obesity. Pediatrics. 2023;151(2):e2022060640. doi:10.1542/peds.2022-
060640
Levinson B. Obesity and metabolic syndrome. Point-of-Care Quick Reference. Pediatric Care Online.
American Academy of Pediatrics. March 9, 2021. Accessed December 7, 2022. Pediatric Care Online
Content Domain
Preventative Pediatrics
A healthy, partially vaccinated 7-year-old is seen for a health supervision visit. His mother states they have
moved frequently and so she was unable to have him vaccinated on time. He received 2 inactivated
poliovirus vaccines at ages 2 and 4 months but has not received further doses.
Of the following, the BEST recommendation regarding polio vaccination for this boy is that he should
Correct answer is B
PREP Pearl(s)
A child older than 4 years who has received 2 doses of poliovirus vaccine should receive one additional
dose.
The only absolute contraindication to receiving the poliovirus vaccine is a history of a severe allergic
reaction to a component of the vaccine or a previous dose.
Immunocompromised individuals should receive only inactivated poliovirus vaccine (not the oral
poliovirus vaccine) owing to the risk of acquiring vaccine-associated paralytic polio.
Critique
The child in the vignette is older than 4 years and has received 2 doses of poliovirus vaccine. According to the
Centers for Disease Control and Prevention guidelines, he should receive 1 more dose. He would need 2
additional doses if he was between the ages of 10 months and 4 years. He does not need to restart the series
even though many years have passed since his last dose. Restarting the poliovirus vaccine series is indicated
only for extenuating circumstances (eg, undergoing a bone marrow transplant).
The poliovirus vaccine has effectively eradicated poliovirus infection in many countries. There are 2 forms of
poliovirus vaccine, an inactivated poliovirus vaccine (IPV) administered intramuscularly or subcutaneously
and an orally administered live virus poliovirus vaccine (OPV). Because of its extremely good safety profile,
the IPV is the only poliovirus vaccine administered in the United States and many other countries. Potential
side effects of the IPV include transient fever with malaise and local injection site reactions.
The live virus OPV can cause vaccine-associated paralytic polio and the development of vaccine-derived
poliovirus strains. In fact, the number of infections caused by vaccine-derived polioviruses is now greater
than those caused by wild polioviruses. Inactivated poliovirus vaccine contains serotypes 1, 2, and 3; the OPV
contained serotype 2 until 2016. An overwhelming number of vaccine-derived infections caused by serotype
2 were derived from the OPV. Oral poliovirus vaccine given before 2016 may not be counted toward the
vaccine schedule unless stated as trivalent.
At least 95% of IPV recipients develop protective antibodies after 2 doses and 99% after 3 doses. Protection
likely lasts a lifetime. Immunocompromised individuals may have a decreased immune response, depending
on their specific condition, but should receive the standard number of doses. Immunocompromised
individuals should only receive IPV (not OPV) owing to the risk of acquiring vaccine-associated paralytic polio.
The only absolute contraindication to receiving the poliovirus vaccine is a history of a severe allergic reaction
to a component of the vaccine or to a previous dose.
Suggested Reading(s)
American Academy of Pediatrics. Poliovirus infections. In: Kimberlin DW, Barnett ED, Lynfield R, Sawyer
MH, eds. Red Book: 2021–2024 Report of the Committee on Infectious Diseases. 32nd ed. American
Academy of Pediatrics; 2021. Accessed September 1, 2023. Red Book Online
Carpenter PA, Englund JA. How I vaccinate blood and marrow transplant recipients. Blood.
2016;127(23):2824-2832. doi:10.1182/blood-2015-12-550475
Estivariz CF, Link-Gelles R, Shimabukuro T. Poliomyelitis. In: Hall E, Wodi AP, Hamborsky J, Morelli V,
Schillie S, eds. Epidemiology and Prevention of Vaccine-Preventable Diseases. 14th ed. Public Health
Foundation; 2021:chap 18. Accessed September 1, 2023.
https://www.cdc.gov/vaccines/pubs/pinkbook/polio.html
Immunization schedules. Center for Disease Control and Prevention. Accessed September 1, 2023.
https://www.cdc.gov/vaccines/schedules/index.html
Content Domain
Preventative Pediatrics
A 17-year-old, new to the practice, is seen for a sports preparticipation evaluation before the start of
basketball season. They are healthy and have no significant medical history. On a standard preparticipation
evaluation form, they acknowledges having had 2 episodes of syncope during cross country meets this
academic year. The first event occurred shortly after the end of a race; they describes feeling dizzy and
developing tunnel vision before losing consciousness. The other episode occurred several hundred yards
before the finish line of a race; they felt “weird” and had difficulty keeping their balance, resulting in a fall. It is
unclear if they truly lost consciousness and they do not have significant recollection of the event. Evaluation
by the school trainer after each incident revealed with no concerning findings. The patient did not seek
further medical attention because they felt better several minutes after each episode. In the office today,
their vital signs, growth, and physical examination findings are normal.
Courtesy of M. Carr
A. clear the adolescent for sports participation and recommend vigorous hydration before and
during sports activities
B. clear the adolescent for sports participation without restrictions or further evaluation
C. Refer the adolescent to a cardiologist and defer clearance for sports participation until after
evaluation
D. refer the adolescent to the emergency department for further evaluation and defer clearance for
sports participation
Correct answer is C
PREP Pearl(s)
Syncope occurs in 15% to 20% of pediatric patients; a neurocardiogenic or vasovagal origin is the most
common cause.
Cardiac syncope is rare in pediatrics; it is vital to identify the circumstances of the event, as well as any
concerning personal or family history to suggest a cardiac cause.
Thorough history and physical examination are key in the evaluation of syncope; a baseline
electrocardiogram is an appropriate additional diagnostic tool.
Critique
The best next step in this adolescent’s management is referral to a cardiologist for evaluation and defer
clearance for sports participation until after their evaluation. Their history of possible exertional syncope and
likely postexertional syncope is concerning for a cardiac cause for her symptoms.
Syncope occurs in 15% to 20% of children and adolescents. Presyncopal symptoms are even more common,
particularly in adolescents. Syncope is defined as a sudden and transient loss of consciousness and postural
muscle tone that reverses without intervention. Presyncope or near-syncope describes symptoms of
lightheadedness, dizziness, changes in vision and/or hearing, or unsteadiness or weakness without loss of
consciousness. Children are often mislabeled as having had a syncopal episode where there is not a loss of
postural tone, only an altered level of consciousness. This issue is more than semantic because the
differential diagnosis is divergent.
True syncope is a concerning symptom for patients, families, and pediatric practitioners. Reassuringly, most
cases do not have a primary cardiac cause; rather, approximately 70% of pediatric syncope is
neurocardiogenic or reflex in origin. However, because syncope can be a primary symptom of a significant
pathologic condition, it is vital to obtain further information to assist in ruling in or out a cardiac cause. The
personal and family history, a detailed description of the syncopal event, and a careful physical examination
help guide clinical decision-making and the need for further testing.
Concerning or “red flag” findings suggestive of a cardiac cause of syncope include the following:
Personal history
Event history
Exertional syncope
Syncope initiated by loud noise or emotion
Absence of prodromal symptoms (dizziness, lightheadedness, or vision changes)
Syncope preceded by acute tachycardia or palpitations or significant chest pain
Syncope in a child younger than 8 years
Family history
Cardiomyopathy
Genetic arrhythmogenic syndromes (eg, long QT, catecholaminergic polymorphic ventricular
tachycardia [CPVT])
Pacemaker or defibrillator placement (especially at a young age)
Heart failure at a young age
Unexplained sudden death before 50 years of age (eg, drowning or unexplained car crash) or known
sudden cardiac death
Several primary cardiac diseases can lead to syncope. Identification of concerning history and/or physical
examination findings can point to specific causes. These cardiac conditions include but are not limited to the
following:
After a careful history review and physical examination, moving forward with a baseline resting
electrocardiogram (ECG) is very reasonable. It is important to remember that ECG findings can aid in the
diagnosis of some conditions, specifically those with a primary electrical abnormality (eg, WPW syndrome,
Brugada syndrome, and long QT) but performs less well with structural disease. Individuals with coronary
abnormalities or CPVT often have a normal resting ECG. Those with HCM, DCM, myocarditis, significant aortic
stenosis, or long-standing pulmonary hypertension can have variable ECG findings; many are abnormal.
The adolescent in the vignette had 1 clear syncopal episode and 1 presyncopal vs syncopal episode; both
occurred during or around physical exertion, which is a concerning feature. Postexertional syncope is often
multifactorial (dehydration, heat-related, or hyperventilation) and is less likely to have a primary cardiac
cause than exertional syncope. However, certain arrhythmias are exacerbated by high sympathetic tone and
can present in this manner. The history surrounding a true exertional syncopal event can often be vague,
making the details of the event difficult to obtain. The adolescent in the vignette has a normal ECG (sinus
bradycardia and sinus arrhythmia [appropriate for an older teenage aerobic athlete]), which is reassuring,
but given their desire to continue to pursue competitive, high-intensity athletics, they require further
evaluation by a cardiologist.
Clearing the adolescent for sports participation without further evaluation is not appropriate, considering
their history and symptoms, even with normal physical examination findings and a normal ECG. Although
their symptoms may ultimately prove to have a neurocardiogenic or vasovagal mechanism, which could
respond to maximized fluid intake, the adolescent still requires consultation with a cardiologist. Counseling
pediatric athletes regarding vigorous hydration before, during, and after high-level or competitive individual
or team activity is important and can mitigate the potential for exertional symptoms. The adolescent is
clinically stable and currently asymptomatic. They do not require urgent evaluation in an emergency
department.
Suggested Reading(s)
Adam HM. Syncope. Point-of-Care Quick Reference. Pediatric Care Online. American Academy of
Pediatrics. September 30, 2020. Accessed December 7, 2021. Pediatric Care Online
Cannon B, Wackel P. Syncope. Pediatr Rev. 2016;37(4):159-167; quiz 168. doi:10.1542/pir.2014-0109
Friedman KG, Alexander ME. Chest pain and syncope in children: a practical approach to the diagnosis
of cardiac disease. J Pediatr. 2013;163(3):896-901.e1-3. doi:10.1016/j.jpeds.2013.05.001
Paris Y, Toro-Salazar OH, Gauthier NS, et al; New England Congenital Cardiology Association (NECCA).
Regional implementation of a pediatric cardiology syncope algorithm using Standardized Clinical
Assessment and Management Plans (SCAMPS) methodology. J Am Heart Assoc. 2016;5(2):e002931.
doi:10.1161/JAHA.115.002931
Redd C, Thomas C, Willis M, Amos M, Anderson J. Cost of unnecessary testing in the evaluation of
pediatric syncope. Pediatr Cardiol. 2017;38(6):1115-1122. doi:10.1007/s00246-017-1625-6
Content Domain
Preventive Pediatrics
The correct answer is: Refer the adolescent to a cardiologist and defer clearance for sports participation until
after evaluation
View Peer Results
AAP PREP 2024 - Question 203/267 Preventative Pediatrics Question 12/27
During a health supervision visit, a parent expresses concern that her 6-year-old is distracted at school and
the teacher often calls regarding her behavior. Her mother states that bedtime is often a difficult experience
and her child may not fall asleep until 11 PM. Counseling is provided regarding the importance of a bedtime
routine.
Correct answer is C
PREP Pearl(s)
Lack of sleep and excess sleep are associated with obesity and mental health disorders.
The ideal duration of sleep varies according to age; infants require 12 to 16 hours total (including
naps), whereas adolescents require 8 to 10 hours.
The importance of sleep quality and duration, facilitated by consistent bedtime routines with
avoidance of screen time, should be discussed at health supervision visits.
Critique
Lack of sleep and too much sleep both have negative behavioral and health effects. A lack of sleep can lead to
inattention, poor behaviors, impulsivity, obesity, depression, and hypertension. Excess sleep is associated
with obesity and mental health disorders. Bedtime routines that improve both sleep quality and duration
lead to better overall health.
Consistent bedtime routines promote healthy sleep. These routines are most successful if they consistently
start at the same time each night (including weekends) rather than when children appear tired. Consistently
following a bedtime activity pattern that children are able to recognize is also helpful. One approach is to
choose something from each of 4 categories:
Participating in these activities in the same order each night helps the child to calm down and be ready for
sleep.
Screen time within 30 minutes of bedtime significantly interferes with good sleep. It is recommended that
there be no screen time for 30 minutes before beginning the bedtime routine. Computers, phones, tablets,
television, and other screen devices should be removed from sleep spaces.
The ideal amount of sleep children need varies by age (Table). Bedtime rituals should take into consideration
school/day care hours. For example, a 6-year-old child who begins school at 7:45 AM and needs to wake by
6:30 AM should be asleep between 6:30 and 9:30 PM
Suggested Reading(s)
American Academy of Pediatrics. Brush, book, bed: how to structure your child’s nighttime routine.
Accessed December 2, 2022. https://www.healthychildren.org/English/healthy-living/oral-
health/Pages/Brush-Book-Bed.aspx/
Jenco M. AAP endorses new recommendations on sleep times. AAP News; June 13, 2016. Accessed
December 2, 2022. https://publications.aap.org/aapnews/news/6630?autologincheck=redirected
Krishna J, Kalra M, McQuillan ME. Sleep disorders in childhood. Pediatr Rev. 2023;44(4):189–202.
doi:10.1542/pir.2022-005521
Mindella JA, Williamson AA. Benefits of a bedtime routine in young children: sleep, development, and
beyond. Sleep Med Rev. 2018;40:93-108. doi:10.1016/j.smrv.2017.10.007
Splaingard ML, May A. Sleep disturbances (nonspecific). In: McInerny TK, Adam HM, Campbell DE, Foy
JM, Kamat DM, eds. American Academy of Pediatrics Textbook of Pediatric Care. 2nd ed. American
Academy of Pediatrics; 2016:chap 194. Pediatric Care Online
Content Domain
Preventive Pediatrics
Courtesy of J. Reed
AAP PREP 2024 - Question 204/267 Preventative Pediatrics Question 13/27
Nine-year-old fraternal twins, new to the practice, are seen for a health supervision visit. Their mother is
concerned that both children are overweight and that her son is much shorter than her daughter. Their
medical history, family history, and results of a review of systems all are unremarkable.
On physical examination, both children have normal vital signs. Their growth charts are shown in Figure 1.
The girl’s physical examination findings are normal; she has a sexual maturity rating of 1. The boy’s physical
examination is remarkable only for a round face and dorsocervical fat pad; his sexual maturity rating is 1.
Courtesy of K. Wu
Of the following, the BEST next step in the care of these siblings is to
A. advise that both children increase their physical activity and eat healthier diets
B. advise that the girl is growing well and the boy’s height will catch up once he begins puberty
C. obtain lipid studies for the boy and thyroid function studies for the girl
D. refer the boy to an endocrinologist and the girl to a nutritionist
Correct answer is D
PREP Pearl(s)
Children who exhibit excessive weight gain in combination with linear growth failure should be
referred to an endocrinologist; those who exhibit excessive weight gain with preservation of linear
growth without signs or symptoms of an underlying disorder may be referred to a nutritionist.
Anthropometric measurements and growth velocities should be interpreted in the context of physical
examination findings and personal and family histories to guide the interpretation and appropriate
evaluation.
Chronic conditions may affect linear growth; affected children should undergo growth monitoring
more frequently, and their health care should involve an interdisciplinary team.
Critique
The growth charts for the siblings in the vignette both exhibit excessive weight gain. The girl’s linear growth is
appropriate. However, the boy’s growth chart exhibits linear growth failure, and on physical examination
there are other signs of glucocorticoid excess (round face, dorsocervical fat pad). He should, therefore, be
referred urgently to an endocrinologist for further evaluation. As the girl does not have signs or symptoms of
an underlying disorder that is causing her increased weight, it would be appropriate to refer her to a
nutritionist for assistance with dietary management.
Advising that both children increase their physical activity and eat healthier diets may be useful, but it would
not address the boy’s likely glucocorticoid excess. Lipid studies are indicated for the boy but would not be
useful in investigating his underlying condition. Thyroid function studies are not recommended in the
evaluation of obesity if linear growth is preserved and no other signs or symptoms of thyroid disease are
present. Reassuring the mother that her daughter is growing well and her son will grow when he begins
puberty does not address the growth concerns about either child.
Growth is often a significant concern raised by families during health supervision visits. Growth parameters
should be measured at every health supervision visit and at visits in which growth is of concern.
Measurements should be plotted on standardized growth charts. The World Health Organization charts are
recommended for children younger than 2 years and the Centers for Disease Control and Prevention charts
for children 2 years or older. Weight-for-length is plotted for children younger than 2 years and body mass
index (BMI) for children 2 years or older.
Both single-point-in-time measurements and growth velocity measurements are necessary to help
differentiate between normal and abnormal growth. A child whose anthropometric measurements cross
percentiles over time may require further investigation. Length or height measurements that are above or
below 2 standard deviations from the mean may be considered abnormal. For children younger than 2 years,
weight-for-length values below the 2nd percentile are considered low and values above the 98th percentile
are considered high. For children 2 years and older, BMI levels below the 5th percentile are in the
underweight category, between the 85th and less than the 95th percentile in the overweight category, and
equal to or above the 95th percentile in the obesity category.
To guide the interpretation and potential evaluation of growth concerns, anthropometric measurements
should be interpreted in the context of physical examination findings and personal and family histories.
Figure 2 demonstrates several linear growth patterns. When evaluating each growth pattern, there are many
possibilities to consider. For example, the growth pattern indicated by the O is more than 2 standard
deviations below the mean. If exploration of this child’s family history reveals familial short stature, this
growth pattern is likely appropriate and the child is expected to meet full genetic height potential. If the
physical examination findings for the child exhibiting the X linear growth pattern reveal early pubertal
development, it could indicate that the child may not reach full genetic height potential. Finally, the linear
growth pattern indicated by the dot may at first raise concern regarding linear growth failure, but evaluation
may reveal delayed puberty and a delayed bone age, which may indicate the potential for future catch-up
growth.
Courtesy of K. Wu
Many chronic conditions affect linear growth. There may be poor nutrient absorption (eg, inflammatory
bowel disease, celiac disease), medication/treatment side effects (eg, chemotherapy, radiation), or growth
effects of genetic syndromes (eg, trisomy 21, Turner syndrome, Noonan syndrome, skeletal dysplasias).
Affected children should undergo growth monitoring more frequently, and the health care team should
include relevant interdisciplinary members—including mental health physicians, because linear growth
patterns that differ from those of peers may cause psychological distress. Indeed, all children with abnormal
growth should be screened for age-appropriate mental health concerns.
Suggested Reading(s)
Braun LR, Marino R. Disorders of growth and stature. Pediatr Rev. 2017;38(7):293–304.
doi:10.1542/pir.2016-0178
Kaplowitz P. Short stature. In: McInerny TK, Adam HM, Campbell DE, Foy JM, Kamat DM, eds. American
Academy of Pediatrics Textbook of Pediatric Care. American Academy of Pediatrics; 2023. Accessed
September 1, 2023. Pediatric Care Online
Hagan JF Jr, Shaw JS, Duncan PM. Promoting healthy weight. In: Hagan JF Jr, Shaw JS, Duncan PM, eds.
Bright Futures Guidelines for Health Supervision of Infants, Children, and Adolescents. 4th ed.
American Academy of Pediatrics; 2017:151-166.
Tang MN, Adolphe S, Rogers SR, Frank DA. Failure to thrive or growth faltering: medical,
developmental/behavioral, nutritional, and social dimensions. Pediatr Rev. 2021;42(11):590-603.
doi:10.1542/pir.2020-001883
Content Domain
Preventive Pediatrics
The correct answer is: refer the boy to an endocrinologist and the girl to a nutritionist
View Peer Results
AAP PREP 2024 - Question 205/267 Preventative Pediatrics Question 14/27
The parents of an 18-month-old in the office for a health supervision visit mention that they are moving to a
new home with an in-ground swimming pool. They ask for guidance on the most effective measure to protect
a child in this age group from drowning in a pool.
Correct answer is D
PREP Pearl(s)
Drowning is a leading cause of injury-related death in children and the leading cause of unintentional
injury-related deaths in children in the United Stated aged 1 to 4 years.
The American Academy of Pediatrics recommends that parents can consider enrolling children in swim
lessons beginning at 1 year of age; decisions should be based on the individual child’s physical and
developmental status.
Pool fencing is one of the most effective preventive methods of childhood drowning in residential
swimming pools.
Critique
Drowning is the leading cause of death by unintentional injury among children aged 1 to 4 years in the United
States. For children younger than 14 years, drowning is the second leading cause of death. Pool fencing is
one of the most effective preventive methods of childhood drowning in residential swimming pools.
Four-sided fencing that separates the pool from the house and yard (Figure) decreases the incidence of
submersion injuries by greater than 50%. The fence should be at least 4 ft high, the opening under the fence
should be less than 4 in, the space between the vertical bars should be less than 4 in, and there should be no
hand- or foot-holds a child could use to climb over it. The fence should allow easy visibility of the pool and
have a self-latching gate that opens away from the pool. Pool gate alarms may be a helpful layer in drowning
prevention, but there is no current evidence to support their effectiveness (Table).
Reprinted with permission from American Academy of Pediatrics. A parent’s guide to water safety. In:
Pediatric Patient Education.
Current terminology describes a drowning or submersion event as fatal or nonfatal and with or without
morbidity. Terms such as near, wet, dry, active, passive, secondary, or delayed drowning are no longer used.
Approximately 10% of fatal drownings occur in bathtubs, 38% in pools, and 43% in open water. It is important
to be aware of risk factors when counseling families about water safety. The highest rate of drowning occurs
between 0 and 4 years of age; 12- to 36-month-old children are the most vulnerable. There is a second peak
of drowning in adolescence; the rate of drowning for adolescent boys is 10 times higher than for adolescent
girls.
Because research has not demonstrated that in-hospital care improves drowning outcomes, focusing efforts
on drowning prevention is critical. Although prompt initiation of cardiopulmonary resuscitation for the
drowning person is associated with significant improvement in survival and long-term neurologic prognosis, it
is a reactionary measure and does not play a role in the prevention of drowning.
The American Academy of Pediatrics supports swim lessons for children older than 1 year. The goal of this
instruction is to reduce drowning risk through ongoing assessment of the child’s water competence (ability to
anticipate, avoid, and survive common drowning situations) as well as to promote parent-child bonding.
However, there is no current evidence to support age-specific recommendations regarding swim lessons and
drowning prevention. What remains critical to prevention is the close, attentive, and constant supervision of
young children near water by a competent adult who is free of distractions.
Suggested Reading(s)
Denny SA, Quan L, Gilchrist J, et al; AAP Council on Injury, Violence, and Poison Prevention. Prevention
of drowning. Pediatrics. 2019;143(5):e20190850. doi:10.1542/peds.2019-0850
Denny SA, Quan L, Gilchrist J, et al; AAP Council on Injury, Violence, and Poison Prevention. Prevention
of drowning. Pediatrics. 2021;148(2):e2021052227. doi:10.1542/peds.2021-052227
Frankel LR. Drowning and near drowning (submersion injuries). In: McInerny TK, Adam HM, Campbell
DE, DeWitt TG, Foy JM, Kamat DM, eds. American Academy of Pediatrics Textbook of Pediatric Care.
American Academy of Pediatrics; 2023. Accessed September 1, 2023. Pediatric Care Online
McCallin TE, Morgan M, Hart MLI, Yusuf Y. Epidemiology, prevention, and sequelae of drowning.
Pediatr Rev. 2021;42(3):123-130. doi:10.1542/pir.2019-0150
Content Domain
Preventive Pediatrics
ABP Content Specification(s) / Content Area(s)
Understand the epidemiology associated with drowning deaths
Counsel parents regarding safety measures for a home pool
The correct answer is: installation of a 4-sided fence separating the pool from the house
View Peer Results
Pre-event Provide close, constant, attentive Install 4-sided fencing that completely Swim where there are lifeguardsa Mandate 4-sided residential pool
supervision of children and poor isolates the pool from the house and fencingb
swimmersa yardb
Evaluate preexisting health conditionc Install self-closing and latching gatesb Attend to warning signagec Mandate life jacket wearb
Develop water competency, including Wear life jackets b
Swim at designated swim sites c
Adopt the Model Aquatic Health
swim abilitya Codec
Know how to choose and fit a life Install compliant pool drainsa Remove toys from pools when not in Increase availability of lifeguardsa
jacketc use to reduce temptation for children to
enter the poolc
Avoid substance usea Install door locksc Empty water buckets and wading poolsc Increase access to affordable and
culturally compatible swim lessonsc
Know the water’s hazards and Enclosures for natural bodies of waterc Lakefront slope gradientd Close high-risk waters during high-
conditionsc risk timesd
Swim at a designated swim sited Promote life jacket loaner programsc Develop designated natural water
swim sitesd
Learn CPRb Role model life jacket use by adultsa Enforce Boating Under the Influence
lawsb
Take a boater education coursec Make rescue devices available at swim
sitesc
Provide ability to call for helpb
Ensure functional watercraftc
Event Water survival skills c
Rescue device availablec Lifeguard or bystander responseb Emergency Medical Systemb
Postevent AED c
Early bystander CPR b
Advanced medical carec
Rescue equipmentc EMS responseb
Abbreviations, AED, automated external defibrillator; CPR, cardiopulmonary resuscitation; EMS, emergency medical services
a
Trials or diagnostic studies with minor limitations; consistent findings from multiple observational studies.
b
Well-designed and conducted trials, meta-analyses on applicable populations.
c
Expert opinion, case reports, reasoning from first principles.
d
Single or few observational studies or multiple studies with inconsistent findings or major limitations.
Reprinted with permission from Denny SA, Quan L, Gilchrist J, et al; AAP Council on Injury, Violence, and Poison Prevention. Prevention of drowning. Pediatrics. 2021;148(2):10.
AAP PREP 2024 - Question 206/267 Preventative Pediatrics Question 15/27
A family new to the practice brings their 2-month-old and 4-year-old children to the office for health
supervision visits. They do not intend to vaccinate their 2-month-old infant and do not want any more
vaccines given to their 4-year-old (he was last vaccinated at 2 years of age).
Of the following, the MOST effective approach to managing this family's concerns is to
A. inform the family that only vaccinated children are seen in the practice
B. provide the family with evidence regarding vaccine safety and efficacy
C. request information regarding their desire to not vaccinate and respond to that concern
Correct answer is C
PREP Pearl(s)
Vaccine hesitancy is best thought of as a period of uncertainty during which the decision to vaccinate
may be affected by many factors.
Alternative vaccine schedules are generally not recommended because of concerns that they will not
provide children with ideal protection. No alternate vaccine schedule has been evaluated and deemed
safer or more effective than the standard schedule.
Best practices regarding conversations with vaccine-hesitant families should focus on communication
that builds rapport.
Critique
Vaccine hesitancy is best thought of as a period of uncertainty during which the decision to vaccinate may be
affected by many factors. It is a period of both vulnerability and opportunity. A 2012 literature review (Leask)
defined 5 categories of vaccine acceptance: the unquestioning acceptor, the cautious acceptor, the hesitant,
the late or selective vaccinator, and the refuser of all vaccines. Most families fall into 1 of the first 2 categories
(45%-65%), whereas the smallest number of families fall into the absolute refusal category (<2%).
The rise of social media and the COVID-19 pandemic have brought the issue of vaccine hesitancy into the
spotlight. Vaccine questioning and reluctance have been amplified by social media platforms. The internet
allows for people with shared beliefs to organize without geographic limitations, creates opportunities for
spreading misinformation, and increases the public's concern around the effectiveness and safety of
vaccines.
Alternative vaccine schedules are generally not recommended because of concerns that they will not provide
children with the ideal level of protection. No alternate vaccine schedule has been evaluated and deemed
safer or more effective. However, individual deviation from the recommended schedule may be considered if
the alternative would be no vaccine acceptance.
No published data are available regarding the outcomes of a strict policy of dismissal from a practice for
refusal to vaccinate. However, there are ethical and legal concerns with refusing to care for these families;
dismissal must take into account state laws prohibiting patient abandonment. There are also public health
concerns regarding unvaccinated families clustered around specific practices.
The most effective strategies used with vaccine-hesitant families include eliciting and responding to the
families’ specific concerns, acknowledging actual vaccine risks and adverse effects, and providing the family
with opportunities to return to the office for further discussion. Best practices regarding these conversations
focus on communication that builds rapport; how the information is discussed is more important than the
specific information provided.
Presumptive language around vaccine delivery increases vaccine acceptance (ie, “Today you are due for your
flu vaccine” as opposed to “Would you like your flu vaccine today?”).
Suggested Reading(s)
Edwards KM, Hackell JM, Committee on Infectious Diseases, Committee on Practice and Ambulatory
Medicine. Countering vaccine hesitancy. Pediatrics. 2016;138(3):e20162146. doi:10.1542/peds.2016-
2146
Larson HJ, Gakidou E, Murray CJL. The vaccine-hesitant moment. N Engl J Med. 2022;387(1):58-65.
doi:10.1056/NEJMra2106441
Leask J, Kinnersley P, Jackson C, et al. Communicating with parents about vaccination: a framework for
health professionals. BMC Pediatr. 2012;12:154. doi:10.1186/1471-2431-12-154
Natbony J, Genies M. Vaccine hesitancy and refusal. Pediatr Rev. 2019;40(suppl 1):22-23.
doi:10.1542/pir.2019-0072
Content Domain
Preventive Pediatrics
The correct answer is: request information regarding their desire to not vaccinate and respond to that
concern
View Peer Results
AAP PREP 2024 - Question 207/267 Preventative Pediatrics Question 16/27
A 6-month-old girl is seen for a health supervision visit. At her 4-month visit she received her second vaccine
doses of diphtheria-tetanus-acellular pertussis (DTaP), inactivated poliovirus (IPV), hepatitis B, Haemophilus
influenzae type B (HIB), pneumococcal 13-valent conjugate vaccine (PCV), and rotavirus as recommended by
the Advisory Committee on Immunization Practices (ACIP). The day after receiving her vaccines, she was
febrile with decreased energy and had a 2-minute tonic-clonic seizure. She was taken to the emergency
department, where her evaluation showed normal findings on a complete blood count with differential,
urinalysis, metabolic panel, and negative blood and urine cultures. She has had no seizures since that time.
Today, she appears healthy with normal physical examination findings and normal development for age.
C. all vaccines routinely recommended for a 6-month-old infant, with DT in place of DTaP
Correct answer is A
PREP Pearl(s)
Contraindications to administration of a diphtheria-tetanus-acellular pertussis (DTaP) vaccine are
limited to an anaphylactic reaction to a component of the vaccine or severe neurologic
sequelae/encephalopathy within 7 days of receiving the vaccine.
Individuals that experience an Arthus type III hypersensitivity reaction to a diphtheria-tetanus-acellular
pertussis (DTaP) vaccine should generally wait at least 10 years before receiving their next dose; risks
vs benefits should be considered.
An individual with a history of Guillain-Barré syndrome should wait 6 weeks before receiving their next
diphtheria-tetanus-acellular pertussis (DTaP) vaccine.
Critique
At this office visit, the girl in the vignette should receive all vaccines routinely recommended for a 6-month-
old infant. Vaccination with diphtheria-tetanus-acellular pertussis (DTaP) vaccine is contraindicated for an
individual that had an anaphylactic allergic reaction to a previous DTaP vaccine or to any of the components
of a DTaP vaccine. Vaccination with a product containing a pertussis component is contraindicated for
anyone that experienced encephalopathy (ie, coma, decreased level of consciousness, prolonged seizure)
within 7 days of receiving a pertussis-containing vaccine. The infant in the vignette had a short, tonic-clonic,
febrile seizure the day after receiving her routine 4-month-old vaccines, which included the DTaP vaccine.
This is not a contraindication to receiving subsequent DTaP vaccines as recommended in the routine vaccine
schedule.
Among the limited indications to alter the recommended childhood vaccination schedule, physicians and
families should consider risks versus benefits of delaying vaccination in the following situations:
Arthus-type hypersensitivity reaction; This is a type III hypersensitivity reaction involving
deposition of Ag-Ab immune complexes at the site of vaccination, causing local vasculitis.
Symptoms include pain, swelling, induration, edema, and, in very severe cases, may progress to
fibrinoid necrosis. The reactions generally develop 6 to 12 hours after a tetanus-containing
vaccine is administered, peak within 12 to 36 hours, and resolve spontaneously over a few days.
Subsequent DTaP/Tdap (tetanus-diphtheria-acellular pertussis) vaccines should be given after a
full 10-year interval, but they may be administered earlier if the benefits outweigh the risks.
Guillain-Barré; DTaP/Tdap is not recommended within 6 weeks of a history of Guillain-Barré
syndrome, but may be administered earlier if the benefits outweigh the risks.
In the late 1920s, Bordetella pertussis infection was responsible for a very high rate of mortality, primarily
among young infants. Developing a vaccine for B pertussis was a high priority, and tetanus-diphtheria-
pertussis (DTP) vaccines were introduced in the United States in the 1940s. The initial vaccine was a whole-
cell pertussis vaccine (DTwP), which was 70%-90% effective in preventing pertussis disease. The DTwP vaccine
was suspected to cause a host of side effects, including significant local reactions, febrile seizures, and more
serious neurologic sequelae including acute encephalopathy. Although there is subsequent uncertainty as to
whether the DTwP vaccine was responsible for these neurologic sequelae, in 1991 an acellular pertussis
vaccine (DTaP) was approved by the FDA. It was initially recommended for the booster doses of the DTP
series (15 months and 4 years), and in 1997 it was recommended for all 5 doses of childhood DTP
vaccination. The DTwP vaccines contain more than 3,000 pertussis antigens as compared with the DTaP
vaccines, which contain between 1 and 5 antigens. In addition, DTaP vaccines contain no liposaccharide (a
contributor to the DTwP vaccine side effects). Diphtheria-tetanus-acellular pertussis vaccines are less reactive
than DTwP vaccine, however, they are also less effective; they are one factor contributing to the resurgence
of B pertussis disease that began in the 1990s. The higher case numbers are also believed to be due to better
recognition of the disease and a simpler diagnosis method, using polymerase chain reaction testing.
Bordetella pertussis infection in adults is common and frequently undiagnosed; it is the major cause of
pertussis infection in infants and children. Early childhood vaccination did not significantly decrease the
incidence of pertussis infection in adolescents and adults, as immunity from the childhood series wanes with
time. Therefore, since 2005, administration of an adolescent dose, using a Tdap formulation, is
recommended for children at age 11 to 12 years.
Suggested Reading(s)
Cherry JD. The 112-year odyssey of pertussis and pertussis vaccines—mistakes made and implications
for the future. J Pediatr Infect Dis. 2019;8(4):334-341. doi:10.1093/jpids/piz005
Centers for Disease Control and Prevention. Diphtheria, tetanus, and pertussis vaccine
recommendations. Updated September 6, 2022. Accessed September 1, 2023.
https://www.cdc.gov/vaccines/vpd/dtap-tdap-td/hcp/recommendations.html
Havers HP, Moro PL, Hariri S, Skoff T. Pertussis. In: The Pink Book. Centers for Disease Control and
Prevention; 2021:chap 16. Accessed September 1, 2023.
https://www.cdc.gov/vaccines/pubs/pinkbook/pert.html
Content Domain
Preventive pediatrics
During family-centered rounds in the neonatal intensive care unit, the mother of a 5-week-old female infant
born at 24 weeks’ gestation asks why a retinopathy screen has not been done yet. She has read that the first
screening eye examination should be done at age 4 weeks. The infant has never been intubated. She was
initially treated with noninvasive ventilation with maximum peak inspiratory pressures of 18 cm H2O, and has
been on continuous positive-pressure ventilation (CPAP) of 6 cm H2O for the last 10 days. Initially, her
maximal inspiratory oxygen requirement was 60% fraction of inspired oxygen (FiO2), for a short while, but
has mostly been 21% to 28% FiO2. The infant has not experienced any significant comorbidities of
prematurity (eg, hypotension, sepsis, intraventricular hemorrhage, or necrotizing enterocolitis). She is
growing well on enteral feedings at 130 cal/kg/d. Her birth weight was 850 g; today’s weight is 1,100 g.
Of the following, the MOST accurate response to the mother’s question is that
Correct answer is C
PREP Pearl(s)
Retinopathy of prematurity screening should be performed for all infants with a birth weight of ≤1,500
g or gestational age ≤30 weeks (postmenstrual age).
Retinopathy of prematurity screening should be performed for infants with a birth weight between
1,500 g and 2,000 g or a gestational age of >30 weeks deemed at risk (eg, history of hypotension
requiring inotropic support, received oxygen supplementation for more than a few days, received
oxygen without saturation monitoring).
Retinopathy of prematurity (ROP) screening should be performed by an ophthalmologist with
experience in the examination of preterm infants for ROP.
Critique
The infant in the vignette’s first retinopathy of prematurity (ROP) screening examination should occur at
postmenstrual age 31 weeks (7 weeks chronologic age).
Retinopathy of prematurity, a leading cause of preventable childhood blindness, occurs due to abnormal
growth of the developing retinal blood vessels in premature infants. It is important for premature infants to
be screened for ROP and followed closely if ROP is identified, because early detection and treatment prevents
significant morbidity. The eye examinations need to be performed by an ophthalmologist with experience in
examining preterm infants for ROP. The American Academy of Pediatrics recommends ROP screening for all
infants with a birth weight ≤1,500 g or a gestational age ≤ 30 weeks. Screening is also recommended for
infants with a birth weight between 1,500 g and 2,000 g, or a gestational age of >30 weeks who are deemed
at risk for ROP due to factors, such as a history of hypotension requiring inotropic support, having received
oxygen supplementation for more than a few days, or having received oxygen without saturation
monitoring.
The onset of significant ROP correlates better with postmenstrual age (PMA) than chronologic age; the more
premature an infant is at birth, the more time it takes to develop serious disease. Therefore, the timing of
initiation of ROP screening should be determined by the infant’s postmenstrual age (Table) and any
significant comorbidities.
The infant in the vignette was born at 24 weeks’ gestation and has not experienced significant morbidities,
therefore her ROP screening can begin at PMA 31 weeks (7 weeks chronological age). Prolonged exposure to
high inspired oxygen levels increases the risk of ROP; intubation is not a specific risk factor. The infant does
not need to achieve a weight threshold for initiation of ROP screening.
Suggested Reading(s)
Campbell DE. Continuing care of the infant after transfer from neonatal intensive care. In: McInerny TK,
Adam HM, Campbell DE, DeWitt TG, Foy JM, Kamat DM, eds. American Academy of Pediatrics Textbook
of Pediatric Care. American Academy of Pediatrics; 2023. Accessed September 1, 2023. Pediatric Care
Online
Fierson WM; American Academy of Pediatrics Section on Ophthalmology; American Academy of
Ophthalmology; American Association for Pediatric Ophthalmology and Strabismus; American
Association of Certified Orthoptists. Screening Examination of Premature Infants for Retinopathy of
Prematurity. Pediatrics. 2018;142(6):e20183061. doi:10.1542/peds.2018-3061
Kim SJ, Port AD, Swan R, Campbell JP, Chan RVP, Chiang MF. Retinopathy of prematurity: a review of
risk factors and their clinical significance. Surv Ophthalmol. 2018;63(5):618-637.
doi:10.1016/j.survophthal.2018.04.002
Sharma M, VanderVeen DK. Identification and treatment of retinopathy of prematurity: update 2017.
NeoReviews. 2017;18(2):e84-e90. doi:10.1542/neo.18-2-e84
Content Domain
Preventive pediatrics
The correct answer is: the first eye examination will occur at a postmenstrual age of 31 weeks
View Peer Results
28 32 4
29 33 4
30 34 4
Older gestational age, high risk factorsb - 4
Shown is a schedule for detecting prethreshold retinopathy of prematurity with 99%
confidence, usually before any required treatment. -, not applicable.
a
This guideline should be considered tentative rather than evidence based for infants with
a gestational age of 22 to 23 weeks because of the small number of survivors in these
postmenstrual age categories.
b
Consider timing on the basis of the severity of comorbidities.
Reprinted with permission from Fierson WM; American Academy of Pediatrics, Section
on Ophthalmology; American Academy of Ophthalmology; American Association for
Pediatric Ophthalmology and Strabismus; American Association of Certified Orthoptists.
Screening examination of premature infants for retinopathy of prematurity. Pediatrics.
2018;142(6):e20183061. doi:10.1542/peds.2018-3061
AAP PREP 2024 - Question 209/267 Preventative Pediatrics Question 18/27
A 17-month-old is seen in the emergency department for injuries to his hands. He was playing near the
fireplace in the living room at home when he started crying. His parents noted burns on his hands,
immediately placed them in cold water, and brought him to the emergency department. The boy’s physical
examination findings are remarkable only for the wounds on his hands (Figure). The family is appropriately
upset and asks for recommendations regarding burn prevention in the home.
Courtesy of J. Reed
Correct answer is C
PREP Pearl(s)
Fireplaces should have a screen around them to prevent burns.
Helmets should be used during all wheeled activities (eg, bicycle, scooter, and rollerblades).
Firearms should be placed unloaded in a locked storage unit (eg, box, cabinet); the ammunition and
the weapon should be stored in separate locked locations.
Critique
The child in the vignette has a second- to third-degree burn on his hands from touching the glass cover in
front of the fireplace. It is recommended that all indoor fireplaces (gas or wood burning) have a screen in
front of them to prevent children from getting close enough to touch the hot exterior. Regarding the risk of
burns, there are no recommendations regarding gas versus wood-burning fireplaces. However, children with
asthma may experience exacerbation of symptoms with a wood-burning fireplace. Children should remain at
least 3 ft from outdoor fire pits or campfires.
Home safety should be addressed at all health supervision visits, focusing on age and developmentally
relevant risks. Children younger than 1 year are rapidly developing new skills, and risks to them can easily be
underestimated. Counseling regarding fall prevention should include using caution with stairs (eg, placing a
safety gate at both the top and bottom of stairs). The use of walkers can increase the risk of injury, especially
around stairs, and has not been shown to improve developmental outcomes. Using caution around changing
tables, beds, couches, and other elevated surfaces should be recommended, as young infants beginning to
roll and scoot can easily fall. Suffocation is a potentially fatal safety hazard for infants and young children (eg,
choking on small toys, balloons, plastic bags, popcorn); these types of items should be kept out of reach at all
times.
To decrease the risk of strangulation injury, caution must be taken with cords from window blinds: avoiding
looped cords, attaching cords to the wall with safety tassels, or using blinds with no cords.
Children between the ages of 1 and 4 years develop physical skills more quickly than cognitive skills.
Important topics to address as children gain the ability to climb include the following:
Avoid putting toys on top of furniture (attracting children to climb to reach them) and ensure furniture
(eg, dressers, bookcases) is secured to the wall; these simple steps can decrease the risk of serious
crush injuries under fallen furniture.
All windows should have guards, in addition to screens that cannot easily be pushed out.
In the kitchen, all pots and pans in use should be placed on back burners with the handles facing
toward the back of the stove; avoid having children in the kitchen while the oven is open.
Avoid storing toxic substances (eg, cleaning substances) in unmarked or easy-to-open containers. All
families should place the telephone number for the Poison Control Center (1-800-222-1222) near a
landline phone or save the number in their cell phones.
Safety measures should be put into place for children as they begin to use wheeled devices, such as scooters,
bicycles, and rollerblades. Children should wear helmets at all times when on these devices, from the time of
first use. When available, the presence of a helmet program in the clinic or provider’s office that provides
helmets at health supervision visits can significantly increase helmet use and prevent injuries. Proper wrist
protection for use with scooters and rollerblades should be discussed, as wrist fractures from falls with
unprotected outstretched hands are common.
All families should be aware of the risk of firearms. Firearms should be kept locked away, with ammunition
and the unloaded firearm locked up separately. The location of the key or passcode to the lockbox should not
be shared with children.
Additional household safety recommendations that can reduce injuries at all ages include the following:
Keeping home water heaters set at less than 120°F decreases the risk of burns.
Parents should be encouraged to never leave a car in the garage after starting the car to warm it up
and to instead move the car into the driveway first.
Install and test, at least yearly, smoke detectors and carbon monoxide detectors.
Suggested Reading(s)
Levin R, Smith GA. Safety and injury prevention. In: McInerny TK, Adam HM, Campbell DE, Foy JM,
Kamat DM, eds. American Academy of Pediatrics Textbook of Pediatric Care. 2nd ed. American
Academy of Pediatrics; 2016:chap 42. Pediatric Care Online
Sanders J, Modilner L. Child safety and injury prevention. Pediatr Rev. 2015;36(6):268-269.
doi:10.1542/pir.36-6-268
Content Domain
Preventive Pediatrics
ABP Content Specification(s) / Content Area(s)
Provide age-appropriate home safety information
Counsel parents regarding stairway safety
Advise a family regarding prevention of poisoning in children
A 2-day-old exclusively breastfed neonate is seen in the clinic. His mother is concerned that he has had a low
temperature (34.9 -35.9 °C) since birth, despite several attempts at “skin-to-skin care.” The neonate was
delivered at home by a midwife at 42 weeks' gestation. In the office, the newborn’s rectal temperature is 35
°C. His other vital signs are appropriate for age. Warming techniques are implemented.
Correct answer is C
PREP Pearl(s)
Small for gestational age is defined as a neonate with birth weight below the 10th percentile for their
gestational age.
A full-term newborn is born between 39 and 41 weeks’ gestation; preterm is defined as a neonate born
at less than 37 weeks’ gestation; post-term neonates are born at 42 weeks’ gestation or later.
Accurate head circumference measurement is critical to distinguish symmetric and asymmetric small
for gestational age neonates. Causes of symmetrical small for gestational age include chromosomal or
congenital anomalies, constitutional (familial) small size, and congenital infections.
Critique
The best next step in this post-term (42 weeks’ gestation) neonate’s management is to perform a point-of-
care glucose test. Postterm (postmature) newborns are usually small for gestational age (SGA) because of
placental insufficiency. They are at risk for hypothermia (temperature <36.5 oC) and hypoglycemia (whole
blood glucose <40 mg/dL [2.22 mmol/L]) because of inadequate subcutaneous fat and liver fat stores.
Therefore, it is important to check this neonate’s temperature, provide thermal care, perform a bedside
glucose test at the earliest opportunity, and provide treatment with intravenous glucose if needed. Although
a lumbar puncture, vitamin K prophylaxis (if not already given), and referral to a lactation consultant may be
needed, these steps should occur later in this neonate’s care.
Small for gestational age is defined as a birth weight below the 10th percentile for gestational age. This can
be further differentiated as symmetrical (weight, length, and head circumference are all below the 10th
percentile) and asymmetrical (head growth is not affected). Accurate head circumference measurement is
critical to distinguish these conditions. The most common cause of asymmetric SGA is placental insufficiency
(occurring most often in the third trimester). Causes of placental insufficiency include preeclampsia, maternal
hypertension, chronic maternal disease (eg, diabetes), and post-term gestation. Causes of symmetrical SGA
include chromosomal or congenital anomalies, constitutional (familial) small size, and congenital infections.
The designation of a newborn as having low birth weight is solely based on a birth weight less than 2,500 g.
Newborns with a birth weight less than 1,500 g are designated as very low birth weight, and those with a
birth weight less than 1,000 g are considered extremely low birth weight.
Characteristic clinical findings can help the physician differentiate among preterm, term, and post-term
newborns (Table). Post-term SGA neonates often have dry, peeling, loose skin and an increased risk of
meconium-stained amniotic fluid and meconium aspiration syndrome. The Ballard score is a bedside tool
that uses physical and neuromuscular assessment to help determine gestational age.
Genitalia
Prominent clitoris, small Labia majora and minora Labia majora cover clitoris and
Female
labia minora equally prominent minora
Infrequent eye
Interaction Alert with eye contact Hyperalertness
movements, no focus
Courtesy of J. Sharma
Suggested Reading(s)
American Academy of Pediatrics. Gestational age and size and associated risk factors. In: PCEP Book 1:
Maternal and Fetal Evaluation and Immediate Newborn Care. American Academy of Pediatrics;
October 2021.
American Academy of Pediatrics. Neonatology for Primary Care. American Academy of Pediatrics;
January 2020.
Ballard JL, Khoury JC, Wedig K, et al. New Ballard Score, expanded to include extremely premature
infants. J Pediatr. 1991;119(3):417-423. doi:10.1016/s0022-3476(05)82056-6
Gooding JR, McClead RE Jr. Initial assessment and management of the newborn. Pediatr Clin North Am.
2015;62(2):345-365. doi:10.1016/j.pcl.2014.12.001
Jackson K, Harrington JW. SGA and VLBW infants: outcomes and care. Pediatr Rev. 2018;39(7):375-377.
doi:10.1542/pir.2017-0091
Content Domain
Preventive Pediatrics
Gestational age <37 weeks Early term: 37-38 weeks ≧42 weeks
Full term: 39-40 weeks
Late term: 41 weeks
Ears Pinna flat stays Well-curved pinna, soft Thick cartilage, stiff
folded or slow with ready recoil
recoil
Genitalia
Female Prominent clitoris, Labia majora and minora Labia majora cover
small labia minora equally prominent clitoris and minora
Courtesy of J. Sharma
AAP PREP 2024 - Question 211/267 Preventative Pediatrics Question 20/27
A registered dietician is teaching a group of medical students about nutrition during their pediatrics rotation.
She explains that nutritional requirements vary by age as well as any medical conditions the child has. She
presents case scenarios of children from 3 weeks to 15 years of age and asks the students which of those
children has the highest average protein needs.
Correct answer is D
PREP Pearl(s)
Among healthy children of all ages, premature infants need the most protein (to support a rate of
growth that mirrors intrauterine growth). The amount of protein required to support growth decreases
from 55% in the first 3 months after birth to 10% by 8 years of age.
Although exercise and physical work increase energy needs, it is unclear whether the protein
requirement is also increased.
There is no tolerable upper limit for dietary proteins because data are insufficient to support a
recommendation in healthy children.
Critique
The 3-week-old girl born at 26 weeks’ gestation has the highest protein requirements. Protein accounts for
80% to 90% of the nutritional support required for normal growth in the third trimester of gestation. It is
estimated that between 24 and 28 weeks’ gestation, a preterm infant needs 4 g/kg daily of protein.
High intake of protein, especially casein, by small infants can result in acidosis, aminoacidemia, and
cylindruria (renal casts). In the past, there were limits set on the amounts of amino acids provided to preterm
infants. It has since become clear that preterm infants tolerate protein intakes as high as 4 g/kg daily and that
this higher concentration of protein results in both short- and long-term improved clinical outcomes.
The recommended daily allowance of protein is the average daily intake required to meet the nutrient needs
of most people of a specific age range and sex. Dietary amino acids that exceed the body's needs cannot be
stored, which is a major reason for the recommendation that children eat at intervals throughout the day.
During preschool and school age, children’s protein requirements decrease. Much of this change is related to
a decreased protein requirement for growth, leaving mostly a maintenance requirement. The daily protein
requirement to support growth decreases from 55% of total intake in the first 3 months after birth to 10% by
8 years of age (Table). Although children with certain conditions (eg, cancer and critical illness) may have
increased protein requirements, an 8-year-old child with well-controlled mild intermittent asthma would not
have an increase in their protein requirement.
During adolescence, maintenance protein requirements are estimated to be 0.66 g/kg daily (adult level).
There is a small increase (slightly higher in boys than girls) in the protein requirement during the adolescent
growth spurt (0.85 g/kg daily).
Although exercise and physical work increase energy needs, it is unclear whether there is a relative increase
in the need for protein. Both endurance and resistance exercise increase amino acid oxidation, which, in
theory, increases protein and amino acid needs. However, evidence has shown that, with continued training,
the increased use of some amino acids during physical exercise is balanced over time by decreased needs
during rest. For individuals wanting to increase muscle mass and enhance body composition, sports nutrition
organizations currently recommend protein intake between 1.4 and 2.0 g/kg daily along with a resistance
training program. For endurance competition, additional protein does not appear to enhance athletic
performance.
The tolerable upper limit is a measure of the level of consumption of a nutrient (eg, protein) that could be
dangerous. There is no upper limit for dietary proteins because the data are insufficient to support a
recommendation in healthy children. However, there are concerns that excessive protein intake before 2
years of age is a risk factor for childhood and adult obesity.
Suggested Reading(s)
Hay WW Jr. Nutritional support strategies for the preterm infant in the neonatal intensive care init.
Pediatr Gastroenterol Hepatol Nutr. 2018;21(4):234-247. doi:10.5223/pghn.2018.21.4.234
Kleinman RE, Greer FR. Proteins. In: Kleinman RE, Greer FR, eds. Pediatric Nutrition. 8th ed. American
Academy of Pediatrics; 2020:449-480.
Taylor SN, Martin CR. Evidence-based discharge nutrition to optimize preterm infant outcomes.
Neoreviews. 2022;23(2):e108-e116. doi:10.1542/neo.23-2-e108
Content Domain
Preventive pediatrics
Intake
(g/kg/d) (% of total)
0.5-3 mo 0.49 55 45
3-6 mo 0.30 43 57
6-12 mo 0.18 31 69
1-3 y 0.10 20 80
4-8 y 0.046 10 90
A 4-month-old boy is seen for a health supervision visit. He currently weighs 5.12 kg, which is only 120 g more
than at his last visit at 3 months of age. His length is 62 cm and tracking well at the 25th percentile (Figure).
The infant typically drinks 4 oz every 3 hours with 1 interval each night of 6 hours between feedings. He
drinks a standard cow milk formula mixed with 1 scoop per 2 oz of water. He does not choke or have reflux
symptoms. He does not have fussiness with feedings. There is no blood or mucus in his stools. He is meeting
age-appropriate developmental milestones. The infant’s mother asks how much formula per bottle he needs
for appropriate weight gain and catch-up growth, if he continues to take 7 bottles daily.
A. 4 ounces
B. 5 ounces
C. 6 ounces
D. 7 ounces
Correct answer is B
PREP Pearl(s)
Failure to thrive is defined as a decrease of ≥2 major weight percentiles after establishing growth on
the World Health Organization growth chart for children 0 to 2 years old or the Center for Disease
Control growth chart for children 2 to 20 years old.
Evaluation of an infant with failure to thrive should be based on the severity of the condition as well as
key findings in the history and physical examination.
Children with failure to thrive and malnutrition are at risk for short stature, developmental delays,
psychosocial delays, and cardiovascular disease and metabolic syndrome as adults.
Critique
The recommended dietary allowance (RDA) for infants 0 to 6 months of age is 108 calories/kg/day and 98
calories/kg/day for infants 6 to 12 months of age. This infant’s current weight is 5.12 kg. Based on his height
at the 25th percentile, his ideal weight for height = 6.4 kg. To determine the infant in the vignette’s caloric
needs for catch-up growth, the formula:
RDA × (ideal weight/current weight) = calories/kg/day should be used. For this infant:
Required calories = 108 calories/kg/day × (6.4 kg/ 5.12 kg) = 135 calories/kg/day
Total daily caloric need = 691.2 calories/day (5.12 kg × 135 cal/kg/day)
Standard cow milk formula contains 20 calories/oz
Formula requirement = 34.56 oz/day (691.2 calories/day ÷ 20 calories/oz) = 7 bottles of 4.9 oz each
Failure to thrive (FTT) is defined as a decrease of less than or equal to 2 major weight percentiles after
establishing growth on the World Health Organization growth chart for children 0 to 2 years old, or the
Center for Disease Control growth chart for children 2 to 20 years old. The prevalence of FTT in children in the
United States is between 2% and 10%. Those with lower socioeconomic status, chronic medical conditions,
refugees, and infants born weighing less than 2,500 g are at higher risk. Often these infants and children have
malnutrition, defined by the American Society for Parenteral and Enteral Nutrition (ASPEN) as an “imbalance
between nutrient requirements and intake that results in cumulative deficits of energy, protein, or
micronutrients that may negatively affect growth, development, and other relevant outcomes.” Children with
FTT and malnutrition are at risk for short stature, developmental delays, psychosocial delays, and even
cardiovascular disease and metabolic syndrome as adults.
The evaluation of an infant with FTT should be based on the severity of the condition and key findings of the
history and physical examination. If the cause appears to be insufficient caloric intake, the first steps are to
educate the family regarding appropriate feeding goals and monitor the weight for the effect of changes
made. Infants that are unable to take sufficient formula volume secondary to coughing or choking during
feedings, and those that have prolonged congestion and frequent respiratory infections, should have a
modified barium swallow study to evaluate for aspiration.
Laboratory evaluation should be based on clues identified from a thorough history (Table 2).
Suggested Reading(s)
Gunn VL, Barone MA. Nutrition. In: Robertson J, Shilkofski N, eds. The Harriet Lane Handbook: A
Manual for Pediatric House Officers. 17th ed. Mosby; 2005:chap 21.
Racine AD. Failure to thrive: pediatric undernutrition. In: McInerny TK, Adam HM, Campbell DE, DeWitt
TG, Foy JM, Kamat DM, eds. American Academy of Pediatrics Textbook of Pediatric Care. American
Academy of Pediatrics; 2021:chap 149. Accessed September 1, 2023. Pediatric Care Online
Tang MN, Adolphe S, Rogers SR, Frank DA. Failure to thrive or growth faltering: medical,
developmental/behavioral, nutritional, and social dimensions. Pediatr Rev. 2021;42(11):590-603.
doi:10.1542/pir.2020-001883
Content Domain
Preventive pediatrics
Table 1. Etiologies of Failure to Thrive by Category. Table 2. Laboratory Evaluation for Children with Failure to Thrive.
Insufficient caloric intake Decreased absorption Increased metabolic need Diagnostic Test Indications
• ●Decreased breast milk • ●Celiac disease • ●Chronic lung disease Complete blood count • ●Findings suggestive of anemia,
supply • ●Cholestasis • ●Chronic inflammation immunodeficiency, or malignancy.
• ●Feeding difficulties (eg, • ●Gastrointestinal • ●Congenital cardiac • ●(If microcytic anemia is present, consider iron
aspiration, hypotonia) infection disease and lead testing)
• ●Improper formula • ●Inflammatory • ●Malignancy
Chemistry panel • ●Concern for renal tubular acidosis
mixing (diluted formula) bowel disease • ●Systemic infection
• ●Monitor for refeeding syndrome in severe
• ●Pancreatic
cases of FTT
insufficiency
Celiac disease testing: • ●Infants that develop FTT after beginning to
Courtesy of J. Reed
Immunoglobulin A (IgA), tissue take gluten containing foods
transglutaminase IgA antibody
Urine • ●Signs and symptoms of Infection
• ●Concern for glycosuria
Stool • ●Evidence of malabsorption
Courtesy of J Reed.
AAP PREP 2024 - Question 213/267 Preventative Pediatrics Question 22/27
A 7-year-old boy who recently moved from another state is undergoing a routine health supervision visit. His
immunization record is shown.
Age Vaccine(s)
DTaP: diphtheria, tetanus, acellular pertussis; Hib: Haemophilus influenzae type b; IPV: inactivated poliovirus
vaccine; MMR: measles, mumps, rubella.
Orders are placed to administer the IPV, MMR, and varicella vaccines.
A. DTaP
B. DTP
C. Td
D. Tdap
Correct answer is D
PREP Pearl(s)
Fever, vaccine site redness and swelling, immunodeficiency, and family history of allergic reactions are
not contraindications to administration of diphtheria, tetanus, acellular pertussis vaccines.
Precautions regarding administration of the diphtheria, tetanus, acellular pertussis vaccines include
development of Guillain-Barré syndrome within 6 weeks after a dose, uncontrolled epilepsy, an
evolving neurological condition, fever >40.5 °C within 48 hours after a dose, and current severe acute
illness.
Contraindications to the diphtheria, tetanus, acellular pertussis vaccines include severe anaphylaxis to
a previous dose and encephalopathy (with no identified etiology) developing within 7 days after a dose.
Critique
Today, in addition to the inactivated poliovirus; measles, mumps, rubella; and varicella vaccines, the 7-year-
old boy in the vignette should receive the tetanus, diphtheria, acellular pertussis (Tdap) vaccine. The boy’s
vaccination status is complete through age 2 years. He has not received the vaccinations recommended to be
administered at age 4 to 6 years (5th dose of tetanus, diphtheria, acellular pertussis [DTaP]; 4th dose of
inactivated poliovirus; 2nd dose of measles, mumps, rubella; 2nd dose of varicella). The Tdap vaccine is
recommended for catch-up if the child is aged 7 years or older. If additional catch-up doses are needed, the
tetanus, diphtheria (Td) vaccine should be administered. Tdap vaccine is routinely administered at age 11 or
12 years. A booster is given every 10 years thereafter, or earlier in cases of exposure to pertussis. See the
Table for guidance on wound prophylaxis.
Five doses of DTaP are recommended as part of the childhood immunization series. The 5th dose is not
necessary if the 4th dose is given after age 4 years. The DTaP vaccine contains diphtheria toxoid, tetanus
toxoid, and inactivated acellular pertussis components. The Tdap vaccine has reduced amounts of diphtheria
toxoids and acellular pertussis components compared with the DTaP vaccine.
The DTP vaccine, which contains the whole cell pertussis, has not been available in the United States since
1997; it has been replaced by the DTaP (acellular pertussis) vaccine. Adverse effects (eg, seizure,
encephalopathy, and hypotensive-hyporesponsive episodes) that were seen with the DTP vaccine are unlikely
with the acellular vaccine (DTaP).
Common adverse effects of the DTaP vaccine include fever, local site redness and swelling, and occasionally
whole-limb swelling; these are not contraindications to future doses.
Relative contraindications to and precautions for the DTaP and Tdap vaccines include the following:
Absolute contraindications to the DTaP and Tdap vaccines include the following:
In cases of an evolving neurological illness of unknown etiology, DTaP can be deferred until the evaluation is
complete. If the decision is made to defer the pertussis component, the DT vaccine can be administered after
the child’s 1st birthday to complete the immunization series.
Immunodeficiency is not a contraindication to administration of the DTaP vaccine, although its effectiveness
may be impaired if the deficiency is severe. Stable neurological conditions, a family history of seizures, and a
family history of an allergic reaction to DTaP are not contraindications to the vaccine.
Combination vaccine products and multiple vaccines can be administered safely on the same day with an
effective immune response. If 2 live vaccines are required, they should be given on the same day. If unable to
be given on the same day, a minimum of 28 days is required between the administration of live vaccines for
an optimal immune response. Serious and specific adverse effects should be reported to the Vaccine Adverse
Effect Reporting System (VAERS).
Suggested Reading(s)
American Academy of Pediatrics. Diphtheria. In: Kimberlin DW, Barnett ED, Lynfield R, Sawyer MH, eds.
Red Book: 2021-2024 Report of the Committee on Infectious Diseases. 32nd ed. American Academy of
Pediatrics; 2021. Accessed September 1, 2023. Red Book Online
American Academy of Pediatrics. Pertussis (whooping cough). In: Kimberlin DW, Barnett ED, Lynfield R,
Sawyer MH, eds. Red Book: 2021-2024 Report of the Committee on Infectious Diseases. 32nd ed.
American Academy of Pediatrics; 2021. Accessed September 1, 2023. DATE. Red Book Online
American Academy of Pediatrics. Tetanus (lockjaw). In: Kimberlin DW, Barnett ED, Lynfield R, Sawyer
MH, eds. Red Book: 2021-2024 Report of the Committee on Infectious Diseases. 32nd ed. American
Academy of Pediatrics; 2021. Accessed September 1, 2023. Red Book Online
Vaccines and immunizations. Centers for Disease Control and Prevention. Accessed March 7, 2023.
www.cdc.gov/vaccines
Content Domain
Preventive Pediatrics
Abbreviatons, DTaP, diphtheria and tetanus toxoids with pertussis; Tdap, tetanus toxoid,
reduced diphtheria toxoid, and acellular pertussis, adsorbed; Td, tetanus and diphtheria
toxoids (adult type); TIG, tetanus immune globulin.
Such as, but not limited to, wounds contaminated with dirt, feces, soil, and saliva; puncture
*
wounds; avulsions; and wounds resulting from missiles, crushing, burns, and frostbite.
Note: DTaP is used for children <7 years of age. Tdap is preferred to Td for underimmunized children
7 years of age or older who have not received Tdap previously.
Adapted and reprinted with permission from Kimberlin DW, Barnett ED, Lynfield R, Sawyer MH, eds.
Red Book: 2021–2024 Report of the Committee on Infectious Diseases. 32nd ed. American Academy
of Pediatrics; 2021:753.
AAP PREP 2024 - Question 214/267 Preventative Pediatrics Question 23/27
A 17-year-old with no notable medical history is seen in the emergency department for evaluation of 3 days
of persistent nausea and vomiting. She was seen 1 month ago by her primary care practitioner and 1 week
ago in the emergency department for the same concerns. She has abdominal pain during the vomiting
episodes, but no diarrhea or fever. She has lost 3 kg in the past 3 months. In a confidential interview, the
adolescent reports no purging behaviors and states that she wants to gain back the weight she lost. There
have been no changes in her living situation. She plays softball on her school team and is a high-achieving
student, but she has frequently missed these activities owing to illness. She reports no depression, suicidal
thoughts, or self-harm history. Her parents comment that she has been spending prolonged amounts of time
in the shower.
The girl’s temperature is 36.9 °C, her blood pressure is 105/60 mm Hg, her heart rate is 92 beats/min, and
her body mass index is at the 30th percentile for age and sex. Her abdomen is soft and nontender to
palpation. Bowel sounds are hypoactive. The remainder of her examination findings are normal.
Of the following, the BEST next step in this adolescent’s evaluation is to confidentially ask about
C. sexual activity
D. sleep routine
Correct answer is A
PREP Pearl(s)
Screening for behavioral risk factors is a critical aspect of adolescent health care in all settings and
should be completed in a confidential manner. Initial questions should be general and open-ended,
and the adolescent’s strengths should be highlighted.
The SSHADESS (Strengths, School, Home, Activities, Drug use, Emotions, Sexuality, Safety) tool is a
strengths-based approach to the adolescent psychosocial history that can be used in place of the
HEADSS (Home, Education, Activities, Drug use, Sexuality, Suicide, Safety) tool.
Critique
The adolescent in the vignette, with persistent nausea, vomiting, and weight loss, has cannabis hyperemesis
syndrome (CHS) or cannabinoid hyperemesis syndrome. Chronic, heavy cannabis use is associated with CHS.
Also evident is the typical CHS history of persistent nausea and vomiting relieved by a hot shower or bath. To
confirm the diagnosis, the best next step in her evaluation is to confidentially ask about drug and alcohol use.
Cannabis hyperemesis syndrome results from overstimulation of the endocannabinoid system receptors
throughout the nervous and gastrointestinal systems. Recently, increasing levels of the psychoactive
component in marijuana, tetrahydrocannabinol, have been found to cause medical and psychiatric
consequences in youth who use cannabis on a daily or weekly basis.
The acute management of CHS includes the administration of antiemetics and intravenous fluid. Laboratory
evaluation and imaging should be performed as indicated according to clinical findings. Physicians should
recommend and assist with cessation of cannabis use, which includes referral for behavioral therapies for
substance use disorder. Affected patients should be assessed and treated for comorbid psychological
conditions.
Marijuana use among high school students has declined in recent years according to the 2021 Youth Risk
Behavior Survey, a behavioral health survey given in all 50 US states every 1 to 2 years. In 2021, 16% of
students reported past-month use of marijuana products, a decrease from 22% in 2019. Marijuana use
among female students did not change. More information on youth behavioral trends from the Youth Risk
Behavior Survey can be found on the Centers for Disease Control and Prevention website. Monitoring the
Future and the National Survey on Drug Use and Health are 2 other national surveys that collect data
regarding substance use and mental health from schools and homes, respectively.
Behavioral risk factors are key components of the history for adolescents in both outpatient and inpatient
settings. Physicians should obtain a psychosocial history with the adolescent alone, to ensure confidentiality
and establish trust. However, physicians must break confidentiality and share information that is essential to
the adolescent’s or another’s safety (eg, abuse, suicidal ideation, homicidal ideation). The HEADSSS (Home,
Education, Activity, Drug use, Sexuality, Suicide, Safety/violence risk) assessment has been used for many
years as a framework for the psychosocial interview. The SSHADESS screening tool (Table) takes an approach
focused on strengths, emotional well being, and risks; it begins with the adolescent’s strengths to build
rapport and addresses school early in the interview, thus providing a general, less personal indicator of
functioning.
The psychosocial interview should begin with open-ended questions that are general and impersonal in
nature, with progression to more personal questions as the adolescent becomes more comfortable.
Physicians should be aware of their own implicit biases and avoid any expression of shock or judgment at the
responses. The goal should never be disclosure of a specific behavior, but rather establishing a relationship
and highlighting the teen’s strengths. More specific screening tools can be used as appropriate to youth-
reported behavioral concerns. For example, for an adolescent who reports cannabis use, the CRAFFT (Car,
Relax, Alone, Friends/Family, Forget, Trouble) questionnaire (Figure) and/or the Screening to Brief
Intervention (S2BI) screening tool could be used to assess for substance use disorder and, if necessary, to
help determine an appropriate intervention.
Suggested Reading(s)
Breuner CC, Alderman EA, Jewell JA; Committee on Adolescence, Committee on Hospital Care. The
hospitalized adolescent. Pediatrics. 2023;151(2):e2022060646. doi:10.1542/peds.2022-060646
Centers for Disease Control and Prevention. Youth Risk Behavior Survey Data Summary & Trends
Report: 2011-2021. Centers for Disease Control and Prevention. Accessed February 21, 2023. Youth
Risk Behavior Survey Data Summary & Trends Report: 2011-2021 (cdc.gov)
Dosani K, Koletic C, Alhosh R. Cannabinoid hyperemesis syndrome in pediatrics: an emerging problem.
Pediatr Rev. 2021;42(9):500–506. doi:10.1542/pir.2019-0097
Ginsburg KR. The SSHADESS screening: a strength-based psychosocial assessment. In: Ginsburg KR, ed.
Reaching Teens: Strength-Based, Trauma-Sensitive Resilience-Building Communication Strategies
Rooted in Positive Youth Development. 2nd ed. American Academy of Pediatrics; 2014:chap 32.
American Academy of Pediatrics Online
Content Domain
Preventive pediatrics
Begin: “I’m going to ask you a few questions that I ask all my patients. Please be honest. I Topic Example Questions
will keep your answers confidential.”
Part A Strengths What do you enjoy? How would you describe yourself
During the PAST 12 MONTHS, on how many days did you:
1. Drink more than a few sips of beer, wine, or any drink containing School What do you like/not like about school? How are your grades? Is
alcohol? Say “0” if none. # of days this any change from last year? What do you want to do when you’re
2. Use any marijuana (cannabis, weed, oil, wax, or hash by smoking,
vaping, dabbing, or in edibles) or “synthetic marijuana” (like “K2,”
older?
“Spice”)? Say “0” if none. # of days
3. Use anything else to get high (like other illegal drugs, pills, Home Who do you live with? Who in your family can you go to if you were
prescription or over-the-counter medications, and things that you
sniff, huff, vape, or inject)? Say “0” if none. # of days
stressed?
Did the patient answer “0” for all questions in Part A? Activities What do you do for fun? What do you do with your friends?
Yes No Drug Use Do your friends ever talk about drugs or alcohol? How many times
have you vaped nicotine in the past year? If you’ve ever used drugs,
Ask 1st question only in Part B, how did it make you feel?
Ask all 6 questions in Part B
then STOP
Part B Circle one Emotions/ Do you feel stressed? Have you had any trouble sleeping? Have you
C Have you ever ridden in a CAR driven by someone (including yourself) who
No Yes depression, noticed a change in your body recently? Do you ever have thoughts of
was “high” or had been using alcohol or drugs?
Do you ever use alcohol or drugs to RELAX, feel better about yourself, or fit
eating hurting yourself or someone else? Have you ever hurt yourself?
R in?
No Yes
Sexuality Are you attracted to anyone (boys, girls, or both)? Have you ever had
A Do you ever use alcohol or drugs while you are by yourself, or ALONE? No Yes
any type of sex? Have you ever been worried about pregnancy or a
F Do you ever FORGET things you did while using alcohol or drugs? No Yes sexually transmitted infection?
Do your FAMILY or FRIENDS ever tell you that you should cut down on your
F drinking or drug use?
No Yes Safety Do you feel safe at home? Is there any bullying at your school? Has
anyone ever touched you physically or sexually when you didn’t want
T Have you ever gotten into TROUBLE while you were using alcohol or drugs? No Yes
them to? Do you ever get into fights?
*Two or more YES answers in Part B suggests a serious problem that needs
further assessment. See back for further instructions
Adapted by R. Savage from Ginsburg KR. The SSHADESS screening: a strength-based
NOTICE TO CLINIC STAFF AND MEDICAL RECORDS:
The information on this page is protected by special federal confidentiality rules (42 CFR Part 2), which prohibit disclosure of this information unless
authorized by specific written consent. psychosocial assessment. In: Ginsburg KR, ed. Reaching Teens: Strength-Based,
Trauma-Sensitive Resilience-Building Communication Strategies Rooted in Positive Youth
© John R. Knight, MD, Boston Children’s Hospital, 2020.
Development. 2nd ed. American Academy of Pediatrics; 2014:225A-228A.
Reproduced with permission from the Center for Adolescent Behavioral Health Research
(CABHRe), Boston Children’s Hospital. [email protected] www.crafft.org
For more information and versions in other languages, see www.crafft.org.
FIGURE: CRAFFT 2.1 screening tool: the CRAFFT interview.
AAP PREP 2024 - Question 215/267 Preventative Pediatrics Question 24/27
An adolescent boy is seen for a routine health supervision visit. His only medical diagnosis is mild
intermittent asthma. Neither he nor his mother has specific concerns. His mother describes him as “moody”
and states that “he just needs his own space sometimes.” During a private interview, the boy states that his
peers are taller than he is, and a friend suggested that protein drinks could help. His romantic interest is in
girls only. He has never had sex and is not currently interested in a relationship. He spends most of his time
with friends playing video games and basketball. He reports no depression or suicidal thoughts. He has a
friend who has a vape device, but he has never vaped nicotine or used any other drugs or alcohol. When
praised for this choice, he says, “I would never do drugs. It’s just dumb.”
A. 11 years
B. 14 years
C. 16 years
D. 18 years
Correct answer is B
PREP Pearl(s)
Adolescence is a complex period of transition from childhood to adulthood that includes physical,
cognitive, emotional, and social development.
Adolescent physical development does not always correlate with the levels of cognitive, social, and
emotional development.
Critique
Of the response choices, the age of the adolescent in the vignette is most likely 14 years. Adolescence is a
period of physical, cognitive, social, and emotional transition and can be divided into 3 periods: early, 10 to 13
years; middle, 14 to 17 years; and late, 18 to 25 years. Brain development continues until approximately age
24 years. Consistent with what is typically seen at the beginning of middle adolescence, the adolescent in the
vignette displays peer comparison (concern about his height), early interest in romantic relationships, desire
for more peer and less family interaction, and concrete thinking (eg, “drugs are dumb”).
Younger adolescents (aged 10-11 years) also display concrete thinking (eg, certain things are always either
good or bad) but are just beginning pubertal changes and may not be as interested in increasing peer
connections and separating from their parents yet. A 16-year-old would likely show more interest in romantic
relationships and have less concrete thinking. An 18-year-old (late adolescence) would typically have a more
mature sense of self and their values and show more abstract reasoning.
Physical development or puberty is the change from a child to an adult body. Physical developmental
milestones occur in a typical sequence, but the timing varies between individuals. Physical development does
not always correlate with the levels of cognitive and emotional development. Cognitive development occurs
when neuronal growth and pruning prepare the brain for mature thinking and learning. The last part of the
brain to develop fully is the prefrontal cortex, the part of the brain responsible for complex thought
formation, executive function, and reasoning skills. Brain development can be altered during adolescence by
substance use, trauma, learning and developmental disorders, mental health disorders, and other stressors.
Emotional development involves the ability to identify and manage emotions. Changing hormonal levels and
activation of various parts of the brain, including the frontal cortex, contribute to emotional development.
Adolescents develop the ability to regulate their emotions as they gain experience in identifying and
managing their thoughts and feelings in various contexts and environments. Identity formation and coping
with stressors are other vital components of emotional development. Practitioners and parents can support
adolescents’ emotional development by using positive communication skills such as open-ended questions,
active listening, inquiring about feelings, and teaching positive coping strategies.
Social development occurs as adolescents begin to empathize with others and perform new roles and
responsibilities. Peer relationships are vital to adolescents’ identity formation, cooperation with others, and
development of conflict resolution skills. Social media and online peer interactions can be useful for social
skill development but are also potentially harmful without boundaries and parental monitoring. Pediatric
practitioners should address bullying, because this is a common form of harassment among adolescents
both in person and online. Adolescence is also important as a time for the formation of one’s own moral code
and, for many, the development of spiritual and faith-based practices and beliefs.
Suggested Reading(s)
Adolescent development explained. US Department of Health and Human Services, Office of
Population Affairs. Accessed February 19, 2023. opa.hhs.gov/adolescent-health/adolescent-
development-explained
Hagan JF, Shaw JS, Duncan PM. Early adolescence: 11 through 14 year visits. In: Hagan JF, Shaw JS,
Duncan PM, eds. Bright Futures: Guidelines for Health Supervision of Infants, Children, and
Adolescents. 4th ed. American Academy of Pediatrics; 2017.
Hagan JF, Shaw JS, Duncan PM. Late adolescence: 18 through 21 year visits. In: Hagan JF, Shaw JS,
Duncan PM, eds. Bright Futures: Guidelines for Health Supervision of Infants, Children, and
Adolescents. 4th ed. American Academy of Pediatrics; 2017.
Hagan JF, Shaw JS, Duncan PM. Middle adolescence: 15 through 17 year visits. In: Hagan JF, Shaw JS,
Duncan PM, eds. Bright Futures: Guidelines for Health Supervision of Infants, Children, and
Adolescents. 4th ed. American Academy of Pediatrics; 2017.
Content Domain
Preventive Pediatrics
A 9-month-old infant is seen in the office for a health supervision visit. He has been healthy and has normal
growth and development. He recently had eruption of the 2 lower central incisors and has no other
detention.
Of the following, the MOST appropriate counseling is that his parent should start
C. non-fluoridated toothpaste, and administer fluoride drops if the water in the community is not
fluoridated
D. non-fluoridated toothpaste until 1 year of age, and then start a fluoridated toothpaste
Correct answer is A
PREP Pearl(s)
Once the first tooth erupts, parents should start brushing their child’s teeth with a smear of fluoridated
toothpaste twice daily.
Children should begin to see a dentist once teeth erupt and no later than 1 year of age.
Children who live in communities where the water is not fluoridated should receive fluoride
supplementation starting at 6 months of age.
Critique
The American Academy of Pediatric Dentistry (AAPD) recommends that parents start brushing their children's
teeth twice a day with a fluoridated toothpaste as soon as the first tooth erupts. The amount of toothpaste
recommended for children under age 3 years is a “smear” or the size of a grain of rice (providing
approximately 0.1 mg of fluoride). Children aged 3 years and older should use a pea-sized amount of
toothpaste, (approximately 0.25 mg of fluoride). These recommended amounts of toothpaste help to limit
the amount of toothpaste swallowed by children, thereby preventing dental fluorosis. Children should have
their first dental visit as soon as teeth erupt or no later than their first birthday.
Educating parents regarding age-appropriate dental hygiene is an important aspect of anticipatory guidance
that should be included in routine health supervision visits. Dental caries is one of the most common chronic
infectious diseases of childhood, affecting 24% of children between the ages of 2 and 4 years; 50% of older
children experience caries. The incidence is higher among children of lower socioeconomic backgrounds and
children with special health care needs, particularly those with developmental delays or neurodevelopmental
disorders (eg, autism).
Bacteria, sugar, saliva, and fluoride affect the balance of the tooth mineralization and demineralization
processes. These variables can be controlled by diet, hygiene practices, and local governmental decisions
regarding fluoridation of water supplies. Fermentation of sugars by oral bacteria decreases the pH of the oral
environment and creates a milieu that allows bacteria to flourish. Saliva plays a role in buffering the pH,
making the oral environment less acidic. Fluoride protects teeth by inhibiting tooth demineralization,
enhancing remineralization of the enamel surface, and inhibiting bacterial enzymes. Water fluoridation is a
highly effective community-level intervention that reduces dental decay at the population level and is
recommended by the federal government. However, policies regarding drinking water fluoridation are made
at the state and local government level and vary across the country. One can determine if their county adds
fluoride to the water using the Centers for Disease Control and Prevention’s ‘My Water’s Fluoride’ website:
https://nccd.cdc.gov/DOH_MWF/Default/Default.aspx.
For children 6 months of age and older whose primary water source has <0.3 ppm of fluoride, the AAPD
recommends that pediatricians prescribe fluoride drops in addition to a parent/caregiver brushing their teeth
with a fluoridated toothpaste.
Fluoride varnish is a topical form of concentrated fluoride that can be applied to teeth and sets upon contact
with saliva. The current AAPD recommendation is to apply fluoride varnish every 3 to 6 months for children at
risk for poor access to dental care. One advantage of fluoride varnish is that it can be applied by dental or
non-dental professionals, including pediatricians. As young children have many more encounters with their
pediatrician than with a dentist, pediatricians can play a crucial role in dental caries prevention by applying
dental varnish during health supervision visits.
Suggested Reading(s)
Centers for Disease Control and Prevention. Children’s Dental Health. Accessed September 1, 2023.
https://www.cdc.gov/oralhealth/publications/features/childrens-dental-health.html
Clark MB, Keels MA, Slayton RL; Section on Oral Health. Fluoride use in caries prevention in the primary
care setting. Pediatrics. 2020;146(6):e2020034637. doi:10.1542/peds.2020-034637
Hagan JF, Shaw JS, Duncan PM, eds. Promoting Oral Health. Bright Futures: Guidelines for Health
Supervision of Infants, Children, and Adolescents, 4th ed. American Academy of Pediatrics; 2017:205-
217.
Slayton RL. Oral health. In: McInerny TK, Adam HM, Campbell DE, DeWitt TG, Foy JM, Kamat DM, eds.
American Academy of Pediatrics Textbook of Pediatric Care. American Academy of Pediatrics;
2021:chap 40. Accessed September 1, 2023. Pediatric Care Online
Content Domain
Preventive pediatrics
A previously healthy 15-month-old is seen for a health supervision visit after returning from international
travel. She received a dose of measles-mumps-rubella (MMR) vaccine at 11 months of age before traveling to
a measles-endemic region. Her family asks how this vaccination will affect the remainder of her MMR vaccine
schedule.
A yes yes
B no no
C no yes
D yes no
A. Response Choice A
B. Response Choice B
C. Response Choice C
D. Response Choice D
Correct answer is A
PREP Pearl(s)
The measles-mumps-rubella (MMR) vaccine should be administered to children between ages 12 and
15 months and ages 4 and 6 years.
People who have immunosuppression (eg,cancer, HIV/acquired immunodeficiency syndrome) or are
receiving medications that weaken the immune system should not receive the measles-mumps-rubella
(MMR) vaccine, as it is a live attenuated vaccine.
Pregnant women should not receive the measles-mumps-rubella (MMR) vaccine; women planning to
become pregnant should wait to conceive until at least 1 month after receiving the MMR vaccine. It is
safe for breastfeeding mothers to receive the MMR vaccine.
Critique
The Centers for Disease Control and Prevention recommends 2 doses of measles-mumps-rubella (MMR)
vaccine: the first administered between 12 and 15 months of age and the second between 4 and 6 years of
age. If a child between the ages of 6 and 12 months will be traveling internationally to a measles-endemic
region, they should receive their first dose of the MMR vaccine before they travel. If the first MMR dose is
given before 12 months of age, the child should still complete the recommended 2-dose series starting at 12
months of age. The interval between doses of the MMR vaccine or other live vaccinations should be at least
28 days.
The MMR vaccine is a live attenuated vaccine. Adverse effects of the vaccine may include mild symptoms (eg,
fever, myalgias) as well as injection site irritation. Some vaccinated individuals may become infected when
exposed to the measles virus but are less likely to have severe illness and are likely to be less contagious.
Administration of an additional dose of MMR vaccine within 3 days of exposure may be considered for
children in close contact with an individual infected with measles (eg, shared home, shared athletic
equipment, close physical contact).
Adults who have never been vaccinated with the MMR vaccine should receive at least 1 dose. Those who live
in close quarters (eg, dormitory, military barracks), who work in health care, or who are about to travel
internationally should receive 2 doses of MMR at least 28 days apart.
The MMR vaccine’s efficacy varies for each included virus, but 2 doses have excellent results (Table).
It is safe for breastfeeding mothers to receive the MMR vaccine. It is generally recommended that the MMR
vaccine be administered postpartum to unvaccinated mothers.
Suggested Reading(s)
Measles, mumps, and rubella (MMR) vaccination: what everyone should know. Centers for Disease
Control and Prevention. Accessed December 2, 2022.
https://www.cdc.gov/vaccines/vpd/mmr/public/index.html
Patient care: immunizations. American Academy of Pediatrics. Accessed December 2, 2022.
https://www.aap.org/en/patient-care/immunizations/
Patient education 2021: VIS—MMR (measles, mumps, and rubella) vaccine. American Academy of
Pediatrics. Accessed December 2, 2022. VIS—MMR (Measles, Mumps, and Rubella) Vaccine | Pediatric
Patient Education | American Academy of Pediatrics (aap.org)
American Academy of Pediatrics. Vaccine ingredients. In: Kimberlin DW, Barnett ED, Lynfield R, Sawyer
MH, eds. Red Book: 2021-2024 Report of the Committee on Infectious Diseases. 32nd ed. American
Academy of Pediatrics; 2021. Accessed September 1, 2023. Red Book Online
Content Domain
Preventive Pediatrics
Courtesy of J. Reed
AAP PREP 2024 - Question 218/267 Preventative Pediatrics Question 27/27
A 25-month-old refugee child from Afghanistan is seen for a new patient health supervision visit. Review of
her immunization record shows that she has not been vaccinated against Haemophilus influenzae type b.
Of the following, the BEST plan for catch-up of this vaccination for this child is to
A. administer conjugated Haemophilus influenzae type b vaccine today; no further doses are
necessary
B. administer conjugated Haemophilus influenzae type b vaccine today, a second dose 4 weeks
from today, and a third and final dose 8 weeks from today
C. administer conjugated Haemophilus influenzae type b vaccine today and a second and final dose
8 weeks from today
Correct answer is A
PREP Pearl(s)
Haemophilus influenzae type b causes invasive infection, most commonly meningitis, epiglottitis,
pneumonia, septic arthritis, bacteremia, and facial cellulitis.
Children younger than 5 years are at greatest risk for invasive Haemophilus influenzae type b infection.
Catch-up immunization schedules should be reviewed when caring for any child who is not up-to-date
based on the standard Advisory Committee on Immunization Practice vaccination schedule.
Critique
The child in the vignette is younger than 5 years and still at risk for serious invasive infection from
Haemophilus influenzae type b. According to the Advisory Committee on Immunization Practice (ACIP) catch-
up schedule, children aged 15 to 59 months should receive 1 dose of conjugated H influenzae type b vaccine.
No additional doses are required based on risk of disease in this age group.
Haemophilus influenzae type b is a gram-negative encapsulated organism known to cause invasive infection
in nonimmune people, with children younger than 5 years at greatest risk. The most common infections
caused by H influenzae type b are meningitis, epiglottitis, pneumonia, septic arthritis, bacteremia, and facial
cellulitis. Children between the ages of 3 months and 5 years are at greatest risk for invasive H influenzae
type b infection because they lack the type b capsule-specific antibodies necessary for bactericidal activity.
Neonates have protective maternal antibodies; older children and adults have acquired protective
antibodies.
The first H influenzae type b vaccines were licensed for use in the United States for children 18 months and
younger in the 1980s. In 1991, the H influenzae type b vaccine became available for use in infants 2 months
or older and had a significant impact on the incidence of invasive disease (Figure 1). Before that,
approximately 1 in 200 children in the United States developed an H influenzae type b invasive infection
before their fifth birthday. The incidence of H influenzae type b meningitis was approximately 12,000 cases
per year, with a mortality of 3% to 6%. Sequelae of H influenzae type b meningitis include intellectual
disability, spasticity or paresis, seizures, hearing loss, and long-term educational and behavioral disability.
The World Health Organization advises that all routine childhood immunization programs include conjugated
H influenzae type b vaccine. In 2019, coverage in the United States was approximately 92%, and the rate of H
influenzae type b invasive disease has decreased to less than 0.3 per 100,000 children (Figure 2).
The vaccine’s impact on H influenzae type b–related invasive disease is seen as vaccine-induced immunity as
well as a reduction of H influenzae type b colonization in the oropharynx of vaccinated individuals, with both
effects impacting herd immunity. It is important to recognize that special populations remain at higher risk
for H influenzae type b–related invasive disease despite immunization, including children with HIV, Native
Alaskans, and Native Americans.
Knowledge of vaccine-preventable illness and the current recommended child and adolescent immunization
schedules is critical for the pediatrician. Although many children seen in the office have been immunized
according to the standard schedule, a substantial number are underimmunized because of gaps in care,
parental decisions, or previous medical care in another country with a different immunization schedule.
Physicians should refer to the immunization catch-up schedule recommended by the ACIP and approved by
the Centers for Disease Control and Prevention and the American Academy of Pediatrics for clinical practice
guidance.
Suggested Reading(s)
Centers for Disease Control and Prevention. Catch-up immunization schedule for children and
adolescents who start late or who are more than 1 month behind. Updated February 17, 2022.
Accessed September 1, 2023. https://www.cdc.gov/vaccines/schedules/hcp/imz/catchup.html
Gilsdorf J. Hib vaccines: their impact on Haemophilus influenzae type b disease. J Infect Dis.
2021;224(12 suppl 2):S321-S330. doi:10.1093/infdis/jiaa537
Kroger AT, Duchin J, Vázquez M. General best practice guidelines for immunization: best practices
guidance of the Advisory Committee on Immunization Practices (ACIP). April 20, 2017. Accessed
September 1, 2023. https://stacks.cdc.gov/view/cdc/46256
Content Domain
Preventive pediatrics
The correct answer is: administer conjugated Haemophilus influenzae type b vaccine today; no further doses
are necessary
View Peer Results
FIGURE 1: Impact of Haemophilus influenzae type B vaccines on kincidence of
disease in the US 1980-2012.
30
0.30
First polysaccharide Hib vaccine licensed for use in children aged ≥18 months
25 0.25
First conjugate Hib vaccine licensed for use in children aged ≥18 months
20 First Hib vaccines licensed for use in infants aged ≥2 months 0.20
Incidence
Incidence
0.15
15
0.10
10
Estimated annual incidence
0.05
Healthy people 2020 goal
5 (0.27/100 000 population)
0.00
2000 2001 2002 2003 2004 2005 2006 2007 2008 2009 2010 2011 2012
0
1980 1982 1984 1986 1988 1990 1992 1994 1996 1998 2000 2002 2004 2006 2008 2010 2012
Year
Year Reprinted with permission from Gilsdorf J. Hib vaccines: their impact on Haemophilus
Reprinted with permission from Gilsdorf J. Hib vaccines: their impact on Haemophilus influenzae type b disease. J Infect Dis. 2021;224(12 suppl 2):S325.
influenzae type b disease. J Infect Dis. 2021;224(12 suppl 2):S325.
FIGURE 2: Estimated annual incidence of Haemophilus influenzae type b
FIGURE 1: Impact of Haemophilus influenzae type B vaccines on incidence of infection in the US 2000-2012.
disease in the US 1980-2012.
AAP PREP 2024 - Question 219/267 Pulmonology Question 1/9
A 12-year-old is evaluated for a nonproductive cough. They have no systemic symptoms other than fatigue,
however, there is a history of recurrent respiratory illnesses over the past 2 years. During past episodes,
physical examination demonstrated variable pulmonary crackles that resolved after oral antibiotic treatment.
Chest radiographs demonstrated variable patchy densities suggestive of pneumonia that cleared within a
week. There is no history of other recurrent infections.
Today, their oxygen saturation is 95% in room air. The patient appears pale with conjunctival pallor and mild
nasal congestion. There is no cervical or supraclavicular adenopathy and their oropharynx is clear. Breath
sounds are coarse with scattered crackles but no wheeze. Cardiovascular examination findings are normal.
There is no digital clubbing. The remainder of the physical examination findings are normal.
Laboratory data are shown:
Hematocrit 31%
This patient is referred for a pulmonology evaluation. Bronchoscopy with bronchoalveolar lavage
demonstrates alveolar macrophages with a positive stain for iron.
Correct answer is C
PREP Pearl(s)
Idiopathic pulmonary hemosiderosis presents with repeated transient pulmonary illnesses and
radiographic changes, in the context of anemia. It is treated with anti-inflammatory agents.
Collagen vascular diseases with capillaritis may cause diffuse alveolar hemorrhage.
Immunoglobulin G–mediated milk intolerance may cause pulmonary hemosiderosis in infants and
young children and is managed with milk protein avoidance.
Critique
This patient has pulmonary hemosiderosis caused by repeated subclinical pulmonary hemorrhage.
Bronchoscopy findings demonstrate hemosiderin laden alveolar macrophages, which are characteristic of
pulmonary hemorrhage (usually repeated events that may or may not be clinically obvious). There is a
protracted history of repeated respiratory symptoms that have been labeled as recurrent infection. However,
the history of anemia, transient radiographic changes, and pulmonary examination findings without other
systemic symptoms suggest previously unrecognized episodes of pulmonary hemorrhage.
Immunologic reaction to avian or other inhaled proteins results in hypersensitivity pneumonitis. This
condition may present with transient symptoms with each exposure, but it is most often associated with
chronic cough, wheezing, and shortness of breath. Anemia is not usually a part of this complex. Findings on
diagnostic testing include radiographic changes consistent with a diffuse interstitial process and a restrictive
pattern on pulmonary function testing with a history of potential protein antigen exposure (eg, bird
droppings or fungal antigens). The diagnosis is often made after multiple episodes of illness. Ongoing
exposure to the offending antigen leads to worsening symptoms that may include hypoxemia, digital
clubbing, and a honeycomb pattern with bronchiectasis may be seen on computed tomography of the chest.
Traumatic injury to the lung can produce intrapulmonary bleeding. A single episode of traumatic pulmonary
hemorrhage can produce hemosiderin laden alveolar macrophages. The patient in the vignette has no
history of chest wall or pulmonary trauma, making pulmonary parenchymal trauma an unlikely cause of their
findings.
Repeated aspiration of stomach contents is associated with lipid-laden, rather than hemosiderin-laden,
alveolar macrophages. Aspiration due to swallowing dysfunction from neurological or anatomical causes or
gastroesophageal reflux promotes transfer of foods containing fat into the airway. The phagocytosis of fat
into macrophages can be identified by an oil-red-O stain. Pulmonary lipid laden macrophages can also
indicate marrow embolism from a large bone fracture or bone infarct.
Pulmonary hemorrhage can result from capillaritis due to collagen vascular disease (eg, granulomatosis with
polyangiitis, Goodpasture syndrome, systemic lupus erythematosus). Capillaritis may manifest as repeated
episodes of cough and shortness of breath, with or without frank hemoptysis.
Infants and young children with milk protein intolerance mediated by IgG4 (Heiner syndrome) may have
pulmonary alveolar bleeding with pulmonary hemosiderosis. Findings may be subtle in this population;
anemia, repeated low-grade pulmonary illnesses, and radiographic changes similar to those of the patient
described in the vignette are characteristic. Serum precipitins to milk proteins may be present but are not
pathognomonic. Heiner syndrome is often a diagnosis of exclusion. Treatment consists of avoidance of milk
proteins. The condition resolves after a period of avoidance. Affected children typically develop tolerance of
milk within a few years.
Inhaled fungal toxins and tobacco smoke exposure have been associated with diffuse pulmonary
hemorrhage in infants. In the 1980s and 1990s, there was an outbreak of pulmonary hemorrhage in infants
living in housing subject to repeated flooding and water damage along the Mississippi River basin that was
eventually linked to exposure to the fungus Stachybotrys chartarum.
Idiopathic pulmonary hemosiderosis is rare but more common in children than in adults. This patient has no
known predisposing conditions and is older than the children usually diagnosed with Heiner syndrome or
other precipitating exposures seen in infancy. The diagnosis of pulmonary hemosiderosis typically requires a
high index of suspicion in the context of repeated unexplained pulmonary symptoms with anemia, and an
evaluation to rule out other causes of diffuse pulmonary hemorrhage. Consultation with a specialist may be
needed. Treatment involves aggressive use of anti-inflammatory drugs. In the past the prognosis was poor,
but recently the 5-year survival rate has been greater than 80%.
Suggested Reading(s)
Reisman S, Chung M, Bernheim A. A review of clinical and imaging features of diffuse pulmonary
hemorrhage. AJR Am J Roentgenol. 2021;216(6):1500-1509. doi:10.2214/AJR.20.23399
Saha B. Idiopathic pulmonary hemosiderosis: a state of the art review. Respir Med. 2021;176:106234.
doi:10.1016/j.rmed.2020.106234
Schroeder SA. Hemoptysis. In: McInerny TK, Adam HM, Campbell DE, DeWitt TG, Foy JM, Kamat DM,
eds. American Academy of Pediatrics Textbook of Pediatric Care. American Academy of Pediatrics;
2023. Accessed September 1, 2023. Pediatric Care Online
Voter K, Ren C. Pulmonary hemorrhage, idiopathic pulmonary hemorrhage and hemosiderosis. In: MJ
Light, ed. Pediatric Pulmonology. American Academy of Pediatrics; 2011:596-597.
Content Domain
Pulmonology
A 10-year-old boy is brought to the office after coughing up blood this morning. He reports no vomiting or
epistaxis. He has had cough, fatigue, malaise, and tactile fever for several weeks. His family emigrated from
Somalia 1 year ago. No one else in the household has been ill recently, but his grandfather, who lives nearby,
has had a chronic cough for many months.
The boy’s temperature is 37.3 °C orally, his blood pressure is 105/65 mm Hg, his heart rate is 100 beats/min,
his respiratory rate is 30 breaths/min, and his oxygen saturation is 97% in room air. His height and weight are
at the 25th percentile. The boy appears tired but not acutely ill. He is breathing comfortably. There is no
cervical tenderness or adenopathy. His nose and pharynx are clear. On lung examination, there is adequate
air exchange with coarse breath sounds throughout, coarse crackles bilaterally in the mid-lung fields, and no
expiratory wheezing. The remainder of the boy’s physical examination findings are normal. Chest
radiography is obtained, and results are pending.
Correct answer is B
PREP Pearl(s)
Infection is the most common cause of hemoptysis in children who do not have underlying severe
pulmonary or cardiac disease.
Hemoptysis may be a presenting symptom of autoimmune diseases with pulmonary capillaritis.
Children with hemoptysis associated with a tracheostomy have a high risk of recurrence, but low
mortality risk. Children with hemoptysis associated with underlying severe heart or pulmonary disease
have a low recurrence risk but a higher mortality risk.
Critique
The boy in the vignette has hemoptysis with symptoms suggestive of systemic illness (fatigue, malaise, tactile
fever) and abnormal pulmonary findings (coarse breath sounds, crackles). The best next step in his care is to
obtain sputum evaluation with acid-fast stain and cultures.
The most common cause of hemoptysis in children without severe underlying cardiac or pulmonary disease
is infection. Based on the history and his physical examination findings, tuberculosis is the most likely cause
of this boy’s hemoptysis. He immigrated from a country where tuberculosis is endemic and has contact with a
family member with chronic cough. Other unusual infections must also be considered, as well as the more
common bacterial causes of pneumonia. Although the chest radiography results should be available soon,
sputum should be obtained to evaluate for a specific infectious etiology without delay. In addition to Gram
stain and bacterial culture, a search for acid-fast (Mycobacterium) and fungal organisms is appropriate.
Neither magnetic resonance imaging of the neck and chest nor computed tomography with contrast would
be part of the initial diagnostic evaluation. These imaging studies might be indicated if there were significant
concern regarding a vascular lesion or mass as a source of bleeding. This boy’s history and physical
examination findings are most suggestive of infection; therefore, these studies are not the best next
management step.
The boy’s condition is stable, and he has no history or physical examination findings suggestive of an upper
airway source of bleeding. Therefore, an otolaryngology consultation is not the best next management step.
Although hemoptysis may occur as an isolated or transient event, the physician should undertake a careful
assessment of the child’s hemodynamic status and stability, as well as evaluation to determine the source
and clinical implications of the bleeding. For a stable child with no signs of impending cardiovascular
compromise and with a clearly benign upper airway source of bleeding, reassurance, discussion of preventive
strategies, and follow-up planning is appropriate. Otherwise, further evaluation is indicated to determine the
etiology and direct the management of the bleeding.
Most airway bleeding in children is due to mucosal irritation. The upper airway (nose, sinuses, pharynx) or
lower airway (trachea, bronchi or bronchioles) mucosa may be involved. Bleeding can result from any of the
following:
Vomiting due to esophageal or stomach bleeding may be mistaken for hemoptysis (expectoration). Infants
and young children may swallow and later vomit upper-airway blood. Table 1 outlines the features
differentiating hemoptysis from a gastric or upper airway blood source.
Certain underlying conditions (eg, presence of a tracheostomy tube, bronchiectasis, congenital heart disease
with pulmonary hypertension or collateral vessels) predispose children to airway inflammation and bleeding.
The hemoptysis may be an isolated occurrence or recurrent. The associated mortality risk depends on the
underlying condition. Hemoptysis associated with tracheostomy infection and local irritation is commonly
recurrent but rarely fatal (an exception is erosion into a major vessel). In contrast, hemoptysis associated
with severe underlying pulmonary or cardiac disease has a higher mortality risk but lower recurrence rate.
The evaluation of a child with hemoptysis associated with an underlying condition is outlined in Table 2.
Systemic conditions associated with hemoptysis include bronchiectasis (cystic fibrosis-related and non–cystic
fibrosis-related), collagen vascular diseases (with sinus and pulmonary foci), arteriovenous anomalies of the
chest or airway, and congenital cardiopulmonary disorders. These conditions may not have been diagnosed
before the episode of hemoptysis, particularly in the case of autoimmune processes with capillaritis as the
source of the bleeding. Evaluation of hemoptysis in a child without a known preexisting condition is outlined
in Table 3.
The initial evaluation and management of hemoptysis should focus on assessment of the child’s
hemodynamic status and potential for deterioration. If the child’s condition is stable and not at risk of
experiencing sudden decompensation, and there are no known underlying conditions, then evaluation
should proceed as in Table 3. Further management depends on the specific cause of bleeding.
Treatment of superficial airway irritation is directed at the identified bleeding site. Children with
tracheostomies require meticulous home care routines to protect the integrity of the site and prevent
superficial infections. Routine evaluation by an otorhinolaryngologist is indicated for management of the
tracheostomy site and any granulomatous tissue within the airway, a potential source of bleeding. Deeper
airway infections, with or without an associated underlying condition, require a more prolonged and
aggressive antibiotic course targeted to the specific infection and underlying process. Bronchoscopy is
indicated to determine if bleeding is from a single site or is diffuse. Management of hemoptysis associated
with cardiovascular disease will require involvement of the pediatric cardiologist to provide direction
regarding the state of specific cardiovascular diagnosis and potentially to address pulmonary arterial
hypertension or presence of collateral circulation.
Suggested Reading(s)
Chiel L, Walsh S, Andren K, et al. Pediatric hemoptysis without bronchiectasis or cardiac disease:
etiology, recurrence, and mortality. J Ped. 2019;214:66-70. doi:10.1016/j.jpeds.2019.07.049
Schroeder SA. Hemoptysis. In: McInerny TK, Adam HM, Campbell DE, DeWitt TG, Foy JM, Kamat DM,
eds. American Academy of Pediatrics Textbook of Pediatric Care. American Academy of Pediatrics;
2023. Accessed September 1, 2023. Pediatric Care Online
Shnayder R, Needleman JP. Hemoptysis. Pediatr Rev. 2018;39(6):319-321. doi:10.1542/pir.2017-0157
Content Domain
Pulmonology
Computed tomography scan (sinus) Mucosal thickening Autoimmune process (Wegener granulomatosis)
Urinalysis Blood or protein Autoimmune process
Complete blood count Anemia Severe or chronic process (autoimmune or idiopathic)
Biopsy of airway lesion Pathologic diagnosis (care should be taken to be sure that bleeding can be
controlled)
A 17-year-old adolescent is brought to the emergency department after experiencing sudden onset of left
upper chest pain 2 hours ago. The pain has improved and is now occurring on deep inspiration. He has no
history of trauma or unusual exertion. There is no personal or family history of pulmonary or connective-
tissue disease.
On physical examination, the patient is tall and thin. He appears uncomfortable but is not in distress. His vital
signs are a blood pressure of 110/65 mm Hg, a heart rate of 100 beats/min, a respiratory rate of 20
breaths/min, and an oxygen saturation (via pulse oximetry) of 97% in room air.
There are decreased breath sounds over the left upper lung fields and axilla, with otherwise good air
exchange, and no adventitious sounds. Cardiac examination findings are normal. Skin turgor is good, with
brisk capillary refill. The remainder of his physical examination findings are unremarkable.
After treatment for pain, he appears more comfortable. His vital signs are now a blood pressure of 112/60
mm Hg, a heart rate of 80 beats/min, a respiratory rate of 16 breaths/min, and an oxygen saturation (via
pulse oximetry) of 97% in room air.
Chest radiographic findings are shown in Figure 1.
Reprinted with permission from Richard Wiggins MD and Zebrack CM, Bratton SL. In: McInerny TK, Adam
HM, Campbell DE, Foy JM, Kamat DM, eds. American Academy of Pediatrics Textbook of Pediatric Care. 2nd
ed. American Academy of Pediatrics; 2016:chap 368.
Figure 1. Chest radiograph for the boy described in the vignette. Arrows demarcate the pleural edge.
D. surgical pleurodesis
Correct answer is C
PREP Pearl(s)
Small to moderate uncomplicated spontaneous pneumothorax can be managed with close outpatient
observation in children and adolescents who are clinically stable. Resolution may take up to 8 weeks.
Tension pneumothorax is a medical emergency; air must be evacuated immediately with a needle or
chest tube.
Except in the context of esophageal rupture, postsurgical status, or trauma, the management of
pneumomediastinum is supportive.
Critique
The patient described in the vignette has a moderate spontaneous pneumothorax in the context of
cardiopulmonary stability and no respiratory distress. The best next step in his treatment is outpatient
observation with close follow-up.
Pediatric data regarding the optimal management of spontaneous pneumothorax are lacking. Historically,
the management of moderate pneumothorax involved hospital admission and administration of 100%
oxygen to hasten resorption of air and reexpansion of the lung. Placement of a small-bore chest tube used to
evacuate the air and then clamped, with reevaluation for reaccumulation after several hours, was also
considered appropriate management.
A 2015 retrospective review showed that resolution of pneumothorax after small-bore catheter evacuation of
air was successful in only about one-half of pediatric cases and was most successful for small air leaks. This
review led to a 2020 randomized study of intervention (eg, evacuation, chest tube placement) versus no
intervention for stable moderate pneumothorax in adolescents and young adults; the findings demonstrated
no difference in resolution by 8 weeks and a lower incidence of recurrence in those who had no intervention.
These findings, although not formally codified into practice guidelines, support nonintervention in stable
patients. Outpatient observation and close follow-up is appropriate management for this patient as long as
he remains hemodynamically stable and in no respiratory distress. Resolution of pneumothorax, with or
without intervention, often takes 8 weeks or more.
Unless there is a tension pneumothorax or hemodynamic instability, immediate use of suction with chest
tube placement is not the standard of care. Tension pneumothorax, with progressive increase in the size of
the air leak and unstable vital signs, is a medical emergency and requires immediate attempt to evacuate the
air with a needle or chest tube.
Pleurodesis is not standard after a first episode of uncomplicated pneumothorax. When pneumothorax is
recurrent, chest computed tomographic evaluation and elective pleurodesis should be considered. However,
surgical pleurodesis may not prevent recurrence.
Beyond the newborn period, acute aspiration of a pneumothorax should be performed with a small-bore
chest tube (eg, pigtail catheter), not with a straight needle. The catheter is then clamped, and several hours
later a decision is made regarding removal (if the pneumothorax has not reaccumulated) or placement to
water seal (if there has been or is concern about recurrence).
Spontaneous pneumothorax occurs in about 3 per 100,000 children and adolescents, most often in those
with a tall, thin body habitus. Adolescent boys are 2 to 4 times more likely than are girls to have spontaneous
pneumothorax, with the highest occurrence at about age 15 years. Smoking and the use of illicit drugs
increases the risk significantly.
Symptoms of spontaneous pneumothorax include chest pain, cough, dyspnea, and irritability. Physical
examination findings include decreased air movement on the affected side, tachycardia, tachypnea, and
sometimes hypoxia. Small apical pleural blebs are often identified on computed tomographic scan in the
setting of primary spontaneous pneumothorax (Figure 2, Figure 3, and Figure 4). Bilateral blebs may be seen
when the pneumothorax is unilateral.
Courtesy of M. Guill
Figure 2. Transverse computed tomographic view of right upper lobe apical blebs.
Courtesy of M. Guill
Figure 3. Coronal computed tomographic view of right upper lobe apical blebs.
Courtesy of M. Guill
Figure 4. Sagittal computed tomographic view of right upper lobe apical blebs.
Suggested Reading(s)
Benbow MK, Nanagas MT. Pneumothorax beyond the newborn period. Pediatr Rev. 2014;35(8):356-
357. doi:10.1542/pir.35-8-356
Brown SGA, Ball EL, Perrin K, et al; PSP Investigators. Conservative versus interventional treatment for
spontaneous pneumothorax. N Engl J Med. 2020;382(5):405-415. doi:10.1056/NEJMoa1910775
Cashen K, Petersen TL. Pleural effusions and pneumothoraces. Pediatr Rev. 2017;38(4):170-181.
doi:10.1542/pir.2016-0088
Lieu N, Ngo P, Chennapragada SM, et al. Update in management of pediatric primary spontaneous
pneumothorax. Paediatr Respir Rev. 2022;41:73-79. doi:10.1016/j.prrv.2021.08.001
Robinson PD, Blackburn C, Babl FE, et al; Paediatric Emergency Departments International
Collaborative (PREDICT) research network. Management of paediatric spontaneous pneumothorax: a
multicentre retrospective case series. Arch Dis Child. 2015;100(10):918-923. doi:10.1136/archdischild-
2014-306696
Zebrack CM, Bratton SL. Pneumothorax and pneumomediastinum. In: McInerny TK, Adam HM,
Campbell DE, Foy JM, Kamat DM, eds. American Academy of Pediatrics Textbook of Pediatric Care.
American Academy of Pediatrics; 2023. Accessed September 1, 2023. Pediatric Care Online
Content Domain
Pulmonology
A 7-year-old boy is seen for evaluation of hoarseness that has been present for several months. He has an
intermittent, nonproductive cough and has recently had difficulty keeping up with his peers when playing,
becoming winded more quickly than his mother and teachers think he should. He has not had fever, recent
respiratory or other illness, snoring, noisy breathing, or trauma to his neck.
On physical examination, he is comfortable at rest. His vital signs are a blood pressure of 105/55 mm Hg, a
heart rate of 80 beats/min, a respiratory rate of 15 breaths/min, and an oxygen saturation of 98% in room air
via pulse oximetry.
His body mass index is at the 40th percentile for age. There is a hemangioma over his left cheek and upper
lip that extends to the oral surface; it has been present since birth and has not changed. He is able to
breathe comfortably through his nose.
Chest examination reveals good aeration with no adventitious sounds, a 1:1 inspiratory-to-expiratory ratio,
and a slight suprasternal tug on inspiration. The remainder of his physical examination findings are
unremarkable.
Of the following, the BEST next step in this boy’s care is
Correct answer is B
PREP Pearl(s)
The most common causes of hoarseness are benign and self-limited (eg, viral infection, voice abuse).
Hoarseness accompanied by evidence of airway compromise should prompt expedited direct
visualization of the airway (laryngoscopy) under controlled conditions.
Neonates with a hoarse cry should undergo prompt evaluation for congenital anomalies of the airway.
Children who require intervention for hoarseness should undergo visualization of the larynx via direct
or fiberoptic laryngoscopy before imaging or referral for speech therapy.
Critique
The boy in the vignette has persistent hoarseness with symptoms of worsening airway obstruction (exercise
intolerance, abnormal inspiratory-to-expiratory ratio) in the context of a facial hemangioma that involves the
mucosal surface. Although hemangiomas usually resolve in infancy or early childhood, they may persist in
older children. A hemangioma that involves the mucosal surface of the oropharynx should raise concern
regarding airway hemangioma. The best next step in this boy’s care is visualization of the larynx with flexible
fiberoptic laryngoscopy.
Per the American Academy of Otolaryngology (AAO) clinical practice guidelines for hoarseness, visualization
of the larynx should be performed before imaging or referral for speech therapy. In most cases of chronic
hoarseness in children and adolescents, there is not an urgent need for airway visualization or imaging.
However, in this case, there is a moderately urgent need for airway visualization because the boy has signs of
an impaired airway.
Etiologies for hoarseness vary by age. The Table outlines some of the many causes of hoarseness by category
and by age. Beyond infancy, viral infection is the most common cause of hoarseness (generally benign and
self-limited). Voice abuse is a common cause of laryngeal nodules, which cause hoarseness at all ages beyond
infancy.
Most causes of pediatric hoarseness are benign and require little or no intervention. Chronic hoarseness is
defined as that which fails to improve after 3 to 4 weeks of voice rest and intervention appropriate to the
suspected cause. Counseling regarding voice rest and hygiene is the best intervention for most children and
adolescents, before hoarseness becomes chronic or at the first evaluation of a stable child with hoarseness
that has lasted at least 3 to 4 weeks. When hoarseness persists beyond 3 or 4 weeks, evaluation for
associated conditions (eg, allergies, laryngopharyngeal reflux) suggested by a careful history and physical
examination may direct intervention for the vocal symptoms. The treatment for vocal cord nodules is speech
therapy; however, the AAO recommends vocal cord visualization before referral to speech therapy, to
provide information that will help guide the intervention.
When stridor or other signs of airway compromise are present, the investigation should be expedited.
Additionally, per the AAO guidelines, an expedited evaluation for hoarseness should occur in children who
have a neck mass or who have had recent surgery (of any type) or recent airway intubation. Although
laryngeal cancer related to smoking is not found in adolescents, smoking is a cause for hoarseness and is an
indication for expedited evaluation. An affected adolescent whose voice is critical to a professional career
should also undergo prompt evaluation.
Infants with a hoarse cry and no history of surgery or airway trauma should undergo expedited evaluation for
congenital anomalies of the airway and neurologic causes. Many of the etiologies for hoarseness in this age
group are associated with poor feeding and aspiration. Early collaboration with specialists regarding upper-
and lower-airway, esophageal, and neurologic concerns is appropriate.
Hoarseness in a child with a history of intubation or surgery involving the neck or chest should prompt direct
visualization of the airway under controlled conditions to evaluate for laryngeal trauma, stenosis, vocal cord
paralysis or dysfunction, or recurrent laryngeal nerve damage. Magnetic resonance imaging and/or other
central nervous system diagnostic procedures should be pursued if there is concern about a neurological
source of hoarseness or abnormal vocal cord function. In the context of cardiovascular abnormalities or
dysmorphic features, referral to a geneticist is appropriate. Specific management is dictated by the
diagnosis.
Suggested Reading(s)
Adam H. Hoarseness. Point-of-Care Quick Reference. Pediatric care Online. American Academy of
Pediatrics. Accessed December 6, 2023. Pediatric Care Online
Parikh SR. Hoarseness. In: McInerny TK, Adam HM, Campbell DE, Foy JM, Kamat DM, eds. American
Academy of Pediatrics Textbook of Pediatric Care. 2nd ed. American Academy of Pediatrics; 2016:chap
165. Accessed September 1, 2023. Pediatric Care Online
Stachler RJ, Francis DO, Schwartz SR, et al. Clinical practice guideline: hoarseness (dysphonia (update)
executive summary. Otolaryngol Head Neck Surg. 2018;158(3):409-426.
doi:10.1177/0194599817751031
Worthen M, Chandran S. Hoarseness in children. Int J Head Neck Surg. 2016;7(2):130-135.
doi:10.5005/jp-journals-10001-1278
Yang J, Xu W. Characteristics of functional dysphonia in children. J Voice. 2018;156.e-1-156.e-4.
doi:10.1016/j.jvoice.2018.07.027
Content Domain
Pulmonology
On physical examination, her temperature is 38.2 °C, her heart rate is 100 beats/min, her respiratory rate is
35 breaths/min, and her oxygen saturation is 92% in room air via pulse oximetry. Her weight is at the 25th
percentile for age and her height at the 75th percentile. She appears uncomfortable and has increased work
of breathing with the use of intercostal and suprasternal muscles. Her breath sounds are decreased
throughout the right lung field with no adventitious sounds. There is dullness to percussion on the right.
There are scattered late inspiratory crackles over the left lung field, with good aeration and no dullness to
percussion. Her liver is palpable 2 to 3 cm below the right costal margin. The remainder of her examination
findings are unremarkable.
Chest radiography shows a large hilar mass and right pleural effusion. Ultrasonography of the chest confirms
a large right-sided pleural effusion. A diagnostic and therapeutic thoracentesis is performed.
Of the following, the MOST likely laboratory finding on the fluid obtained is a
B. uid pH of 7.8
Correct answer is D
PREP Pearl(s)
Pneumonia is the most common cause of pleural effusion in children.
Therapeutic thoracentesis is not necessary if the cause of a pleural effusion is clear and the child’s
condition is stable and improving with treatment.
Exudative pleural effusions have a high protein content and are more likely than transudates to
become complex.
Critique
The girl described in the vignette most likely has a lymphatic malignancy. She has a hilar mass and had rapid
accumulation of a pleural effusion, which precipitated respiratory distress. Malignant effusions are generally
exudative. Of the response choices, the finding on thoracentesis most consistent with an exudate is a pleural
fluid–to–serum protein ratio of greater than 0.5. Other characteristics of an exudative effusion include an
elevated cell count, an abnormal lymphocyte count, high protein and lactate dehydrogenase content, a low
pH, and often a low glucose concentration.
Characteristics of exudative and transudative effusions are outlined in Table 1. There is nothing in this girl’s
history or clinical presentation to suggest intrapleural bleeding; thus, a fluid hematocrit greater than serum
hematocrit would not be expected, even in a malignant effusion.
An infection with dense hilar adenopathy could have a similar clinical presentation and imaging findings. An
exudative effusion would also be expected in the context of infection. The most common cause of pleural
effusion in children is pulmonary infection. Although Streptococcus and Staphylococcus are the most
common bacterial etiologies of pneumonia that result in an effusion, tuberculosis and viral, fungal, and
parasitic infections/infestations can also cause effusions. Causes of exudative effusions are outlined in Table
2.
Transudates are characterized by a normal fluid pH, as well as by low protein and lactate dehydrogenase
content (Table 1). Conditions associated with transudative effusions are outlined in Table 2. Because of their
low protein content, transudative pleural effusions are unlikely to become loculated. Transudative effusions
are generally not therapeutically addressed unless they are causing respiratory distress via their volume.
Treatment is aimed at the underlying process.
Therapeutic thoracentesis is not necessary, even in the context of infection, unless the child is not improving,
is worsening despite treatment of the underlying condition, or is in respiratory distress. Early diagnostic
thoracentesis should be performed when the effusion’s etiology is not clear, and the results will inform
treatment of the underlying condition. Evidence is mixed regarding the value of early drainage and chest tube
insertion for exudative effusions when the underlying cause is clear and the child’s condition is stable. Some
data suggest that early intervention results in earlier mobilization and hospital discharge, but other study
findings are conflicting.
Suggested Reading(s)
Cashen K, Petersen TL. Pleural effusions and pneumothoraces. Pediatr Rev. 2017;38(4):170-179.
doi:10.1542/pir.2016-0088
Fischer GB, Mocelin HT, Andrade CF, Sarria EE. When should parapneumonic pleural effusions be
drained in childhood? Paediatric Respir Rev. 2018;26:27-30. doi:10.1016/j.prrv.2017.05.003
Florin TA, Ambroggio L, Brokamp C, et al. Biomarkers and disease severity in children with community
acquired pneumonia. Pediatrics. 2020;145(6):e20193728. doi:10.1542/peds.2019-3728
McGraw MD, Robinson K, Kupfer O, Brinton JT, Stillwell PC. The use of Light’s criteria in hospitalized
children with a pleural effusion of unknown etiology. Pediatr Pulmonol. 2018;53(8):1101-1106.
doi:10.1002/ppul.24065
Sharma GD. Pleural effusion (nonbacterial). In: Light MJ, Blaisdell CJ, Homnick DN, eds. Pediatric
Pulmonology. American Academy of Pediatrics; 2011:559-569.
American Academy of Pediatrics. Dyspnea. Point-of-Care Quick Reference. Pediatric Care Online.
American Academy of Pediatrics; 2022. Accessed September 1, 2023. Pediatric Care Online
Content Domain
Pulmonology
A 5-year-old boy is seen for a health supervision visit. His mother is concerned about a decrease in school
performance, which she attributes to fatigue from attending his first year of all-day school. He takes a nap in
the afternoon after school, which he did not do while attending half-day preschool. He snores most nights
and sleeps restlessly. His mother has not witnessed apnea but has heard him move and rouse from sleep
after a period of snoring. He has also begun to wet the bed at night once or twice weekly after being fully
continent for more than 2 years.
On physical examination, the boy’s weight is at the 75th percentile and height is at the 40th percentile for
age. In the past year, his weight has increased 6 kg and his height has increased 4 cm. His blood pressure is
110/70 mm Hg, heart rate is 100 beats/min, respiratory rate is 15 breaths/min, and oxygen saturation via
pulse oximetry is 98% in room air. He is breathing easily with no increased work of breathing. His speech is
somewhat nasal, but he is able to breathe through his nose. His oropharynx reveals tonsils 2+ in size without
inflammation and a Mallampati score of 3 (assessment of the size of the base of the tongue relative to the
oropharyngeal cavity). The remainder of his physical examination findings are normal.
A home overnight pulse oximetry recording shows a baseline oxygen saturation of 98% with several grouped
episodes of >4% desaturation. Two episodes went as low as 85% oxygen saturation; these resolved promptly
with arousal.
Correct answer is B
PREP Pearl(s)
Complete overnight polysomnography should be performed to diagnose obstructive sleep apnea and
plan for an appropriate intervention.
Some children with nasal congestion and mild sleep apnea or hypopnea can be treated with topical
nasal corticosteroids and oral montelukast. (Note, montelukast has a black box warning for serious
neuropsychiatric events.)
Tonsilloadenoidectomy (T&A) is recommended as the first-line treatment for obstructive sleep apnea
due to adenotonsillar hypertrophy.
Critique
The boy in the vignette likely has obstructive sleep apnea (OSA) manifested by snoring, restless sleep,
borderline hypertension, daytime somnolence, and poor school performance. His physical examination
findings of modest-sized tonsils (2+) in a crowded pharynx (Mallampati score of 3) (Figure) are consistent with
OSA. Clinical findings (Table 1) and objective evidence of physiologic impact are required to meet the
diagnostic criteria for OSA. The boy’s overnight oximetry is suggestive of sleep-disordered breathing with
episodic mild desaturation, but there is not adequate evidence for a definitive diagnosis of obstructive apnea.
The best next management step is to obtain complete overnight polysomnography to document obstructive
sleep apnea and then plan an appropriate intervention.
Table 2 presents key action statements from the most recent American Academy of Pediatrics Clinical Practice
Guideline for Diagnosis and Management of Childhood Obstructive Sleep Apnea Syndrome. Tonsilloadenoidectomy
(T&A) is recommended as the first-line treatment for OSA due to adenotonsillar hypertrophy. Table 3 outlines risk
factors for postoperative complications of T&A. In some children with nasal congestion and mild sleep apnea or
hypopnea, topical nasal corticosteroids and oral montelukast (recognizing the black box warning on montelukast for
behavior concerns) are effective in decreasing or delaying the need for T&A. This regimen, paired with lifestyle
modification, may be an appropriate intervention for the boy in the vignette if he has evidence of mild sleep
apnea/hypopnea on polysomnography or to treat mild persistent OSA after surgical intervention for more severe
OSA. Table 4 outlines potential therapies for OSAS. Table 5 shows the disease states associated with or complicated
by chronic sleep-disordered breathing in children.
Lateral neck radiography can be used in conjunction with clinical and physiologic findings to assess adenoid
size and upper airway narrowing, but this boy still would need polysomnography for diagnosis of his likely
obstructive apnea. The boy in the vignette does not have findings of acute decompensation, for which an
urgent T&A would be indicated.
Suggested Reading(s)
Chan KC, Au CT, Hui LL, Ng SK, Wing YK, Li AM. How OSA evolves from childhood to young adulthood:
natural history from a 10-year follow-up study. Chest. 2019;156(1):120-130.
doi:10.1016/j.chest.2019.03.007
Gipson K, Lu M, Kinane TB. Sleep-disordered breathing in children. Pediatr Rev. 2019;40(1):3-12.
doi:10.1542/pir.2018-0142
Marcus CL, Brooks LJ, Draper KA, et al; American Academy of Pediatrics. Diagnosis and management of
childhood obstructive sleep apnea syndrome. Pediatrics. 2012;130(3):576-584. doi:10.1542/peds.2012-
1671
Papadakis CE, Chaidas K, Chimona TS, et al. Use of oximetry to determine need for adenotonsillectomy
for sleep disordered breathing. Pediatrics. 2018;142(3):e20173382. doi:10.1542/peds.2017-3382
Thompson R, Splaingard M. Management of snoring. Pediatr Rev. 2021;42(8):471-473.
doi:10.1542/pir.2020-000950
Content Domain
Pulmonology
Table 2. American Academy of Pediatrics 2012 Guideline Table 3. Risk Factors for Postoperative Respiratory Complications
Recommendations for the Diagnosis and Treatment of Obstructive in Children with Obstructive Sleep Apnea Syndrome Undergoing
Sleep Apnea in Children and Adolescents. Adenotonsillectomy.
• ●Cardiac complications of OSA
1. All children/adolescents should be screened for snoring.
• ●Craniofacial anomalies
2. Polysomnography should be performed in children/adolescents with snoring and
• ●Failure to thrive
symptoms/signs of OSA; if polysomnography is not available, then alternative
• ●Neuromuscular disorders
diagnostic tests or referral to a specialist for more extensive evaluation may be
• ●Obesity
considered.
• ●Respiratory infection, current
3. Adenotonsillectomy is recommended as the first-line treatment for patients with
• ●Severe OSA on polysomnography
adenotonsillar hypertrophy.
• ●Younger than age 3 years
4. High-risk patients should be monitored as inpatients postoperatively.
5. Patients should be reevaluated postoperatively to determine whether further Abbreviation: OSA, obstructive sleep apnea.
treatment is required. Objective testing should be performed in patients who are
high risk or have persistent symptoms/signs of OSA after therapy. Table 4. Therapies for Obstructive Sleep Apnea.
6. Continuous positive airway pressure is recommended as treatment if
adenotonsillectomy is not performed or if OSA persists postoperatively. Therapy Indications
7. Weight loss is recommended in addition to other therapy in patients who are
Adenotonsillectomy Adenoidal or tonsillar hypertrophy, with
overweight or obese.
strong clinical evidence of OSA or a definitive
8. Intranasal corticosteroids are an option for children with mild OSA in whom
sleep study (of note, adenoidal tissue may
adenotonsillectomy is contraindicated or for mild postoperative OSA
reaccumulate after adenotonsillectomy).
Abbreviation: OSA, obstructive sleep apnea.
CPAP/Bi-level PAP is the commonest noninvasive treatment
for OSA, providing the additional pressure
needed to help the patient overcome an upper
airway obstruction.
Tracheostomy Principally indicated in cases in which a
Table 5. Disease Associations With and Complications of Sleep- patient is failing to thrive or has otherwise
Disordered Breathing. been exceptionally refractory to less-intensive
OSA therapy. Tracheostomy may be helpful in
System Associated Disease/Complication
cases of severe craniofacial abnormalities or
Cardiovascular • ●Hypertension neurologic issues.
• ●Pulmonary hypertension, cor pulmonale
Mandibular distraction Surgical separation of the mandible is paired
Hematologic • ●Sickle cell disease with an implanted mechanical distraction
device, which permits the controlled
Nutritional/Metabolic • ●Failure to thrive (malnutrition)
separation of opposing bone at a rate that
• ●Insulin resistance
promotes bone remodeling and, thus, gradual
• ●Obesity
extension of the mandible, resulting in a
Neuropsychiatric • ●Attention-deficit/hyperactivity disorder lengthening of the lower jaw and that yields
• ●Developmental delay less crowding of the tongue and associated
• ●Major depressive disorder soft tissue in the oropharynx.
Pulmonary • ●Bronchopulmonary dysplasia Palatal expansion and distraction Use of these orthodontic devices expands the
osteogenesis maxillary expansion hard palate, correcting associated narrowing
Adapted from Gipson K, Lu M, Kinane TB. Sleep-disordered breathing in children. Pediatr
of the nasal passages.
Rev. 2019;40(1):4.
Hypoglossal nerve stimulator Presently in the investigational stage in
children, these devices supply a titratable
electric stimulus to the hypoglossal nerve
while the patient is asleep, inducing the
tongue to push forward and alleviate airway
obstruction. This device is actuated remotely
by the parents each evening at bedtime so as
not to cause disruption to speech or comfort
during waking hours.
Montelukast* and intranasal Leukotriene receptor antagonists, used
corticosteroids principally as an adjunctive therapy to
intranasal corticosteroids, likely improves
nasal airflow by reducing the inflammatory bulk
of the nasal turbinates and adenoidal tissue.
*Montelukast was issued a black box warning for serious neuropsychiatric events
including, but not limited to, agitation, aggression, depression, sleep disturbances, and
suicidal thoughts and behavior.
Abbreviations, Bi-level, bilevel positive airway pressure; CPAP, continuous positive airway
pressure; OSA, obstructive sleep apnea; PAP, positive airway pressure.
AAP PREP 2024 - Question 225/267 Pulmonology 7/9
A 3-year-old girl has wheezing with viral respiratory infections, intermittently requiring treatment with
inhaled bronchodilators and systemic corticosteroids. She has a night cough several times per month, when
she is otherwise well. The girl is the youngest of 3 children living in the home. Family pets include 1 dog and 2
cats. The girl attends day care 3 days a week. She has had 2 or 3 episodes of otitis media each year and has
mild atopic dermatitis. There is no other notable medical history.
Of the following, the factor MOST predictive of persistence of this girl’s condition at age 6 to 12 years is
A. an indoor pet
B. atopic dermatitis
Correct answer is B
PREP Pearl(s)
Early transient wheezing that resolves by school age occurs most often in children with prenatal
tobacco smoke exposure, low lung function at birth that improves by school age, and a lack of atopy or
parental history of asthma.
Children with late-onset wheezing (age 6+ years) or persistent wheezing from infancy are more likely to
have a personal atopic condition and a parental history of asthma, and a greater risk of symptoms that
persist into adolescence and adulthood.
Most individuals with severe persistent asthma had onset of symptoms before school age.
Critique
Of the response choices, the factor most predictive of persistent asthma at age 6 to 12 years for the girl in the
vignette is atopic dermatitis. The most frequently occurring predictive factors for a diagnosis of asthma in a
school-aged child with early wheezing are physician-diagnosed atopic dermatitis and a parental history of
asthma. Day care attendance and the presence of older siblings in the home are not predictive of asthma at
school age. These factors are associated with an increased frequency of viral upper respiratory infections,
which may trigger wheezing in a child with atopy who has a genetic predisposition for asthma or may result
in a nonasthmatic wheezing illness. Indoor pets may trigger wheezing in children with atopy who have pet
sensitization, but their presence is not independently predictive of the development of asthma.
The prevalence of asthma in the United States is 7% to 8% for school-aged children, with up to 14% of
children having a diagnosis of asthma at some time in their lives. Data from the Tucson Children’s Respiratory
Study (TCRS) forms the basis for current knowledge of the epidemiology and natural history of asthma in the
United States. This study began in the early 1980s; the investigators enrolled more than 1,200 infant/family
sets prenatally, most of whom were followed into the children’s adulthood, and carefully documented
multiple attributes, including family history, socioeconomic and environmental influences, respiratory
illnesses, lung function, and immunologic markers.
Data from the TCRS allowed for characterization of the relationship between early wheezing and the
development of asthma at school age. In this study, 60% of children with wheezy lower respiratory tract
infections in the first 2 years after birth had no wheezing at ages 6, 11, and 16 years. These children with
early transient wheezing were more likely to have been exposed to tobacco smoke prenatally; they were also
more likely to have low lung function in the first 3 months after birth, which improved by school age to near
normal. They did not develop atopic conditions or have a parental history of asthma. In children with early
frequent wheezing, the presence of 1 major or 2 minor criteria is predictive of asthma at age 6 years and
beyond (Table).
A second subset of children in the TCRS who had early wheezing also had persistent wheezing at age 6 years
and into adolescence. These children had normal lung function in infancy that declined by age 6 years. Sixty
percent had a persistently elevated serum IgE concentration by age 9 months and were documented to be
sensitized to at least one allergen by age 6 years. A parental history of asthma was common in this group.
A third subset of TCRS children did not have early wheezing, but they developed symptoms around age 6
years. These children typically had atopy and normal lung function both in infancy and at age 6 years.
Personal findings of atopy and a parental history of asthma are predictive of persistence of wheezing
symptoms into school age and adolescence and a diagnosis of persistent asthma. Children with late-onset
wheezing or persistent wheezing at age 6 years are at risk of experiencing asthma that persists into
adolescence and adulthood. Some, particularly those who do not have atopy, may outgrow their asthma and
become asymptomatic with advancing age.
Other factors contributing to the prevalence and severity of asthma include lower socioeconomic status, poor
living conditions, environmental air pollution, stress, and obesity. Genetic factors and specific gene
associations that are not yet identified likely play a significant role in asthma risk. Factors associated with an
asthma diagnosis should be differentiated from those that precipitate episodes of illness in children with
asthma. For example, early tobacco smoke exposure is not predictive of persistent asthma, but tobacco
smoke is a trigger for illness in children with a diagnosis of asthma. Similarly, the presence of a pet in the
home is not predictive of asthma, but it can be a trigger for children with atopic sensitization to pets. Air
pollution, weather conditions, stress, and some viral illnesses are other factors associated with an increased
asthma prevalence.
Retrospective studies of asthma in adolescents and adults suggest that most individuals with persistent
asthma, particularly those with more severe asthma, had onset of symptoms before age 6 years; in some
cases, the asthma was not recognized. Many studies of chronic obstructive pulmonary disease, particularly in
nonsmokers, suggest that airway remodeling resulting from unrecognized or undertreated asthma is a
precipitating factor.
Suggested Reading(s)
Anderson HM, Jackson DJ. Microbes, allergic sensitization, and the natural history of asthma. Curr Opin
Allergy Clin Immunol. 2017;17(2):116-122. doi:10.1097/ACI.0000000000000338
Dinakar C. Asthma. In: McInerny TK, Adam HM, Campbell DE, Foy JM, Kamat DM, eds. American
Academy of Pediatrics Textbook of Pediatric Care. 2nd ed. American Academy of Pediatrics; 2016:chap
218. Pediatric Care Online
Patel SJ, Teach SJ. Asthma. Pediatr Rev. 2019;40(11):549-567. doi:10.1542/pir.2018-0282
Postma DS, Bush A, van den Berge M. Risk factors and early origins of chronic obstructive pulmonary
disease. Lancet. 2015;385(9971):899-909. doi:10.1016/S0140-6736(14)60446-3
Content Domain
Pulmonology
The correct answer is: atopic dermatitis Table. Asthma-predictive Criteria in Children With Early Frequent Wheezing.*
Criteria Category Criteria
View Peer Results Major Parental history of asthma
Physician-diagnosed eczema
Minor Physician-diagnosed allergic rhinitis
Wheezing apart from colds
Peripheral eosinophil count ≥4%
The presence of 1 major or 2 minor criteria is predictive of asthma at age 6 years and
*
beyond.
AAP PREP 2024 - Question 226/267 Pulmonology 8/9
A 3-year-old boy is seen in the emergency department for coughing and shortness of breath. He developed
upper respiratory infection (URI) symptoms 2 days ago and has had worsening cough and di culty breathing
despite the use of an albuterol inhaler with holding chamber 2 pu s every 4 hours. He attends day care and
experiences frequent URIs with prolonged coughing after each episode. Twice in the last 6 months, he has
been treated with oral corticosteroids in addition to inhaled albuterol for similar but less severe episodes.
Between episodes he seems well, although his mother notes nighttime coughing several times monthly and
has concerns that he fatigues with exercise more quickly than his siblings did at the same age.
On physical examination, the boy’s temperature is 37.9 °C, his heart rate is 140 beats/min, his respiratory
rate is 35 breaths/min, and his oxygen saturation is 92% in room air. He appears uncomfortable and has
increased work of breathing with use of intercostal and neck muscles. Air exchange is fair and breath sounds
are diminished throughout, with a prolonged expiratory phase. His liver is palpable 1 to 2 cm below the right
costal margin. His spleen is not palpable. There is no digital clubbing. The remainder of his physical
examination findings are unremarkable.
The boy is admitted to the hospital and treated with oxygen, inhaled bronchodilators every 2 hours, and
systemic corticosteroids. Because of a slow response to treatment and delayed return of oxygen saturation
to normal, chest radiography is obtained; the findings demonstrate right middle lobe atelectasis. Follow-up
chest radiography 2 weeks later shows resolution of the atelectasis.
Of the following, the BEST long-term treatment approach for this boy is to
A. instruct his parents to take him to the emergency department at respiratory symptom onset
B. prescribe daily low-dose inhaled corticosteroid and short-acting inhaled β-agonist to be used
as needed
Correct answer is B
PREP Pearl(s)
Diagnostic criteria for asthma include repeated symptoms of bronchospasm and demonstration of
reversible airflow obstruction. Asthma is classified as intermittent or persistent, with a spectrum from
mild to severe.
Persistent asthma should be treated with daily inhaled corticosteroids for prevention of severe
exacerbations.
Asthma exacerbations not responding quickly to inhaled bronchodilators should be treated with
systemic corticosteroids.
Critique
The boy in the vignette has a moderately severe asthma exacerbation related to a viral upper respiratory
infection (URI). He meets criteria for persistent asthma on the basis of his nocturnal cough, exercise
intolerance, and need for systemic corticosteroids twice in the last 6 months. His long-term treatment should
address chronic airway inflammation in addition to acute symptoms. Use of daily low-dose inhaled
corticosteroids (ICS) supplemented by inhaled bronchodilators as needed represents step 2 of the asthma
management algorithm defined in the 2020 update to the National Institutes of Health (NIH) Asthma
Management Guidelines (Table 1). Persistent asthma is a spectrum of disease, from mild to severe. If the use
of low-dose daily ICS does not prevent severe exacerbations requiring oral corticosteroids, then his treatment
should be escalated to step 3 or 4.
Failure to address the boy’s asthma with a home action plan that includes intervention before going to the
emergency department is not appropriate. The use of short-acting bronchodilators alone has failed to
prevent the need for an emergency department visit. A prescription of oral corticosteroids for use as needed
without a trial of chronic ICS puts the boy at unnecessary risk of experiencing long-term systemic toxicity
caused by use of oral corticosteroids.
Asthma is diagnosed on the basis of recurrent episodes of bronchospasm with at least partial response to
bronchodilators. For school-aged children and adolescents, this response is best documented with
pulmonary function testing before and after use of inhaled bronchodilator, or before and after exercise if
there is suspected asthma with normal lung function and no or little bronchodilator response. In clinical
practice, it is common to make the diagnosis on the basis of symptoms, clinical findings, and a therapeutic
trial of bronchodilator as the measure of response. In wheezing preschool children for whom lung function
testing is not a viable option, and sometimes for older children with acute symptoms, use of a single dose of
an inhaled bronchodilator with repeat physical examination in 15 to 20 minutes is a reasonable clinical
diagnostic tool. For young children with more subtle but chronic symptoms, a therapeutic trial of scheduled
use of an inhaled bronchodilator for several days is a reasonable approach to diagnosis of reversible
bronchospasm; however, care must be taken to communicate that this is for diagnostic purposes and is not a
long-term treatment plan.
Classification of asthma severity incorporates the frequency of daytime and nocturnal symptoms and
response to a short-acting bronchodilator. The classification of asthma as intermittent or persistent, with
categories of mild, moderate, and severe, did not change with the 2020 NIH guidelines. However, at any age,
symptoms (even mild) occurring more than 2 days per week or nocturnal symptoms occurring more than 2
nights per month are classified as persistent rather than intermittent asthma under the 2020 NIH guidelines.
The tables outline the classification according to age group (Table 2) and initial management of asthma
according to severity (Table 1, Table 3, Table 4).
Regardless of the severity category assigned at the time of treatment initiation, asthma exacerbations not
readily responding to inhaled bronchodilators should be treated with corticosteroids. The 2020 Asthma
Management Guidelines recommend consideration of a short course of ICS with bronchodilators at the onset
of symptoms for preschool children with intermittent asthma triggered by viral URIs. School-aged and older
children may be treated with both daily and as-needed use of a combination of ICS and formoterol (Table 3
and Table 4). Use of a combined ICS/formoterol inhaler as needed is appropriate treatment for step 2 care in
older children and adolescents. For more severe asthma episodes not adequately responding to inhaled
corticosteroids with bronchodilators, use of systemic corticosteroids is needed.
Short-term adverse effects of systemic corticosteroids include behavioral changes, gastritis, and sleep
disturbances. A less common (<1%) but more serious adverse effect of systemic corticosteroids is infection
due to immune suppression. Long-term adverse effects of continuous or repeated courses of systemic
corticosteroids most commonly include weight gain, Cushingoid features, and growth suppression.
Additionally, suppression of the hypothalamic-pituitary-adrenal axis, suppression of immune responses,
gastritis, gastrointestinal bleeding, behavior changes, and skin manifestations may occur. Oral candidiasis
and hoarseness are local adverse effects of ICS, mitigated by proper delivery using a holding chamber and
rinsing the mouth after each dose.
Comorbidities and complications (eg, atelectasis, pneumonia, secondary infection) must be considered when
children and adolescents do not respond to appropriate therapy for an acute asthma exacerbation. Although
not routinely indicated, evaluation with chest radiography may be appropriate and antibiotic treatment may
be necessary to treat infectious complications. Monitoring of electrolytes and glucose levels for adverse
effects of pharmacologic intervention is indicated for prolonged or severe asthma episodes.
Suggested Reading(s)
Cloutier MM, Teach SJ, Lemanske RF Jr, Blake KV. The 2020 Focused Updates to the NIH Asthma
Management Guidelines: key points for pediatricians. Pediatrics. 2021;147(6):e2021050286.
doi:10.1542/peds.2021-050286
Dinakar C. Asthma. In McInerny TK, Adam HM, Campbell DE, DeWitt TG, Foy JM, Kamat DM, eds.
American Academy of Pediatrics Textbook of Pediatric Care. American Academy of Pediatrics;
2021:chap 218. Accessed September 1, 2023. Pediatric Care Online
Patel SJ, Teach SJ. Asthma. Pediatr Rev. 2019;40(11):549-567. doi:10.1542/pir.2018-0282
Rosen CM. Corticosteroids. Pediatr Rev. 2019;40(10):546-548. doi:10.1542/pir.2018-0186
Content Domain
Pulmonology
The correct answer is: prescribe daily low-dose inhaled corticosteroid and short-acting inhaled β-agonist to
be used as needed
Source: National Asthma Education and Prevention Program Coordinating Committee Expert Panel Working Group. 2020 Focused Updates to the Asthma Management Guidelines. US Department of
Health and Human Services, National Institutes of Health, National Heart, Lung and Blood Institute. December 2020.
Table 1: Stepwise treatment of asthma in children 0-4 years old.
Table 2. Asthma Severity in Children Aged 0-18 Years.
Factor Symptoms Age Symptom Severity
Intermittent Persistent
Mild Moderate Severe
Impairment Daytime symptoms All ages <2 days/week >2 days/week but Daily Throughout the day
not daily
Nighttime symptoms 0-4 years none 1-2 times/month 3-4 times/month
Lung function ≥5 yr FEV1>80% predicted FEV1>80% predicted FEV1 60-80% FEV1 <60% predicted
FEV1/FVC >0.85 FEV1/FVC >0.8 predicted FEV1/FVC <0.75
FEV1/FVC 0.75-.08
Risk Exacerbations requiring 0-1 time/year for 0-4 years: Grade severity on
systemic steroids all ages >2 in 6 months and/ frequency of episodes
or >4 episodes of and intervention
wheezing/year lasting needed
>1 day
≥5 years:
≥ 2 times/year
Abbreviations: FEV1, forced expiratory volume in 1 second; FVC, forced vital capacity; FEV1/FVC, ratio of two functions; SABA, short-acting β-agonist.
Modified from National Heart, Lung, and Blood Institute, National Asthma Education and Prevention Program. Expert Panel Report 3: Guidelines for the Diagnosis and Management of
Asthma. Full Report 2007. National Heart, Lung, and Blood Institute; 2007. Accessed September 12, 2022. https://www.nhlbi.nih.gov/sites/default/files/media/docs/EPR-3_Asthma_Full_
Report_2007.pdf
Source: National Asthma Education and Prevention Program Coordinating Committee Expert Panel Working Group. 2020 Focused Updates to the Asthma Management Guidelines. US Department of
Health and Human Services, National Institutes of Health, National Heart, Lung and Blood Institute.
Table 3: Stepwise treatment of asthma in children 5-11 years old.
Source: National Asthma Education and Prevention Program Coordinating Committee Expert Panel Working Group. 2020 Focused Updates to the Asthma Management Guidelines.
US Department of Health and Human Services, National Institutes of Health, National Heart, Lung and Blood Institute. December 2020.
Table 4: Stepwise treatment of asthma in children and adolescents 12 years and older.
AAP PREP 2024 - Question 227/267 Pulmonology 9/9
A 12-year-old has a history of recurrent multifocal pneumonia, sinusitis, and otitis media. An extensive
evaluation for humoral and cellular immunodeficiencies and cystic fibrosis was unrevealing. Although they
do not have dextrocardia or situs inversus on radiographic examination, primary ciliary dyskinesia is
suspected and confirmed via electron microscopic evaluation of tracheal cilia.
A. aspiration
B. bronchiectasis
C. digital clubbing
D. hemoptysis
Correct answer is B
PREP Pearl(s)
Bronchiectasis most commonly occurs with cystic fibrosis (CF). Non-CF cases are most commonly a
complication of recurrent viral or bacterial infections. Up to 30% of non-CF bronchiectasis is idiopathic.
The characteristic clinical manifestation of bronchiectasis is a chronic wet cough.
The definitive diagnostic test for bronchiectasis is non-contrast high-resolution computed tomography
of the chest.
Critique
The child described in the vignette has primary ciliary dyskinesia (PCD). Of the response choices, the most
likely complication of PCD is bronchiectasis. Bronchiectasis is precipitated by repeated lower respiratory
infections. Aspiration of oral contents may lead to recurrent pulmonary infection and bronchiectasis;
however, aspiration is not a complication of PCD. Digital clubbing is an uncommon complication in individuals
who do not have cystic fibrosis (CF), but it can be a late complication of bronchiectasis from any cause.
Hemoptysis is not generally seen in PCD but can be a complication of extensive and late-stage bronchiectasis.
Other complications of bronchiectasis include distal airway mucus plugging, focal and diffuse air trapping,
and hyperinflation; chronic changes may be seen on chest radiography.
The primary clinical manifestation of bronchiectasis, regardless of cause, is a chronic wet cough. Most
affected individuals are diagnosed with recurrent lower respiratory tract infections. Although bronchiectasis
has no distinguishing physical examination findings, lung examination may demonstrate wheezing,
decreased breath sounds, and intermittent crackles. Lung examination findings are most often abnormal
during an acute exacerbation and may be entirely normal between illnesses.
With bronchiectasis, chest radiography findings may show central peribronchial thickening or patchy areas of
increased density. In advanced cases of bronchiectasis, a “tram track” appearance, created by a prominent
airway between thickened bronchial walls, may be seen. A definitive diagnosis of bronchiectasis can be made
radiographically with non-contrast high-resolution computed tomography. Findings consistent with
bronchiectasis include dilated airways with surrounding inflammation in the affected segment(s) (“signet ring
effect”), which occurs when the airway is larger than the adjacent blood vessel, lack of central-to-peripheral
airway lumen tapering, and focal or general hyperinflation (Figure). Lung function testing usually shows a
moderate-to-severe obstruction that may not fully reverse with inhaled bronchodilator.
The common pathophysiology in cases of bronchiectasis is focal inflammation from airway injury or recurrent
infection. While the most common cause of bronchiectasis is CF, there are a variety of other causes,
including:
Less common causes include foreign body aspiration, congenital disorders (including PCD), connective tissue
disorders, injury from obstruction or toxic inhalation, and allergic bronchopulmonary aspergillosis.
Severe or recurrent infection (eg, tuberculosis, measles) is a more common cause of bronchiectasis in low-
income populations. Immunodeficiencies as an underlying cause for infection are less frequently identified in
these populations and foreign body aspiration is more common.
Children with recurrent aspiration from any cause are at risk for bronchiectasis. Examples include the
following:
The Table outlines the differential diagnosis and approach to the evaluation of bronchiectasis.
Allergic bronchopulmonary aspergillosis (ABPA) is an immunologic injury that leads to central saccular
bronchiectasis in individuals with underlying allergies and asthma. It is characterized by persistent wheezing
despite appropriate intervention, often with expectoration of golden brown mucus plugs which are culture
positive for Aspergillus fumigatus. The high-resolution computed tomographic pattern of bronchiectasis in
ABPA is different from that of bronchiectasis due to infection or airway injury.
The initial evaluation of suspected bronchiectasis (history, physical examination, radiography, testing) should
focus on the most common etiologies (Table):
Cystic fibrosis
Primary immunodeficiency
Primary ciliary dyskinesia
If the initial evaluation is unrevealing, further evaluation of bronchiectasis should expand (based on
symptoms) to tests that evaluate for the next most commonly seen causes, which include the following:
Postinfectious
Chronic oral aspiration
Foreign body aspiration or airway obstruction
Congenital structural airway anomaly
α1-antitrypsin deficiency
Suggested Reading(s)
Adam H. Recurrent infections. Point-of-Care Quick Reference. Pediatric Care Online. American
Academy of Pediatrics. April 15, 2021. Accessed September 1, 2023. Pediatric Care Online
Chang AB, Bush A, Grimwood K. Bronchiectasis in children: diagnosis and treatment. Lancet.
2018;392:866-879. DOI: 10.1016/S0140-6736(18)31554-X
Gaffar S, Lei WW, Harrington JW. Bronchiectasis. Pediatr Rev. 2021;42(2):103-105. doi:10.1542/pir.2020-
0110
Kasi AS, Kamerman-Kretzmer RJ. Cough. Pediatr Rev. 2019;40:157-167. doi.org/10.1542/pir.2018-0116
O’Grady AF, Drescher BJ, Goyal V, et al. Chronic cough postacute respiratory illness in children: a cohort
study. Arch Dis Child. 2017;102:1044-1048.
Content Domain
Pulmonology
Courtesy of M. Guill
FIGURE: Bronchiectasis on high-resolution computed tomography of chest.
Yellow arrows demonstrate dilated peripheral airways with surrounding
inflammation in atelectatic segment. Note signet ring effect in area circled
in blue.
Table. Differential Diagnosis and Evaluation of Bronchiectasis.
Disease Process Possible Diagnosis Evaluation
A physician working in a busy pediatric intensive care unit is entering orders for recently admitted patients. A
2-year-old boy with a seizure disorder is admitted for pneumonia, and a 13-year-old girl is admitted for
diabetic ketoacidosis (DKA). The nurse in charge of the 2-year-old confirms the correct home medication of
levetiracetam and informs the physician, who inadvertently places the order in the electronic medical record
of the patient with DKA. The error is caught before the medication is administered to the girl admitted for
DKA. The medication is then entered in the correct patient’s chart, but as a result of the error, administration
of his usual dose is delayed. The 2-year-old has a tonic-clonic seizure lasting 2 minutes before the dose is
dispensed from the pharmacy. The physician taking care of these patients is appropriately distressed about
this event.
Of the following, the BEST advice for this physician is to
Correct answer is C
PREP Pearl(s)
Medical errors, including near-miss events, should be disclosed to families in a timely manner and
reported through the institution’s patient safety reporting system.
Barriers to provider reporting of adverse events include personal discomfort and fear of
repercussions.
Institutions should promote a culture of safety by focusing on system change rather than individual
blame.
Critique
The best advice for the physician in the vignette is to disclose the medication error to both families. The
parents of the 2-year-old boy should be informed that there was a delay in administering the antiepileptic
medication owing to an error in order entry. Although it is impossible to prove that the delay directly caused
the seizure, his parents should be informed that it may have been a contributing factor. Even though the
medication error was caught before causing harm to the 13-year-old girl, her parents should still be informed
of the near-miss (ie, potential adverse event). Disclosing and reporting near-miss events leads to analysis that
helps uncover potential system issues before harm is done. The error and its impact on both patients should
also be reported through the institution’s patient-safety reporting system.
Physicians are often uncomfortable about reporting errors and fear repercussions. However, failure to
disclose errors in a timely manner can lead to distrust in the patient-provider relationship. Parents often
appreciate honest conversations and prefer to hear about a medical error from their provider. The decision
about whether to include a pediatric patient in a disclosure talk depends on the child’s developmental stage
and maturity level, as well as the parents’ preference.
Patient safety is one of the most important quality care domains as defined by the National Academy of
Medicine. Medical errors and adverse events are estimated to cause up to 250,000 deaths annually. Children
are at higher risk of experiencing medical errors because of weight-based medication dosing, their varied
developmental stages, and the need for corroborating information from families. A medical error (ME) is
defined as an act of commission or omission that has potential for patient harm. An ME that leads to actual
patient harm is considered a preventable adverse event (AE). An ME that did not cause actual harm but had
the potential to do so is considered a near-miss or potential AE (Figure). Factors contributing to medical error
include physician fatigue/distraction, similar patient names, look-alike or sound-alike drugs, over-reliance on
electronic medical record systems, and lack or failure of second-check systems. Adverse events lead to
emotional distress not only for patients and their families but also for healthcare providers; their feelings of
guilt and blame may cause ‘‘second victim syndrome.”
Barriers to provider reporting of adverse events include personal discomfort, lack of training in disclosing
errors, fear of legal repercussions, fear of blame, and loss of academic standing among colleagues.
Institutions that follow a “just culture” framework create a fair accountability system by understanding
human mistakes, focusing on system changes, and reserving punitive actions for reckless behaviors. Event
reporting should be anonymous, to minimize fear of repercussions. However, the data collected and
analyzed should be transparent, and there should be mechanisms that promote system changes to prevent
future errors. Graduate medical education training programs should provide a robust curriculum on quality
improvement and patient safety. An institutional culture of safety dedicated to improving system errors,
rather than placing individual blame, facilitates reporting of errors and ultimately leads to reduced patient
harm.
Suggested Reading(s)
Committee on Medical Liability and Risk Management; Council on Quality Improvement and Patient
Safety. Disclosure of adverse events in pediatrics. Pediatrics. 2016;138(6):e20163215.
doi:10.1542/peds.2016-3215
Mueller BU, Neuspiel DR, Fisher ERS; Council on Quality Improvement and Patient Safety, Committee
on Hospital Care. Principles of pediatric patient safety: reducing harm due to medical care. Pediatrics.
2019;143(2):e20183649. doi:10.1542/peds.2018-3649
Shaikh SK, Cohen SP. Disclosure of medical errors. Pediatr Rev. 2020;41(1):45-47. doi:10.1542/pir.2018-
0228
Neuspiel DR. Medical errors, adverse events, and patient safety. In: McInerny TK, Adam HM, Campbell
DE, DeWitt TG, Foy JM, Kamat DM, eds. American Academy of Pediatrics Textbook of Pediatric Care.
American Academy of Pediatrics; 2023. Accessed September 1, 2023. Pediatric Care Online
Content Domain
Quality Improvement/Patient Safety
Reprinted with permission from McDonnell WM, Altman RL, Bondi SA, Fanaroff JM, Narang
SK, et al. Disclosure of adverse events in pediatrics. Pediatrics. 2016;138(6): e20163215.
Abbreviations: AEs, Adverse events.
FIGURE: Types of medical errors.
AAP PREP 2024 - Question 229/267 Quality Improvement/Patient Safety Question 2/2
A 6-month-old girl is brought to the emergency department by her maternal aunt after an unwitnessed fall
from the bed to a carpeted floor. Her maternal aunt does not speak English but is able to sign the girl in at
the front desk with the help of a cousin who speaks English. The family is brought to the triage area and the
nurse measures the patient’s vital signs. With the infant’s cousin acting as translator, the attending physician
obtains the child’s medical history from the aunt, including information about the fall. The girl is known to be
developmentally delayed and cannot yet roll. On physical examination, she has a bruise on her right cheek,
which her aunt reports was from the girl laying on a stuffed animal last week. No other abnormal findings
were noted on her examination. The physician documents that there is a low suspicion for nonaccidental
trauma, and the girl is discharged home with her aunt.
The girl is brought to the emergency department 1 week later after having a seizure. On physical
examination, she is noted to have bilateral retinal hemorrhages. Imaging reveals acute and chronic subdural
hemorrhages and healing posterior rib fractures. On review of this case, the emergency department
leadership determines that a systems approach to preventing a similar incident is required.
B. implement a patient safety training session for the emergency department staff
Correct answer is C
PREP Pearl(s)
Root cause analysis is a standardized process that identifies factors involved when there is an
undesired outcome, with the goal of improving the system to enhance prevention.
Quality improvement methods rely on scientific and structured methods to implement effective
change in a health care system.
Critique
The approach most likely to be successful in this situation is to organize an interdisciplinary team to review
the case, focusing on the events that led to the unintended outcome. This approach can be accomplished by
following the principles of root cause analysis (RCA).
An RCA is used to identify factors that may lead to medical errors; the goal of this analysis is prevention. This
approach enables the health care organization to focus on the systems in place that may contribute to an
error that reaches a patient rather than the specific event or actions of an individual. The American Academy
of Pediatrics’ 2019 policy statement Principles of Pediatric Patient Safety: Reducing Harm Due to Medical Care
guides pediatricians to focus on “the significance of pediatric patient safety, the science behind the culture of
safety, and strategies to ensure patient safety.” The Joint Commission requires accredited health care
institutions to have a standardized process in place for situations similar to the one in the vignette.
The first step of RCA is to form a team to identify all the events that played a role in the unintended outcome.
The Joint Commission provides a template to assist with this analysis. Some example questions from this
template include the following:
The RCA process helps identify systems problems. The team then works together to identify systems
solutions that will help prevent future events. Strategies and a timeline for implementation of new or
modified processes and procedures are then defined. This analysis and plan are communicated to
organizational leadership, who will need to provide support for the plan (eg, dissemination, education, and
resources).
Quality improvement methods are an important process for enhancing the delivery of health care to patients.
Quality improvement initiatives approach problems at the patient level and seek to improve care, often by
implementing evidence-based guidelines. This scientific method focuses on answering the question of why
change is needed (global aim) and how to create change by using the SMART acronym (Specific, Measurable,
Attainable, Relevant, Time-bound). A key driver diagram (Figure 1) can assist the quality improvement team in
determining the actions needed to make a change.
Generally, several rapid Plan-Do-Study-Act (PDSA) cycles are used to test proposed changes. The steps
include the following:
Plan - defines the details of the project and determines how data will be collected
Do - change is implemented and observed (data collection)
Study - data are analyzed
Act - a determination is made regarding the next steps for the change
The PDSA cycle is repeated until there is enough information gathered to determine whether the change is
improving the system. A run chart (Figure 2) is helpful for visualizing the impact of the change. Once the PDSA
cycles demonstrate improvement, changes can be implemented more broadly; this may include the
implementation of new policies and procedures. Training for all those involved is necessary to sustain the
change.
An RCA might identify several events in this infant’s care that would be considered systems problems or
medical errors that may have contributed to the undesired outcome. The girl was brought to the emergency
department by her aunt (not her mother or father); guardianship was not confirmed. Her aunt did not speak
English, and the staff relied on a family member to communicate rather than a translating service. The
attending physician also failed to verify guardianship and relied on the family member to translate. The girl’s
history and physical examination findings should have raised suspicion for child abuse. For any infant being
evaluated after an unwitnessed injury, there are recommended laboratory and imaging studies to screen for
child abuse. In addition, the girl’s examination revealed a bruise on her cheek, with an explanation that is not
plausible (lying on a stuffed animal), and the infant’s developmental level is not consistent with the story that
she fell off the bed (she is unable to roll).
Implementing a child abuse pathway alert in the electronic medical record, organizing a patient safety
training session for the nursing staff, and planning a grand rounds lecture on child abuse for the emergency
department attending physicians are all ways to potentially improve the system and decrease missed child
abuse cases. However, these changes should occur after careful analysis of the events by a multidisciplinary
team.
Suggested Reading(s)
Joint Commission. The framework for root cause analysis and corrective action. Accessed November
22, 2022. https://www.jointcommission.org/-/media/tjc/documents/resources/patient-safety-
topics/sentinel-event/rca_framework_101017.pdf?
db=web&hash=B2B439317A20C3D1982F9FBB94E1724B&hash=B2B439317A20C3D1982F9FBB94E1724
B.
Parikh K, Hochberg E, Cheng JJ, et al. Apparent cause analysis: a safety tool. Pediatrics.
2020;145(5):e20191819. doi:10.1542/peds.2019-1819
Simpson B, Statile AM, Schondelmeyer AC. How to perform quality improvement projects. Pediatr Rev.
2022;43(10):549-560. doi:10.1542/pir.2021-005314
Content Domain
Quality improvement/patient safety
Reprinted with permission from Simpson B, Statile AM, Schondelmeyer AC. How to perform
quality improvement projects. Pediatr Rev. 2022;43(10):556.
FIGURE 2: Sample run chart.
AAP PREP 2024 - Question 230/267 Racism/DEI Question 1/1
A previously healthy 2-year-old Native American child is evaluated in the emergency department after 4 days
of vomiting and profuse watery diarrhea. His mother has offered him fluids frequently, but today he has only
had 2 ounces of a pediatric oral electrolyte replacement solution. His immunizations are up-to-date.
On physical examination, the boy appears ill but nontoxic. His heart rate is 150 beats/min, respiratory rate is
30 breaths/min, and blood pressure is 86/44 mm Hg. He has dry mucous membranes and a capillary refill
time of 4 seconds.
Intravenous 25% dextrose (2 ml/kg) and isotonic fluid boluses (20 ml/kg) are administered, after which, he is
still unable to take fluids orally and is admitted to the hospital.
Of the following, the GREATEST risk factor for hospitalization of this boy is
A. age
C. race
D. vaccination status
Correct answer is C
PREP Pearl(s)
Racism leads to healthcare disparities in several groups including American Indian, African American,
and Hispanic. These disparities persist despite adjustments for income and insurance coverage.
Healthcare disparities related to racism include infant mortality rates, delayed diagnosis of autism,
delays in referral to subspecialty care, and the risk of hospitalization for disease processes including
urinary tract infections, pneumonia, asthma, bronchiolitis and influenza.
Critique
Of the response choices, this child’s race is the greatest risk factor for admission to the hospital. Native
American (Indigenous, First Nations) children are at increased risk of hospitalization for a number of illnesses,
including gastroenteritis. Gastroenteritis is a common diagnosis in children that can lead to hospitalization if
they are unable to maintain adequate hydration with oral intake. Unlike the risk for affected infants, this
child’s age does not place him at high risk for requiring hospitalization for gastroenteritis. The boy was
offered appropriate electrolyte- and glucose-containing fluids; therefore, the oral fluid type was not a risk
factor for admission for this child. His vaccine status is an unlikely risk factor for the boy, as he has received
all of the recommended vaccines.
Racism is a major factor contributing to health and health care disparities. Even when adjusting for income
and education, disparities persist. Various studies report disparities among populations, including infant
mortality rates, delayed diagnosis of autism, and delays in referral to subspecialty care. Their risk of
hospitalization is increased for many disease processes including gastroenteritis, urinary tract infections,
pneumonia, asthma, bronchiolitis and influenza. Furthermore, the length of hospital stay and time to the
operating room are statistically longer.
The cause of this disparity is likely multifactorial. The incidence of poverty is higher, with about 60% of the
people from these races experiencing poverty versus 30% of people who identify as white. Additional
contributing factors include a greater likelihood of living in high-poverty neighborhoods with increased
environmental exposures (eg, pollution, small particles, and tobacco), and less job flexibility.
Personal factors, including concern for being treated unfairly by medical providers, lead many to avoid the
healthcare system. There is an increased risk of comorbidities (eg, obesity) and barriers to adherence to
healthcare recommendations due to challenges with health literacy, transportation, and clear
communication. Family-centered rounds (involving family members/guardians in care discussions and
planning) lead to improved outcomes and have become the standard of care. However, the need for an
interpreter during rounds creates an additional barrier for families who do not fluently speak English; the
family may view this as a burden, making family-centered rounds less productive.
Clear communication during office visits and hospitalizations is vitally important to good care and is linked to
improved outcomes. Cultural differences, language barriers, and varying levels of health literacy and
education can all contribute to families not fully understanding medical information shared with them, and
key points in the history may be missed.
Suggested Reading(s)
American Academy of Pediatrics. The impact of racism on child and adolescent health. Pediatrics.
2019;144(2). doi:10.1542/peds.2019-1765
American Academy of Pediatrics; Patient Education. Using books to talk to kids about race and racism.
American Academy of Pediatrics; 2022. https://doi.org/10.1542/ppe_document257
Jindal M, Trent M, Mistry K. The intersection of race racism and child and adolescent health. Pediatr
Rev. 2022;43(8):415-425. doi:10.1542/pir.2020-004366
McKay S, Parente V. Health disparities in the hospitalized child. Hosp Pediatr. 2019;9(5):317-325.
doi:10.1542/hpeds.2018-0223
Content Domain
Racism/DEI
An 8-year-old boy is brought to the office for follow-up. He was seen 2 weeks ago for a sore throat, at which
time he was noted to have an elevated blood pressure. Laboratory testing performed at that visit
demonstrated normal renal function and an unremarkable urinalysis. Today, the boy’s weight is at the 25th
percentile and his height is at the 50th percentile for age. His heart rate is 88 beats/min, his respiratory rate
is 16 breaths/min, and his blood pressure in the right arm is 128/88 mm Hg. Completion of 4 limb blood
pressure measurements shows readings of 126/86 mm Hg in the left arm, 136/94 mm Hg in the left thigh,
and 138/96 mm Hg in the right thigh. His physical examination findings are otherwise remarkable only for a
bruit heard in the upper abdomen.
C. echocardiography
Correct answer is B
PREP Pearl(s)
Renal disease and renovascular disease are the most common secondary causes of hypertension in
children.
Renal artery stenosis should be suspected in a child with stage 2 hypertension, significant diastolic
hypertension, hypokalemia, abdominal bruit, and a kidney size discrepancy on renal ultrasonography.
Doppler ultrasonography of the renal vessels is used to screen for renal artery stenosis. Computed
tomographic angiography or magnetic resonance angiography are noninvasive tests used to diagnose
renal artery stenosis.
Critique
The boy in the vignette has stage 2 hypertension, a normal upper and lower limb blood pressure gradient,
and an abdominal bruit on physical examination. The best next step in his evaluation is to obtain Doppler
renal ultrasonography to evaluate for renal artery stenosis, the most likely cause of his hypertension.
For children and adolescents, normal blood pressure levels are defined based on age, sex, and height. The
American Academy of Pediatrics 2017 Clinical Practice Guideline for Screening and Management of High
Blood Pressure in Children And Adolescents (Flynn et al) defines normal blood pressure and the stages of
hypertension as follows:
The 95th percentile blood pressure for an 8-year-old boy (50th percentile for height) is 114/74 mm Hg. The
boy’s blood pressure in the office today is 128/88 mm Hg, which classifies him as having stage 2
hypertension.
Hypertension in children is further classified as primary (no identifiable cause) or secondary (with identifiable
cause). Primary hypertension is usually asymptomatic, and often associated with obesity and a family history
of hypertension. Children with secondary hypertension are usually younger than 6 years, have an elevated
diastolic blood pressure, and exhibit signs and symptoms of the underlying cause. Causes of secondary
hypertension in children are shown in Table 1. Renal disease and renovascular disease are the most common
secondary causes.
The diagnostic evaluation for children with hypertension (Table 2) includes a detailed history, physical
examination, urinalysis, basic metabolic panel (including renal function tests), lipid profile, and renal
ultrasonography. Renal artery stenosis (RAS) should be suspected in a child with stage 2 hypertension,
significant diastolic hypertension, hypokalemia, abdominal bruit, and a kidney size discrepancy on renal
ultrasonography. Doppler ultrasonography of the renal vessels is used to screen for RAS in normal-weight
children at least 8 years old who can cooperate with the procedure. Computed tomographic angiography and
magnetic resonance angiography are noninvasive tests used to diagnose RAS. Renal arteriography is the
reference standard for diagnosis of RAS.
A dimercaptosuccinic acid (DMSA) scan is indicated when renal scarring is suspected as a cause of
hypertension. The boy in the vignette has normal renal function, an unremarkable urinalysis finding, and no
history of urinary tract infections; therefore, a DMSA scan is not indicated. Coarctation of the aorta is
suspected when the systolic blood pressure is 10 to 20 mm Hg higher in the upper extremity than in the
lower. The boy in the vignette has a normal blood pressure gradient between his upper and lower
extremities. Echocardiography will not identify the cause of this boy’s hypertension, because there is no
evidence of coarctation of the aorta on his physical examination, so this test is not the best next step in his
evaluation. However, echocardiography may be indicated in the future to assess left ventricular mass,
geometry, and function, as well as cardiac damage. Magnetic resonance imaging of the adrenal glands is
indicated to diagnose pheochromocytoma, which is unlikely for this boy owing to the absence of episodic
headache, tachycardia, and sweating.
Suggested Reading(s)
Adam HM. Hypertension. Point-of-Care Quick Reference. Pediatric Care Online. American Academy of
Pediatrics. April 6, 2020. Accessed September 1, 2023. Pediatric Care Online
Flynn JT, Kaelber DC, Baker-Smith CM, et al. Clinical practice guideline for screening and management
of high blood pressure in children and adolescents. Pediatrics. 2017;140(3):e20171904.
doi:10.1542/peds.2017-1904
Weaver DJ Jr. Hypertension in children and adolescents. Pediatr Rev. 2017;38(8):369-382.
doi:10.1542/pir.2016-0106
Content Domain
Renal
Renal parenchymal disease • Failure to thrive • Edema • Blood urea nitrogen level
• Family history of renal disease • Pallor • Complete blood cell count
• Gross hematuria • Palpable mass • Electrolyte levels
• History of oligohydramnios • Short stature • Genetic testing for monogenetic forms of
• Muscle weakness hypertension (for consideration)
• Nocturia • Renal ultrasonography
• Polyuria • Serum creatinine level
• Swelling • Urinalysis
• Urinary tract infections
Renovascular disease • Neonatal history of an umbilical artery catheter • Abdominal mass • Angiography
• Adenoma sebaceum • Computed tomographic or magnetic
• Ash leaf spots resonance imaging
• Café-au-lait spots • Renal ultrasonography with Doppler
• Carotid or abdominal bruit • Serum aldosterone level
• Neurofibromas • Serum renin level
Adapted and reprinted with permission from Weaver DJ Jr. Hypertension in children and adolescents. Pediatr Rev. 2017;38(8):373.
AAP PREP 2024 - Question 232/267 Renal Question 2/7
A 1-day-old male neonate is evaluated in the nursery. He was born to a 25-year-old primigravida mother at
40 weeks’ gestation via vaginal delivery. Prenatal ultrasonographic findings were significant for moderate
hydronephrosis of the right kidney. He is breastfeeding well and has good urine output. The neonate’s weight
is 3.2 kg, and his length is 50 cm. He is vigorous and alert, and his vital signs are normal. His physical
examination findings, including those of the genitourinary examination, are unremarkable.
Of the following, the BEST next step in this neonate’s evaluation is to perform
Correct answer is C
PREP Pearl(s)
Common causes of prenatal hydronephrosis are transient physiologic hydronephrosis, ureteropelvic
junction obstruction, vesicoureteral reflux, and ureterovesical junction obstruction.
Neonates with prenatal unilateral hydronephrosis should undergo postnatal renal ultrasonography 1
to 2 weeks after birth.
Neonates with prenatal bilateral hydronephrosis or a solitary kidney with hydronephrosis should
undergo postnatal renal ultrasonography within 48 hours after birth.
Critique
The neonate in the vignette has unilateral moderate hydronephrosis demonstrated on prenatal
ultrasonography. The postnatal ultrasonographic examination, in a case of unilateral hydronephrosis, should
be performed when the neonate has nearly regained birth weight, usually 1 to 2 weeks after birth.
Hydronephrosis noted on prenatal ultrasonography has a reported incidence of 1% to 5%. The most common
causes are transient physiologic hydronephrosis, ureteropelvic junction obstruction, vesicoureteral reflux
(VUR), and ureterovesical junction obstruction. These and other common etiologies of unilateral and bilateral
prenatal hydronephrosis are outlined in the Table. Infants and children with hydronephrosis are commonly
asymptomatic but can present with urinary tract infection. Severe hydronephrosis can present with
abdominal mass and/or hematuria. Bilateral hydronephrosis, such as that seen with posterior urethral
valves, can lead to urinary incontinence, poor urinary stream, failure to thrive, and renal failure.
Renal function testing (eg, serum creatinine level) is not recommended in the first 7 to 10 days after birth for
neonates with unilateral hydronephrosis and a normal contralateral kidney, as earlier testing will be reflective
of the mother’s creatinine level rather than that of the neonate. However, if bilateral hydronephrosis is
present, renal function testing should be performed to assess for the presence of renal dysfunction.
Renal and bladder ultrasonography is the preferred initial imaging study in the evaluation of prenatal
hydronephrosis. The timing of initial renal ultrasonography depends on whether there is unilateral or
bilateral involvement or the presence of solitary kidney on prenatal ultrasonography. Renal ultrasonography
obtained within 48 hours after birth can show a false absence or reduction of hydronephrosis because
neonatal urinary flow is reduced in the first 1 to 2 days after birth. For the most accurate results, a neonate
with prenatally diagnosed unilateral hydronephrosis with a normal contralateral kidney should undergo renal
ultrasonography 1 to 2 weeks after birth. Neonates with bilateral prenatal hydronephrosis or a solitary kidney
with hydronephrosis should undergo postnatal renal ultrasonography within 48 hours after birth to prevent
renal injury due to a delay in diagnosis. Neonates with bilateral hydronephrosis may have posterior urethral
valves, which should be promptly managed via urinary catheter insertion and surgical correction. Neonates
with hydronephrosis in a solitary kidney may have severe VUR, and early intervention is required.
The presence of a dilated ureter is helpful in narrowing the differential diagnosis of hydronephrosis.
Ureteropelvic junction obstruction, which impairs urinary flow from the renal pelvis to the ureter, results in
dilation of the renal pelvis (hydronephrosis without a dilated ureter). Ureterovesical junction obstruction,
which impairs urinary flow from the ureter to the bladder, leads to dilation of the ureter and structures up to
the renal pelvis (hydroureteronephrosis). Other uncommon causes of obstruction to urine flow that lead to
hydroureteronephrosis include ureterocele (cystic dilation of the ureter at the insertion into the bladder) and
ectopic ureter (abnormal insertion of the ureter into the bladder). Vesicoureteral reflux can also cause
hydronephrosis with a dilated ureter.
The severity and persistence of hydronephrosis in the postnatal period determines the need for further
evaluation. Neonates with unilateral mild hydronephrosis are monitored with serial renal ultrasonography. A
voiding cystourethrogram is indicated for (a) unilateral moderate-to-severe hydroureteronephrosis to
diagnose VUR and (b) bilateral hydronephrosis to diagnose bilateral VUR and posterior urethral valves.
Suggested Reading(s)
Campbell DE, Nemerofsky SL, Iyare A, Mauch TJ, Schwend RM. Postnatal assessment of common
prenatal sonographic findings. In: McInerny TK, Adam HM, Campbell DE, DeWitt TG, Foy JM, Kamat DM,
eds. American Academy of Pediatrics Textbook of Pediatric Care. American Academy of Pediatrics;
2017:chap 97. Accessed September 1, 2023. Pediatric Care Online
Nguyen HT. Obstructive uropathy and vesicoureteral reflux. In: McInerny TK, Adam HM, Campbell DE,
DeWitt TG, Foy JM, Kamat DM, eds. American Academy of Pediatrics Textbook of Pediatric Care.
American Academy of Pediatrics; 2017:chap 299. Accessed September 1, 2023. Pediatric Care Online
Schlomer BJ, Copp HL. Antenatal hydronephrosis. NeoReviews. 2013;14(11):e551-e561.
doi:10.1542/neo.14-11-e551
Content Domain
Renal
A 3-year-old with end-stage renal disease secondary to renal dysplasia is seen in the office for immunization
before undergoing a kidney transplant. The surgery will occur in 8 weeks; their mother will be the donor. The
child will receive long-term immunosuppressive medications (tacrolimus and mycophenolate mofetil) to
prevent rejection after the transplant. They have received all of their routine age-appropriate immunizations.
The child is at the 5th percentile for height and 25th percentile for weight for their age. Vital signs and physical
examination findings are unremarkable.
Of the following, the MOST appropriate immunization to administer to this child today is
B. inactivated poliovirus
D. pneumococcal conjugate
Correct answer is C
PREP Pearl(s)
Live virus vaccines should be given at least 4 weeks and inactivated vaccines at least 2 weeks before
kidney transplant.
Live virus vaccines are contraindicated after kidney transplant; the child will receive long-term
immunosuppressive medication and will be at risk of experiencing a virulent vaccine strain that causes
severe illness.
Inactivated vaccines can be given 3 to 6 months after kidney transplant, except for the inactivated
influenza vaccine, which can be given as early as 1 month after transplant.
Critique
The child in the vignette, whose routine immunizations are up to date, should receive the measles, mumps,
and rubella (MMR) vaccine at this visit. The MMR vaccine is routinely recommended at 12 to 15 months and 4
to 6 years of age. Because it is a live virus vaccine, the MMR vaccine is contraindicated after transplantation.
Therefore, to enhance the child’s immunity, a second dose of the MMR vaccine should be administered at
least 4 weeks before the kidney transplant (as long as it has been at least 28 days since their first MMR dose).
Children and adolescents requiring kidney and other solid-organ transplants (eg, heart, liver, and lung)
should receive all age-appropriate immunizations before receiving the transplant. Live virus vaccines are
contraindicated after transplantation owing to the risk of a virulent vaccine strain causing severe illness in a
child with immunosuppression. If indicated, live virus vaccines (eg, MMR, varicella, and live attenuated
influenza virus) should be given at least 4 weeks before the transplant. In cases of deceased donor
transplant, because the timing of transplantation is not planned, the child is made “inactive” on the
transplant list for at least 4 weeks after immunization with a live virus vaccine.
Inactivated vaccines, if indicated, should be given at least 2 weeks before transplantation. The child in the
vignette is up to date on their primary series of Haemophilus influenzae type b, inactivated poliovirus, and
pneumococcal conjugate vaccine; thus, additional doses are not indicated at this time. Pneumococcal
polysaccharide vaccine is indicated for transplant candidates older than 2 years for protection against
additional pneumococcal strains. Inactivated virus vaccines can be given 3 to 6 months after transplantation;
early administration leads to poor antibody response due to intense immunosuppression. The only exception
is inactivated influenza vaccine, which can be given as early as 1 month after transplant and then annually.
Children who are not up to date with their immunizations should receive catch-up vaccinations before the
transplant surgery. After transplantation, primary care providers should work closely with a transplant
specialist to ensure that immunosuppressed children continue to receive age-appropriate inactivated
vaccines.
Suggested Reading(s)
American Academy of Pediatrics. Immunization and other considerations in immunocompromised
children. In: Kimberlin DW, Barnett ED, Lynfield R, Sawyer MH, eds. Red Book: 2021–2024 Report of the
Committee on Infectious Diseases, Committee on Infectious Diseases. 32nd ed. American Academy of
Pediatrics; 2021. Accessed September 1, 2023. Red Book Online
Katz DT, Torres NS, Chatani B, et al. Care of pediatric solid organ transplant recipients: an overview for
primary care providers. Pediatrics. 2020;146(6):e20200696. doi:10.1542/peds.2020-0696
Spinner JA, Denfield SW. Immunosuppressant drugs and their effects on children undergoing solid
organ transplant. Pediatr Rev. 2022;43(2):71-86. doi:10.1542/pir.2020-000620
Content Domain
Renal
A 16-year-old boy is seen for follow-up after an emergency department visit 2 weeks earlier for right flank
pain and bright red urine with passage of clots. At that time he had no dysuria. His urinalysis revealed 3+
blood, no protein, 50 to 100 red blood cells/high-power field, and <5 white blood cells/high-power field. Renal
ultrasonography showed a 5-mm echogenic focus with acoustic shadowing in the right lower pole of the
kidney. The boy was treated with pain medication and intravenous fluids and discharged home after his pain
improved.
Today, he has no pain and his urine color is yellow. His physical examination findings, including vital signs,
are normal.
A. hypocalciuria
B. hypocitraturia
C. hypooxaluria
D. hypouricosuria
Correct answer is B
PREP Pearl(s)
Adolescents with nephrolithiasis usually have the classic symptom of flank pain; children younger than
age 5 years may be asymptomatic or have recurrent abdominal pain.
Renal ultrasonography is the first-line imaging modality to detect nephrolithiasis.
A 24-hour urine test for metabolic abnormalities predisposing to nephrolithiasis may reveal
hypercalciuria, hypocitraturia, hyperoxaluria, hyperuricosuria, or cystinuria.
Critique
The boy in the vignette has a history of flank pain, hematuria with clots, and renal ultrasonography findings
suggestive of nephrolithiasis. A 24-hour urine test for a metabolic abnormality may reveal one or more of the
common risk factors for nephrolithiasis including hypercalciuria, hypocitraturia, hyperoxaluria,
hyperuricosuria, or cystinuria. Of the response choices, hypocitraturia is the only finding associated with
renal stone formation.
Kidney stones are commonly composed of calcium oxalate, calcium phosphate, struvite, cysteine, or uric acid.
A renal stone, when retrieved, should be sent for composition analysis. Factors that increase the risk of stone
formation include low urine volume, increased solute excretion (eg, calcium, uric acid), a decreased level of
stone inhibitors (eg, citrate, magnesium), urinary tract infection, and renal structural abnormalities (eg, renal
cyst). A 24-hour urine collection should be obtained after an acute episode to identify treatable urinary
metabolic abnormalities such as hypercalciuria, hyperoxaluria, hypocitraturia, hyperuricosuria, or cystinuria.
Identification of a specific urinary metabolic abnormality allows for targeted interventions (eg, dietary,
medications) to prevent recurrence of nephrolithiasis.
The incidence of nephrolithiasis in children is increasing, most likely due to dietary changes and the rising
prevalence of obesity. Adolescents with nephrolithiasis usually demonstrate the classic symptoms of flank
pain or renal colic as the stone tries to pass through the ureterovesical junction. Children younger than age 5
years may be asymptomatic or experience recurrent abdominal pain. Other presenting features of
nephrolithiasis include gross hematuria, dysuria, urinary urgency, and urinary tract infection.
When evaluating a child with nephrolithiasis, practitioners should obtain a history that includes any urinary
tract anomalies (eg, obstruction, renal cyst), recurrent urinary tract infections (eg, Proteus, Klebsiella),
metabolic conditions (eg, malabsorption syndrome, ketogenic diet), medications (eg, sulfadiazine, indinavir),
and family history. The physical examination should include blood pressure measurement, growth
parameters, and a thorough abdominal examination. It is important to assess for a mass that could result
from a urinary obstruction. The initial laboratory evaluation should include urinalysis with microscopic
examination for the presence of crystals; urine culture to rule out urinary tract infection; urine calcium-to-
creatinine ratio; and renal function tests.
Renal ultrasonography is the first-line imaging modality recommended to detect nephrolithiasis, which
appears as a focal area of echogenicity with acoustic shadowing (Figure). However, renal ultrasonography
may not detect small stones or ureteral stones. A plain abdominal radiograph detects radiopaque stones.
Noncontrast helical computed tomography is the most sensitive test for detection of nephrolithiasis.
Figure. Right kidney (RK) showing hyperechoic calculi (orange arrow) in the renal pelvis with posterior
acoustic shadowing (green arrows).
The acute management of nephrolithiasis includes pain control, aggressive fluid administration, and
facilitating passage or removal of the stone. Identification of the stone’s composition and risk factors for
stone recurrence facilitates long-term management with targeted dietary intervention and medications. For
children with hypercalciuria, reduced sodium intake and a thiazide diuretic may be indicated. Moderate
dietary oxalate restriction and avoidance of excess vitamin C intake are indicated for the management of
hyperoxaluria. Citrate supplementation is indicated for those with hypocitraturia to decrease the risk of stone
formation.
Suggested Reading(s)
Gellin CE. Urinary tract stones. Pediatr Rev. 2019;40(3):154-156. doi:10.1542/pir.2017-0235
McKay CP. Renal stone disease. Pediatr Rev. 2010;31(5):179-188. doi:10.1542/pir.31-5-179
Miah T, Kamat D. Pediatric nephrolithiasis: a review. Pediatr Ann. 2017;46(6):e242-e244.
doi:10.3928/19382359-20170517-02
Content Domain
Renal
A 5-year-old boy is brought to the emergency room for evaluation of body swelling and weight gain over the
past 3 days. The swelling started over his eyelids and now involves his legs. He has normal urine output.
There is no change in urine color, dysuria, fever, or rash. His vital signs are a blood pressure of 105/65 mm
Hg, a heart rate of 92 beats/min, and a respiratory rate of 16 breaths/min.
The boy’s weight is at the 85th percentile and height is at the 50th percentile for age. He is alert.
He has swelling of the upper eyelids, abdominal distension, and pitting edema of the lower legs. The
remainder of the boy’s physical examination findings are unremarkable.
Urine
pH 5.7
Protein 4+
Blood Negative
D. water retention
Correct answer is D
PREP Pearl(s)
Nephrotic syndrome is characterized by heavy proteinuria (urine protein >40 mg/m2/hr or urine-
protein-to-creatinine ratio of >2 mg/mg), hypoalbuminemia (serum albumin <2.5 g/dL [25 g/L]), edema,
and hyperlipidemia.
Children with nephrotic syndrome have mild hyponatremia because of water retention.
The glomerular filtration rate (indicated by blood urea nitrogen and serum creatinine concentrations)
is usually normal in nephrotic syndrome.
Critique
The boy in the vignette has nephrotic syndrome (NS), evidenced by his eyelid and leg edema,
hypoalbuminemia, and 4+ proteinuria. His hyponatremia is dilutional secondary to water retention. The low
effective circulatory volume secondary to hypoalbuminemia in NS causes a decrease in urine sodium.
Nephrotic syndrome usually presents with periorbital edema and a progressive increase in swelling that
leads to pedal edema, ascites, pleural effusion, vulvar or scrotal edema, and anasarca. The typical age of NS
onset is between 2 and 10 years. Most cases of NS in children are idiopathic (primary), but some have
secondary causes (underlying systemic disease such as vasculitis or systemic lupus erythematosus). Minimal
change disease is the most common histology in children with idiopathic NS.
Laboratory evaluation of suspected NS typically begins with a urinalysis that shows proteinuria. Microscopic
hematuria (3-5 red blood cells/high-power field) occurs in 25% of children with idiopathic NS. The serum
albumin concentration is low because of increased urinary losses. The serum cholesterol concentration is
elevated because of low oncotic pressure from loss of serum albumin, increased synthesis, and decreased
degradation of products in the cholesterol pathway. The serum calcium concentration may be low in NS as a
result of hypoalbuminemia. However, ionized calcium concentrations are normal.
Children with NS have mild dilutional hyponatremia because of water retention, which is a result of lower
effective intravascular volume from hypoalbuminemia in addition to inappropriate secretion of antidiuretic
hormone. A normal amount of water intake causes edema and hyponatremia in NS. Hyponatremia can also
result from diuretic therapy used in the treatment of NS. Hyponatremia in NS is not due to the urinary loss of
sodium. Urine sodium excretion is decreased due to low effective intravascular volume. The hyponatremia in
NS is true hyponatremia and is not a factitious or pseudohyponatremia.
The glomerular filtration rate (indicated by blood urea nitrogen and serum creatinine concentrations) is
usually normal in NS, as seen with the child in the vignette. Mildly increased blood urea nitrogen and serum
creatinine concentrations may occur in NS owing to decreased effective circulatory blood volume, but this is
not the cause of hyponatremia in NS. A decrease in glomerular filtration rate (azotemia), hypertension, and
macroscopic hematuria are commonly seen in nephritic syndrome or acute glomerulonephritis.
Prednisone is the first-line treatment for idiopathic NS. Hyponatremia in NS is treated with water and sodium
restriction and by addressing the hypoalbuminemia.
Suggested Reading(s)
Rodriguez-Ballestas E, Reid-Adam J. Nephrotic syndrome. Pediatr Rev. 2022;43(2):87-99.
doi:10.1542/pir.2020-001230
Varade WS. Nephrotic syndrome. In: McInerny TK, Adam HM, Campbell DE, DeWitt TG, Foy JM, Kamat
DM, eds. American Academy of Pediatrics Textbook of Pediatric Care. 2nd ed. American Academy of
Pediatrics; 2016:chap 295. Accessed September 1, 2023. Pediatric Care Online
Content Domain
Renal
A 6-year-old boy is seen in the office for evaluation of a rash over his lower extremities of 7 days’ duration
and mild, diffuse, colicky abdominal pain of 2 days’ duration. He has had no diarrhea or vomiting. The boy’s
blood pressure is 120/80 mm Hg, his heart rate is 80 beats/min, and his respiratory rate is 14 breaths/min.
He has a nonblanching, erythematous, papular rash over the posterior aspects of his bilateral lower limbs
(Figure). There is mild pitting edema of his lower extremities without joint swelling. His abdomen is soft, with
mild diffuse tenderness on palpation. The remainder of the boy’s examination findings are normal.
Reprinted with permission from Quddusi FI, Youssef MJ, Davis DMR. Pediatr Rev. 2021;42(12):662.
Blood 3+
Urine Test Result
Protein 3+
Correct answer is D
PREP Pearl(s)
Signs and symptoms of Henoch-Schönlein purpura, the most common vasculitis affecting children,
may include palpable purpura over the lower extremities and buttocks, diffuse abdominal pain,
arthritis, hematuria, and proteinuria.
Children with Henoch-Schönlein purpura and mild nephritis (microscopic hematuria, non–nephrotic-
range proteinuria) can be closely monitored.
Children with Henoch-Schönlein purpura and macroscopic hematuria, worsening proteinuria,
nephrotic syndrome, or acute kidney injury should be referred to a pediatric nephrologist for a kidney
biopsy. Individuals with nephrotic-range proteinuria, nephrotic syndrome, or acute kidney injury are at
risk of progressing to chronic kidney disease and end-stage kidney disease.
Critique
The child in the vignette has Henoch-Schönlein purpura (HSP). This diagnosis is supported by the presence of
palpable purpura, abdominal pain, and renal involvement (hematuria, proteinuria, edema, and
hypertension). The best next step is to refer him to a pediatric nephrologist for a kidney biopsy before
starting treatment with prednisone.
Henoch-Schönlein purpura is the most common vasculitis in children, with a peak incidence at age 4 to 6
years. Inflammation of the small vessels of the skin, joints, gastrointestinal tract, and kidneys leads to its
classical manifestations. The skin findings of HSP include asymptomatic, erythematous, edematous papules
(palpable purpura) over the lower extremities and buttocks. The face, trunk, and forearms may be involved in
infants and young children. These skin lesions tend to occur in crops, last up to 10 days, may recur in the first
3 months, and resolve without scarring. Skin biopsy shows leukocytoclastic vasculitis with IgA immune
complex deposition. However, skin biopsy is not routinely performed; it is indicated only when the diagnosis
is suspected but the rash or clinical presentation is atypical. The child in the vignette has typical skin
manifestations of HSP, so referral to a dermatologist for skin biopsy is not indicated.
Arthritis occurs in 60% to 80% of children with HSP; it presents as swelling and tenderness of the ankle, wrist,
knee, finger joints, or a combination of these. Gastrointestinal tract manifestations, occurring in 50% to 70%
of HSP cases, may include intermittent colicky abdominal pain, vomiting, and blood in the stool. Common
complications of HSP are intestinal ischemia, perforation, and small bowel intussusception.
Kidney involvement occurs in 50% to 70% of children with HSP. A urinalysis should be performed at the time
of presentation, repeated weekly or biweekly for the first 2 months, and then monthly for 6 months to
monitor for hematuria and proteinuria. Children with HSP may have microscopic hematuria, macroscopic
hematuria, proteinuria, nephrotic syndrome, or rapidly progressive acute glomerulonephritis. Kidney
function tests and complement levels should be obtained in children with hematuria, proteinuria, and
hypertension.
Henoch-Schönlein purpura is a self-limiting illness in the majority of affected children. The symptoms are
treated according to their severity. Mild arthritis is managed conservatively, whereas severe arthritis may
require treatment with nonsteroidal anti-inflammatory drugs, steroids, or a combination of these. However,
nonsteroidal anti-inflammatory drugs should be used cautiously in children with renal involvement to avoid
worsening of acute kidney injury. The child in the vignette does not have joint manifestations, so a
prescription for ibuprofen is not indicated.
Children with mild abdominal pain may be treated supportively. Children with severe abdominal pain and
gastrointestinal bleeding are usually treated with steroids for 2 to 4 weeks. The child in the vignette has mild
abdominal pain, so treatment with prednisone is not indicated.
Children with mild HSP nephritis (microscopic hematuria, non-nephrotic–range proteinuria) can be closely
monitored. Those with macroscopic hematuria, worsening proteinuria, nephrotic syndrome, or acute kidney
injury should be referred to a pediatric nephrologist for a kidney biopsy, which will confirm the diagnosis of
IgA nephropathy, reveal the severity of kidney involvement, and guide treatment with prednisone or other
immunosuppressive medication. The kidney biopsy should be performed before starting prednisone
treatment so as to avoid masking the findings. The duration of prednisone treatment for HSP nephritis is up
to 12 weeks to avoid chronic kidney damage. Severe HSP nephritis (including crescentic glomerulonephritis) is
managed aggressively with intravenous steroids, cyclophosphamide, intravenous immunoglobulin,
plasmapheresis, or a combination of these.
Children with mild HSP have an excellent prognosis. Microscopic hematuria resolves spontaneously. The
long-term prognosis depends on the severity of kidney involvement. Children with HSP and nephrotic-range
proteinuria, nephrotic syndrome, or acute kidney injury are at risk of progressing to chronic kidney disease
and end-stage kidney disease.
Suggested Reading(s)
Adrogue HE, Hayde NA. Henoch-Schönlein purpura. In: McInerny TK, Adam HM, Campbell DE, DeWitt
TG, Foy JM, Kamat DM, eds. American Academy of Pediatrics Textbook of Pediatric Care. 2nd ed.
American Academy of Pediatrics; 2016:chap 264. Accessed September 1, 2023. Pediatric Care Online
Kamat DM. Henoch-Schonlein purpura. Henoch-Schönlein purpura. Point-of-Care. Quick Reference.
American Academy of Pediatrics; 2020. Accessed February 23, 2023. Pediatric Care Online
Quddusi FI, Youssef MJ, Davis DMR. Dermatologic manifestations of systemic diseases in childhood.
Pediatr Rev. 2021;42(12):655-671. doi:10.1542/pir.2020-000679
Reid-Adam J. Henoch-Schönlein purpura. Pediatr Rev. 2014;35(10):447-449 doi:10.1542/pir.35-10-447
Content Domain
Renal
A 14-year-old boy with type 1 diabetes mellitus is brought to the emergency department having experienced
2 days of abdominal pain, vomiting, and fatigue. He has had decreased urine output for 1 day. He has no
fever, cough, diarrhea, or upper respiratory infection symptoms. He was diagnosed with diabetes at age 9
years and currently takes insulin glargine subcutaneously once daily and insulin lispro subcutaneously with
meals.
On physical examination, the boy’s weight is at the 5th percentile and height is at the 10th percentile for age.
His blood pressure is 100/70 mm Hg, heart rate is 102 beats/min, and respiratory rate is 20 breaths/min. He
has dry mucous membranes, and his capillary refill time is 3 seconds. His abdomen is soft and nontender.
The remainder of the boy’s examination findings are unremarkable.
Laboratory results are shown:
32 mg/dL
β-Hydroxybutyrate
(reference range, 0.2-2.8 mg/dL)
Urine
pH 5.5
Ketones Present
Blood Negative
Protein Negative
Laboratory Test Result
Of the following, the additional laboratory finding MOST likely to be seen in this adolescent is
Correct answer is A
PREP Pearl(s)
The etiology of acute kidney injury can be divided into prerenal (decreased effective renal blood flow),
renal (intrinsic renal damage), and postrenal (obstruction to the flow of urine).
The fractional excretion of sodium is less than 1% in prerenal acute kidney injury.
The initial step in the management of prerenal acute kidney injury is to restore intravascular volume
using a 10 to 20 mL/kg intravenous bolus of 0.9% saline solution.
Critique
The adolescent in the vignette has diabetic ketoacidosis (DKA) (hyperglycemia, metabolic acidosis, high serum
β-hydroxybutyrate concentration, and ketonuria) and acute kidney injury (AKI) (elevated blood urea nitrogen
and serum creatinine concentrations) secondary to dehydration from osmotic diuresis and vomiting. He most
likely has prerenal AKI (urinalysis with no blood, protein, or casts; high urine specific gravity; and blood urea
nitrogen–to–serum creatinine ratio >20). In the setting of prerenal AKI, his fractional excretion of sodium
would be <1%, indicating reabsorption of the filtered sodium and water in response to renal hypoperfusion.
In acute kidney injury, there is a decrease in urine output or an increase in serum creatinine, leading to a
decrease in glomerular filtration rate. The etiology of AKI (Table) can be divided into the following categories:
Findings on urinalysis with microscopic examination and urinary indices (fractional excretion of sodium, renal
failure index) are useful to differentiate the cause of AKI. Prerenal AKI is associated with a normal urinalysis
result, with no blood, protein, cells, or casts. The presence of granular or epithelial cell casts on urinalysis is
suggestive of acute tubular necrosis and red blood cell casts are associated with acute glomerulonephritis;
both conditions are renal causes of AKI. A blood urea nitrogen–to–serum creatinine ratio <20 and a urine-to-
serum osmolality ratio <1.5 support a diagnosis of renal AKI.
The fractional excretion of sodium (FENa) differentiates prerenal AKI from renal AKI and is calculated by using
the following formula:
(urine sodium × serum creatinine) ÷ (serum sodium × urine creatinine) × 100
A FENa less than 1% is suggestive of prerenal AKI, and a FENa of greater than 2% favors renal AKI (eg, acute
tubular necrosis). The fractional excretion of urea (FEUrea) is also used to differentiate prerenal AKI from
renal AKI. The FEUrea is a more accurate test than FENa in children who are receiving diuretic therapy. A
FEUrea of less than 35% is suggestive of prerenal AKI and a FEUrea greater than 50% is seen in renal AKI. The
Figure defines an algorithm to differentiate the types of AKI and their initial management.
Reprinted with permission from Jain A, Mattoo TK. Oliguria and anuria. In: McInerny TK, Adam HM,
Campbell DE, DeWitt TG, Foy JM, Kamat DM, eds. American Academy of Pediatrics Textbook of Pediatric
Care. 2nd ed. American Academy of Pediatrics; 2017:chap 301.
Figure. Algorithm for determination of the type and management of acute kidney injury.
The management of DKA with its associated dehydration and prerenal AKI is challenging. The goal is to
correct the dehydration and acidosis with simultaneous slow correction of hyperglycemia to minimize the risk
of cerebral edema. The initial step is to restore intravascular volume using an intravenous isotonic crystalloid
fluid bolus (eg, 10-20 mL/kg of 0.9% saline). This fluid is administered before insulin therapy begins. The need
for further fluid boluses is determined according to the child’s vital signs, urine output, and fluid status.
Maintenance and deficit fluid calculations should be performed. The goal is to correct the fluid deficit over 24
to 48 hours using 0.45% to 0.9% saline solution.
After the initial fluid resuscitation, a continuous insulin infusion should be started, with dextrose added to the
intravenous fluid to maintain stable serum glucose concentrations. Bicarbonate administration is not
routinely recommended for DKA management. Laboratory data (eg, glucose, sodium, potassium
concentrations) should be carefully monitored. Correction of the fluid deficit and hyperglycemia improves the
DKA and AKI. Once the acidosis is resolved and the child is tolerating oral intake, transition to subcutaneous
insulin is indicated.
Suggested Reading(s)
Cashen K, Petersen T. Diabetic ketoacidosis. Pediatr Rev. 2019;40(8):412-420. doi:10.1542/pir.2018-
0231
Glaser N, Fritsch M, Priyambada L, et al. ISPAD clinical practice consensus guidelines 2022: diabetic
ketoacidosis and hyperglycemic hyperosmolar state. Pediatr Diabetes. 2022;23(7):835-856.
doi:10.1111/pedi.13406
Jain A, Mattoo TK. Oliguria and anuria. In: McInerny TK, Adam HM, Campbell DE, DeWitt TG, Foy JM,
Kamat DM, eds. American Academy of Pediatrics Textbook of Pediatric Care. 2nd ed. American
Academy of Pediatrics; 2017:chap 301. Accessed September 1, 2022. Pediatric Care Online
Selewski DT, Symons JM. Acute kidney injury. Pediatr Rev. 2014;35(1):30-41. doi:10.1542/pir.35-1-30
Content Domain
Renal
A 17-year-old girl is evaluated for an increase in painful menstrual bleeding. Her menarche was at age 14
years. She reports having monthly cycles lasting 5 to 7 days until 3 months ago, when she bled for 14 days in
a row. Since then, her menses has occurred every 2 weeks. She describes crampy abdominal and back pain
with every menses, causing her to miss multiple school days. She is currently menstruating. The adolescent
has one male sexual partner and reports 100% condom use. She denies vaginal discharge or dysuria. She has
no past medical problems, takes no medications, and has no family history of clotting disorders. Physical
examination reveals normal vital signs for age and no pain with abdominal palpation.
B. perform a urine pregnancy test and test for sexually transmitted infection
C. prescribe hormonal treatment with a combined oral contraceptive pill, patch, or ring
Correct answer is B
PREP Pearl(s)
Most adolescents with dysmenorrhea have no pelvic pathology (primary dysmenorrhea) and can be
treated empirically with nonsteroidal anti-inflammatory drugs at appropriate doses.
Secondary dysmenorrhea requires further evaluation, which may include testing for sexually
transmitted infection, pelvic ultrasonography, or referral for evaluation for endometriosis.
Critique
The adolescent girl in the vignette has dysmenorrhea (painful menstruation) accompanied by a recent change
in her menstrual bleeding pattern; this presentation meets criteria for abnormal uterine bleeding (AUB).
According to the International Federation of Gynecology and Obstetrics criteria, this adolescent’s menstrual
bleeding pattern can be categorized as frequent (more than 4 episodes in a 90-day period) and prolonged
(menstrual periods that exceed 8 days on a regular basis). The Table lists and defines the categories of AUB
according to the International Federation of Gynecology and Obstetrics classification system. Painful menses
in the setting of AUB requires testing for causes of secondary dysmenorrhea, including pregnancy and
sexually transmitted infection (STI).
Primary dysmenorrhea, painful menstruation with no pelvic pathology or abnormality, can be treated
empirically with appropriate doses of nonsteroidal anti-inflammatory drugs (NSAIDs). If NSAIDs do not
provide adequate pain relief, hormonal agents such as combined hormonal contraceptive options (eg, pill,
patch, or ring) have been shown to be beneficial in the treatment of dysmenorrhea. The adolescent in the
vignette has secondary dysmenorrhea, which requires further evaluation; therefore, treatment with NSAIDs
or hormonal agents is not the best next management step. Pelvic ultrasonography is an appropriate step in
evaluating secondary dysmenorrhea, but it is not the best initial option for this girl given her history of sexual
activity and rapid need to determine pregnancy and STI status.
Most adolescents who experience dysmenorrhea have primary dysmenorrhea. Dysmenorrhea occurs in up
to 90% of adolescent girls and is the most common cause of short-term school absences. Most adolescents
with primary dysmenorrhea respond to treatment with NSAIDs or hormonal suppression. If painful menses
persists after 3 to 6 months of appropriate treatment, further evaluation for causes of secondary
dysmenorrhea should be performed (see ACOG Committee Opinion No. 760).
For secondary dysmenorrhea, pelvic examination is indicated to evaluate for cervicitis or pelvic inflammatory
disease, especially in the presence of suprapubic abdominal pain elicited by palpation, vaginal discharge, or
strong suspicion of STI. Pelvic ultrasonography is indicated in most cases of secondary dysmenorrhea.
Pregnancy and STI testing should always be performed in the sexually active adolescent. Endometriosis is the
leading cause of secondary dysmenorrhea in adolescents. Referral to a gynecologist for diagnostic
laparoscopy is indicated if endometriosis is suspected.
Suggested Reading(s)
ACOG Committee Opinion No. 760: dysmenorrhea and endometriosis in the adolescent. Obstet
Gynecol. 2018;132(6):e249-e258. doi:10.1097/AOG.0000000000002978
Dinerman LM. Dysmenorrhea. In: McInerny TK, Adam HM, Campbell DE, DeWitt TG, Foy JM, Kamat DM,
eds. American Academy of Pediatrics Textbook of Pediatric Care. American Academy of Pediatrics;
2023. Accessed September 1, 2023. Pediatric Care Online
Kho KA, Shields JK. Diagnosis and management of primary dysmenorrhea. JAMA. 2020;323(3):268-269.
doi:10.1001/jama.2019.16921
Content Domain
Reproductive Health
The correct answer is: perform a urine pregnancy test and test for sexually transmitted infection
Disorders in regularity
• ●Irregular menstrual bleeding (variation; >20 days over a period of one year)
• ●Absent menstrual bleeding (amenorrhea; no bleeding in a 90 day period)
Disorders in frequency
• ●Infrequent menstrual bleeding (oligomenorrhea; one or two episodes in a
90-day period)
• ●Frequent menstrual bleeding (more than four episodes in a 90-day period)
Disorders in amount of flow
• ●Heavy menstrual bleeding (excessive blood loss which interferes with the woman’s
physical, emotional, social and material quality of life and which can occur alone
or with other symptoms)
• ●Heavy and prolonged menstrual bleeding (excessive blood loss exceeding
eight days)
• ●Light menstrual bleeding (bleeding less than 5 mL in a period).
Disorders of duration of flow
• ●Prolonged menstrual bleeding (menstrual periods that exceed eight days on a
regular basis.)
• ●Shortened menstrual bleeding (menstrual bleeding lasting less than two days)
A 16-year-old adolescent is seen at the county youth detention center for an intake physical examination. She
moved from another state 1 month ago to be near a boyfriend she met online. A review of systems is positive
for one week of mild, crampy abdominal pain, light menstrual bleeding, and a white vaginal discharge.
Menarche was at age 13 years. She had regular monthly menses until her last normal period 6 weeks ago.
She reports 1 male sex partner in the past 6 months.
On physical examination, the adolescent’s vital signs are normal. Her abdomen is soft and mildly tender to
palpation in the left lower quadrant without rebound or guarding. Pelvic examination is significant for a
scant, bloody, nonodorous vaginal discharge originating from the cervical os. There is no cervical or adnexal
tenderness. The remainder of her physical examination findings are unremarkable.
Swabs are obtained for sexual transmitted infection testing. A urine pregnancy test result is positive.
Correct answer is D
PREP Pearl(s)
Pregnant adolescents carry a high risk of adverse health outcomes for both adolescent and child.
Long-acting reversible contraception is a highly effective method of preventing adolescent pregnancy.
Pregnant individuals with first-trimester menstrual bleeding must undergo pelvic or transvaginal
ultrasonography to evaluate for a ruptured ectopic pregnancy, a life-threatening condition.
Critique
The adolescent in the vignette is pregnant and experiencing first-trimester menstrual bleeding, a common
phenomenon (occurring in 20%-40% of pregnancies) with various causes. Bleeding due to early loss or
threatened miscarriage occurs in 15% to 20% of pregnancies. Ectopic pregnancy is a much less common
cause of early pregnancy bleeding (2%), but rupture of an ectopic pregnancy is life threatening and the most
serious early pregnancy complication. It is imperative to exclude ectopic pregnancy in every case of early
pregnancy menstrual bleeding.
The best next step in this adolescent’s treatment is pelvic ultrasonography to evaluate the location of the
pregnancy and rule out an ectopic pregnancy. Handheld Doppler ultrasonography can be used after 10 to 12
weeks of gestation to evaluate for fetal heart tones; this would not be helpful in ruling out an ectopic
pregnancy. Implantation bleeding is a possible cause of this adolescent’s symptoms but is a diagnosis of
exclusion. Whereas serial measurements of human chorionic gonadotropin levels may be useful in the first 6
weeks of pregnancy to evaluate for early pregnancy loss, they are not the best next management step; the
results must be correlated with ultrasonography findings.
Other causes of first-trimester pregnancy bleeding include cervical, vaginal, or uterine pathology (eg, sexually
transmitted infection [STI], cervicitis, vaginal laceration, or polyps). Pelvic examination with a speculum is
required in any patient with first-trimester bleeding to evaluate for these causes. Given her physical
examination findings (abdominal tenderness, bloody vaginal discharge), the adolescent in the vignette should
undergo full STI testing, including for HIV and syphilis.
Despite an overall decline in the teen birth rate in the United States over the last 30 years, the rate is higher
than that in other developed countries. Within the United States, birth rates differ according to region of the
country and socioeconomic status (SES). The majority (80%) of pregnant adolescents aged 15 to 19 years
report that their pregnancy was unintended. Most adolescents at risk of becoming pregnant report using
some form of contraception, but a much lower proportion report using highly effective contraception such as
long-acting reversible contraception (eg, intrauterine devices and the subcutaneous implant). Long-acting
reversible contraception has been shown to decrease unintended pregnancies in adolescents. Risk factors for
repeat teen pregnancy include depression, history of abortion, and low SES. Protective factors against repeat
pregnancy include use of long-acting reversible contraception and attainment of a higher education level.
On establishing a pregnancy, the physician should spend time eliciting the pregnant individual’s thoughts and
feelings regarding the pregnancy, assisting with notification of her support system if necessary, and providing
resources for counseling and prenatal care. The physician should also provide a prescription for a folic acid–
containing prenatal vitamin and counseling about abstaining from and/or cessation of use of alcohol and
other substances. Adolescents carry an increased risk of adverse pregnancy outcomes (eg, preterm birth and
fetal death) for unclear reasons; this may be related to SES or biological immaturity.
The adolescent in the vignette has multiple risk factors for adverse pregnancy outcomes, including her social
setting (detention center), young age, and cause for concern about human trafficking (moved to the area as a
minor to be near a “boyfriend”). Social services and mental health support systems are integral aspects of her
care. Providers should be aware of all factors affecting pregnant and recently pregnant adolescents to
provide them with appropriate health care, social, and psychological support systems.
Suggested Reading(s)
Elfenbein DS, Felice ME. Adolescent pregnancy and parenthood. In: McInerny TK, Adam HM, Campbell
DE, Foy JM, Kamat DM, eds. American Academy of Pediatrics Textbook of Pediatric Care. 2nd ed.
American Academy of Pediatrics; 2023. Accessed September 1, 2023. Pediatric Care Online
Hornberger LL; Committee on Adolescence. Options counseling for the pregnant adolescent patient.
Pediatrics. 2017;140(3):e20172274. doi:10.1542/peds.2017-2274
Trends in teen pregnancy and childbearing. US Department of Health and Human Services, Office of
Population Affairs. Accessed September 1, 2023. https://opa.hhs.gov/adolescent-health/reproductive-
health-and-teen-pregnancy/trends-teen-pregnancy-and-childbearing
Content Domain
Reproductive health
A previously healthy 15-year-old with no surgical history is seen in the emergency department for 2 days of
emesis that started with acute onset of abdominal pain. She is not experiencing diarrhea. She had menarche
at 11 years of age. Her periods have always been irregular, occurring every 1 to 3 months. Her last period
was 6 weeks ago. She denies illicit substance use or sexual activity. Her vital signs are a temperature of 37.7
°C, heart rate of 110 beats/min, respiratory rate of 20 breaths/min, and blood pressure of 110/70 mm Hg.
Her abdomen is nondistended, and there is right lower quadrant tenderness. The remainder of her
examination findings are normal. Abdominal ultrasonography shows a normal appendix. She is taken to the
operating room for an exploratory laparoscopy, which reveals an ovarian torsion.
Of the following, the MOST likely underlying condition that led to this girl’s intraoperative findings is a/an
B. ovarian malignancy
Correct answer is A
PREP Pearl(s)
Most cases of ovarian torsion in the pediatric population have an underlying cystic pathology of the
ovary.
The clinical presentation of ovarian torsion in children is nonspecific; the diagnosis usually requires
exploratory laparoscopy.
Conservative management of adnexal torsion, without oophorectomy, is the best strategy; in most
cases, ovarian function is preserved
Critique
Up to 25% of pediatric ovarian torsion occurs with a previously normal ovary; however, in most cases there is
an underlying ovarian pathology. In approximately 51% to 84% of cases, cystic structures are identified (eg,
hemorrhagic cysts, cystic teratomas, and dermoid, paraovarian, or tubal cysts). The risk of torsion is greater
when an ovarian mass is benign or is 5 cm or larger. However, females with enlarged ovaries due to
polycystic ovarian syndrome rarely experience torsion. The median age for children to present with ovarian
torsion is 11 years.
Ovarian torsion is estimated to occur in 4.9 per 100,000 or approximately 3% of female children with acute
abdominal pathology. Children and adolescents account for 15% of ovarian torsion in all age groups. The
nonspecific presentation in children makes diagnosis challenging. Most children present with acute onset of
abdominal pain, often accompanied by emesis. The pain is often felt in the right lower abdomen, making it
difficult to distinguish from appendicitis.
Isolated ovarian torsion refers to a torsion of the ovary alone, with twisting on the mesovarium. Adnexal
torsion refers to torsion of both the fallopian tube and ovary, which is the more common occurrence,
constituting approximately two-thirds of all torsions. Torsion is more common on the right due to the
increased mobility of the ilium and cecum. Similarly, the attaching ligament on the right side is slightly longer
than on the left, increasing the risk for torsion.
No laboratory test is reliably diagnostic for ovarian torsion; a complete blood cell count may show a slight
leukocytosis. When considering the diagnosis of appendicitis or ovarian torsion in a postpubertal girl, it is
important to perform a pregnancy test because ectopic pregnancy must be considered in the differential
diagnosis. Doppler ultrasonography is the most appropriate imaging study to evaluate for ovarian torsion.
Findings may include a unilateral enlarged ovary, a heterogeneous appearance of the ovary (due to edema),
displacement of an ovary or other pelvic structures, and/or the whirlpool sign (twisting of the ovarian pedicle,
resulting in the twisting of associated blood vessels). Although the absence of vascular flow on Doppler
ultrasonography is highly suggestive of torsion, the sensitivity of this imaging modality is as low as 40% to
70%.
In the pediatric population, laparoscopic surgery remains the best diagnostic and therapeutic approach when
ovarian torsion is suspected. Prompt surgical exploration is necessary to prevent irreversible adnexal
damage. Pain lasting more than 10 hours is associated with an increased risk of necrotic tissue. In the past,
surgeons would perform an oophorectomy if an ovary appeared necrotic; however, it has become apparent
that even overtly necrotic ovaries may recover after detorsion. Ovarian malignancy is very rare in children
and is an uncommon indication for ovary removal.
Uncertainty remains regarding the clinical outcomes after surgery for ovarian torsion. Some studies suggest
that removal of a single ovary does not affect future fertility, whereas others show a negative impact. Studies
of girls treated conservatively, without oophorectomy, show very high rates of normal follicular function.
Therefore, in the pediatric population, conservative laparoscopic management of adnexal torsion is the
preferred strategy and in most cases preserves ovarian function.
Suggested Reading(s)
Bolli P, Schädelin S, Holland-Cunz S, Zimmermann P. Ovarian torsion in children: development of a
predictive score. Medicine (Baltimore). 2017;96(43):e8299. doi:10.1097/MD.0000000000008299
Childress KJ, Dietrich JE. Pediatric ovarian torsion. Surg Clin North Am. 2016;97(1):209-222.
doi:10.1016/j.suc.2016.08.008
Content Domain
Reproductive Health
A 2-month-old boy is seen for a routine health supervision visit. He has a unilateral undescended testis that
was noted at birth. He is otherwise healthy. His growth and development are normal. His right testis is not
palpable in the scrotal sac or the inguinal canal. His right hemiscrotum is underdeveloped. His left testis is
easily palpable in the appropriately-developed left hemiscrotum. His penis length is normal, and the urethral
meatus is normally located. The remainder of the boy’s physical examination findings are normal.
Of the following, this infant is at HIGHEST risk of developing
A. adrenal crisis
B. malignancy
C. short stature
Correct answer is B
PREP Pearl(s)
The inability to palpate a testis in the scrotum may be due to cryptorchidism, ectopic location, or
retraction or absence of the testis.
The 2 major risks associated with cryptorchidism are testicular cancer and infertility. Orchiopexy
performed before puberty decreases the risk of malignancy; however, the risk remains higher
throughout life than in those without cryptorchidism.
Infants with cryptorchidism should be referred to a urologist by age 6 to 9 months; orchiopexy should
be performed by age 18 months.
Critique
The infant in the vignette has unilateral cryptorchidism. Of the response choices, he is at highest risk of
developing a testicular malignancy. The risk of testicular cancer is 2 to 8 times higher in boys with
cryptorchidism. Although orchiopexy performed before puberty decreases the risk of malignancy, the risk
remains higher throughout life than in those without cryptorchidism. Orchiopexy allows for easier
examination and surveillance because the testis is located in the scrotum.
Infertility is the other major risk associated with cryptorchidism. Orchiopexy is recommended by age 18
months to preserve the full fertility potential. Higher rates of germ cell loss and infertility occur the longer the
testis remains undescended.
When an undescended testis is associated with an inguinal hernia, there is a risk of acute strangulation.
Additionally, testicular torsion is more difficult to diagnose when it occurs in an undescended testis.
A testis may be nonpalpable because of arrest of descent (cryptorchidism), ectopic location, retraction into
the inguinal canal (retractile testis), or intrauterine torsion (vanishing testis). Arrest of descent can occur at
any location along the line of migration: intra-abdominal, the inguinal canal (internal and external rings), or
the upper pole of the scrotal sac. The most common location for a cryptorchid testis is the inguinal canal,
followed by the prescrotal location (distal to the external ring of the inguinal canal).
Cryptorchidism is the most common genital disorder identified at birth, with a prevalence of 2% to 3% in term
infants. Because testicular descent occurs during the 3rd trimester (typically by 36 weeks’ gestation), it has a
notably higher prevalence (20% to 30%) in premature infants. The etiology of cryptorchidism is multifactorial
and includes genetic predisposition, hormonal imbalances, and environmental factors. For relatives of a boy
with cryptorchidism, there is a 10-fold increased risk in twins and a 3-fold increased risk in brothers. Several
maternal factors (eg, obesity, smoking, diabetes) are associated with an increased risk of cryptorchidism. Low
birth weight, breech presentation, and estrogen exposure are also associated with cryptorchidism.
The diagnosis of cryptorchidism is made via careful physical examination. If a testis is not palpable,
techniques such as moving one hand along the inguinal canal and applying gentle pressure to close the
external ring while using the other hand to palpate for the testis in the scrotum may be helpful. A retractile
testis can be brought into the scrotal sac on physical examination, but the cremasteric reflex may cause it to
retract back into the canal. A testis may be palpated in an ectopic location (eg, perineum, femoral triangle,
contralateral scrotum, or superficial inguinal area). The presence of unilateral or bilateral cryptorchidism
should be noted, as well as any other genital abnormalities (eg, hypospadias and micropenis).
Spontaneous descent of a cryptorchid testis into the scrotal sac can occur up to 6 months after birth
(corrected for gestational age). The likelihood of descent after age 6 months is extremely low. Therefore,
infants should be referred to a urologist by age 6 to 9 months if both testes are not present in the scrotal sac.
Infants with a retractile testis should be examined closely at every health supervision visit; if the condition is
persistent, they should be referred to a urologist by age 6 to 9 months owing to the risk of ascent and
acquired cryptorchidism.
An infant with bilateral cryptorchidism or unilateral cryptorchidism in the presence of other abnormal genital
findings should be evaluated for a difference of sexual development (DSD). An infant with 46,XX
chromosomes and severe virilization due to congenital adrenal hyperplasia can be mistaken for a male with
bilateral cryptorchidism. Prompt recognition of a possible DSD is critical, because some DSDs are associated
with adrenal insufficiency. The infant in the vignette has unilateral cryptorchidism and otherwise normal
external genitalia, making a DSD and the associated risk of an adrenal crisis unlikely.
This infant has a normal penile length, making growth hormone deficiency with resulting short stature an
unlikely diagnosis. Growth hormone deficiency can be associated with micropenis. Urinary tract infections are
not associated with isolated cryptorchidism.
Suggested Reading(s)
Inouye B, Tourchi A, Gearhart JP. Hypospadias, epispadias, and cryptorchidism. In: McInerny TK, Adam
HM, Campbell DE, Foy JM, Kamat DM, eds. American Academy of Pediatrics Textbook of Pediatric Care.
2nd ed. American Academy of Pediatrics; 2016:chap 272. Accessed March 3, 2023. Pediatric Care
Online
Kolon TF, Herndon CDA, Baker LA, et al. Evaluation and treatment of cryptorchidism: AUA guideline. J
Urol. 2014;192(2):337-345. doi:10.1016/j.juro.2014.05.005
Wu WJ, Gitlin JS. The male genital system. Pediatr Rev. 2020;41(3):101-111. doi:10.1542/pir.2017-0316
Content Domain
Reproductive Health
A 13-year-old girl with no notable medical history is seen for a health supervision visit. She complains of
heavy, painful, and irregular menses. Menarche was at age 12 years. Her menses typically occur every 2
months and last 2 to 3 weeks. Her last menstrual period began 1 month ago and has continued daily since
then. The girl has missed school because of heavy bleeding and pain. Nonsteroidal anti-inflammatory
medications provide incomplete relief of her pain. She has never had sex. There is no known family history of
hormonal or bleeding disorders. The girl’s vital signs and physical examination findings are normal. Her body
mass index is at the 50th percentile for age and sex. Her sexual maturity rating is 4 for both breast
development and pubic hair. There is no evidence of hyperandrogenism.
On point-of-care testing, the adolescent’s hemoglobin level is 9.5 g/dL (95.0 g/L), and the result of a urine
pregnancy test is negative. A complete blood cell count with differential, iron studies, coagulation studies,
and thyroid function testing are ordered. Hormonal treatment options for abnormal uterine bleeding are
discussed with the girl and her family.
Of the following, the BEST additional laboratory tests to obtain today are
Correct answer is D
PREP Pearl(s)
Heavy menstrual bleeding since menarche, with or without a family or personal history of bleeding
disorders, should prompt an evaluation for von Willebrand disease.
Von Willebrand disease test results may be altered by acute illness, severe anemia, pregnancy,
estrogen exposure, and other physiologic stress. Abnormal results should prompt referral to a
hematologist; repeat testing should occur after clinical stabilization.
Critique
The girl in the vignette is experiencing abnormal uterine bleeding (AUB), defined as bleeding from the uterus
that is abnormal in frequency, duration, regularity, volume, or a combination of these. Her history of heavy
menstrual bleeding since menarche raises concern regarding a disorder of hemostasis. Von Willebrand
disease (VWD) affects 10% to 36% of women with AUB and is the most common disorder of hemostasis.
Therefore, the best additional laboratory tests to obtain are von Willebrand factor antigen and platelet-
dependent von Willebrand factor activity levels to screen for VWD. Factor VIII activity and ristocetin cofactor
are frequently included on combined VWD testing panels.
When an adolescent is experiencing AUB, an evaluation to determine the etiology of the bleeding should be
performed after ensuring they are hemodynamically stable. Abnormal uterine bleeding is divided into
structural and nonstructural causes according to the mnemonic PALM-COEIN (polyps, adenomyosis,
leiomyomas, malignancy, hyperplasia–coagulopathies, ovulatory disorders, endometrial disorders, iatrogenic,
not-yet-classified]). Adolescents aged 19 years and younger typically have nonstructural causes of AUB;
ovulatory dysfunction is the most common nonstructural cause. Adolescents may take up to 3 years after
menarche to establish a normal menstrual pattern of 7 days or less of bleeding every 24 to 45 days. Thus, the
early postmenarcheal adolescent with irregular cycles may not warrant a full hormonal evaluation. However,
if cycles are heavy and prolonged, an AUB evaluation should be performed, including an appropriate physical
examination (including pelvic examination, if indicated) and the following laboratory tests:
Von Willebrand factor and activity levels may be falsely elevated by acute illness, severe anemia, pregnancy,
estrogen exposure, and other types of physiologic stress. Thus, although it is important to test for VWD
before initiation of estrogen-containing hormonal treatment, the VWD screening tests should be repeated
after the adolescent is no longer acutely ill. Delays in laboratory processing time may falsely lower the levels.
An adolescent with an abnormal VWD screening test result should undergo repeat testing by a hematologist.
This testing should be performed when hormonal therapy is stable, after iron repletion has begun, and when
the adolescent is free from active illness; the sample should be sent to an appropriate laboratory with rapid
processing times.
Abnormal uterine bleeding or heavy menstrual bleeding has now replaced the previous terminology of
dysfunctional uterine bleeding, menorrhagia, metrorrhagia, and menometrorrhagia. The Federation of
International Gynecology and Obstetrics has provided recommendations regarding the definition and
classification of causes of AUB. Recent guidelines emphasize the negative impact AUB can have on quality of
life and the importance of inquiring about quality of life when discussing menstrual patterns with
adolescents.
The management of AUB should focus on controlling the current heavy bleeding and reducing blood loss
during subsequent cycles, especially if anemia is present. Hormonal treatments include intravenous
conjugated equine estrogen (indicated for severe anemia when inpatient stabilization is needed), estrogen-
containing combined oral contraceptive pills, norethindrone acetate pills, and medroxyprogesterone acetate
injections. Nonsteroidal anti-inflammatory drugs also provide some bleeding control. Tranexamic acid can be
administered for acute, heavy menstrual bleeding.
It is important to obtain free and total testosterone levels before beginning hormonal treatment to evaluate
for polycystic ovary syndrome. However, the adolescent in the vignette has no signs of androgen excess, and
it is more urgent to evaluate for a disorder of hemostasis. Follicle-stimulating hormone and estradiol levels
are helpful in the evaluation of anovulatory bleeding due to ovarian insufficiency or hypothalamic
dysfunction, which cause amenorrhea or oligomenorrhea. Measurement of follicle-stimulating hormone and
estradiol levels is not indicated in the evaluation of prolonged, heavy bleeding. Obtaining liver transaminase
levels may be appropriate if the adolescent has a known medical condition or is taking a medication that can
cause liver dysfunction leading to coagulopathy, but testing for VWD is a higher priority.
Suggested Reading(s)
Adeyemi-Fowode O, Stambough KC; Committee on Clinical Consensus. General approaches to medical
management of menstrual suppression. ACOG Clinical Consensus. 2022;140(3):528-541.
doi:10.1097/AOG.0000000000004899
American College of Obstetrics and Gynecologists. Opinion No. 557: Management of acute abnormal
uterine bleeding in nonpregnant reproductive-aged women. Obstet Gynecol. 2013;121(4):891-896.
doi:10.1097/01.AOG.0000428646.67925.9a
Borzutzky C, Jaffray J. Diagnosis and management of heavy menstrual bleeding and bleeding disorders
in adolescents. JAMA Pediatr. 2020;174(2):186-194. doi:10.1001/jamapediatrics.2019.5040
Graham RA, Davis JA, Corrales-Medina FF. The adolescent with menorrhagia: diagnostic approach to a
suspected bleeding disorder. Pediatr Rev. 2018;39(12):588–600. doi:10.1542/pir.2017-0105
Trent M, Joffe A. Vaginal bleeding. In, McInerny TK, Adam HM, Campbell DE, DeWitt TG, Foy JM, Kamat
DM, eds. American Academy of Pediatrics Textbook of Pediatric Care. American Academy of Pediatrics;
2016:chap 204. Pediatric Care Online
Content Domain
Reproductive health
The correct answer is: von Willebrand factor antigen and activity levels
View Peer Results
AAP PREP 2024 - Question 243/267 Research Question 1/1
A drug utilization study was performed through a retrospective medical record review to compare
prescribing patterns of 2 medications (drug X and drug Y) used for pulmonary complications of an
autoimmune disorder.
Drug Y
The prescribing patterns of these medications, based on the patient’s insurance carrier, are shown:
Of the following, the findings in this medical record review BEST represent a dilemma of the ethical principle
of
A. autonomy
B. bene cence
C. justice
D. nonmaleficence
Correct answer is C
PREP Pearl(s)
Medical ethics is the set of principles or guidelines used in practicing medicine to distinguish what is
considered right or wrong.
The 4 principles of medical ethics include autonomy, beneficence, justice, and nonmaleficence.
Justice is the principle that actions are compatible with both the patient's rights and the law and that
the distribution of health care resources should be fair and equitable.
Critique
The findings in this medical record review best represent a dilemma of the ethical principle of justice. In
medical ethics, justice is the principle that all patients should be treated fairly without regard to their
socioeconomic status, race, ethnicity, or educational level.
Medical ethics is the set of principles or guidelines used in practicing medicine to distinguish what is
considered right or wrong. These principles apply not only to physicians but also to the health care system as
a whole, and they must be considered in the context of managed health care.
There are 4 principles of medical ethics that must be considered for the ethical practice of medicine:
Autonomy: The ability of a patient to make decisions regarding their health care that are fully
informed and free from coercion.
Beneficence: The expectation that a physician or health care system treats a patient with the
intent to do good and perform actions that they believe lead to positive outcomes.
Justice: The principle that actions are fair and balanced, compatible with both the patient's rights
and the law, and that the distribution of health care resources will be equitable and fair.
Nonmaleficence: The obligation of a physician or health care system to neither cause nor allow
harm to a patient.
Core aims of a managed health care system include controlling health care costs and providing oversight of
quality of care. Intrinsic to the managed health care system is some limitation of autonomy in patient choice
of physicians and treatments; this limitation has the potential to impact quality of care.
The drug utilization medical record review described in the vignette does not identify a reason for the
discrepancy in the prescribing patterns among the insurance carriers. It only demonstrates an unequal
distribution of health resources on the basis of insurance carrier status (and possibly socioeconomic status).
A potential factor contributing to the discrepancy could be full or partial coverage of the cost of drug X and
drug Y by the insurance carrier, which may influence prescribing patterns or patient choice of medication.
Whatever the cause, there is clear inequality in therapeutic management for these different patient
populations, making justice the ethical dilemma in this case.
Although decreased patient autonomy could be responsible for the difference seen in this utilization study,
even a fully informed decision made to use drug X was likely influenced by a difference in out-of-pocket cost,
making the principle of justice the best answer. Both drug X and drug Y are appropriate therapies for
treatment of this condition, so there is no concern for nonmaleficence or beneficence.
Suggested Reading(s)
Beauchamp TL, Childress JF. Principles of Biomedical Ethics. 6th ed. Oxford University Press; 2009.
Page K. The four principles: can they be measured and do they predict ethical decision making? BMC
Med Ethics. 2012;13:10. doi:10.1186/1472-6939-13-10
Content Domain
Research
A 6-year-old girl with no significant medical history is evaluated for an 8-month history of intermittent left
knee pain with morning stiffness. Her mother has noticed her limping 2 to 3 times per week in the morning
or after long car rides. The girl is able to participate in recess at school with no limitations. She has no known
history of injury to the knee. Vital signs are normal for age, and physical examination findings are
unremarkable other than in the left lower extremity. Examination of the left knee demonstrates mild
warmth, a moderate joint effusion, and a flexion contracture (Figure 1). The left knee appears larger than the
right (Figure 2), and there is a leg-length discrepancy of 3/4 cm. Laboratory studies were obtained to further
evaluate the child’s suspected diagnosis.
Courtesy of D. Fleck
Of the following, the laboratory finding MOST associated with an increased risk of ocular complications in this
child is a(n)
Correct answer is A
PREP Pearl(s)
Juvenile idiopathic arthritis is an autoimmune condition defined as arthritis affecting 1 or more joints
for greater than 6 weeks, with no other causes, in a child younger than 16 years.
Chronic joint findings of juvenile idiopathic arthritis due to long-standing joint inflammation include
flexion contracture, bony overgrowth, and leg-length discrepancy.
Uveitis is a common extra-articular manifestation of juvenile idiopathic arthritis. Children younger than
7 years with a positive antinuclear antibody titer result are at the highest risk.
Critique
The girl in the vignette likely has juvenile idiopathic arthritis (JIA), formerly known as juvenile rheumatoid
arthritis (JRA). Children diagnosed with JIA at younger than 7 years who have a positive antinuclear antibody
titer result have an increased risk of developing uveitis.
Juvenile idiopathic arthritis is a common autoimmune condition seen in childhood. It is defined as arthritis
affecting 1 or more joints exceeding 6 weeks’ duration in a child younger than 16 years in the absence of
other causes (eg, infectious or postinfectious, malignant, or other inflammatory or noninflammatory
connective tissue–related disorders). Arthritis is defined as a joint effusion alone or 2 of the following 4 joint
findings:
Decreased range of motion
Joint warmth
Pain with movement of the joint
Joint tenderness
Long-standing joint inflammation due to JIA can lead to chronic joint findings on physical examination,
including bony overgrowth of the joint, flexion contracture, and leg-length discrepancy. These manifestations
are likely due to the effects of inflammatory cytokines on the adjacent growth plates.
Medications used to treat JIA include nonsteroidal anti-inflammatory drugs, intra-articular corticosteroid
injections, disease-modifying antirheumatic drugs, and biologic therapies (eg, tumor necrosis factor inhibitors
and interleukin 6 inhibitors). Adequate control of joint inflammation can help prevent progression of chronic
joint manifestations. Nonpharmacologic interventions include physical therapy and occupational therapy.
Orthotics (eg, heel lifts) for children with significant leg-length discrepancies can compensate for asymmetry
and improve mobility.
All children with JIA are at risk for uveitis, an inflammatory condition of the middle layer of the eye. Untreated
uveitis can lead to keratopathy, cataracts, glaucoma, and permanent vision loss. It is important that all
children diagnosed with JIA undergo ophthalmologic examinations shortly after initial diagnosis and routine
screening at appropriate intervals based on risk classification for early detection of uveitis, preservation of
vision, and prevention of long-term ocular complications. Children, particularly girls, younger than 7 years
with a positive antinuclear antibody titer result have the highest risk of uveitis and require slit-lamp
evaluations every 3 to 4 months for the first 4 years after diagnosis. The first-line treatment for uveitis is
corticosteroids (topical or oral). Many children require additional treatment with systemic
immunosuppressive therapies (eg, methotrexate or tumor necrosis factor inhibitors).
Although an elevated C-reactive protein level, positive rheumatoid factor, and anemia of chronic disease are
indicators of more severe disease that may indicate the need for more aggressive treatment of joint
manifestations, they are not indicators of increased risk of uveitis in children with JIA.
Suggested Reading(s)
Angeles-Han ST, Ringold S, Beukelman T, et al. 2019 American College of Rheumatology/Arthritis
Foundation Guideline for the Screening, Monitoring, and treatment of Juvenile Idiopathic Arthritis-
Associated Uveitis. Arthritis Care Res. 2019;71(6):703-716. doi:10.1002/acr.23871
Malleson PN, Mackinnon MJ, Sailer-Hoeck M, Spencer CH. Review for the generalist: the antinuclear
antibody test in children - when to use it and what to do with a positive titer. Pediatr Rheumatol Online
J. 2010;8:27. doi:10.1186/1546-0096-8-27
Sen ES, Clarke SL, Ramanan AV. The child with joint pain in primary care. Best Pract Res Clin
Rheumatol. 2014;28(6):888-906. doi:10.1016/j.berh.2015.04.008
Shenoi S. Juvenile idiopathic arthritis - changing times, changing terms, changing treatments. Pediatr
Rev. 2017;38(5):221-232. doi:10.1542/pir.2016-0148
Siegel DM, Gewanter HL, Sahai S. Rheumatologic diseases.In: McInerny TK, Adam HM, Campbell DE,
DeWitt TG, Foy JM, Kamat DM, eds. American Academy of Pediatrics Textbook of Pediatric Care.
American Academy of Pediatrics; 2021:chap 324. Accessed September 1, 2023. Pediatric Care Online
Content Domain
Rheumatology
ABP Content Specification(s) / Content Area(s)
Recognize the long-term complications associated with juvenile idiopathic arthritis
Plan the appropriate management of juvenile idiopathic arthritis, while recognizing side
The correct answer is: antinuclear antibody titer of 1:640
View Peer Results
AAP PREP 2024 - Question 245/267 Rheumatology Question 2/8
A 25-month-old, previously healthy girl is seen in the office for a progressively severe rash on her chest and
legs first noted 3 to 4 months ago. She developed joint pain and swelling over the past 2 to 3 weeks, and has
had malaise and a slight decrease in appetite. She has had no fevers. There were no recent sick contacts. The
girl’s vitals signs are normal for age. There are large bilateral knee and ankle joint effusions with overlying
redness and warmth; joint range of motion remains normal, but there is pain elicited on range-of-motion
testing. There is swelling and redness of the proximal interphalangeal joints with moderate contractures, and
a scattered rash over the chest and abdomen consisting of reddish-brown, circular, 6-mm lesions. Over the
lower extremities, there are mildly tender, subcutaneous, 1-cm nodules with mild overlying redness. The girl
was referred to a dermatologist; a skin biopsy revealed noncaseating epithelioid cell granulomas.
Laboratory test results are shown.
C. order echocardiography
D. refer to ophthalmology
Correct answer is D
PREP Pearl(s)
Early-onset sarcoidosis (Blau syndrome) has a median onset age of 26 months; the clinical
presentation is a triad of inflammatory arthritis, rash, and uveitis.
Pediatric-onset adult sarcoidosis presents in adolescents with symptoms of fever, malaise, and weight
loss; nearly all have pulmonary involvement.
The diagnosis of sarcoidosis is best made by biopsy of affected tissues with histological findings of
noncaseating granulomas.
Critique
This patient’s skin biopsy demonstrated the pathologic finding of noncaseating epithelioid granulomas, a
hallmark of sarcoidosis. Due to the associated risk of progressive uveitis, the best next step in her evaluation
is ophthalmology referral.
Early-onset sarcoidosis (EOS), also known as Blau syndrome, is a granulomatous inflammatory process. The
clinical triad of polyarthritis, rash, and uveitis presents between the ages of 2 months and 14 years (median
age of onset is 26 months). The manifestations of EOS include symmetric polyarthritis involving both large
and small joints including knees, ankles, wrists, and proximal interphalangeal joints. The inflammatory
arthritis is very prominent; pathologic evaluation of the synovium, if performed, shows characteristic
noncaseating epithelioid granulomas. Tendon and tendon sheath involvement of the hands (tenosynovitis)
may occur and can lead to an inability to fully straighten the fingers.
The most common cutaneous manifestation is small (5-7 mm), pale pink to tan lesions over the trunk, arms,
and face. These lesions may be minimally palpable; over time, the lesions may desquamate or take on a scaly
appearance. Affected children may develop painful subcutaneous nodules on the lower extremities that
appear identical to erythema nodosum. Less common cutaneous manifestations include urticaria,
leukocytoclastic vasculitis, and erysipelas-like lesions. Ocular manifestations of EOS may include a
granulomatous posterior uveitis. While initially asymptomatic, this condition progresses leading to profound
ocular complications, and up to one-third of those affected experience long-term visual deficits. Early-onset
sarcoidosis is often associated with a genetic mutation in the NOD2/CARD15 on chromosome 16. Cases can
be both familial and sporadic.
A second form of sarcoidosis seen in the pediatric population is pediatric-onset adult sarcoidosis. This
condition is similar to the classical sarcoidosis seen in adults. Children present with weight loss, fever, and
fatigue. Nearly all affected children experience pulmonary involvement, and approximately half have hilar
lymphadenopathy on radiographic imaging. The kidneys, muscles, nervous system, skin, joints, liver, and
heart may also be involved. This form of sarcoidosis is generally seen in adolescents, with increasing
frequency as the age of presentation approaches adulthood.
Sarcoidosis is a pathologic diagnosis best made through biopsy of affected tissue. Laboratory findings in
children and adolescents with sarcoidosis may include elevated inflammatory markers and
hypergammaglobulinemia. Angiotensin-converting enzyme (ACE) levels may be elevated due to secretion by
the formed granulomas; elevated ACE levels are supportive of the diagnosis, but are not confirmatory due to
low specificity and sensitivity. Hypercalcemia may also be present.
Medications used to treat sarcoidosis consist of immunosuppressive therapies (eg, corticosteroids), disease-
modifying antirheumatic drugs (DMARDs) (eg, methotrexate), and biologics (eg, tumor necrosis factor
inhibitors).
Chest radiography and echocardiography may eventually be recommended for the girl in the vignette to
screen for organ involvement, but in early-onset sarcoidosis the risk of pulmonary and cardiac involvement is
significantly lower compared with the risk of uveitis. An ophthalmology evaluation would be a higher priority.
Interferon-γ release assay testing is specific for Mycobacterium tuberculosis infection, which produces
caseating (necrotic) granulomas. The girl in the vignette has noncaseating granulomas, thus this would not be
an appropriate test.
Suggested Reading(s)
Ellis JC, Faber BG, Uri IF, Emerson SJ. Early onset sarcoidosis (Blau syndrome): erosive and often
misdiagnosed. Rheumatol. 2020; 59(5):1179-1180. doi:10.1093/rheumatology/kez484
Shetty AK, Gedalia A. Childhood sarcoidosis: a rare but fascinating disorder. Pediatr Rheumatol Online
J. 2008;6:16. doi:10.1186/1546-0096-6-16
Siegel DM, Gewanter HL, Sahai S. Cystic fibrosis. In: McInerny TK, Adam HM, Campbell DE, DeWitt TG,
Foy JM, Kamat DM, eds. American Academy of Pediatrics Textbook of Pediatric Care. American
Academy of Pediatrics; 2023. Accessed September 1, 2023. Pediatric Care Online
Content Domain
Rheumatology
A 5-month-old boy is seen in the office for an 8-day history of fevers between 38.5 and 39.5° C. He has been
fussier than usual and has been taking 1 to 2 ounces less formula per feeding than usual without vomiting.
He continues to have 5 to 6 wet diapers per day and 1 to 2 normal-appearing stools without diarrhea. He was
seen at an urgent care center on day 2 of fever and diagnosed with a left acute otitis media, which was
treated with amoxicillin. Two days into antibiotic treatment, he developed a rash, and his mother stopped
the medication. The rash, located over the chest and back, has waxed and waned. His mother has been using
an over-the-counter emollient cream for the rash on his trunk, a diaper cream for a rash in his groin area,
and acetaminophen for the fever.
In the office, his temperature is 39 °C, and his heart rate is 175 beats/min. His respiratory rate and blood
pressure are normal for age and his oxygen saturation is 98% in room air. His mucous membranes are moist;
his lips are dry and cracked; there is a patchy, erythematous, blanchable rash over his chest and an
erythematous, desquamating rash in his groin. His hands and feet are erythematous. The remainder of his
examination findings are unremarkable.
Of the following, the BEST next step in the evaluation and management of this infant is
A. a complete blood count, basic metabolic panel, respiratory virus panel, urinalysis and next day
follow-up
Correct answer is D
PREP Pearl(s)
Kawasaki disease is a clinical diagnosis, which can be supported by laboratory and echocardiographic
data.
Incomplete Kawasaki disease is more common in children ages younger than 6 months and older than
5 years and is often diagnosed late due to the lack of classic symptoms and atypical patient age.
The differential diagnosis of Kawasaki disease (KD) is broad; if considered, evaluation for KD should be
expedited to ensure timely treatment.
Critique
The best next step in the evaluation and management of this infant is referral to an emergency department
for evaluation and treatment for Kawasaki disease (KD), specifically incomplete Kawasaki disease.
Kawasaki disease is a medium-vessel vasculitis of uncertain etiology, which primarily affects children between
ages 6 months to 5 years The incidence of KD in the United States is ~25 per 100,000 children younger than
age 5 years; it is higher in children of Asian and Pacific Island descent compared with other populations. The
incidence is ten times higher in Japan, where the entity was first described. It occurs more commonly in
males than females (1.5:1). There is an increased rate of recurrence in individuals and family members that
suggests a genetic predisposition. Current theory is that KD is caused by a viral pathogen; no specific virus
has been identified. There is controversy surrounding the possible association between certain
environmental exposures and KD risk.
The diagnosis of classic KD is based on the following criteria:
Kawasaki disease is a clinical diagnosis. The typical symptoms can wax and wane over the course of the
disease and may not all be present at one time. This often leads to a delay in the diagnosis. Kawasaki disease
is not an echocardiographic diagnosis; there is no specific echocardiographic parameter that defines the
disease. Children <6 months old (as well as older children) have a higher incidence of incomplete KD, defined
as prolonged, unexplained fever and fewer than 4 of the principal clinical features, with additional laboratory
and/or echocardiographic abnormalities.
The criteria for incomplete KD include:
Fever >5 days with 2 or 3 principal clinical features or an infant ≤6 months of age with
unexplained fever for ≥7 days
Laboratory findings
CRP ≥3.0 mg/dL and/or ESR ≥40 mm/h
3 or more supplemental laboratory findings
Anemia for age
Platelet count ≥450,000//μL (450 x109/L) after the 7th day of illness
Albumin <3.0 g/dL (30 g/L)
Elevated ALT
WBC count of ≥15,000/μL (15.0 x 109/L)
Urine ≥10 WBC/hpf
OR POSITIVE Echocardiographic findings (any of 3 conditions)
z score of left anterior descending (LAD) or right coronary artery (RCA) ≥2.5
True coronary aneurysm
Other (must be ≥3 findings)
Decreased left ventricular systolic function
Mitral regurgitation
Pericardial effusion
z score of the LAD or RCA of 2.0-2.5
Echocardiography can be utilized in making the diagnosis of incomplete KD, and should be part of the
evaluation in children with persistent fever and a concern for KD who do not meet classic criteria. All patients
meeting clinical criteria for classic KD should undergo baseline echocardiography to define the initial extent
of coronary artery involvement, which aids in short and long-term management, as well as serial monitoring.
If a patient meets clinical criteria for classic KD, prompt treatment should be undertaken and not withheld
while awaiting echocardiographic results.
The differential diagnosis of incomplete KD is even wider, including the diagnoses above, as well as urinary
tract infection, lymphadenitis, cytomegalovirus, Epstein-Barr virus, meningitis (viral or bacterial), hepatitis,
appendicitis, and septic shock. The infant in the vignette’s age, duration of fever, additional clinical findings,
and lack of a clear etiology should prompt further evaluation. Symptomatic treatment with acetaminophen
without further evaluation potentially delays identification of an underlying diagnosis that needs treatment.
In the case of KD (classic and incomplete), prompt treatment significantly lowers the risk of coronary artery
changes, specifically aneurysm formation.
Although an outpatient laboratory evaluation with follow-up can be useful in select patients with persistent
fevers, this infant’s history and clinical and laboratory findings warrant a more urgent and complete
evaluation. A positive rapid respiratory viral panel may be seen in a subset of patients ultimately diagnosed
with KD, and should not be considered evidence against the diagnosis of KD in the presence of principal
clinical findings. Although admission is an appropriate next step in the evaluation of an infant with persistent
fevers and an unclear diagnosis, the infant in the vignette does not appear to be dehydrated based on the
clinical history and physical examination findings. Kawasaki disease and incomplete KD are often missed (or
diagnosed late) due to fractured care and common diagnostic error biases (eg, framing, premature closure,
anchoring, and confirmation).
Suggested Reading(s)
Bukulmez H. Current understanding of multisystem inflammatory syndrome (MIS-C) following COVID-
19 and its distinction from Kawasaki disease. Curr Rheumatol Rep. 2021;23(8):58. doi:10.1007/s11926-
021-01028-4
Jackson MA. Kawasaki disease. In: McInerny TK, Adam HM, Campbell DE, DeWitt TG, Foy JM, Kamat DM,
eds. American Academy of Pediatrics Textbook of Pediatric Care. American Academy of Pediatrics;
2023. Accessed September 1, 2023. Pediatric Care Online
McCrindle BW, Rowley AH, Newburger JW, et al; American Heart Association Rheumatic Fever,
Endocarditis, and Kawasaki Disease Committee of the Council on Cardiovascular Disease in the Young;
Council on Cardiovascular and Stroke Nursing; Council on Cardiovascular Surgery and Anesthesia; and
Council on Epidemiology and Prevention. Diagnosis, treatment, and long-term management of
Kawasaki disease: a scientific statement for health professionals from the American Heart Association.
Circulation. 2017;135(17):e927-e999. doi:10.1161/CIR.0000000000000484
Rowley AH. The complexities of the diagnosis and management of Kawasaki disease. Infect Dis Clin North Am.
2015;29(3):525-537. doi:10.1016/j.idc.2015.05.006
Son MBF, Newburger JW. Kawasaki disease. Pediatr Rev. 2018;39(2):78-90. doi:10.1542/pir.2016-0182
Content Domain
Rheumatology
A 5-year-old boy with a past medical history of mild atopic dermatitis presents with acute onset of non-
painful purpuric lesions on the lower extremities and buttocks over the past week. Upon awakening this
morning the boy developed mild colicky abdominal pain. His appetite is normal, and there is no diarrhea or
hematochezia. His vital signs, including blood pressure, are normal for age. There are palpable purpuric
lesions over his legs and buttocks and nonspecific edema of both hands. His abdomen is soft with no
palpable masses; there is mild discomfort to deep palpation but no guarding.
B. obtain a urinalysis
D. provide reassurance
Correct answer is B
PREP Pearl(s)
IgA vasculitis (Henoch-Schönlein purpura) is an immune complex–mediated vasculitis presenting with
palpable purpura and petechiae with lower extremity predominance along with abdominal pain,
arthralgia/arthritis, and/or renal involvement (hematuria, proteinuria).
Treatment for IgA vasculitis (Henoch-Schönlein purpura) is symptomatic with the use of nonsteroidal
antiinflammatory drugs (in the setting of normal renal function) for pain control. Glucocorticoids may
be required for severe gastrointestinal symptoms or renal involvement.
IgA vasculitis (Henoch-Schönlein purpura) is transient and self resolving, but requires diligent
monitoring for renal involvement with blood pressure checks and urinalysis testing for 6 months after
diagnosis.
Critique
The boy in the vignette has lower-extremity dominant, palpable purpura and abdominal pain, a presentation
most consistent with IgA vasculitis (Henoch-Schönlein purpura). Urinalysis is the best next step in his
management. Timely assessment for renal involvement, a common manifestation of this condition, is
essential due to the high morbidity risk if unrecognized. Monitoring for renal involvement with blood
pressure checks and urinalysis testing is required over the course of 6 months, as renal manifestations may
occur later in the course of IgA vasculitis.
IgA vasculitis is an IgA immune-complex–mediated, small-vessel vasculitis with predominant skin, joint,
gastrointestinal, and renal involvement. The current Pediatric Rheumatology European Society diagnostic
criteria include palpable purpura with lower extremity predominance, along with one of the following:
Courtesy of Edgar O. Ledbetter. Reprinted with permission from Adrogue HE, Hayde NA. Henoch-Schönlein
purpura. In: McInerny TK, Adam HM, Campbell DE, DeWitt TG, Foy JM, Kamat DM, eds. American Academy
of Pediatrics Textbook of Pediatric Care. American Academy of Pediatrics; 2021:chap 264.
Joint pain and/or arthritis affects over 80% of children with IgA vasculitis. Large joints are more commonly
involved, but small and medium joints may also be affected. The arthritis is transient and resolves within a
few days without chronic joint damage.
Gastrointestinal (GI) involvement commonly includes generalized abdominal pain, nausea, and vomiting.
There may be occult blood in the stool, and some children have hematochezia. Severe GI complications
include intussusception, bowel ischemia, and intestinal perforation. Less frequent GI manifestations include
pancreatitis, protein-losing enteropathy, strictures, and bowel obstruction.
There is no diagnostic testing for IgA vasculitis. Abnormal laboratory findings may include mild
thrombocytosis, normocytic anemia, leukocytosis, positive stool guaiac, and hematuria/proteinuria.
Treatment is symptomatic. Nonsteroidal anti-inflammatory drugs may be used for pain control in the setting
of normal renal function. Severe GI involvement and nephritis may warrant glucocorticoid steroid therapy.
Refractory cases may warrant treatment with immunomodulatory therapies such as mycophenolate mofetil,
cyclophosphamide, or azathioprine.
Abdominal ultrasonography would be warranted in the presence of severe abdominal pain to rule out
intussusception, which is not the case for the child in the vignette. The serum IgA level may be elevated in
some cases, but this is neither diagnostic nor a predictor of disease course or severity. Reassurance alone
would not be appropriate for this boy, as monitoring for renal involvement is critically important for children
with IgA vasculitis.
Suggested Reading(s)
Adrogue HE, Hayde NA. Henoch-Schönlein purpura. In: McInerny TK, Adam HM, Campbell DE, DeWitt
TG, Foy JM, Kamat DM, eds. American Academy of Pediatrics Textbook of Pediatric Care. American
Academy of Pediatrics; 2021:chap 264. Accessed September 1, 2023. Pediatric Care Online
Leung AKC, Barankin B, Leong KF. Henoch-Schönlein purpura in children: an updated review. Curr
Pediatr Rev. 2020;16(4):265-276. doi:10.2174/1573396316666200508104708
Reid-Adam J. Henoch-Schönlein purpura. Pediatr Rev. 2014;35(10):447-449. doi:10.1542/pir.35-10-447
Content Domain
Rheumatology
A 7-year-old girl with no significant medical history is being evaluated for a 9-month history of clumsiness.
Her father reports that she falls approximately 10 times per day, and recently she has had difficulty going up
stairs. Five months ago, she developed a raised, red rash over the joints of her fingers. During the past
month, she has developed a red rash around her eyelids, elbows, fingers, knees, and toes. On physical
examination, her vital signs are normal for age. She has difficulty climbing onto the examination table. There
are no joint effusions. Findings on strength testing include the following: shoulder adduction, 3/5; arm
flexion/extension, 4/5; hip flexion, 3/5; leg flexion, 4/5; hand strength, 5/5; and foot dorsiflexion/plantar
flexion, 4/5. She has a positive Gower sign and Trendelenburg sign. Skin examination reveals a subtle
violaceous hue of the upper eyelids, periorbital erythema, and erythematous to violaceous plaques over the
extensor surfaces of the distal and proximal interphalangeal joints, metacarpophalangeal joints, elbows, and
knees.
Laboratory data are shown:
γ-Glutamyltransferase 21 U/L
A. abdominal ultrasonography
Correct answer is D
PREP Pearl(s)
Juvenile dermatomyositis is an inflammatory myositis that presents with proximal muscle weakness,
cutaneous changes (heliotrope rash and Gottron papules), and elevated serum muscle enzymes.
Musculoskeletal magnetic resonance imaging of the proximal muscles (shoulder girdle and hip/pelvic)
will demonstrate inflammatory myopathy in children with juvenile dermatomyositis.
Juvenile dermatomyositis may involve the muscles of respiration. Low pharyngeal tone increases the
risk of aspiration, and vasculopathy of the gastrointestinal tract may lead to abdominal pain, bleeding,
or perforation of the bowel.
Critique
The girl in the vignette has signs and symptoms consistent with a diagnosis of juvenile dermatomyositis
(JDM). The sensitivity of magnetic resonance imaging (MRI) in detecting inflammatory myopathy has largely
replaced the need for electromyography and/or biopsy for the diagnosis of JDM. Electromyography and
biopsy can be performed if there is uncertainty in the diagnosis.
Juvenile dermatomyositis is the most common chronic idiopathic inflammatory myositis in children. It is
characterized by inflammation of both the muscles and skin due to an autoimmune angiopathy. The
incidence of JDM is 3.2 per million, with a peak incidence between the ages of 5 and 14 years; there is a
female predominance.
The diagnostic criteria for JDM include the following:
Reprinted with permission from Schroeder JC, Frantz T, Osten AW, Cho S. Visual diagnosis: rash and fatigue
in a 6-year-old girl. Pediatr Rev. 2021;42(4):e14.
Figure 1. Heliotrope rash of juvenile dermatomyositis with violaceous patches over the upper eyelids and
nasolabial and melolabial folds.
Reprinted with permission from Schroeder JC, Frantz T, Osten AW, Cho S. Visual diagnosis: rash and fatigue
in a 6-year-old girl. Pediatr Rev. 2021;42(4):e15.
Figure 2. Gottron papules: purple or red flat papules or plaques on the extensor surfaces of the small joints
of the hand.
Clinical manifestations of JDM include easy fatigability, proximal muscle weakness, muscle pain and
tenderness, and classic dermatologic findings (heliotrope rash and Gottron papules). Onset is generally slow
and insidious, but rapid onset occurs in approximately one-third of patients. The violaceous or reddish purple
heliotrope rash over the upper eyelids can resemble the malar rash of systemic lupus erythematosus but is
often less demarcated. Gottron papules are symmetrical, erythematous, scaly plaques over the extensor
surfaces of the joints. They are often hypertrophic and can range from light pink early in the disease course
to bright red later. Periungual skin and capillary bed changes can be seen on capillaroscopy. Approximately
one-third of affected children develop calcinosis in the subcutaneous tissues, which will usually
spontaneously regress or liquify and drain but may persist in 25% of cases.
The proximal muscles (shoulder girdle and hip/pelvic) are affected more than the distal muscles. Affected
individuals may have difficulty raising their arms above their head and performing sit-ups. They may exhibit a
positive Gower maneuver and Trendelenburg sign. In addition, JDM may involve the muscles of respiration.
Low pharyngeal tone may increase the risk of aspiration. Vasculopathy of the gastrointestinal (GI) tract may
lead to abdominal pain, GI bleeding, or perforation.
Laboratory test result abnormalities seen in children with JDM include elevation of muscle enzyme levels
(creatine kinase, aldolase, aspartate aminotransferase, alanine aminotransferase, and lactate
dehydrogenase). The degree of elevation does not correlate with disease severity. Anemia may be present
because of GI bleeding.
After addressing any life-threatening issues (eg, perforation, ventilatory insufficiency, and aspiration risk), the
short-term management of JDM is focused on reducing inflammation. Initial medical management includes
intravenous pulse methylprednisolone followed by a prolonged oral corticosteroid taper and first-line
steroid-sparing therapy with weekly subcutaneous methotrexate. Intravenous immunoglobulin, rituximab,
and cyclophosphamide can be used in severe or refractory cases. Recalcitrant skin disease can be treated
with hydroxychloroquine or dapsone. Adjunct therapies may include physical and occupational therapy (to
maintain or improve function), topical emollient therapies, and broad-spectrum UV protection.
Abdominal ultrasonography and hepatitis C serologic testing would not be warranted at this time. This child’s
elevated transaminase levels are secondary to muscle inflammation rather than hepatic disease. Given the
classic cutaneous findings of dermatomyositis, genetic testing for muscular dystrophy (analysis of X-linked
dystrophin gene) would not be indicated at this time.
Suggested Reading(s)
Schroeder JC, Frantz T, Osten AW, Cho S. Visual diagnosis: rash and fatigue in a 6-year-old girl. Pediatr
Rev. 2021;42(4):e13-e16. doi:10.1542/pir.2018-0218
Siegel DM, Gewanter HL, Sahai S. Rheumatologic diseases.In: McInerny TK, Adam HM, Campbell DE,
DeWitt TG, Foy JM, Kamat DM, eds. American Academy of Pediatrics Textbook of Pediatric Care.
American Academy of Pediatrics; 2023. Accessed September 1, 2023. Pediatric Care Online
Vehe RK, Riskalla MM. Collagen vascular diseases: SLE, dermatomyositis, scleroderma, and MCTD.
Pediatr Rev. 2018;39(10):501-515. doi:10.1542/pir.2017-0262
Content Domain
Rheumatology
A previously healthy 13-year-old adolescent boy is seen in the office for the third time in a month. He was
initially evaluated for 2 days of nonbloody diarrhea, abdominal pain, and fever and was diagnosed with viral
gastroenteritis. After a week of persistent symptoms, stool cultures revealed Campylobacter jejuni. The boy
was treated with azithromycin, and his symptoms resolved. Today, 3 weeks after resolution of his previous
symptoms, he has come to the office for evaluation of 2 days of eye redness and right knee pain and
swelling, without known injury.
The boy’s vital signs are normal for age. Physical examination reveals bilateral conjunctival injection without
drainage. His right knee is mildly swollen, with a small effusion and pain on passive range of motion. There is
no warmth or erythema. The remainder of his physical examination findings are unremarkable.
Of the following, the BEST next step in this boy’s care is
C. arthrocentesis with culture of the knee effusion and administration of intravenous antibiotics
Correct answer is A
PREP Pearl(s)
Postinfectious arthritis is managed with supportive care and nonsteroidal anti-inflammatory
medications.
Findings on laboratory and imaging studies obtained while evaluating for causes of arthritis are usually
nonspecific in postinfectious arthritis.
Some children with postinfectious arthritis experience chronic symptoms that evolve into ankylosing
spondylitis.
Critique
The adolescent boy in the vignette most likely has postinfectious arthritis. Of the response choices,
administration of a nonsteroidal anti-inflammatory medication and provision of supportive care is the best
next management step. The absence of fever and knee erythema or warmth makes infectious arthritis less
likely, so culture of the knee effusion and intravenous antibiotics are not indicated. Similarly, referral to an
orthopedic surgeon and avoidance of weight bearing are not necessary given the low likelihood of a
mechanical etiology. Prednisone and referral to a rheumatologist should be considered when there are
persistent or recurrent symptoms, but these measures are not indicated at this time.
Postinfectious arthritis may occur after a variety of infections, including the following:
Chlamydia pneumoniae
Chlamydia trachomatis
Infectious enteritis
Meningococcal infection
Viral infections
Large joints are affected more commonly than are small joints. Conjunctivitis and urethritis may also be
present. Joint symptoms begin days to weeks after the inciting infection and may last days to months. Some
children experience chronic symptoms that evolve into ankylosing spondylitis. Causes of arthritis requiring
urgent treatment (eg, osteomyelitis, septic arthritis) must be excluded. Laboratory and imaging studies
obtained to evaluate for other causes of arthritis usually reveal nonspecific findings.
Suggested Reading(s)
Sarwark JF, LaBella CR. Septic arthritis. In: Sarwark JF, LaBella CR, eds. Pediatric Orthopaedics and
Sports Injuries: A Quick Reference Guide. American Academy of Pediatrics; 2021:chap 8. Accessed
September 1, 2023. Pediatric Orthopaedics Online
Siegel DM, Marston B. Joint pain. In: McInerny TK, Adam HM, Campbell DE, DeWitt TG, Foy JM, Kamat
DM, eds. American Academy of Pediatrics Textbook of Pediatric Care. American Academy of Pediatrics;
2017:chap 171. Accessed September 1, 2023. Pediatric Care Online
Content Domain
Rheumatology
A 13-year-old girl is seen for a progressive rash on her lower extremities. The lesions began as small
erythematous spots, which have increased in size and spread. The results of a review of systems are
otherwise negative. On physical examination, there are firm and indurated lesions on both legs with a shiny,
waxy appearance (Figure 1, Figure 2). There is limited eversion and inversion of her ankles bilaterally, without
pain on range of motion testing and palpation. The remainder of her examination findings are normal.
Courtesy of D. Fleck
Of the following, the MOST likely cause of this child’s examination findings is
C. psoriasis
D. tinea versicolor
Correct answer is B
PREP Pearl(s)
Morphea (localized scleroderma) is an inflammatory condition that leads to hardening and fibrosis of
the skin and subcutaneous tissues.
Complications of morphea (localized scleroderma) include joint contractures, limb atrophy, and
angular deformities, leading to severe disability.
Juvenile systemic sclerosis is an inflammatory condition that features fibrosis of the skin with
multiorgan involvement, including the cardiovascular, gastrointestinal, musculoskeletal, pulmonary,
and renal systems.
Critique
The most likely cause of the findings of the girl in the vignette is morphea. Morphea (localized scleroderma) is
a cutaneous or subcutaneous inflammatory disease characterized by excessive collagen deposition, leading
to fibrosis and hardening of the skin and subcutaneous tissues. Morphea is classified into 5 types based on
appearance:
Circumscribed (plaque) morphea (Figure 3)
Linear scleroderma (Figure 4)
Generalized morphea (Figure 5)
Pansclerotic morphea
Mixed morphea
Reprinted with permission from Morphea. Point-of-Care Quick Reference. American Academy of Pediatrics.
Pediatric Care Online.
Reprinted with permission from Morphea. Point-of-Care Quick Reference. American Academy of Pediatrics.
Pediatric Care Online.
Morphea is a slowly evolving disease process, which often leads to a delay in diagnosis. Lesions start as
violaceous or erythematous plaques that progress to areas of induration with pigmentation changes, a firm
texture, and waxy appearance. Violaceous or erythematous borders are reflective of an active inflammatory
state. Affected skin may have alopecia and/or atrophy of the subcutaneous tissue and dermal or epidermal
layer.
Long-term complications of morphea are dependent on the areas affected. Musculoskeletal complications
may include joint contractures, limb atrophy, limb length discrepancies, and angular deformities. These
deformities can cause severe disability. Involvement of the face and head can lead to ocular (eg, uveitis,
episcleritis, and lacrimal gland) or neurologic (eg, headaches and seizures) complications.
Laboratory findings are often nonspecific; there are no serum antibodies associated with morphea. Magnetic
resonance imaging can be useful in assessing the extent of subcutaneous tissue, muscle, joint, and tendon
involvement. Biopsy of affected areas can aid in the diagnosis, but is not required. Treatment of morphea
includes topical therapies (eg, tacrolimus, calcipotriol, and corticosteroids). Ultraviolet-A therapy can be
considered for isolated morphea lesions. Affected individuals often require systemic therapy with
corticosteroids and methotrexate.
In contrast to morphea, the inflammation and fibrosis in juvenile systemic sclerosis involve not only the skin
and musculoskeletal system but also multiple organ systems, including the following:
Pulmonary
Interstitial lung disease and fibrosis
Pulmonary arterial hypertension
Cardiovascular
Raynaud phenomenon
Digital infarction
Myocardial fibrosis with subsequent congestive heart failure or arrhythmia
Gastrointestinal
Fibrosis of muscles of mastication, leading to dysphagia
Diffuse involvement with a “top-down” progression from the esophagus to the anal
sphincter (leading to impaired peristalsis and malnutrition)
Renal
Typically mild involvement
Scleroderma renal crisis (a rare medical emergency with microangiopathic hemolytic
anemia, hypertension, and renal insufficiency that is fatal if untreated)
Autoantibodies associated with juvenile systemic sclerosis include Scl-70, centromere, U1-RNP, PM-Scl, and
RNA polymerase. Each antibody is associated with a different phenotypic presentation.
Treatment for juvenile systemic sclerosis (dependent on disease severity and the organ systems involved)
includes the following:
Corticosteroids
Cyclophosphamide
Methotrexate
Mycophenolate mofetil
Rituximab
Hematopoietic stem cell therapy
The complications of Raynaud disease and digital ischemia are treated with the following:
Calcium channel blockers
Phosphodiesterase-5 inhibitors
Topical vasodilators
Occupational and physical therapy are required for joint contractures and impaired mobility.
Although cutaneous lupus and psoriasis are inflammatory disorders of the skin, they are not associated with
fibrosis and hardening. Tinea versicolor may cause skin hyperpigmentation or hypopigmentation; it is not
associated with fibrosis.
Suggested Reading(s)
Siegel DM, Gewanter HL, Sahai S. Rheumatologic diseases. In: McInerny TK, Adam HM, Campbell DE,
DeWitt TG, Foy JM, Kamat DM, eds. American Academy of Pediatrics Textbook of Pediatric Care.
American Academy of Pediatrics; 2023. Accessed September 1, 2023. Pediatric Care Online
Vehe RK, Riskalla MM. Collagen vascular diseases: SLE, dermatomyositis, scleroderma, and MCTD.
Pediatr Rev. 2018;39(10):501-515. doi:10.1542/pir.2017-0262
Content Domain
Rheumatology
ABP Content Specification(s) / Content Area(s)
Understand the prognosis of patients with scleroderma as opposed to those with systemic sclerosis
Recognize the clinical findings associated with localized scleroderma
The correct answer is: morphea (localized scleroderma)
A 17-year-old girl is seen for her annual health supervision visit. She has systemic lupus erythematosus,
diagnosed at the age of 12 years, which was initially treated with mycophenolate mofetil and corticosteroids
and has been in remission for 4 years. She is currently taking hydroxychloroquine. She is a junior in high
school. She is not active in sports but enjoys spending her summers swimming.
Correct answer is D
PREP Pearl(s)
Systemic lupus erythematosus is a chronic multisystem autoimmune disease. Associated serologic
markers include antinuclear antibody, anti-double-stranded DNA, and anti-Sm antibodies
An antinuclear antibody titer greater than or equal to 1:80 is required for diagnosis of systemic lupus
erythematosus but is not diagnostic because it is also positive in up to 20% of unaffected children.
Skin damage from UV radiation can lead to systemic lupus erythematosus flares; this can be mitigated
by sun avoidance and routine use of sunscreen.
Critique
For the girl in the vignette, the best intervention to prevent flares of her systemic lupus erythematosus (SLE)
is routine use of sunscreen. Ultraviolet radiation, which leads to both cytokine release and damage to
keratinocytes, is linked with the pathogenesis and activation of SLE and cutaneous lupus. Avoidance of UV-
induced cell damage with sun avoidance and sunscreen can prevent SLE activation and flares.
Appetite decrease
Fatigue
Symptoms
Fever
Weight loss
Alopecia
Bullous disease
Cutaneous vasculitis (Figure 1)
Discoid lupus (Figure 2)
Cutaneous findings
Malar (butterfly) rash (Figure 3)
Photosensitive annular rashes on limbs/face (Figure 4)
Raynaud disease
Subacute cutaneous lupus (Figure 5)
Inflammatory arthritis
Other physical Hepatosplenomegaly
examination findings Lymphadenopathy
Painless ulceration of nasal septum or posterior palate
Courtesy of Y. Kimura.
Reprinted with permission from Weiss JE. Pediatric systemic lupus erythematosus: more than a positive
antinuclear antibody. Pediatr Rev. 2012;33(2):66.
Reprinted with permission from Mancini AJ, Krowchuk DP. Systemic lupus erythematosus (SLE). In:
Pediatric Dermatology: A Quick Reference Guide. 4th ed. American Academy of Pediatrics; 2021.
Reprinted with permission from Mancini AJ, Krowchuk DP. Systemic lupus erythematosus (SLE). In:
Pediatric Dermatology: A Quick Reference Guide. 4th ed. American Academy of Pediatrics; 2021.
Reprinted with permission from Gupta D, Goldberg L, Dickinson D, et al. An 8-year-old boy with prolonged
fever and subcutaneous nodules. Pediatrics. 2022;149(5):e2021052974.
There is no specific laboratory test that confirms a diagnosis of SLE. Antinuclear antibody (ANA) titers equal to
or greater than 1:80 are required for diagnosis of SLE. On its own, this finding is not predictive of a diagnosis
of SLE; up to 20% of healthy children will have a positive ANA result. A positive ANA result in the context of
clinical suspicion for SLE requires further testing. Systemic lupus erythematosus–specific antibodies include
anti–double-stranded DNA and anti-Sm. Laboratory abnormalities are highly variable in SLE. The most
common laboratory test result abnormalities include the following:
Anemia (hemolytic anemia and anemia of chronic disease)
Lymphopenia
Thrombocytopenia
Elevated C-reactive protein level
Elevated erythrocyte sedimentation rate
Low C3 and C4 levels
It is important that renal function and urine testing be performed in children with or suspected to have SLE.
Approximately 50% of affected children will develop progressive glomerulonephritis.
Figure 6 outlines the diagnostic criteria for SLE. Systemic lupus erythematosus can affect nearly every organ
system. Conditions that may be seen include (but are not limited to) the following:
The primary treatment for SLE consists of corticosteroids (intravenous and oral) and hydroxychloroquine.
Disease severity and organ system involvement dictate further therapies; a plethora of immune modulating
medications are available.
No dietary changes have been found to prevent flares of SLE. Physical activity is important for individuals with
SLE, due to their increased risk of cardiovascular disease, but it will not prevent disease flares. Laboratory
monitoring may detect early serologic changes indicating active SLE, but monitoring does not help prevent
disease flares.
Suggested Reading(s)
Siegel DM, Gewanter HL, Sahai S. Rheumatologic diseases. In: McInerny TK, Adam HM, Campbell DE,
DeWitt TG, Foy JM, Kamat DM, eds. American Academy of Pediatrics Textbook of Pediatric Care.
American Academy of Pediatrics; 2021:chap 324. Accessed September 1, 2023. Pediatric Care Online
Vehe RK, Riskalla MM. Collagen vascular diseases: SLE, dermatomyositis, scleroderma, and MCTD.
Pediatr Rev. 2018;39(10):501-515. doi:10.1542/pir.2017-0262
Weiss JE. Pediatric systemic lupus erythematosus: more than a positive antinuclear antibody. Pediatr
Rev. 2012;33(2):62–74. doi:10.1542/pir.33-2-62
Content Domain
Rheumatology
A 12-year-old football athlete presents with right ankle pain and swelling after he was tackled two weeks ago.
He was unable to walk after the injury. The results of radiography of the foot and ankle were negative and he
was sent home with crutches, rest, ice, and elevation. The boy continues to have pain with weight bearing.
On physical examination, there is bruising along the lateral ankle. There is no tenderness to palpation over
the anterior talofibular, posterior talofibular, or calcaneofibular ligaments. He has pain with palpation over
the distal fibular physis. Ankle anterior drawer, squeeze, and Kleiger test results are negative. Repeat
radiography of the ankle demonstrates widening of the distal fibular physis with callus formation into the
metaphysis of the fibula.
B. peroneal tendonitis
D. syndesmotic sprain
Correct answer is C
PREP Pearl(s)
The physis is the weakest structure in the growing skeleton and most susceptible to injury during the
adolescent growth spurt.
Persistent pain at the physis and a limited ability to bear weight warrant repeat ankle radiography to
reevaluate the physis for an injury not visualized on initial radiographs.
Pain elicited with a squeeze test (examiner performs a compress and release motion midway up the
calf) and/or Kleiger test (lower leg is stabilized with one hand, the other hand grasps the medial aspect
of the foot while supporting the ankle in a neutral position, and the foot is externally rotated and
dorsiflexed) supports the diagnosis of a syndesmotic (high) ankle sprain.
Critique
The boy in the vignette has a Salter-Harris type II fracture of the distal fibula. The healing physis (growth
plate) injury is seen on his repeat radiographs performed 2 weeks after the event. Repeat radiography may
be required to identify bony healing at the site of a physis injury because the injury may not be visible on
initial radiographs. Persistent pain at the physis and a limited ability to bear weight warrant repeat
radiography. The physis is the weakest structure in the growing skeleton and most susceptible to injury
during the adolescent growth spurt. Physis fractures account for 30% of childhood fractures; they are most
commonly seen in the radius, tibia, and fibula.
The Salter-Harris classification is most widely used to describe these fractures (Table and Figure).
Reprinted with permission from Jones C, Wolf M, Herman M. Acute and chronic growth plate injuries.
Pediatr Rev. (2017) 38 (3):132.
Type I
Disruption through the physis and does not affect surrounding bone
Often nondisplaced; periosteum is intact
Mechanism: shear across the hypertrophic zone of the physis
Radiographs usually appear normal
Healing: 3 to 4 weeks
Typically without growth disruption
Type II
Disruption through the physis extending through metaphysis of the bone
75% of growth plate fractures
Mechanism: shear or avulsion with angular force
Radiographs: separation across physis; small portion of metaphysis attached
May require reduction, depending on angulation and displacement
Healing: within 4 weeks
Fractures of the distal tibia and femur are more likely to cause growth disruption
Type III
Disruption through the physis and epiphysis
Anatomical alignment is key for this fracture, which extends into the joint
Early and accurate reduction is required to prevent growth and joint disruption
Type IV
Disruption through the metaphysis, physis, and epiphysis
Risk of articular cartilage injury and growth arrest
Displaced fractures require open reduction and internal fixation (ORIF)
Poor prognosis for growth regardless of accuracy of reduction
Type V:
Crush injuries to all or part of the physis due to significant axial loading
Can occur at any age
Often does not require reduction or ORIF
Significant compression of physis results in destruction of growth potential
Full physeal injuries cause limb length discrepancy
Partial physeal injury often results in angulation of long bones (uninjured physis continues
normal growth and injured portion develops growth arrest)
The peroneus longus tendon, one of the tendons that stabilize the foot and ankle, runs posterior and inferior
to the lateral malleolus and attaches underneath the foot. Peroneus longus tendon injuries can be acute
(ankle sprain) or chronic (repetitive use). Injury results in pain and swelling posterior to the lateral malleolus
or along the tendons around this bony prominence.
A common injury in contact sports, a syndesmotic ankle sprain (high ankle sprain) occurs when the affected
foot is planted and the ankle forcibly dorsiflexed and externally rotated. The tibiofibular syndesmosis is a
fibrous joint made up of 4 ligaments: anterior inferior tibiofibular ligament, posterior inferior tibiofibular
ligament, transverse ligament, and interosseous ligament. Injury results in sharp pain felt along the
anterolateral ankle, which worsens with weight bearing. Pain elicited in the area of the syndesmosis when
performing a squeeze test (examiner performs a compress and release motion midway up the calf) and/or
Kleiger test (lower leg is stabilized with one hand, the other hand grasps the medial aspect of the foot while
supporting the ankle in a neutral position, and the foot is externally rotated and dorsiflexed). Pain elicited
with the squeeze or Kleiger test supports the diagnosis.
A common mechanism for lateral ankle sprain is inversion of the ankle. Affected individuals have pain and
swelling of 1 or multiple lateral ankle ligaments (anterior talofibular, posterior talofibular, and
calcaneofibular). The ankle anterior drawer test assesses the lateral integrity of the anterior talofibular
ligament, which was normal on the boy’s examination. Ankle sprain is 1 of the most common lower-extremity
injuries and can be classified as follows:
Grade I sprains (stretch, microtear) are mechanically stable.
Grade II sprains (partial tear) involve some joint laxity.
Grade III sprains (full tear) show clinical and/or radiologic evidence of instability.
For lateral ankle sprains, early mobilization and focused range-of-motion exercises reduce pain and shorten
recovery time compared with prolonged rest. Rehabilitation of the grade I or II ankle sprain is more effective
than immobilization; balance training and neuromuscular training should be performed throughout. Chronic
ankle instability with recurrent sprains is associated with deficits in proprioception and/or muscle strength
and ligamentous laxity. Use of a brace after ankle injury may reduce recurrent injury by increasing
proprioceptive feedback; ankle taping has not shown the same benefits.
Suggested Reading(s)
Jones C, Wolf M, Herman M. Acute and chronic growth plate injuries. Pediatr Rev. 2017;38(3):129-138.
doi:10.1542/pir.2015-0160
Moseley M, Rivera-Diaz Z, Fein DM. Ankle Injuries. Pediatr Rev. 2022;43(3):185–187.
doi:10.1542/pir.2021-004992
Fractures and dislocations. Point-of-Care Quick Reference. Pediatric Care Online. American Academy of
Pediatrics. 2015. Accessed September 1, 2023. Pediatric Care Online
Content Domain
Sports Medicine
Pattern Through physis Physis to metaphysis Physis to epiphysis Through metaphysis, physis, Crushed physis
and epiphysis
Radiograph finding Normal to subtle Possible displacement or Possible displacement, rotation, Possible displacement, rotation, Normal to subtle irregularity
irregularity rotation/angulation angulation, or joint disturbance angulation, or joint disturbance
Management Relative immobilization Possible reduction and Closed versus surgical Closed vs surgical reduction Possible surgical reduction,
immobilization reduction and immobilization and immobilization immobilization and
management of growth arrest
Potential complication Prolonged pain Malunion or growth arrest Malunion, arthritis, or Late displacement, malunion, Growth arrest
growth arrest arthritis, or growth arrest
AAP PREP 2024 - Question 253/267 Sports Medicine Question 2/11
A 15-year-old athlete is being evaluated after a batted (line drive) ball struck her left eye during a high school
softball game. She immediately fell to the ground in pain and now reports diffuse pain around the eye and
orbit. Physical examination of the left eye demonstrates a normal pupil shape and a thin layer of blood in the
anterior chamber of the left eye. The remainder of her examination findings are normal.
Correct answer is D
PREP Pearl(s)
Eye injury assessment includes visual acuity, pupillary response, extraocular eye movements, and
palpation around the orbit.
A hyphema, a collection of blood in the anterior chamber of the eye, needs urgent evaluation and
monitoring by an ophthalmologist to assess intraocular pressure and perform a dedicated eye
examination.
An individual with a hyphema should be positioned with the head elevated to at least 30°.
Critique
The adolescent in the vignette sustained an eye injury when she was hit with a projectile (softball). The high
velocity of this impact increases her risk of serious injury to the globe and orbit. Her physical examination
finding of blood in the anterior chamber is consistent with a traumatic hyphema (Figure). The first step in the
evaluation of an athlete with an eye injury should be an assessment of visual acuity (Snellen eye chart),
followed by evaluation of pupillary response, extraocular eye movements, and palpation around the orbit. If
red flag findings are present (eg, vision changes, decreased pupillary response to light, an irregular pupil, or
bony tenderness of the orbit), the athlete should be transported to the emergency department for further
evaluation. Fluorescein evaluation for corneal abrasion or the administration of topical anesthetic or steroid
drops should not take place before assessment for more severe or penetrating injuries to the globe.
Reprinted with permission from Lavrich JB, Hersink S. Red eye/pink eye. In: McInerny TK, Adam HM,
Campbell DE, DeWitt TG, Foy JM, Kamat DM, eds. American Academy of Pediatrics Textbook of Pediatric
Care. 2nd ed. Itasca, IL: American Academy of Pediatrics; 2016:chap 188.
Figure. Hyphema.
Typically painful, a hyphema is a collection of blood in the anterior chamber of the eye, between the cornea
and iris. The blood may cover part or all of the iris and pupil, causing visual obstruction. Hyphema is a serious
injury with significant risk of permanent vision loss; this injury requires urgent evaluation by an
ophthalmologist. An individual with a hyphema should be positioned with their head elevated to at least 30°
to layer the blood inferiorly and outside the visual axis. The anterior chamber blood typically resolves over
time. Close monitoring by an ophthalmologist is required. Acute complications include intraocular
hypertension (typically occuring shortly after the injury) and rebleeding (2-5 days after injury in approximately
30% of cases). Return to play is not recommended until full resolution.
Hyphema must be differentiated from subconjunctival hemorrhage, which involves a painless conjunctival
blood vessel rupture and spares the anterior chamber of the eye. Subconjunctival hemorrhage is usually self-
limited, resolving in 7 to 10 days. Affected athletes can usually return to the field immediately if they have no
vision changes.
A sharp pain or foreign-body sensation, tearing, and photosensitivity after eye injury suggest a corneal
abrasion. Fluorescein staining of the eye and use of a Wood lamp will show uptake of the fluorescein stain in
the abrasion. Corneal abrasion is treated with antibiotic eye drops. Contact lenses should not be worn until
antibiotic treatment is finished.
Suggested Reading(s)
Canty G, Nilan L. Return to play. Pediatr Rev. 2015;36(10):438-447. doi:10.1542/pir.36-10-438
Micieli JA, Easterbrook M. Eye and orbital injuries in sports. Clin Sports Med. 2017;36(2):299-314.
doi:10.1016/j.csm.2016.11.006
Rao RC, Bohra L, Roarty JD. 2016. Ocular trauma. In: McInerny TK, Adam HM, Campbell DE, DeWitt TG,
Foy JM, Kamat DM, eds. American Academy of Pediatrics Textbook of Pediatric Care. 2nd ed. American
Academy of Pediatrics; 2017:chap 300. Pediatric Care Online
Whiteside JW. Management of head and neck injuries by the sideline physician. Am Fam Physician.
2006;74(8):1357-1362. PMID:17087430
Content Domain
Sports medicine
The correct answer is: visual acuity evaluation with a Snellen eye chart
View Peer Results
AAP PREP 2024 - Question 254/267 Sports Medicine Question 3/11
A 16-year-old male soccer player is evaluated for right knee pain. He recalls no trauma. The medial knee has
gradually become more painful during activity in the past several weeks. He now limps with activity and
reports locking of the right knee. Physical examination reveals a limp, mild effusion of the right knee, and
limited extension of the right leg. The remainder of his physical examination findings are unremarkable,
including testing of the ligaments and meniscus. Four-view knee radiographs are obtained. The anterior
posterior knee image is shown in Figure 1.
Reprinted with permission from Atanda A Jr, Reddy D, Rice JA, Terry MA. Injuries and chronic conditions of
the knee in young athletes. Pediatr Rev. 2009;30(11):424.
B. bone contusion
C. meniscus tear
D. osteochondritis dissecans
Correct answer is D
PREP Pearl(s)
Osteochondritis dissecans of the knee typically develops in the femoral condyles.
The presence of a joint effusion and joint locking warrants complete radiographic evaluation. Children
6 to 10 years of age have normal baseline irregularity (ossification variant) of the posterior femoral
condyles, which can mimic osteochondritis dissecans on radiographs.
Orthopedic consultation is recommended regarding nonoperative treatment of osteochondritis
dissecans (OCD) (stable OCD lesions with an open physis) vs operative treatment (unstable OCD
lesions, closed physis, or failure of conservative treatment).
Critique
The adolescent in the vignette’s history and physical examination findings are most consistent with
osteochondritis dissecans (OCD) (Figure 2). The joint effusion is concerning and could be secondary to an
anterior cruciate ligament or meniscus tear; however, the lack of trauma and the insidious onset of
symptoms are inconsistent with those diagnoses. Additionally, the findings on ligament and meniscus
examination are normal. The limited extension of the right knee is likely caused by a loose body in an
unstable joint with OCD. A bone bruise would not have the gradual, insidious onset of signs and symptoms
seen in this adolescent.
Reprinted with permission from Atanda A Jr, Reddy D, Rice JA, Terry MA. Injuries and chronic conditions of
the knee in young athletes. Pediatr Rev. 2009;30(11):424
Figure 2. Osteochondral dissecans lesion seen on the medial femoral condyle on A. plain radiograph and B.
magnetic resonance T1-weighted coronal image.
Osteochondritis dissecans is a focal abnormality in the subchondral bone of the knee, elbow, or ankle joint,
which can progress to joint instability. Joint instability can lead to bone and overlying articular cartilage
detachment, resulting in a loose body within the joint space. A loose body or irregular bony surface leads to
pain, swelling, locking of the affected joint, and limited joint range of motion. The most common location of
knee OCD is the medial femoral condyle, although OCD of the lateral femoral condyle, trochlea, patella, and
tibia have been reported. Osteochondritis dissecans is best visualized by 4-view knee radiographs that
include anterior-posterior, lateral, tunnel, and sunrise projections. The tunnel view will best demonstrate
OCD in the posterior femoral condyles. The sunrise view may identify a patellar or trochlear OCD.
Osteochondritis dissecans in the knee may be asymptomatic or may present with symptoms such as pain,
swelling, locking, and limited range of motion. During adolescence, OCD is more common in boys than girls.
Notably, children 6 to 10 years of age have normal baseline irregularity (ossification variant) of the posterior
femoral condyles, which can mimic OCD on radiographs; age and development should always be taken into
account when interpreting imaging studies. In children older than 10 years, irregularity of the femoral
condyles warrants further evaluation. Magnetic resonance imaging (MRI) is typically used to characterize the
size and stability of OCD.
Both nonoperative and operative options exist for treatment of OCD; orthopedic surgery consultation is
recommended regarding management. The ultimate goal is to achieve bony healing and prevent breakdown
of the joint. Nonoperative management may be appropriate in the case of an open femoral physis (indicating
healing potential) if the OCD appears stable on MRI. When a radiographic diagnosis of OCD is made
incidentally, an asymptomatic OCD may be followed with serial radiographs until full ossification or healing is
seen. A symptomatic OCD that is stable on imaging with an open femoral physis may also be followed with
serial radiographs every 3 to 6 months. In such a case, the patient and family should be counseled to restrict
high-impact exercise, such as running, jumping, and pivoting, while the OCD is being monitored. If after 3 to 6
months, there is persistent pain, joint instability, or no radiographic evidence of healing, surgery should be
considered. Adolescents with a closed femoral physis typically require surgery to drill, fix, or graft the OCD
lesion. Operative treatment is also recommended for all unstable OCD lesions seen initially on MRI.
Suggested Reading(s)
Atanda A Jr, Reddy D, Rice JA, Terry MA. Injuries and chronic conditions of the knee in young athletes.
Pediatr Rev. 2009;30(11):419-428; quiz 429-430. doi:10.1542/pir.30-11-419
Cruz AI Jr, Shea KG, Ganley TJ. Pediatric knee osteochondritis dissecans lesions. Orthop Clin North Am.
2016;47(4):763-775. doi:10.1016/j.ocl.2016.05.001
Kannikeswaran N, Srinivasan Suresh S. Sports musculoskeletal injuries. In: McInerny TK, Adam HM,
Campbell DE, DeWitt TG, Foy JM, Kamat DM, eds. American Academy of Pediatrics Textbook of Pediatric
Care. American Academy of Pediatrics; 2021:chap 334. Pediatric Care Online
Uppstrom TJ, Gausden EB, Green DW. Classification and assessment of juvenile osteochondritis
dissecans knee lesions. Curr Opin Pediatr. 2016;28(1):60-67. doi:10.1097/MOP.0000000000000308
Wall EJ, Brtko K. The nonoperative treatment of osteochondritis dissecans of the knee. Curr Opin
Pediatr. 2021;33(1):59-64. doi:10.1097/MOP.0000000000000976
Content Domain
Sports Medicine
A 15-year-old is evaluated in the emergency department after a fall during a soccer game. The patient fell
with their right arm abducted and externally rotated, and felt a shift of the right shoulder as they hit the
ground. Their shoulder “popped” again as they pulled on their arm and was unable to continue participation.
They had no prior injuries to the right shoulder and is right hand dominant. Physical examination is notable
for swelling of the anterior shoulder, no ecchymosis, no deformity, and a limited range of motion in all
directions (forward flexion, extension, internal rotation, and external rotation). There is decreased sensation
over the deltoid/lateral shoulder. Their reflexes are intact and pulses are full and equal. Three-view
radiographs of the shoulder (anterior-posterior, axillary, scapular) demonstrate proper joint alignment, a
fracture of the anteroinferior glenoid rim, and a depression fracture of the posterolateral humeral head.
Of the following, the BEST next step in management is to
Correct answer is C
PREP Pearl(s)
A bony Bankart lesion and Hill-Sachs deformity are fracture patterns that may be seen on radiographs
after anterior shoulder dislocation.
Magnetic resonance imaging evaluates for labral pathology and joint stability in a child or adolescent
with shoulder dislocation.
The recurrence rate after primary shoulder dislocation in children and adolescents younger than 20
years of age is 70% to 80%, and a discussion of the risks and benefits of nonoperative versus operative
management is recommended.
Critique
The patient in the vignette has an anterior shoulder dislocation per the history, physical examination, and
radiographic findings. Further imaging with magnetic resonance imaging (MRI) is recommended in youth to
evaluate for labral pathology and joint stability due to radiograph findings. Physical therapy starts with a
three phase protocol after a shoulder dislocation. First phase incorporates gentle range of motion and
isometric exercises, the second phase moves to elastic band exercises, and third phase incorporates
plyometrics, endurance, and resisted weight exercises.
With an anterior inferior shoulder dislocation, typically, the arm is held abducted and externally rotated. A
shoulder dislocation may injure the axillary nerve, causing a loss of sensation over the lateral shoulder.
Dislocations can self-reduce or require manipulation in the emergency department. There are various
techniques to reduce shoulder dislocation acutely. After reduction, the initial management should include
rest, ice, sling, and swathe. Gentle range-of-motion exercises are safe to begin after joint reduction.
Compression with an elastic bandage is not recommended for this condition.
The humeral head and glenoid of the scapula articulate over a small surface area, creating significant mobility
of the shoulder joint, which may naturally lead to instability and dislocations. The labrum and glenohumeral
ligaments (superior, middle, inferior) are the primary soft tissue stabilizers. The rotator cuff, long head of
biceps, and periscapular muscles act as dynamic stabilizers of the shoulder. Anterior inferior is the most
common form of shoulder dislocation. Multidirectional shoulder instability and posterior shoulder
dislocations present differently and require different clinical approaches to treatment.
Primary anterior shoulder dislocation often produces a pathological change at the humeral head or glenoid.
Standard shoulder radiograph views (ie, anterior posterior, scapular, axillary) can demonstrate associated
bony pathologies, includin;g a Hill-Sachs deformity (depression fracture of the posterolateral humeral head)
and a bony Bankart lesion (fracture of the anterior inferior glenoid rim). In addition, in 20% of youth shoulder
dislocations, labral tears are present. Labral tears occur when the humeral head shifts anteroinferiorly and
contacts the anterior glenoid rim. An intact labrum serves as a natural bumper or ridge along the glenoid
adding to joint stability. Magnetic resonance imaging is used to evaluate the extent of joint damage, laxity,
and labral integrity, and results can guide treatment. The recurrence rate of shoulder dislocation in children
and adolescents younger than 20 years of age is 70% to 80%, whereas in adults the recurrence rate is close to
20%. This high recurrence rate of shoulder dislocation is attributed to the combination of the following: rate
of return to contact sports, general ligamentous laxity, and damage to the labrum.
Surgical fixation is an option that may be considered at a young age. Maintaining joint health and integrity by
preventing exacerbation of shoulder instability is the goal. Significant findings on MRI, a bony Bankart lesion,
or desired participation in high contact or overhead activity may be indications for operative treatment after
a primary dislocation. Recurrent shoulder instability further damages the labrum and glenoid, accelerates
arthritis of the joint, and causes surgical complexity. Surgeons may use computed tomography (CT) for
evaluation of the glenoid joint. The availability of 3D reconstruction of CT images is helpful for surgical
planning when there is a bony Bankart injury necessitating calculation of the surface area of the affected
joint. A large bony Bankart lesion (>20% of the glenoid surface area) requires surgical management. Referral
to an orthopedic surgeon, patient and family education, and shared decision-making are critical to
management in the pediatric population. Families may opt to proceed with conservative treatment and
physical therapy, understanding the risks and high recurrence rate.
Suggested Reading(s)
Franklin CC, Weiss JM. The natural history of pediatric and adolescent shoulder dislocation. J Pediatr
Orthop. 2019;39(6)(Suppl_1):S50-S52. doi:10.1097/BPO.0000000000001374
Kannikeswaran N, Suresh S. Sports musculoskeletal injuries. In: McInerny TK, Adam HM, Campbell DE,
DeWitt TG, Foy JM, Kamat DM, eds. American Academy of Pediatrics Textbook of Pediatric Care.
American Academy of Pediatrics; 2021:chap 334. Accessed September 1, 2023. Pediatric Care Online
Lin KM, James EW, Spitzer E, Fabricant PD. Pediatric and adolescent anterior shoulder instability:
clinical management of first-time dislocators. Curr Opin Pediatr. 2018;30(1):49-56.
doi:10.1097/MOP.0000000000000566
Content Domain
Sports Medicine
A 16-year-old boy is seen in the office for a sports preparticipation physical examination for clearance to play
football. He has no significant medical history and is taking no medications or supplements. He plans to
supplement his diet with creatine monohydrate to “bulk up” for the football season. Of the following, the
MOST accurate statement is that the use of this supplement may
Correct answer is B
PREP Pearl(s)
Protein, creatine, and caffeine are the most commonly used performance-enhancing substances by
adolescents.
Performance-enhancing supplementation does not produce greater benefits than naturally acquired
throughout adolescence when coupled with proper nutrition and fundamental training.
Critique
Creatine monohydrate may result in short-term performance and strength gains in adults. Creatine
monohydrate is a popular performance-enhancing substance that can alter physique and improve short,
high-intensity exercise performance. These effects are appealing to youth. Approximately 10% to 30% of high
school and 30% to 40% of college athletes have reported using creatine. Polypharmacy of other performance-
enhancing substances along with creatine monohydrate is also seen in this population.
Creatine is a nonessential amino acid produced endogenously in the liver (1 g/d), kidney, pancreas and
exogenously ingested from fish or red meat. Approximately 95% of systemic creatine is stored in skeletal
muscle. Smaller reservoirs for creatine storage are the blood, brain, and testes. The total daily requirement of
creatine is 2 g/d. Creatine monohydrate is a nutritional supplement easily obtained over the counter; it is
often supplemented at 2 to 3 times the recommended daily allowance for performance enhancement.
Creatine acts as a major energy source for skeletal muscle during initial bouts of anaerobic activity after free
adenosine triphosphate is consumed. Creatine combines with a phosphoryl group and is stored as
phosphocreatine, which then dephosphorylates to recycle adenosine diphosphate into adenosine
triphosphate in an enzymatic pathway. Maximizing creatine and phosphocreatine storage is critically
important for high-intensity, short bursts of repetitive exercise (eg, weight lifting, sprints, and jumping
events). Creatine shortens recovery time and increases training load to create strength gain, which is ideal for
athletic performance. Phosphocreatine stores can be increased 15% to 40% through oral supplementation.
Although evidence supports strength gains and performance enhancement with creatine supplementation in
adults, it is a temporary gain. Creatine monohydrate does not improve performance in aerobic exercise
performance or endurance training. Additionally, no formal study or randomized clinical trial data are
available on the safety considerations in youth or adolescent athletes; all published research has been
completed in adults.
Adverse effects of creatine supplementation include water retention and weight gain from increased total
body water. A rapid increase in muscle size due to fluid retention is believed to play a role in the
development of chronic exertional compartment syndrome secondary to creatine monohydrate. Other
adverse effects include gastrointestinal cramping, fatigue, and diarrhea. Supplementation with creatine
monohydrate does not cause dehydration, electrolyte imbalances, or renal impairment.
The American Academy of Pediatrics policy statement on performance-enhancing substances (Labotz 2016)
notes that naturally occurring strength gains during puberty are greater than those from performance-
enhancing substances, such as creatine. Performance-enhancing supplements do not produce greater
benefits than are naturally acquired throughout adolescence when coupled with proper nutrition and
fundamental training.
Suggested Reading(s)
Dandoy C, Gereige RS. Performance-enhancing drugs. Pediatr Rev. 2012;33(6):265-71;
doi:10.1542/pir.33-6-265
Jagim AR, Kerksick CM. Creatine supplementation in children and adolescents. Nutrients.
2021;13(2):664. doi:10.3390/nu13020664
LaBotz M, Griesemer BA. Council on Sports Medicine and Fitness. Use of performance-enhancing
substances. Pediatrics. 2016;138(1):e20161300. doi:10.1542/peds.2016-1300
Content Domain
Sports Medicine
The correct answer is: cause short-term performance and strength gains in adults
A 15-year-old female high school soccer player is seen in the office for a preparticipation physical evaluation
in late August. She also participates on the school track team in the spring. Last track season she experienced
severe shin splints; the symptoms have fully resolved. Her family history is significant for her father having a
myocardial infarction at 48 years of age. The adolescent has been well, but today developed mild rhinorrhea.
Her physical examination findings are remarkable only for a temperature of 38.5 °C and clear nasal
discharge. Her hearing screening results are normal. Her vision screening results are 20/30 OD and 20/20 OS.
The girl is informed that she will need to return in 1 week for reassessment for participation clearance. She
asks what made her ineligible for sports clearance today.
Correct answer is A
PREP Pearl(s)
Fevers alter the body’s core temperature, leading to temperature dysregulation.
Participation in exercise with a fever can increase the risk of heat illness.
A positive family history of sudden cardiac death at younger than 35 years, placement of a defibrillator
or pacemaker at younger than 35 years, or a genetic, electrical, or structural heart condition warrant
further investigation and may delay or disqualify medical clearance for sports participation.
Critique
The adolescent in the vignette’s febrile illness makes her currently medically ineligible for sports participation.
Fever elevates the core temperature, increasing the individual’s metabolism and heart rate. The altered core
temperature leads to temperature dysregulation; the body inherently stores more heat. Exercise further
increases heat production, which increases the risk of heat illness. Fever also increases insensible fluid losses,
impairs coordination and concentration, and decreases muscle strength, aerobic power, and endurance. Viral
illnesses contribute to tissue wasting, muscle catabolism, and a negative nitrogen balance.
The preparticipation physical evaluation (PPE) addresses both adolescent wellness and safe sport
participation. The PPE form is a screening tool used to identify medical and musculoskeletal conditions that
may affect the adolescent during physical activity. The goals of this screening are to identify risk factors for
and to prevent sudden cardiac death, promote safe physical activity participation, perform injury assessment
and injury prevention, and meet liability standards of sport participation for local leagues or schools.
The American Heart Association guidelines recommend the sports preparticipation evaluation as a case
finding screening tool that includes a personal cardiac history (7 questions), family cardiac history (4
questions), and cardiac physical examination. Positive personal history, family history, and/or physical
examination findings trigger further cardiac investigation. An isolated family history of myocardial infarction
in this adolescent’s father at 48 years of age is not a cause for medical ineligibility. A positive family history of
sudden cardiac death at younger than 35 years, placement of a defibrillator or pacemaker at younger than 35
years, or a genetic, electrical or structural heart condition warrant further investigation and may delay or
disqualify medical clearance.
The standard PPE form allows the medical practitioner to assign the child or adolescent to 1 of 4 categories:
The preparticipation evaluation also screens for musculoskeletal conditions that affect the adolescent athlete
(eg, fractures, stress fracture, or ligament, tendon, muscle, or joint pathology). Unresolved musculoskeletal
injuries may warrant additional evaluation or make an athlete medically ineligible. Resolved severe shin
splints are not a cause for medical ineligibility. Rather, the history of shin splints should lead to counseling
regarding training errors, neuromuscular training, and consideration of physical therapy in the preseason.
Visual acuity should be checked during the PPE. If the corrected vision is worse than 20/20, the athlete should
be referred for evaluation by an eye care specialist. If the best-corrected vision is worse than 20/40 in one
eye, the individual is considered functionally 1-eyed, and eye protection should be strongly encouraged to
protect the unaffected eye. Eye protection is essential for high-risk sports with small projectiles (eg, paintball,
air rifle, and BB gun), hard projectiles or sticks (eg, baseball, softball, basketball, cricket, fencing, field hockey,
ice hockey, lacrosse, racquetball, and squash), and combat sports (eg, boxing and martial arts). The American
Academy of Ophthalmology recommends that athletes who are functionally 1-eyed should not participate in
sports that do not allow athletes to wear eye protection (eg, boxing, wrestling, or full-contact martial arts).
Suggested Reading(s)
American Academy of Family Physicians, American Academy of Pediatrics, American College of Sports
Medicine, American Medical Society for Sports Medicine, American Orthopaedic Society for Sports
Medicine and the American Osteopathic Academy of Sports Medicine. Preparticipation Physical
Evaluation. 5th ed. American Academy of Pediatrics; 2019.
Dick NA, Diehl JJ. Febrile illness in the athlete. Sports Health. 2014;6(3):225-231.
doi:10.1177/1941738113508373
Harris MD. Infectious disease in athletes. Curr Sports Med Rep. 2011;10(2):84-89.
doi:10.1249/JSR.0b013e3182142381
Manuel C, Feinstein R. Sports participation for young athletes with medical conditions: seizure
disorder, infections and single organs. Curr Probl Pediatr Adolesc Health Care. 2018;48(5-6):161-171.
doi:10.1016/j.cppeds.2018.06.004
Content Domain
Sports Medicine
A 13-year-old boy is seen after a collision during a soccer match. His right thigh came in contact with the
opponent's cleat. The collision caused immediate pain. He had difficulty weight bearing and was unable to
continue in the game. The boy was immediately evaluated at a local urgent care center, where the results of
radiography of the femur were negative. He has been resting, icing, and using a compression sleeve on the
thigh. Six days have passed with some improvement. Physical examination reveals no mass, no thigh swelling
or bruising, full range of motion at the hip, and limited knee flexion. He exhibits a mild decrease in strength
on quadriceps testing (4/5 strength with extension) on the affected side. He is able to bear weight with
minimal limp. He would like to know if he can participate in the championship soccer game today.
Of the following, the MOST appropriate guidance to provide the boy is that he can return to play
B. when he gains full strength, full range of motion, and normal gait
Correct answer is B
PREP Pearl(s)
Return-to-play decisions should take into account the athlete’s health status (physical, psychological,
and functional), participation risk, and decision modifiers.
A return-to-play decision model can provide a strategic approach for individual clinical scenarios.
Critique
The adolescent in the vignette presents with a strength and gait deficit that precludes clearance to play
sports. He should be advised to return to play when he achieves full strength, full range of motion, and a
normal gait. A normal gait pattern is essential for running in soccer. Additionally, the limbs should be
symmetric in strength and motion. For athletes, adequate rehabilitation of a current injury is important in the
prevention of future injury.
Returning to play after an injury or illness must take into account multiple factors. An evidence-based model
(Creighton 2010) recommends considering health status, participation risk, and decision modification. The 3
steps can be applied to individual patients and their clinical scenarios to provide a strategic approach for
return to play decisions.
Step 1 addresses the individual’s health status. The physical, psychological, and functional status of the child
are reviewed to evaluate the seriousness of the injury. The injury history, past medical history, and current
symptoms are considered. The physical examination includes assessment of function, healing, strength, and
range of motion. Radiographic imaging, advanced imaging, and laboratory studies may be used. The child’s
readiness and confidence to return are assessed.
Step 2 evaluates the child’s participation risk. The type of sport, level of sport, and player position are key in
the decision-making process. Higher contact and collision sports typically require complete resolution of an
injury for return to play. A noncontact sport or one in which the athlete could modify the level of play (eg,
change of player position) may allow for earlier return to play. The ability to wear padding may enter the
decision-making process (eg, a stable fracture of the finger, hand, or wrist may be casted and padded to allow
an athlete return to sport before full bony healing).
Step 3 involves decision modifiers specific to the clinical scenario. The advantages and disadvantages of the
timing of the sport season are potential drivers for clearance. An athlete in the off-season vs an athlete
playing in a championship game may influence the clinical decision. There often exists external pressure from
the athlete, parents, coach, team, and community that needs to be addressed and documented for both
short-term and long-term health. Open discussion regarding any conflict of interest and documentation is
important for every scenario.
Suggested Reading(s)
Canty G, Nilan L. Return to play. Pediatr Rev. 2015;36(10):438–447. doi:10.1542/pir.36-10-438
Creighton DW, Shrier I, Shultz R, Meeuwisse WH, Matheson GO. Return-to-play in sport: a decision-
based model. Clin J Sport Med. 2010;20(5):379-385. doi:10.1097/JSM.0b013e3181f3c0fe
Content Domain
Sports Medicine
The correct answer is: when he gains full strength, full range of motion, and normal gait
While playing in a middle school football game, a 13-year-old wide receiver sustains a helmet-to-helmet
collision. He falls to the ground, then stands immediately and takes his helmet off. Several seconds later he
kneels and is assisted off the field by the athletic trainer. During the sideline evaluation, he is awake and
talking but clearly confused, unable to move both upper extremities, and describing paresthesia of both
hands.
Of the following, the BEST next step in this boy’s management is to
A. advise rest with return to play when his stinger injury symptoms have resolved
Correct answer is D
PREP Pearl(s)
Appropriate management of an acute sports-related neck injury requires the cervical spine to remain
in proper alignment. In the case of a football injury, before transport, the face mask should be
removed while the helmet and shoulder pads are kept in place. If the helmet has been removed, the
shoulder pads should also be removed and a cervical collar placed.
Knowledge and practice of appropriate cervical spine injury management are critical when providing
sideline coverage.
Diagnostic imaging is indicated when there is bilateral paresthesia after trauma to the head and/or
neck during contact sports.
Critique
The boy in the vignette is demonstrating progressive neurologic symptoms and an altered mental status,
raising concern for cervical spine and/or intracranial injury after a helmet-to-helmet collision. Urgent
transport to a trauma center for formal evaluation is indicated, and cervical spine immobilization is needed.
The injured athlete should be placed on a rigid spine board before removal from the field.
Typically, when a football player has a suspected cervical spine injury, the face mask should be removed from
the football helmet to establish airway access, leaving the helmet and shoulder pads in place, before
transferring the athlete to a trauma center. Sideline health care professionals should be trained in face mask
removal and have the appropriate tools at their disposal. If an injured football player is found without a
helmet, as in this case, the shoulder pads should be promptly removed and a cervical collar placed before
transferring the athlete to a rigid spine board. The helmet and shoulder pads function as a single unit to help
maintain cervical spinal alignment; both should either stay on the athlete or be removed.
It is critically important to maintain cervical spine alignment when transferring an athlete to a rigid spine
board. The preferred technique depends on the position of the injured athlete. An 8-person lift technique is
preferred when the athlete is supine, and a log-roll technique is preferred when the athlete is prone.
Although the boy in the vignette may have a concussion, a cervical spine injury or intracranial injury is of
greater concern, given his neurologic symptoms and altered mental status. A stinger injury is a unilateral
brachial plexus injury and produces unilateral symptoms; this diagnosis is unlikely in this individual with
bilateral neurologic symptoms. Emergency intubation in the field is not indicated for this boy because he is
currently maintaining his airway.
For any athlete with bilateral neurologic symptoms after trauma to the head and/or neck while playing a
contact sport, cervical magnetic resonance imaging (MRI) is indicated. An MRI provides the best assessment
of the functional reserve of cerebrospinal fluid around the spinal cord. Any loss of functional reserve
(functional spinal stenosis) places the athlete at increased risk for injury to the spinal cord. Measurement of
the anteroposterior diameter of the spinal canal with cervical spine radiographs is not as sensitive as MRI for
diagnosing functional spinal stenosis. Cervical spine radiographs, with flexion and extension views, should be
performed to assess stability. Return to sport is controversial after a cervical injury with transient signs and
symptoms. Normal imaging findings and full return of function are required to consider clearance for contact
collision sports; sports clearance is provided on a case by case basis.
Suggested Reading(s)
Fischer PE, Perina DG, Delbridge TR, et al. Spinal motion restriction in the trauma patient: a joint
position statement. Prehosp Emerg Care. 2018;22(6):659-661. doi:10.1080/10903127.2018.1481476
Herman MJ, Brown KO, Sponseller PD, et al. Pediatric cervical spine clearance: a consensus statement
and algorithm from the Pediatric Cervical Spine Clearance Working Group. J Bone Joint Surg Am.
2019;101(1):e1. doi:10.2106/JBJS.18.00217
Mills BM, Conrick KM, Anderson S, et al. Consensus recommendations on the prehospital care of the
injured athlete with a suspected catastrophic cervical spine injury. J Athl Train. 2020;55(6):563-572.
doi:10.4085/1062-6050-0434.19
Swartz EE, Boden BP, Courson RW, et al. National Athletic Trainers' Association position statement:
acute management of the cervical spine-injured athlete. J Athl Train. 2009;44(3):306-331.
doi:10.4085/1062-6050-44.3.306
Content Domain
Sports Medicine
The correct answer is: remove his shoulder pads and place a cervical collar
AAP PREP 2024 - Question 260/267 Sports Medicine Question 9/11
A 17-year-old basketball player is seen in the office with right ankle pain and swelling after landing awkwardly
and stepping on another player's foot. At the time he was unable to bear weight. The results of initial
radiography of the foot and ankle were negative, and he was provided crutches and an aircast. Today, 10
days after the injury, he continues to have pain with weight bearing, diffuse ankle swelling, and bruising
along the medial ankle. He has tenderness to palpation only over the anterior inferior tibiofibular ligament
and medial malleolus. The results of a squeeze test and dorsiflexion external rotation test are positive.
Correct answer is D
PREP Pearl(s)
The ankle contains 3 main ligament complexes, lateral, medial, and syndesmotic, any of which can be
injured in a sprain.
Supination and inversion most often result in lateral ankle injury.
Eversion most often leads to medial and syndesmotic ankle injuries.
Critique
The findings of the adolescent in the vignette are most consistent with a syndesmotic ankle sprain. Ankle
sprain is a common injury that is frequently evaluated in the emergency department, urgent care, or office
setting. The ankle contains 3 main ligament complexes, lateral, medial, and syndesmotic, any of which can be
injured in a sprain. Supination and inversion injuries most often result in a lateral ankle injury. Eversion of the
ankle leads to medial and syndesmotic ankle injuries.
The tibiofibular syndesmosis is a fibrous joint made up of 4 ligaments: anterior inferior tibiofibular, posterior
inferior tibiofibular, transverse, and interosseous. Syndesmotic ankle sprains (high ankle sprains) are
commonly encountered in contact sports, occurring when the affected foot is planted and the ankle forcibly
dorsiflexed and externally rotated. This injury results in sharp pain along the anterolateral ankle that is worse
with weight bearing. The squeeze test (compression and release of the leg at midcalf) and Kleiger test
(external rotation and dorsiflexion of the foot while the lower leg is stabilized) are classically positive with a
syndesmotic ankle sprain; pain is felt in the area of the syndesmosis.
Lateral ankle sprains are more common than syndesmotic ankle sprains. A lateral ankle sprain typically
involves plantarflexion and subtalar inversion. Three ligaments make up the lateral ligament complex: the
anterior talofibular ligament (ATFL), calcaneofibular ligament (CFL), and the posterior talofibular ligament
(PTFL). The ATFL is under maximal stress with foot inversion and is most commonly injured in lateral sprains.
The CFL is the second most common lateral ligament injured. The PTFL is injured with forced dorsiflexion; it is
the least frequently injured. All lateral ligaments can tear proximally, midsubstance, or at the distal
attachment. With a lateral ankle sprain, pain and swelling are often localized to 1 or more lateral ankle
ligaments, and anterior drawer testing demonstrates ligamentous laxity.
Lateral ligament injuries can be classified based on laxity noted on examination:
The peroneus longus tendon runs posteriorly and inferiorly to the lateral malleolus and attaches underneath
the foot to provide ankle stability. Peroneus longus tendon injuries can be either acute (ankle sprain) or
chronic (repetitive use), resulting in pain and swelling posterior to the lateral malleolus. The boy in the
vignette has medial ankle findings on examination, making this diagnosis unlikely.
Lateral malleolus fracture is an unlikely diagnosis for this adolescent. The results of radiography of the ankle
and foot, obtained on the day of injury based on the Ottawa ankle rules (inability to bear weight immediately
following the injury), were negative. If the boy had bony tenderness over the lateral malleolus on today’s
examination, repeat radiography would be appropriate to rule out a fracture.
Suggested Reading(s)
Canares TL, Lockhart G. Sprains. Pediatr Rev. 2013;34(1):47-49. doi:10.1542/pir.34-1-47
Moseley M, Rivera-Diaz Z, Fein DM. Ankle Injuries. Pediatr Rev. 2022;43(3):185-187.
doi:10.1542/pir.2021-004992
Content Domain
Sports Medicine
A 16-year-old wrestler is seen in the emergency department after he fell on his outstretched right hand
during a match, resulting in acute elbow pain and deformity. He was unable to flex or extend the elbow.
Findings on physical examination of the right upper extremity were consistent with shifting of the olecranon
posteriorly, indicating a posterior elbow dislocation. There was no fracture or intra-articular loose body noted
on radiography. Successful reduction of the elbow dislocation was achieved with supination of the forearm,
elbow flexion, countertraction on the humerus, and gentle axial traction on the forearm. The elbow was then
immobilized in a posterior splint at 90°.
Of the following, the MOST accurate anticipatory guidance regarding this injury is to expect
Correct answer is B
PREP Pearl(s)
Pediatric elbow dislocations are rare injuries.
Pediatric elbow dislocations often have an associated fracture; it is important to obtain radiographs
before reduction.
Simple stable posterior elbow dislocations require a short period of immobilization, then guided active
range of motion is initiated to prevent elbow stiffness.
Critique
For the adolescent in the vignette, the most accurate anticipatory guidance regarding what to expect after
this injury is loss of 10° terminal extension in the elbow.
Elbow dislocations are rare injuries in children, constituting 5% of all pediatric joint dislocations. The peak
incidence is at 12 years of age. These injuries occur more commonly in males and in the nondominant
extremity. Elbow dislocations are classified by the direction of the dislocation: posterior, posterior lateral,
posterior medial, anterior, medial, and lateral. Ninety percent of pediatric elbow dislocations are classified as
posterior.
Approximately 60% of pediatric elbow dislocations have associated fractures of the radial head through the
physis, radial neck, coronoid process, olecranon process, lateral condyle, or avulsion of the medial
epicondyle. Therefore, it is important that radiographs be obtained before reduction. Associated fractures
can have a significant impact on the outcome of elbow dislocations. For example, the “terrible triad” of radial
head fracture, coronoid process fracture, and medial collateral ligament injury (elbow dislocation) often leads
to long-term joint stiffness, instability, and degenerative changes.
Neurovascular complications from a posterior elbow dislocation, which can affect the ulnar nerve or brachial
artery, are rare. A transient ulnar neuropathy will produce tingling in the fourth and fifth digits and the
hypothenar eminence. A brachial artery injury causes pain that radiates down the arm, decreases skin
temperature and pulse strength, and produces notable extremity pallor. Early identification of a brachial
artery injury is critical to prevent a Volkmann ischemic contracture (contracture of the flexor muscles and
nerve palsy of the median and ulnar nerves).
Simple, stable elbow dislocations can be treated conservatively with a splint or cast for 2 to 3 weeks. The
appropriate immobilized position has the elbow flexed 90° and the forearm in pronation to reduce the radial
head dislocation and promote healing of the lateral ligament complex. After a short period of immobilization,
advancement to active range of motion is generally guided by an occupational or physical therapist. Simple
posterior dislocations can be treated using a hinged elbow brace that blocks terminal extension at the angle
noted to maintain stability, which allows for immediate range of motion to mitigate stiffness. The therapist
will guide the range of motion to full extension over 4 weeks.
Loss of approximately 10° of extension is expected after an elbow dislocation, which does not limit future
function. Surgical fixation is usually limited to individuals with displaced elbow fractures or incongruent
reductions. The typical time frame for return to athletic activities is 6 to 8 weeks after a simple posterior
elbow dislocation and longer for more complicated dislocations with fracture.
Suggested Reading(s)
Gottlieb M, Suleiman L. Current approach to the management of forearm and elbow dislocations in
children. Pediatr Emerg Care. 2019;35(4):293-298. doi:10.1097/PEC.0000000000001805
Potocki R, Canares T. Elbow injuries in children. Pediatr Rev. 2022;43(4):236-238. doi:10.1542/pir.2021-
004986
Content Domain
Sports Medicine
The correct answer is: loss of 10° terminal extension in the elbow
A 14-year-old female athlete presents with pain in the hypothenar region of her right hand. She is an avid
golfer and right hand dominant. Pain occurs with gripping the club tightly. She reports decreased grip
strength and numbness involving the fourth and fifth fingers.
Of the following, the BEST initial test to diagnose this girl’s condition is a
Correct answer is A
PREP Pearl(s)
Hook of the hamate fracture most often occurs in athletes participating in sports using a racquet, club,
or bat.
Carpal tunnel radiography is the first-line imaging test recommended for identification of a hook of
hamate fracture.
Computed tomography (CT) of the wrist is the most sensitive imaging test for a hook of hamate
fracture; CT is recommended when a hook of the hamate fracture is suspected despite negative carpal
tunnel radiographic findings.
Critique
The adolescent in the vignette most likely has a hamate fracture or injury. Carpal tunnel radiography is the
imaging study that will visualize the hook of the hamate.
The hook of the hamate arises from the body of the hamate and extends into the hypothenar area of the
palm; it is located 1 to 2 cm distal and radial to the pisiform. An easy way to palpate the hook of hamate is to
palpate the pisiform and then roll into the palm radially—the next bony prominence felt is the hook. The
hook serves as an attachment site for the flexor carpi ulnaris, flexor digiti minimi, and opponens digiti minimi
and as the radial border for the Guyon canal. The ulnar nerve travels within the Guyon canal and splits into
sensory and motor branches. The superficial sensory branch provides innervation to the medial fourth digit
and fifth digits and travels close to the hook of hamate tip. The deep motor branch travels at the base of the
hamate and innervates the hypothenar muscles. Injury to the hook of hamate can cause symptoms in the
ulnar nerve distribution, weakness in finger flexion, and grip weakness.
Hook of the hamate fractures commonly occur in athletes participating in racquet sports, golf, softball, or
baseball. Direct compression by a racquet, club, or bat can cause a hook of the hamate fracture; affected
individuals often recall a forceful swing. Forceful grip, repetitive microtrauma, and shearing forces contribute
to further injury. Hook of the hamate fractures comprise 2% to 4% of all carpal fractures. A high index of
suspicion, based on the mechanism of injury and physical examination findings, can prompt proper
diagnostic testing via radiography.
Carpal tunnel radiography with the wrist in hyperextension (maximum dorsiflexion) and angled 20° to 30°
can visualize the hook of the hamate, although the base of the hook may not be visualized. Sensitivity is 40%
to 50% for a hook of the hamate fracture with radiography. Computed tomography (CT) of the wrist has 92%
to 100% sensitivity for detection of a hook of the hamate fracture. Magnetic resonance imaging provides
evaluation of the neurovascular structures within the Guyon canal and ulnar nerve, as well as bone, but cost
can be a limiting factor. A CT of the wrist is recommended for evaluation when a hook of the hamate fracture
is suspected despite negative carpal tunnel radiographic findings.
Both nonoperative and operative treatment options are available for hook of the hamate fractures.
Treatment must take into account the fracture location, child’s age, and activity level. An orthopedic hand
surgeon should be consulted. With early diagnosis, nonoperative treatment consisting of immobilization in a
cast for 6 to 8 weeks can heal the fracture, but there is a significant risk of nonunion. Delayed diagnosis
increases the risk of nonunion of the fracture. Surgical excision of the hook fracture has become a gold
standard for treatment, leading to quick resolution of symptoms and return to play.
Suggested Reading(s)
Bauer AS, Bae DS. Upper Extremity Injuries in Young Athletes. Springer; 2018. doi:10.1007/978-3-319-
56651-1
Spencer J, Hunt SL, Zhang C, Walter C, Everist B. Radiographic signs of hook of hamate fracture:
evaluation of diagnostic utility. Skeletal Radiol. 2019;48(12):1891-1898. doi:10.1007/s00256-019-03221-
0
Tian A, Goldfarb CA. Hook of hamate fractures. Hand Clin. 2021;37(4):545-552.
doi:10.1016/j.hcl.2021.06.013
Content Domain
Sports medicine
An 18-month-old girl with a history of sickle cell disease is evaluated for 2 days of fever, cough, and vomiting.
A rapid influenza diagnostic test is performed due to her symptoms and the high incidence of influenza A in
the community. The test has 55% sensitivity and 95% specificity for influenza.
Of the following, the MOST likely error type to occur when using this test is
A. type I
B. type II
C. type III
D. type IV
Correct answer is B
PREP Pearl(s)
Tests with a low sensitivity have a higher likelihood of false-negative results or type II errors.
Tests with a low specificity have a higher likelihood of false-positive results or type I errors.
Critique
The most likely error type to occur when using the rapid influenza diagnostic test is a type II error. A type II
error occurs when the diagnostic test result is negative but the patient has the disease being tested for (ie,
false negative). When the sensitivity of a test is low, as in this case, any negative test result has a high
probability of being a false negative or type II error. In this situation, the practitioner must consider both the
child’s risk of complications from influenza (eg, increased risk for this child with sickle cell disease) and the
likelihood of a false-negative test result and decide whether to believe the test result, administer antiviral
therapy based on clinical suspicion, or perform further influenza testing of respiratory specimens by
molecular assays.
A type I error occurs when a diagnostic test result is positive but the patient does not have the disease being
tested for (ie, false positive). Tests with a low specificity have a higher likelihood of a false-positive result
(Table).
A type III error occurs in statistics when you correctly reject the null hypothesis, but it is rejected for the
wrong reason. One arrives at the correct conclusion but for the wrong reason.
A type IV error occurs when you correctly reject the null hypothesis but make a mistake interpreting the
results.
Suggested Reading(s)
Centers for Disease Control and Prevention. Rapid influenza diagnostic tests. Accessed October 22,
2022. https://www.cdc.gov/flu/professionals/diagnosis/clinician_guidance_ridt.htm
Jennings JM, Sibinga E. Demystifying type I and type II errors. Pediatr Rev. 2010;37(11):209-210.
doi:10.1542/pir.31-5-209
Content Domain
Statistics
ABP Content Specification(s) / Content Area(s)
Distinguish between type I and type II statistical errors
Understand standard deviation in the interpretation of results
Understand standard error in the interpretation of results
Understand confidence interval in the interpretation of results
Infected Not-Infected
Reprinted with permission from Jennings JM, Sibinga E. Demystifying type I and type II
errors. Pediatr Rev. 2010;37(11):210.
AAP PREP 2024 - Question 264/267 Substance Use and Addictions Question 1/3
An 18-year-old high school senior is seen for a preparticipation physical examination. He is a high-performing
basketball player at his school. His review of symptoms is significant for abdominal pain and nausea that
occurs once per week, typically before basketball games. He has difficulty falling asleep because of racing
thoughts. On a confidential substance-use screening tool, he reports daily use of a nicotine-containing vape
device, monthly marijuana use, and binge drinking once or twice in the past year. The patient desires to stop
vaping, as he is concerned about the potential negative effect on his performance. However, he feels it is the
only thing that helps him “calm down.” He reports morning cravings for nicotine and withdrawal symptoms if
he goes more than 1 day without using his vape device.
He is referred to a counselor for behavioral support, and the option of initiating a selective serotonin
reuptake inhibitor is discussed.
Of the following, the BEST next step in the management of this adolescent’s nicotine use is to
Correct answer is B
PREP Pearl(s)
Many adolescents use substances such as nicotine to manage stress and anxiety. Physicians should
screen all adolescents for substance use (including nicotine) in a confidential manner.
Nicotine replacement therapy is an important component of treatment for an adolescent with nicotine
use disorder.
Although nicotine replacement therapy (NRT) is approved by the US Food and Drug Administration for
individuals 18 years and older, the American Academy of Pediatrics recommends off-label use of NRT
for adolescents younger than 18 years who show signs of nicotine dependance.
Critique
The best next step for this patient is to prescribe nicotine replacement therapy (NRT), specifically a nicotine
patch (21 mg) and nicotine gum or lozenges (4 mg). The patient in the vignette is experiencing somatic
manifestations of stress and anxiety (abdominal pain, nausea, insomnia) and is using nicotine as a coping
strategy. In addition to discussing treatment for anxiety (eg, behavioral therapy, selective serotonin reuptake
inhibitor), the physician should offer treatment options for nicotine use, especially since the patient is
concerned about the negative consequences of his nicotine use.
Adolescents prescribed NRT should be counseled on how to use it. For example, the “chew and park” method
of using nicotine gum should be recommended, because it increases absorption and decreases the side
effects (eg, nausea) that occur with incorrect use. The American Academy of Pediatrics (AAP) has published
guidelines for NRT in adolescents, including dosing and instructions (Table). Although NRT is approved by the
US Food and Drug Administration for individuals 18 years and older, the AAP recommends off-label use of
NRT for adolescents younger than 18 years who show signs of nicotine dependence.
Many adolescents use substances such as nicotine or marijuana to cope with negative feelings associated
with stress. Adolescents who use nicotine, alcohol, marijuana, and other substances are at higher risk of
experiencing addiction than are adults, owing to the developing adolescent brain and highly responsive
reward pathway. It is important for pediatricians to identify substance use and substance use disorders in
adolescents, provide appropriate treatment, and encourage cessation.
Nicotine use disorder (NUD) is characterized by a problematic pattern of nicotine use that continues despite
causing difficulty in multiple life domains (eg, social, educational, personal health). Many adolescents with
NUD have tried to quit or cut down but were unsuccessful. Daily nicotine use is highly predictive of having a
severe NUD and typically has associated withdrawal symptoms and cravings. Nicotine withdrawal can be
assessed with the Hooked on Nicotine Checklist (HONC). Symptoms of nicotine withdrawal can mimic those
of anxiety (eg, irritability, restlessness, difficulty concentrating). Thus, it is vital to inquire about and address
nicotine use in an adolescent patient who has a suspected anxiety disorder.
Although the HONC screening tool would help to identify trends over time, the adolescent patient in the
vignette has already reported many indicators of nicotine dependence (eg, daily use of nicotine, withdrawal
symptoms, morning cravings), and thus HONC screening is unnecessary. Smartphone apps, telephone
quitlines, and text messaging support are all useful behavioral interventions, but they should be used as
adjunctive treatments in addition to NRT. Setting a quit date has been shown to be one of the best indicators
of the likelihood of quitting. However, on the basis of his level of use, the adolescent patient in the vignette
primarily needs medication support with NRT.
Suggested Reading(s)
American Academy of Pediatrics. Section on Tobacco Control. Nicotine replacement therapy and
adolescent patients: information for pediatricians. Updated November 2019. Accessed September 1,
2022. AAP Patient Care
American Academy of Pediatrics. Tobacco use: considerations for clinicians. March 8, 2022. Accessed
September 1, 2022. AAP Patient Care
Section on Tobacco Control; Groner JA, Nelson KE, Etzel RA, et al. Clinical practice policy to protect
children from tobacco, nicotine, and tobacco smoke. Pediatrics. 2015;136(5):1008-1017.
doi:10.1542/peds.2015-3108
Tanski SE. Tobacco and nicotine use. In: McInerny TK, Adam HM, Campbell DE, DeWitt TG, Foy JM,
Kamat DM, eds. American Academy of Pediatrics Textbook of Pediatric Care. American Academy of
Pediatrics; 2023. Accessed September 1, 2023. Pediatric Care Online
Content Domain
Substance use and addictions
The correct answer is: prescribe a nicotine transdermal patch and nicotine gum
View Peer Results
Table. Types of Nicotine Replacement Therapy for Adolescent
Patients.
Nicotine Transdermal Patch Dosage:
(OTC for 18+ • 21mg, 14mg, 7mg
Rx for <18) Use Instructions:
Cost: • Apply patch to clean skin, change patch every 24
Over-the-counter retail hours
cost ranges from $25-$70 • 8-10 week treatment regimen:
for 28 patches.
Out-of-pocket
° Use first dose for 6 weeks, then “step down” to lower
dose
prescription costs will
vary by insurance plan. ° Use lower dose for 2 weeks, then “step down” to
lowest dose for 2 more weeks
• See package for full details
Side Effects:
• Skin irritation, sleep disturbance
Advantages:
• Sustained blood levels of nicotine, compliance is
relatively easy
During a health supervision visit, a 15-year-old girl is screened for substance use. She reports monthly
marijuana use, generally when she is feeling overwhelmed or stressed at school. She denies other drug use
and has never ridden in a car driven by someone under the influence of drugs or alcohol.
A. engage a parent (with the patient’s permission) in the discussion of this behavior and refer the
patient for counseling with an appropriate specialist
C. provide positive reinforcement of her judgment regarding automobile safety; follow-up in 1 year
D. use motivational interviewing to help her compare the benefits of continuing this behavior
with the benefits of reducing or stopping it
Correct answer is D
PREP Pearl(s)
Substance use screening, brief intervention, and referral to treatment are important components of
the standard adolescent health supervision visit.
Assessment with a substance use screening tool is preferable to clinical assessment alone.
Alcohol, tobacco, and marijuana are the substances most commonly used by youth in the United
States.
Critique
The best next step in management of the adolescent in the vignette is to use the brief intervention strategy of
motivational interviewing to help her compare the benefits of continuing substance use with the benefits of
reducing or stopping use.
Substance use screening, brief intervention, and referral to treatment are important components of the
adolescent health supervision visit. However, self-reported data demonstrate that screening for substance
use by pediatricians varies from less than 50% to 86%; many rely on clinical impression rather than a
screening tool. Alcohol, tobacco, and marijuana are the substances most commonly used by youth in the
United States.
For the adolescent in the vignette, exhibiting a mild or moderate substance use disorder (SUD), the use of a
motivational interview technique that respects their autonomy while helping them to implement a behavior
change is an appropriate brief intervention strategy. This approach can help the adolescent identify the
benefits of reducing or abstaining from substance use. A specific plan can be developed by the pediatrician
and adolescent together, including a plan for close follow-up.
When screening reveals no substance use, the pediatrician should use positive reinforcement to credit the
adolescent with making an active decision to abstain. This approach can help delay the initiation of alcohol
use, in particular among adolescents. This adolescent exhibits behaviors consistent with mild to moderate
SUD; therefore, positive reinforcement and praise for not driving with someone under the influence would
not be the recommended approach to counseling.
The use of tobacco, alcohol, or marijuana once or twice in the past year is classified as substance use without
SUD (Table 1). The appropriate brief intervention in such a case is to provide clear advice on stopping
substance use, including a brief discussion of the negative effects of continued use. This technique has not
been shown to be effective for individuals, such as the girl in the vignette, with mild to moderate SUD.
Weekly substance use is classified as severe SUD or addiction. For those with severe SUD, the best next steps
are to refer them for a comprehensive evaluation by a substance use specialist and to consider assessment
for a concurrent mental health disorder. Assessment for the acute risk of harm is also critically important.
The pediatrician must use clinical judgment when considering the role of a parent or guardian when
counseling and managing an adolescent with severe SUD. Adolescents are unlikely to follow through with
referrals without the help of an adult. An open discussion with the adolescent about why confidentiality may
no longer be upheld and engaging them in the decision about how to inform their parent or guardian is
helpful in maintaining trust. This girl does not yet meet the criteria for severe SUD; therefore, referral to an
addiction medicine specialist is not warranted at this time.
There are many levels of care for the treatment of the adolescent with SUD (Table 2). For successful
management, it is important for the pediatrician to be knowledgeable about services available in their
community, participate as an active member of the adolescent’s multidisciplinary team during treatment, and
be prepared to care for the adolescent after discharge from treatment.
Suggested Reading(s)
Bagley S, Levy S. Substance use disorders. In: McInerny TK, Adam HM, Campbell DE, DeWitt TG, Foy JM,
Kamat DM, eds. American Academy of Pediatrics Textbook of Pediatric Care. American Academy of
Pediatrics; 2023. Accessed September 1, 2023. Pediatric Care Online
Levy S, Bagley S. Substance use: initial approach in primary care. In: McInerny TK, Adam HM, Campbell
DE, DeWitt TG, Foy JM, Kamat DM, eds. American Academy of Pediatrics Textbook of Pediatric Care.
American Academy of Pediatrics; 2023. Accessed September 1, 2023. Pediatric Care Online
Levy SJ, Williams JS. Substance use screening, brief intervention, and referral to treatment. Pediatrics.
2016;138(1):e20161211. doi:10.1542/peds.2016-1211
Ryan SA, Kokotailo P; AAP Committee on Substance Use and Prevention. Alcohol use by youth.
Pediatrics. 2019;144(1):e20191357. doi:10.1542/peds.2019-1357
Content Domain
Substance use and addictions
The correct answer is: use motivational interviewing to help her compare the benefits of continuing this
behavior with the benefits of reducing or stopping it
View Peer Results
Table 1. Substance Use Spectrum and Goals for Brief Intervention.
Stage Description Brief Intervention Goals
Abstinence The time before an individual has ever used drugs or alcohol more Prevent or delay initiation of substance use through positive
than a few sips. reinforcement and patient/parent education.
Substance use without a disorder Limited use, generally in social situations, without related problems. Advise to stop. Provide counseling regarding the medical harms of
Typically, use occurs at predictable times, such as on weekends. substance use. Promote patient strengths.
Mild–moderate substance use disorder Use in high-risk situations, such as when driving or with strangers. Brief assessment to explore patient-perceived problems associated
Use associated with a problem, such as a fight, arrest, or school with use. Give clear, brief advice to quit. Provide counseling regarding
suspension. Use for emotional regulation, such as to relieve stress the medical harms of substance use. Negotiate a behavior change to
or depression. Defined as meeting 2 to 5 of the 11 criteria for an quit or cut down. Close patient follow-up. Consider referral to
substance use disorder in the DSM-5. substance use disorder treatment. Consider breaking confidentiality.
Severe substance use disorder Loss of control or compulsive drug use associated with neurologic As above. Involve parents in treatment planning whenever possible.
changes in the reward system of the brain. Defined as meeting ≥6 of Refer to the appropriate level of care. Follow up to ensure compliance
the 11 criteria for an substance use disorder in the DSM-5. with treatment and to offer continued support.
DSM-5, Diagnostic and Statistical Manual of Mental Disorders, Fifth Edition.
Reprinted with permission from Levy SJ, Williams JS. Substance use screening, brief intervention, and referral to treatment. Pediatrics. 2016;138(1):e20161211.
Table 2. American Society of Addiction Medicine Levels of Care for Treatment of Substance Use Disorders.
Description
Outpatient
Individual Adolescents with substance use disorders should receive specific treatment of their substance use; general supportive counseling may be a useful adjuvant but
counseling should not be a substitute. Several therapeutic modalities (motivational interviewing, cognitive behavioral therapy, contingency management, etc) have all shown
promise in treating adolescents with substance use disorders.
Group therapy Group therapy is a mainstay of substance use disorders treatment of adolescents with substance use disorders. It is a particularly attractive option because it is
cost-effective and takes advantage of the developmental preference for congregating with peers. However, group therapy has not been extensively evaluated as a
therapeutic modality for this age group, and existing research has produced mixed results.
Family therapy Family-directed therapies are the best-validated approach for treating adolescent substance use disorders. A number of modalities have all been shown to be
effective. Family counseling typically targets domains that figure prominently in the etiology of substance use disorders in adolescents: family conflict,
communication, parental monitoring, discipline, child abuse/neglect, and parental substance use disorders.
IOP IOPs serve as an intermediate level of care for patients who have needs that are too complex for outpatient treatment but do not require inpatient services. These
programs allow individuals to continue with their daily routine and practice newly acquired recovery skills both at home and at work.
IOPs generally comprise a combination of supportive group therapy, educational groups, family therapy, individual therapy, relapse prevention and life skills,
12-step recovery, case management, and after-care planning. The programs range from 2 to 9 hours per day, 2 to 5 times per week, and last 1 to 3 months. These
programs are appealing because they provide a plethora of services in a relatively short period of time.
Partial hospital Partial hospitalization is a short-term, comprehensive outpatient program in affiliation with a hospital that is designed to provide support and treatment of patients
program with substance use disorders. The services offered at these programs are more concentrated and intensive than regular outpatient treatment because they are
structured throughout the entire day and offer medical monitoring in addition to individual and group therapy. Participants typically attend sessions for 7 or 8 hours
per day, at least 5 days per week, for 1–3 weeks. As with IOPs, patients return home in the evenings and have a chance to practice newly acquired recovery skills.
Inpatient/residential
Detoxification Detoxification refers to the medical management of symptoms of withdrawal. Medically supervised detoxification is indicated for any adolescent who is at risk
for withdrawing from alcohol or benzodiazepines and may also be helpful for adolescents withdrawing from opioids, cocaine, or other substances. Detoxification
may be an important first step but is not considered definitive treatment. Patients who are discharged from a detoxification program should then begin either an
outpatient or residential substance use disorders treatment program.
ART ART is a short-term (days–weeks) residential placement designed to stabilize patients in crisis, often before entering a longer-term residential treatment program.
ART programs typically target adolescents with co-occurring mental health disorders.
Residential Residential treatment programs are highly structured live-in environments that provide therapy for those with severe substance use disorders, mental illness, or
treatment behavioral problems that require 24-hour care. The goal of residential treatment is to promote the achievement and subsequent maintenance of long-term
abstinence as well as equip each patient with both the social and coping skills necessary for a successful transition back into society. Residential treatment
programs are classified by the length of the program; short-term refers to programs of ≤30 days’ duration, long-term refers to programs of >30 days’ duration.
Residential treatment programs generally comprise individual and group therapy sessions plus medical, psychological, clinical, nutritional, and educational
components. Residential facilities aim to simulate real living environments with added structure and routine to prepare individuals with the framework necessary
for their lives to continue drug and alcohol free on completion of the program.
Therapeutic Therapeutic boarding schools are educational institutions that provide constant supervision for their students by professional staff. These schools offer a highly
boarding structured environment with set times for all activities, smaller, more specialized classes, and social and emotional support. In addition to the regular services offered
school at traditional boarding schools, therapeutic schools also provide individual and group therapy for adolescents with mental health or substance use disorders.
Abbreviations: ART, acute residential treatment; IOP, intensive outpatient program.
Adapted and reprinted with permission from Levy SJ, Williams JS. Substance use screening, brief intervention, and referral to treatment. Pediatrics. 2016;138(1):e20161211.
AAP PREP 2024 - Question 266/267 Substance Use and Addictions Question 3/3
A 16-year-old boy is seen because of parental concern for substance use. He has been hanging out after
school with a new group of friends, his grades have dropped this semester, he is spending a lot more time
alone in his room, and he is resisting family time (eg, meals together). His parents are concerned that he is
depressed. They ask about the prevalence of adolescent substance use and whether it has been affected by
the COVID-19 pandemic. The physician shares data regarding substance use among adolescents in the 10th
to 12th grades during the COVID-19 pandemic (2020 and 2021).
Of the following, the MOST accurate information the physician can provide is that for this age group during
this time frame there was an increase in
A. alcohol consumption
B. marijuana use
C. nicotine use
Correct answer is D
PREP Pearl(s)
Substance use is most commonly initiated between the ages of 15 and 17 years but may begin as early
as 10 years of age.
Alcohol, marijuana, and nicotine are the most common substances used by adolescents.
During the COVID-19 pandemic, all illicit substance use decreased among adolescents; however, opioid
overdose fatalities increased because of illicitly manufactured fentanyl.
Critique
The most accurate information the physician can provide the adolescent’s parents regarding substance use
during the COVID-19 pandemic is that there was an increase in fatalities from opioid overdose. The majority
of adolescent substance use occurs between the ages of 15 and 17 years but may start as early as 10 years of
age. Brain development during this age range predisposes adolescents to both a heightened response to
immediate reward satisfaction and an immature ability to manage impulsivity.
Historically, alcohol has been the substance most commonly used by adolescents, followed by marijuana and
nicotine. In 2017, 19.9% of 10th graders reported alcohol use, 14.0% reported marijuana use, and 19.5%
reported nicotine use. At 18 years of age, reported use increased to 64%, 45%, and 31%, respectively.
Although these statistics remained relatively stable during the years before the COVID-19 pandemic, cigarette
use has been decreasing in inverse correlation with nicotine vaping, and there has been a steady increase in
the use of marijuana by youth, mirroring changes in attitudes around marijuana use and its increased
legalization for adults in the United States.
The National Institute on Drug Abuse, part of the National Institutes of Health, funds the Monitoring the
Future Survey, an annual national survey of drug use among 8th, 10th, and 12th graders. Survey data
obtained in both 2020 and 2021 showed a decrease in the use of marijuana, nicotine, and alcohol across all
surveyed age groups (Figure). In 2020, this decrease was thought to be due to social distancing resulting from
COVID-19–related school shutdowns.
Source: National Institute on Drug Abuse. Monitoring the Future 2021 Survey Results.
Although illicit drug use decreased for middle and high school students from 2019 to 2020, overdose deaths
among adolescents between 14 and 18 years of age increased 94% from 2019 to 2020 and 20% from 2020 to
2021. This increase is mostly due to the prevalence of illicitly manufactured fentanyl (IMF) and the
proliferation of pills resembling prescription drugs but containing IMF.
The American Academy of Pediatrics recommends routine screening for illicit substance use at health
supervision visits starting at 11 years of age. Children and adolescents are unlikely to disclose substance use
unless they are asked directly. A validated screening tool, such as the Brief Screener for Tobacco, Alcohol, and
other Drugs (BSTAD), Screening to Brief Intervention (S2BI), or CRAFFT (Car, Relax, Alone, Forget, Friends,
Trouble), is indicated. Pediatricians are trusted and respected adults who teens can turn to for reliable
information about substance use.
Suggested Reading(s)
Gray KM, Squeglia LM. Research review: what have we learned about adolescent substance use? J Child
Psychol Psychiatr. 2018;59(6):618-627. doi:10.1111/jcpp.12783
National Institute on Drug Abuse. Percentage of adolescents reporting drug use decreased significantly
in 2021 as the COVID-19 pandemic endured. Accessed December 26, 2022. https://nida.nih.gov/news-
events/news-releases/2021/12/percentage-of-adolescents-reporting-drug-use-decreased-significantly-
in-2021-as-the-covid-19-pandemic-endured
Tanz LJ, Dinwiddie AT, Mattson CL, et al. Drug overdose deaths among persons aged 10–19 years —
United States, July 2019–December 2021. MMWR Morb Mortal Wkly Rep. 2022;71:1576-1582.
doi:10.15585/mmwr.mm7150a2
Wang GS, Hoyte C. Common substances of abuse. Pediatr Rev. 2018;39(8):403-414.
doi:10.1542/pir.2017-0267
Content Domain
Substance Use and Addictions
A 17-year-old with type 1 diabetes mellitus is making a health supervision visit accompanied by her mother
and three siblings: a 5-year-old with asthma, a 10-year-old with attention-deficit/hyperactivity disorder, and a
healthy 12-year-old.
Of the four children, the physician should have a discussion about transitioning to adult health care with
D. the 17-year-old
Correct answer is C
PREP Pearl(s)
A stepwise approach to health care transition from a child/family-centered to an adult-centered model
of care is vital for adolescent patients both with and without special health care needs.
The first step in health care transition planning is to discuss the practice’s transition policy; this may
begin when the patient is as young as 12 years old.
Got Transition (https://www.gottransition.org) is a useful website to assist providers in the health care
transition process.
Critique
Planning for transition from an adolescent to an adult health care model is a process that should begin at age
12 years. The first step in transition planning should be discussion of the transition policy. The vignette
describes a family with four children, three of whom have chronic health conditions. Although discussions
around transition planning are especially important for children with chronic health conditions or other
special health care needs, all adolescent patients should begin to receive information about transitioning to
adult care starting at age 12 years.
Health care transition (HCT) is the process of changing from a child/family-centered to an adult-centered
model of care; this does not necessarily involve changing physicians. The American Academy of Pediatrics’
(AAP’s) 2018 clinical report recommends transition in a stepwise fashion and outlines a suggested timeline.
This is based on the Six Core Elements of HCT (Figure). The Six Core Elements are as follows:
The AAP clinical report and the Got Transition website (https://www.gottransition.org) provide details
regarding specific activities within each element. All pediatric practices should develop a transition policy to
share with patients and families starting when patients are aged 12 years. Resources and tools are available
at the Got Transition website to assist in policy formation and for all other steps in the transition process.
Suggested Reading(s)
Got Transition. Six Core Elements. Accessed September 1, 2023. https://www.gottransition.org/six-
core-elements/
Hagan JF, Shaw JS, Duncan PM, eds. Bright Futures: Guidelines for Health Supervision of Infants,
Children, and Adolescents. 4th ed. American Academy of Pediatrics; 2017.
Society for Adolescent Health and Medicine. Transition to adulthood for youth with chronic conditions
and special health care needs. J Adolesc Health. 2020;66(5):631-634.
doi:10.1016/j.jadohealth.2020.02.006
White PH, Cooley WC; Transitions Clinical Report Authoring Group. Supporting the health care
transition from adolescence to adulthood in the medical home. Pediatrics. 2018;142(5):e20182587.
doi:10.1542/peds.2018-2587
Content Domain
Transitions of Care/Care Coordination